Патофізіологія - тести

Етіологія та патогенез

1. У працівників підприємства по виробленню килимових покриттів відмічені риніт, дерматит, бронхіальна астма. За якою класифікацією ці хвороби можуть бути віднесені до одної групи?

A. За віком

B. Топографо – анатомічна

C. Екологічною

D. Патогенетична

E. За статтю

2. Перебіг гарячки, гіпоксії, голодування, запалення не залежать від виду, віку та статі тварини. Яку назву мають патологічні зміни, що розвиваються за однаковими основними закономірностями?

A. Патологічний стан

B. Період розвитку хвороби

C. Типовий патологічний процес

D. Патологічна реакція

E. Головна ланка патогенезу

3. Після того, як хлопчик промочив ноги, у нього підвищилася температура тіла, з'явився біль у горлі. Діагностовано гостре респіраторне захворювання. Який фактор є причиною хвороби в даному разі?

A. Алергія

B. Охолодження

C. Гіповітаміноз

D. Зниження імунітету

E. Інфекція

4. Стан хворого, який протягом 4 років хворіє на пептичну виразку шлунка, особливо погіршується восени і навесні, коли виникають печія, нудота, біль у животі. Як можна характеризувати стан хворого в період між загостреннями хвороби?

A. Одужання

B. Типовий патологічний процес

C. Ремісія

D. Патологічна реакція

E. Патологічний процес

5. Хлопчик, 12 років, після повернення зі школи почав скаржитися на головний біль, озноб, біль у м’язах, втрату апетиту, втому. Для якого періоду хвороби характерні такі симптоми?

A. Латентний

B. Інкубаційний

C. Розпалу захворювання

D. Закінчення хвороби

E. Продромальний

6. Хворому, що страждає на карієс, ускладнений гострим гнійним запаленням навколозубної тканини (періодонтитом), який супроводжується підвищенням температури тіла, було зроблено екстракцію зуба. Яку зміну в даного хворого можна віднести до поняття «патологічний стан»?

A. Карієс

B. Відсутність зуба

C. Періодонтит

D. Запалення

E. Лихоманка

7. Чоловік 40 років у результаті переохолодження занедужав на крупозну пневмонію. Яка причина виникнення такої форми запалення легенів?

A. Cтафілокок

B. Зниження реактивності організму в результаті переохолодження

C. Вік

D. Пневмокок

E. Вплив на організм низької температури

8. При рентгенологічному обстеженні у хворого виразковою хворобою виявлено стеноз воротаря. Це порушення є:

A. Типовим патологічним процесом

B. Патологічною реакцією

C. Захворюванням

D. Патологічним процесом

E. Патологічним станом

9. У жінки, 55 років, стоматолог виявив кілька зубів, уражених карієсом. Яким нозологічним поняттям можна визначити цей стан?

A. Патологічний стан

B. Ремісія

C. Хвороба

D. Патологічний процес

E. Компенсаторна реакція.

10. У здорової людини, яка перебувала в контакті з хворим на дифтерію, при бактеріологічному дослідженні змиву з носоглотки виявлено збудника дифтерії. Який період хвороби спостерігається в даному випадку?

A. Продромальний

B. Латентний

C. Розпал

D. Кінець

E. Одужання

11.Хто із зазначених нижче вчених уперше ввів у експериментальну практику метод хронічного експерименту й вважається засновником цього методу ?

A. Мєчніков І.І

B. Сєченов І.М.

C. Бернар К.

D. Павлов І.П.

E. Пашутін В.В.

12.Хто із перерахованих нижче вчених у своїх дослідженнях широко використовував порівняльно-патологічний метод?

A. Пашутін В.В.

B. Сєченов І.М.

C. Павлов І.П.

D. Бернар К.

E. Мєчніков І.І.


Фізичні фактори

13.До приймального відділення надійшов чоловік, що постраждав під час вибуху в шахті. Загальний стан задовільний. Свідомість збережена. На обличчі, руках і тулубі визначаються ділянки гіперемованої шкіри з пухирями. Який фактор переважно чинив дію на шахтаря?

A. Повітряна хвиля

B. Хімічний

C. Термічний

D. Психологічний

E. Токсичний

14.Чоловіка приблизно 50 років було знайдено на вулиці при температурі повітря +3оС. Об'єктивно: температура тіла 35,2оС, хворий промерзлий, не може самостійно рухатися. Температура пальців ніг 20oС. ЧД - 16/хв., АТ - 100/60 мм рт. ст. Тони серця приглушені, ЧСС 55 уд./хв. Найімовірніше у хворого спостерігається:

A. Відмороження ніг

B. Загальне переохолодження в стадії компенсації

C. Загальне переохолодження в стадії декомпенсації

D. Серцево-судинна недостатність

E. Гіпоглікемічна кома

15.Який механізм компенсує надмірну втрату солей натрію в організмі при тривалому потінні в умовах жаркого мікроклімату?

A. Зниження реабсорбции гідрокарбонату в нирках

B. Зниження вироблення альдостерону

C. Зниження реабсорбции гідрокарбонату в нирках

D. Зниження вироблення реніну

E. Підвищення симпатичних впливів на потові залози
Підвищення вироблення альдостерону

16.У разі дії на організм низької температури підвищується активність механізмів терморегуляції. Який з нижченаведених механізмів найефективніше обмежує тепловіддачу?

A. Брадипное.

B. Розширення судин шкіри.

C. Спазм судин шкіри.

D. Посилення потовиділення.

E. Брадикардія.

17.У хворого з опіками 40 % поверхні тіла розвинувся опіковий шок. Який механізм розвитку шоку домінує на його початку?

A. Зневоднення

B. Порушення білкового обміну

C. Аутоімунізація

D. Порушення мінерального обміну

E. Больовий

18.При температурі повітря -230С у селянина шкіра бліда, холодна, суха, температура тіла залишається нормальною. Назвіть основний критерій виникнення стадії декомпенсації при гіпотермії.

A. Зменшення випаровування з дихальних шляхів

B. Зменшення тепловипромінювання з поверхні тіла

C. Зменшення теплопроведення з поверхні тіла

D. Зниження температури ядра тіла нижче норми

E. Зниження теплопродукції

19.Який механізм тепловіддачі найбільш ефективно спрацьовує при перебуванні людини в умовах 40% вологості й температури навколишнього середовища +35оС?

A. Теплопровідність

B. Радіація

C. Потовиділення

D. Конвекція

E. Випаровування з слизової дихальних шляхів

20.У робітника, який влітку працював у щільному костюмі, різко підвищилася температура тіла, з’явилися задишка, тахікардія, запаморочення, судоми; потім він знепритомнів. Що стало причиною тяжкого стану робітника?

A. Посилення тепловіддачі.

B. Зниження тепловіддачі.

C. Зниження теплопродукції.

D. Дія пірогенів

E. Роз’єднання окислення і фосфорування

21.При температурі повітря +330С у робітника "гарячого" цеху температура тіла залишається нормальною. Назвіть основний механізм, що попереджує розвиток стадії декомпенсації:

A. Випаровування з дихальних шляхів

B. Тепловипромінювання з поверхні тіла

C. Теплопроведення з поверхні тіла

D. Посилення потовиділення

E. Зниження теплопродукції

22.У разі дії на організм високої температури підвищується активність механізмів терморегуляції. Який з нижченаведених механізмів сприяє посиленню тепловіддачі?

A. Скорочення скелетних м’язів

B. Спазм судин шкіри.

C. Брадипное.

D. Розширення судин шкіри.

E. Брадикардія.

23.В умовах високої температури повітря зменшується здатність стабілізувати системний артеріальний тиск при переході з горизонтального положення у вертикальне (ортостатична стійкість). Основна причина цього полягає в тому, що стає неможливим достатнє:

A. Рефлекторне звуження венозних ємкісних судин

B. Рефлекторне збільшення частоти серцевих скорочень

C. Рефлекторне звуження артеріальних судин опору

D. Рефлекторне збільшення сили серцевих скорочень

E. Збільшення об'єму циркулюючої крові

24.У разі дії на організм низької температури підвищується активність механізмів терморегуляції. Який з нижченаведених механізмів сприяє посиленню теплопродукції?

A. Брадипное.

B. Розширення судин шкіри.

C. Спазм судин шкіри.

D. Скорочення скелетних м’язів.

E. Брадикардія.

25.У хлопчика 10 років після тривалого перебування на пляжі виникли почервоніння і набряк шкіри плечей, обличчя і спини, пекучий біль у почервонілих ділянках, підвищення температури тіла до 37,2оС. Яке порушення спостерігається у хлопчика?

A. Ендогенна гіпертермія

B. Тепловий удар

C. Опіки

D. Опіковий шок

E. Лихоманка

26.Під час ліквідації аварії на ЧАЕС у пожежника з'явилися нудота, блю­ван­ня, пронос із кров'ю. АТ 100/60 мм. рт. ст., Ер. 4 Т/л, гемоглобін 140 г/л. Яка форма гострої променевої хвороби розвинулася в постраждалого?

A. Церебральна

B. Кістково - мозкова

C. Кишкова

D. Токсемічна (судинна)

E. Кісткова

27.Ліквідатор аварії на ЧАЕС одержав дозу опромінення 3 Гр. Які зміни в крові спостерігались у перший день після опромінення?

A. Тромбоцитопенія

B. Лейкопенія

C. Гранулоцитопенія

D. Лімфопенія

E. Еритропенія

28.Під час ліквідації наслідків аварії на Чорнобильській АЕС робітник одержав дозу опромінення 5 Гр. Скаржиться на головний біль, нудоту, запаморочення. Які зміни кількості лейкоцитів можна очікувати у хворого через 10 годин після опромінення?

A. Лейкопенію.

B. Нейтрофільний лейкоцитоз.

C. Еозинофілію

D. Лімфоцитоз.

E. Агранулоцитоз.

29.Через декілька тижнів після опромінення (7 Гр) у хворого виникла кровотеча. Які зміни в крові мають вирішальне значення в патогенезі геморагічного синдрому при гострій променевій хворобі?

A. Нейтропенія

B. Еритропенія

C. Лімфопенія

D. Еозинопенія

E. Тромбоцитопенія

30.Ліквідатор наслідків аварії на АЕС отримав дозу іонізуючого опромінення 6 Гр. Які зміни лейкоцитарної формули слід очікувати через 10 діб?

A. Гранулоцитоз, лімфопенія

B. Агранулоцитоз, лімфопенія

C. Базофілія

D. Лімфоцитоз

E. Еозинофілія

31.Через 8 діб після опромінення у потерпілого розвинувся виразково-некротичний стоматит. У крові: еритроцити - 3,0 • 1012/л, гемоглобін - 5,6 ммоль/л, лейкоцити - 1,2 • 109/л, тромбоцити – 80 • 109/л. Для якого періоду кістково-мозкової форми гострої променевої хвороби це характерно?

A. Кінця хвороби

B. Удаваного благополуччя

C. Продромальний

D. Розгорнутих клінічних ознак

E. Первинних променевих реакцій

32.Через 4 доби після опромінення потерпілий почуває себе задовільно. У крові: еритроцити - 4,2 • 1012/л, гемоглобін - 8,0 ммоль/л, лейкоцити - 5,2 • 109/л, тромбоцити – 150 • 109/л. Для якого періоду гострої променевої хвороби, кістково-мозкової форми, характерні описані зміни?

A. Кінця хвороби

B. Продромальний

C. Розгорнутих клінічних ознак

D. Прихований

E. Первинних реакцій

33.Ліквідатор аварії на ЧАЕС отримав дозу опромінення 5 Гр. Через тиждень у нього виявлено агранулоцитоз. Який патогенетичний механізм є провідним у його виникненні?

A. Збільшене руйнування лейкоцитів

B. Збільшений перехід гранулоцитів у тканини

C. Пригнічення лейкопоезу

D. Порушення виходу зрілих лейкоцитів з кісткового мозку

E. Розвиток аутоімунного процессу

34.У хворого на променеву хворобу з’явилися ознаки геморагічного синдрому. Який механізм має найбільше значення у патогенезі цього синдрому?

A. Згущення крові

B. Гіпопродукція тромбоцитів кістковим мозком

C. Руйнування тромбоцитів

D. Посилення продукції тромбопоетинів

E. Споживання тромбоцитів

35.Працівника АЕС доставили в клініку після одноразового опромінення зі скаргами на головний біль, непритомність, підвищення температури тіла, слабість, блювання, пронос. В аналізі крові - лейкоцитоз із лімфопенією. Який період променевої хвороби найбільш імовірний у пацієнта?

A. Продромальный

B. Первинних реакцій

C. Латентний

D. Розгорнутої клінічної картини

E. Удаванного благополуччя

36.Через добу після опромінення (3,5 Гр) у потерпілого в крові: еритроцити - 4,7 • 1012/л, гемоглобін - 9,0 ммоль/л, лейкоцити – 11 • 109/л, тромбоцити – 270 • 109/л. У мазку крові: базофіли – 1 %, еозинофіли – 3 %, нейтрофіли – 81 %, лімфоцити – 11 %, моноцити – 4 %. З яким періодом гострої променевої хвороби пов’язані зазначені зрушення?

A. Прихований

B. Первинних реакцій

C. Розпал хвороби

D. Відновлення

E. Завершення

37.Пацієнт потрапив до лікарні через 5 діб після опромінення (3 Гр). Недостатність якої функціональної системи є головною ланкою патогенезу гострої променевої хвороби у хворого?

A. Ендокринної

B. Дихання

C. Травлення

D. Крові

E. Кровообігу

38.Чоловік потрапив до лікарні через 3 доби після впливу іонізуючого випромінювання в дозі 3 Гр. Зміни з боку якого органу стали головними в патогенезі порушень у цьому випадку?

A. Щитоподібної залози.

B. Серця.

C. Червоного кісткового мозку.

D. Кори головного мозку.

E. Тонкої кишки.

39.Електрик, випадково доторкнувся до оголеного електропроводу обома руками і загинув. Що спричинило смерть у даному випадку?

A. Повна блокада серця

B. Синусова брадикардія

C. Фібриляція серця

D. Синусова тахікардія

E. Тампонада серця

40.Електрик, випадково доторкнувся до оголеного електропроводу однією рукою і отримав опіки. Що спричинило ураження у даному випадку?

A. Електротермічна дія струму

B. Електромеханічна дія струму

C. Електрохімічна дія струму

D. Електрофізична дія струму

E. Електробіологічна дія струму

41.Електрик, випадково доторкнувся до оголеного електропроводу однією рукою і не зміг його відпустити, доки не вимкнули струм. Що спричинило ураження у даному випадку?

A. Електротермічна дія струму

B. Електромеханічна дія струму

C. Електрохімічна дія струму

D. Електрофізична дія струму

E. Електробіологічна дія струму

42.При розгерметизації літака на висоті 15 км льотчик загинув. Причиною смерті було гостре порушення кровообігу. Який вид емболії найбільш вірогідно виник при цьому?

A. Повітряна

B. Газова

C. Тромбоемболія

D. Жирова

E. Тканинна

43.При зануренні на 100 м у аквалангіста з'явилися порушення вищої нервової діяльності і координації рухів. Що найбільш ймовірно викликає ці розлади?

A. Гіпоксемія

B. Десатурація азоту

C. Гіпокапнія

D. Гіперсатурація азоту

E. Гіперкапнія

44.Після занурення водолаза на глибину 60 м у нього з’явилися симптоми порушення функцій ЦНС: збудження, ейфорія, ослаблення уваги, професійні помилки. Токсичною дією на нейрони якої речовини зумовлені ці симптоми?

A. Кисню.

B. Вуглекислого газу.

C. Лактату.

D. Азоту.

E. Аміаку.

45.Під час аварійного підйому з глибини у водолаза розвинулися судоми із знепритомненням. Що є головною ланкою в патогенезі синдрому декомпресії?

A. Газова емболія

B. Гіперкапнія

C. Токсична дія кисню

D. Гіпоксія

E. Токсична дія азоту

46.При розвитку гірської хвороби розлади функцій організму багато в чому залежать від ступеня чутливості різних органів до гіпоксії. Вкажіть, який орган або система має найбільшу чутливість до нестачі кисню ?

A. Спинний мозок

B. Скелетна мускулатура

C. Ендокринна система

D. Тонкий кишечник

E. Головний мозок

47.На якій висоті над рівнем моря виникає закипання рідини в організмі при нормальній температурі тіла людини?

A. 15 000 м

B. 4 000 м

C. 10 000 м

D. 2 500 м

E. 19 000 м

48.Альпініст протягом декількох діб підіймався в гору. На висоті 5 000 м почали турбувати тахіпное, тахікардія, біль у вухах, головний біль розпираючого характеру. Вкажіть можливу причину явищ, що спостерігались.

A. Недостатня вентиляція легенів

B. Підвищення парціального тиску кисню в повітрі

F. Зниження барометричного тиску

C. Газова емболія

D. Зниження температури повітря

49.У туристів, які піднялися на висоту 3000 м, дихання стало частим і глибоким. Ці зміни є наслідком стимуляції:

A. Барорецепторів каротидного синуса

B. Барорецепторів дуги аорти

C. Механорецепторів легеневих альвеол

D. Хеморецепторів каротидних синусів

E. Міоцитів дихальних м'язів

50.У водолаза, що проводив роботи на великій глибині, при швидкому поверненні його в умови нормального атмосферного тиску з'явився біль у суглобах, свербіж шкіри, порушення зору, непритомність. Як називається описане явище ?

A. Баротравма

B. Хвороба декомпресії

C. Стан невагомості

D. Синдром вибухової декомпресії

E. Гіпероксія

51.При підйомі в гори на висоті 5 000 м в учасників альпіністської групи з'явилися скарги на задишку, прискорене серцебиття, головний біль, запаморочення, шум у вухах. Який патологічний фактор викликав зазначені явища?

A. Гіпотермія

B. Гіпокаліемія

C. Збільшення кисневої ємності крові

D. Лактацидемія

E. Гіпоксемія

52.При розгерметизації літака на висоті 6 км у льотчика виникли біль у вухах та носова кровотеча. Яка можлива причина вказаних симптомів?

A. Зниження парціального тиску кисню в повітрі.

B. Газова емболія.

C. Зниження барометричного тиску повітря.

D. Недостатня вентиляція легень.

E. Зниження температури повітря.

53.У водолаза, що проводив роботи на великій глибині, при швидкому поверненні його в умови нормального атмосферного тиску з'явився біль у суглобах, свербіж шкіри, порушення зору, непритомність. Як називається описане явище ?

A. Гірська хвороба

B. Кесонна хвороба

C. Стан невагомості

D. Синдром компресії

E. Синдром вибухової декомпресії

54.При розвитку гірської хвороби розвиваються зміни ряду гомеостатичних показників організму. Яка зміна носить власне патологічний характер?

A. Гіпертензія

B. Тахіпное

C. Тахікардія

D. Гіпоксемія

E. Централізація кровообігу

55.Після вимушеного швидкого підняття водолаза з глибини на поверхню у нього з’явилися симптоми кесонної хвороби: біль у суглобах, свербіння шкіри, мерехтіння в очах, запаморочення свідомості. Яким видом емболії зумовлений розвиток цих симптомів?

A. Жирова

B. Тромбоемболія

C. Тканинна

D. Газова

E. Повітряна


Спадковість та конституція

56. До медико-генетичної консультації звернулася жінка з приводу ризику захворювання на гемофілію у свого сина. Її чоловік страждає на дане захворювання з народження. Жінка та її батьки здорові стосовно гемофілії. Яка імовірність народження хворого хлопчика в цій сім'ї?

A. 50 % хлопчиків будуть хворими

B. 75 % хлопчиків будуть хворими

C. 25 % хлопчиків будуть хворими

D. Усі хлопчики будуть здорові

E. Усі хлопчики будуть хворі

57. До медико-генетичної консультації звернулася жінка з приводу ризику захворювання на фосфопенічний рахіт у своєї дочки. Її мати страждає на дане захворювання з народження. Жінка та її чоловік здорові. Яка імовірність народження хворої дівчинки у цій сім'ї?

A. Усі дівчатка будуть здорові

B. 75 % дівчаток будуть хворими

C. 25 % дівчаток будуть хворими

D. 50 % дівчаток будуть хворими

E. Усі дівчатка будуть хворі

58. До медико-генетичної консультації звернулася жінка з приводу ризику захворювання на дальтонізм у своєї дочки. Її батько страждає на дане захворювання з народження. Жінка та її чоловік здорові стосовно дальтонізму. Яка імовірність народження хворої дівчинки в цій сім'ї?

A. 25 % дівчатка будуть здорові

B. 75 % дівчаток будуть хворими

C. Усі дівчаток будуть хворими

D. 50 % дівчаток будуть хворими

E. Усі дівчатка будуть здорові

59. До медико-генетичної консультації звернулася жінка з приводу ризику захворювання на дальтонізм у своєї дитини. Її чоловік страждає на дане захворювання з народження. Жінка та її батьки здорові стосовно дальтонізму. Яка імовірність народження хворої дитини в цій сім'ї?

A. 75 % діти будуть здорові

B. Усі дітей будуть хворими

C. 25 % дітей будуть хворими

D. 50 % дітей будуть хворими

E. Усі діти будуть здорові

60. До медико-генетичної консультації звернулася жінка з приводу ризику захворювання на гемофілію у своєї дитини. Її батько страждає на дане захворювання з народження. Жінка та її чоловік здорові стосовно гемофілії. Яка імовірність народження хворої дитини в цій сім'ї?

A. Усі діти будуть хворі

B. 75 % дітей будуть хворими

C. Усі діти будуть здорові

D. 50 % дітей будуть хворими

E. 25 % дітей будуть хворими

61. Яка вірогідність народження хворих синів у здорової жінки, батько якої страждає на атрофію здорових нервів (тип успадкування такий, як при гемофілії), а чоловік здоровий?

A. 25%

B. 0%

C. 50%

D. 75%

E. 100%.

62. Яка вірогідність народження хворих дочок у подружньої пари, де чоловік хворіє на гіпофосфатемічний рахіт, а жінка здорова?

A. 75%.

B. 0%

C. 25%

D. 50%

E. 100%

63. Чоловік хворіє на брахідактилію, а дружина здорова. Відомо, що його мати була здоровою, а батько хворів на брахідактилію. Яка вірогідність народження дітей з брахідактилією у цієї подружньої пари?

A. 100%

B. 0%

C. 25%

D. 75%

E. 50%.

64. До лікаря-генетика звернулися батьки юнака 18 років. Об’єктивно виявлені виражена розумова відсталість та гіпогонадизм. Попередній діагноз: синдром Клайнфельтера. Який метод медичної генетики дозволяє підтвердити цей діагноз?

A. Генеалогічний

B. Цитологічний

C. Популяційно-статистичний

D. Біохімічний

E. Близнюковий

65. Під час обстеження юнака з розумовою відсталістю виявлено євнухоїдну будову тіла, недорозвиненість статевих органів. Який метод генетичного дослідження слід застосувати для уточнення діагнозу?

A. Близнюковий

B. Генеалогічний

C. Цитологічний

D. Біохімічний

E. Популяційно-статистичний

66. У пологовому будинку народилася дитина з численними порушеннями як зовнішніх так і внутрішніх органів – серця, нирок, травної системи. Був встановлений попередній діагноз – синдром Дауна. Який метод генетичного дослідження слід застосувати для підтвердження діагнозу?

A. Популяційно-статистичний

B. Біохімічний

C. Цитологічний

D. Близнюковий

E. Генеалогічний

67. Лікар запідозрив у дитини синдром Шерешевського – Тернера. Який метод генетичного обстеження дасть можливість встановити цей діагноз?

A. Цитологічний

B. Генеалогічний

C. Біохімічний

D. Демографо-статистичний

E. Близнюковий

68. У населення України частота гетерозигот за геном фенілкетонурії становить 3 %. Який метод генетичного дослідження використовується для раннього виявлення фенілкетонурії у новонароджених?

A. Генеалогічний

B. Цитогенетичний

C. Близнюковий

D. Біохімічний

E. Популяційно-статистичний

69. При дослідженні каріотипу 7-річного хлопчика виявлено 46 хромосом. Одне плече 15-ї хромосоми коротше від іншого. Який вид мутації має місце у цієї дитини?

A. Нестача

B. Дуплікація

C. Інверсія

D. Ампліфікація

E. Делеція

70. У клітинах людини відбулося відновлення пошкодженої ділянки молекули ДНК за непошкодженим ланцюгом з допомогою специфічних ферментів. Яку назву має це явище?

A. Реплікація

B. Термінація

C. Дуплікація

D. Репарація

E. Ініціація

71. У клітинах людини під дією ультрафіолетового випромінювання відбулося пошкодження молекули ДНК, але спрацювала система репарації ДНК. Який з названих ферментів входить до цієї системи?

A. Ліпаза

B. Каталаза

C. Трипсин

D. Амілаза

E. Лігаза

72. Внаслідок впливу гама-випромінювання ділянка ланцюга ДНК подвоїлась. Який з наведених видів мутацій виник у ланцюзі ДНК?

A. Делеція

B. Інверсія

C. Реплікація

D. Транслокація

E. Дуплікація

73. Під час медико-генетичного консультування родини зі спадковою патологією виявлено, що аномалія проявляється через покоління у чоловіків. Який тип успадковування притаманний для цієї хвороби?

A. Х-зчеплене домінантне

B. Х-зчеплене рецесивне домінантне

C. Y-зчеплене

D. Аутосомне домінантне

E. Аутосомне рецесивне

74. Під час диспансерного обстеження хлопчика 7 років встановлено діагноз: дальтонізм. Батьки здорові, колірний зір нормальний. У дідуся по материнській лінії виявлена така сама аномалія. Який тип успадкування цієї патології?

A. Домінантний, зчеплений зі статтю

B. Рецесивний, зчеплений зі статтю

C. Аутосомно-рецесивний

D. Неповне домінування

E. Аутосомно-домінантний

75. Під час медико-генетичного консультування родини зі спадковою патологією виявлено, що аномалія проявляється у кожному поколінні у чоловіків. Який тип успадковування притаманний для цієї хвороби?

A. Аутосомне домінантне

B. Х-зчеплене домінантне

C. Х-зчеплене рецесивне

D. Y-зчеплене

E. Аутосомне рецесивне

76. Під час медико-генетичного консультування родини з ахондроплазією виявлено, що аномалія проявляється у кожному поколінні як у чоловіків, так і в жінок. Який тип успадковування притаманний для цієї хвороби?

A. Неповне домінанування

B. Аутосомне рецесивне

C. Аутосомне домінантне

D. Х-зчеплене рецесивне

E. Y-зчеплене

77. Під час медико-генетичного консультування дитини з фенілкетонурією виявлено, що це захворювання у сім'ї діагностоване вперше. Який тип успадковування притаманний для цієї спадкової аномалії?

A. Аутосомне домінантне

B. Х-зчеплене домінантне

C. Х-зчеплене рецесивне

D. Y-зчеплене

E. Аутосомне рецесивне

78. Аналіз родоводу встановив аутосомно-домінантний тип успадкування захворювання. Яка з названих нижче хвороб має такий тип успадковання?

A. Галактоземія.

B. Гемоглобінопатія.

C. Полідактилія.

D. Глікогеноз.

E. Синдром Шерешевського-Тернера.

79. Аналіз родоводу встановив полігенний тип успадкування захворювання. Яка з названих нижче хвороб має такий тип успадковання?

A. Ахондроплазія.

B. Вітамін D2-резистентний рахіт.

C. Хвороба Дауна.

D. Цукровий діабет.

E. Гемофілія.

80. Дитина страждає на спадкову хворобу. Яка хвороба з названих нижче успадковується за рецесивним типом, зчепленим з Х-хромосомою?

A. Серпоподібноклітинна анемія.

B. Синдром Клайнфельтера.

C. Трисомія за Х-хромосомою.

D. Гіпогаммаглобулінемія Брутона.

E. Гіпофосфатемічний рахіт.

81. Дитина страждає на спадкову хворобу. Яка з названих нижче хвороб передається за аутосомно-рецесивним типом?

A. Алкаптонурія.

B. Короткопалість.

C. Гіпогаммаглобулінемія Брутона.

D. Гемофілія.

E. Дальтонізм.

82. Дитина страждає на спадкову хворобу. Яка з названих нижче хвороб передається за аутосомно-домінантним типом?

A. Фенілкетонурія.

B. Гемофілія.

C. Галактоземія.

D. Полідактилія.

E. Дауна.

83. Дитина страждає на спадкову хворобу. Яка з названих нижче хвороб передається за рецесивним типом, зчепленим з Х-хромосомою?

A. Гіпофосфатемічний рахіт.

B. Полідактилія.

C. Серпоподібноклітинна анемія.

D. Гемофілія А.

E. Фенілкетонурія.

84. Дитина страждає на спадкову хворобу. Яка з названих нижче хвороб передається за аутосомно-рецесивним типом?

A. Полідактилія.

B. Короткопалість.

C. Альбінізм.

D. Гіпогаммаглобулінемія Брутона.

E. Дальтонізм.

85. У хворої людини в ядрах соматичних клітин виявлено одне тільце Барра (статевий хроматин). При якому варіанті каріотипу це спостерігається?

A. 47, XY, 21+

B. 45, ХО

C. 45, XY

D. 47, XX, 15+

E. 48, XXXY.

86. У хворого виявлено синдром Клайнфельтера. Який у нього каріотип?

A. 47, XY, 15

B. 45, ХО

C. 45, YO

D. 47, XXY

E. 48, XXXX.

87. У хворої людини в ядрах соматичних клітин виявлено одне тільце Барра (статевий хроматин). При якому варіанті каріотипу це спостерігається?

A. 47, XY, 15+

B. 45, Х

C. 47, XY, 21+

D. 47, XXY

E. 48, XXXY.

88. У хворого виявлено “ускладнений” синдром Клайнфельтера. Який каріотип може бути у нього?

A. 45, ХО

B. 49, XXXXY.

C. 45, YO

D. 47, XY, 15

E. 46, XY

89. У дитини діагностовано вроджений сифіліс. Виберіть найбільш точне визначення вроджених хвороб.

A. Хвороби, зумовлені патологією пологів.

B. Це Усі спадкові хвороби.

C. Спадкові хвороби з домінантним типом успадкування.

D. Хвороби, з якими людина народжується.

E. Трансплацентарні інфекційні хвороби.

90. Дитина народилась зі спадковою хворобою. Яка хвороба є водночас вродженою і спадковою?

A. Хорея Гентінгтона.

B. Гемолітична хвороба новонароджених.

C. Брахідактилія.

D. Подагра

E. Вроджений сифіліс.

91. Спадкова хороба вперше виявила себе у дорослому віці. Яка хвороба є спадковою, але не вродженою?

A. Синдром Дауна.

B. Сифіліс.

C. Полідактилія.

D. Спадкова глухонімота.

E. Хорея Гентінгтона.

92. Дитина народилась з патологією. Яка хвороба є водночас вродженою і не спадковою?

A. Імунодефіцит з тромобоцитопенією та екземою.

B. Серпоподібноклітинна анемія.

C. Гемолітична хвороба новонароджених (резус-конфлікт).

D. Фенілкетонемія.

E. Брахідактилія.

93. У хлопчика діагностовано хворобу Дауна. Яка зміна у хромосомах є причиною цієї хвороби?

A. Трисомія 21

B. Трисомія 13

C. Моносомія Х

D. Трисомія 18

E. Трисомія Х

94. Чоловік 28 років звернувся до лікаря з приводу безпліддя. Має високий зріст, довгі кінцівки. У ядрах букальних епітеліоцитів пацієнта виявлено по одному тільцю Барра. Для якої патології характерні зазначені порушення?

A. Синдром Луї - Барр

B. Синдром Віскотта - Олдріча

C. Синдром Клайнфельтера

D. Акромегалія

E. Гігантизм

95. У жінки 45 років народився хлопчик з розщепленням верхньої щелепи (“заяча губа” та “вовча паща”). Під час додаткового обстеження виявлені значні порушення з боку нервової, серцево-судинної систем та зору. При дослідженні каріотипу діагностована трисомія по 13 хромосомі. Який синдром має місце у хлопчика?

A. Едвардса

B. Шерешевського-Тернера

C. Патау

D. Дауна

E. Клайнфельтера

96. До медико-генетичної консультації звернулася жінка з приводу відхилень фізичного та статевого розвитку. Під час мікроскопії клітин слизової оболонки ротової порожнини статевого хроматину не виявлено. Для якої хромосомної патології це характерно?

A. Трисомія за Х-хромосомою

B. Синдром Клайнфельтера

C. Хвороба Дауна

D. Синдром Віскотта – Олдріча

E. Синдром Шерешевського—Тернера

97. У експериментального собаки слабкий тип конституції. За якою класифікацією виділено цей тип?

A. Чорноруцького

B. Сиго

C. Павлова

D. Кречмера

E. Богомольця

98. Хворий на шизофренію має астенічний тип конституції. За якою класифікацією виявляється такий зв'язок між будовою тіла та схильністю до психічних захворювань?

A. Гіпократа

B. Богомольця

C. Сиго

D. Кречмера

E. Павлова

99. Юнак, 18 років, високий на зріст, з міцними кістками та розвиненими м’язами. Який у нього тип конституції за Сиго?

A. Травний

B. М’язовий

C. Дихальний

D. Мозковий

E. Гіперстенічний

100. Хлопець 18 років, зріст 180см, вага 130 кг, дещо надмірне відкладення жиру в області живота і стегон, страждає запорами. Який у нього тип конституції за Сиго?

A. Мозковий

B. М’язовий

C. Дихальний

D. Травний

Е. Гіперстенічний

101. Хлопець, 15 років високий на зріст, худий, з тонкими кістками, часто хворіє на респіраторні захворювання. Який у нього тип конституції за Богомольцем?

A. Астенічний

B. Дихальний

C. Слабкий

D. Нормостенічний

E. Гіперстенічний

102. У чоловіка атлетичний тип конституції за класифікацїєю Кречмера. Серед хворих на які хвороби частіше зустрічається цей тип конституції?

A. Туберкульоз легень.

B. Вірусні респіраторні інфекції.

C. Епілепсію.

D. Гіпоацидний гастрит.

E. Шизофренію.

103. У хворого виявлено астенічний тип конституції за класифікацією Кречмера. Серед хворих на які хвороби частіше зустрічається цей тип конституції?

A. Гіпоацидний гастрит.

B. Вірусні респіраторні інфекції.

C. Туберкульоз легень.

D. Шизофренію.

E. Істерію.

104. У хворого гіперстенічний тип конституції за М.В.Чорноруцьким. Який принцип покладений в основу цієї класифікації?

A. Особливості основних функцій організму і обміну речовин.

B. Переважний розвиток фізіологічної системи.

C. Переважання одного з основних соків організму.

D. Переважання першої або другої сигнальної системи.

E. Особливості структури сполучної тканини.

105. У дитини з екзематозними ураженнями шкіри виявлено ексудативно-катаральний діатез. Що таке діатез?

A. Аномалія конституції з неадекватною реакцією організму на подразники.

B. Захворювання раннього дитячого віку.

C. Аномалія внутрішньоутробного розвитку.

D. Схильність людини до певних захворювань.

E. Спадкові ферментопатії.

106. У дитини з хронічним тонзилітом та аденоїдними вегетаціями виявлено лімфатико-гіпопластичного діатез. Що ще характерне для цього діатезу?

A. Збудливість нервової системи.

B. Зменшення лімфатичних вузлів.

C. Лімфопенія.

D. Затримка інволюції вилочкової залози.

E. Подагра.

107. Людина має астенічний тип конституції за класифікацією О.О.Богомольця. Що характерне для цього типу?

A. Схильність до гіпертонічної хвороби.

B. Переважання процесу гальмування над збудженням у нервовій системі.

C. Нерішучість, пригніченість, відлюдкуватість.

D. Переважний розвиток системи дихання.

E. Тонка, ніжна сполучна тканина.

108. Людина має мозковий тип конституції за класифікацією Сиго. Який ще тип конституції відноситься до цієї класифікації?

A. Гіперстенік.

B. Травний.

C. Ліпоматозний.

D. Пастозний.

E. Пікнічний.

109. Людина має астенічний тип конституції за класифікацією О.О.Богомольця Який принцип покладений в основу цієї класифікації?

A. Переважання одного з основних соків організму.

B. Переважний розвиток фізіологічної системи.

C. Особливості структури сполучної тканини.

D. Переважання першої або другої сигнальної системи.

E. Особливості основних функцій організму і обміну речовин.


Біологічні фактори

110. Хворі з парціальною недостатністю В-системи імунітету і, відповідно, гуморальної відповіді, страждають на різні бактеріальні інфекції. У той же час, вони захищені від вірусів, рикетсій, хламідії, найпростіших та грибів. Яка властивість мікроорганізмів є визначальною для ввімкнення конкретного типу імунної відповіді – гуморальної або клітинної?

A. Вірулентність

B. Геометричні розміри

C. Розташування відносно клітини (поза-, внутрішньо-)

D. Патогенність

E. Стійкість до антибіотика

111. У хворого на бактеріальну (стафілококкову) пневмонію в крові виражений лейкоцитоз, високі титри антитіл G. Які властивості збудника інфекції є причиною ввімкнення гуморальної відповіді імунної системи?

A. Вірулентність

B. Внутрішньоклітинне розміщення

C. Геометричні розміри

D. Позаклітинне розміщення

E. Патогенність

112. У хворого на кавернозний туберкульоз легені в крові лімфо-, моноцитоз. Утворення каверни є наслідком гіперергічного розвитку запалення в легені, під час якого лімфоцити – хелпери намагаються допомогти макрофагам знищувати мікобактерії. Які властивості збудника інфекції є причиною ввімкнення клітинної відповіді імунної системи?

A. Позаклітинне розміщення

B. Геометричні розміри

C. Вірулентність

D. Патогенність

E. Внутрішньоклітинне розміщення

113. У крові хворого на вітряну віспу визначаються лімфо-, моноцитоз. Які властивості вірусної інфекції є причиною ввімкнення клітинної відповіді імунної системи?

A. Геометричні розміри

B. Внутрішньоклітинне розміщення

C. Вірулентність

D. Патогенність

E. Позаклітинне розміщення

114. Місце запалення при хламідійній інфекції характеризується вираженою інфільтрацією тканини лімфоцитами і моноцитами. Які властивості хламідійної інфекції є причиною ввімкнення клітинної відповіді імунної системи?

A. Позаклітинне розміщення

B. Геометричні розміри

C. Вірулентність

D. Патогенність

E. Внутрішньоклітинне розміщення

115. При вірусній інфекції віруси здатні виживати і розмножуватися в клітині. Які саме властивості вірусних білків дозволяють їм це робити?

A. Маленькі розміри

B. Термостабільність

C. Аналогія з клітинними білками

D. Стійкість до антисептиків

E. Правильна відповідь відсутня

116. Стратегією виживання деяких вірусів є їх здатність спричинювати надмірну проліферацію інфікованих клітин та виживати в несприятливих умовах імунної відповіді. Такі перетворення вірус-інфікованих клітин визначаються як пухлинна трансформація, а віруси, здатні її спричинювати – онкогенні. Які властивості вірусних білків дозволяють їм це робити?

A. Інактивувати транскрипцію

B. Активувати апоптоз

C. Інактивувати ініціатори проліферації

D. Вмикати поділ, уникати апоптозу

E. Правильна відповідь відсутня

117. Які віруси не мають онкогенних властивостей (можливості індукції пухлинної трансформації клітин)?

A. Герпес

B. Адено

C. Ентеро

D. Лімфоцитотропні ( HTLV, HIV)

E. Папіломи

118. Губчаста енцефалопатія підтвердила діагноз хвороби Крейтцфельда-Якоба в померлого. До якої групи належить інфекційний агент, що спричинює це захворювання?

A. Пріони

B. Найпростіші

C. Віруси

D. Бактерії

E. Гриби

119. Спонгіоформна енцефалопатія у хворих на хворобу Крейтцфельда-Якоба спричинена дією пріонного білка PrPSc. Чим відрізняється патогенний білок PrPSc від нативного білка людини PrPC?

A. Ферментативною активністю

B. Первинною структурою

C. Конформацією

D. Ліпотропністю

E. Гідрофільністю

120. У чоловіка 46 років, хворого на деменцію, діагностовано губчасту енцефалопатію. Що є головною ланкою в патогенезі дегенеративних змін нейронів при повільних пріонових інфекціях?

A. Перевантаження нейронів власними спотвореними протеїнами PrPC

B. Цитотоксична дія антитіл

C. Знищення PrPSc – перевантажених нейронів лімфоцитами

D. Ушкодження PrPSc - інфікованих нейронів цитокінами

E. Дефіцит ендогенних опіатів

121. Причиною інфекційного мононуклеозу є вірус Епстайна - Барра. Який білок на поверхні лімфоцитів цей вірус використовує для проникнення до клітин?

A. Рецептор СD8

B. Рецептор СD4

C. Рецептори комплементу 2 типу (СR2)

D. Глікопротеїн gp120

E. Глікопротеїн gp105

122. У ВІЛ–інфікованих людей відбувається поступове зменшення кількості лімфоцитів–хелперів. Який білок на поверхні лімфоцитів ВІЛ використовує для проникнення до клітин?

A. СD 8

B. Рецептор інтерлейкіну 1

C. Рецептор інтерлейкіну 2

D. СD 4

E. gp 120

123. У хворого на пневмоцистну пневмонію діагностовано СНІД. Який власний білок ВІЛ використовує для проникнення в клітини - мішені?

A. CD21

B. СD56

C. СD4

D. СD8

E. gp120

124. Після інфікування у хворого розвинувся хронічний активний гепатит В. Що є механізмом розвитку цього захворювання?

A. Вірусна інфекція

B. Імунне ураження гепатоцитів

C. Поєднана бактеріальна інфекція

D. Цитотоксична дія вірусу

E. Цитотоксична дія антибіотиків

125. У хворого виявлений дифузний шкірний і прогресуючий вісцеральний лейшманіоз. Який фермент дозволяє лейшманії швидко знешкоджувати активні форми кисню і уникати фагоцитозу?

A. РНК-полімераза

B. Каталаза

C. Лактамаза

D. Супероксиддисмутаза

E. ДНК-полімераза

126. У дитини, яка часто хворіє на піодермію, діагностовано порушення фагоцитозу, що спричинено спадковим дефектом обміну полісахаридів у фагоцитах (хвороба Альдера). Які саме клітини недієздатні при цьому?

A. Еозинофіли

B. Моноцити

C. Нейтрофіли

D. Лімфоцити

E. Дендритні клітини

127. Лікування дитини сироваткою проти кору привело до одужання маленького пацієнта. Зі створенням якого виду резистентності пов’язано одужання дитини при цьому?

A. Неспецифічна пасивна

B. Неспецифічна активна

C. Специфічна активна

D. Специфічна пасивна

E. Первинна

128. Дівчина, яка довго знаходилася на холоді, змерзла. Щоб зігрітися, вона намагається активно рухатися. Який універсальний адаптивний механізм забезпечує підтримання температури її тіла в межах норми за таких надзвичайних умов?

A. Фагоцитоз

B. Шок

C. Імунна відповідь гуморальна

D. Імунна відповідь клітинна

E. Стрес

129. Чоловік з переломом кінцівки втратив велику кількість крові. При цьому в нього артеріальний тиск крові 100/60 мм. рт. ст., частота серцевих скорочень 96 за 1 хв. Який універсальний адаптивний механізм забезпечує діяльність системи кровообігу в межах норми за таких надзвичайних умов?

A. Колапс

B. Шок

C. Стрес

D. Імунна відповідь

E. Фагоцитоз

130. Загальний стан хворого з невеликим інфарктом лівого шлуночка відносно задовільний. Артеріальний тиск крові 115/75 мм. рт. ст., частота серцевих скорочень 96 за 1 хв. Який універсальний адаптивний механізм забезпечує діяльність системи кровообігу в межах норми за таких надзвичайних умов?

A. Стрес

B. Шок

C. Колапс

D. Імунна відповідь

E. Фагоцитоз

131. Після крововтрати артеріальний тиск крові в пацієнта є 105/65 мм. рт. ст., частота серцевих скорочень 98 за 1 хв. Які гормони вмикають адаптивні перебудови в організмі при цьому?

A. Альдостерон, АДГ

B. Інсулін, тестостерон

C. Адреналін, кортизол, АДГ

D. Інсулін, соматотропін

E. Інсулін, глюкагон

132. У дитини, яка хворіє на піодермію, встановлено порушення фагоцитозу, яке пов’язують з дефіцитом НАДФ - оксидази в нейтрофілах. Яка стадія фагоцитозу порушена при цьому?

A. Поглинання

B. Хемотаксис

C. Адгезія

D. Перетравлення

E. Створення фагосоми

133. У дитини після вакцинації виникла стійкість до дифтерії. Як найповніше охарактеризувати резистентність, яка є в основі цієї стійкості:

A. Набута неспецифічна активна

B. Природжена активна

C. Набута специфічна пасивна

D. Набута специфічна активна

E. Природжена неспецифічна пасивна

134. Хворий спадковим синдромом Чедіака - Хігасі страждає від гнійничкових уражень шкіри, часто хворіє на гнійні отити, тонзиліти. Дефект яких органел фагоцитів є причиною недостатності фагоцитозу при цьому синдромі?

A. Рибосоми

B. Мітохондрії

C. Пероксисоми

D. Лізосоми

E. Комплекс Гольджі

135. Собака захворів чумкою, тоді як його господар залишився здоровим. Який вид реактивності лежить в основі такої стійкості господаря?

A. Індивідуальна

B. Видова

C. Групова

D. Патологічна специфічна

E. Гіперергічна

136. У пацієнта часто виникає ангіоневротичний набряк ока або губи. Зі спадковим дефіцитом якого білка пов’язаний розвиток набряку Квінке?

A. α1- антитрипсину

B. Мієлопероксидази

C. Інгібітора комплементу С1

D. Фібриногену

E. Антитіл А


Імунологічна реактивність

137. У піддослідних шимпанзе викликали стан стану первинної недостатності клітинного імунітету. Яка з експериментальних методик може бути застосована для цього?

A. Введення глюкокортикоїдів

B. Видалення тимуса

C. Застосування цитостатиків

D. Опромінення

E. Інфікування ВІЛ

138. Хворому на гострий лейкоз проведена операція з трансплантації донорського кісткового мозку. Яке ускладнення операції може створювати реальну загрозу для життя хворого в післяопераційному періоді?

A. Імунодепресія

B. Цитотоксична реакція 2-го типу

C. «Трансплантат проти хазяїна

D. Анафілаксія

E. Стрес

139. П’ятирічний хлопчик з народження хворіє на різні інфекції (вірусні, бактеріальні, грибкові). При обстеженні в нього виявлено зменшення кількості лімфоцитів, відсутність IgА, замала кількість IgG. Як можна визначити порушення функції імунної системи у хворого?

A. Стрес

B. Первинний гуморальний імунодефіцит

C. Первинний комбінований імунодефіцит

D. Вторинний імунодефіцит

E. Алергія

140. Дитина 1 року постійно хворіє на вірусні та грибкові інфекції. При обстеженні виявлено відсутність Т - лімфоцитів. Що може бути причиною такого стану імунітету в дитини?

A. Гіперплазія тимусу

B. Внутрішньоутробна гіпотрофія

C. Пологова травма

D. Стрес

E. Гіпоплазія тимусу

141. Хлопчик 9 місяців страждає на гнійні інфекції. При обстеженні: в кістковому мозку - нормальна кількість пре-В-клітин, відсутність плазматичних клітин; в крові - відсутність В-лімфоцитів та антитіл, нормальна кількість Т-клітин. Кваліфікуйте стан імунної системи в дитини:

A. СНІД

B. Вроджений комбінований імунодефіцит

C. Первинна недостатність фагоцитозу

D. Вроджена недостатність гуморального імунітету

E. Парціальна недостатність клітинного імунітету

142. Однорічний хлопчик постійно страждає на гнійні інфекції. Обстеження показало в дитини спадкове захворювання імунної системи – гіпогаммаглобулінемію Бруттона. Як успадковується патологічний ген, що є причиною цього захворювання?

A. Кодомінантно

B. Зчеплено з У-хромосомою

C. Аутосомно – домінантно

D. Аутосомно – рецесивно

E. Зчеплено з Х – хромосомою

143. Хлопчик 1,5 років постійно хворіє на інфекції дихальних шляхів, травного каналу. Спадкова недостатність продукції яких антитіл є найбільш вірогідною причиною частого інфікування дитини?

A. Ig G

B. Ig M

C. Ig A

D. Ig E

E. Ig E, М

144. Батьків 6-ти річного хлопчика турбує зниження апетиту і поганий сон у дитини. При обстеженні з’ясовано, що хлопчик інфікований аскаридами. Антитіла якого класу сприяють еміграції еозинофілів до місця глистяної інвазії і, таким чином, мають значення в захисті від гельмінтів?

A. Ig E

B. Ig A

C. Ig M

D. Ig G

E. Ig D

145. У ВІЛ-інфікованого пацієнта діагностовано зниження функції антигенпрезентуючих клітин. Які це клітини?

A. Т хелпери

B. Т кілери

C. Макрофаги

D. Нейтрофіли

E. Природні кілери


146. У ВІЛ-інфікованого пацієнта діагностовано зниження функції антигенпрезентуючих клітин. З молекулами клітинної поверхні якого типу відбувається презентація антигенів Т-хелперам антигенпрезентуючими клітинами?

A. МНС І

B. МНС ІІ

C. СD4

D. CD8

E. CD21

147. Однією з причин недостатньої імунної відповіді на Herpes інфекцію є здатність Herpes вірусів зменшувати експресію білків на поверхні інфікованих клітин. За якими молекулами відбувається ідентифікація інфікованих клітин як мішеней лімфоцитами- кілерами?

A. CD8

B. МНС ІІ

C. СD4

D. МНС І

E. CD21

148. Після інтенсивної хіміотерапії злоякісної пухлини у хворого виникло порушення імунної системи. Який вид недостатності імунітету виник у нього?

A. Вторинна недостатність В - системи

B. Первинний комбінований імунодефіцит

C. Первинна недостатність Т - системи

D. Парціальна недостатність фагоцитозу

E. Вторинний комбінований імунодефіцит

149. Пацієнту з вираженим кандидозом ротової порожнини рекомендовано подальше обстеження для вирішення питання про можливу ВІЛ- інфекцію. Зі зменшенням кількості яких клітин в організмі пов'язаний розвиток СНІДу?

A. Т CD8

B. Т CD4

C. Природні кілери CD56

D. Макрофаги

E. Еозинофіли

150. Чоловік потрапив до лікарні з симптомами менінгіту. При обстеженні: криптококовий менінгіт; недостатність функції макрофагів, критичне зменшення кількості CD4 Т лімфоцитів в крові та величини співвідношення CD4/CD8 до 0,5. Кваліфікуйте стан імунної системи в пацієнта:

A. Первинна недостатність клітинної відповіді

B. Латентна ВІЛ - інфекція

C. Первинна недостатність фагоцитозу

D. СНІД

E. Первинна недостатність гуморальної відповіді

151. Хворий скаржиться на загальну слабкість, постійний кашель, пітливість, підвищення температури тіла. При обстеженні діагностовано пневмоцистну пневмонію, СНІД. Первинна недостатність функції яких клітин є причиною опортуністичної інфекції при СНІДі?

A. Природні кілери, Т- кілери

B. Т- кілери, еозинофіли

C. Макрофаги, Т - хелпери

D. В - лімфоцити

E. Кровотворні клітини

152. У хворого на ожиріння, гіперглікемію, інфекцію, що постійно рецидивує, виявлено пухлину кори наднирників, що продукує надмірну кількість гормонів. Надлишок яких гормонів є причиною недостатності імунітету в пацієнта?

A. Альдостерону

B. Адреналіну

C. Кортизолу

D. Андрогенів

E. Інсуліну

153. У патогенезі ураження ЦНС при СНІДі має значення вторинна інфекція, запалення і токсична дія цитокінів на нейрони. Які клітини транспортують ВІЛ до ЦНС?

A. Еритроцити

B. Нейтрофіли

C. Макрофаги

D. В-лімфоцити

E. Т-лімфоцити

154. Огляд показав множинний карієс у дитини. Недостатня продукція яких антитіл може бути причиною полегшеного інфікування ротової порожнини Strеptococcus mutans з подальшим порушенням їх фагоцитозу нейтрофілами?

A. Ig Е, G

B. Ig A , E

C. Ig Е, M

D. Ig A, G

E. Ig D, M


155. Хлопчик, 2 років, народжений від ВІЛ – інфікованої матері, також ВІЛ – інфікований. Він регулярно піддається обстеженню з метою завчасної діагностики СНІДу. Зменшення кількості яких клітин в крові є достовірним критерієм розвитку СНІДу?

A. CD8 Т

B. CD4 Т

C. CD21 В

D. CD56 НК

E. Моноцити

156. Немовля з вродженим стенозом аорти страждає на грибкову і вірусну інфекцію. При обстеженні: нормальна кількість пре-В- і плазматичних клітин у кістковому мозку; нормальна кількість В-, відсутність Т-клітин у циркуляції. Який стан імунітету у дитини?

A. Імунодепрессія

B. Гіпогаммаглобулінемія Бруттона

C. СНІД

D. Комбінований імунодефіцит

E. Синдром Ді Джорджи

157. Немовля з гіпоплазією тимуса та прищитовидних залоз страждає на тетанію, інфекції. При обстеженні: нормальна кількість пре-В- і плазматичних клітин у кістковому мозку; нормальна кількість В-, відсутність Т-клітин у циркуляції. Яка інфекція є типовою для такого стану імунітету?

A. Гельмінти

B. Стрептококи

C. Віруси

D. Пріони

E. Стафілококи

158. У чоловіка, що вже місяць хворіє на пневмонію, виявляються збільшені лімфатичні вузли в ділянці шиї, під пахвами. В крові хворого майже відсутні зрілі нейтрофіли. Дослідження кісткового мозку показало «Гострий лейкоз». Інфекція якого типу є закономірною для станів з недостатністю нейтрофілів та фагоцитозу?

A. Піогенні коки

B. Гельмінти

C. Віруси, що спричинюють цитоліз

D. Віруси, що «брунькуються»

E. Грибки

159. Після курсу терапії лімфогранулематозу в пацієнта розвинулася пневмонія. Який з терапевтичних заходів став причиною недостатності імунітету?

A. Застосування наркотичних анальгетиків

B. Застосування вітаміну А

C. Застосування вітаміну С

D. Хіміотерапія

E. Застосування вітаміну В6

160. У хлопчика з лімфомою розвинулася важка імунодепресія після застосування хіміотерапії і гормонів під час лікування лімфоми. Які гормони спричинюють апоптоз лімфоцитів?

A. Андрогени

B. Естрогени

C. Глюкокортикоїди

D. Інсулін

E. Соматотропін


Автоімунні хвороби

161. Провідною ланкою в патогенезі автоімунних хвороб є зняття толерантності імунної системи до власних антигенів. Що може бути причиною цього порушення імунної системи?

A. Недостатність фагоцитів

B. Апоптоз лімфоцитів

C. Знищення лімфоцитів в тимусі

D. Інактивація лімфоцитів цитокінами

E. Презентація автоантигенів

162. Провідною ланкою в патогенезі автоімунних хвороб є зняття толерантності імунної системи до власних антигенів. Що може бути причиною цього порушення імунної системи?

A. Відсутність презентації автоантигенів

B. Модифікація автоантигену

C. Інактивація лімфоцитів цитокінами

D. Апоптоз лімфоцитів

E. Недостатність фагоцитів

163. Яке автоімунне захворювання розвивається за загальним з "сироватковою хворобою" механізмом?

A. Автоімунна гемолітична анемія

B. Дифузний токсичний зоб (хв. Базедова)

C. Цукровий діабет 1 типу

D. Системний червоний вовчак

E. Автоімунна тромбоцитопенія

164. Системний червоний вовчак (СЧВ) є автоімунним захворюванням, при якому надвелика кількість імунних комплексів автоантиген-антитіло-комплемент спричинюють пошкодження і запалення нирок, суглобів, шкіри, ЦНС, інших тканин. На який автоантиген реагує імунна система при СЧВ?

A. Білки поверхні В-клітин островків Лангенгарса

B. Білки поверхні еритроцитів

C. Білки поверхні тромбоцитів

D. Ядерні білки (нуклеопротеїди)

E. Сироваткові альбуміни

165. При хворобі Базедова щитоподібна залоза продукує надмірну кількість гормонів Т3, Т4. Що є причиною гіперфункції тироцитів при цьому автоімунному захворюванні?

A. Дефіцит антитіл A

B. Стимуляція антитілами G

C. Стимуляція антитілами E

D. Пошкодження, індуковане імунними комплексами

E. Агресія кілерів

166. Імунна система не толерантна до органів і тканин, які «приховані» від неї за спеціалізованими гематопаренхіматозними бар’єрами. При пошкодженні бар’єру якого органа може виникати автоімунне захворювання?

A. Головний мозок

B. Печінка

C. Нирки

D. Серце

E. Кістковий мозок

167. У хворого з травмою ока виникло автоімунне запалення у другому оці, що призвело до значного зниження зору. Який механізм призвів до автоімунного ушкодження ока?

A. Інфікування ока

B. Втрата толерантності

C. Модифікація автоантигену

D. Демаскування автоантигену

E. Гіперпродукція антитіл Е

168. З метою попередження розвитку реакції «трансплантат проти хазяїна», пацієнту з пересадженим кістковим мозком призначено курс імунодепресивної терапії. Які гормони застосовують з цією метою?

A. Андрогени

B. Мінералокортикоїди

C. Глюкокортикоїди

D. Естрогени

E. Соматотропін

169. Автоімунний інсуліт, що є причиною розвитку цукрового діабету 1 типу, розвивається за механізмом автоімунних реакцій 4-го типу, за Кумбсом і Джеллом. Що є причиною пошкодження β-клітин островців підшлункової залози?

A. В-лімфоцити

B. IgA

C. IgE

D. Імунні комплекси

E. Т-лімфоцити

170. При цукровому діабеті 1 типу в організмі створюється важкий дефіцит інсуліну з причини зменшення кількості В-клітин в островках Лангенганса, що, в 95% випадків, є наслідком автоімунного запалення (інсуліту). Зазначте механізм знищення В-клітин островків підшлункової залози при цьому:

A. Пошкодження, індуковане антитілами А

B. Токсична дія надмірної кількості антитіл E

C. Агресія кілерів

D. Пошкодження, індуковане антитілами G, комплементом

E. Пошкодження, індуковане імунними комплексами

171. Жінка скаржиться на погіршення пам’яті, сухість шкіри, набряклість обличчя. Діагностовано гіпофункцію щитоподібної залози з причини автоімунного тиреоїдиту, що розвивається за механізмом реакцй 4-го типу, за Кумбсом і Джеллом. Що є механізмом знищення тироцитів при автоімунному тироїдиті:

A. Цитотоксична дія антитіл E

B. Агресія кілерів

C. Пошкодження, індуковане антитілами А

D. Пошкодження, індуковане антитілами G, комплементом

E. Пошкодження, індуковане імунними комплексами


Алергія

172. У хворої на системний червоний вовчак виявлені протеїнурія, гематурія, набряки, ураження шкіри та суглобів. Алергічні реакції якого типу (за Кумбсом і Джеллом) є причиною тканинного пошкодження в патогенезі цього захворювання?

A. Опосередковані IgG клітинні дисфункції

B. Опосередковані IgE

C. Опосередковані IgG, цитотоксичні

D. Опосередковані імунними комплексами

E. Опосередковані макрофагами і Т- лімфоцитами

173. Через годину після вживання амідопірину пацієнт скаржиться на біль в горлі, неможливість ковтання слини. При обстеженні: некротична ангіна, у крові є антилейкоцитарні антитіла, нейтропенія. Алергічні реакції якого типу, за класифікацією Кумбса і Джелла, лежать в основі цієї патології?

A. Опосередковані IgG, цитотоксичні

B. Опосередковані IgE

C. Опосередковані IgG клітинні дисфункції

D. Опосередковані імунними комплексами

E. Опосередковані макрофагами і Т- лімфоцитами

174. До лікаря звернулася жінка зі скаргами на те, що після вживання горіхів і меду у неї виникає уртикарний висип на обличчі та шиї. Алергічні реакції якого типу за Кумбсом і Джелом має місце у цьому випадку?

A. Опосередковані імунними комплексами

B. Опосередковані IgG, цитотоксичні

C. Опосередковані IgG клітинні дисфункції

D. Опосередковані IgE

E. Опосередковані макрофагами і Т- лімфоцитами

175. Через місяць після встановлення зубних протезів у пацієнта з’явилися локальні (в місці металевих коронок) почервоніння й ущільнення ясен. Які алергічні реакції визначають розвиток гінгівіту в цьому випадку?

A. Опосередковані імунними комплексами

B. Опосередковані IgE

C. Опосередковані IgG, цитотоксичні

D. Опосередковані IgG клітинні дисфункції

E. Опосередковані макрофагами і Т- лімфоцитами

176. У хворого на правець через 2 тижні після введення значної дози лікувальної (кінської) сироватки виникли уртикарний висип, біль в суглобах та м’язах, гарячка. Алергічна реакція якого типу розвинулася у хворого?

A. Опосередкована IgG, цитотоксична

B. Опосередкована IgE

C. Опосередкована імунними комплексами

D. Опосередкована IgG клітинна дисфункція

E. Опосередкована макрофагами і Т- лімфоцитами

177. Пацієнт відмічає почервоніння, ущільнення шкіри долоней після контакту з пральним порошком. Алергічні реакції якого типу є причиною розвитку контактного дерматиту?

A. Опосередковані імунними комплексами

B. Опосередковані IgE

C. Опосередковані IgG, цитотоксичні

D. Опосередковані IgG клітинні дисфункції

E. Опосередковані макрофагами і Т- лімфоцитами

178. У зубного техніка виражений контактний дерматит. Алергічні реакції якого типу, за Кумбсом і Джеллом, визначають розвиток цього захворювання?

A. Опосередковані макрофагами і Т- лімфоцитами

B. Опосередковані IgE

C. Опосередковані IgG, цитотоксичні

D. Опосередковані IgG клітинні дисфункції

E. Опосередковані імунними комплексами

179. У хворого на сифіліс на другому тижні від початку терапії великими дозами пеніціліну, з'явилися симптоми гломерулонефриту, артриту, уртикарні висипи на шкірі, що було схоже на "сироваткову хворобу". Алергічні реакції якого типу, за Кумбсом і Джеллом, є причиною ускладнення протимікробної терапії у пацієнта?

A. Опосередковані імунними комплексами

B. Опосередковані IgE

C. Опосередковані IgG, цитотоксичні

D. Опосередковані IgG клітинні дисфункції

E. Опосередковані макрофагами і Т- лімфоцитами

180. У хворого з вітамін В12–дефицитною анемією, що резистентна до терапії вітаміном В12, виявлена велика кількість IgG проти рецепторів ентероцитів, що забезпечують рецептор-залежний ендоцитоз комплексу «вітамін В12–внутрішній фактор Касла». Імунні реакції якого типу (за Кумбсом і Джеллом) є головною ланкою в патогенезі перніціозної анемії в пацієнта?

A. Опосередковані імунними комплексами

B. Опосередковані IgE

C. Опосередковані IgG, цитотоксичні

D. Опосередковані IgG клітинні дисфункції

E. Опосередковані макрофагами і Т- лімфоцитами

181. На 8 день після введення протиправцевої сироватки з приводу брудної рани стопи у пацієнта температура тіла підвищилася до 38 оС, з'явилися біль в суглобах, висип, свербіж, протеїнурія. Алергічна реакція якого типу розвинулася в цьому випадку?

A. Опосередкована макрофагами і Т- лімфоцитами

B. Опосередкована IgE

C. Опосередкована IgG, цитотоксична

D. Опосередкована IgG клітинна дисфункція

E. Опосередкована імунними комплексами

182. Після введення антитіл до білків базальної мембрани ниркових клубочків у тварини розвинувся гострий гломерулонефрит. Алергічні реакції якого типу, за класифікацією Кумбса і Джелла, лежить в основі цієї патології?

A. Опосередковані IgG клітинні дисфункції

B. Опосередковані IgE

C. Опосередковані IgG, цитотоксичні

D. Опосередковані імунними комплексами

E. Опосередковані макрофагами і Т- лімфоцитами

183. Через декілька хвилин після ін’єкції ліків у пацієнта з’явилися відчуття нестачі повітря, утруднення видоху, шкіра вкрилася потом, артеріальний тиск крові знизився до 80/40 мм.рт.ст. Алергійні реакції якого типу, за Кумбсои і Джелом, зумовили вказані патологічні симптоми?

A. I типу, системні

B. I типу, локальні

C. II типу

D. III типу

E. IV типу

184. Пацієнт скаржиться на напади серцебиття, пітливість, дратівливість. Обстеження показало в нього дифузний токсичний зоб. Імунні реакції якого типу, за Кумбсом і Джеллом, є причиною гіперфункції щитоподібної залози при Базедовій хворобі?

A. Опосередковані IgE

B. Опосередковані IgG клітинні дисфункції

C. Опосередковані IgG, цитотоксичні

D. Опосередковані імунними комплексами

E. Опосередковані макрофагами і Т- лімфоцитами

185. У дитини визначається позитивна реакція Манту. Імунні реакції якого типу, за Кумбсом і Джеллом, є причиною почервоніння, ущільнення і некротичних змін шкіри у місці введення туберкуліну?

A. II типу

B. I типу, локальні

C. I типу, системні

D. IV типу

E. III типу

186. Жінка звернулася зі скаргами на свербіж та печіння в очах, сльозотечу, виділення з носа. Симптоми з’явилися під час поїздки за місто. Діагностовано поліноз. Алергічні реакції якого типу, за Кумбсом і Джелом, спричиюють розвиток полінозу?

A. І типу, системні

B. I типу, локальні

C. II типу

D. III типу

E. IV типу

187. Ділянка шкіри в місці контакту з металевим браслетом почервоніла, ущільнилась. До сплаву, з якого виготовлений браслет, входять солі нікелю і хрому. Алергічні реакції якого типу, за Кумбсом і Джеллом, є причиною контактного дерматиту в пацієнта?

A. Опосередковувані антитілами G

B. Анафілактичні, локальні

C. Анафілактичні, системні

D. Опосередковувані макрофагами і лімфоцитами

E. Імунокомплексні

188. У хворого з протеїнурією, гематурією, набряками та артеріальною гіпертензією встановлено діагноз «Гострий гломерулонефрит». Алергічні реакції якого типу, за Кумбсом і Джеллом, є причиною гломерулонефриту у 80% випадків?

A. Опосередковувані IgG

B. Анафілактичні

C. Опосередковувані IgЕ

D. Опосередковувані імунними комплексами

E. Опосередковувані лімфоцитами - кілерами

189. У паціента з недостатністю дихання діагностовано атопічну бронхіальну астму. Алергічні реакції якого типу, за Кумбсом і Джеллом, є причиною обструкції нижніх дихальних шляхів і недостатності альвеолярної вентиляції в хворого?

A. Опосередковувані IgG, клітинні дисфункції

B. Опосередковувані IgG, цитотоксичні

C. Опосередковувані IgЕ

D. Опосередковувані імунними комплексами

E. Опосередковувані лімфоцитами

190. Через тиждень після застосування протиправцевої сироватки у пацієнта підвищилася температура тіла, з'явився висип, напухли суглоби, у сечі з'явився білок. Алергічні реакції якого типу (за Кумбсом і Джеллом) є головною ланкою в патогенезі сироваткової хвороби у пацієнта?

A. III типу

B. I типу, локальні

C. І типу, місцеві

D. II типу

E. IV типу

191. У пацієнта з ідіопатичною тромбоцитопенічною пурпурою у крові визначається високий титр антитромбоцитарних IgG. Імунні реакції якого типу, за Кумбсом і Джеллом, визначають розвиток тромбоцитопенії у цього хворого?

A. IV типу

B. I типу, локальні

C. І типу, системні

D. III типу

E. ІI типу

192. Усі члени сім'ї пацієнта Д., 22 років, страждають від алергічних реакцій анафілактичного типу. Яким терміном описують спадкову схильність до розвитку анафілактичних реакцій?

A. Десенсибілізація

B. Сенсибілізація

C. Атопія

D. Гіперчутливість

E. Псевдоанафілаксія

193. У Каті З. вживання в їжу червоної риби завжди супроводжується відчуттям дискомфорту, болями в животі, іноді проносом. Як називається локальна анафілактична реакція в травному тракті?

A. Анафілактичний шок

B. Алергічний гастроентерит

C. Алергічний нежить

D. Кропив'янка

E. Поліноз

194. При будь-якому контакті з сухим кормом для рибок Женя Т. миттєво починає задихатися. Вкажіть клінічний прояв алергічної реакції анафілактичного типу в нижніх дихальних шляхах:

A. Поліноз

B. Кропив'янка

C. Алергічний нежить

D. Атопічна бронхіальна астма

E. Анафілактичний шок

195. У деяких людей є генетичні передумови розвитку певних анафілактичних реакцій на харчові антигени. Як називаються клінічні прояви локальної анафілаксії в шкірі?

A. Кропив'янка

B. Алергічний нежить

C. Поліноз

D. Анафілактична бронхіальна астма

E. Анафілактичний шок

196. Через 10 днів після введення протидифтерійної сироватки у дитини з'явився висип на шкірі, який супроводжувався сильним свербінням, підвищилася температура тіла до 38 оC, з'явився біль у суглобах. Яке ускладнення виникло?

A. Анафілактична реакція

B. Гіперчутливість сповільненого типу

C. Сироваткова хвороба

D. Контактний дерматит

E. Атопія

197. У дитини після вживання червоної малини виникли уртикарні висипи на шкірі, які супроводжувалися сильним свербінням, підвищилася температура тіла до 37,5 оC. Яке захворювання виникло?

A. Контактний дерматит

B. Анафілактичний шок

C. Сироваткова хвороба

D. Кропив’янка

E. Поліноз

198. Під час внутрішньовенної ін’єкції ліків шкіра хворого вкрилася холодним потом, кінцівки захолонули, з’явилися задишка і утруднення дихання. Назвіть системну анафілактичну реакцію, що розвинулася у пацієнта:

A. Алергічний нежить

B. Кропив'янка

C. Анафілактичний шок

D. Поліноз

E. Сіроваткова хвороба

199. Одразу після внутрішньом’язового введення антибіотика у пацієнта з'явилися задуха, відчуття страху, акроціаноз. Назвіть системну анафілактичну реакцію у пацієнта:

A. Сироваткова хвороба

B. Кропив'янка

C. Алергічна нежить

D. Поліноз

E. Анафілактичний шок

200. У місцях розвитку анафілактичних реакцій виражені гіперемія, накопичення ексудату, набряки і свербіння. Який медіатор анафілаксії визначає розвиток вищезазначених симптомів?

A. Фактор хемотаксису еозинофілів

B. Гепарин

C. Гістамін

D. Інтерферон

E. Інтерлейкін 2

201. У жінки з бронхіальною астмою після контакту з собакою виник напад задухи. Які медіатори анафілаксії є причиною тривалого бронхоспазму?

A. Лейкотриєни

B. Інтерлейкіни

C. Катехоламіни

D. Кініни

E. Фактори хемотаксису

202. У патогенезі гострої недостатності дихання при анафілактичному шоку є бронхоспазм, гіперсекреція слизу, набряк слизової оболонки нижніх дихальних шляхів. Які з медіаторів зумовлюють розвиток даних порушень?

A. Інтерлейкіни 1, 2

B. Гепарин, серотонін

C. Кініни

D. Гістамін, лейкотрієни

E. Білки комплементу

203. Під час внутрішньовенної інекції ліків у пацієнта розвинувся анафілактичний шок. Які медіатори анафілаксії спричинюють безпосередньо обструкцію нижніх дихальних шляхів і гостру недостатність дихання при цьому?

A. Хемотаксичні фактори

B. Гістамін, лейкотрієни

C. Цитокіни

D. Інтерлейкіни

E. Катехоламіни

204. Катехоламіни / адреноміметики можуть істотно поліпшити прохідність нижніх дихальних шляхів і тому використовуються для надання невідкладної допомоги пацієнтам з алергічними реакціями:

A. Алергічний нежить

B. Кропив'янка

C. Анафілактичний шок

D. Поліноз

E. Сироваткова хвороба

205. Вже багато років жінка страждає на алергійний риніт. Вкажіть головну ланку в патогенезі цього захворювання:

A. Алерген-індукована активація макрофагів

B. Преципітація імунних комплексів

C. Гіперпродукція імунних комплексів

D. Алерген – індукована дегрануляція тканинних базофілів

E. Гіперпродукція антитіл А

206. У Маші Д., 7 років, алергія на волоський горіх. Вживання в їжу будь-якого продукту, що містить горіхи, призводить до появи у дитини висипу, що свербить. З якими антитілами пов'язаний розвиток алергічних реакцій анафілактичного типу?

A. IgD

B. IgA

C. IgM

D. IgG

E. IgE

207. Для постановки туберкулінової проби дитині внутрішньошкірно введено туберкулін. Через 24 години в місці введення туберкуліну відзначена виражена гіперемія, ущільнення шкіри з некрозом в центрі. Який механізм лежить в основі розвитку даних змін?

A. Опосередкована антитілами цитотоксичність

B. Клітинна цитотоксичність

C. Опосередкована антитілами клітинна дисфункція

D. Алерген-індукована дегрануляція тканинних базофілів

E. Преципітація імунних комплексів, активація комплементу

208. Для постановки туберкулінової проби дитині внутрішньошкірно введено туберкулін. Через 24 години в місці введення відзначена виражена гіперемія, ущільнення тканини. Що спричинює пошкодження і запалення у цьому випадку?

A. В-лімфоцити, нейтрофіли

B. IgA

C. IgE

D. Імунні комплекси

E. Т-лімфоцити, макрофаги

209. У жінки віком 45 років, яка тривалий час хворіє на атопічну бронхіальну астму, виник напад ядухи. Що є причиною нестачі повітря в хворої при цьому?

A. Порушення рухомості грудної клітини

B. Ушкодження дихального центру

C. Спазм, обтурація дрібних бронхів

D. Порушення перфузії легенів

E. Втрата еластичності легеневої тканини

210. Одазу після введення анестетика в пацієнта розвинувся анафілактичний шок. Який механізм визначає розвиток гострої недостатності кровообігу при цьому?

A. Біль

B. Гіперволемія

C. Вазодилатація

D. Активація симпато-адреналової системи

E. Зниження скоротливої функції серця

211. Хворому М. перед введенням лікувальної дози протиправцевої сироватки було зроблено пробу на чутливість до неї. Проба виявилася позитивною. Яким чином найбільш правильно провести специфічну гіпосенсибілізацію за Безредкою?

A. Введенням завершальної дози сироватки

B. Введенням малих доз сироватки

C. Введенням антигістамінних препаратів

D. Введенням катехоламінів

E. Введенням глюкокортикоїдів

212. Дитині 10 років, що під час гри порізала ногу відламком скла, з метою попередження ускладнень протиправцеву сироватку вводили за Безредкою. Який механізм лежить в основі подібного способу гіпосенсибілізації?

A. Зв'язування специфічних IgE

B. Блокування дегрануляції тучних клітин

C. Блокування Н1-рецепторів гістаміну

D. Виникнення імунологічної толерантності

E. Зв'язування рецепторів до IgE на тучних клітинах

213. Під час парентерального введення ліків у пацієнта розвинувся анафілактичний шок. Антитіла якого класу відіграють головну роль у патогенезі анафілактичного шоку у людини?

A. IgA

B. IgG

C. IgD

D. IgM

E. IgE

214. У резус-негативної дитини, народженої від резус-позитивної матері, виникла гемолітична хвороба новонароджених. Антитіла якого класу спричинюють гемоліз еритроцитів і, таким чином, відіграють головну роль у патогенезі захворювання?

A. IgM

B. IgD

C. IgG

D. IgA

E. IgE

215. У тварини при моделюванні феномену Артюса на шкірі з’явився осередок геморагічного некрозу. Активація якої системи призводить до пошкодження судинної стінки при імунокомплексних реакціях?

A. Калікреїн-кінінової

B. Антиоксидантної

C. Комплементу

D. Антикоагулянтної

E. Фібринолізу

216. У пацієнта з некротичною ангіною при обстеженні у крові виявлені нейтропенія й антилейкоцитарні антитіла. Що є одним з механізмів пошкодження нейтрофілів у цьому випадку?

A. Дія NK-клітин

B. Дія Т-кілерів

C. Дія цитотоксичних лімфокінів

D. Комплемент-залежний цитоліз

Е. Дія В-лімфоцитів


ЗМІСТОВИЙ МОДУЛЬ 2 «ТИПОВІ ПАТОЛОГІЧНІ ПРОЦЕСИ»

Пошкодження клітини

217. У експерименті кроля опромінили рентгенівськими променями і отримали гостру променеву хворобу. Вкажіть патогенетичний варіант пошкодження клітин у цьому випадку:

A. Оборотне

B. Гостре

C. Хронічне

D. Цитопатичне

E. Насильне

218. У експерименті кроля тримали в барокамері за умов розрідженого повітря. Вкажіть патогенетичний варіант пошкодження клітин у цьому випадку:

A. Гостре

B. Насильне

C. Цитопатичне

D. Хронічне

E. Оборотне

219. У експерименті кроля тримали на дієті, що не містить вітамінів С і Е. Вкажіть патогенетичний варіант пошкодження клітин у цьому випадку:

A. Насильне

B. Гостре

C. Хронічне

D. Цитопатичне

E. Первинне

220. У експерименті кроля опромінили рентгенівськими променями і отримали гостру променеву хворобу. Який механізм викликав активацію перекисного окислення ліпідів у цьому випадку?

A. Дія детергентів

B. Дефіцит антиоксидантних ферментів

C. Гіповітаміноз Е

D. Активація гліколізу

E. Посилення утворення вільних радикалів

221. У експерименті кроля тримали у барокамері за умов підвищеного тиску кисню. Який механізм викликав активацію перекисного окислення ліпідів у цьому випадку?

A. Гіповітаміноз Е

B. Дефіцит антиоксидантних ферментів

C. Посилення утворення вільних радикалів

D. Активація гліколізу

E. Дія детергентів

222. У експерименті кролю вводили великі дози вітаміну Д. Який механізм викликав активацію перекисного окислення ліпідів у цьому випадку?

A. Дефіцит антиоксидантних ферментів

B. Посилення утворення первинних вільних радикалів

C. Гіповітаміноз Е

D. Порушення пентозного циклу і циклу Кребса

E. Дія детергентів

223. У експерименті кроля тримали на дієті, що не містить вітамінів С і Е. Який механізм викликав активацію перекисного окислення ліпідів у цьому випадку?

A. Недостатність первинних антиоксидантів

B. Посилення утворення первинних вільних радикалів

C. Дефіцит прооксидантів

D. Порушення пентозного циклу і циклу Кребса

E. Дія детергентів

224. При інфаркті міокарду в клітинах серцевого м’яза активуються ліпідні механізми пошкодження клітин. Який з перерахованих механізмів відноситься до ліпідних?

A. Набряк клітини

B. Активація перекисного окислення ліпідів

C. Денатурація білків

D. Активація лізосомних гідролаз

E. Активація протеолізу

225. При інфаркті міокарду в клітинах серцевого м’яза підвищується концентрація іонів кальцію, що значно посилює їх пошкодження. Який з перерахованих механізмів відноситься до кальцієвих?

A. Активація мембранних фосфоліпаз

B. Активація перекисного окислення ліпідів

C. Детергентна дія жирних кислот

D. Втрата мембранного потенціалу

E. Порушення транскрипції

226. При ішемії в клітинах підвищується концентрація іонів водню, що значно посилює їх пошкодження. Який з перерахованих механізмів відноситься до ацидотичних?

A. Порушення трансляції

B. Контрактура скоротливих клітин

C. Втрата клітиною потенціалу спокою

D. Детергентна дія жирних кислот

E. Підвищення проникності мембран

227. При інфаркті міокарду в клітинах серцевого м’яза підвищується концентрація іонів кальцію, що значно посилює їх пошкодження. Який з перерахованих механізмів відноситься до кальцієвих?

A. Набряк клітини

B. Втрата клітиною потенціалу спокою

C. Роз’єднання окиснення та фосфорилювання

D. Осмотичне розтягнення мембран

E. Денатурація білків

228. При гіпоксії у клітинах порушується робота іонних насосів, що значно посилює пошкодження. Який з перерахованих механізмів пошкодження клітин відноситься до електролітно-осмотичних?

A. Порушення реплікації

B. Порушення функції білків

C. Роз’єднання окиснення та фосфорилювання

D. Детергентна дія жирних кислот

E. Втрата клітиною потенціалу спокою

229. При голодуванні в клітинах порушується структура та функція білків, що значно посилює пошкодження клітини. Який з перерахованих механізмів відноситься до протеїнових?

A. Набряк клітини

B. Зменшення активності ферментів

C. Втрата клітиною потенціалу спокою

D. Детергентна дія жирних кислот

E. Активація перекисного окислення ліпідів

230. При дефіциті вітаміну В12 в клітинах порушується синтез азотистих основ, що значно посилює пошкодження клітини. Який з перерахованих механізмів відноситься до нуклеїнових?

A. Зменшення реплікації

B. Набряк клітини

C. Втрата клітиною потенціалу спокою

D. Детергентна дія жирних кислот

E. Активація перекисного окислення ліпідів

231. При ішемії у міокардіоцитах виникає контрактура міофібрил, роз’єднання окиснення та фосфорування та активація фосфоліпази А2. Які механізми пошкодження клітин переважають у цьому випадку?

A. Протеїнові

B. Ліпідні

C. Ацидотичні

D. Електолітно-осмотичні

E. Кальцієві

232. При дії іонізуючої радіації у клітинах значно активується перекисне окиснення ліпідів. Які механізми пошкодження клітин переважають у цьому випадку?

A. Кальцієві

B. Ліпадні

C. Ацидотичні

D. Електолітно-осмотичні

E. Протеїнові

233. У клітині виникли підвищення проникності мембран, активація лізосомних ферментів, порушення активності ферментів та структурних білків. Які механізми пошкодження клітин переважають у цьому випадку?

A. Кальцієві

B. Ліпідні

C. Ацидотичні

D. Електолітно-осмотичні

E. Протеїнові

234. У клітині виникли внутрішньоклітинний набряк, осмотичне розтягнення мембран та втрата потенціалу спокою. Які механізми пошкодження клітин переважають у цьому випадку?

A. Ліпідні

B. Електолітно-осмотичні

C. Кальцієві

D. Ацидотичні

E. Протеїнові


Місцеві розлади кровообігу

235. У кроля після видалення правого верхнього шийного симпатичного вузла спостерігаються почервоніння й підвищення температури шкіри правої половини голови. Який вид артеріальної гіперемії розвинувся в кролика?

A. Вакатна

B. Нейропаралітична

C. Постішемічна

D. Нейротонічна

E. Метаболічна

236. У хворого на хронічну серцеву недостатність часто виникають головний біль, запаморочення та знепритомнення. Який вид порушень місцевого кровообігу виник у головному мозку в цьому випадку?

A. Стаз

B. Артеріальна гіперемія

C. Венозна гіперемія

D. Тромбоз

E. Ішемія

237. Експериментатор при подразненні вагуса спостерігав збільшення коронарного кровотоку. Який вид артеріальної гіперемії виник при цьому?

A. Нейротонічна

B. Робоча

C. Реактивна

D. Гуморальна

E. Нейропаралітична

238. Експериментатор при подразненні horda tympani спостерігав гіперемію піднижньощелепної слинної залози. Який вид артеріальної гіперемії виник при цьому?

A. Робоча

B. Нейротонічна

C. Реактивна

D. Гуморальна

E. Нейропаралітична

239. У хворого на хронічну серцеву недостатність ноги набряклі, ціанотичні, холодні на дотик. Який вид порушень місцевого кровообігу є головним у виникненні цих змін?

A. Стаз

B. Артеріальна гіперемія

C. Ішемія

D. Емболія

E. Венозна гіперемія

240. Відновлення коронарного кровообігу у хворого з тромбозом вінцевої артерії супроводжувалося зниженням скорочувальної здатності серця і погіршенням загального стану пацієнта. Яке явище стало причиною ускладнення?

A. Венозна гіперемія

B. Ішемія

C. Артеріальна гіперемія

D. Реперфузійний синдром

E. Резорбційно-некротичний синдром

241. У хворого з облітеруючим ендартеріїтом після гангліонарної симпатектомії спостерігається позитивний лікувальний ефект, пов’язаний з розвитком артеріальної гіперемії нижніх кінцівок. Який вид гіперемії виник у пацієнта?

A. Гуморальна

B. Робоча

C. Реактивна

D. Нейропаралітична

E. Нейротонічна

242. У чоловіка, 57 років, який скаржиться на біль у ділянці серця після негативних емоцій, встановлено ішемічну хворобу серця. Який механізм ішемії найбільш імовірний?

A. Ангіоспастичний.

B. Механічний.

C. Обтураційний.

D. Странгуляційний

E. Компресійний.

243. Після нанесення ацетилхоліну на вухо кролика місцево виникло почервоніння цієї ділянки шкіри. Який вид артеріальний гіперемії виник у кролика?

A. Метаболічна

B. Робоча

C. Реактивна

D. Нейропаралітична

E. Нейротонічна

244. Тривале перебування на холоді викликало збліднення шкіри вуха у дитини 5 років. Яка речовина зумовила розвиток ішемії?

A. Вазопресин

B. Ангіотензин 1

C. Норадреналін

D. Na+

E. Ca2+

245. Відновлення коронарного кровообігу у хворого на інфаркт міокарда супроводжувалося зниженням скорочувальної здатності серця і погіршенням загального стану пацієнта. Які реакції визначають збільшення пошкодження кардіоміоцитів в умовах реперфузії?

A. Трансамінування

B. Глікозилювання

C. Декарбоксилювання

D. Вільнорадикальні

E. Дефосфорилювання

246. У хворої внаслідок емболії малого кола кровообігу виникли гіпертензія в малому колі, гіпотензія у великому колі кровообігу та збільшення центрального венозного тиску. Що є основною причиною порушення кровообігу в цьому випадку?

A. Лівошлуночкова недостатність

B. Механічна закупорка аорти

C. Гіповолемія

D. Правошлуночкова недостатність

E. Фібриляція шлуночків

247. Внутрішньовенне введення тромбопластину тварині призвело до тромбозу. Який механізм лежить в основі його розвитку?

A. Уповільнення течії крові

B. Ушкодження судинної стінки

C. Активація системи зсідання крові

D. Активація тромбоцитарного гемостазу

E. Зменшення активності антикоагулянтів

248. Відомо, що у венах тромби утворюються в п'ять разів частіше, ніж у артеріях, а у венах нижніх кінцівок - у три рази частіше, ніж у венах верхніх кінцівок. Вкажіть, з яким патогенетичним фактором, що зумовлює процес тромбоутворення, пов'язане це явище?

A. Підвищенням активності системи згортання крові

B. Ушкодженням судинної стінки

C. Уповільненням кровообігу

D. Підвищенням в'язкості крові

E. Зниженням активності системи згортання крові

249. У хворої внаслідок емболії малого кола кровообігу виникли гіпертензія в малому колі, гіпотензія у великому колі кровообігу та збільшення центрального венозного тиску. Що є основною причиною правошлуночкової недостатності в цьому випадку?

A. Інфаркт міокарда

B. Портальна гіпертензія

C. Гіповолемія

D. Спазм артеріол легень

E. Фібриляція шлуночків

250. Низка патологічних процесів (атеросклероз судин, запалення, механічна травма, некроз тканин) супроводжується тромбоутворенням. Який патогенний фактор у механізмі тромбоутворення відіграє роль ініціатора при цих процесах?

A. Уповільнення кровообігу

B. Ушкодження ендотелію судин

C. Зміна фізико-хімічних властивостей крові

D. Підвищення активності системи згортання крові

E. Зниження активності системи антикоагулянтів

251. У хворого з пораненням яремної вени встановлено зниження артеріального, збільшення центрального венозного тиску, ціаноз. Яке порушення периферичного кровообігу виникло у хворого?

A. Емболія малого кола кровообігу

B. Емболія великого кола кровообігу

C. Емболія портальної вени

D. Тромбоемболія

E. Ішемія

252. У хворого 73-х років з закритим переломом правої стегнової кістки діагностовано емболію судин головного мозку. Назвіть вид емболії, що найбільш часто виникає при переломах трубчастих кісток:

A. Газова

B. Тканинна

C. Жирова

D. Ретроградна

E. Повітряна

253. У хворої внаслідок емболії малого кола кровообігу виникли гіпертензія в малому колі, гіпотензія у великому колі кровообігу та збільшення центрального венозного тиску. Що є основною причиною смерті в цьому випадку?

A. Гіповолемія

B. Портальна гіпертензія

C. Правошлуночкова недостатність

D. Лівошлуночкова недостатність

E. Інфаркт міокарда

254. Ембол, що утворився у великому колі кровообігу знову потрапив у судини великого кола, минаючи мале коло. Як називається такий вид емболії?

A. Ретроградна

B. Парадоксальна

C. Ортоградга

D. Тканинна

E. Жирова

255. Нанесення гіпертонічного розчину NaCl на брижу жаби призвело до швидкого уповільнення і зупинки течії крові в судинах. Яке порушення кровообігу спостерігалося при цьому?

A. Стаз венозний

B. Венозна гіперемія

C. Стаз ішемічний

D. Стаз істинний

E. Тромбоз

256. У хворої на бешиху виникла облітерація лімфатичних судин, що призвело до «слоновості». Який різновид порушень мікроциркуляції виник у хворої?

A. Резорбційна лімфатична недостатність

B. Динамічна лімфатична недостатність

C. Механічна лімфатична недостатність

D. Ангіоспастична ішемія

E. Венозна гіперемія

257. У експерименті після внутрішньовенного введення декстрину в мікросудинах утворилась велика кількість агрегатів еритроцитів, що суттєво порушили мікроциркуляцію. Який головний механізм виникнення цих змін?

A. Порушення суспензійної стабільності крові

B. Сповільнення кровообігу

C. Пошкодження судинної стінки

D. Переважання прокоагулянтів на антикоагулянтами

E. Сепарація плазми крові

258. У хворої на рак молочної залози після видалення реґіонарних лімфовузлів виник набряк руки. Який різновид порушень мікроциркуляції виник у хворої?

A. Резорбційна лімфатична недостатність

B. Динамічна лімфатична недостатність

C. Механічна лімфатична недостатність

D. Ангіоспастична ішемія

E. Венозна гіперемія

259. У хворого на хронічну серцеву недостатність виникло порушення лімфотоку в нижніх кінцівках. Який різновид порушень мікроциркуляції виник у хворого?

A. Динамічна лімфатична недостатність

B. Механічна лімфатична недостатність

C. Резорбційна лімфатична недостатність

D. Ангіоспастична ішемія

E. Венозна гіперемія

260. У хворого на цироз печінки різко знизився артеріальний тиск. Який патологічний механізм є ініціатором порушення гемодинаміки при цьому?

A. Збільшення депонування крові в портальному руслі

B. Спазм коронарних артерій

C. Зменшення ударного об'єму серця

D. Збільшення діастолічного об'єму серця

E. Розширення вен передньої стінки живота

261. У експерименті після внутрішньовенного введення метилцелюлози в мікро­судинах утворилась велика кількість агрегатів еритроцитів, що суттєво порушили мікроциркуляцію. Який головний механізм виникнення цих змін?

A. Сповільнення кровообігу

B. Порушення суспензійної стабільності крові

C. Пошкодження судинної стінки

D. Переважання прокоагулянтів на антикоагулянтами

E. Сепарація плазми крові

262. У хворого з алкогольним цирозом печінки раптово виникли загальна слабкість, задишка, набряки, асцит, розширення шкірних вен передньої стінки живота. Яке порушення периферичного кровообігу має місце у цього хворого?

A. Емболія легеневої артерії

B. Тромбоз яремної вени

C. Тромбоз селезінкової артерії

D. Емболія брижової артерії

E. Емболія ворітної вени

263. У експерименті після внутрішньовенного введення метилцелюлози в мікро­судинах утворилась велика кількість агрегатів еритроцитів.. Яке порушення мікроциркуляції виникло?

A. Ішемія

B. Сладж

C. Тромбоз

D. Емболія

E. Стаз

264. У хворої на варикозне розширення вен після невдалої операції виникла облітерація лімфатичних судин, що призвело до «слоновості». Який різновид порушень мікроциркуляції виник у хворої?

A. Резорбційна лімфатична недостатність

B. Динамічна лімфатична недостатність

C. Механічна лімфатична недостатність

D. Ангіоспастична ішемія

E. Венозна гіперемія

Запалення

265. У місці опіку з’явилась ділянка некрозу. Який фактор викликає первинну альтерацію?

A. Набряк

B. Гіперосмія

C. Флогогенний агент

D. Медіатори запалення

E. Гіпоксія

266. Після опіку в зоні гіперемії та набряку шкіри у хворого з’явилася ділянка некрозу. Який механізм спричинює посилення руйнівних явищ в осередку запалення?

A. Хемотаксис нейторфілів

B. Діапедез еритроцитів

C. Первинна альтерація

D. Вторинна альтерація

E. Хемотаксис макрофагів

267. У дитини з гнійним апендицитом виявлене значне ушкодження тканини апендикса. Які фактори зумовлюють розвиток вторинної альтерації в осередку запалення?

A. Серотонін

B. Ферменти мікроорганізмів

C. Гістамін

D. Простациклін

E. Лізосомні ферменти

268. У центрі опіку з’явилась ділянка некрозу. Які медіатори запалення можуть брати участь у вторинній альтерації?

A. Простагландини

B. Серотонін

C. Вільні радикали

D. Фібринолізин

E. Фактор Хагемана

269. Хворий скаржиться на біль у місці опіку. Які біологічно активні речовини є медіаторами болю при запаленні?

A. Інтегрини

B. Селектини

C. Гепарин, антитромбін

D. Гістамін, кініни

E. Кейлони

270. В осередку запалення шкіра набрякла, червона, гаряча. Що спричинює місцеве почервоніння тканини в місці запалення?

A. Спазм артеріол

B. Розширення артеріол

C. Тромбоутворення

D. Порушення венозного відтоку

E. Ексудація

271. На брижі жаби під мікроскопом спостерігали розширення судин, прискорення кровотоку, осьовий рух крові в судинах у місці запалення. Яка артеріальна гіперемія розвинулась при цьому?

A. Нейротонічна

B. Фізіологічна

C. Метаболічна

D. Реактивна

E. Робоча

272. На брижі жаби в осередку запалення спостерігали артеріальну гіперемію. Який фактор є головним у патогенезі цього порушення кровообігу?

A. Зменшення вмісту калію в осередку запалення

B. Накопичення кальцію в осередку запалення

C. Дія біологічно активних речовин

D. Підвищення тонусу альфа-адренергічних нервів

E. Параліч вазодилятаторів

273. У місці опіку спостерігається почервоніння. Які медіатори запалення викликають артеріальну гіперемію?

A. Гістамін

B. Тромбоксани

C. Лізосомні ферменти

D. Лімфокіни

E. Фібринолізин

274. При позитивній реакції Манту шкіра в місці введення туберкуліну ущільнена, червоного кольору. Яка речовина може бути причиною почервоніння тканини?

A. CO

B. NO

C. Ендотеліни

D. Селектини

E. Інтегрини

275. Дерматит супроводжувався почервонінням. Які речовини можуть цьому сприяти?

A. Інтерлейкіни

B. Тромбоксани

C. Лізосомні ферменти

D. Простагландини

E. Фактори росту

276. При позитивній реакції Манту шкіра в місці введення туберкуліну червоного кольору. Вплив якого іону на артеріоли може бути причиною почервоніння тканини?

A. Fe2+

B. Na+

C. Cа2+

D. К+

E. Cl

277. В осередку гострого запалення активно взаємодіють різні клітини. Більшість з них є універсальним джерелом неспецифічних медіаторів гострого запалення, які утворюються з фосфоліпідів клітинних мембран. Які це медіатори?

A. Серотонін

B. Гістамін

C. Ейкозаноїди

D. Інтерлейкіни

E. Гепарин

278. Основними медіаторами гострого запалення є похідні арахідонової кислоти. Назвіть ці медіатори?

A. Інтерлейкін 1, інтерлейкін 2

B. Гістамін, серотонін

C. Інтерферон α, інтерферон γ

D. Простагландини, лейкотрієни

E. Калідин, брадикінін

279. У осередку запалення шкіри спостерігається набряк. Які медіатори запалення підвищують проникність судин?

A. Гістамін

B. Тромбоксани

C. Ендотеліни

D. Інтерлейкіни

E. Лімфокіни

280. У осередку гострого запалення спостерігається набряк. Які медіатори запалення підвищують проникність судин?

A. Інтерферони

B. Фактори росту

C. Альбуміни

D. Простагландини

E. Імуноглобуліни

281. Через дві години після травми в місці пошкодження утворився набряк. Що зумовлює ексудацію при запаленні?

A. Алкалоз

B. Медіатори

C. Гарячка

D. Збільшення ШОЕ

E. Лейкоцитоз

282. У хворого отримали 100 мл гнійного ексудату з плевральної порожнини. Який механізм зумовлює ексудацію в даному випадку?

A. Гіпоксія

B. Спазм судин

C. Ішемія

D. Підвищення судинної проникності

E. Алкалоз

283. У місці гострого запалення тканина набрякла. Що сприяє ексудації в осередку запалення?

A. Місцеве підвищення температури

B. Хемотаксичні речовини

C. Білки гострої фази запалення

D. Підвищення гідростатичного тиску крові

E. Проліферація клітин

284. При ангіні мигдалики у хворого червоні набряклі. Який механізм зумовлює ексудацію в осередку запалення?

A. Експресія селектинів

B. Спазм артеріол

C. Розширення венул

D. Хемотаксис лімфоцитів

E. Підвищення судинної проникності

285. У місці опіку з’явився набряк. Який механізм сприяє ексудації в осередку запалення?

A. Підвищення гідростатичного тиску в тканині

B. Зниження гідростатичного тиску в тканині

C. Підвищення онкотичного тиску в тканині

D. Підвищення онкотичного тиску крові

E. Підвищення осмотичного тиску крові

286. У пацієнта карієс ускладнився пульпітом, що супроводжується нестерпним болем. Що є основною причиною виникнення болю у даного хворого?

A. Стаз

B. Первинна альтерація

C. Ішемія

D. Еміграція лейкоцитів

E. Ексудація

287. Хворий з фурункулом на руці скаржиться на сильний біль, набряк та почервоніння у місці запалення, неможливість нормальних рухів рукою, гарячку, сонливість. Який з цих симптомів відноситься до місцевих проявів запалення?

A. Гарячка

B. Сонливість

C. Набряк

D. Лейкоцитоз

E. Підвищення ШОЕ

288. У хворого на гострий апендицит виявили у крові лейкоцитоз, підвищення ШОЕ, появу білків гострої фази запалення. Яка речовина зумовила ці загальні прояви запалення?

A. Лізосомні ферменти

B. Катіонні білки

C. Комплемент

D. Інтерлейкін 1

E. Калідин

289. У хворого на остеомієліт спостерігаються гарячка, лейкоцитоз, підвищення ШОЕ, міалгії, артралгії, сонливість. Які клітини вивільнюють медіатор запалення, що зумовлює появу цих загальних реакцій організму?

A. Лімфоцити

B. Тканинні базофіли

C. Еозинофіли

D. Макрофаги

E. Еритроцити

290. Хворий з фурункулом на руці скаржиться на сильний біль, набряк та почервоніння у місці запалення, неможливість нормальних рухів рукою, гарячку, сонливість. Який з цих симптомів відноситься до загальних проявів запалення?

A. Почервоніння (rubor)

B. Гарячка (febris)

C. Біль (dolor)

D. Набряк (tumor)

E. Місцеве підвищення температури (calor)

291. У зоні запалення лівої гомілки на другу добу відзначається: припухлість, почервоніння, біль, жар. Який провідний механізм зумовив місцеве підвищення температури?

A. Артеріальна гіперемія

B. Ексудація

C. Активація лізосомальних ферментів

D. Активація ендотеліоцитів

E. Підвищення концентрації брадикініну

292. При запаленні артеріальна гіперемія переходить у венозну. Які фактори цьому сприяють?

A. Активація фібринолізу

B. Набухання ендотелію.

C. Активація згортання крові.

D. Зниження осмотичного тиску в судинах

E. Збільшення осмотичного тиску в судинах

293. В експерименті одержано алергічне запалення шкіри у тварини. Спостерігається гіперемія, набряк та еміграція моноцитів в осередок запалення. З дією якого медіатора пов’язаний хемотаксис моноцитів?

A. Еластаза

B. Гістамін

C. Серотонін

D. Тромбоксан

E. Лімфокіни

294. Під час другої стадії фагоцитозу об’єкт фіксується до поверхні фагоциту. Які речовини стимулюють цей процес і є опсонінами при запаленні?

A. Інтерлейкіни

B. Інтегрини

C. Інтерферони

D. Імуноглобуліни

E. Кініни

295. Після поглинання об’єкту фагоцитозу відбувається його знищення спеціальними бактерицидними механізмами. Яка ферментативна система нейтрофілів є основою їх бактерицидності?

A. Калікреїн-кінінова система

B. Мієлопероксидазна система

C. Система комплементу

D. Глютатіонпероксидазна система

E. Ренін-ангіотензинова система

296. Після поглинання мікроорганізмів фагоцитами відбувається їх знищення бактерицидними речовинами. Назвіть такі речовини?

A. Інтегрини

B. Моноаміни

C. Кініни

D. Інтерлейкіни

E. Активні кисневі радикали

297. У стадію прилипання фагоцитозу об’єкт фіксується до поверхні фагоцита. Який механізм цього явища?

A. Респіраторний вибух

B. Дегрануляція базофілів

C. Утворення простагландинів

D. Опсонізація

E. Активація фібринолізу

298. Внаслідок гіперонкії в місці запалення з'явився набряк. Який механізм є провідним у виникненні підвищеного онкотичного тиску?

A. Активація ендотеліоцитів

B. Підвищення концентрації недоокислених продуктів

C. Посилення синтезу білків

D. Скупчення іонів у тканинній рідині

E. Скупчення макромолекул у тканинній рідині

299. Внаслідок гіперонкії в місці запалення з'явився набряк. Який механізм є провідним у виникненні підвищеного онкотичного тиску в тканинній рідині?

A. Підвищення проникності судинної стінки

B. Підвищення концентрації недоокислених продуктів

C. Посилення синтезу білків

D. Скупчення іонів у тканинній рідині

E. Активація ендотеліоцитів

300. Під час запалення спостерігається вихід лейкоцитів з судин у тканину. Які гуморальні медіатори стимулюють хемотаксис лейкоцитів у осередок запалення?

A. Простагландини

B. Компоненти комплементу С3а, С5а

C. Лізосомальні ферменти

D. Гістамін

E. Кініни

301. У хворого з позитивною реакцією Манту шкіра інфільтрована макрофагами і лімфоцитами. Які медіатори спричинюють це явище?

A. Антитіла

B. Кініни

C. Білки комплементу

D. Лімфокіни

E. Лізосомні ферменти

302. Під час запалення спостерігається вихід лейкоцитів з судин у тканину. Які медіатори стимулюють адгезію лейкоцитів до стінок судин (крайове стояння)?

A. Кініни

B. Простагландини

C. Лізосомальні ферменти

D. Гістамін

E. Селектини

303. При вивченні під мікроскопом розвитку запалення на язиці жаби спостерігали крайове стояння лейкоцитів та їх еміграцію крізь судинну стінку. Який фактор є провідним у цьому процесі?

A. Збільшення гідростатичного тиску крові

B. Гіперонкія в осередку запалення

C. Зниження онкотичного тиску крові

D. Поява адгезивних білків на ендотелії в осередку запалення

E. Зменшення гідростатичного тиску крові

304. У порожнину абсцесу, індукованого скипидаром у піддослідної тварини, ввели смертельну дозу правцевого токсину, а тварина не загинула. Яка ймовірна причина такого результату досліду?

A. Посилення васкуляризації місця запалення.

B. Активація дезінтоксикаційної функції фагоцитів.

C. Стимуляція лейкопоезу при запаленні.

D. Активація синтезу антитіл при запаленні.

E. Формування бар’єру навколо вогнища запалення.

305. У зоні запалення виникла інтенсивна деструкція клітин. Яке місцеве фізико-хімічне явище буде пов'язане з цим?

A. Гіперонкія

B. Гіперкалійіонія

C. Гіпоосмія

D. Ацидоз

E. Гіпернатрійіонія

306. У зоні запалення виникла інтенсивна протезна деструкція клітин. Яке місцеве фізико-хімічне явище буде пов'язане з цим?

A. Гіперонкія

B. Гіпоосмія

C. Ацидоз

D. Гіпернатрійіонія

E. Алкалоз

307. У експерименті на кролях було виявлено підвищення кількості макромолекул у міжклітинному середовищі при запаленні. Яка фізико-хімічна зміна виникла?

A. Місцеве підвищення температури.

B. Ацидоз.

C. Гіперосмія.

D. Гіперонкія.

E. Гіпоксія.

308. У експерименті на кролях було виявлено підвищення активності гліколізу в осередку запалення. Яка фізико-хімічна зміна виникла?

A. Гіпоксія.

B. Гіперонкія.

C. Гіперосмія.

D. Місцеве підвищення температури.

E. Ацидоз.

309. У експерименті було виявлено підвищення кількості мікромолекул у позаклітинному середовищі при запаленні. Яка фізико-хімічна зміна виникла?

A. Гіперонкія.

B. Ацидоз.

C. Гіперосмія.

D. Місцеве підвищення температури.

E. Гіпоксія.

310. У експерименті на брижі жаби було виявлено ексудацію в осередок запалення. Який механізм першої фази підвищення проникності судинної стінки?

A. Гіперкальціємія.

B. Ацидоз.

C. Гіперосмія.

D. Гіперонкія.

E. Заокруглення ендотеліоцитів.

311. Після незначної травми структура пошкодженої тканини повністю відновилася. Як називається такий різновид проліферації?

A. Репарація

B. Регенерація

C. Альтерація

D. Ексудація

E. Еміграція

312. Після значної травми сформувався рубець і структура пошкодженої тканини повністю не відновилася. Як називається такий різновид проліферації?

A. Репарація

B. Регенерація

C. Альтерація

D. Ексудація

E. Еміграція

313. У хворого з рваною раною кількість гною значно зменшилась, на дні рани з’явились грануляції. Які біологічно-активні речовини стимулюють проліферацію в осередку запалення?

A. Гістамін

B. Серотонін

C. С-реактивний протеїн

D. Фактори росту

E. Простациклін

314. У місці операційного розрізу сформувався рубець. Проліферація яких клітин у осередку запалення супроводжується продукцією білків сполучної тканини?

A. Нейтрофіли

B. Лімфоцити

C. Фібробласти

D. Тканинні базофіли

E. Еозинофіли

315. При дослідженні запальної тканини виявлено підвищення інтенсивності анаболічних процесів. Для якої стадії запалення найбільш характерна ця зміна?

A. Ексудації

B. Альтерації

C. Проліферації

D. Судинної реакції

E. Еміграції

316. У пацієнта утворився келоїдний рубець. Яка стадія запалення була порушена ?

A. Ексудація

B. Альтерації первинна

C. Альтерація вторинна

D. Еміграція

E. Проліферація

317. У хворого на цукровий діабет трофічна виразка довго не загоюється. Яка стадія запалення порушена в даному випадку?

A. Вторинна альтерація.

B. Еміграція.

C. Проліферація.

D. Ексудація.

E. Первинна альтерація.

318. У дитини, 6 років, погіршення перебігу запалення верхніх дихальних шляхів зумовлене гіперпродукцією прозапальних гормонів. Який з гормонів має прозапальну дію?

A. Кортизол.

B. Соматотропін.

C. Адреналін.

D. Альдостерон.

E. Тестостерон.

319. У клінічній практиці глюкокортикоїди застосовуються як протизапальні засоби. Який механізм їх протизапальної дії зменшує вторинну альтерацію?

A. Пригнічення синтезу гістаміну.

B. Пригнічення синтезу простагландинів.

C. Стабілізація мембран.

D. Пригнічення синтезу серотоніну.

E. Пригнічення синтезу лейкотриєнів.

320. У клінічній практиці широке застосування як протизапальні засоби отримали глюкокортикоїди. Який механізм їх протизапальної дії зменшує проліферацію?

A. Пригнічення синтезу білка.

B. Стабілізація мембран лізосом.

C. Пригнічення синтезу ейкозаноїдів.

D. Пригнічення синтезу гістаміну.

E. Пригнічення синтезу серотоніну.

321. У клінічній практиці широке застосування як протизапальні засоби отримали глюкокортикоїди. Який механізм їх протизапальної дії зменшує ексудацію?

A. Пригнічення синтезу білка.

B. Активація лізосомних ферментів

C. Стимуляція дегрануляції базофілів.

D. Активація синтезу простагландинів.

E. Зменшення утворення ейкозаноїдів.

322. Пацієнт з бронхітом відкашлює багато мокроти. У нього в анамнезі – ендокринопатія. Який гормон має прозапальну дію і посилює ексудацію?

A. Тироксин

B. Альдостерон

C. Інсулін

D. Тестостерон

E. Кортизол

323. Після місцевого застосування глюкокортикоїдів у хворого на екзему значно зменшились набряк, почервоніння, біль. Яким чином глюкокортикоїди зменшують ексудацію?

A. Стимулюють продукцію простагландинів

B. Збільшують проникність судин

C. Пригнічують продукцію ейкозаноїдів

D. Стимулюють глюконеогенез

E. Пригнічують поділ клітин

324. В досліді Конгейма спостерігали крайове стояння лейкоцитів. Що зумовлює початкову стадію еміграції лейкоцитів?

A. Альбуміни

B. Білки гострої фази

C. Адгезивні білки

D. Гепарин

E. Кініни


Гарячка

325. Відомо, що первинні пірогени ініціюють утворення вторинних. Яким чином вторинні пірогени спричинюють підвищення температури?

A. Стимулюють виділення тиреоїдних гормонів

B. Стимулюють скоротливий термогенез

C. Роз’єднують окислення і фосфорування

D. Впливають на роботу центру терморегуляції

E. Викликають підвищення синтезу глюкокортикоїдів

326. Жінка 46 років скаржиться на загальну слабкість, озноб, біль у горлі. Об'єктивно: почервоніння в зоні мигдаликів. Температура тіла 38,60С. Які з перерахованих клітин є головним джерелом ендогенних пірогенів, що викликають лихоманку у хворої?

A. Базофіли

B. Макрофаги

C. Еозинофіли

D. В-лімфоцити

E. Тканинні базофіли

327. У дитини із запаленням легенів спостерігається підвищення температури тіла. Яка з перерахованих біологічно активних речовин відіграє провідну роль у виникненні цього явища?

A. Гістамін

B. Брадикінін

C. Серотонін

D. Інтерлейкін - 1

E. Лімфокін

328. Хворому на грип лікар у комплексній терапії не призначив жарознижувальних препаратів, виходячи із захисної дії підвищеної температури. Який з механізмів такої дії підвищеної температури включається при лихоманці?

A. Пригнічення еритропоезу

B. Стимуляція розмноження збудника

C. Активація продукції інтерферону

D. Пригнічення продукції лізоциму

E. Пригнічення антитілоутворення

329. У хворої на гостру пневмонію з’явився озноб з підвищенням температури тіла до 39 °C, загальна слабкість, сухий кашель. Який з перерахованих медіаторів запалення має властивості ендогенного пірогену?

A. Серотонін.

B. Тромбоксан А2.

C. Брадикінін.

D. Гістамін.

E. Інтерлейкін 1.

330. При гарячці первинні пірогени зумовлюють синтез вторинних. Вкажіть місце утворення останніх.

A. Клітини щитоподібної залози

B. Макрофаги

C. Лімфоцити та еозинофіли

D. Нейрони гіпоталамічної ділянки

E. Клітини юкстагломерулярного апарату нирок

331. З метою отримання гарячки тварині ввели ендотоксин. Вкажіть, як зміниться чутливість нейронів центру терморегуляції до холодових та теплових сигналів?

A. До теплових – підвищиться, до холодових – знизиться

B. Знизиться

C. Підвищиться

D. До теплових – знизиться, до холодових – підвищиться

E. Не зміниться

332. Експериментальній тварині з метою відтворення лихоманки було введено пірогенал. Який механізм при цьому запускає процес підвищення температури тіла?

A. Підвищення установчої точки нейронів терморегуляторного центру

B. Активація скорочувального термогенезу

C. Активація нескорочувального термогенезу

D. Зменшення тепловіддачі

E. Роз'єднання тканинного дихання й окислювального фосфорилювання

333. Розвиток лихоманки у хворого супроводжувався зсувом установчої точки терморегуляторного центру на більш високий рівень із послідовним чергуванням таких стадій:

A. Decrementi, fastigii, incrementi

B. Fastigii, incrementi, decrementi

C. Incrementi, fastigii, decrementi

D. Fastigii, decrementi, incrementi

E. Incrementi,decrementi, fastigii

334. У хворого спостерігається збліднення шкірних покривів, «гусяча шкіра», тахіпное, тахікардія. Якій стадії лихоманки відповідає даний стан?

A. Intermittens

B. Incrementi

C. Fastigii

D. Hectica

E. Decrementi

335. Після введення пірогеналу у хворого 36 років підвищилася температура тіла, шкірні покриви стали блідими, холодними на дотик, з'явився озноб, збільшилося споживання кисню. Який механізм регуляції температури в описаному періоді лихоманки є первинним?

A. Збільшується тепловіддача

B. Збільшується теплопродукція

C. Тепловіддача дорівнює теплопродукції

D. Знижується теплопродукція

E. Знижується тепловіддача

336. В патогенезі гарячки розрізняють три стадії. Вкажіть, як змінюється терморегуляція в першій стадії гарячки?

A. Підвищена теплопродукція відповідає збільшеній тепловіддачі

B. Теплопродукція зростає, тепловіддача не змінюється

C. Теплопродукція не змінюється, тепловіддача знижується

D. Теплопродукція зростає, тепловіддача знижується

E. Тепловіддача перевищує теплопродукцію

337. У хворого на грип відзначається блідість шкірних покривів, «гусяча шкіра», озноб. Яке співвідношення процесів утворення й віддачі тепла в описаному періоді лихоманки?

A. Теплопродукція дорівнює тепловіддачі

B. Посилення теплопродукції без зміни тепловіддачі

C. Теплопродукція перевищує тепловіддачу

D. Зниження тепловіддачі при незміненій теплопродукції

E. Теплопродукція нижча за тепловіддачу

338. У хворого на ГРВІ блідість шкіри змінилася гіперемією, виникло відчуття жару, шкіра на дотик стала гарячою. Якій стадії лихоманки відповідає цей стан?

A. Fastigii

B. Intermittens

C. Incrementi

D. Hectica

E. Decrementi

339. У хворого на крупозну пневмонію різко підвищилася температура до 39,5 оС та утримувалася такою протягом 7 діб. Вкажіть, як змінюється терморегуляція у другій стадії гарячки?

A. Теплопродукція не змінюється, тепловіддача знижується

B. Теплопродукція підвищується, тепловіддача не змінюється

C. Підвищена теплопродукція відповідає збільшеній тепловіддачі

D. Теплопродукція зростає, тепловіддача знижується

E. Тепловіддача перевищує теплопродукцію

340. У хворого на ГРВІ блідість шкіри змінилася гіперемією, виникло відчуття жару, шкіра на дотик стала гарячою. Яке співвідношення процесів утворення й віддачі тепла в описаному періоді лихоманки?

A. Теплопродукція перевищує тепловіддачу

B. Посилення теплопродукції без зміни тепловіддачі

C. Теплопродукція дорівнює тепловіддачі на більш високому рівні

D. Зниження тепловіддачі на фоні незміненої теплопродукції

E. Теплопродукція нижча за тепловіддачу

341. При обстеженні хворого виявлені такі клінічні ознаки: шкірні покриви рожеві, теплі на дотик, сухі, ЧСС – 92 у хвилину, ЧД – 22 за хвилину, температура тіла – 39,2 оС. Яке співвідношення процесів утворення й віддачі тепла в описаному періоді лихоманки?

A. Теплопродукція перевищує тепловіддачу

B. Теплопродукція нижча за тепловіддачу

C. Зниження тепловіддачі при незміненій теплопродукції

D. Посилення теплопродукції без зміни тепловіддачі

E. Теплопродукція дорівнює тепловіддачі

342. У патогенезі гарячки розрізняють три стадії. Вкажіть, як змінюється терморегуляція у третій стадії гарячки?

A. Теплопродукція зростає, тепловіддача не змінюється

B. Тепловіддача перевищує теплопродукцію

C. Теплопродукція не змінюється, тепловіддача знижується

D. Теплопродукція зростає, тепловіддача знижується

E. Підвищена теплопродукція відповідає збільшеній тепловіддачі

343. При обстеженні хворого виявлено такі об'єктивні дані: шкірні покриви гіперемовані, вологі на дотик, відзначається поліурія, полідипсія, температура тіла - 37,20С. Якій стадії лихоманки відповідає цей стан?

A. Decrementi

B. Intermittens

C. Fastigii

D. Incrementi

E. Hectica

344. У хворого з лихоманкою внаслідок лікування температура тіла почала швидко знижуватися. Яка причина зниження температури при цьому?

A. Захисна активація імунної системи

B. Зменшення теплопродукції внаслідок зниження метаболізму

C. Зниження утворення пірогенів

D. Зниження резистентності організму до дії пірогенів

E. Підвищення резистентності організму до дії пірогенів

345. Після різкого зниження температури хворий знепритомнів. Який механізм призвів до зниження артеріального тиску і виникнення колапсу?

A. Звуження судин

B. Збільшення тепловіддачі

C. Зниження теплопродукції

D. Зниження хвилинного об’єму серця

E. Розширення судин

346. У хворого на ГРВІ з’явилось профузне потовиділення. Шкіра червона, гаряча, волога. Яке співвідношення процесів утворення й віддачі тепла в описаному періоді лихоманки?

A. Теплопродукція перевищує тепловіддачу

B. Посилення теплопродукції без зміни тепловіддачі

C. Теплопродукція дорівнює тепловіддачі на більш високому рівні

D. Тепловіддача перевищує теплопродукцію

E. Зниження тепловіддачі на фоні незміненої теплопродукції

347. При обстеженні хворого виявлено такі об'єктивні дані: шкірні покриви гіперемовані, вологі на дотик, відзначається поліурія, полідипсія, температура тіла - 37,20С. Якій стадії лихоманки відповідає цей стан?

A. Теплопродукції

B. Рівноваги

C. Стояння температури

D. Підйому температури

E. Падіння температури

348. У хворого на туберкульоз протягом двох місяців ранкова температура тіла була в межах 36,4 – 36,9 0С, до вечора вона підвищувалася до 37,3 – 37,9 0С. Яка лихоманка за рівнем температури відзначалася у хворого?

A. Субфебрильна

B. Гіперпіретична

C. Помірна

D. Висока

E. Надмірна

349. У хворого з гарячкою температура трималася на рівні 39,6 оС протягом 7 діб. Після цього критично знизилася і була нормальною. Через 6 днів приступ гарячки повторився. Для якого захворювання характерна така температурна крива?

A. Грип

B. Сепсис

C. Поворотний тиф

D. Крупозна пневмонія

E. Малярія

350. У хворого з гарячкою температура трималася на рівні 38,6-39,4 оС протягом 6 діб. Після цього температура літично знизилась і хворий одужав. Для якого захворювання характерна така температурна крива?

A. Малярія

B. Сепсис

C. Поворотний тиф

D. Пневмонія

E. Абсцес легені

351. У хворого з гарячкою температура протягом доби декілька разів коливалася від 37,2 оС до 40,8 оС. Для якого захворювання характерна така температурна крива?

A. Поворотний тиф

B. Грип

C. Сепсис

D. Малярія

E. Крупозна пневмонія

352. У хворого з гарячкою температура різко піднялася до 40,1 оС, утримувалася на високому рівні протягом 2 годин, а потім критично знизилася до 35,9 оС. Через день приступ гарячки повторився. Для якого захворювання характерна така температурна крива?

A. Малярія

B. Сепсис

C. Грип

D. Поворотний тиф

E. Крупозна пневмонія

353. У хворого з гарячкою температура трималася на рівні 39,6 оС протягом 7 діб. Після цього критично знизилася і була нормальною. Через 6 днів приступ гарячки повторився. Який тип температурної кривої у хворого?

A. Febris intermittens.

B. Febris continua.

C. Febris recurrens.

D. Febris hectica.

E. Febris remittens.

354. У хворого після переохолодження температура тіла підвищилася до 39,7 °C і коливалася від 39 °C до 39,8 °C протягом 3 діб. Який тип температурної кривої у хворого?

A. Febris hectica.

B. Febris recurrens.

C. Febris intermittens.

D. Febris continua.

E. Febris remittens.

355. У хворого з гарячкою температура протягом доби декілька разів коливалася від 37,1 оС до 41,1 оС. Який тип температурної кривої у хворого?

A. Febris hectica.

B. Febris continua.

C. Febris recurrens.

D. Febris intermittens.

E. Febris remittens.

356. У хворого протягом доби температура тіла підвищується і залишається високою від 1 до 3 годин, а потім знижується до нормального рівня. Таке підвищення температури тіла спостерігається періодично через 3 дні на четвертий. Який це тип температурної кривої?

A. Febris intermittens.

B. Febris recurrens.

C. Febris continua.

D. Febris hectica.

E. Febris remittens.


Пухлини

357. Чоловік тривалий час працює в нафтопереробній промисловості. Який з наведених класів канцерогенів зустрічається у його оточенні?

A. Нітрозаміни

B. Аміноазосполуки

C. Поліциклічні ароматичні вуглеводні

D. Канцерогени біологічного походження

E. Аміни

358. З анамнезу пацієнта з раком легень відомо, що протягом 20 років він палив до 30 цигарок на добу. До якої групи належать канцерогени тютюнового диму?

A. Гетероциклічні вуглеводні

B. Нітрозаміни

C. Аміни

D. Поліциклічні ароматичні вуглеводні

E. Аміноазосполуки

359. У працівника складу, де зберігали арахіс, виникла пухлина печінки. Яка речовина найімовірніше є причиною виникнення пухлини у хворого?

A. Нітрозаміни

B. Афлатоксин

C. 3,4-Бензпірен

D. β-Нафтиламін

E. Амідопірин

360. Хворий на рак печінки довгий час контактував з диметиламіноазобензолом (ДАБ). Чим пояснюється виникнення пухлини саме в цьому органі?

A. Органотропністю ДАБ

B. Токсичною дією ДАБ

C. Порушеннями жовчовиділення

D. Попереднім захворюванням на гепатит

E. Місцевою канцерогенною дією ДАБ

361. Епідеміологічне дослідження пухлин показало високу кореляцію розвитку пухлин легенів з палінням тютюну. З дією якого канцерогену найбільш імовірно пов'язане виникнення цього виду патології?

A. Метилхолантрену

B. Ортоаміноазотолуолу

C. Афлатоксину

D. 3,4-бензпірену

E. Діетилнітрозаміну

362. У експерименті пухлини часто індукують шляхом введення безклітинного фільтрату з пухлини. Хто вперше успішно поставив такі досліди?

A. Ямагіва та Ішикава

B. Новинський

C. Шоуп

D. Біттнер

E. Еллерман і Банг

363. За даними гінекологічного обстеження у жінки діагностовано рак шийки матки. Який вірус може бути віднесеним до етіологічних факторів розвитку пухлини в даному випадку?

A. Цитомегаловірус

B. HTLV-1

C. Герпеса

D. Аденовірус

E. Грипу

364. У 1910 році Раус в експерименті викликав саркому в курей шляхом введення їм безклітинного фільтрату з саркоми. Який метод експериментального моделювання використовував автор?

A. Експлантації

B. Індукування

C. Ізотрансплантації

D. Гомотрансплантації

E. Гетеротрансплантації

365. У 1908 році Елерман і Банг вперше довели, що віруси можуть викликати лейкоз у курей. Стосовно яких онковірусів доведено, що вони здатні викликати пухлини у людини?

A. Вірус Біттнера

B. Вірус саркоми Роуса

C. Вірус Епстайна – Барра

D. Вірус папіломи Шоупа

E. Вірус поліоми

366. Одну групу щурів тримали в умовах прямого сонячного опромінення, а іншу – в закритих камерах. У тварин, які перебували у відкритих камерах, виникли пухлини непокритих шерстю частин шкіри. Вплив якого фактора зумовив канцерогенез?

A. Біологічних канцерогенів

B. Сонячного тепла

C. Ультрафіолетового опромінення

D. Екзогенних хімічних канцерогенів

E. Інфрачервоного опромінення

367. У 1915 році Ішикава та Ямагіва в експерименті викликали рак шкіри втираючи в неї кам’яновугільну смолу протягом 6 місяців. Який механізм канцерогенезу характерний для хімічних канцерогенів?

A. Мутаційний

B. Епігеномний

C. Трансплантаційний

D. Індукційний

E. Продукційний

368. У 1910 році Раус в експерименті викликав саркому в курей шляхом введення їм безклітинного фільтрату, отриманого з саркоми курки. Який механізм канцерогенезу вмикають лише онкогенні віруси?

A. Трансплантаційний

B. Мутаційний

C. Епігеномний

D. Індукційний

E. Продукційний

369. У постраждалого після опромінення виник гострий лейкоз. Який механізм трансформації характерний для фізичних канцерогенів?

A. Продукційний

B. Епігеномний

C. Трансплантаційний

D. Індукційний

E. Мутаційний

370. Дослідження каріотипу трансформованих клітин хворого на хронічний мієлолейкоз показало наявність філадельфійської хромосоми. Яка хромосомна аберація спричинює активацію протоонкогену і лейкозогенез у цьому випадку?

A. Дуплікація

B. Делеція

C. Транслокація

D. Інверсія

E. Кільцева хромосома

371. Експериментальне вивчення геному пухлинних клітин ретинобластоми показало, що у механізмах канцерогенезу може мати значення:

A. Блокада гена теломерази

B. Активація індукторів поділу

C. Збільшення синтезу ферментів циклу Кребса

D. Транслокація Делеція генів інгібіторів клітинного поділу (генів супресорів)

E. Активація гену білка р53

372. Спостереження за ростом пухлинних клітин в культурі тканин виявило необмеженість їх поділу. Які зміни в пухлинних клітинах зумовлюють відсутність ліміту Хейфліка?

A. Стимуляція апоптозу

B. Активація ДНК-ази

C. Дерепресія гену теломерази

D. Експресія генів супресорів (антионкогенів)

E. Активація циклу Кребса

373. Аналіз експериментальних даних по вивченню онкобілків показав, що їх функція полягає в тому, що вони

A. Зумовлюють контактне гальмування клітинного поділу

B. Є стимуляторами мітозу

C. Мають функцію запуску апоптозу

D. Стимулюють роботу онкогенів

E. Зумовлюють ліміт Хейфліка

374. Шкіру тварини змащували мікрокількостями метилхолантрену. Пізніше ту саму ділянку шкіри змащували кротоновим маслом, і це призвело до виникнення пухлини. З якою стадією канцерогенезу пов’язана дія кротонового масла?

A. Промоція

B. Прогресія

C. Трансформація

D. Транслокація

E. Ініціація

375. У досліді Беренблюма – Моттрама доведено, що кротонове масло, яке саме не є канцерогеном, може бути промотором і сприяє розвитку пухлин. Який ефект на клітину справляє промотор?

A. Зменшує чутливість до цитостатиків

B. Порушує регуляцію поділу

C. Гальмує диференціювання

D. Стимулює до поділу

E. Посилює обмін речовин

376. У експерименті виявлено, що в пухлинних клітинах значно активується синтез певних груп білків. Що таке онкобілки?

A. Метаболіти пухлинних клітин

B. Альбуміни

C. Глобуліни

D. Гормони

E. Продукти онкогенів

377. Після третього курсу хіміотерапії у хворого на злоякісну пухлину легені відсутні будь-які позитивні зміни. Яка властивість пухлин зумовлює розвиток їх резистентності до хіміотерапії?

A. Інвазивний ріст

B. Автономний ріст

C. Прогресія

D. Експансивний ріст

E. Промоція

378. Пацієнта турбує те, що шкіра навкруги пігментного невуса почервоніла. Дослідження показало інвазивний ріст пухлини у хворого. Що можна зазначити як механізм пухлинної прогресії?

A. Інактивація протоонкогена

B. Активація протоонкогена

C. Транслокація протоонкогена

D. Додаткові мутації в трансформованих клітинах

E. Інактивація гена – репресора поділу

379. Після обстеження хворому поставлено діагноз „Рак печінки”. Який білок в крові може свідчити про наявність злоякісної пухлини у хворого?

A. Альбумін

B. Альфа- фетопротеїн

C. Глобулін

D. Парапротеїн

E. С- реактивний

380. Гістологічне дослідження тканини раку шийки матки показало високий ступінь злоякісності пухлини у хворої. Що з нижчезазначеного є показником морфологічної анаплазії пухлини?

A. Наявність спеціалізованих органел

B. Ацидофілія цитоплазми

C. Поліхроматофілія цитоплазми

D. Високе співвідношення ядро/цитоплазма

E. Секреторні гранули в цитоплазмі

381. У хворого виявлено недиференційований рак шлунка з вираженими ознаками анаплазії. Що таке анаплазія пухлинної тканини?

A. Здатність розсувати нормальні тканини

B. Збільшення маси пухлини

C. Перетворення в інший тип тканин

D. Здатність проростати в судини

E. Дедиференціювання пухлинних клітин

382. На саркомі Ієнсена досліджено, що споживання глюкози з артерії, що веде до пухлини, значно збільшується, також наявний приріст вмісту молочної кислоти у відвідній вені. Про що свідчить це явище?

A. Посилення гліколізу

B. Посилення окисного фосфорування

C. Окислення білків

D. Зменшення гліколізу

E. Посилення β-окислення ліпідів

383. У хворого виявлено низькодиференційований рак шкіри. Вкажіть прояв анаплазії пухлин.

A. Безмежний ріст

B. Автономний ріст

C. Антигенне спрощення

D. Нерегульований поділ

E. Відсутність ліміту Хейфліка

384. Клітини гепатоми не здатні до синтезу жовчних кислот. Як може бути названа така властивість пухлинних клітин?

A. Метастазування

B. Морфологічна анаплазія

C. Метаплазія

D. Прогресія

E. Функціональна анаплазія

385. Біохімічний аналіз пухлини, вилученої у хворого, виявив інтенсивний гліколіз навіть в умовах достатнього надходження кисню до клітини. Така особливість обміну речовин пухлинної клітини є проявом:

A. Фізико-хімічної анаплазії

B. Пухлинної прогресії

C. Морфологічної анаплазії

D. Біохімічної анаплазії

E. Метаплазії

386. У хворого на рак легень виявлено гіперпродукцію пухлиною адренокортикотропного гормону. Яка властивість пухлини зумовлює такий розвиток подій?

A. Безмежність росту

B. Метаплазія

C. Моноклональність росту

D. Морфологічна анаплазія

E. Фізико-хімічна анаплазія

387. Під час видалення пухлини м’якої тканини встановлено, що скупчення пухлинних клітин спостерігаються не тільки в осередку, але й серед оточуючих тканин, вздовж судин та нервів. Який тип росту пухлини виявлено в цьому випадку?

A. Генералізований

B. Метастатичний

C. Локальний

D. Інфільтративний

E. Експансивний

388. Хворому встановлено діагноз: злоякісна пухлина легенів. Яка характеристика пухлинного росту найбільш імовірно свідчить про його злоякісність?

A. З однієї клітини

B. Експансивний

C. Безмежний

D. Інфільтративний

E. Нерегульований


Голодування

389. При обстеженні жінки 50 років, що тривалий час перебувала на рослинній дієті, виявлено гіпотермію, гіпотонію, м'язову слабкість, негативний азотистий баланс. Який фактор найімовірніше викликав такий стан пацієнтки?

A. Недостатня кількість вуглеводів у раціоні

B. Надмірна кількість вуглеводів у раціоні

C. Недостатня кількість білків у раціоні

D. Надмірне вживання рідини

E. Недостатня кількість жирів у раціоні

390. При обстеженні чоловіка 45 років, що перебував тривалий час на рослинній дієті, виявлений негативний азотистий баланс. Яка особливість раціону стала причиною цього явища?

A. Надмірна кількість води

B. Надмірна кількість вуглеводів

C. Недостатня кількість жирів

D. Недостатня кількість білків

E. Недостатня кількість жирів і білків

391. У людини внаслідок тривалого голодування швидкість клубочкової фільтрації зросла на 20%. Що є причиною зміни фільтрації в зазначених умовах?

A. Збільшення ниркового плазмотоку

B. Зменшення онкотичного тиску плазми крові

C. Збільшення системного артеріального тиску

D. Збільшення коефіцієнта фільтрації

E. Збільшення проникності ниркового фільтра

392. У тварини, яка тривалий час знаходилася переважно на вуглеводній їжі, виникли набряки. Що стало їх причиною?

A. Гіпопротеїнемія

B. Гіперглікемія

C. Гіпоглікемія

D. Калорійна недостатність

E. Гіповітаміноз А

393. У хворого зі стенозом пілоричного відділу шлунка, який утруднює прийом їжі, спостерігається схуднення, м'язова слабкість, набряки нижніх кінцівок. Який вид голодування у хворого?

A. Часткове (білкове)

B. Водне

C. Абсолютне

D. Неповне (недоїдання)

E. Повне з водою

394. У дитини, хворої на квашиоркор, спостерігаються порушення ороговіння і злущування епідермісу, набряки. Що є найбільш важливим механізмом виникнення цих порушень?

A. Вітамінне голодування

B. Вуглеводне голодування

C. Жирове голодування

D. Білкове голодування

E. Мінеральне голодування

395. У дитини, яка тривалий час голодувала, спостерігаються порушення росту, відсталість розумового розвитку, тяжке виснаження, депігментація шкіри і волосся, набряки. Як називається ця форма білково-калорійної недостатності?

A. Мінеральне голодування

B. Квашиоркор

C. Аліментарна дистрофія

D. Гіповітаміноз

E. Аліментарний маразм

396. У чоловіка, яка тривалий час голодував, спостерігаються тяжке виснаження, депігментація шкіри і волосся, набряки. Як називається ця форма білково-калорійної недостатності?

A. Квашиоркор

B. Аліментарна дистрофія

C. Аліментарний маразм

D. Гіповітаміноз

E. Мінеральне голодування

397. В експерименті в умовах абсолютного голодування водна недостатність частково компенсується утворенням ендогенної води. Яке з наведених речовин при окисненні дає найбільшу кількість оксидаційної води?

A. Амінокислоти

B. Глюкоза

C. Жири

D. Глікоген

E. Білки

398. Пацієнт 28 років голодує 48 годин. Які речовини використовуються м'язовими тканинами як джерело енергії в цьому випадку?

A. Глюкоза

B. Амінокислоти

C. Кетонові тіла й жирні кислоти

D. Лактат

E. Піруват

399. У голодуючої тварини в першу годину повного голодування енергетичні потреби забезпечуються вуглеводами. Якому клінічному періоду голодування відповідають ці зміни?

A. Термінальному

B. Збудження

C. Паралічів

D. Пригнічення

E. Байдужості

400. У щура в експерименті при повному голодуванні з водою спостерігаються незначне підвищення основного обміну, дихальний коефіцієнт рівний 1. Тварина активно шукає їжу. Якому періоду голодування відповідають ці зміни?

A. Термінальному

B. Байдужості

C. Паралічів

D. Пригнічення

E. Збудження

401. У чоловіка 45 років на 4 добу від початку голодування зберігається відчуття голоду, відзначається загальна слабість, пригнічення психіки. Глюкоза крові 2,8 ммоль/л, дихальний коефіцієнт 1,0. В якому періоді голодування за патофізіологічною характеристикою перебуває пацієнт?

A. Максимального пристосування

B. Неощадливої витрати енергії

C. Пригнічення

D. Термінальному

E. Байдужості

402. У тварини під час повного голодування основний обмін на 10% нижчий, ніж в нормі. Дихальний коефіцієнт дорівнює 1,0. Азотистий баланс негативний. У якому періоді голодування за патофізіологічною класифікацією перебуває тварина?

A. Початковому

B. Максимального пристосування

C. Термінальному

D. Неекономної трати енергії

E. Латентному

403. У щура при повному голодуванні з водою спостерігаються незначне підвищення основного обміну, дихальний коефіцієнт 1,0. Тварина активно шукає їжу. Які зміни обміну речовин характерні для цього періоду голодування?

A. Активація ліпогенезу

B. Активація синтезу білка

C. Активація циклу Кребса

D. Активація глікогенолізу

E. Активація глікогенезу

404. Після тривалого голодування в крові людини відзначається підвищення вмісту кетонових тіл. Що зумовлює це явище?

A. Надлишок ацетилКоА

B. Гальмування β-окислення жирних кислот

C. Активація цикла Кребса

D. Гіполіпацидемія

E. Зниження ліполізу

405. При голодуванні (у другому періоді) у хворих може підвищуватися кількість ліпідів крові. Яка форма гіперліпемії наявна в цьому випадку?

A. Церебральна

B. Траспортна

C. Ретенційна

D. Аліментарна

E. Діенцефальна

406. В експериментальної тварини, якій відтворено повне харчове голодування, основний обмін знижений, дихальний коефіцієнт дорівнює 0,7. У крові відзначається кетонемія, у сечі - кетонурія. Який клінічний період голодування характеризують такі зміни?

A. Початковий

B. Байдужності

C. Пригнічення

D. Збудження

E. Паралічів

407. Хворий на злоякісну пухлину стравоходу протягом тижня не приймав їжу. Яким чином змінився гормональний статус у цього хворого?

A. Підвищилася секреція інсуліну

B. Знизилася секреція кортизолу

C. Знизилася секреція адреналіну

D. Знизилася секреція кортикотропіну

E. Підвищилася секреція глюкагону

408. У щурів під час повного голодування дихальний коефіцієнт становить 0,7. Азотистий баланс негативний. У крові ‑ гіперліпацидемія. У якому періоді голодування за патофізіологічною характеристикою перебувають тварини?

A. Байдужості

B. Неощадливої витрати енергії

C. Пригнічення

D. Термінальному

E. Максимального пристосування

409. Під час голодування в організмі виникає перехід на ендогенне харчування. Що з перерахованого найімовірніше варто очікувати на 8 день повного голодування дорослої людини?

A. Дихальний коефіцієнт дорівнює 0,8

B. Інтенсифікація кетогенезу в печінці

C. Пригнічення глюконеогенезу в печінці

D. Зниження ліполізу в жировій тканині

E. Інтенсифікація ліпогенезу в жировій тканині

410. У щура в експерименті на 2-у добу повного голодування з водою дихальний коефіцієнт 0,7, основний обмін знижений на 15%. Тварина пасивна. Які зміни обміну речовин характерні для цього періоду голодування?

A. Активація глікогенолізу

B. Активація синтезу білка

C. Активація ліпогенезу

D. Активація β-окислення жирних кислот

E. Активація глікогенезу

411. У щура в експерименті на 6-у добу повного голодування з водою спостерігаються незначне підвищення основного обміну, дихальний коефіцієнт рівний 0,8. Тварина вкрай виснажена, практично не рухається. Які зміни обміну речовин характерні для цього періоду голодування?

A. Активація ліпогенезу

B. Активація циклу Кребса

C. Активація протеолізу

D. Активація глікогенолізу

E. Активація глікогенезу

412. У щурів при повному голодуванні дихальний коефіцієнт становить 0,8. У сечі багато азоту, калію, фосфору. Тварини виснажені, різко загальмовані. Яка речовина використовується як джерело енергії у цьому періоді голодування?

A. Жирні кислоти

B. Глюкоза

C. Білки

D. Глікоген

E. Кетонові тіла

413. Внаслідок тривалого голодування у людини з’явились ознаки інтоксикації, набряки по всьому тілу, збільшилось виділення із сечею азоту та калію. Посилення якого процесу зумовило ці явища?

A. Катаболізму вуглеводів

B. Катаболізму білка

C. Синтезу білка

D. Катаболізму ліпідів

E. Синтезу глікогену


Гіпоксія

414. У хворої Т. 42 років під час нападу бронхіальної астми при вивченні газового складу крові виявлено наявність гіперкапнії, гіпоксемії та газового ацидозу. Який вид гіпоксії спостерігається в цьому випадку?

A. Дихальна

B. Гіпоксична

C. Циркуляторна

D. Гемічна

E. Гістотоксична

415. Після аварії на хімічному виробництві внаслідок отруєння нітросполуками до лікарні вступили 18 працівників підприємства. Потерпілі скаржилися на різку слабість, головний біль, задишку, запаморочення. У чому полягає причина розвитку гіпоксії?

A. Гальмуванні цитохромоксидази

B. Гальмуванні дегідрогеназ

C. Утворенні метгемоглобіну

D. Зниженні функції флавінових ферментів

E. Утворенні карбоксигемоглобіну

416. Під час пожежі із закритого приміщення, повного диму, винесли в непритомному стані чоловіка приблизно 50 років. Які зміни газового складу крові ймовірні в цьому випадку?

A. Гіпокапнія

B. Зменшення артеріовенозної різниці за киснем

C. Гіпероксія

D. Збільшення артеріовенозної різниці за киснем

E. Зменшення кисневої ємності крові

417. До лікарні звернувся хворий 28 років зі скаргами на слабість, головний біль, зниження працездатності, задишку під час ходьби. При обстеженні газового складу крові виявлено збільшення артеріовенозної різниці за киснем. Яка з перерахованих причин можливо призвела до розвитку гіпоксії в цьому випадку?

A. &nbnbsp; Отруєння ціанідами

B. Зменшення кількості гемоглобіну

C. Серцева недостатність

D. Дихальна недостатність

E. Отруєння чадним газом

418. Група відпочиваючих у кардіологічному санаторії пішла на екскурсію в гори на висоту 1200 м над рівнем моря. Через 2 години від початку походу в одного з екскурсантів з'явилися тахікардія, задишка. Який з перерахованих видів гіпоксії призвів до цих порушень?

A. Тканинна

B. Циркуляторна

C. Гіпоксична

D. Дихальна

E. Гемічна

419. Після марафонського бігу спортсмен почуває себе дуже стомленим. У нього діагностовано гіпоксію напруження. Які зміни показників крові в цих умовах наявні?

A. Підвищення вмісту креатину

B. Підвищення вмісту молочної кислоти

C. Зниження вмісту молочної кислоти

D. Підвищення вмісту сечовини

E. Зниження вмісту сечовини

420. Судово-медичний експерт при розтині трупа 18-річної дівчини встановив, що смерть настала внаслідок отруєння ціаністим калієм. Який фермент найбільшою мірою гальмується ціанідами?

A. Гемсинтетаза

B. Сукцинатдегідрогеназа

C. Піруваткарбоксидаза

D. Цитохромоксидаза

E. 5.Аспартатамінотрансфераза

421. До лікарні машиною швидкої допомоги доставлено з вулиці чоловіка 60 років у непритомному стані. Шкірні покриви бліді. Пульс і дихання прискорені. Кількість еритроцитів 2,52 Г/л, вміст гемоглобіну – 72 г/л, результати газового аналізу: рО2 – 75 мм рт.ст., киснева емність крові – 10 об.%. Який вид гіпоксії наявний?

A. Гемічна

B. Дихальна

C. Гіпоксична

D. Циркуляторна

E. Гістотоксична

422. Робітниця виробництва анілінових барвників доставлена в медичний пункт із ознаками отруєння аніліном. Обличчя синюшне, шкірні покриви і слизові оболонки ціанотичні. Спостерігається задишка. Пульс - 100 уд/хв. Яка причина розвитку гіпоксії найбільш імовірна в цьому випадку?

A. Роз'єднання процесів окислювання та фосфорилювання

B. Утворення карбоксигемоглобіну

C. Гальмування цитохромоксидази

D. Гальмування дегідрогеназ

E. Утворення метгемоглобіну

423. Чоловік 37 років скаржиться на задишку, кашель із виділенням мокротиння, загальну слабість, підвищення температури до 390С. Занедужав після переохолодження. АТ - 110/60 мм рт.ст. Пульс - 98 уд/хв. У процесі обстеження виявлено крупозну пневмонію. Який вид гіпоксії наявний у цьому випадку?

A. Циркуляторна застійна

B. Циркуляторна ішемічна

C. Дихальна

D. Гіпоксична

E. Тканинна

424. При дослідженні впливу гострої гіпоксичної гіпоксії на білих щурів виявлено такі прояви захисно-пристосувальних реакцій: тахіпное з гіпокапнією, тахікардія, збільшення кількості еритроцитів і гемоглобіну. Яким зрушенням кривої дисоціації оксигемоглобіну супроводжуються ці явища?

A. Вгору

B. Вниз

C. Вправо

D. Вліво

E. Не змінюється

425. Чоловік 70 років страждає на хронічну серцеву недостатність. При обстеженні виявлено ціаноз, задишку, періодичні напади ядухи. Який вид гіпоксії , обумовлений даною патологією міг виникнути у хворого?

A. Циркуляторна ішемічна

B. Гемічна

C. Змішана

D. Циркуляторна застійна

E. Тканинна

426. У хворого після перенесеного грипу виникло запалення серцевого м'яза - міокардит. Об'єктивно: задишка, тахікардія, ціаноз, які свідчать про розвиток гіпоксії. Визначте тип гіпоксії :

A. Гіпоксична

B. Циркуляторна

C. Гемічна

D. Респіраторна

E. Тканинна

427. У члена високогірної експедиції на висоті 6 км виникло запаморочення, слабість. Альпініст знепритомнів, дихання зупинилося. Ці порушення виникли внаслідок:

A. Недостатнього звільнення О2 гемоглобіном

B. Недостатнього виведення СО2 з організму

C. Недостатнього утворення СО2 у тканинах

D. Надмірної утилізації О2 тканинами

E. Надмірного виведення СО2 з організму

428. Потерпілого доставили в лікарню з гаража, де він перебував у непритомному стані при працюючому моторі автомобіля. Попередній діагноз - отруєння чадним газом. Розвиток гіпоксії в потерпілого пов'язаний з накопиченням у крові:

A. Дезоксигемоглобіну

B. Карбгемоглобіну

C. Оксигемоглобіну

D. Карбоксигемоглобіну

E. Метгемоглобіну

429. У пацієнта з гострим інфарктом міокарда з'явилися ознаки гіпоксії - тахікардія, задишка, ціаноз, Вкажіть, який вид гіпоксії у хворого?

A. Тканинна

B. Гіпоксична

C. Гемічна

D. Респіраторна

E. Циркуляторна

430. При підйомі на «висоту» у барокамері в щура з'явилися часте дихання, тахікардія, зниження напруги рО2 у крові. Яка форма гіпоксії наявна в цьому випадку?

A. Гемічна

B. Циркуляторна

C. Гіпоксична

D. Тканинна

E. Анемічна

431. До відділення реанімації доставлено хворого, у крові якого виявлено високий вміст сульфгемоглобіну. Який тип гіпоксії наявний у цьому випадку?

A. Респіраторний

B. Гемічний

C. Циркуляторний

D. Тканинний

E. Екзогенний

432. Лікар-дослідник у складі альпіністської експедиції піднявся на висоту 5 км. На 3 день перебування в нього з'явилися ознаки гірської хвороби: задишка, головний біль, втрата апетиту, загальна слабість, ціаноз. Який тип гіпоксії наявний у цьому випадку?

A. Застійний

B. Циркуляторний

C. Гіпоксичний

D. Гемічний

E. Тканинний

433. До клініки доставлено потерпілого у результаті отруєння окисом вуглецю. Яка форма гіпоксії тут наявна?

A. Гемічна

B. Гіпоксична

C. Циркуляторна

D. Дихальна

E. Тканинна

434. При проведенні рятувальних робіт із шахти було винесено шахтаря у непритомному стані без зовнішніх ознак ушкодження. У повітрі шахти містився у великій кількості метан. Який вид гіпоксії розвинувся в потерпілих?

A. Дихальна

B. Гемічна

C. Циркуляторна

D. Тканинна

E. Гіпоксична

435. При клінічному обстеженні жителів гірського селища, розташованого на висоті 3000 метрів, виявлено підвищену кількість еритроцитів у крові. Причиною цього є:

A. Збільшення обсягу циркулюючої крові

B. Підвищення утворення еритропоетинів

C. Збільшення синтезу вітаміну В12

D. Зміна функції селезінки

E. Згущення крові

436. Людина постійно живе високо в горах. Яку зміну показників крові може бути виявлено в неї?

A. Зменшення кольорового показника

B. Зниження показників вмісту гемоглобіну.

C. Появу в крові еритробластів

D. Збільшення кількості еритроцитів

E. Зниження кількості ретикулоцитів

437. Водій після роботи заснув у гаражі у машині із працюючим двигуном. Прокинувшись, він відчув головний біль, почалося блювання. Утворення якої сполуки в крові стало причиною цього стану?

A. Дезоксигемоглобіну

B. Карбгемоглобіну

C. Карбоксигемоглобіну

D. Метгемоглобіну

E. Оксигемоглобіну

438. Пацієнт звернувся зі скаргами на напади утрудненого дихання, запаморочення. З'ясувалося, що він працює на хімічному підприємстві з виробництва синильної кислоти. З порушенням функції якого ферменту можуть бути пов'язані ці симптоми?

A. Каталази

B. Сукцинатдегідрогенази

C. Лактатдегідрогенази

D. Цитохромоксидази

E. Піруватдегідрогенази

439. В експерименті на кролі через 2 тижні після звуження ниркової артерії виявлено збільшення кількості еритроцитів і гемоглобіну в крові внаслідок стимуляції еритропоезу еритропоетинами. Що підсилює утворення еритропоетинів?

A. Гіперосмія

B. Гіпоксія

C. Гіпоосмія

D. Гіповолемія

E. Гіперкапнія

440. У приміщенні відзначається підвищений вміст вуглекислого газу. Як зміниться дихання (глибина і частота) у людини, що увійшла до цього приміщення?

A. Збільшиться частота

B. Зменшиться глибина

C. Зменшиться частота

D. Збільшиться глибина

E. Збільшиться глибина й частота

441. Хвора 13 років перебуває на стаціонарному лікуванні в гематологічному відділенні обласної дитячої лікарні з діагнозом залізодефіцитна анемія. Який тип гіпоксії є в цієї хворої?

A. Гіпоксична

B. Змішана

C. Дихальна

D. Гемічна

E. Циркуляторна

442. Велика травма стегна (з руйнуванням м'яких тканин, нервових стовбурів, судин), що супроводжується масивною кровотечею, зумовила розвиток травматичного шоку. Який вид гіпоксії виникає в цьому випадку?

A. Змішана

B. Гіпоксична

C. Гемічна

D. Циркуляторна

E. Тканинна

443. Декількох працівників виробництва з виготовлення вибухівки доставили в лікарню із симптомами отруєння. Спектральний аналіз крові показав, що це пов'язане з дією бертолетової солі. Який тип гіпоксії розвивається при цьому?

A. Тканинна

B. Гіпоксична

C. Дихальна

D. Циркуляторна

E. Гемічна

444. Чоловік, якого винесли під час пожежі із заповненої димом кімнати, перебуває в непритомному стані. Який вид гіпоксії виник у постраждалого?

A. Циркуляторна.

B. Дихальна.

C. Гемічна.

D. Тканинна.

E. Навантаження.

445. Хворий на крупозну пневмонію скаржиться на задишку, кашель із виділенням мокротиння, загальну слабість, гарячку. Який вид гіпоксії наявний у цьому випадку?

A. Циркуляторна ішемічна

B. Дихальна

C. Циркуляторна застійна

D. Гіпоксична

E. Тканинна

446. У хворого внаслідок передозування наркотичної речовини (уретану) рО2 у венозній крові 80 мм рт.ст., а артеріовенозна різниця за киснем зменшилась. Який вид гіпоксії наявний в цьому випадку?

A. Гемічна

B. Гіпоксична

C. Циркуляторна

D. Тканинна

E. Дихальна

447. У хворого 60 років шкірні покриви бліді, пульс і дихання прискорені. Кількість еритроцитів 2,52 Г/л, вміст гемоглобіну – 72 г/л. Який вид гіпоксії наявний?

A. Гістотоксична

B. Дихальна

C. Гіпоксична

D. Циркуляторна

E. Гемічна

448. При гострій дихальній гіпоксії у щурів виявлені гіпоксемія, гіперкапнія, ацидоз, тахіпное, тахікардія, збільшення кількості еритроцитів і гемоглобіну. Яке зрушення кривої дисоціації оксигемоглобіну буде у щурів?

A. Не змінюється

B. Вправо

C. Вниз

D. Вгору

E. Вліво

449. У хворого з хронічною правошлуночковою серцевою недостатністю виявлені ознаки циркуляторної гіпоксії. Які зміни газового складу крові найбільш ймовірні в цьому випадку?

A. Зменшення рО2 і збільшення рСО2 в артеріальній крові

B. Зменшення артеріовенозної різниці за киснем

C. Збільшення артеріовенозної різниці за киснем

D. Зменшення кисневої ємності крові

E. Збільшення рО2 і рСО2 в артеріальній крові

450. У хворого зі скаргами на задишку, втомлюваність у крові знижені кількість еритроцитів, гемоглобіну, колірний показник, у мазку мікроанізоцитоз, пойкілоцитоз. Що є головним критерієм розвитку гемічної гіпоксії?

A. Зниження кисневої ємності крові.

B. Гіпоксемія та гіпокапнія.

C. Зниження артеріо-венозної різниці за киснем.

D. Збільшення артеріо-венозної різниці за киснем.

E. Гіпоксемія та гіперкапнія.

451. В альпініста на висоті 4000 м над рівнем моря виникла ейфорія, неадекватна оцінка обставин, галюцинації. Яка причина розвитку цих ознак гірської хвороби?

A. Фізичне навантаження.

B. Розширення повітря в лобних пазухах.

C. Зниження парціального тиску кисню в повітрі.

D. Зниження парціального тиску вуглекислого газу в повітрі.

E. Снігова офтальмія.

452. У лікарню доставлено непритомного чоловіка після ремонту автомобіля в гаражному приміщенні. Утворення якої сполуки має провідне значення в патогенезі гемічної гіпоксії в цього пацієнта?

A. Карбгемоглобіну

B. Сульфгемоглобіну

C. Метгемоглобіну

D. Карбоксигемоглобіну

E. Оксиду азоту

453. Після аварії в цеху при роботі з нітросполуками до лікарні потрапили 18 робітників зі скаргами на різку слабість, головний біль, задишку, запаморочення. У чому полягає причина розвитку гіпоксії?

A. Гальмуванні дегідрогеназ

B. Утворенні метгемоглобіну

C. Гальмуванні цитохромоксидази

D. Зниженні функції флавінових ферментів

E. Утворенні карбоксигемоглобіну


ЗМІСТОВИЙ МОДУЛЬ 3 «ТИПОВІ ПОРУШЕННЯ ОБМІНУ РЕЧОВИН»

Енергетичний обмін

454. У тварин, які були адаптовані до холоду, відбувається порушення спряження процесів окислення субстратів і синтезу АТФ. Це явище називається:

A. Пригнічення аеробного окислення глюкози

B. Пригнічення анаеробного окислення глюкози

C. Активація окислювального фосфорилювання

D. Роз'єднання окислення і фосфорилювання

E. Активація β-окислення жирних кислот

455. У хворого зі скаргами на дратівливість, тахікардію, тремтіння пальців рук, екзофтальм виявлене збільшення основного обміну на 20%. Яке ендокринне порушення зумовило ці зміни?

A. Гіпокортицизм

B. Гіпотиреоз

C. Гіпертиреоз

D. Гіперкортицизм

E. Гіпогонадизм

456. У хворого з ожирінням недостатньо розвинені вторинні статеві ознаки. Виявлене зменшення основного обміну на 10%. Яке ендокринне порушення зумовило ці зміни?

A. Гіперкортицизм

B. Гіпотиреоз

C. Гіпертиреоз

D. Гіпокортицизм

E. Гіпогонадизм

457. У хворого з хронічною серцевою недостатністю, зумовленою ревматичною вадою серця, основний обмін підвищений на 30%. Який чинник викликав цю зміну?

A. Посилення роботи серця і органів дихання

B. Посилення роботи скелетних м'язів

C. Роз'єднання окислення і фосфорилювання

D. Зменшення потужності циклу Кребса

E. Активація протеолізу

458. В експерименті тварині ввели динітрофенол. В результаті значно зросло виділення тепла організмом. Вкажіть причину цього порушення.

A. Пригнічення анаеробного окислення глюкози

B. Роз'єднання окислення і фосфорилювання

C. Активація окислювального фосфорилювання

D. Пригнічення аеробного окислення глюкози

E. Активація β -окислення жирних кислот

459. У хворого зі скаргами на зниження пам’яті, сонливість, зниження температури тіла, ожиріння, набряки виявлене зменшення основного обміну на 30%. Яке ендокринне порушення зумовило ці зміни?

A. Гіперкортицизм

B. Гіпертиреоз

C. Гіпокортицизм

D. Гіпотиреоз

E. Гіпогонадизм

460. У хворих на цукровий діабет виникають значні порушення енергетичного обміну. Вкажіть первинну зміну, що зумовлює ці порушення.

A. Активація гліколізу

B. Збільшення β-окислення жирних кислот

C. Зменшення потужності циклу Кребса

D. Активація протеолізу

E. Активація глікогенезу

461. У хворих з бері-бері (гіповітамінозом В1) спостерігаються судоми і психози, зумовлені порушенням енергозабезпечення нейронів головного мозку. Який процес порушується в першу чергу в цьому випадку?

A. Протеоліз

B. β-окислення жирних кислот

C. Анаеробне окислення глюкози (гліколіз)

D. Глікогеноліз

E. Цикл Кребса

462. У хворих з пелагрою (гіповітамінозом РР) один з головних механізмів захворювання пов'язаний з порушенням енергетичного обміну. Робота яких ферментів порушується в першу чергу в цьому випадку?

A. Глікогенолізу

B. β-окислення жирних кислот

C. Анаеробного окислення глюкози (гліколізу)

D. Дихального ланцюга

E. Циклу Кребса

463. У хворих з арибофлавінозом (гіповітамінозом В2) один з головних механізмів захворювання пов'язаний з порушенням енергетичного обміну. Робота яких ферментів порушується в першу чергу в цьому випадку?

A. Дихального ланцюга

B. β-окислення жирних кислот

C. Анаеробного окислення глюкози (гліколізу)

D. Глікогенолізу

E. Циклу Кребса

Вуглеводний обмін

464. Дитина, хвора на цукровий діабет І типу, натще ввела собі інсулін. Через 15 хвилин у неї з’явилися дріж, інтенсивне потовиділення, запаморочення. Що в даному випадку зумовило розвиток гіпоглікемії в дитини?

A. Посилення гліколізу

B. Збільшення споживання глюкози тканинами

C. Посилення глюконеогенезу

D. Посилення ліполізу

E. Посилення глікогенолізу

465. Дитина, хвора на цукровий діабет І типу, натще ввела собі інсулін. Через 15 хвилин у неї з’явилися дріж, інтенсивне потовиділення, запаморочення. Яка зміна гомеостазу викликала ці зміни в організмі дитини?

A. Гіперліпемія

B. Гіперглікемія

C. Кетонемія

D. Гіпоглікемія

E. Глюкозурія

466. У хворого на цукровий діабет після випадкового передозування інсуліну виникли гостре відчуття голоду, тахікардія, підвищення АТ, блідість шкіри, сильна пітливість. Чим зумовлені прояви з боку системи кровообігу?

A. Зниженням активності парасимпатичної нервової системи

B. Підвищенням активності парасимпатичної нервової системи

C. Підвищенням активності симпатичної нервової системи

D. Зниженням активності парасимпатичної нервової системи

E. Зменшенням споживання глюкози

467. У хлопчика, хворого на цукровий діабет, що ввів чергову дозу інсуліну, але не поїв, виникли гостре почуття голоду, дратівливість, агресивна поведінка, згодом запаморочення, судоми. Чим зумовлені зміни з боку центральної нервової системи?

A. Зниженням активності парасимпатичної нервової системи

B. Підвищенням активності симпатичної нервової системи

C. Підвищенням активності парасимпатичної нервової системи

D. Зниженням активності симпатичної нервової системи

E. Зменшенням рівня глюкози в крові

468. У хворого на цукровий діабет після інтенсивного м'язового навантаження на тлі звичайних доз інсуліну виникли гостре відчуття голоду, тахікардія, підвищення АТ, блідість шкіри, сильна пітливість. Нижче якого рівня глюкози в крові виникає гіпоглікемічна кома?

A. 6,1 ммоль/л

B. 1,5 ммоль/л

C. 3,0 ммоль/л

D. 2,5 ммоль/л

E. 4,6 ммоль/л

469. У хворого з недостатністю кори надниркових залоз (хворобою Аддісона) спостерігається виражена схильність до гіпоглікемії. Чим це зумовлено?

A. Надлишком інсуліну

B. Дефіцитом контрінсулярних гормонів

C. Надлишком контрінсулярних гормонів

D. Дефіцитом інсуліну

E. Дефіцитом вуглеводів у їжі

470. У хворого з недостатністю аденогіпофіза (пангіпопітуітаризмом) спостерігається виражена схильність до гіпоглікемії. Чим це зумовлено?

A. Дефіцитом вуглеводів у їжі

B. Надлишком інсуліну

C. Надлишком контрінсулярних гормонів

D. Дефіцитом інсуліну

E. Дефіцитом контрінсулярних гормонів

471. У хворого з аденомою пучкової зони кори надниркових залоз (синдромом Іценка – Кушинга) спостерігається виражена схильність до гіперглікемії. Чим це зумовлено?

A. Дефіцитом контрінсулярних гормонів

B. Надлишком інсуліну

C. Дефіцитом інсуліну

D. Надлишком контрінсулярних гормонів

E. Надлишком вуглеводів у їжі

472. У хворого з еозинофільною аденомою гіпофіза (акромегалією) спостерігається виражена схильність до гіперглікемії. Чим це зумовлено?

A. Дефіцитом інсуліну

B. Надлишком інсуліну

C. Надлишком контрінсулярних гормонів

D. Дефіцитом контрінсулярних гормонів

E. Надлишком вуглеводів у їжі

473. У хворого з пухлиною з β-клітин острівців Лангерганса спостерігається виражена схильність до гіпоглікемії. Чим це зумовлено?

A. Дефіцитом інсуліну

B. Надлишком контрінсулярних гормонів

C. Надлишком інсуліну

D. Дефіцитом контрінсулярних гормонів

E. Дефіцитом вуглеводів у їжі

474. У другий період повного голодування з водою в організмі порушується обмін речовин, у крові збільшується кількість ліпідів, з’являються кетонові тіла, але вміст глюкози зберігається на рівні нижньої межі норми. Що забезпечує еуглікемію?

A. Гліколіз

B. Кетогенез

C. Ліпогенез

D. Глікогеноліз

E. Глюконеогенез

475. Хворому на цукровий діабет І типу введено інсулін. Через деякий час у хворого з’явилися загальна слабкість, гостре почуття голоду, дратівливість, запаморочення. Який основний механізм розвитку проявів гіпоглікемії у пацієнта?

A. Зменшення глюконеогенеpзу.

B. Вуглеводне голодування головного мозку

C. Посилення кетогенезу.

D. Посилення глікогенолізу.

E. Посилення ліпогенезу.

476. У дитини з затримкою фізичного та психічного розвитку, схильністю до гіпоглікемії, гіперліпідемією, лактацидемією, гепатомегалією та остеопорозом діагностовано хворобу Гірке (глікогеноз). Що спричинило гіпоглікемію у хворого?

A. Активація гліколізу

B. Зниження синтезу глікогену

C. Зниження глюконеогенезу

D. Зниження глікогенолізу

E. Активація синтезу глікогену

477. У хворого з аденомою пучкової зони кори надниркових залоз (синдромом Іценка – Кушинга) гіперглікемія зумовлена:

A. Активацією глюконеогенезу в печінці

B. Пригніченням глікогенезу в печінці

C. Пригніченням глікогенолізу в печінці

D. Активацією глікогенезу в печінці

E. Активацією глікогенолізу в печінці

478. Відразу після больового нападу у хворого рівень глюкози у крові склав 7,5 ммоль/л. Чим можна пояснити гіперглікемію?

A. Зниженням активності парасимпатичної нервової системи

B. Підвищенням активності парасимпатичної нервової системи

C. Зниженням активності симпатичної нервової системи

D. Підвищенням активності симпатичної нервової системи

E. Зменшенням споживання глюкози тканинами

479. У хворого з феохромоцитомою (пухлиною мозкової речовини наднирників) спостерігається підвищення рівня глюкози в крові. Гіперглікемія зумовлена:

A. Активацією глікогенезу в печінці

B. Пригніченням глікогенезу в печінці

C. Пригніченням глікогенолізу в печінці

D. Пригніченням глюконеогенезу в печінці

E. Активацією глікогенолізу в печінці

480. У хлопчика 7 років через 2 години після вживання глюкози її вміст у крові складає 20 ммоль/л. Виявлено цукровий діабет 1 типу (інсулінозалежний). Що є причи­ною гіперглікемії при цьому захворюванні?

A. Надлишок інсуліну

B. Резистентність до інсуліну

C. Абсолютний дефіцит інсуліну

D. Дефіцит глюкокортикоїдів

E. Надлишок глюкокортикоїдів

481. У чоловіка 47 років знижена толерантність до вуглеводів. Виявлено цукровий діабет 2 типу. Який механізм веде до гіперглікемії при цьому захворюванні?

A. Активація гліколізу

B. Зниження глюконеогенезу

C. Активація глікогенезу

D. Зниження споживання глюкози тканинами

E. Активація цикла Кребса

482. У хворого на цукровий діабет рівень глюкози в крові складає 32 ммоль/л. Яке з перерахованих порушень обміну речовин є безпосереднім наслідком гіперглікемії?

A. Гіперазотемія

B. Гіперосмія крові

C. Гіперхолестеринемія

D. Аміноацидемія

E. Кетонемія

483. У хворого на цукровий діабет рівень глюкози в крові складає 15 ммоль/л. Яке з перерахованих порушень обміну речовин є безпосереднім наслідком гіперглікемії?

A. Гіперліпацидемія

B. Лактацидемія

C. Гіпохолестеринемія

D. Глюкозурія

E. Кетоацидоз

484. У хворого на цукровий діабет виявлено глюкозурію. При якому мінімальному (пороговому) рівні глюкози в крові виникає глюкозурія?

A. 8,88 ммоль/л

B. 11 ммоль/л

C. 6,1 ммоль/л

D. 4,6 ммоль/л

E. 7,1 ммоль/л

485. У хворого на цукровий діабет виявлено глюкозурію. Яке порушення обміну речовин є безпосереднім наслідком глюкозурії?

A. Кетоацидоз

B. Гіперглікемія

C. Поліурія

D. Ожиріння

E. Схуднення

486. У хворого на цукровий діабет виявлено глюкозурію і поліурію. Яке порушення обміну речовин є безпосереднім наслідком поліурії?

A. Ожиріння

B. Гіперглікемія

C. Кетоацидоз

D. Зневоднення

E. Схуднення

487. У хворого на цукровий діабет рівень глюкози в крові складає 15 ммоль/л. Яке з перерахованих порушень обміну речовин є безпосереднім наслідком гіперглікемії?

A. Кетоацидоз

B. Лактацидемія

C. Гіпохолестеринемія

D. Гіперліпемія

E. Глікерування білків

488. У хворого на цукровий діабет 1-го типу спостерігаються виражені порушення жирового обміну. Яке порушення жирового обміну характерне для цього захворювання?

A. Лактацидемія

B. Гіпохолестеринемія

C. Гіполіпемія

D. Ожиріння

E. Кетонемія

489. У хворого на цукровий діабет 2-го типу спостерігаються виражені порушення жирового обміну. Яке порушення жирового обміну характерне для цього захворювання?

A. Ожиріння

B. Гіпохолестеринемія

C. Кетонемія

D. Гіполіпемія

E. Лактацидемія

490. У хворого на цукровий діабет 2-го типу спостерігаються виражені порушення жирового обміну. Яке порушення жирового обміну характерне для цього захворювання?

A. Кетонемія

B. Гіпохолестеринемія

C. Гіперхолестеринемія

D. Гіполіпемія

E. Лактацидемія

491. У хворого на цукровий діабет 2-го типу спостерігаються виражені порушення жирового обміну. Яке порушення жирового обміну характерне для цього захворювання?

A. Гіполіпемія

B. Гіпохолестеринемія

C. Кетонемія

D. Гіперліпопротеїнемія

E. Лактацидемія

492. У хлопчика 7 років, що значно схуд за останній місяць, виявлено цукровий діабет 1 типу. Що є причи­ною схуднення при цьому захворюванні?

A. Резистентність до інсуліну

B. Абсолютний дефіцит інсуліну

C. Надлишок інсуліну

D. Дефіцит глюкокортикоїдів

E. Надлишок глюкокортикоїдів

493. У хворого на цукровий діабет знижені процеси регенерації, довго не загоюються рани. Які зміни в обміні речовин це спричинюють?

A. Накопичення кетонових тіл.

B. Порушення жирового обміну.

C. Пригнічення синтезу білків.

D. Зменшення надходження глюкози в клітини.

E. Ацидоз.

494. У хворого на цукровий діабет 1-го типу спостерігаються виражені порушення білкового обміну. Яке з перерахованих порушень білкового обміну характерне для цього захворювання?

A. Гіперпротеїнемія

B. Активація синтезу білка

C. Пригнічення протеолізу у м'язах

D. Пригнічення дезамінування амінокислот

E. Негативний азотистий баланс

495. У хворого на цукровий діабет 1-го типу часто спостерігаються порушення водно-електролітного обміну. Яке порушення водно-електролітного обміну характерне для цього захворювання?

A. Гіпергідрія

B. Гіпогідрія

C. Набряки

D. Гіпонатріємія

E. Гіпокаліємія

496. У хворого на цукровий діабет 1-го типу часто спостерігається зневоднення. Що зумовлює це порушення?

A. Поліурія

B. Полідипсія

C. Поліфагія

D. Схуднення

E. Ожиріння

497. Хворий 68 років доставлений у стаціонар непритомним, дихання часте, глибоке, ЧСС – 110 уд/хв., АТ 90/45 мм рт. ст., шкіра в'яла, суха. Лабораторно: глюкоза крові – 15 ммоль/л, молочна кислота крові – 12 ммоль/л. Що є найбільш вірогідною причиною лактацидемії у хворого?

A. Поліфагія

B. Полідипсія

C. Гіпоксія

D. Алкалоз

E. Схуднення

498. У хворого на цукровий діабет може виникати гіперлактацидемія. Що зумовлює це порушення?

A. Схуднення

B. Глюконеогенез

C. Глікогеноліз у печінці

D. Поліфагія

E. Глікогеноліз у м’язах

499. У хворого на цукровий діабет може виникати гіпоксія. Що зумовлює це порушення?

A. Алкалоз

B. Полідипсія

C. Поліфагія

D. Зневоднення

E. Схуднення

500. У експерименті тварині вводили β-цитотропні віруси. Який патологічний стан моделювали в даному експерименті?

A. Відносну інсулінову недостатність

B. Абсолютну інсулінову недостатність

C. Діабет вагітних

D. Нирковий цукровий діабет

E. Нецукровий діабет

501. У експерименті тварині вводили антитіла до інсулінових рецепторів. Який патологічний стан моделювали в даному експерименті?

A. Нирковий цукровий діабет

B. Абсолютну інсулінову недостатність

C. Цукровий діабет вагітних

D. Відносну інсулінову недостатність

E. Нецукровий діабет

502. У експерименті тварині вводили великі дози кортизолу. Який патологічний стан моделювали у даному експерименті?

A. Абсолютну інсулінову недостатність

B. Стероїдний цукровий діабет

C. Нирковий діабет

D. Гіпофізарний цукровий діабет

E. Нецукровий діабет

503. У хворого М. виявлений цукровий діабет 2-го типу. Яка з перерахованих ознак характерна для даного захворювання?

A. Виражені метаболічні порушення і частий кетоз

B. Абсолютна інсулінова недостатність

C. Молодий вік

D. Інсулінорезистентність тканин

E. Аутоімунне ураження β-клітин

504. У дівчини 14 років, яка скаржиться на значне схуднення, спрагу, поліурію діагностовано цукровий діабет 1-го типу. Який патогенетичний механізм є провідним у виникненні цього захворювання?

A. Поява антитіл до інсуліну

B. Інсулінорезистентність тканин

C. Надлишок контрінсулярних гормонів

D. Порушення рецепції інсуліну

E. Аутоімунне ушкодження β-клітин

505. У хворого на аденому гіпофіза (хворобу Іценка – Кушинга) спостерігаються гіперглікемія, глюкозурія, поліурія, поліфагія, полідипсія. Який вид діабету спостерігається у хворого?

A. Вторинний цукровий діабет

B. Первинний цукровий діабет

C. Діабет вагітних

D. Нирковий діабет

E. Нецукровий діабет

506. У чоловіка 64 років, який скаржиться на спрагу, поліурію, ожиріння діагностовано цукровий діабет 2-го типу. Який патогенетичний механізм є провідним у виникненні цього захворювання?

A. Аутоімунне ушкодження β-клітин

B. Надлишок контрінсулярних гормонів

C. Інсулінорезистентність тканин

D. Хронічний панкреатит

E. Пухлина з β-клітин

507. У хворого М. виявлено цукровий діабет 1-го типу. Яка з перерахованих ознак характерна для даного захворювання?

A. Вік після 40 років

B. Абсолютна інсулінова недостатність

C. Незначні метаболічні порушення

D. Зменшення кількості рецепторів до інсуліну на клітинах мішенях

E. Часте ожиріння

508. У дівчини 14 років, яка скаржиться на значне схуднення, спрагу, поліурію виявлено антитіла проти острівцевих клітин. Який варіант цукрового діабету у хворої?

A. 3-й тип

B. 2-й тип

C. 1-й тип

D. 4-й тип

E. Нирковий

509. У вагітної жінки 34 років, яка скаржиться на спрагу, поліурію, вперше виявлено глюкозурію та зниження толерантності до глюкози. Який варіант цукрового діабету у хворої?

A. 1-й тип

B. 4-й тип

C. 2-й тип

D. 3-й тип

E. Нирковий

510. У хворого на аденому кори наднирника (синдром Іценка – Кушинга) спостерігаються гіперглікемія, глюкозурія, поліурія, поліфагія, полідипсія. Який вид діабету спостерігається у хворого?

A. 4-й тип

B. 1-й тип

C. 2-й тип

D. 3-й тип

E. Нирковий

511. У хлопчика з еозинофільною аденомою гіпофіза на фоні значного збільшення швидкості росту із збереженням нормальних пропорцій тіла спостерігається виражена схильність до гіперглікемії. Який варіант цукрового діабету у хворого?

A. Нирковий

B. 1-й тип

C. 2-й тип

D. 4-й тип

E. 3-й тип

512. Жінки 34 років перенесла гострий панкреатит, після чого в неї виникли спрага та поліурія. При обстеженні виявленї глюкозурія та зниження толерантності до глюкози. Який варіант цукрового діабету у хворої?

A. 2-й тип

B. 1-й тип

C. 3-й тип

D. 4-й тип

E. Нирковий

513. У чоловіка 64 років, який скаржиться на спрагу, поліурію, ожиріння діагностовано цукровий діабет з інсулінорезистентністю тканин. Який варіант цукрового діабету у хворого?

A. 3-й тип

B. 1-й тип

C. 2-й тип

D. 4-й тип

E. Нирковий

514. Після перенесеного епідемічного паротиту дитина почала худнути, постійно відчуває спрагу; наявні поліурія, глюкозурія. Вміст глюкози в крові становить 16 ммоль/л, кетонових тіл - 400 мкмоль/л. Що спричинює цукровий діабет у дитини?

A. Надлишок вазопресину

B. Відносний дефіцит інсуліну

C. Надлишок глюкокортикоїдів

D. Дефіцит вазопресину

E. Абсолютний дефіцит інсуліну

515. У хворого виявили глюкозурію на фоні нормальних рівнів глюкози в крові. Який варіант цукрового діабету у хворого?

A. Нирковий

B. 1-й тип

C. 2-й тип

D. 3-й тип

E. 4-й тип

516. У хворого на цукровий діабет розвинулась гіперглікемічна кома (рівень глюкози 54 ммоль/л). Яке патологічне явище лежить в основі порушення функцій клітин мозку в даному випадку?

A. Енергодефіцит

B. Токсичне ураження

C. Гіпогідратація клітин мозку

D. Іонний дисбаланс

E. Гіпергідратація клітин мозку (набухання)

517. У хворого на цукровий діабет 1 типу виникла кома. Який характер коми є найімовірнішим у даному випадку?

A. Гіпоосмолярна

B. Гіпоглікемічна

C. Кетонемічна

D. Гіперосмолярна

E. Лактатацидемічна

518. Хворий 35 років доставлений у стаціонар непритомним, дихання часте, глибоке (типу Куссмауля), АТ 90/45 мм рт. ст., з рота запах ацетону. Глюкоза крові – 18 ммоль/л, у сечі – глюкозурія, кетонурія. Яка кома виникла у даного хворого?

A. Гіпоглікемічна

B. Кетоацидотична

C. Гіперосмолярна

D. Печінкова

E. Лактацидотична

519. Хворий 55 років доставлений у стаціонар непритомним, дихання часте, поверхневе, АТ 90/45 мм рт. ст., шкіра в'яла, суха. Лабораторно: глюкоза крові – 55 ммоль/л, Na – 150 ммоль/л, Cl – 15 ммоль/л Яка кома виникла у даного хворого?

A. Лактацидотична

B. Гіпоглікемічна

C. Кетоацидотична

D. Печінкова

E. Гіперосмолярна

520. Чоловік С., 45 років, хворий на цукровий діабет 2 типу, має вдруге інфаркт міокарда. Яке ускладнення діабету прискорює розвиток ішемічної хвороби серця в хворого?

A. Нейропатія

B. Ретинопатія

C. Макроангіопатія

D. Гломерулопатія

E. Фетопатія

521. У хворого на цукровий діабет 1 типу під час кетонемічної коми розвинулося патологічне гіперпное. Що спричинило розвиток дихання Куссмауля у хворого?

A. Ацидоз видільний

B. Алкалоз газовий

C. Алкалоз видільний

D. Токсична дія ацетону

E. Ацидоз метаболічний

522. Жінка, хвора на цукровий діабет 1 типу, скаржиться на погіршення зору. Обстеження показало в неї діабетичну ретинопатію. Яке ускладнення діабету може бути причиною втрати зору?

A. Макроангіопатія

B. Мікроангіопатія

C. Атеросклероз

D. Нейропатія

E. Гломерулопатія

523. Хворий 68 років доставлений у стаціонар непритомним, дихання часте, глибоке, ЧСС – 110 уд/хв., АТ 90/45 мм рт. ст., шкіра в'яла, суха. Лабораторно: глюкоза крові – 15 ммоль/л, молочна кислота крові – 12 ммоль/л. Яка кома виникла у даного хворого?

A. Лактацидотична

B. Гіпоглікемічна

C. Гіперосмолярна

D. Кетоацидотична

E. Печінкова

524. Дівчина, хвора на цукровий діабет, чекає на донорську нирку. Яке ускладнення діабету є причиною хронічної ниркової недостатності?

A. Атеросклероз

B. Нейропатія

C. Мікроангіопатія

D. Макроангіопатія

E. Ретинопатія

525. У чоловіка 55 років, хворого на цукровий діабет 2-го типу, після незначної травми ступні виникла гангрена ноги. Яке ускладнення діабету є головною причиною гангрени?

A. Кетоацидоз

B. Лактатацидоз

C. Нефропатія

D. Макроангіопатія

E. Ретинопатія

526. У чоловіка 55 років, хворого на цукровий діабет 2-го типу, після незначної травми ступні виникла гангрена ноги. Діагностовано атеросклероз стегнових артерій. Який механізм прискорює розвиток атеросклерозу при цукровому діабеті?

A. Лактацидемія

B. Кетонемія

C. Глікерування апопротеїнів

D. Гіпохолестеринемія

E. Гіполіпацидемія

527. До лікарні доставили хворого на цукровий діабет у стані непритомності. Дихання типу Куссмауля, артеріальний тиск 80/50 мм рт. ст., запах ацетону з рота. Накопиченням в організмі яких речовин можна пояснити виникнення цих розладів?

A. Вугільної кислоти.

B. β-Ліпопротеїдів.

C. Складних вуглеводів.

D. Кетонових тіл.

E. Молочної кислоти.

528. У дитини з затримкою розвитку, розумовою відсталістю, катарактою та цирозом печінки виявлено галактозурію. Активність якого фермента знижена у хворого?

A. Кетогексокінази

B. Галактозо-1-фосфат-уридилтрансферази

C. Фосфорилази

D. Аміло-1,6-глюкозидази

E. Галактозо-1-фосфатази

529. У дитини з ураженням печінки та нирок після вживання фруктів діагностовано фруктозурію та гіпоглікемією. Активність якого ферменту знижена у хворого?

A. Кетогексокінази

B. Галактозо-1-фосфат-уридилтрансферази

C. Фруктокінази

D. Аміло-1,6-глюкозидази

E. Фруктозодифосфат-альдолази

530. У дитини з затримкою фізичного та психічного розвитку, гіпоглікемією, гіперліпідемією, лактацидемією, гепатомегалією та остеопорозом діагностовано хворобу Гірке. Яка речовина накопичується у тканинах хворого?

A. Фруктоза

B. Мукополісахариди

C. Глікоген

D. Сорбітол

E. Глікозаміноглікани

531. У чоловіка з гротескними рисами обличчя, множинними деформаціями кісток та суглобів, ураженням печінки, серця та селезінки діагностовано мукополісахаридоз. Накопичення якої речовини в тканинах спричинило захворювання?

A. Глікогену

B. Фруктозо-1,6-дифосфату

C. Галактозо-1-фосфату

D. Глікозаміногліканів

E. Тригліцеридів

532. У новонародженої дитини після годування молоком спостерігаються диспепсія, блювота. При годуванні розчином глюкози ці явища зникають. Недостатня активність якого ферменту призводить до даних розладів?

A. Амілази

B. Лактази

C. Сахарази

D. Ізомальтази

E. Мальтази

533. У новонародженої дитини діагностована спадкова недостатність сахарази. Який процес при цьому порушений у дитини?

A. Секреторна функція підшлункової залози

B. Порожнинне травлення

C. Екскреторна функція кишок

D. Мембранне травлення

E. Секреція шлункового соку


Жировий обмін

534. У хворої з обтураційною жовтяницею при обстеженні виявлена стеаторея. Відсутність якого компоненту жовчі в кишках призвела до порушення всмоктування та перетравлення жирів?

A. Жирних кислот

B. Прямого білірубіну

C. Холестерину

D. Жовчних кислот

E. Ліпази

535. Хворий 67 років страждає на атеросклероз судин головного мозку. При обстеженні виявлено гіперліпідемію. Підвищення вмісту яких ліпопротеїнів плазми крові найбільше сприяє атерогенезу?

A. Комплексів жирних кислот з альбумінами

B. Ліпопротеїнів високої щільності

C. Альфа-ліпопротеїнів

D. Хіломікронів

E. Ліпопротеїнів низької щільності

536. Хворий 67 років страждає на атеросклероз судин головного мозку. При обстеженні виявлено гіперліпідемію. Зниження вмісту яких ліпопротеїнів плазми крові найбільше сприяє атерогенезу?

A. Хіломікронів

B. Ліпопротеїнів низької щільності

C. Ліпопротеїнів дуже низької щільності

D. Ліпопротеїнів високої щільності

E. Комплексів жирних кислот з альбумінами

537. У хворого з еруптивними ксантомами, гепатоспленомегалією, абдомінальними коліками та панкреатитом виявлено значне збільшення вмісту хіломікронів у плазмі крові. Який дефект лежить в основі виникнення первинної гіперліпопротеїнемії у хворого?

A. Дефект рецептора апоВ/Е

B. Дефіцит ліпопротеїнліпази

C. Підвищена продукція апоВ

D. Аномальний апоЕ

E. Підвищена продукція тригліцеридів, ЛПДНЩ печінкою

538. У хворого з еруптивними ксантомами, гепатоспленомегалією, абдомінальними коліками та панкреатитом виявлено значне збільшення вмісту хіломікронів у плазмі крові. Яка первинна гіперліпопротеїнемія (ГЛП) у хворого?

A. ГЛП I типу

B. ГЛП IIа типу

C. ГЛП IIб типу

D. ГЛП III типу

E. ГЛП IV типу

539. У дитини 10 років з сухожильними ксантомами та тяжкими нападами стенокардії виявлено значне збільшення вмісту ліпопротеїнів низької щільності у плазмі крові. Яка первинна гіперліпопротеїнемія (ГЛП) у хворого?

A. ГЛП III типу

B. ГЛП I типу

C. ГЛП IIб типу

D. ГЛП IIа типу

E. ГЛП IV типу

540. У дитини 10 років з сухожильними ксантомами та тяжкими нападами стенокардії виявлено значне збільшення вмісту ліпопротеїнів низької щільності у плазмі крові. Який дефект лежить в основі виникнення первинної гіперліпопротеїнемії у хворого?

A. Аномальний апоЕ

B. Дефіцит ліпопротеїнліпази

C. Підвищена продукція апоВ

D. Дефект рецептора апоВ/Е

E. Підвищена продукція тригліцеридів, ЛПДНЩ печінкою

541. У хворого 45 років при обстеженні виявлений вміст холестерину в одиниці об’єму крові 5,8 ммоль/л. Який ризик розвитку атеросклерозу в хворого?

A. Дуже високий

B. Дуже низький

C. Низький

D. Високий

E. Середній

542. У хворого 45 років при обстеженні виявлений вміст тригліцеридів в одиниці об’єму крові 2,8 ммоль/л. Який ризик розвитку атеросклерозу в хворого?

A. Низький

B. Дуже низький

C. Високий

D. Середній

E. Дуже високий

543. У хворого 45 років при обстеженні виявлений вміст ЛПНЩ в одиниці об’єму крові 5 г/л. Який ризик розвитку атеросклерозу в хворого?

A. Високий

B. Дуже низький

C. Низький

D. Середній

E. Дуже високий

544. У хворої 45 років при обстеженні виявлений вміст ЛПВЩ в одиниці об’єму крові 6,1 г/л. Який ризик розвитку атеросклерозу в хворої?

A. Низький

B. Дуже низький

C. Середній

D. Високий

E. Дуже високий

545. У хворих на цукровий діабет часто виникає дефіцит ліпокаїчної субстанції. Яке порушення буде наслідком цієї зміни?

A. Аліментарна гіперліпемія

B. Жирова інфільтрація печінки

C. Кетогенез

D. Ожиріння

E. Атеросклероз

546. У хворого встановлений діагноз – вторинне ожиріння. Що могло стати його причиною?

A. Вторинна недостатність лептину

B. Первинна недостатність лептину

C. Гіперпродукція кортизолу

D. Зменшення утворення інсуліну

E. Перегодовування в грудному віці

547. У мишей лінії Цукера (ob/ob) спонтанно виникає первинне ожиріння з ненаситним апетитом. Який механізм виникнення ожиріння у цих мишей?

A. Гіпогонадизм

B. Вторинна недостатність лептину

C. Гіперкортицизм

D. Енцефаліт

E. Первинна недостатність лептину

548. У хворого встановлений діагноз – гіперпластичне ожиріння. Що могло стати його причиною?

A. Вторинна недостатність лептину

B. Первинна недостатність лептину

C. Перегодовування в грудному віці

D. Гіперпродукція інсуліну

E. Психогенна булемія

549. У хворого встановлений діагноз – вторинне ожиріння. Що могло стати його причиною?

A. Перегодовування в грудному віці

B. Первинна недостатність лептину

C. Вторинна недостатність лептину

D. Зменшення утворення інсуліну

E. Енцефаліт

550. У мишей лінії db/db спонтанно виникає ненаситний апетит, гіпотермія та гіпометаболізм, і розвивається ожиріння, при якому маса тіла значно перевищує норму. Який механізм виникнення ожиріння?

A. Гіперкортицизм

B. Первинна недостатність лептину

C. Вторинна недостатність лептину

D. Енцефаліт

E. Гіпогонадизм

551. Після введення ауротіоглюкози у частини тварин виникли гіперфагія, посилення ліпогенезу, зниження термогенезу, викликаного вживанням їжі, та ожиріння. Який механізм виникнення ожиріння?

A. Гіперкортицизм

B. Первинна недостатність лептину

C. Вторинна недостатність лептину

D. Ураження вентромедіальних ядер гіпоталамуса

E. Ураження вентролатеральних ядер гіпоталамуса

552. У людини діагностовано первинне ожиріння. З порушенням утворення якого гормону воно пов’язане?

A. Інсуліну

B. Лептину

C. Адреналіну

D. Кортизолу

E. Соматотропіну

553. У людини, яка перехворіла важкою формою грипу, з’явилися ожиріння, судинні кризи, сильний головний біль. Який вид ожиріння (за етіологією) виник у хворого?

A. Аліментарно-конституціональне

B. Аліментарне

C. Церебральне

D. Спадкове

E. Ендокринне

554. У мишей зі спадковим ожирінням і цукровим діабетом 2-го типу виявлено гіперглікемію та зниження кількості інсулінових рецепторів у міоцитах. Який первинний патогенетичний механізм посилення ліпогенезу в цих тварин?

A. Гіперплазія ліпоцитів.

B. Гіперінсулінемія.

C. Зниження толерантності до глюкози.

D. Порушення мобілізації жирів.

E. Гіпоінсулінізм.

555. У хворих на первинне ожиріння значно збільшені апетит та споживання їжі. Яка речовина стимулює центр голоду і активізує харчову поведінку людини?

A. Нейропептин Y

B. Адреналін

C. Нейропептин I

D. Лептин

E. Кахексин

556. У хворих з ожирінням у крові збільшені рівень фібриногену, загальна коагуляцій на активність та знижений рівень гепарину. Які порушення можуть виникнути у хворого при таких змінах?

A. Вірусні інфекції

B. Крововиливи

C. Бактеріальні інфекції

D. Тромбози

E. Гематурія

557. Хвора на нейрогенну анорексію вкрай виснажена. Маса тіла дівчини нижча за нормальну на 48%. Як називається такий стан?

A. Лікувальне голодування

B. Екзогенна кахексія

C. Ожиріння

D. Ендогенна кахексія

E. Фізіологічне голодування

558. У хворого із опіками 35% площі шкіри та зниженням маси на 38% діагностовано опікову хворобу у стадії опікового виснаження. Яка речовина, що пригнічує центр голоду та стимулює катаболізм, викликала виснаження у хворого?

A. Соматотропін

B. Інсулін

C. Нейропептин Y

D. Кахексин

E. Лептин

559. У хворого із злоякісною пухлиною спостерігається значне схуднення і виснаження. Яка речовина, що пригнічує центр голоду та стимулює катаболізм, викликала виснаження у хворого?

A. Лептин

B. Інсулін

C. Нейропептин Y

D. Соматотропін

E. Кахексин

560. У хворого на цукровий діабет 1-го типу Спостерігається значна втрата маси тіла. Яка зміна обміну речовин викликала схуднення у хворого?

A. Збільшення продукції соматотропіну

B. Збільшення продукції нейропептину Y

C. Відсутність анаболічної дії інсуліну

D. Лептинова недостатність

E. Збільшення продукції кахексину


Білковий обмін

561. У хворого на цироз печінки встановлене значне зниження в крові вмісту альбумінів. Яке порушення в організмі найбільш вірогідно може бути наслідком такої гіпопротеїнемії?

A. Набряки

B. Геморагії

C. Анемія

D. Енцефалопатія

E. Жовтяниця

562. У хворого з гепатитом вміст загального білка в крові становить 40 г/л, вміст альбумінів і фібриногену знижений. Які зміни якісного білкового складу крові наявні в цьому випадку?

A. Гіперазотемія

B. Гіперпротеїнемія

C. Диспротеїнемія

D. Гіпопротеїнемія

E. Парапротеїнемія

563. Чоловік 65 років, що страждає на подагру, скаржиться на болі в зоні нирок. При ультразвуковому дослідженні встановлено наявність ниркових каменів. Яка зміна веде до утворення ниркових каменів у цьому випадку?

A. Протеїнурія

B. Гіперурикемія

C. Гіперпротеїнемія

D. Гіперазотемія

E. Гіпербілірубінемія

564. Через місяць після видалення у хворого значного відрізка тонкої кишки у нього досліджували кров на вміст білка в плазмі, альбумін склав 12 г/л. Яка зміна білкового складу крові виникла?

A. Гіперазотемія

B. Диспротеїнемія

C. Гіперпротеїнемія

D. Парапротеїнемія

E. Гіпопротеїнемія

565. Пацієнтка 60 років звернулася до лікаря зі скаргами на болі в малих суглобах ніг і рук. Суглоби збільшені, мають вигляд потовщених вузлів. У сироватці крові підвищений вміст уратів. Порушення обміну якої речовини може бути причиною розвитку цього стану?

A. Амінокислот

B. Піримідинів

C. Пуринів

D. Ліпідів

E. Вуглеводів

566. У хворого з інфекційним захворюванням концентрація білка в крові складає 120 г/л. Яка кількісна зміна білкового складу крові наявна?

A. Гіпопротеїнемія

B. Гіперпротеїнемія

C. Парапротеїнемія

D. Диспротеїнемія

E. Гіперазотемія

567. До лікаря звернувся чоловік 65 років зі скаргами на гострий біль у великих пальцях ніг. Він любить і часто вживає пиво. Виникла підозра на подагру. Вміст якої речовини необхідно визначити в крові для підтвердження діагнозу?

A. Кетонових тіл

B. Білірубіну

C. Сечової кислоти

D. Сечовини

E. Лактату

568. У хворого з вірусним гепатитом А вміст загального білка в крові становить 60 г/л, вміст альбумінів і фібриногену знижений, гамма-глобуліну підвищений. Які зміни якісного білкового складу крові наявні в цьому випадку?

A. Парапротеїнемія

B. Гіпопротеїнемія

C. Гіперпротеїнемія

D. Гіперазотемія

E. Диспротеїнемія

569. У дитини 10 років з сечокам’яною хворобою, уповільненням росту та мегалобластною анемією, резистентною до вітамінотерапії, виявлено оротоацидурію. Синтез якої речовини порушений у хворого?

A. Уридину

B. Сечової кислоти

C. Кетонових тіл

D. Пуринів

E. Лактату

570. У людини, що виконувала важку фізичну роботу в умовах підвищеної температури навколишнього середовища, змінився білковий склад плазми крові. Яка форма зміни білкового складу крові буде спостерігатися при цьому?

A. Абсолютна гіперпротеїнемія

B. Відносна гіперпротеїнемія

C. Гіпопротеїнемія

D. Диспротеїнемія

E. Парапротеїнемія

571. Альбіноси погано переносять вплив сонця: засмага не розвивається, а з’являються опіки. Порушення метаболізму якої амінокислоти лежить в основі цього явища?

A. Глутамінової

B. Гістидину

C. Триптофану

D. Тирозину

E. Метіоніну

572. При аналізі сечі тримісячної дитини виявлено підвищену кількість гомогентизинової кислоти, сеча при стоянні на повітрі набуває темного забарвлення. Для якого з перелічених нижче захворювань характерні описані зміни?

A. Цистинурії

B. Альбінізму

C. Аміноацидурії

D. Алкаптонурії

E. Фенілкетонурії

573. При клінічному обстеженні у вагітної жінки виявлено збільшення вмісту фенілаланіну в крові. Що з названого нижче може виникнути у дитини?

A. Синдром Шерешевського-Тернера

B. Галактоземія

C. Олігофренія

D. Хвороба Вільсона

E. Синдром Дауна

574. Відомо, що фенілкетонурія виникає внаслідок мутації гена, відповідального за перетворення амінокислоти фенілаланіну й розпаду її до кінцевих продуктів обміну - СО2 і Н2О. Вкажіть, який шлях обміну фенілаланіну призведе до розвитку фенілкетонурії?

A. Фенілаланін –> тирозин –> меланін

B. Фенілаланін –> тирозин –> тироксин

C. Фенілаланін –> тирозин –> норадреналін

D. Фенілаланін –> тирозин –> алкаптон

E. Фенілаланін –> фенілпіруват –> кетокислоти

575. При обстеженні у хворого було встановлено діагноз алкаптонурії - спадкового захворювання, зумовленого дефіцитом ферменту:

A. Тироксингідроксилази

B. Оксидази гомогентизинової кислоти

C. Фенілаланінгідроксилази

D. Тирозинази

E. Моноамінооксидази

576. У дитини 6 місяців з уповільненим моторним і психічним розвитком відзначається посвітління шкірних покривів, волосся, райдужної оболонки ока, позитивна проба Фелінга (з 3% розчином трихлороцтового заліза). Яке з зазначених спадкових захворювань виявлено в дитини?

A. Хвороба Дауна

B. Галактоземія

C. Алкаптонурія

D. Фенілкетонурія

E. Альбінізм

577. У хворого 25 років білі волосся та шкіра, рожево-червоні очі. Скаржиться на поганий зір вдень та підвищену чутливість шкіри до сонця – фотодерматит. Дефіцит якого фермента зумовлює це захворювання?

A. Тирозинази

B. Тироксингідроксилази

C. Оксидази гомогентизинової кислоти

D. Фенілаланінгідроксилази

E. Моноамінооксидази

578. У дитини на 6-му місяці життя з’явилося тяжке ураження центральної нервової системи, посвітління шкіри та волосся. Порушення метаболізму якої амінокислоти лежить в основі цього явища?

A. Фенілаланіну

B. Гістидину

C. Триптофану

D. Глутаминової

E. Метіоніну

579. У дитини з розумовою відсталістю встановлено кретинізм. Який механізм є головним у розвитку пошкодження ЦНС при цьому?

A. Посилення екскреції з сечею фенілкетонових тіл

B. Зменшення синтезу тирозину

C. Збільшення синтезу триптофану

D. Зменшення синтезу тироксину

E. Накопичення в крові фенілала­ніну і фенілкетонів

580. У дитини 8 місяців виникли прояви пелагри. Порушення метаболізму якої амінокислоти лежить в основі цього явища?

A. Глутамінової

B. Гістидину

C. Фенілаланіну

D. Триптофану

E. Метіоніну

581. У хворого на адреногенітальний синдром спостерігається позитивний азотистий баланс. Який механізм зумовив це явище?

A. Протеїнурія

B. Гіперпродукція гормону росту

C. Гіперпродукція тиреоїдних гормонів

D. Гіперпродукція андрогенів

E. Голодування

582. У хворих з печінковою комою накопичується амоній, який зумовлює порушення енергозабезпечення нейронів головного мозку. Який процес порушується в першу чергу в цьому випадку?

A. Анаеробне окислення глюкози (гліколіз)

B. β-окислення жирних кислот

C. Цикл Кребса

D. Глікогеноліз

E. Протеоліз

583. Пацієнт, в якого наявна лихоманка протягом трьох тижнів, втратив вагу. Визначення залишкового азоту крові показало, що він становить 50 ммоль/л. Який азотистий баланс у хворого?

A. Відносний

B. Позитивний

C. Нормальний

D. Негативний

E. Абсолютний

584. У хворого на цукровий діабет залишковий азот крові становить 35 ммоль/л. Який вид гіперазотемії виник у хворого?

A. Продукційна

B. Ретенційна

C. Транспортна

D. Відносна

E. Абсолютна

585. Пацієнт, що одужує після тяжкої форми крупозної пневмонії, почав потроху відновлювати вагу. Який азотистий баланс у хворого?

A. Негативний

B. Позитивний

C. Нормальний

D. Відносний

E. Абсолютний

586. У хворого на хронічну ниркову недостатність у стадії уремії залишковий азот крові становить 60 ммоль/л. Який вид гіперазотемії виник у хворого?

A. Відносна

B. Продукційна

C. Транспортна

D. Ретенційна

E. Абсолютна

587. У хворого на еозинофільну аденому гіпофіза спостерігається позитивний азотистий баланс. Який механізм зумовив це явище?

A. Гіперпродукція тиреоїдних гормонів

B. Гіперпродукція андрогенів

C. Гіперпродукція гормону росту

D. Протеїнурія

E. Голодування

588. У хворого на цукровий діабет 1-го типу спостерігається негативний азотистий баланс. Який механізм зумовив це явище?

A. Голодування

B. Втрата білків

C. Гіперпродукція анаболічних гормонів

D. Гіперпродукція катаболічних гормонів

E. Гіпопродукція анаболічних гормонів

589. У хворого на опікову хворобу спостерігається негативний азотистий баланс. Який механізм зумовив це явище?

A. Втрата білків

B. Гіперпродукція анаболічних гормонів

C. Гіпопродукція анаболічних гормонів

D. Гіперпродукція катаболічних гормонів

E. Гіпопродукція катаболічних гормонів

590. У пацієнта, хворого на гломерулонефрит, виявлена протеїнурія. Вміст білків у крові становить 30 г/л. Який азотистий баланс у хворого?

A. Відносний

B. Позитивний

C. Нормальний

D. Негативний

E. Абсолютний

591. У хворого на спонгіоформну губчасту енцефалопатію (хворобу Крейтцфельдта – Якоба) у клітинах центральної нервової системи накопичуються убіквітинізовані протеїни. Який процес порушений у нейронах при цій хворобі?

A. Лізосомний протеоліз

B. Протеасомний протеоліз

C. Синтез білків

D. Трансамінування амінокислот

E. Дезамінування амінокислот


Водно–електролітний обмін

592. Піддослідна тварина знаходилась в термостаті при +36 оС. Протягом тривалого часу у неї спостерігалось значне збільшення частоти і глибини дихання. Яке порушення водно-електролітного обміну виникло при цьому?

A. Дегідратація ізоосмолярна

B. Дегідратація гіпоосмолярна

C. Дегідратація гіперосмолярна

D. Позитивний водний баланс

E. Набряк

593. У хворого на гострий ентерит з тяжкою діареєю розвинулося порушення водно-електролітного обміну (ВЕО). Яке порушення ВЕО спостерігається в даному випадку?

A. Гіпергідратація гіпоосмолярна

B. Дегідратація гіпоосмолярна

C. Дегідратація ізоосмолярна

D. Дегідратація гіперосмолярна

E. Гіпергідратація гіперосмолярна

594. У хворого з травматичною комою, який знаходиться у стані непритомності 4-у добу, спостерігаються зниження тургору тканин, сухість слизових оболонок, гіпотензія. Який тип розладу водно-електролітного балансу характеризує поява зазначених ознак?

A. Гіпергідратація ізоосмолярна

B. Гіпергідратація гіперосмолярна

C. Гіпергідратація гіпоосмолярна

D. Дегідратація гіпоосмолярна

E. Дегідратація гіперосмолярна

595. Хворому з опіками II ступеню тяжкості 40% поверхні тіла вводять знеболювальні препарати та протишокові розчини. Яке порушення водно-електролітного обміну найбільш імовірно виникає при опіковій хворобі?

A. Гіпоосмолярна дегідратація

B. Гіпоосмолярна гіпогідрія

C. Ізоосмолярна дегідратація

D. Гіперосмолярна дегідратація

E. Гіперосмолярна гіпергідрія

596. Робітник гарячого цеху, який протягом робочого дня пив водопровідну воду, відчув запаморочення. Спрага відсутня, АТ 90/55 мм рт. ст. Яке порушення водно-електролітного обміну спостерігається в даному випадку?

A. Гіпергідратація ізоосмолярна

B. Дегідратація гіперосмолярна

C. Дегідратація ізоосмолярна

D. Дегідратація гіпоосмолярна

E. Гіпергідратація гіперосмолярна

597. У хворої на гіпофізарну форму нецукрового діабету розвинулося порушення водно-електролітного обміну (ВЕО). Яке порушення ВЕО спостерігається в даному випадку?

A. Дегідратація гіперосмолярна

B. Дегідратація гіпоосмолярна

C. Дегідратація ізоосмолярна

D. Гіпергідратація гіпоосмолярна

E. Гіпергідратація гіперосмолярна

598. У адреналектомованої тварини значно знизився артеріальний тиск. Яке порушення ВЕО спостерігається в даному випадку?

A. Дегідратація гіперосмолярна

B. Дегідратація гіпоосмолярна

C. Дегідратація ізоосмолярна

D. Гіпергідратація гіпоосмолярна

E. Гіпергідратація гіперосмолярна

599. Гіперосмолярна дегідратація розвивається при нецукровому діабеті та при посиленій втраті поту. Зміни якого з перерахованих показників будуть різноспрямованими при цих видах патології?

A. Осмолярності крові

B. Об'єму циркулюючої крові

C. Внутрішньоклітинного об’єму

D. Осмолярності сечі

E. Концентрації натрію в плазмі крові

600. У хворого на цукровий діабет виникла значна спрага, поліурія, порушення психічної діяльності. Який тип розладу водно-електролітного балансу характеризує поява зазначених ознак?

A. Гіпергідратація ізоосмолярна

B. Дегідратація ізоосмолярна

C. Гіпергідратація гіпоосмолярна

D. Дегідратація гіпоосмолярна

E. Дегідратація гіперосмолярна

601. У постраждалого виявлено внутрішню кровотечу, що супроводжується гіпотензією, зниженням тургору тканин, сухістю слизових оболонок, спрагою. Який тип розладу водно-електролітного балансу характеризує поява зазначених ознак?

A. Дегідратація гіпоосмолярна

B. Гіпергідратація гіперосмолярна

C. Гіпергідратація гіпоосмолярна

D. Дегідратація ізоосмолярна

E. Гіпергідратація ізоосмолярна

602. У хворого на синдром Пархона (гіперпродукція вазопресину) розвинулося порушення водно-електролітного обміну (ВЕО). Яке порушення ВЕО спостерігається в даному випадку?

A. Дегідратація гіперосмолярна

B. Гіпергідратація гіпоосмолярна

C. Дегідратація гіпоосмолярна

D. Дегідратація ізоосмолярна

E. Гіпергідратація гіперосмолярна

603. У хворого на гломерулонефрит виникла гостра ниркова недостатність з анурією, головним болем, нудотою, блювотою та судомами. Яке порушення водно-електролітного балансу найбільш імовірно виникло у хворого?

A. Гіпоосмолярна дегідратація

B. Ізоосмолярна гіпогідратація

C. Гіпоосмолярна гіпергідратація

D. Гіперосмолярна гіпергідратація

E. Ізоосмолярна гіпергідратація

604. У хворого на вторинний гіперальдостеронізм розвинулося порушення водно-електролітного обміну (ВЕО). Яке порушення ВЕО спостерігається в даному випадку?

A. Дегідратація гіпоосмолярна

B. Гіпергідратація гіперосмолярна

C. Дегідратація ізоосмолярна

D. Гіпергідратація гіпоосмолярна

E. Дегідратація гіперосмолярна

605. При лікуванні зневоднення безсольовими розчинами на фоні різко зниженої видільної функції нирок з'явилося погіршення загального стану, судомна готовність, явища набухання мозку. Яке порушення водного обміну наявне?

A. Ізоосмолярна гіпергідратація

B. Гіперосмолярна гіпергідратація

C. Гіперосмолярна гіпогідратація

D. Ізоосмолярна гіпогідратація

E. Гіпоосмолярна гіпергідратація

606. У працівника гарячого цеху внаслідок посиленого потовиділення виникла спрага, яку він тамує питтям води без солі. Яке порушення водно-електролітного обміну може при цьому виникнути?

A. Гіперосмолярна гіпергідратація

B. Ізоосмолярна гіпогідратація

C. Гіпоосмолярна гіпергідратація

D. Гіперосмолярна гіпогідратація

E. Ізоосмолярна гіпергідратація

607. У хворого з травмою уретри виникла рефлекторна анурія з головним болем, нудотою, блювотою та судомами. Яке порушення водно-електролітного балансу найбільш імовірно виникло у хворого?

A. Ізоосмолярна гіпогідратація

B. Гіпоосмолярна гіпергідратація

C. Гіпоосмолярна дегідратація

D. Гіперосмолярна гіпергідратація

E. Ізоосмолярна гіпергідратація

608. Хворому разом з лікарськими препаратами внутрішньовенно введено 500,0 мл 5% розчину глюкози. Яке порушення водно-електролітного балансу найбільш імовірно виникне у хворого?

A. Гіперосмолярна гіпергідратація

B. Ізоосмолярна гіпогідратація

C. Гіпоосмолярна дегідратація

D. Гіпоосмолярна гіпергідратація

E. Ізоосмолярна гіпергідратація

609. У одного з моряків, які через аварію на судні в океані змушені були пити морську воду, виникли нестерпне почуття спраги, гарячка, непритомність. Яке порушення водно-електролітного балансу викликало ці прояви?

A. Гіперосмолярна гіпергідратація

B. Ізоосмолярна гіпогідратація

C. Гіпоосмолярна дегідратація

D. Ізоосмолярна гіпергідратація

E. Гіпоосмолярна гіпергідратація

610. Водне навантаження у адреналектомованої тварини призвело до блювання, судом та розвитку коми. Яке порушення водно-електролітного балансу найбільш імовірно виникло у хворого?

A. Гіпоосмолярна дегідратація

B. Ізоосмолярна гіпогідратація

C. Гіпоосмолярна гіпергідратація

D. Гіперосмолярна гіпергідратація

E. Ізоосмолярна гіпергідратація

611. У тварини після введення сулеми виникла гостра ниркова недостатність з анурією, головним болем, нудотою, блювотою та судомами. Яке порушення водно-електролітного балансу найбільш імовірно виникло у хворого?

A. Гіперосмолярна гіпергідратація

B. Ізоосмолярна гіпогідратація

C. Гіпоосмолярна дегідратація

D. Гіпоосмолярна гіпергідратація

E. Ізоосмолярна гіпергідратація

612. При огляді хворого виявлена виражена припухлість в ділянці лівого передпліччя, що виникла після укусу бджоли. Назвіть провідний патогенетичний механізм цього набряку:

A. Лімфогенний

B. Гіперволемічний

C. Застійний

D. Гіпопротеїнемічний

E. Мембраногенний

613. У хворого тяжка нефропатія. Лабораторні дослідження визначають значну протеїнурію, циліндрурію, гіпопротеїнемію, гіперліпемію, гіперкаліємію. Що є найбільш істотною ланкою в патогенезі набряків у цього хворого?

A. Збільшення гідродинамічного тиску крові

B. Зниження онкотичного тиску крові

C. Збільшення тиску позаклітинної рідини

D. Блокада лімфовідтоку

E. Збільшення проникності мікросудин

614. У хворого з серцевою недостатністю за лівошлуночковим типом з'явилися клінічні ознаки набряку легенів. Який з перерахованих патогенетичних механізмів є первинним при цій патології?

A. Лімфогенний

B. Гіперволемічний

C. Гіпопротеїнемічний

D. Застійний

E. Мембраногенний

615. Чоловік 64 років скаржиться на задишку, часте серцебиття, швидку стомлюваність. Увечері з'являються набряки на нижніх кінцівках. Що з переліченого нижче є ведучим патогенетичним фактором цих набряків?

A. Підвищення гідростатичного тиску крові

B. Порушення лімфовідтоку

C. Зниження онкотичного тиску крові

D. Підвищення онкотичного тиску тканинної рідини

E. Підвищення проникності капілярів

616. Значна частина випадків аліментарного голодування супроводжується вираженими набряками. Який з патогенетичних факторів набряку є провідним у цьому випадку?

A. Зниження гідростатичного тиску в тканинах

B. Зниження онкотичного тиску плазми крові

C. Підвищення осмотичного тиску

D. Підвищення онкотичного тиску в міжклітинній рідині

E. Підвищення гідродинамічного тиску в капілярах

617. У хворого 35 років, що переніс гепатит і продовжував вживати алкоголь, розвинулися ознаки цирозу печінки з асцитом і набряками на нижніх кінцівках. Які зміни складу крові стали вирішальними в розвитку набряку?

A. Гіпохолестеринемія

B. Гіпокаліємія

C. Гіпоглобулінемія

D. Гіпоальбумінемія

E. Гіпоглікемія

618. У жінки спостерігається пастозність повік, губ, шиї, язика, які виникли після того, як вона з'їла апельсин. Який патогенетичний механізм є провідним у розвитку набряку в жінки?

A. Порушення лімфовідтоку

B. Підвищення гідродинамічного тиску крові в капілярах

C. Підвищення проникності капілярів

D. Зниження онкотичного тиску крові

E. Підвищення онкотичного тиску тканинної рідини

619. При запаленні в судинах мікроциркуляторного русла відзначається підвищення їх проникності, збільшення гідродинамічного тиску. У тканинній рідині наявні зрушення рН у кислий бік, підвищення осмотичної концентрації й дисперсності білкових структур. Який вид набряку буде спостерігатися в цьому випадку?

A. Колоїдно-осмотичний

B. Мембраногенний

C. Гідродинамічний

D. Змішаний

E. Лімфогенний

620. У хворого, що страждає на серцеву недостатність, спостерігається збільшення печінки, набряки нижніх кінцівок, асцит. Який механізм є провідним в утворенні цього набряку?

A. Гіперволемічний

B. Застійний

C. Гіпопротеїнемічний

D. Лімфогенний

E. Мембраногенний

621. Хворій 34 років 3 роки тому було поставлено діагноз хронічного гломерулонефриту. За останні 6 місяців з'явилися набряки. Що лежить в основі їх розвитку?

A. Гіперосмолярність плазми

B. Гіперальдостеронізм

C. Порушення білковосинтетичної функції печінки

D. Гіперпродукція вазопресину

E. Протеїнурія

622. У хворого на первинний гіперальдостеронізм (синдром Конна) виникли м’язова слабкість, зниження моторики шлунка і кишок, гіпотензія. Яке порушення викликало ці зміни?

A. Гіпомагніемія

B. Гіпонатріемія

C. Гіпокаліемія

D. Гіпокальціемія

E. Гіпофосфатемія

623. У дитини 10 місяців, хворої на рахіт, виникли дратівливість, спазмофілія, порушення мінералізації кісток та їх деформації. Яке порушення викликало ці зміни?

A. Гіпомагніемія

B. Гіпонатріемія

C. Гіпокаліемія

D. Гіпокальціемія

E. Гіпофосфатемія

624. У хворого на хворобу Аддісона вміст Na+ у плазмі крові 100 ммоль/л. Чим може проявитися такий стан?

A. Набуханням клітин

B. Набряками

C. Аритміями

D. Дегідратацією організму

E. Зупинкою серця

625. При експериментальному гіпервітамінозі Д у тварини розвинулись артеріосклероз Менкеберга в аорті, гіперсекреція в шлунку та камені в нирках. Яке порушення викликало ці зміни?

A. Гіперфосфатемія

B. Гіпернатріемія

C. Гіперкаліемія

D. Гіпермагніемія

E. Гіперкальціемія

626. У хворого у плазмі крові: вміст Na+ 170 ммоль/л, осмолярність – 330 мосмоль/л. Чим може проявитися такий стан?

A. Набуханням клітин

B. Набряками

C. Зневодненням клітин

D. Гіповолемією

E. Дегідратацією організму

627. У дитини 5 місяців, хворої на рахіт, виникли дратівливість, спазмофілія, порушення мінералізації кісток. Яке порушення викликало ці зміни?

A. Гіповітаміноз A

B. Гіпервітаміноз Д

C. Гіперпаратиреоз

D. Гіпопаратиреоз

E. Гіповітаміноз Д

628. Хворий на первинний гіперальдостеронізм (синдром Конна) скаржиться на артеріальну гіпертензію. Яке порушення викликало ці зміни?

A. Гіперкальціемія

B. Гіперкаліемія

C. Гіпермагніемія

D. Гіпернатріемія

E. Гіперфосфатемія

629. Після адреналектомії у тварини виникла зупинка серця в діастолі і вона загинула. Яке порушення викликало зупинку серця?

A. Гіперфосфатемія

B. Гіпернатріемія

C. Гіпермагніемія

D. Гіперкальціемія

E. Гіперкаліемія

630. У дітей найбільші прояви рахіту припадають на ранню весну. Що зумовлює загострення хвороби в зимовий час?

A. Руйнування вітаміну Д при термічній обробці їжі

B. Руйнування вітаміну Д при зберіганні продуктів

C. Зниження інсоляції

D. Зростання потреби в кальції

E. Холодна погода

631. Який механізм компенсує надмірну втрату солей натрію в організмі при тривалому потінні в умовах жаркого мікроклімату?

A. Зниження вироблення альдостерону

B. Підвищення вироблення альдостерону

C. Зниження реабсорбции гідрокарбонату в нирках

D. Зниження вироблення реніну

E. Підвищення симпатичних впливів на потові залози

632. При профілактичному огляді дітей закарпатського селища у багатьох виявляється поширений карієс. Дефіцит якої речовини в їжі має відношення до розвитку карієсу?

A. Брому

B. Йоду

C. Фтору

D. Нікелю

E. Цинку

633. У тварини, яка тривалий час знаходилась на дієті з недостатністю одного з катіонів, значно знизилась збудливість нервових и м’язових клітин, зменшився тонус судин. Якого іону не вистачає в організмі?

A. Кальцію

B. Натрію

C. Магнію

D. Калію

E. Заліза

634. У щурів, які тривалий час отримували лише вуглеводну їжу, спостерігалось накопичення води в тканинах. Який патогенетичний механізм є головним у розвитку набряку в даному випадку?

A. Гіпоонкотичний

B. Мембраногенний

C. Дисрегуляторний

D. Лімфогенний

E. Гіперосмолярний

635. У хворого з серцевою декомпенсацією при пункції черевної порожнини отримано прозору лимонно-жовтого кольору рідину, відносна щільність - 1012, альбумінів - 1%, глобулінів - 0,2%, фібриногену немає, одиничні еритроцити, 1-3 лейкоцити в полі зору. Назвіть провідний патогенетичний механізм утворення цієї рідини:

A. Колоїдний

B. Мембраногенний

C. Лімфогенний

D. Гідростатичний

E. Осмотичний

636. У хворого з гіпертонічним кризом виник набряк легенів. Що є головним чинником у розвитку цього стану?

A. Підвищення проникності судин малого кола

B. Збільшення онкотичного тиску тканин

C. Підвищення гідростатичного тиску в капілярах легенів

D. Підвищення опору легеневих судин

E. Зниження онкотичного тиску плазми крові

637. Внаслідок захворювання нирок у пацієнта відзначаються набряки. В аналізах сечі значна кількість білка. Яким механізмом можна пояснити виникнення набряків у такого пацієнта?

A. Зниженням онкотичного тиску лімфи

B. Зниженням онкотичного тиску плазми крові

C. Зниженням фільтраційного тиску в нирках

D. Зниженням онкотичного тиску інтерстицію

E. Підвищенням осмотичного тиску плазми крові

638. У працівника гарячого цеху внаслідок посиленого потовиділення виникла спрага. Яке порушення водно-електролітного обміну може при цьому виникнути?

A. Гіперосмолярна гіпогідратація

B. Ізоосмолярна гіпогідратація

C. Гіперосмолярна гіпергідратація

D. Гіперосмолярна гіпогідрія

E. Гіпоосмолярна гіпергідрія

639. Хворий 40 років скаржиться на загальну слабкість, задишку, серцебиття, набряки на ногах. При огляді: частота дихання - 32 за хвилину, обличчя ціанотичне, асцит, печінка збільшена, пульс - 124 уд./хв., АТ - 170/90 мм рт.ст. Який провідний фактор патогенезу набряку у цього хворого?

A. Підвищення онкотичного тиску міжклітинної рідини

B. Підвищення гідростатичного тиску крові в капілярах

C. Підвищення проникності судинної стінки

D. Порушення лімфовідтоку

E. Підвищення онкотичного тиску крові

640. У хворого на алергію виник набряк Квінке (набряк м'яких тканин тіла). Який з патогенетичних факторів набряку є пусковим у цьому випадку?

A. Підвищення гідродинамічного тиску в капілярах

B. Зниження міжклітинного гідростатичного тиску

C. Зниження онкотичного тиску плазми крові

D. Підвищення проникності стінок капілярів

E. Підвищення осмотичного тиску в тканинах

641. До клініки потрапив чоловік 30 років із тяжкою діареєю протягом 12 годин. Блювання не було. Які зміни водно-електролітного балансу й кислотно-лужної рівноваги спостерігаються у хворого?

A. Негазовий ацидоз із дегідратацією

B. Газовий алкалоз із дегідратацією

C. рН крові без зміни, дегідратація

D. Газовий ацидоз із дегідратацією

E. Негазовий алкалоз із дегідратацією

642. Чоловік 32 років протягом 4 років страждає на хронічний гломерулонефрит. Відзначаються набряки на обличчі, останнім часом з'явилися набряки на ногах і тулубі, тобто гломерулонефрит перебігає з нефротичним синдромом. Який з перерахованих механізмів найбільш характерний для розвитку набряків у цього хворого?

A. Утруднення лімфовідтоку

B. Підвищення онкотичного тиску тканинної рідини

C. Зниження онкотичного тиску крові

D. Підвищення гідродинамічного тиску крові в капілярах

E. Підвищення проникності капілярів

643. Жінку 32 років вжалила оса. На шкірі лівої щоки (у місці укусу) - набряк і гіперемія. Який механізм набряку є первинним у цьому випадку?

A. Підвищення онкотичного тиску тканинної рідини

B. Зниження онкотичного тиску крові

C. Утруднення лимфовідтоку

D. Підвищення гідродинамічного тиску крові в капілярах

E. Підвищення проникності капілярів


Кислотно – основний стан (КОС)

644. У хворого з дихальною недостатністю рН крові 7,36. Визначення рСО2 показала наявність гіперкапнії.Сеча має кислу реакцію. Яка форма порушення кислотно-основного стану в даному випадку?

A. Ацидоз метаболічний компенсований

B. Алкалоз газовий компенсований

C. Алкалоз газовий некомпенсований

D. Ацидоз газовий компенсований

E. Ацидоз метаболічний декомпенсований

645. У хворого виник декомпенсований негазовий ацидоз. Який з показників вказує на декомпенсацію ацидозу?

A. Підвищення рН вище 7,44

B. Зниження бікарбонатів крові

C. Зниження рН нижче 7,36

D. Нормальна величина рН

E. Зниження рСО2

646. У хворого виник компенсований газовий алкалоз. Який з показників вказує на компенсацію алкалозу?

A. Зниження бікарбонатів крові

B. Нормальна величина рН

C. Зниження рН нижче 7,36

D. Підвищення рН вище 7,44

E. Зниження рСО2

647. У хворого виник декомпенсований газовий ацидоз. Який з показників вказує на декомпенсацію ацидозу?

A. Нормальна величина рН

B. Зниження бікарбонатів крові

C. Підвищення рН вище 7,44

D. Зниження рН нижче 7,36

E. Зниження рСО2

648. У хворого виник компенсований негазовий алкалоз. Який з показників вказує на компенсацію алкалозу?

A. Нормальна величина рН

B. Зниження бікарбонатів крові

C. Зниження рН нижче 7,36

D. Підвищення рН вище 7,44

E. Зниження рСО2

649. У хворого виник декомпенсований газовий алкалоз. Який з показників указує на декомпенсацію алкалозу?

A. Зниження рСО2

B. Зниження бікарбонатів крові

C. Зниження рН нижче 7,36

D. Нормальна величина рН

E. Підвищення рН вище 7,44

650. У хворого виник компенсований газовий ацидоз. Який з показників указує на компенсацію ацидозу?

A. Зниження рН нижче 7,36

B. Зниження бікарбонатів крові

C. Нормальна величина рН

D. Підвищення рН вище 7,44

E. Зниження рСО2

651. У хворого виник декомпенсований негазовий алкалоз. Який з показників указує на декомпенсацію алкалозу?

A. Нормальна величина рН

B. Зниження бікарбонатів крові

C. Зниження рН нижче 7,36

D. Підвищення рН вище 7,44

E. Зниження рСО2

652. У хворого виник компенсований негазовий ацидоз. Який з показників указує на компенсацію ацидозу?

A. Зниження бікарбонатів крові

B. Нормальна величина рН

C. Зниження рН нижче 7,36

D. Підвищення рН вище 7,44

E. Зниження рСО2

653. У вагітної жінки розвинувся токсикоз з важкою повторною блювотою протягом доби. До кінця доби почали з’являтися тетанічні судоми і зневоднення організму. Яке порушення КОС викликало описані зміни?

A. Негазовий метаболічний ацидоз

B. Негазовий видільний алкалоз

C. Газовий алкалоз

D. Газовий ацидоз

E. Негазовий видільний ацидоз

654. У хворого діагностовано первинний гіперальдостеронізм (хвороба Конна) Яке порушення КОС може мати місце при цьому?

A. Газовий екзогенний ацидоз

B. Газовий алкалоз

C. Негазовий метаболічний ацидоз

D. Негазовий видільний алкалоз

E. Негазовий видільний ацидоз

655. При підйомі в гори у альпініста розвинулися ейфорія, головний біль, запаморочення, задишка, яка чергувалася з апное. Про яке порушення кислотно-основного стану слід думати?

A. Негазовий алкалоз

B. Газовий ацидоз

C. Екзогенний ацидоз

D. Видільний ацидоз

E. Газовий алкалоз

656. У новонародженої дитини з пілоростенозом спостерігається багаторазова блювота, апатія, слабкість, підвищення тонусу м'язів, іноді виникають судоми. Яка форма порушення кислотно-основного стану розвинулася у дитини?

A. Видільний алкалоз

B. Видільний ацидоз

C. Газовий ацидоз

D. Метаболічний ацидоз

E. Газовий алкалоз

657. У хворого під час операції з надмірною штучною вентиляцією легень виникли судоми. Їх викликало наступне порушення КОС:

A. Негазовий метаболічний ацидоз

B. Негазовий видільний алкалоз

C. Газовий алкалоз

D. Газовий екзогенний ацидоз

E. Негазовий видільний ацидоз

658. У хворого діагностовано вторинний гіперальдостеронізм. Яке порушення КОС може мати місце при цьому?

A. Газовий екзогенний ацидоз

B. Газовий алкалоз

C. Негазовий метаболічний ацидоз

D. Негазовий видільний алкалоз

E. Негазовий видільний ацидоз

659. У хворої з гіперацидним гастритом, яка постійно вживала великі кількості лужних мінеральних вод, раптово виникли тетанія та судоми. Їх викликало наступне порушення КОС:

A. Газовий алкалоз

B. Негазовий екзогенний алкалоз

C. Негазовий видільний алкалоз

D. Негазовий метаболічний ацидоз

E. Негазовий видільний ацидоз

660. У хворого з закритою черепно-мозковою травмою спостерігається виражене гіперпное. Яка форма порушення кислотно-основного стану розвинулася у постраждалого?

A. Газовий алкалоз

B. Видільний алкалоз

C. Видільний ацидоз

D. Газовий ацидоз

E. Метаболічний ацидоз

661. У стоматологічного хворого значно виражений синдром гіперсалівації (птіалізм), унаслідок чого він вимушений випльовувати слину. Яка з форм порушення кислотний-основного стану може розвинутися з часом у даного хворого?

A. Видільний алкалоз

B. Метаболічний ацидоз

C. Газовий алкалоз

D. Видільний ацидоз

E. Газовий ацидоз

662. У хворого на цукровий діабет виявлені високий рівень гіперглікемії, кетонурія, глюкозурія, гіперстенурія і поліурія. Яка форма порушення кислотно-основної рівноваги має місце?

A. Метаболічний алкалоз

B. Газовий ацидоз

C. Метаболічний ацидоз

D. Газовий алкалоз

E. Видільний алкалоз

663. Хвора на цукровий діабет доставлена в лікарню у непритомному стані. При дослідженні кислотно-основного стану (КОС) виявлений негазовий ацидоз. Який первинний механізм зумовив виявлені зміни КОС?

A. Зниження виведення СО2

B. Утворення недоокислених продуктів

C. Надходження екзогенних кислот

D. Порушення буферних систем крові

E. Виведення лужних компонентів з сечею

664. На 10 добу лікувального голодування хворий відчув збудження, з'явилося глибоке, часте дихання, артеріальний тиск знизився до 90/60 мм рт.ст., зменшилося виділення сечі, сеча із запахом ацетону, рН 7,3. Яке порушення КОС у хворого?

A. Гіпоглікемія

B. Газовий ацидоз

C. Видільний ацидоз

D. Метаболічний ацидоз

E. Ниркова недостатність

665. У хворого на холеру в результаті тривалого проносу відбувається значна втрата кишкового вмісту, що стало причиною порушення кислотно-основного стану в організмі. Яка з перерахованих форм порушення КОС має місце ?

A. Газовий ацидоз

B. Газовий алкалоз

C. Негазовий видільний ацидоз

D. Негазовий видільний алкалоз

E. Метаболічний ацидоз

666. Циркуляторна гіпоксія, що виникла у хворого внаслідок хронічної серцевої недостатності, призвела до порушення КОС. Про яку зміну КОС слід думати?

A. Газовий ацидоз

B. Метаболічний кетоацидоз

C. Негазовий алкалоз

D. Газовий алкалоз

E. Метаболічний лактатацидоз

667. У людини з гострою нирковою недостатністю (ГНН) спостерігаються артеріальна гіпертензія, серцева аритмія, дихання Куссмауля, судоми. Яке зрушення КОС характерне для ГНН?

A. Негазовий метаболічний ацидоз

B. Негазовий видільний алкалоз

C. Негазовий видільний ацидоз

D. Газовий алкалоз

E. Газовий ацидоз

668. У хворого діагностовано хронічну недостатність надниркових залоз (хворобу Аддісона). Яке порушення КОС може мати місце в даному випадку?

A. Газовий алкалоз

B. Негазовий видільний ацидоз

C. Негазовий видільний алкалоз

D. Негазовий метаболічний ацидоз

E. Газовий екзогенний ацидоз

669. Внаслідок отруєння морфіном у хворого виникло значне пригнічення дихального центру і порушення легеневої вентиляції. Які зрушення кислотно-основного стану можна виявити у цього хворого?

A. Видільний ацидоз

B. Видільний алкалоз

C. Газовий алкалоз

D. Газовий ацидоз

E. Метаболічний ацидоз

670. У хворого виявлено порушення прохідності дихальних шляхів на рівні дрібних і середніх бронхів. Які порушення кислотно-основного стану в цього хворого?

A. Кетоацидоз

B. Респіраторний ацидоз

C. Метаболічний ацидоз

D. Респіраторний алкалоз

E. Метаболічний алкалоз

671. У хворого на цукровий діабет розвинулася діабетична кома внаслідок порушення кислотно-основного стану. Який вид порушення виник при цьому?

A. Видільний алкалоз

B. Газовий ацидоз

C. Газовий алкалоз

D. Метаболічний алкалоз

E. Метаболічний ацидоз

672. У людини з хронічним гломерулонефритом наростає загальна слабкість, різка тахікардія з періодичною аритмією, загальмованість і сонливість. Яке зрушення КОС супроводжує наближення уремічної коми?

A. Негазовий метаболічний ацидоз

B. Негазовий видільний алкалоз

C. Негазовий видільний ацидоз

D. Газовий алкалоз

E. Газовий ацидоз

673. У хворого на цукровий діабет унаслідок накопичення β-оксимасляної і ацетооцтової кислот має місце порушення кислотно-основного стану, яке називається:

A. Газовий алкалоз

B. Негазовий видільний алкалоз

C. Негазовий метаболічний лактатацидоз

D. Негазовий метаболічний кетоацидоз

E. Газовий ацидоз

674. При обстеженні хворого визначається наявність гіперглікемії, кетонурії, поліурії і глюкозурії. Яка форма порушення кислотно-основної рівноваги має місце в даній ситуації?

A. Газовий алкалоз

B. Метаболічний ацидоз

C. Газовий ацидоз

D. Метаболічний алкалоз

E. Видільний алкалоз

675. У тварини, яка тривалий час провела в умовах надлишку СО2, виникло наступне порушення КОС:

A. Газовий екзогенний ацидоз

B. Газовий екзогенний алкалоз

C. Газовий респіраторний ацидоз

D. Негазовий видільний ацидоз

E. Негазовий екзогенний ацидоз


Обмін вітамінів

676. У хворого на хронічний алкоголізм спостерігаються слабкість та болі в ногах, атрофія м’язів кінцівок, балакучість та ретроградна амнезія. Діагностована хвороба бері-бері. Дефіцит якого вітаміну зумовлює цю патологію?

A. С

B. А

C. В12

D. В1

E. Д

677. У хворого на бері-бері вживання вуглеводів значно погіршило стан хворого і спровокувало психотичні прояви. Який біохімічний процес порушений в організмі при гіповітамінозі В1?

A. Декарбоксилювання

B. Β-Окислення жирних кислот

C. Ліполіз

D. Синтез ДНК

E. Окислення пірувату до ацетилКоА

678. У хворого, що тривалий час знаходився на кукурудзяно-злаковій дієті, спостерігаються дерматит, діарея, деменція. Діагностовано пелагру Дефіцит якого вітаміну зумовлює цю патологію?

A. А

B. PP

C. В1

D. В12

E. К

679. У хворих на арибофлавіноз один з головних механізмів захворювання пов'язаний з порушенням ферментів дихального лагцюга. Дефіцит якого вітаміну зумовив цю патологію?

A. В2

B. РР

C. А

D. В1

E. К

680. У хворих на гіповітаміноз В2 спостерігаються хейлоз, ангулярний стоматит, себорейний дерматит. Функція яких ферментів порушується при цій патології?

A. Синтезу ДНК

B. Ліпогенезу

C. Ліполізу

D. Флавінових дегідрогеназ

E. Синтезу РНК

681. Лікування гіпохромної анемії в пацієнта з хронічним алкоголізмом препаратами заліза була неефективним. При подальшому обстеженні діагностована сидеробластна анемія. Дефіцит якої речовини може бути її причиною?

A. Вітаміну В6

B. Фолієвої кислоти

C. Вітаміну В12

D. Заліза

E. Цинку

682. У хворого на туберкульоз та хронічний алкоголізм після лікування фтивазидом виник дефіцит вітаміну В6 та діагностована залізорефрактерна анемія. Порушення синтезу яких речовин є причиною анемії у хворого?

A. ДНК

B. РНК

C. Порфіринів

D. Еритропоетинів

E. Глікопротеїнів

683. У дитини, хвороі на туберкульоз, після лікування фтивазидом виник дефіцит вітаміну В6, що призвело до значного сповільнення росту. Порушення обміну яких речовин є причиною цієї зміни?

A. РНК

B. Амінокислот

C. ДНК

D. Еритропоетинів

E. Глікогену

684. Через деякий час після резекції пухлини тонкої кишки в хворого виявлено фолієводефіцитну анемію. Який процес найбільше порушується при цьому?

A. Ліполіз

B. Глікогенез

C. Синтез білка

D. Ліпогенез

E. Мітоз

685. У жінки, яку лікували від лейкозу аміноптерином, виявлено мегалобластну анемію і лейкопенію. Порушення синтезу якої речовини виникає при дії антагоністів фолієвої кислоти?

A. Гема

B. РНК

C. Порфіринів

D. ДНК

E. Глобіну

686. Після завершення курсу хіміотерапії лейкозу в хворої розвинулася пневмонія. Застосування яких препаратів стало причиною недостатності імунітету?

A. Вітаміну С

B. Вітаміну А

C. Антагоністів фолієвої кислоти

D. Вітаміну Е

E. Вітаміну В6

687. У жінки, яка хворіє на атрофічний гастрит, виявлено мегалобластну анемію і лейкопенію. Порушення синтезу якої речовини виникає при дефіциті ціанкобаламіну?

A. Глобіну

B. РНК

C. Порфіринів

D. Гема

E. ДНК

688. У чоловіка, якому 5 років тому видалили шлунок, стоматолог виявив атрофічно-запальні процеси слизової ротової порожнини: яскраво-червоний, гладенький («лакований») язик, гінгівіт. Чим зумовлені ці зміни?

A. Дефіцитом вітаміну В12.

B. Психоемоційним перенапруженням.

C. Дефіцитом вітаміну В6.

D. Дефіцитом вітаміну С.

E. Гіперсалівацією.

689. Хворий з гіповітамінозом В12 скаржиться на відчуття „повзання мурашок” по тілу. Чим зумовлене це порушення чутливості?

A. Пошкодженням рецепторів

B. Демієлінізацією чутливих нервів

C. Підвищенням чутливості рецепторів

D. Руйнуванням мотонейронів

E. Мієлінізацією рухових нервів

690. Жінку, яка хворіє на атрофічний гастрит, турбує її швидка втомлюваність, задишка, відчуття оніміння в кінцівках. При обстеженні виявлено гіперхромну макроцитарну анемію, лейкопенію. Що є причиною зменшення кількості еритроцитів і лейкоцитів у крові хворої?

A. Дефіцит вітаміну В12

B. Дефіцит заліза

C. Дефіцит фолієвої кислоти

D. Гемоліз еритроцитів

E. Хронічна крововтрата

691. У хворого, якому 5 років тому зробили резекцію шлунка, діагностовано гіперхромну анемію, зумовлену дефіцитом ціанкобаламіну. Що спричинило розвиток анемії у хворого?

A. Аліментарна нестача фолієвої кислоти

B. Дефіцит заліза

C. Аліментарна нестача ціанокобаламіну

D. Дефіцит внутрішнього фактора Касла

E. Дефіцит вітаміну А

692. Після тривалого застосування сульфаніламідів у хворого з’явились крововиливи на шкірі та слизових оболонках. Нестача якого вітаміну виникла у хворого?

A. С

B. А

C. K

D. В

E. Д

693. На 2 день життя у немовляти виникли кровотеча з пуповини, мелена (чорний дьогтеподібний кал, з червоним обідком на пелюшці), крововиливи на шкірі та слизових оболонках. Нестача якого вітаміну викликала ці прояви?

A. A

B. K

C. С

D. В

E. Д

694. У жінки, що страждає на жовчно-кам’яну хворобу виявлено геморрагічний синдром, зумовлений дефіцитом вітаміну К. Які з перелічених факторів є неповноцінним при гіповітамінозі К?

A. Фактори II, VII, IX, X

B. Фактори VIII, XII

C. Фактор Віллебранда

D. Фактори XIII, V

E. Фактори I, IV

695. У хворого з гіповітамінозом К виявлено зниження вмісту протромбіну (ф. II) в крові. Це призведе до порушення перш за все:

A. Першої фази коагуляційного гемостазу

B. Фібринолізу

C. Третьої фази коагуляційного гемостазу

D. Судинно-тромбоцитарного гемостазу

E. Другої фази коагуляційного гемостазу

696. У хворого 45 років діагностована механічна жовтяниця, що супроводжується стеатореєю та геморагіями на шкірі. У крові - зменшення вмісту протромбіну та інших факторів зсідання крові. Яка причина порушення коагуляційного гемостазу в даного хворого?

A. Підвищена секреція в кров протеолітичних ферментів

B. Токсичне ушкодження гепатоцитів

C. Порушення всмоктування вітаміну К

D. Порушення перетравлення та всмоктування білка

E. Коагулопатія споживання

697. У хворого через наявність каменю у загальній жовчній протоці припинилося надходження жовчі в кишечник. Порушення якого з процесів спостерігається при цьому?

A. Всмоктування білків

B. Всмоктування жиророзчинних вітамінів

C. Перетравлення білків

D. Перетравлення вуглеводів

E. Всмоктування вуглеводів

698. Після тривалого введення великих доз ергокальциферолу у тварини виникли склерозування та кальциноз середньої оболонки артерій. Який патологічний процес моделювали в експерименті?

A. Артеріолосклероз

B. Атеросклероз Маршана

C. Артеріосклероз Менкеберга

D. Вікові зміни артерій

E. Ендартеріїт

699. При експериментальному гіпервітамінозі Д у тварини розвинулись артеріосклероз Менкеберга в аорті, гіперсекреція в шлунку та камені в нирках. Яке порушення викликало ці зміни?

A. Гіпернатріемія

B. Гіперкальціемія

C. Гіперкаліемія

D. Гіпермагніемія

E. Гіперфосфатемія

700. У дитини 10 місяців, хворої на рахіт, виникли дратівливість, спазмофілія, порушення мінералізації кісток та їх деформації. Яке порушення викликало ці зміни?

A. Гіпомагніемія

B. Гіпонатріемія

C. Гіпокаліемія

D. Гіпокальціемія

E. Гіпофосфатемія

701. У дитини 5 місяців, хворої на рахіт, виникли дратівливість, спазмофілія, порушення мінералізації кісток. Яке порушення викликало ці зміни?

A. Гіповітаміноз А

B. Гіпервітаміноз А

C. Гіповітаміноз К

D. Гіпервітаміноз Д

E. Гіповітаміноз Д

702. У дітей найбільші прояви рахіту припадають на ранню весну. Що зумовлює загострення хвороби в зимовий час?

A. Руйнування вітаміну Д при термічній обробці їжі

B. Руйнування вітаміну Д при зберіганні продуктів

C. Зниження інсоляції

D. Зростання потреби в кальції

E. Холодна погода

703. Дитина 9 місяців збуджена, вередлива. У неї порушена мінералізація та ріст кісток. Нестача якого вітаміну виникла у хворого?

A. A

B. Д

C. С

D. К

E. В

704. Внаслідок порушення всмоктування вітаміну у травному каналі у хворого спостерігаєсться ксерофтальмія та кератомаляція. Дефіцит якого вітаміну зумовив цю патологію?

A. К

B. В

C. С

D. Д

E. А

705. Внаслідок тривалого дотримання суворого посту у хворого спостерігаєсться порушення зору у вечірні та нічні години («куряча сліпота»). Дефіцит якого вітаміну зумовив цю патологію?

A. С

B. В

C. А

D. Д

E. К

706. Внаслідок тривалого одноманітного харчування на початку весни у хворого спостерігаюсться кровоточивість ясен та петехії на шкірі. Діагностована цинга. Дефіцит якого вітаміну зумовив цю патологію?

A. В

B. С

C. А

D. Д

E. К

707. Внаслідок тривалого одноманітного харчування на початку весни у хворого спостерігаюсться кровоточивість ясен та петехії на шкірі. Діагностована цинга. Порушення якого процесу є причиною геморагічної вазопатії?

A. Синтез колагену

B. Ліполіз

C. Синтез ДНК

D. Синтез РНК

E. Ліпогенез

Патофізіологія системи крові 1. У жінки 42 років з пухлиною наднирників, що продукує альдостерон, артеріальний тиск крові складає 160/90 мм.рт.ст. Обстеження не виявило відхилень в клітинному складі крові, лише величина гематокриту є 0,32. Яке порушення загального об’єму крові є в хворої?

A. Анемія

B. Еритроцитоз

C. Гіповолемія поліцитемічна

D. Гіперволемія поліцитемічна

E. Гіперволемія олігоцитемічна


2. У дитини з опікової хворобою (площа опіків біля 25 % поверхні тіла), гематокрит складає 0,65. Яке порушення загального об’єму крові має місце в даному випадку?

A. Анемія

B. Еритроцитоз

C. Гіповолемія поліцитемічна

D. Гіповолемія олігоцитемічна

E. Гіперволемія олігоцитемічна


3. У хворого з залізодефіцитною анемією гематокрит складає 0,31. Як можна охарактеризувати зміни загального об’єму крові в даному випадку?

A. Гіповолемія ізоцитемічна

B. Гіповолемія олігоцитемічна

C. Гіповолемія поліцитемічна

D. Гіперволемія поліцитемічна

E. Ізоволемія олігоцитемічна


4. При пораненні пацієнт втратив 15 % об’єму крові. Як можна охарактеризувати зміни загального об’єму крові через 5 діб після крововтрати?

A. Гіповолемія ізоцитемічна

B. Гіповолемія олігоцитемічна

C. Гіповолемія поліцитемічна

D. Ізоволемія поліцитемічна

E. Ізоволемія олігоцитемічна


5. На 7 добу після гострої шлункової кровотечі в 50-річного чоловіка з’явились ознаки нетермінових механізмів компенсації. Які механізми компенсації відносяться до нетермінових?

A. Надходження тканинної рідини до судин

B. Збільшення синтезу білка в печінці

C. Збільшення глибини і частоти дихання

D. Спазм кровоносних судин

E. Збільшення сили і частоти серцевих скорочень


6. При пораненні пацієнт втратив 15 % об’єму крові. Який з нижченаведених механізмів забезпечить відновлення кисневої ємності крові?

A. Посилення еритропоезу

B. Перехід на мегалобластичний еритропоез

C. Посилення лейкопоезу

D. Надходження тканинної рідини до судин

E. Збільшення синтезу білків в печінці


7. При пораненні пацієнт втратив 10 % об’єму крові. Який з нижченаведених механізмів швидко компенсує зниження об’єму крові?

A. Посилення еритропоезу

B. Перехід на мегалобластичний еритропоез

C. Посилення лейкопоезу

D. Надходження тканинної рідини до судин

E. Збільшення синтезу білків в печінці


8. При пораненні пацієнт втратив 15 % об’єму крові. Який з нижченаведених механізмів швидко нормалізує артеріальний тиск?

A. Посилення еритропоезу

B. Посилення дихання

C. Посилення лейкопоезу

D. Спазм кровоносних судин

E. Збільшення синтезу білків в печінці


9. При пораненні пацієнт втратив 15 % об’єму крові. Який з нижченаведених механізмів швидко забезпечить компенсацію гіпоксії?

A. Збільшення хвилинного об’єму серця

B. Зниження артеріального тиску

C. Посилення еритропоезу

D. Надходження тканинної рідини до судин

E. Збільшення синтезу білків в печінці


10. Хворий скаржиться на біль у животі, кал з домішками крові, загальну слабкість. При обстеженні виявлено пухлину товстої кишки і анемію: гемоглобін 4,4 ммоль/л, еритроцити 2,8 • 10 12 /л, колірний показник 0,7. У мазку крові: анізоцитоз, пойкілоцитоз, помірна кількість поліхроматофільних еритроцитів. Яка анемія в хворого?

A. Хронічна постгеморагічна

B. Гіпопластична

C. В12 - дефіцитна

D. Спадкова гемолітична

E. Набута гемолітична


11. При обстеженні пацієнта, що скаржиться на головний біль та порушення зору, виявлено: еритроцити 7 • 10 12 /л, гемоглобін 12 ммоль/л, лейкоцити – 6,8 • 10 9 /л, тромбоцити – 220 • 10 9 /л. Охарактеризуйте зміни в крові пацієнта:

A. Лейкоцитоз

B. Лейкопенія

C. Тромбоцитопенія

D. Анемія

E. Еритроцитоз


12. На восьмому місяці вагітності в жінки розвинулася мегалобластна анемія. Які клітини периферичної крові свідчать про мегалобластичний тип кровотворення?

A. Мегалоцити

B. Анулоцити

C. Мікроцити

D. Ретикулоцити

E. Нормоцити


13. На шостому місяці вагітності в жінки розвинулася залізодефіцитна анемія. Які клітини периферичної крові свідчать про дефіцит гемоглобіну в еритроцитах?

A. Гіперхромні

B. Гіпохромні

C. Нормохромні

D. Ретикулоцити

E. Нормобласти


14. На шостому місяці вагітності в жінки розвинулася гіпохромна анемія. Які клітини периферичної крові свідчать про низький вміст гемоглобіну в еритроцитах при залізодефіцитній анемії?

A. Мегалоцити

B. Анулоцити

C. Мікросфероцити

D. Ретикулоцити

E. Нормобласти


15. У хворого вміст гемоглобіну в крові складає 4,5 ммоль/л, еритроцитів – 2,5 Т/л, в достатній кількості присутні клітини, які свідчать про активність фізіологічних механізмів регенерації «червоної» крові. Які це клітини периферичної крові?

A. Пойкілоцити

B. Мікроцити

C. Мегалоцити

D. Гіпохромні еритроцити

E. Поліхроматофільні еритроцити


16. Тиждень тому пацієнт втратив 15 % об’єму крові. При обстеженні: зменшення вмісту еритроцитів, гемоглобіну, гематокриту, кольорового показника крові, ретикулоцитов - 10%. Дайте характеристику стану «червоної крові» в пацієнта:

A. Анемія апластична

B. Анемія мегалобластна

C. Анемія регенераторна

D. Анемія гіперхромна

E. Еритроцитоз


17. При анеміях в периферичній крові виявляються регенеративні, дегенеративні і патологічні форми еритроцитів. Назвіть регенеративну форму еритроцитів.

A. Пойкілоцити

B. Ретикулоцити

C. Анізоцити

D. Анізохромоцити

E. Сфероцити


18. Пацієнтка С., 17 років, скаржиться на загальну слабкість, хронічну втому. При обстеженні в неї виявлено анемію. Які з регенеративних форм еритроцитів не можуть бути в периферичній крові при анемії?

A. Ретикулоцити

B. Поліхроматофільні еритроцити

C. Мегалобласти

D. Нормобласти

E. Еритробласти


19. У крові хворої після хіміотерапії: гемоглобін 5,0 ммоль/л, еритроцити 2,5 Т/л, КП 1,0, ретикулоцити 0,01%. Визначте анемію в хворої:

A. Гіпохромна

B. Гіперхромна

C. Регенераторна

D. Арегенераторна

E. Залізодефіцитна


20. Для з’ясування стану кісткового мозку при анеміях бажано оцінити його регенераторну здатність. Визначення кількості яких клітин в крові дозволяє це зробити?

A. Ретикулоцити, нормобласти

B. Мікроцити, макроцити

C. Овалоцити, ехіноцити

D. Анізоцити, анізохромоцити

E. Пойкілоцити


21. Пацієнт скаржиться на загальну слабкість, швидку втому при невеликому навантаженні. У його крові зменшено вміст еритроцитів і гемоглобіну. Наявність вільного гемоглобіну в крові дозволяє діагностувати такий механізм анемії в хворого:

A. Крововтрату

B. Гемоліз

C. Недостатність еритропоезу

D. Нестачу вітаміну В12

E. Нестачу заліза


22. Привертає увагу жовтувато-лимонний відтінок шкіри в жінки. У крові: гемоглобін 4,6 ммоль/л, еритроцити 2,8 Т/л, КП 0,82, ретикулоцити 7%; збільшено вміст непрямого білірубіну. Склад «білої крові» не змінений. Для яких захворювань системи крові така картина крові є типовою:

A. Фолієводефіцитні анемії

B. Залізодефіцитні анемії

C. Мегалобластні анемії

D. Гемолітичні анемії

E. Хронічні лейкози


23. Пацієнт скаржиться на загальну слабкість, втомлюваність, жовтяницю. У крові зменшено вміст анемія, ретикулоцитів - 5%, збільшення вмісту непрямого білірубіну, гемоглобінурія. Зазначте механізм анемії в хворого:

A. Гостра крововтрата

B. Хронічна крововтрата

C. Гемоліз інтраваскулярний

D. Гемоліз позаваскулярний

E. Недостатність еритропоезу


24. Пацієнт скаржиться слабість, жовтушність шкіри. У крові зменшено вміст гемоглобіну, еритроцитів, КП крові 0,8, ретикулоцитів 5%; кількість лейкоцитів і тромбоцитів в межах норми. Що може бути механізмом цих змін в крові пацієнта?

A. Дефіцит еритропоетинів

B. Хронічна крововтрата

C. Гемоліз еритроцитів

D. Зменшення синтезу гемоглобіну

E. Зменшення клітинного поділу


25. У дитини з наявним резус-антигеном на еритроцитах, що народилася від резус-негативної матері, зменшено вміст гемоглобіну в крові. Алергічні реакції якого типу (за Кумбсом і Джелом) лежать в основі цього захворювання?

A. I типу (анафілактичні)

B. II типу (гуморальні цитотоксичні )

C. III типу (імунокомплексні)

D. IV типу (клітинні цитотоксичні)

E. V типу (стимулюючі)


26. Пацієнтка скаржиться на напади болю в спині, які супроводжуються появою сечі бурого кольору; обстеження показало пароксизмальну нічну гемоглобінурію. Надайте патогенетичну характеристику анемії в хворої:

A. Гемолітична спадкова

B. Гемолітична набута

C. Дисеритропоетична

D. Залізодефіцитна

E. Гіперхромна


27. Точкова мутація гена β-ланцюга гемоглобіну є причиною анемії. Вкажіть типову для даної анемії патологічну форму гемоглобіну.

A. Hb A2

B. Hb A

C. Hb F

D. Hb S

E. Барт- Hb


28. У пацієнтки з жовчнокам’яною хворобою діагностовано спадкову анемію (мембранопатію). Для яких анемій холелітіаз є типовим ускладненням?

A. Після крововтрати

B. Гемолітичні

C. Залізодефіцитні

D. Фолієводефіцитні

E. Апластичні


29. Лікування препаратами заліза не дало позитивного ефекту в хворої на мікроцитарну, гіпохромну анемію. Подальше обстеження дозволило встановити діагноз таласемії. Дайте характеристику цієї анемії:

A. Ферментопатія

B. Мембранопатія

C. Гемоглобінопатія

D. Імунна

E. Залізодефіцитна


30. У хворої на системний червоний вовчак в крові визначаються антиеритроцитарні антитіла, зменшення кількості еритроцитів і вмісту гемоглобіну. Надайте етіопатогенетичну характеристику цієї анемії:

A. Постгеморагічна

B. Гемолітична набута імунна

C. Гемолітична набута токсична

D. Гемолітична спадкова

E. Дисеритропоетична


31. Лікування гіпохромної анемії в пацієнта препаратами заліза була неефективним. При подальшому обстеженні встановлено діагноз залізорефрактерна, сидеробластна анемія. Дефіцит якої речовини може бути її причиною?

A. Фолієвої кислоти

B. Вітаміну В 12

C. Вітаміну В 6

D. Заліза

E. Цинку


32. Через деякий час після резекції пухлини тонкої кишки в хворого виявлено фолієводефіцитну анемію. Які клітини є типовими в периферичній крові при цьому?

A. Мікроцити

B. Анулоцити

C. Мегалобласти

D. Нормобласти

E. Еритробласти


33. У працівника типографії лікування гіпохромної анемії препаратами заліза не було ефективним. При подальшому обстеженні діагностована залізорефрактерна анемія. Порушення синтезу яких речовин є причиною цієї анемії?

A. РНК

B. ДНК

C. Порфіринів

D. Еритропоетинів

E. Білків


34. У жінки, яка хворіє на атрофічний гастрит, виявлено мегалобластну анемію і лейкопенію. Порушення синтезу якої речовини є причиною виникнення мегалобластного типу кровотворення?

A. РНК

B. ДНК

C. Порфіринів

D. Гема

E. Глобіну


35. Хвора скаржиться на слабкість, тривалі менорагії. Об’єктивно: фіброміома матки і гіпохромна гіпорегенераторна анемія. Який механізм виникнення анемія в хворої?

A. Дефіцит В 6

B. Втрата заліза

C. Дефіцит В 12

D. Імунний гемоліз

E. Токсичний гемоліз


36. Жінку, яка хворіє на атрофічний гастрит, турбує її швидка втомлюваність, задишка, відчуття оніміння в кінцівках. При обстеженні виявлено гіперхромну макроцитарну анемію, лейкопенію. Що є причиною зменшення кількості еритроцитів і лейкоцитів у крові хворої?

A. Дефіцит заліза

B. Дефіцит фолієвої кислоти

C. Дефіцит вітаміну В 12

D. Гемоліз еритроцитів

E. Хронічна крововтрата


37. Жінка скаржиться на зниження працездатності, сонливість, задишку при швидкій ході. При обстеженні: еритроцити – 4,0 Т/л, гемоглобін – 4,8 ммоль/л, КП – 0,6; анулоцитоз, мікроцитоз. Що може бути причиною такого стану крові?

A. Дефіцит фолієвої кислоти

B. Дефіцит вітаміну В 12

C. Дефіцит заліза

D. Дія гемолітичної отрути

E. Недостатність еритропоезу


38. У хворого, якому 5 років тому зробили резекцію шлунка, діагностовано гіперхромну анемію. Що спричинило розвиток анемії у хворого?

A. Дефіцит заліза

B. Аліментарна нестача ціанокобаламіну

C. Аліментарна нестача фолієвої кислоти

D. Дефіцит внутрішнього фактора Касла

E. Дефіцит вітаміну А


39. У хворої на лімфогранулематоз після променевої терапії: гемоглобін 6,0 ммоль/л, еритроцити 3,0 •10 12 /л, КП 1,0, ретикулоцити 0,1%. Визначте анемію в хворої:

A. Постгеморагічна

B. Гемолітична

C. Гіпопластична

D. Залізодефіцитна

E. Мегалобластична


40. У жінки, віком 55 років, яка тривалий час страждає на залізодефіцитну анемію, виявлені дистрофічні зміни в міокарді. Що могло стати причиною дистрофії міокарда у хворої?

A. Зниження активності клітинних фосфоліпаз

B. Зниження активності залізовмісних ферментів

C. Зникнення гемосидерину в макрофагах печінки

D. Зникнення трансферину з плазмі крові

E. Зниження кількості сидеробластів у кістковому мозку


41. У приймальне відділення лікарні поступила дівчинка 9 років з болем у животі. Обстеження показало гострий апендицит. Як змінюється клітинний склад периферичної крові при гострому нормергічному запаленні?

A. Анемія

B. Агранулоцитоз

C. Нейтропенія

D. Лімфопенія

E. Нейтрофільний лейкоцитоз


42. Дослідження показало, що загальна кількість лейкоцитів в крові пацієнта становить 3,0 Г/л, лімфоцити складають 10%. Що може бути причиною змін клітинного складу "білої" крові у цьому випадку?

A. Опіки

B. Травма

C. Інфекція

D. Опромінення

E. Отруєння чадним газом


43. Загальна кількість лейкоцитів у крові пацієнта складає 10 Г/л; при цьому нейтрофілів - 82%. Вкажіть можливу причину такого складу "білої" крові:

A. Вірусна інфекція

B. Травма, опіки

C. Гельмінти

D. Хронічне запалення

E. Анафілактичні реакції


44. У крові пацієнта загальна кількість лейкоцитів складає 3,0 Г/л; при цьому лімфоцити складають 80%. Охарактеризуйте зміни клітинного складу "білої" крові в зазначеному випадку:

A. Нейтропенія

B. Нейтрофілія

C. Еозинопенія

D. Лімфоцитоз

E. Лімфопенія


45. Мати хлопчика 4-х років скаржиться на те, що дитина погано їсть, дратівлива. Обстеження показало, що хлопчик інфікований аскаридами. Які зміни з боку периферичної крові є типовими для гельмінтозів?

A. Лімфоцитоз

B. Моноцитоз

C. Нейтрофілія

D. Еозинофілія

E. Базофілія


46. До лікарні поступив чоловік 47 років із загрудинним болем. За результатами ЕКГ в нього діагностовано інфаркт міокарда. Які зміни клітинного складу периферичної крові індуковані некротичними змінами в міокарді?

A. Тромбоцитоз

B. Лімфопенія

C. Лімфоцитоз

D. Нейтропенія

E. Нейтрофілія


47. До лікарні поступила жінка з опіками. Як в типових випадках при гострому пошкодженні тканин змінюється клітинний склад периферичної крові?

A. Еритропенія

B. Лімфопенія

C. Нейтропенія

D. Лімфоцитоз

E. Нейтрофілія


48. Юнак звернувся до лікаря зі скаргами на нежить, біль у горлі, гарячку. При обстеженні, у крові: лейкоцити 10 Г/л, лімфоцити - 55%, нейтрофіли - 45%. Інфекція якого типу найбільш ймовірно є причиною гострого респіраторного захворювання в пацієнта?

A. Коки

B. Віруси

C. Грибки

D. Мікобактерії

E. Гельмінти


49. Загальна кількість лейкоцитів у крові пацієнта складає 10 Г/л; при цьому лімфоцити складають 56%, нейтрофіли 40 %. Охарактеризуйте зміни клітинного складу "білої" крові в зазначеному випадку:

A. Агранулоцитоз

B. Гранулоцитоз

C. Нейтропенія

D. Лімфопенія

E. Лімфоцитоз


50. Загальна кількість лейкоцитів у крові пацієнта 7 Г/л, еозинофіли складають 15%. Що може бути причиною такого порушення складу "білої" крові?

A. Гостре запалення / вірусна інфекція

B. Гостре запалення / травма

C. Гостре запалення / кокова інфекція

D. Глистяна інфекція

E. ВІЛ – інфекція


51. У пацієнта з макроцитарною гіпорегенераторною анемією вміст лейкоцитів в крові складає 4 Г/л, лімфоцитів 40%, нейтрофілів 40%. Ядра нейтрофілів містять 5-7 сегментів. Вкажіть зміни "білої" крові у пацієнта.

A. Лейкоцитоз

B. Лімфоцитоз

C. Ядерний нейтрофільний зсув вліво

D. Ядерний нейтрофільний зсув вправо

E. Агранулоцитоз


52. Загальна кількість лейкоцитів у крові пацієнта 10 Г/л; при цьому нейтрофілів 75%, серед них 10% - паличкоядерні. Охарактеризуйте зміни клітинного складу "білої" крові в цьому випадку:

A. Нейтропенія

B. Агранулоцитоз

C. Лімфоцитоз

D. Ядерний зсув нейтрофілів вліво

E. Ядерний зсув нейтрофілів вправо


53. Охарактеризуйте ядерне зрушення нейтрофілів у хворого на гостру бактеріальну пневмонію: лейкоцити 11 Г/л, лімфоцити 27%, нейтрофіли - 70%, з них: ю- 2%, п/я - 8%, с/я - 58%.

A. Вліво регенераторне

B. Вліво гіперрегенераторне

C. Вліво дегенеративне

D. Вліво регенераторно-дегенеративне

E. Вправо


54. У крові хворого на бактеріальну пневмонію збільшено вміст нейтрофілів. Яке зрушення ядер нейтрофілів периферичної крові є типовим для гострого нормергічного запалення?

A. Вліво регенераторне

B. Вліво гіперрегенераторне

C. Вліво дегенеративне

D. Вліво регенераторно-дегенеративне

E. Вправо


55. Пацієнт скаржиться на часті інфекційні захворювання. При обстеженні: лейкоцити 3,8 Г/л: лімфоцити - 40 %, нейтрофіли: п/я — 1 %, с/я — 49 %, гіперсегментація ядер нейтрофілів. Як можна визначити зрушення "білої" крові у пацієнта?

A. Вліво регенеративне

B. Вліво гіперрегенеративне

C. Вліво регенеративно-дегенеративне

D. Вліво дегенеративне

E. Вправо


56. У дитини з гострим апендицитом у крові: лейкоцити – 11 Г/л, лімфоцити - 23%, нейтрофіли: юні - 4%, п/я - 8%, с/я - 58%. Яке зрушення нейтрофілів периферичної крові у дитини?

A. Вліво регенеративне

B. Вліво гіперрегенеративне

C. Вліво дегенеративне

D. Вліво регенеративно-дегенеративне

E. Вправо


57. У юнака з ангіною у крові: Лейк. - 11•10 9 /л, Лімф. - 21%, Мон. - 5%, Еоз. - 2 %, Баз. - 0%. Нейтр.: юні - 5%, п/я - 10%, с/я - 57%. Яке зрушення нейтрофілів периферичної крові є типовим для гострого запалення?

A. Вліво регенеративне

B. Вліво гіперрегенеративне

C. Вліво дегенеративне

D. Вліво регенеративно-дегенеративне

E. Вправо


58. При обстеженні у пацієнтки виявлена лейкопенія. У формулі: лімфоцити - 34%, нейтрофіли: юні - 0%, п/я - 20%, с/я- 39%. Яке зрушення нейтрофілів периферичної крові спостерігається в даному випадку?

A. Вліво регенеративне

B. Вліво гіперрегенеративне

C. Вліво дегенеративне

D. Вліво регенеративно-дегенеративне

E. Вправо


59. У пацієнта хворого на крупозну пневмонію у крові: лейкоцити 16 Г/л. Лімфоцити - 26%, нейтрофіли: мієлоцити - 2%, юні - 6%, п/я 12%, с/я - 50%. Яке зрушення нейтрофілів периферичної крові у хворого?

A. Вліво регенеративне

B. Вліво гіперрегенеративне

C. Вліво дегенеративне

D. Вліво регенеративно-дегенеративне

E. Вправо


60. У хворого на гострий бронхіт у крові: лейкоцити – 11 Г/л, лімфоцити - 23%, нейтрофіли: юні - 4%, п/я - 8%, с/я- 58%. Лейкоцити з токсичною зернистістю. Яке зрушення нейтрофілів периферичної крові спостерігається в даному випадку?

A. Вліво регенеративне

B. Вліво гіперрегенеративне

C. Вліво дегенеративне

D. Вліво регенеративно-дегенеративне

E. Вправо


61. У крові пацієнта: знижено вміст гемоглобіну та еритроцитів, лейкоцитів 27 Г/л, гранулоцитоз, ядерне зрушення нейтрофілів вліво до промієлоцитів. Що може бути причиною вищезазначених змін?

A. Анемія

B. Хронічний лімфолейкоз

C. Хронічний мієлолейкоз

D. Гострий лейкоз

E. Еритроцитоз


62. Хворий 67 р., скаржиться на втомлюваність, схуднення, жовтушність шкіри і склер. При обстеженні: анемія, лейкоцити — 54 Г/л, тромбоцити — 200 Г/л. Серед лейкоцитів 90% складають малі та середні лімфоцити. Що може бути причиною такого стану?

A. Дефіцит фолієвої кислоти

B. Надлишок глюкокортикоїдів

C. Гострий лейкоз

D. Хронічний лімфолейкоз

E. Хронічний мієлолейкоз


63. В абсолютній більшості випадків хронічного мієлолейкозу в клітинах пухлинного клону є "Філадельфійська" хромосома. Яка подія відбувається із стовбуровою кровотворною клітиною кісткового мозку внаслідок зазначеної хромосомної аберації?

A. Трансформація

B. Прогресія

C. Припинення поділу

D. Порушення дозрівання

E. Вихід у кров


64. Трансплантація стовбурових клітин кісткового мозку здатна повністю відновити кровотворну і лімфатичну системи реципієнта, що був під дією опромінення або цитотоксичних хіміопрепаратів. Вкажіть потенційне джерело великої кількості стовбурових кровотворних клітин:

A. Звичайна венозна кров

B. Звичайна артеріальна кров

C. Пуповинна кров

D. Плацента

E. Шкіра


65. Хворий на гострий лейкоз після кірсу хіміотерапії почіває себе задовільно. Що можна трактувати як показник стану ремісії при гострому лейкозі?

A. Вміст гемоглобіну в крові більше 10 ммоль/л

B. Нейтрофіли в крові менше 1 Г/л

C. Бласти в крові менше 10%

D. Бласти в кістковому мозку менше 5%

E. Бласти в кістковому мозку менше 30%


66. Хвора скаржиться на часті інфекційні захворювання останнім часом. При обстеженні: анемія, тромбоцити — 220 Г/л, лейкоцити — 10 Г/л; 70% складають нейтрофіли, з них: 45% - промієлоцити, 25% сегментоядерні. Що може бути причиною такого стану в системі крові?

A. Хронічний мієлолейкоз

B. Хронічний лімфолейкоз

C. Гострий лейкоз

D. Інтоксикація

E. Гостра інфекція


67. Хвора Р., 47 р., скаржиться на втомлюваність, носові кровотечі, біль у кістках. При обстеженні: еритроцити — 2,7 Т/л, гемоглобін — 4,0 ммоль/л, лейкоцити — 14 Г/л, тромбоцити — 70 Г/л. Серед лейкоцитів крові 7% складають бласти. Що може бути причиною цих змін?

A. Еритроцитоз

B. Анемія

C. Нейтропенія

D. Гострий лейкоз

E. Хронічний лейкоз


68. У крові пацієнта зменшено вміст всіх клітин: анемія, нейтропенія, тромбоцитопенія. Дослідження клітинного складу кісткового мозку показало гострий мієлолейкоз. Що є специфічною ознакою гострих лейкозів?

A. Велика кількість зрілих гранулоцитів в крові

B. Збільшення кількості дозріваючих гранулоцитів в крові

C. Збільшення кількості незрілих клітин в кістковому мозку

D. Велика кількість незрілих клітин в крові

E. Велика кількість зрілих та дозріваючих клітин в


кістковому мозку 69. У хворого виявлені анемія, тромбоцитопенія, гранулоцитоз. Загальна кількість лейкоцитів у крові складає 57 Г/л, ядерний зсув нейтрофілів вліво до промієлоцитів. У кістковому мозку - гранулоцитоз. Яка патологія наявна в хворого?

A. Гострий лейкоз

B. Хронічний мієлолейкоз

C. Хронічний лімфолейкоз

D. Агранулоцитоз

E. Еритроцитоз


70. Пацієнту, який скаржиться на загальну слабкість та біль в кістках, встановлено діагноз "Хронічний мієлолейкоз". Назвіть хромосомну аномалію, типову для більшості випадків цього захворювання.

A. Делеція Х

B. Делеція 12

C. Трисомія 21

D. Транслокація 9 на 22

E. Транслокація 21 на 15


71. У крові пацієнта: анемія, тромбоцитопенія, загальна кількість лейкоцитів складає 3,0 г/л, лімфоцити складають 60%. Що може бути причиною вищеописаної зміни складу "білої" крові:

A. Гостре запалення / травма / бактеріальна інфекція

B. Глистяна інфекція

C. Дефіцит заліза

D. Хронічний лейкоз

E. Гострий лейкоз


72. Чоловік 50 р. звернувся до лікаря з приводу збільшення лімфатичного вузла на шиї. Будь-які неприємні відчуття в цьому місці в нього відсутні. За результатами біопсії діагностовано злоякісну пухлину з клітини лімфатичного вузла. Як звуться такі пухлини системи крові?

A. Гострий лейкоз

B. Хронічний лейкоз

C. Лімфома

D. Карцинома

E. Аденома


73. Хворому на лейкоз зроблено аналіз крові. Що найімовірніше вказує на наявність гострого лейкозу?

A. Наявність лейкемічного провалу

B. Лейкоцитоз

C. Анемія

D. Дегенерація лейкоцитів

E. Поява в крові бластів


74. Після третього курсу хіміотерапії у хворого на хронічний мієлолейкоз відсутні будь–які позитивні зміни. Яка властивість пухлин пов’язана з розвитком їх резистентності до хіміотерапії?

A. Автономний ріст

B. Інвазивний ріст

C. Експансивний ріст

D. Прогресія

E. Промоція


75. У 1908 році Елерман і Банг вперше довели, що віруси можуть викликати лейкоз у курей. Стосовно якого онковірусу доведено, що він здатен викликати лейкоз у людини?

A. Вірус герпесу

B. Вірус гепатиту В

C. Вірус HTLV

D. Аденовірус

E. Вірус поліоми


76. У хворого на хронічний мієлолейкоз визначається гіперплазія ясен, стоматит. Дослідження біоптату слизової оболонки ротової порожнини виявило її лейкоцитарну інфільтрацію. З якою властивістю пухлини пов’язане ураження ротової порожнини у хворого?

A. Автономність росту

B. Метастазування

C. Імморталізація

D. Анаплазія

E. Метаплазія


77. У постраждалого після опромінення виник гострий лейкоз. Який механізм трансформації характерний для фізичних канцерогенів?

A. Епігеномний

B. Мутаційний

C. Трансплантаційний

D. Індукційний

E. Продукційний


78. У хворого на хронічний мієлолейкоз на фоні резистентності пухлини до хіміотерапії виник бластний криз, що було розцінено лікарем як прояв пухлинної прогресії. Що складає суть пухлинної прогресії?

A. Здатність до безмежного поділу

B. Антигенне спрощення клітин

C. Розповсюдження по організму

D. Малігнізація

E. Експансивний ріст


79. У 1907 році Елерман і Банг в експерименті викликали лейкоз у курей шляхом введення їм безклітинного фільтрату, отриманого з крові курки. Який механізм канцерогенезу вмикають лише онкогенні віруси?

A. Епігеномний

B. Мутаційний

C. Трансплантаційний

D. Індукційний

E. Продукційний


80. Стан хворого на хронічний мієлолейкоз раптово значно погіршився, звичайні протипухлинні препарати перестали діяти. Що можна трактувати як гематологічний прояв пухлинної прогресії хронічного мієлолейкозу?

A. Анемія

B. Еритроцитоз

C. Тромбоцитоз

D. Гранулоцитоз

E. Лейкемічний провал


Патологія гемостазу 81. У хворого на цингу спостерігаються кровоточивість ясен та петехії на шкірі. Що спричинює порушення гемостазу при цьому захворюванні?

A. Тромбоцитопенія

B. Надлишок антикоагулянтів

C. Активація фібринолізу

D. Дефіцит прокоагулянтів

E. Порушення синтезу колагену


82. У дівчинки після перенесеної ангіни з’явився петехіальний висип на шкірі кінцівок та тулуба. Об’єктивно: кількість тромбоцитів 80 Г/л, антитромбоцитарні антитіла. Алергічні реакції якого типу (за Кумбсом і Джелом) лежать в основі цього захворювання?

A. I типу (анафілактичні)

B. II типу (гуморальні цитотоксичні )

C. III типу (імунокомплексні)

D. IV типу (клітинні цитотоксичні)

E. V типу (стимулюючі)


83. У хворого порушена адгезія тромбоцитів до колагену та спостерігаються кровотечі з дрібних судин. Порушення якої ланки гемостазу можна припустити у хворого?

A. Коагуляційної, І фази

B. Судинно-тромбоцитарної

C. Коагуляційної, ІІІ фази

D. Фібринолізу

E. Коагуляційної, ІІ фази


84. У хворого діагностовано тромбоцитопенію. Які клінічні прояви типові для порушень тромбоцитарно – судинного гемостазу?

A. Петехії, екхімози (синці)

B. Гемартрози

C. Гематоми

D. Зменшення часу кровотечі

E. Збільшення часу згортання крові


85. У хворої при обстеженні виявили тромбоцитопатію. Вкажіть, яка зміна відіграє важливу роль в патогенезі тромбоцитопатій?

A. Зниження активності антикоагулянтів

B. Продукція патологічних тромбоцитів кістковим мозком

C. Гіперактивація тромбоцитопоезу

D. Підвищення концентрації в крові прокоагулянтів

E. Пригнічення фібринолізу.


86. Перед проведенням оперативного втручання виявлено, що в людини час кровотечі збільшений до 10 хв. Дефіцит яких формених елементів у складі крові може бути причиною таких змін?

A. Еритроцитів

B. Моноцитів

C. Лімфоцитів

D. Тромбоцитів

E. Лейкоцитів


87. У пацієнтки тривале вживання аспірину викликало крововиливи. Об’єктивно: тромбоцитопенія з порушенням функціональної активності тромбоцитів. Тромбоцитопатія в даному випадку зумовлена пригніченням активності:

A. Цитохромоксидази

B. Ліпооксигенази

C. Циклооксигенази

D. Супероксиддисмутази

E. Фосфоліпази А 2 .


88. У хворого діагностовано зменшення продукції ендотелієм судин фактора Віллебранда. Яке порушення судинно-тромбоцитарного гемостазу спостерігається при цьому?

A. Порушення агрегації тромбоцитів

B. Гіперкоагуляція

C. Порушення полімеризації фібрину

D. Порушення адгезії тромбоцитів

E. Посилений фібриноліз


89. У дитини з геморагічним висипом, що виник після ГРВІ, діагностовано геморагічний васкуліт (хвороба Шенляйн – Геноха). Алергічні реакції якого типу (за Кумбсом і Джелом) лежать в основі цього захворювання?

A. I типу (анафілактичні)

B. II типу (гуморальні цитотоксичні )

C. III типу (імунокомплексні)

D. IV типу (клітинні цитотоксичні)

E. V типу (стимулюючі)


90. У дитини з геморагічним висипом, що виник після ГРВІ, діагностовано геморагічний васкуліт (хвороба Шенляйн – Геноха). Що спричинює порушення гемостазу при цьому захворюванні?

A. Спадковий дефект сполучної тканини судинної стінки

B. Пошкодження судинної стінки

C. Спадковий дефіцит антикоагулянтів

D. Пригнічення фібринолізу

E. Спадковий дефіцит прокоагулянтів


91. У жінки, що страждає на жовчно-кам’яну хворобу виявлено геморрагічний синдром, зумовлений дефіцитом вітаміну К. Який з перелічених факторів є неповноцінним при гіповітамінозі К?

A. Антигемофільний глобулін А (ф. VIII)

B. Фактор Віллебранда

C. Фібринстабілізуючий (ф. XIII)

D. Стюарта-Прауера (ф. X)

E. Фібриноген (ф. I)


92. У дитини з бронхіальною астмою спостерігається підвищена кровоточивість, сповільнення процесу згортання крові, що зумовлено вивільненням однієї з біологічно активних речовин з тканинних базофілів. Яка речовина призвела до патології згортання крові в даному випадку?

A. Гістамін

B. Серотонін

C. Лейкотриєн

D. Тромбоксан

E. Гепарин


93. У пацієнта при обстеженні було виявлено тромбофілію (прискорення процесу зсідання крові ). Що могло стати причиною порушення?

A. Підвищення концентрації простацикліну

B. Зниження концентрації тромбіну в крові

C. Підвищення концентрації гепарину в крові

D. Дефіцит інгібіторів протеолітичних ферментів

E. Підвищення концентрації фібринолізину в крові


94. У хворого із захворюванням печінки виявлено зниження вмісту протромбіну в крові. Це призведе до порушення перш за все:

A. Фібринолізу

B. Третьої фази коагуляційного гемостазу

C. Другої фази коагуляційного гемостазу

D. Судинно-тромбоцитарного гемостазу

E. Першої фази коагуляційного гемостазу


95. У хворого, що страждає на стрептококову інфекцію, розвинувся геморагічний синдром. Яка причина підвищеної кровоточивості при цьому?

A. Посилений фібриноліз

B. Зменшення кількості гепарину в плазмі крові

C. Нестача вітаміну А

D. Збільшення кількості калікреїну в плазмі крові

E. Нестача вітаміну С


96. У хворого на хронічний гепатит видалили зуб. Кровотечу, що виникла після цього, не вдавалося зупинити протягом 2-х годин. Проведене дослідження гемостазу встановило зменшення вмісту декількох факторів згортання крові. Який вид гемостазу порушений у цьому випадку?

A. Тромбоцитарний

B. Коагуляційний

C. Тромбоцитарно-судинний

D. Судинний

E. Змішаний


97. У хворого 45 років діагностована механічна жовтяниця, що супроводжується стеатореєю та геморагіями на шкірі. У крові - зменшення вмісту протромбіну та інших факторів зсідання крові. Яка причина порушення коагуляційного гемостазу в даного хворого?

A. Токсичне ушкодження гепатоцитів

B. Підвищена секреція в кров протеолітичних ферментів

C. Порушення перетравлення та всмоктування білка

D. Коагулопатія споживання

E. Порушення всмоктування в кишечнику вітаміну К


98. У хлопчика 7 років, після падіння з велосипеда виник гемартроз колінного суглоба. Введення кріопреципітату та відкачування крові з суглоба призвело до значного поліпшення стану дитини. Про яке захворювання слід думати?

A. Геморагічний васкуліт.

B. Тромбоцитопатія

C. Тромбоцитопенія

D. Ревматоїдний артрит

E. Гемофілія А


99. У хлопчика з вираженим геморагічним синдромом в плазмі крові відсутній антигемофильній глобулін А (фактор VIII). Яка фаза гемостазу первинно порушена у цієї дитини?

A. Ретракція кров’яного згустка

B. Перетворення фібриногену на фібрин

C. Перетворення протромбіну на тромбін

D. Зовнішній шлях активації протромбінази

E. Внутрішній шлях активації протромбінази


100. Хлопчик страждає на гемофілію. Якими клінічними ознаками проявляються порушення коагуляційного гемостазу?

A. Петехіальними крововиливами

B. Мікрогематурією

C. Екхімозами (синцями)

D. Порушенням гостроти зору

E. Гематомами, тривалими кровотечами


101. У постраждалого внаслідок ДТП через деякий час після перенесеної політравми розвинувся синдром дисемінованого внутрішньосудинного зсідання крові (ДВЗ). Який фактор був ініціатором цього ускладнення?

A. Фібриноген (ф. I)

B. Антигемофільний глобулін А (ф. VIII)

C. Фактор Стюарта-Прауера (ф. X)

D. Тканинний тромбопластин (ф. III)

E. Антигемофільний глобулін В (ф. IX)


102. У хворого на гострий панкреатит розвинувся синдром дисемінованого внутрішньосудинного зсідання крові (ДВЗ). Яка речовина була ініціатором цього ускладнення?

A. Фібриноген (ф. I)

B. Антигемофільний глобулін А (ф. VIII)

C. Фактор Стюарта-Прауера (ф. X)

D. Трипсин

E. Антигемофільний глобулін В (ф. IX)


103. У хворого на політравму з гострою нирковою недостатністю, стан ускладнився внутрішньою кровотечею. Що є головною ланкою патогенезу цієї стадії ДВЗ - синдрому?

A. Викид лейкоцитів з депо

B. Тромбоцитоз

C. Активація протромбінази

D. Пригнічення фібринолізу

E. Споживання факторів згортання крові


104. У хворого з опіковою хворобою як ускладнення розвинувся ДВЗ - синдром. Яку стадію ДВЗ - синдрому можна запідозрити, якщо відомо, що кров хворого згортається менше ніж за 3 хв.?

A. Гіпокоагуляції

B. Відновлення

C. Гіперкоагуляції

D. Латентну

E. Термінальну


105. У хворого з травмою розвинувся ДВЗ - синдром. Які зміни гемостазу спостерігаються в ІІ фазу ДВЗ -синдрому?

A. Гіперкоагуляція.

B. Гіпокоагуляція

C. Фібриноліз

D. Тромбоцитопенія

E. Тромбоцитопатія


106. У пацієнта після переливання несумісної крові виник ДВЗ – синдром. Що є головною ланкою в патогенезі цього ускладнення при гемолізі?

A. Надходження у кров внутрішньоклітинних протеаз

B. Накопичення білірубіну в крові

C. Надлишок у крові тромбопластину

D. Надлишок у крові протромбіну

E. Збільшення вмісту плазміногену


107. У вагітної жінки відбулося відшарування плаценти, розвилася перша стадія ДВЗ - синдрому, що характеризується прискоренням процесу згортання крові. Який фактор спричинив виникнення цього синдрому в даному випадку?

A. Нестача у крові тромбопластину

B. Надлишок у крові тромбопластину

C. Надлишок у крові протромбіну

D. Збільшення вмісту плазміногену

E. Підвищення вмісту іонів Са++


108. У хворого на тлі хронічної ниркової недостатності розвився ДВЗ-синдром. При обстеженні виявлено збільшення часу згортання крові, тромбоцитопенію, підвищення рівня фібрин-мономерних комплексів та продуктів деградації фібрину. Про яку стадію ДВЗ-синдрому слід думати?

A. Гіперкоагуляції

B. Гіпокоагуляції

C. Відновлення

D. Латентну

E. Нестабільну


109. У пацієнта перебіг токсичної гемолітичної анемії ускладнено розвитком гострої ниркової недостатності. Що є головною ланкою в патогенезі цього ускладнення при гемолізі?

A. ДВЗ - крові

B. Накопичення білірубіну в крові

C. Ретикулоцитоз

D. Еритроцитоз

E. Диспротеїнемія


110. У хворого на хронічний лімфолейкоз виникли геморагії внаслідок розвитку ДВЗ - синдрому. Які зміни показників периферичної крові будуть спостерігатися при цьому?

A. Еритроцитоз, підвищення в’язкості крові

B. Тромбоцитоз, зменшення часу згортання крові

C. Гіперкоагуляція, збільшення агрегації тромбоцитів

D. Гіпокоагуляція, тромбоцитопенія

E. Підвищена активність прокоагулянтів


Патофізіологія системи кровообігу 111. У жінки, 22 років, діагностовано міокардит, є ознаки серцевої недостатності. Який вид серцевої недостатності за патогенезом виник у хворої?

A. Від перевантаження серця підвищеним опором відтоку крові.

B. Від ушкодження міокарда.

C. Від зниження об’єму циркулюючої крові.

D. Від підвищення опору в судинах малого кола.

E. Від перевантаження міокарда збільшеним об’ємом крові.


112. У хворого наслідком гіпертонічного кризу стала гостра серцева недостатність. Який головний механізм виникнення серцевої недостатності у цьому випадку? А. Абсолютна коронарна недостатність. Б. Порушення ритму серця. С. Перевантаження серця об’ємом.

D. Перевантаження серця опором. Е. Ушкодження міокарда. 113. У хворого з недостатністю мітрального клапана виникла гостра серцева недостатність. Який патофізіологічний варіант недостатності серця спостерігається в цьому випадку?

A. Від перевантаження серця.

B. Внаслідок гіпоксичного ушкодження міокарда

C. Внаслідок коронарогенного ушкодження серця

D. Внаслідок нейрогенного ушкодження серця

E. Внаслідок гострої тампонади серця


114. У хворого 65 років діагностовано повну атріо-вентрикулярну блокаду, яка ускладнилась серцевою недостатністю. Який патогенетичний варіант серцевої недостатності у хворого?

A. Перевантаження серця опором.

B. Позаміокардіальна.

C. Перевантаження серця об’ємом.

D. Міокардіальна, аритмічна.

E. Міокардіальна, внаслідок ушкодження міокарда.


115. У хворого спостерігається різке підвищення артеріального тиску за рахунок зміни тонусу судин. Який компенсаторний механізм забезпечує збільшення сили скорочень міокарду у цьому випадку?

A. Пригнічення симпатичної нервової системи.

B. Гетерометричний.

C. Активація парасимпатичної нервової системи.

D. Активація ренін-ангіотензинової системи.

E. Гомеометричний.


116. У хворого після гостро перенесеного захворювання діагностовано недостатність мітрального клапану, але ознак серцевої недостатності не виявлено. Який негайний механізм компенсації забезпечує в такому випадку гіперфункцію серця?

A. Гіпертрофія лівого передсердя

B. Гіпертрофія лівого шлуночка

C. Гетерометричний механізм

D. Гомеометричний механізм

E. Тахікардія


117. У чоловіка, 25 років, виявлено недостатність мітрального клапана без порушення кровообігу. Який довготривалий механізм забезпечує компенсацію?

A. Міогенна дилатація.

B. Гомеометричний.

C. Механізм Боудича.

D. Гетерометричний.

E. Гіпертрофія міокарда.


118. Через місяць після виникнення експериментальної артеріальної гіпертензії в собаки товщина стінки лівого шлуночка серця збільшилась в 1,7 рази. Надалі маса серця не збільшувалась, хвилинний об’єм серця нормалізувався. Яка стадія гіпертрофії міокарда спостерігається у тварини?

A. Декомпенсації.

B. Прогресуючого кардіосклерозу.

C. Початкова.

D. Завершеної гіпертрофії.

E. Аварійна.


119. У тварини з недостатністю аортальних клапанів розвинулась гіпертрофія лівого шлуночка серця. В окремих його ділянках визначаються локальні контрактури. Накопичення якої речовини в кардіоміоцитах зумовило контрактури?

A. Молочної кислоти.

B. Натрію.

C. Кальцію.

D. Вуглекислого газу.

E. Калію.


120. У хворого з недостатністю мітрального клапана виникла гіпертрофія лівого шлуночка серця. Який механізм розвитку гіпертрофії є провідним?

A. Активація гліколізу.

B. Збільшення споживання жирних кислот кардіоміоцитами.

C. Активація генетичного аппарату кардіоміоцитів

D. Збільшення надходження іонів калію в клітину.

E. Збільшення інтенсивності клітинного дихання.


121. У хворого 25 років після перенесеного гострого ендокардиту виникла недостатність клапанів аорти, що викликала гіпертрофію лівого шлуночка серця. Через вісім тижнів маса серця збільшилась на 65 % і ріст її зупинився. Що викликало припинення подальшої гіпертрофії?

A. Погіршення пластичного забезпечення кардіоміоцитів

B. Порушення судинного забезпечення серця

C. Погіршення енергетичного забезпечення міокардіоцитів

D. Порушення регуляторного забезпечення серця

E. Нормалізація навантаження на одиницю м’язової маси


серця 122. Через 1 годину після накладання кільця, що звужує аорту, у собаки зросла сила та частота серцевих скорочень, а об’єм циркулюючої крові та товщина стінки лівого шлуночка не відрізнялися від вихідних показників. Яка стадія гіпертрофії міокарда спостерігається у тварини?

A. Завершеної гіпертрофії.

B. Відносно стійкої гіперфункції.

C. Декомпенсації.

D. Прогресуючого кардіосклерозу.

E. Аварійна.


123. У дитини 14 років з відкритою артеріальною протокою маса серця в 3 рази перевищує вікову норму. Стан дитини прогресивно погіршується: спостерігаються стійкий ціаноз, задишка, періодичні запаморочення. Яка особливість гіпертрофованого міокарда сприяє виникненню декомпенсації?

A. Зниження енергозабезпечення міокардіоцитів.

B. Активація гліколізу.

C. Активація генетичного апарату.

D. Активація кальцієвих насосів.

E. Збільшення кількості судин і нервів


124. У дитини 14 років з відкритою артеріальною протокою маса серця в 3 рази перевищує вікову норму. Стан дитини прогресивно погіршується: спостерігаються стійкий ціаноз, задишка, періодичні запаморочення. Яка стадія гіпертрофії міокарда спостерігається у дитини?

A. Завершеної гіпертрофії.

B. Відносно стійкої гіперфункції.

C. Компенсації.

D. Прогресуючого кардіосклерозу.

E. Аварійна.


125. У тварини з мітральним стенозом розвинулась гіпертрофія лівого передсердя. Який показник свідчить про перехід гіпертрофії у другу стадію?

A. Збільшення інтенсивності функціонування структур.

B. Нормалізація інтенсивності функціонування структур.

C. Зменшення інтенсивності функціонування структур.

D. Збільшення маси серця на 100%.

E. Нормалізація хвилинного об’єму серця.


126. У собаки з аортальним стенозом розвинулась гіпертрофія лівого шлуночка серця. Який показник свідчить про наявність першої стадії гіпертрофії?

A. Збільшення інтенсивності функціонування структур.

B. Нормалізація інтенсивності функціонування структур.

C. Зменшення інтенсивності функціонування структур.

D. Збільшення маси серця на 100%.

E. Нормалізація хвилинного об’єму серця.


127. У хворого розвинулася гостра лівошлуночкова недостатність, яка швидко ускладнилася набряком легень. Що є головною ланкою в патогенезі кардіогенного набряку легень?

A. Збільшення гідростатичного тиску крові

B. Збільшення онкотичного тиску тканинної рідини

C. Зменшення гідростатичного тиску крові

D. Зменшення онкотичного тиску крові

E. Зменшення онкотичного тиску тканинної рідини


128. У хворого на гіпертонічну хворобу під час фізичного навантаження з’явились відчуття нестачі повітря, ціаноз, дистанційні вологі хрипи. Що є найтяжчим клінічним проявом лівошлуночкової недостатності серця?

A. Асцит

B. Набряк легень

C. Стенокардія

D. Тахікардія

E. Набряк на ногах


129. У хворого на гіпертонічну хворобу під час фізичного навантаження з’явились відчуття м’язової слабкості, нестачі повітря. Який рефлекс запобігає розвитку набряку легень при лівошлуночкової недостатності серця?

A. Пульмо-коронарний

B. Пульмо-кардіальний

C. Китаєва

D. Крачмера

E. Бейнбріджа


130. У хворого на правошлуночкову серцеву недостатність виникли асцит та набряки. Який основний патогенетичний механізм розвитку набряків у цього хворого?

A. Збільшення проникності стінок судин

B. Збільшення онкотичного тиску крові

C. Збільшення гідростатичного тиску крові у венах

D. Збільшення онкотичного тиску міжклітинної рідини

E. Зменшення осмотичного тиску крові


131. У хворого на ревматизм розвинувся міокардит з проявами недостатності кровообігу. Яке порушення гемодінаміки спостерігається при цьому?

A. Зниження систолічного артеріального тиску.

B. Зниження венозного тиску.

C. Підвищення систолічного артеріального тиску.

D. Зростання швидкості кровообігу.

E. Зниження діастолічного артеріального тиску.


132. Хворий, 39 років, з хронічною серцевою недостатністю скаржиться на задишку при невеликому фізичному навантаженні, набряки на ногах. Що може бути свідченням декомпенсації функції серця у хворого?

A. Тахікардія

B. Збільшення швидкості кровотоку

C. Анемія

D. Підвищення артеріального тиску

E. Зменшення венозного тиску


133. Хвора 45 років скаржиться на задишку при невеликому фізичному навантажені, набряки на ногах. Діагностовано недостатність кровообігу. Який гемодинамічний показник свідчить про декомпенсацію серця?

A. Зменшення хвилинного об’єму серця

B. Зменшення об’єму циркулюючої крові

C. Зниження венозного тиску

D. Підвищення артеріального тиску

E. Задишка


134. У хворого з міокардіодистрофією розширені порожнини серця, знижена сила серцевих скорочень, збільшилися кінцевий систолічний об’єм шлуночків та кровонаповнення вен. Про який функціональний стан серця свідчать ці порушення?

A. Гіпертрофія серця, аварійна стадія

B. Гіпертрофія серця, стадія завершеної гіпертрофії

C. Гіпертрофія серця, стадія гіперфункції

D. Міогенна дилатація

E. Тоногенна дилатація


135. Хворий 65 років скаржиться на загальну слабість, серцебиття й задишку при помірному фізичному навантаженні, набряки та ціаноз нижніх кінцівок. Діагностовано серцеву недостатність. Чим зумовлений ціаноз у хворого?

A. Збільшенням напруги кисню в артеріальній крові

B. Зменшенням напруги вуглекислого газу в артеріальній крові

C. Зменшенням напруги кисню в артеріальній крові

D. Зменшенням кількості гемоглобіну в крові

E. Збільшенням кількості відновленого гемоглобіну в


крові 136. У підлітка після перенесеного інфекційного захворювання спостерігається коливання частоти серцевих скорочень залежно від фази дихання. Який механізм виникнення дихальної аритмії у хворого?

A. Порушення збудливості серця

B. Рефлекс Бейнбріджа

C. Порушення скоротливої функції серця

D. Порушення провідності серця

E. Коливання тонусу блукаючого нерва


137. Дані ЕКГ пацієнта з гіпертонічною хворобою: ритм синусовий, правильний, частота серцевих скорочень - 92 за 1 хв., тривалість інтервалу Р-Q 0,19 с., комплекс QRS не змінений. Яка функція серця порушена?

A. Автоматизм

B. Провідність

C. Збудливість

D. Скоротливість

E. Збудливість і провідність


138. Після тяжкої інфекції у хворого розвинувся міокардит з ушкодженням провідної системи серця і виникненням синдрому Морганьї – Адамса - Стокса. Яке порушення ритму у пацієнта?

A. Перехід неповної атріовентрикулярної блокади в повну.

B. Блокада правої ніжки пучка Гіса.

C. Пароксизмальна тахікардія.

D. Блокада лівої ніжки пучка Гіса.

E. Передчасне збудження шлуночків.


139. У хворого виявлено екстрасистолію. Дані ЕКГ: відсутній зубець Р, комплекс QRS деформований і розширений, наявна повна компенсаторна пауза. Вкажіть локалізацію ектопічного вогнища збудження в цьому випадку?

A. Передсердно-шлуночковий вузол.

B. Шлуночки.

C. Пучок Гіса.

D. Передсердя.

E. Синусовий вузол.


140. У хворого на дифузний токсичний зоб часто виникають серцебиття, слабкість, дефіцит пульсу. На ЕКГ виявлено відсутність зубців Р, замість них численні хвилі f, неоднакові інтервали R-R. Яке порушення серцевого ритму у хворого?

A. Миготіння передсердь

B. Пароксизмальна тахікардія

C. Синусова екстрасистолія

D. Атріовентрикулярна блокада

E. Фібриляція шлуночків


141. Нормальний ритм серця у хворої раптово перервався нападом скорочень частотою 200 уд/хв. тривалістю до 5 хв. Назвіть можливий наслідок тривалої пароксизмальної тахікардії.

A. Зменшення серцевого викиду

B. Збільшення серцевого викиду

C. Збільшення ударного об’єму серця

D. Підвищення систолічного тиску

E. Збільшення коронарного кровотоку


142. На ЕКГ лікар виявив у окремих серцевих циклах скорочення інтервалу Т-Р, двофазний зубець Р, комплекс QRS без порушень, неповна компенсаторна пауза. Для якої аритмії характерні такі зміни на ЕКГ?

A. Синусової аритмії

B. Передсердно-шлуночкової екстрасистолії

C. Шлуночкової екстрасистолії

D. Пароксизмальної тахікардії

E. Передсердної екстрасистолії


143. У хворого, що страждає на важку форму порушення водно-сольового обміну, настала зупинка серця в діастолі. Що є найбільш імовірною причиною зупинки серця в діастолі?

A. Гіперкаліємія

B. Гіперкальціємія

C. Дегідратація організму

D. Гіпомагніємія

E. Гіпонатріємія


144. На ЕКГ у хворої на міокардит виявлено подовження PQ інтервалу до 0,4 сек. Яка властивість провідної системи порушена?

A. Рефрактерність

B. Автоматизм

C. Скоротність

D. Збудливість

E. Провідність


145. Під час операційного втручання трапилося рефлекторне посилення впливу блукаючого нерва на серце. Що при цьому може виникнути?

A. Зупинка серця

B. Збільшення провідності атріовентрикулярного вузла

C. Збільшення збудливості міокарда

D. Посилення скорочень міокарда

E. Збільшення частоти серцевих скорочень


146. Чоловік, 57 років, скаржиться на біль у ділянці серця, який виник після негативних емоцій. Нітрогліцерин достатньо швидко усунув біль. Який механізм ішемії найбільш імовірний?

A. Ангіоспастичний.

B. Механічний.

C. Обтураційний.

D. Странгуляційний

E. Компресійний.


147. У хворого, що скаржиться на стенокардію у спокої, рівень холестерину у крові становить 8 ммоль/л. Що, найімовірніше, є причиною вінцевої недостатності у хворого?

A. Атеросклероз вінцевих артерій

B. Фізичне навантаження

C. Стрес

D. Артеріосклероз Менкеберга

E. Міокардит


148. При ішемії у міокардіоцитах виникає контрактура міофібрил, активація фосфоліпази А 2 та роз’єднання окиснення з фосфоруванням. Який механізм пошкодження клітин домінує у цьому випадку?

A. Ліпідний

B. Кальцієвий

C. Ацидотичний

D. Електолітно-осмотичний

E. Протеїновий


149. Емоційний стрес у пацієнта похилого віку був причиною виникнення гострого болю за грудиною. Лікар швидкої медичної допомоги встановив ішемічну хворобу серця. Що є причиною розвитку стенокардії у хворого?

A. Інфаркт міокарда

B. Хронічна дихальна недостатність

C. Хронічна серцева недостатність

D. Гостра вінцева недостатність

E. Асфіксія


150. У експерименті на коронарну артерію наклали лігатуру, а через 60 хв. зняли, що призвело до збільшення зони некрозу порівняно з очікуваною. Що лежить в основі даного явища?

A. Реперфузійний синдром

B. Резорбційно-некротичний синдром

C. Больовий синдром

D. Синдром Дресслера

E. Астенічний синдром


151. У хворого, який раптово помер, патологоанатомічно виявили обтуруючий тромб у лівій коронарній артерії, але не виявили некрозу. Що, найімовірніше, стало причиною ранньої серцевої смерті у хворого?

A. Реперфузійний синдром

B. Фібриляція шлуночків

C. Пароксизмальна тахікардія

D. Больовий шок

E. Синдром Дресслера


152. Відновлення коронарного кровообігу у хворого на інфаркт міокарда супроводжувалося зниженням скорочувальної здатності серця і погіршенням загального стану пацієнта. Які реакції визначають збільшення пошкодження кардіоміоцитів в умовах реперфузії?

A. Глікозилювання

B. Фосфорилювання

C. Вільнорадикальні

D. Трансамінування

E. Дефосфорилювання


153. Через 3 тижні після ангіни у хворого виник осередковий некроз міокарда. У крові виявили міокардіотоксичні антитіла. Алергічні реакції якого типу лежать в основі виникнення некрозу серця у даному випадку?

A. I типу (анафілактичні)

B. II типу (гуморальні цитотоксичні )

C. V типу (стимулюючі)

D. III типу (імунокомплексні)

E. IV типу (клітинні цитотоксичні)


154. У хворого з інфарктом міокарда артеріальний тиск різко знизився до 70/40 мм рт. ст. Який первинний механізм розвитку артеріальної гіпотензії?

A. Гіповолемія.

B. Втрата організмом натрію.

C. Затримка калію в організмі.

D. Зменшення хвилинного об’єму серця.

E. Вазоконстрикція периферичних судин.


155. У хворого з інфарктом міокарда розвинулася серцева недостатність. Який патогенетичний механізм її розвитку?

A. Реперфузійне ураження міокарда

B. Гостра тампонада серця

C. Перевантаження серця об'ємом

D. Перевантаження серця опором

E. Зменшення маси функціонуючих кардіоміоцитів


156. Після перев’язки однієї з гілок коронарних артерій у собаки розвинувся інфаркт міокарда, який супроводжувався проявами резорбційнонекротичного синдрому. Яка характерна ознака розвитку цього синдрому?

A. Біль за грудиною.

B. Зниження хвилинного об’єму крові.

C. Підвищення рівня катехоламінів у крові.

D. Фібриляція шлуночків.

E. Збільшення вмісту креатинфосфокінази в крові.


157. Хворий з інфарктом міокарда скаржиться на гарячку, лабораторно виявлені нейтрофільний лейкоцитоз, підвищення ШОЕ. Яка речовина викликала ці зміни?

A. Гістамін

B. Інтерферон

C. Інтерлейкін 1

D. Калідин

E. Лізоцим


158. У пацієнта, у якого 1,5 міс тому виник інфаркт міокарда, діагностовано синдром Дресслера з характерною тріадою: перикардитом, плевритом, пневмонією. Який головний механізм цього ускладнення?

A. Зниження резистентності організму до інфекції.

B. Викид у кров міокардіальних ферментів.

C. Автоімунне ураження.

D. Інтоксикація організму продуктами некрозу.

E. Активація сапрофітної мікрофлори.


159. У тварини, якій у порожнину перикарда вводили фізіологічний розчин виникла гостра тампонада серця. Який процес первинно порушується при цьому виді недостатності серця?

A. Скорочення міокардіоцитів

B. Автоматизм серця

C. Проведення імпульсів

D. Діастолічне розширення серця

E. Збудження міокардіоцитів


160. У тварини, якій у порожнину перикарда вводили фізіологічний розчин виникла гостра тампонада серця. Який компенсаторний механізм спрацьовує при цьому виді недостатності серця?

A. Гомеометричний

B. Гетерометричний

C. Гіпертрофія міокарда

D. Інотропна дія катехоламінів

E. Тахікардія


161. Хворий 67 років страждає на атеросклероз судин головного мозку. При обстеженні виявлено гіперліпопротеїнемію IIб типу. Який клас ліпопротеїнів плазми крові значно підвищений у хворого?

A. Ліпопротеїни низької щільності

B. Ліпопротеїни високої щільності

C. Альфа-ліпопротеїни

D. Хіломікрони

E. Комплекси жирних кислот з альбумінами


162. Після тривалого введення великих доз ергокальциферолу у тварини виникли склерозування та кальциноз середньої оболонки артерій. Який патологічний процес моделювали в експерименті?

A. Атеросклероз Маршана

B. Артеріолосклероз

C. Артеріосклероз Менкеберга

D. Вікові зміни артерій

E. Ендартеріїт


163. У дитини 10 років з сухожильними ксантомами та тяжкими нападами стенокардії виявлено значне збільшення вмісту ліпопротеїнів низької щільності у плазмі крові. Який дефект лежить в основі виникнення первинної гіперліпопротеїнемії у хворого?

A. Дефіцит ліпопротеїнліпази

B. Дефект рецептора апоВ/Е

C. Підвищена продукція апоВ

D. Аномальний апоЕ

E. Глікерування апоВ/Е


164. У хворого при ангіографії виявили значне атеросклеротичне ураження ниркових артерій. До яких наслідків призводить ця патологія?

A. Ішемічна хвороба серця

B. Артеріальна гіпертензія

C. Печінкова недостатність

D. Геморагічний інсульт

E. Дихальна недостатність


165. У хворого на цукровий діабет виявлено інтенсивний прогресуючий розвиток атеросклерозу, що проявлялося розвитком ішемічної хвороби серця, стенозуванням ниркових артерій, порушеннями мозкового кровообігу. Що є найбільш імовірною причиною розвитку атеросклерозу ?

A. Надмірне вживання холестерину з їжею

B. Ушкодження судин токсичними речовинами

C. Глікерування апоВ/Е

D. Дефекти ліпопротеїдліпази

E. Спадкові дефекти рецепторів апоВ/Е


166. У тварини змоделювали артеріосклероз Менкеберга. Що є його основними проявами?

A. Кальциноз та склероз медії

B. Кальциноз та склероз інтими

C. Ліпідоз інтими

D. Ліпідоз медії

E. Інфільтрація інтими


167. Хворий 67 років страждає на атеросклероз судин головного мозку. При обстеженні виявлено гіперхолестеринемію. Який клас ліпопротеїнів плазми крові найбільш імовірно значно знижений у хворого?

A. Ліпопротеїни низької щільності

B. Ліпопротеїни високої щільності

C. Ліпопротеїни дуже низької щільності

D. Хіломікрони

E. Комплекси жирних кислот з альбумінами


168. У кроля, якого тримали на раціоні з великим вмістом тригліцеридів, виникло атеросклеротичне ураження артерій. Що було головною причиною розвитку атеросклерозу в даному випадку?

A. Ендогенна гіперхолестеринемія

B. Екзогенна гіперхолестеринемія

C. Первинне пошкодження судинної стінки

D. Генетичний дефект рецепторів апоВ/Е

E. Генетичний дефект ліпопротеїнліпази


169. У хворого на цукровий діабет виявлено інтенсивний прогресуючий розвиток атеросклерозу, що проявлялося розвитком ішемічної хвороби серця, стенозуванням ниркових артерій, порушеннями мозкового кровообігу. Що є найбільш імовірною причиною розвитку атеросклерозу?

A. Надмірне вживання холестерину з їжею

B. Ушкодження судин токсичними речовинами

C. Поява модифікованих ліпопротеїнів

D. Дефекти ліпопротеїдліпази

E. Спадкові дефекти рецепторів апоВ/Е


170. У хворого при ангіографії виявили значне атеросклеротичне ураження внутрішніх сонних артерій. До яких наслідків може призвести ця патологія?

A. Ішемічна хвороба серця

B. Артеріальна гіпотензія

C. Хронічна ниркова недостатність

D. Інфаркт міокарда

E. Ішемічний інсульт


171. У хворого, 43 років, артеріальна гіпертензія є наслідком помірного збільшення серцевого викиду і загального периферичного опору. Який гемодинамічний варіант артеріальної гіпертензії має місце в цьому випадку?

A. Дискінетичний.

B. Гіпокінетичний.

C. Змішаний.

D. Гіперкінетичний.

E. Еукінетичний.


172. Дівчина астенічної конституції скаржиться на загальну слабкість, швидку втомлюваність, зменшення працездатності. Об’єктивно: пульс - 98 за 1 хв., АТ - 90/60 мм. рт. ст. Який механізм зумовив гіпотензію у хворої?

A. Депонування крові в портальній системі

B. Зниження периферійного судинного опору

C. Збільшення серцевого викиду

D. Збільшення хвилинного об’єму крові

E. Активація симпатичної нервової системи


173. У хворого виявлено артеріальну гіпертензію з гіпокінетичним типом кровообігу. Який фактор найбільш істотно впливає на артеріальний тиск?

A. Ударний об’єм серця

B. Частота серцевих скорочень

C. Периферичний судинний опір

D. Венозне повернення до серця

E. Хвилинний об’єм серця


174. У хворого з гіпертонічною хворобою різко підвищився артеріальний тиск до 180/105 мм рт. ст. Яке ускладнення, найбільш вірогідно, може виникнути у хворого в цьому випадку?

A. Гостра ниркова недостатність

B. Хронічна ниркова недостатність

C. Гостра серцева недостатність

D. Хронічна серцева недостатність

E. Хронічна дихальна недостатність


175. При критичному зниженні температури тіла в третьому періоді лихоманки на фоні тахікардії АТ = 80 / 60 мм рт. ст. До якої форми порушення судинного тонусу належить це явище?

A. Хронічної гіпертензії

B. Гострої гіпертензії

C. Гострої гіпотензії

D. Хронічної гіпотензії

E. Гіпотонічної хвороби


176. У хворого з гіпертонічною хворобою різко підвищився артеріальний тиск до 210/150 мм рт. ст. Яке ускладнення, найбільш вірогідно, може виникнути у хворого в цьому випадку?

A. Гостра ниркова недостатність

B. Хронічна серцева недостатність

C. Геморагічний інсульт

D. Гіпотрофія лівого шлуночка серця

E. Гіпотрофія міокарда


177. У чоловіка, що тривалий час страждає на артеріальну гіпертензію, останнім часом систолічний тиск почав знижуватися, а діастолічний залишається підвищеним. Який гемодинамічний тип артеріальної гіпертензії у хворого?

A. Дискінетичний.

B. Еукінетичний.

C. Гіперкінетичний.

D. Гіпокінетичний.

E. Нормокінетичний.


178. Хворий страждає на гіпертонічну хворобу 5 років. Яке ускладнення, найбільш вірогідно, може виникнути в цьому випадку?

A. Гіпертрофія правого шлуночка серця

B. Гіпертрофія лівого шлуночка серця

C. Гіпотрофія лівого шлуночка серця

D. Гостра ниркова недостатність

E. Жовтяниця


179. У чоловіка, що 15 років хворіє на гіпертонічну хворобу, виявлено гіпокінетичний тип кровообігу. Що є причиною стійкого збільшення судинного опору при цьому?

A. Атеросклероз

B. Артеріосклероз Менкеберга

C. Артеріолосклероз

D. Розширення судин

E. Підвищена в’язкість крові


180. У хворого, 18 років, артеріальна гіпертензія є наслідком значного збільшення серцевого викиду при нормальному периферичному опорі судин. Який гемодинамічний варіант артеріальної гіпертензії має місце в цьому випадку?

A. Дискінетичний.

B. Гіпокінетичний.

C. Змішаний.

D. Гіперкінетичний.

E. Еукінетичний.


181. У хворого на стійку артеріальну гіпертензію значно погіршився стан під час лікування артриту індометацином. Що призвело до посилення артеріальної гіпертензії?

A. Гіперпродукція реніну.

B. Гіпопродукція кінінів.

C. Гіперпродукція кортизолу.

D. Гіпопродукція простагландинів.

E. Гіперпродукція альдостерону.


182. У тварини, якій видалили обидві нирки, на фоні регулярного гемодіалізу виникла стійка артеріальна гіпертензія. Що призвело до підвищення артеріального тиску?

A. Гіперпродукція реніну.

B. Гіперпродукція кінінів і простагландинів.

C. Гіперпродукція кортизолу.

D. Гіпопродукція простагландинів і кінінів.

E. Гіперпродукція альдостерону.


183. Артеріальна гіпертензія у хворого зумовлена вираженим атеросклерозом ниркових артерій. Гіперактивність якої системи є головною ланкою в патогенезі цієї форми гіпертензії?

A. Ренін-ангіотензинової

B. Симпато-адреналової

C. Калікреїн-кінінової

D. Парасимпатичної

E. Гіпоталамо-гіпофізарної


184. У хворого на феохромоцитому (пухлину мозкової речовини наднирників) розвинулася артеріальна гіпертензія. Що стало причиною підвищення артеріального тиску?

A. Гіпопродукція простагландинів

B. Вазодилатація

C. Гіпернатріємія

D. Збільшення хвилинного об’єму серця

E. Гіповолемія


185. У хворого на мікседему розвинулася артеріальна гіпотензія. Що стало причиною зниження артеріального тиску?

A. Спазм судин

B. Тахікардія

C. Гіпонатріємія

D. Зниження хвилинного об’єму серця

E. Гіперволемія


186. У жінки 25 років, що скаржиться на болі в шиї та голові, виявлені при обстеженні остеохондроз шийного відділу хребта та нестійка артеріальна гіпертензія. Який різновид симптоматичної гіпертензії у хворої?

A. Центрально-ішемічна.

B. Невротична.

C. Рефлексогенна.

D. Реноваскулярна.

E. Ренопривна.


187. У хворого на синдром Конна (первинний гіперальдостеронізм) розвинулася артеріальна гіпертензія. Що стало причиною підвищення артеріального тиску?

A. Гіперкаліємія

B. Гіперкальціємія

C. Гіпернатріємія

D. Ацидоз

E. Гіповолемія


188. У хворого на синдром Іценка - Кушинга розвинулася артеріальна гіпертензія. Що є механізмом підвищення артеріального тиску?

A. Гіповолемія

B. Гіперволемія

C. Зменшення опору судин

D. Депонування крові у портальній системі

E. Гіпогідрія


189. У чоловіка з хворобою Аддісона травма викликала значне погіршення стану. Які зміни в системі кровообігу характерні для таких хворих?

A. Гіпотензія

B. Гіпертензія

C. Гіперволемія

D. Гіперглікемія

E. Гіпертрофія міокарда


190. У хворого на Базедову хворобу розвинулася артеріальна гіпертензія. Що стало причиною підвищення артеріального тиску?

A. Збільшення хвилинного об’єму серця

B. Зменшення опору судин

C. Гіповолемія

D. Активація калікреїн-кінінової системи

E. Системна дегрануляція тканинних базофілів


Патофізіологія системи зовнішнього дихання 191. У недоношеної дитини при народженні діагностували рестриктивну дихальну недостатність. Яка причина лежить в основі недостатності дихання в цьому випадку?

A. Набряк легенів

B. Внутрішньоутробна гіперкапнія

C. Потрапляння навколоплідних вод

D. Токсоплазмоз

E. Дефіцит сурфактанта


192. При обстеженні у хворого виявили обструктивну форму дихальної недостатності. Яка причина могла б її викликати?

A. Запалення легенів

B. Порушення цілісності плевральної порожнини

C. Зниження утворення сурфактанта

D. Запалення бронхів

E. Пневмосклероз


193. У непритомного юнака, що отруївся морфіном, спостерігається рідке поверхневе дихання. Яка дихальна недостатність виникла при цьому?

A. Вентиляційна рестриктивна

B. Вентиляційна обструктивна

C. Вентиляційна дисрегуляторна

D. Перфузійна

E. Дифузійна


194. У хворого з поліомієлітом діагностовано значні патологічні зміни альвеолярної вентиляції. Порушення функції якої ланки системи зовнішнього дихання спостерігається в цьому випадку?

A. Дихального центру

B. Мотонейронів спинного мозку

C. Нервово-м’язового синапсу

D. Дихальних м’язів

E. Легеневої паренхіми


195. Хворий з мітральним стенозом скаржиться на важку задишку, ціаноз, кашель. Яка дихальна недостатність спостерігається при цьому?

A. Вентиляційна рестриктивна

B. Вентиляційна обструктивна

C. Вентиляційна дисрегуляторна

D. Перфузійна

E. Дифузійна


196. При обстеженні хворого виявили перфузійну форму дихальної недостатності. Яка причина могла її викликати?

A. Емболія легеневої артерії

B. Порушення цілісності плевральної порожнини

C. Зниження утворення сурфактанту

D. Запалення бронхів

E. Емфізема


197. У шахтаря виявлений фіброз легенів, який супроводжується порушенням альвеолярної вентиляції. Який провідний механізм виникнення вентиляційної недостатності в цьому випадку?

A. Зменшення розтяжності легенів

B. Порушення нервової регуляції дихання

C. Обмеження рухливості грудної клітки

D. Спазм бронхів

E. Звуження верхніх дихальних шляхів


198. У підлітка, 12 років, виник важкий напад бронхіальної астми: виражена експіраторна задишка, блідість шкіри. Який вид порушення альвеолярної вентиляції має місце?

A. Обструктивний

B. Центральний

C. Нервово-м’язовий

D. Торакодіафрагмальний

E. Рестриктивний


199. У хворого на цукровий діабет виникла діабетична кома, яка супроводжувалася глибоким гучним диханням. Як називається такий тип дихання?

A. Поліпное

B. Агональне дихання

C. Гіперпное

D. Періодичне дихання

E. Брадипное


200. У хворого, 62 років, після мозкового крововиливу спостерігається наростання глибини і частоти дихання, а потім зменшення до повного припинення, після чого цикл дихальних рухів відновлюється. Який тип дихання виник у хворого?

A. Апнейстичне

B. Куссмауля

C. Гаспінг-дихання

D. Чейна-Стокса

E. Біотта


201. У результаті виробничої травми у хворого виявлено перелом декількох ребер. Який характер дихання за звичай спостерігається в таких випадках?

A. Гіперпное

B. Тахіпное

C. Еупное

D. Брадипное

E. Апное


202. Анафілактичний шок у морської свинки зумовив зміну характеру дихання. Який тип дихання спостерігається в цьому випадку?

A. Експіраторна задишка

B. Інспіраторна задишка

C. Стенотичне

D. Кусмауля

E. Брадипное


203. Хворий з черепно-мозковою травмою госпіталізований у важкому стані. Його дихання характеризується судомним подовженим вдихом, який переривається рідкими короткими видихами. Для якого типу дихання це характерно?

A. Чейна-Стокса

B. Гаспінг-дихання

C. Біотта

D. Куссмауля

E. Апнейстичного


204. У хворого з двостороннім запаленням легенів з’явилась задишка. Назвіть найбільш суттєву ознаку для визначення диспное.

A. Збільшення глибини дихання

B. Зміна частоти дихання

C. Зміна ритму дихання

D. Збільшення хвилинного об’єму дихання

E. Відчуття нестачі повітря


205. У тварин було перерізано обидва блукаючих нерва. Яким стане дихання в цьому випадку?

A. Періодичним

B. Рідким та поверхневим

C. Частим та поверхневим

D. Частим та глибоким

E. Рідким та глибоким


206. При обстеженні у хворого діагностували обструкцію нижніх дихальних шляхів. Яке порушення дихання виникло?

A. Інспіраторна задишка

B. Експіраторна задишка

C. Тахіпное

D. Апнейстичне

E. Гаспінг


207. Хворий був госпіталізований в стані діабетичної коми. Дихання шумне, часте, після глибокого вдиху йде посилений видих. Який тип дихання спостерігається у цього хворого?

A. Біотта

B. Апнейстичне

C. Чейна-Стокса

D. Куссмауля

E. Гаспінг


208. У хворого в стані агонії після короткочасної зупинки дихання почали реєструватися поодинокі зітхання, сила і частота яких поступово зменшувалися. Якому типу дихання відповідають подібні прояви?

A. Апнейстичному

B. Чейна-Стокса

C. Біотта

D. Гаспінг-диханню

E. Куссмауля


209. У хворого на дифтерію виник набряк гортані. Назвіть характерний тип дихання при цьому.

A. Часте та глибоке

B. Часте та поверхневе

C. Рідке та глибоке (стенотичне)

D. Рідке та поверхневе

E. Біотта


210. Під час операції неадекватно висока вентиляція легенів ускладнилась розвитком судом у пацієнта. Який розлад кислотно-основного стану може бути причиною їх розвитку?

A. Ацидоз газовий

B. Ацидоз метаболічний

C. Ацидоз видільний

D. Алкалоз газовий

E. Алкалоз метаболічний


211. У хворого 76 років виявлено значне збільшення залишкового об’єму легенів. Яке захворювання може викликати ці зміни?

A. Пневмонія

B. Плеврит

C. Бронхіт

D. Емфізема

E. Туберкульоз


212. Під час аускультації хворого попросили глибоко дихати. Після 5 хвилин інтенсивних дихальних рухів хворий відчув запаморочення. Це викликано:

A. Гіпероксією

B. Розширенням судин мозку

C. Перерозподілом крові в нижні кінцівки

D. Гіпокапнією

E. Гіпоксією


213. У травмпункт доставили потерпілого з травматичним ураженням грудної клітки. На рентгенограмі виявлено газ у плевральній порожнині. Назвіть характерну для пневмотораксу ознаку.

A. Періодичне дихання

B. Зниження транспульмонального тиску

C. Пригнічення дихального центру

D. Порушення прохідності трахеї

E. Підвищення транспульмонального тиску


214. У хворого внаслідок патологічного процесу збільшилась товщина альвеолокапілярної мембрани. Безпосереднім наслідком цього буде зменшення:

A. Альвеолярної вентиляції

B. Дифузійної здатності легенів

C. Кисневої ємності крові

D. Резервного об’єму видиху

E. Хвилинного об’єму дихання


215. У хворого на фіброзно-кавернозний туберкульоз легенів різко посилились кашель з виділенням гнійно-слизової мокроти з домішками крові та слабкість. Яка причина вентиляційної недостатності, що виникла у хворого?

A. Зменшення кількості функціонуючих альвеол

B. Порушення функції дихального центру

C. Порушення функції нервово-м’язового синапсу

D. Порушення рухомості грудної клітки

E. Порушення прохідності дихальних шляхів


216. Хворий на хронічний бронхіт скаржиться на задишку під час фізичного навантаження, постійний кашель з виділенням мокроти. При обстеженні діагностовано ускладнення – емфізема легенів. Вона зумовлена зменшенням:

A. Еластичних властивостей легенів

B. Розтяжності легенів

C. Залишкового об’єму легенів

D. Дифузійної здатності легенів

E. Перфузії легенів


217. У хворого на туберкульоз легенів спостерігається значний дефіцит сурфактанту. Які патологічні зміни в легеневій тканині можуть виникнути в цьому випадку?

A. Бронхоспазм

B. Емфізема

C. Набряк

D. Лімфостаз

E. Ателектаз


218. У хворого внаслідок хронічного захворювання органів дихання на тлі задишки, тахікардії та ціанозу під час дослідження газового складу крові виявлено розвиток гіпоксемії та гіперкапнії. Яке порушення зовнішнього дихання спостерігається у цього хворого?

A. Гіпервентиляція

B. Гіповентиляція

C. Гіпердифузія

D. Гіперперфузія

E. Гіпоперфузія


219. У хворого в стані спокою значно збільшена робота дихальних м’язів. Що з нижче перерахованого може бути причиною цього явища?

A. Пригнічення дихального центру

B. Негативний плевральний тиск

C. Обструкція дихальних шляхів

D. Гіпокапнія

E. Алкалоз


220. У робітника цементного заводу, що хворіє на пневмоконіоз, спостерігається задишка, акроціаноз. Що є причиної цього?

A. Обструкція великих бронхів

B. Еритроцитоз

C. Зниження дифузійної здатності легенів

D. Токсична дія SiO 2 на дихальний центр

E. Параліч діафрагми


Патофізіологія системи травлення 221. Після отруєння фосфорорганічними речовинами у хворого виникло тривале підвищення слиновиділення. До якого порушення в організмі може призвести гіперсалівація?

A. Нейтралізації шлункового соку

B. Підсилення травлення у шлунку

C. Гіпоосмолярної дегідратації

D. Гіпоосмолярної гіпергідратації

E. Пригнічення пристінкового травлення


222. У чоловіка, якому 5 років тому видалили шлунок, стоматолог виявив атрофічно-запальні процеси слизової ротової порожнини: яскраво-червоний, гладенький («лакований») язик, гінгівіт. Чим зумовлені ці зміни?

A. Психоемоційним перенапруженням.

B. Дефіцитом вітаміну В 6 .

C. Дефіцитом вітаміну С.

D. Дефіцитом вітаміну В 12 .

E. Гіперсалівацією.


223. У тварини в експерименті індукували розвиток карієсу. Який найважливіший компенсаторний механізм включається при розвитку цього захворювання?

A. Пригнічення фагоцитозу

B. Гіпотрофія слинних залоз

C. Гіперфункція паращитоподібних залоз

D. Новоутворення емалі

E. Утворення вторинного дентину


224. Хворий звернувся до лікаря зі скаргами на гнійні виділення з ясенних кишень і розхитування зубів. Що є причиною гнійного пародонтиту у пацієнта?

A. Вірусна інфекція

B. Гиперсалівація

C. Місцева анафілаксія

D. Бактеріальна інфекція

E. Генералізована анафілаксія


225. До гастроентерологічного відділення потрапив хворий 57 років з підозрою на синдром Золлінгера - Еллісона, про що свідчило різке збільшення рівня гастрину у сироватці крові. Яке порушення секреторної функції шлунка найбільш імовірне в цього хворого?

A. Ахілія

B. Гіпосекреція гіперацидна

C. Гіперсекреція гіперацидна

D. Гіпосекреція гіпоацидна

E. Гіперсекреція гіпоацидна


226. У хворого, який отримав тривалий курс лікування глюкокортикоїдами, виявлені виразки шлунка. Який механізм є головним у їх розвитку?

A. Зменшення секреції шлункового соку

B. Зменшення секреції гастрину

C. Зменшення регенерації епітелію шлунка

D. Збільшення секреції простагландинів Е1, Е2

E. Підвищення тонусу парасимпатичної нервової системи


227. Хворий скаржиться на нудоту, печію, біль у епігастральній області, закріп, схуднення. При обстеженні загальна НСl - 96 ммоль/л, вільна - 60 ммоль/л. Який стан кислотоутворюючої функції у даного хворого?

A. Ахілія

B. Гіпоацидний

C. Анацидний

D. Гіперацидний

E. Нормацидний


228. Для моделювання виразки шлунку тварині ввели в шлункові артерії атофан, який викликає їх склерозування. Який механізм пошкодження слизової оболонки шлунка є провідним у даному випадку?

A. Механічний

B. Нейродистрофічний

C. Гіпоксичний

D. Дисрегуляторний

E. Нейрогуморальний


229. У пацієнта 35 років, хворого на невроз, виявили виразкову хворобу. Який чинник, найімовірніше, міг стати причинним у виникненні цього захворювання?

A. Зменшення концентрації адреналіну в крові

B. Порушення кірково-підкіркових взаємовідносин

C. Зниження продукції глюкокортикоїдів

D. Порушення режиму харчування

E. Зниження впливу парасимпатичної нервової системи


230. У хворого з синдромом Zollinger-Ellison (пухлина з G-клітин з локалізацією в підшлунковій залозі) відзначається збільшення секреції HCl та пептичні виразки. Яка з перерахованих речовин викликає цей комплекс симптомів?

A. Вазоактивний інтестинальний пептид

B. Гастрин

C. Пепсин

D. Трипсин

E. Секретин


231. У хворої В., яка довгий час приймала саліцилати для лікування ревматизму, під час фіброгастроскопії знайдена дуоденальна виразка. Який провідний механізм розвитку виразкової хвороби в даному випадку?

A. Зниження синтезу простагландинів

B. Зниження продукції муцину

C. Зниження секреції пепсину

D. Зниження секреції соляної кислоти

E. Зниження моторики шлунка


232. У жінки 52 років з хворобою Іценка - Кушинга підвищена кислотність шлункового соку. Який механізм забезпечив підвищення секреції в шлунку?

A. Посилення секреції секретину

B. Зниження синтезу простагландинів

C. Гіпосекреція гастрину

D. Збільшення вмісту гістаміну

E. Зменшення виділення гастрину


233. Аналіз матеріалу біопсії слизової оболонки шлунка людини, хворої на хронічний гастрит, показав різке зменшення кількості парієнтальних клітин. Як це відбивається на показниках аналізу шлункового соку?

A. Збільшення кислотності

B. Зменшення продукції слизу

C. Зменшення кислотності

D. Збільшення вільної хлористоводневої кислоти

E. Зменшення продукції ферментів


234. У хворого зі скаргами на оперізуючий біль в епігастральній області, повторну блювоту різко знизився системний артеріальний тиск. При лабораторному обстеженні знайдено підвищений вміст діастази в сечі. Про яке ускладнення гострого панкреатиту повинен подумати лікар у першу чергу?

A. Гострий апендицит

B. Виразку шлунка

C. Панкреатичний шок

D. Панкреатичну недостатність

E. Діарею


235. У хворої зі скаргами на оперізуючий біль в епігастральній області при лабораторному обстеженні знайдено підвищений вміст діастази в сечі, а також стеаторея. Для якої форми патології травного каналу найбільш характерні такі явища?

A. Гострий апендицит

B. Виразкова хвороба шлунка

C. Гострий панкреатит

D. Гострий коліт

E. Інфекційний гепатит


236. У жінки віком 67 років, яка тривалий час страждала на холецистит, після їжі раптово виник різкий біль у верхній частині живота, нудота, блювання. Встановлено діагноз – гострий панкреатит. Що є основною ланкою патогенезу цього захворювання?

A. Передчасна активація ферментів підшлункової залози

B. Зниження рівня ферментів у панкреатичному соку

C. Підвищення активації ферментів у дванадцятипалій кишці

D. Зниження секреції панкреатичного поліпептиду

E. Підвищення рівня холецистокініну


237. У хворого виявлений синдром подразненої кишки, який характеризується значним посиленням моторики. Виділення якого інкрету найбільш імовірно збільшується при цьому?

A. Мотиліну

B. Інтестинального пептиду

C. Глюкагону

D. Урогастрону

E. Секретину


238. До дитячої лікарні потрапила 8-місячна дитина з діареєю, здуттям живота, гіпотрофією. Симптоми з'явилися і почали наростати після введення в раціон харчування борошняних виробів. Діагностовано целіакія. Який процес у травному каналі порушений?

A. Всмоктування в кишках

B. Секреція шлункового соку

C. Евакуація шлункового вмісту

D. Секреція жовчі

E. Секреція підшлункового соку


239. Хворий, що лікувався антибіотиками з приводу пневмонії, скаржиться на прискорене виділення рідких випорожнень із специфічним запахом, біль і бурчання в животі. Що, найімовірніше, стало причиною розвитку проносу?

A. Дисбактеріоз

B. Механічна стимуляція моторики

C. Імунне пошкодження кишок

D. Зниження моторики кишок

E. Гіперосмолярність химуса


240. У новонародженої дитини діагностована спадкова недостатність лактази. Який процес при цьому був порушений у дитини?

A. Порожнинне травлення

B. Мембранне травлення

C. Екскреторна функція кишок

D. Секреторна функція підшлункової залози

E. Секреція шлункового соку


241. У хворого, що тривалий час страждає на хронічний ентероколіт, після вживання молока виникли метеоризм, діарея, коліки. З нестачею якого ферменту в кишечнику це пов'язано?

A. Мальтаза

B. Лактаза

C. Глікогенсинтетаза

D. Амілаза

E. Сахараза


242. У хворого після апендектомії припинилося просування химуса. Який вид кишкової непрохідності за механізмом виник у даному випадку?

A. Странгуляційна

B. Обтураційна

C. Паралітична

D. Низька

E. Висока


243. У хворого з ущемленою паховою грижею порушене просування химуса. Який вид кишкової непрохідності має місце в даному випадку?

A. Странгуляційна

B. Обтураційна

C. Паралітична

D. Спастична

E. Атонічна


244. Хворий з високою кишковою непрохідністю скаржиться на біль у животі, багаторазову блювоту з жовчю, задишку. Об'єктивно: ЧСС – 110 за 1 хв., АТ – 90/50 мм рт. ст., гематокрит - 0,52, гіпонатріємія. Що є провідним механізмом порушень кровообігу у хворого?

A. Негазовий алкалоз

B. Артеріальна гіпертензія

C. Газовий ацидоз

D. Кишкова автоінтоксикація

E. Дегідратація, гіповолемія


245. Хворий з високою кишковою непрохідністю скаржиться на біль у животі, багаторазову блювоту з жовчю, задишку. Об'єктивно: ЧСС – 110 за 1 хв., АТ – 90/50 мм рт. ст., гематокрит - 0,52, гіпонатріємія. Яке порушення кислотно-основного гомеостазу в хворого?

A. Негазовий алкалоз

B. Видільний ацидоз

C. Газовий ацидоз

D. Газовий алкалоз

E. Екзогенний ацидоз


Патофізіологія печінки 246. У хворого з жовтяницею виявлено: збільшення в крові вмісту прямого білірубіну та холестерину, білірубінурія, ахолія, поява в крові жовчних кислот. Для якого виду жовтяниці характерні такі зміни?

A. Печінковоклітинної

B. Механічної

C. Гемолітичної

D. Фізіологічної

E. Ядерної


247. У жінки, 57 років, після нападу печінкової коліки з'явилася жовтяниця. УЗД показало закупорку загальної жовчної протоки каменем. Підвищення кількості якої речовини в крові свідчить про механічну жовтяницю?

A. Непрямого білірубіну

B. Білівердину

C. Вердоглобіну

D. Прямого білірубіну

E. Стеркобіліну


248. Хворий відзначає часті проноси, особливо після вживання жирної їжі, втрату маси тіла. Лабораторні дослідження показали наявність стеатореї; кал ахолічний. Що може бути причиною такого стану?

A. Дефіцит панкреатичної амілази

B. Запалення слизової оболонки тонкої кишки

C. Дефіцит панкреатичної ліпази

D. Обтурація жовчних шляхів

E. Незбалансована дієта


249. У хворого з жовтяницею виявлені гіпербілірубінемія за рахунок прямого і непрямого білірубіну, у сечі – прямий білірубін, уробілиноген. Є також гіпохолестеринемія, гіпопротеїнемія, уповільнення згортання крові. Для якого виду жовтяниці характерні такі зміни?

A. Паренхіматозної

B. Механічної

C. Гемолітичної

D. Фізіологічної

E. Ядерної


250. У хворого з жовтяницею діагностовано синдром Криглера - Наджара. Яке порушення обміну жовчних пігментів призвело до жовтяниці у хворого?

A. Посилений гемоліз

B. Знижене захоплення білірубіну гепатоцитами

C. Знижена коньюгація білірубіну гепатоцитами

D. Знижена екскреція білірубіну гепатоцитами

E. Обтурація жовчного протока


251. Чоловік скаржиться на гарячку, нудоту, біль у правому підребер'ї. Об'єктивно: жовті шкіра та склери, печінка збільшена, сеча темна, кал знебарвлений, гіпербілірубінемія (білірубін прямий та непрямий), білірубінурія, уробілінурія, гіпопротеїнемія. Для якого з названих станів характерні такі зміни?

A. Паренхіматозної жовтяниці

B. Підпечінкової жовтяниці

C. Надпечінкової гемолітичної жовтяниці

D. Гострого холециститу

E. Гострого панкреатиту


252. У хворого з жовтяницею діагностовано синдром Жильбера. Яке порушення обміну жовчних пігментів призвело до жовтяниці у хворого?

A. Посилений гемоліз

B. Знижене захоплення білірубіну гепатоцитами

C. Знижена кон’югація білірубіну гепатоцитами

D. Знижена екскреція білірубіну гепатоцитами

E. Обтурація жовчного протока


253. У хворого розвинулась імунна гемолітична анемія. Який показник сироватки крові збільшиться найбільше?

A. Прямий білірубін

B. Непрямий білірубін

C. Стеркобіліноген

D. Мезобіліноген

E. Протопорфірин


254. До лікаря звернувся пацієнт зі скаргами на пожовтіння склер і шкіри, потемніння сечі і калу. Аналіз крові: еритроцити - 2,5 • 10 12 /л, Нв - 80 г/л, кольоровий показник – 0,9, ретикулоцитоз. Яка жовтяниця розвинулася у цього пацієнта?

A. Шунтова

B. Ядерна

C. Паренхіматозна

D. Гемолітична

E. Механічна


255. У новонародженого з гемолітичною хворобою розвинулася енцефалопатія. Збільшення якої речовини в крові викликало ураження ЦНС?

A. Білірубіну, не зв'язаного з альбуміном

B. Жовчних кислот

C. Комплексу білірубін-альбумін

D. Вердоглобіну

E. Білірубін-глюкуроніду


256. В експерименті після дії жовчі на серце жаби спостерігається брадикардія. Який механізм лежить в основі цього ефекту?

A. Блокада адренорецепторів білірубіном

B. Активація n. vagus прямим білірубіном.

C. Активація n. vagus жовчними кислотами

D. Кардіодепресорний вплив холестерину

E. Активація холінестерази під дією жовчі


257. У хворого з жовтяницею спостерігається артеріальна гіпотензія, брадикардія, свербіння шкіри, дратівливість, сонливість вдень і безсоння вночі. Як називається такий синдром?

A. Ахолічний

B. Холемічний

C. Холестатичний

D. Гіпохолічний

E. Дисхолічний.


258. Жінка, 55 років, скаржиться на швидку стомлюваність, дратівливість, безсоння вночі і сонливий стан вдень, свербіння шкіри. Попередній діагноз: жовчнокам'яна хвороба із закупоркою каменем загальної жовчної протоки. Чим зумовлені зміни з боку нервової системи у хворої?

A. Гипербілірубінемією

B. Холалемією

C. Гіперхолестеринемією

D. Порушенням всмоктування жирів

E. Порушенням всмоктування жиророзчинних вітамінів


259. У хворого з жовтяницею з’явилися порушення травлення, стеаторея, знебарвлення калу, порушення згортання крові, ознаки авітамінозу. Про розвиток якого синдрому свідчать дані зміни?

A. Холемічного

B. Астенічного

C. Ахолічного

D. Гіперхолічного

E. Холестатичного


260. У хворого, який страждає на жовчнокам’яну хворобу, внаслідок обтурації жовчовивідних шляхів спостерігається знебарвлений жирний кал. Відсутність якого компонента жовчі зумовлює стеаторею?

A. Жирних кислот

B. Холестерину

C. Жовчних пігментів

D. Жовчних кислот

E. Лужної фосфатази


261. У хворого через наявність каменю у загальній жовчній протоці припинилося надходження жовчі в кишечник. Порушення якого з процесів спостерігається при цьому?

A. Перетравлення жирів

B. Всмоктування білків

C. Перетравлення білків

D. Перетравлення вуглеводів

E. Всмоктування вуглеводів


262. У хворого з патологією печінки є ознаки геморагічного синдрому. Який можливий механізм його виникнення?

A. Підвищення судинної проникності під дією жовчних пігментів

B. Гемоліз еритроцитів під дією жовчних кислот

C. Зниження утворення фактора Віллебранда

D. Порушення утворення протромбіну

E. Зниження здатності тромбоцитів до адгезії і агрегації


263. У хворого з алкогольним цирозом печінки скарги на загальну слабкість, задишку. Встановлено зниження артеріального тиску, асцит, розширення поверхневих вен передньої стінки живота, спленомегалію. Яке порушення гемодинаміки спостерігається у хворого?

A. Синдром портальної гіпертензії

B. Недостатність лівого шлуночку серця

C. Недостатність правого шлуночку серця

D. Колапс

E. Тотальна серцева недостатність


264. У хворого, що страждає на цироз печінки, спостерігається зниження артеріального тиску, розширення поверхневих вен передньої стінки живота, спленомегалія, асцит. Який механізм є провідним у виникненні асциту?

A. Застійний

B. Мембраногенний

C. Лімфогенний

D. Гіперпротеїнемічний

E. Гіперволемічний


265. У хворого 35 років, що переніс гепатит і продовжував приймати алкоголь, розвинулися ознаки цирозу печінки з асцитом і набряками на нижніх кінцівках. Які зміни складу крові стали вирішальними в розвитку набряку?

A. Гіпокаліємія

B. Гіпоглобулінемія

C. Гіпоальбумінемія

D. Гіпохолестеринемія

E. Гіпоглікемія


266. Хвора на хронічний гепатит скаржиться на підвищену чутливість до барбітуратів, які раніше вона переносила без симптомів інтоксикації. З порушенням якої функції печінки це пов’язане найбільше?

A. Фагоцитарної

B. Екскреторної

C. Гемодинамічної

D. Гемопоетичної

E. Метаболічної


267. Скарги і об'єктивні дані дозволяють припустити наявність у хворого запального процесу в жовчному міхурі, порушення колоїдних властивостей жовчі, вірогідність утворення жовчних каменів. Яка речовина, головним чином, може зумовити утворення каменів?

A. Холестерин

B. Хлориди

C. Фосфати

D. Урати

E. Оксалати


268. У людини унаслідок хронічного захворювання печінки істотно порушена її білковосинтезуюча функція. Який параметр гомеостазу зменшиться найбільш вірогідно?

A. рН

B. Онкотичний тиск плазми крові

C. Осмотичний тиск плазми крові

D. Гематокрит

E. Об’єм крові


269. У хворого на цироз печінки низький артеріальний тиск. Який механізм порушення гемодинаміки при цьому?

A. Збільшення опору судин

B. Депонування крові в портальному руслі

C. Збільшення ударного об'єму серця

D. Збільшення діастолічного об'єму серця

E. Розширення вен передньої стінки живота


270. Тварині після попередньої підготовки (фістула Екка – Павлова) видалили печінку. Що стане причиною загибелі тварини?

A. Гіперглікемічна кома

B. Гіпоглікемічна кома

C. Портальна гіпертензія

D. Ахолія

E. Холемія


Патофізіологія нирок 271. У хворого на аденому простати знизилась швидкість клубочкової фільтрації. Який механізм цього ускладнення?

A. Зниження гідростатичного тиску крові

B. Підвищення гідростатичного тиску в капсулі клубочка

C. Підвищення онкотичного тиску крові

D. Зменшення кількості функціонуючих нефронів

E. Зменшення площі фільтруючої поверхні


272. 43-річний хворий скаржиться на масивні набряки та артеріальну гіпертензію. У сечі: відносна щільність 1,017, білок 4,0 г/л, Ер – 15-18 в полі зору (вилужені). Який нирковий процес переважно порушений у хворого?

A. Фільтрація

B. Реабсорбція

C. Секреція

D. Інкреція

E. Екскреція


273. У хворого А., 38 років, на 3-му році захворювання на системний червоний вовчак виявилось дифузне ураження нирок, що супроводжувалося нефротичним синдромом. Який механізм розвитку протеїнурії у хворого?

A. Автоімунне ушкодження клубочків нефронів

B. Запальне пошкодження канальців нефронів

C. Ішемічне ушкодження канальців

D. Збільшення рівня протеїнів у крові

E. Ураження сечовидільних шляхів


274. При обстеженні хворого встановлено, що кліренс ендогенного креатиніну після 24-х годинного збору сечі становить 50 мл/хв (при нормі – 110-150 мл/хв.). Про порушення якого ниркового процесу свідчить наявність такої ознаки?

A. Клубочкової фільтрації

B. Канальцевої реабсорбції

C. Ниркової інкреції

D. Канальцевої секреції

E. Екскреції сечі


275. У хворого на мієломну хворобу з вираженою гіперпарапротеїнемією знизилась швидкість клубочкової фільтрації (ШКФ). Який фактор зумовив зменшення ШКФ в даному випадку?

A. Зниження гідростатичного тиску крові

B. Підвищення гідростатичного тиску в капсулі клубочка

C. Підвищення онкотичного тиску крові

D. Зменшення маси діючих нефронів

E. Зменшення площі фільтруючої поверхні


276. У хворого, віком 58 років, з гострою серцевою недостатністю, спостерігалось зменшення добової кількості сечі – олігурія. Який механізм цього явища?

A. Підвищення гідростатичного тиску крові в клубочкових капілярах

B. Зниження кількості функціонуючих нефронів

C. Збільшення онкотичного тиску крові

D. Зниження гідростатичного тиску крові в клубочкових капілярах

E. Зниження проникності клубочкової мембрани


277. Хворий після автомобільної катастрофи непритомний, АТ 70/40 мм рт. ст. За добу виділяє біля 300 мл сечі. Який механізм порушення сечоутворення у даному випадку?

A. Підсилення клубочкової фільтрації

B. Зменшення клубочкової фільтрації

C. Послаблення канальцевої реабсорбції

D. Підсилення канальцевої реабсорбції

E. Зменшення канальцевої секреції


278. У хворого з гострою нирковою недостатністю в стадії поліурії азотемія не тільки не зменшилась, але й продовжувала підвищуватись. Що в даному випадку викликало поліурію?

A. Збільшення фільтрації

B. Збільшення секреції

C. Збільшення реабсорбції

D. Зменшення фільтрації

E. Зменшення реабсорбції


279. У результаті порушення техніки безпеки відбулося отруєння сулемою (хлорид ртуті). Через 2 дні добовий діурез становив 200 мл. У хворого з'явилися головний біль, блювання, судоми, задишка, у легенях – вологі хрипи. Як називається така патологія?

A. Гостра ниркова недостатність

B. Хронічна ниркова недостатність

C. Нефротичний синдром

D. Гломерулонефрит

E. Пієлонефрит


280. Хворий доставлений у лікарню із шлунковою кровотечею в тяжкому стані. АТ - 80/60 мм рт. ст. Діурез 60-80 мл сечі за добу з відносною щільністю 1,028-1,036. Який механізм зумовив зменшення добового діурезу?

A. Підвищення онкотичного тиску крові

B. Зниження гідростатичного тиску в капілярах клубочків

C. Підвищений рівень залишкового азоту в крові

D. Підвищення колоїдно-осмотичного тиску в крові

E. Підвищення гідростатичного тиску в капсулі


Шумлянського - Боумена 281. У хворого, який був госпіталізований після того, як перебував під завалом протягом 4 годин, спостерігається різке зниження діурезу, гіперазотемія, ознаки набряку головного мозку. В якій стадії гострої ниркової недостатності перебуває хворий?

A. Початковій

B. Олігоануричній

C. Поліуричній

D. Одужання

E. Термінальній


282. У хворого після тяжкої травми грудної клітки розвинувся шок та з’явилися ознаки гострої ниркової недостатності (ГНН). Що є провідним механізмом розвитку ГНН в даному випадку?

A. Порушення відтоку сечі

B. Зменшення артеріального тиску

C. Підвищення тиску в капсулі клубочка

D. Підвищення тиску в ниркових артеріях

E. Зменшення онкотичного тиску крові


283. У хворого, що тривалий час страждає на хронічний гломерулонефрит, виникла азотемія при помірній поліурії. Що є головною ланкою в патогенезі цих симптомів при хронічній нирковій недостатності?

A. Зменшення маси діючих нефронів

B. Підвищення клубочкової фільтрації

C. Зниження канальцевої секреції

D. Порушення проникності клубочкових мембран

E. Недостатність вазопресину


284. У чоловіка 48 років, що давно хворіє на хронічний гломерулонефрит, обстеження виявило наявність хронічної недостатності нирок. Чим пояснюється гіперазотемія при хронічній нирковій недостатності?

A. Зниженням клубочкової фільтрації

B. Зниженням канальцевої реабсорбції

C. Зниженням канальцевої екскреції

D. Порушенням білкового обміну

E. Порушенням водно-електролітного балансу


285. У хворого з хронічною нирковою недостатністю встановлено зменшення кліренсу за інуліном до 60 мл/хв. З порушенням якої функції нирок це пов’язано?

A. Клубочкової фільтрації

B. Канальцевої секреції

C. Реабсорбції в проксимальному відділі нефрону

D. Реабсорбції в дистальному відділі нефрону

E. Реабсорбції в збиральних ниркових трубочках


286. У хворого на хронічну ниркову недостатність з’явилися анорексія, диспепсія, порушення ритму серця, свербіння шкіри. Який механізм розвитку цих порушень є головним?

A. Розвиток ниркового ацидозу

B. Порушення ліпідного обміну

C. Зміни вуглеводного обміну

D. Гіперазотемія

E. Порушення водно-електролітного обміну


287. У хворого, що тривалий час страждає на цукровий діабет І типу, виникли азотемія, олігурія, гіпо-, ізостенурія, протеїнурія. Що є головною ланкою в патогенезі цих симптомів при хронічній нирковій недостатності?

A. Порушення проникності клубочкових мембран

B. Підвищення клубочкової фільтрації

C. Зниження канальцевої секреції

D. Зменшення маси діючих нефронів

E. Посилення реабсорбції натрію


288. У хворого з хронічною патологією нирок після проведення проби Зимницького виявлено гіпостенурію. Які зміни сечовиділення будуть спостерігатися при цьому?

A. Анурія

B. Олігурія

C. Поліурія

D. Полакіурія

E. Ніктурія


289. У хворого М. діагностовано хронічний гломерулонефрит. Внаслідок значних склеротичних змін маса функціонуючих нефронів зменшилася до 10%. Яке з перерахованих нижче порушень зумовлює уремічний синдром?

A. Артеріальна гіпертензія

B. Порушення водного гомеостазу

C. Порушення осмотичного гомеостазу

D. Ниркова остеодистрофія

E. Азотемія


290. Білому щуру ввели під шкіру сулему в дозі 5 мг/кг маси тіла. Через 24 години в плазмі крові концентрація креатиніну збільшилася в декілька разів. Який механізм ретенційної гіперазотемії в даному випадку?

A. Зростання клубочкової фільтрації

B. Збільшення утворення креатиніну в м`язах

C. Зростання реабсорбції креатиніну

D. Зниження клубочкової фільтрації

E. Збільшення секреції креатиніну в канальцях нирок


291. У результаті гострої ниркової недостатності у хворого виникла олігурія. Яка добова кількість сечі відповідає даному симптому?

A. 100-500 мл

B. 1500-2000 мл

C. 1000-1500 мл

D. 500-1000 мл

E. 50-100 мл


292. В результаті гострої ниркової недостатності у хворого виникла анурія. Яка добова кількість сечі відповідає даному симптому?

A. 100-500 мл

B. 1500-2000 мл

C. 1000-1500 мл

D. 500-1000 мл

E. 50-100 мл


293. У хворого на первинний нефротичний синдром встановлений вміст загального білку крові 40 г/л. Яка причина зумовила гіпопротеїнемію?

A. Порушення всмоктування білка у кишечнику

B. Вихід білка з судин у тканини

C. Зниження синтезу білка у печінці

D. Підвищений протеоліз

E. Протеїнурія


294. У хворого на хронічний пієлонефрит розвинулась ізостенурія. Яка питома вага сечі має бути у цього хворого?

A. 1,000

B. 1,003

C. 1,010

D. 1,020

E. 1,030


295. Хворій Н. 3 роки тому був поставлений діагноз хронічний гломерулонефрит. Останні 6 місяців з'явились набряки. Що головним чином спричинює їх розвиток?

A. Гіпоальдостеронізм

B. Гіперпродукція вазопресину

C. Порушення білковоутворюючої функції печінки

D. Гіперосмолярність плазми

E. Протеїнурія


296. У 20-річного хлопця через 2 тижня після перенесеної лакунарної ангіни був поставлений діагноз: гострий гломерулонефрит. Які патологічні зміни в складі сечі найбільш імовірні?

A. Наявність свіжих еритроцитів

B. Циліндрурія

C. Лейкоцитурія

D. Протеїнурія

E. Натрійурія


297. В експерименті кролю ввели нефроцитотоксичну сироватку морської свинки. Яке захворювання нирок моделювалося в цьому досліді?

A. Нефротичний синдром

B. Гострий дифузний гломерулонефрит

C. Гострий пієлонефрит

D. Хронічна ниркова недостатність

E. Хронічний пієлонефрит


298. У хворого з гломерулонефритом виявлено: АТ – 185/105 мм рт.ст., анемія, лейкоцитоз, гіперазотемія, гіпопротеїнемія. Який показник свідчить про ускладнення гломерулонефриту нефротичним синдромом?

A. Анемія

B. Лейкоцитоз

C. Гіперазотемія

D. Артеріальна гіпертензія

E. Гіпопротеїнемія


299. У хворого на гострий гломерулонефрит спостерігається протеїнурія з молекулярною масою білка понад 70000

D. Яке порушення спричинило це явище?

A. Зниження фільтрації білка

B. Ушкодження клубочкової мембрани

C. Зниження реабсорбції білка

D. Зниження канальцевої секреції

E. Ушкодження канальців


300. У дитини 5 років через 2 тижні після перенесеної ангіни виник гострий дифузний гломерулонефрит, що характеризувався олігурією, протеїнурією, гематурією, збільшенням ОЦК. Порушення якого з ниркових процесів найбільш суттєве для виникнення цих симптомів?

A. Ниркової інкреції

B. Канальцевої реабсорбції

C. Канальцевої секреції

D. Концентрування сечі

E. Клубочкової фільтрації


Патофізіологія ендокринної системи 301. У хворого Н., 25 років, після перенесеної інфекції розвинувся нецукровий діабет. Дефіцит якого гормону призвів до розвитку даної патології?

A. Альдостерону

B. Вазопресину

C. Кортизолу

D. Реніну

E. Інсуліну.


302. Хворий скаржиться на постійну спрагу, знижений апетит. Випиває за добу 9 л. Поліурія, сеча знебарвлена, відносна щільність – 1,005. Найбільш імовірною причиною розвитку даної патології у хворого є пошкодження:

A. Наднирників

B. Гіпоталамічних ядер

C. Аденогіпофізу

D. Епіфізу

E. Базальної мембрани капілярів клубочків


303. У дитини 2,5 років спостерігається затримка у фізичному розвитку, поганий сон та апетит, спрага, поліурія. Цукор у сечі не виявляється. Яка ендокринна патологія є причиною порушень водно-мінерального обміну?

A. Гіперсекреція антидіуретичного гормону

B. Гіпосекреція антидіуретичного гормону

C. Вторинний гіперальдостеронізм

D. Гіпосекреція соматотропного гормону

E. Гіпосекреція адренокортикотропного гормону


304. У хворого 40 років у зв'язку з ураженням гіпоталамо-гіпофізарного провідного шляху виникла поліурія (10-12 л за добу), полідипсія. При дефіциті якого гормону виникають такі порушення?

A. Вазопресину

B. Тиреотропіну

C. Кортикотропіну

D. Соматотропіну

E. Окситоцину


305. До лікаря звернувся чоловік 27 років. При огляді було виявлено збільшення кистей рук, ступнів та нижньої щелепи. Крім того є імпотенція, атрофія яєчок. Функції якої залози порушені?

A. Передньої частини гіпофізу

B. Задньої частини гіпофізу

C. Надниркових залоз

D. Щитоподібної залози

E. Прищитоподібних залоз


306. Хворий звернувся до лікаря зі скаргами на біль у голові, зміни у кінцівках, збільшення рук та ніг. Зовні: масивні надбрівні дуги, губи. При надлишку якого гормону виявляються такі зміни?

A. Тиреотропного

B. Адренокортикотропного

C. Соматотропного

D. Глюкокортикоїдів

E. Адреналіну


307. Батьки хлопчика 15 років звернулися до лікаря зі скаргами на значне відставання сина в рості. Об’єктивно: статура правильна, зріст 98 см, інтелект не порушений. При нестачі якого гормону з'являються такі зміни?

A. Тиреотропного

B. Адренокортикотропного

C. Соматотропного

D. Гонадотропного

E. Пролактину


308. У хворого В., 46 років, виявлено непропорційне збільшення кистей рук, ступнів, носа, вух, надбрівних дуг. Лабораторно гіперглікемія, глюкозурія, порушення тесту толерантності до глюкози. Причиною цукрового діабету є:

A. Гіпосекреція інсуліну

B. Гіперсекреція кортикотропіну

C. Гіперсекреція соматотропного гормону

D. Гіпосекреція вазопресину

E. Гіперсекреція глюкокортикоїдів


309. У хворої, що страждає на гіпертиреоз, підвищені основний обмін, температура тіла, стійкість до гіпотермії. Що можна зазначити як головну ланку в патогенезі цих проявів?

A. Збільшення споживання О 2 мітохондріями

B. Зменшення окисного фосфорування

C. Активація глікогенолізу

D. Активація гліколізу

E. Інактивація ферментів дихального ланцюга


310. Хворій Д., 50 років, був поставлений діагноз мікседема. Порушення утворення яких гормонів призводить до розвитку даної патології?

A. Тироксину і трийодтироніну

B. Кортизолу і альдостерону

C. АКТГ і СТГ

D. Окситоцину і вазопресину

E. Інсуліну і глюкагону


311. Пацієнт скаржиться на напади серцебиття, пітливість і дратівливість. Обстеження показало наявність у нього дифузного токсичного зобу. Імунні реакції якого типу (за Кумбсом і Джеллом) є головною ланкою в патогенезі Базедової хвороби?

A. I типу (анафілактичні)

B. II типу (гуморальні цитотоксичні )

C. III типу (імунокомплексні)

D. IV типу (клітинні цитотоксичні)

E. V типу (стимулюючі)


312. У ліквідатора аварії на ЧАЕС через деякий час з'явились скарги на підвищену збудливість, нервозність, серцебиття, зниження маси тіла, постійну слабкість, тремтіння тіла, відчуття жару. Гіперфункція якої залози може бути причиною вказаних змін:

A. Щитоподібної залози

B. Аденогіпофізу

C. Кіркової речовини наднирників

D. Мозкової речовини наднирників

E. Паращитоподібної залози


313. У хворого з базедовою хворобою виявлено тахікардію та ізольовану систолічну артеріальну гіпертензію. Збільшення чутливості до якого гормону є головною ланкою в патогенезі цих проявів при гіпертиреозі?

A. Соматотропіну

B. Соматостатину

C. Адреналіну

D. Альдостерону

E. ТТГ


314. У пацієнта 20 років після хірургічного видалення щитоподібної залози спостерігається гіпоплазія емалі, порушення утворення дентину. Недостатність якого гормону стала причиною вказаної патології?

A. Паратгормону

B. Тироксину

C. Тиреотропіну

D. Трийодтироніну

E. Тиреокальцитоніну


315. У жительки Закарпаття з’явились слабкість, сонливість, апатія, зниження пам’яті, набряки. Після обстеження встановлений діагноз “ендемічний зоб”. Брак якої речовини у воді та їжі може призвести до цієї хвороби?

A. Цинку

B. Фтору

C. Заліза

D. Йоду

E. Магнію


316. У жительки Закарпаття діагностовано гіпотиреоїдний зоб. Вміст якого гормону в крові збільшується при ендемічному зобі?

A. АКТГ

B. ТТГ

C. СТГ

D. МСГ

E. АДГ


317. Дитина 3-х років млява, малорухлива, різко відстає від однолітків у фізичному і психічному розвитку, понижені основний обмін і температура тіла. Яке захворювання у дитини?

A. Кретинізм

B. Базедова хвороба

C. Тиреотоксикоз

D. Ендемічний зоб

E. Спорадичний зоб


318. У хворого на правошлуночкову серцеву недостатність є асцит та набряки. Яке ендокринне порушення має значення в патогенезі серцевих набряків?

A. Первинний гіперальдостеронізм (синдром Конна)

B. Вторинний гіперальдостеронізм

C. Вторинний гіперкортицизм (хвороба Іценка – Кушинга)

D. Первинний гіперкортицизм (синдром Іценка – Кушинга)

E. Первинний гіпокортицизм (хвороба Адісона)


319. У хворого, 39 років, з набряками на ногах, задишкою і іншими ознаками правошлуночкової серцевої недостатності діагностована гіперпродукція альдостерону. Чим викликана ця зміна?

A. Гіперпродукцією реніну

B. Гіперпродукцією кортикотропіну

C. Гіперпродукцією адреналіну

D. Гіпопродукцією кортикотропіну

E. Гіпопродукцією адреналіну


320. У хворого виявлена аденома клубочкової зони кори наднирників. Внаслідок цього розвинувся первинний гіперальдостеронізм або хвороба Конна. На обмін якого іону впливає ця пухлина?

A. Натрію

B. Хлору

C. Магнію

D. Кальцію

E. Заліза


321. У хворого на аденому клубочкової зони кори наднирників (хворобу Конна) спостерігаються м’язова слабкість, атонія кишок, аритмії серця, парестезії. Дефіцит якого катіону є головною ланкою в патогенезі цих розладів?

A. Натрію

B. Калію

C. Кальцію

D. Магнію

E. Заліза


322. У хворого на аденому клубочкової зони кори наднирників (хворобу Конна) спостерігаються артеріальна гіпертензія, м’язова слабкість, аритмії серця, судоми. Яке порушення кислотно-основного гомеостазу в хворого?

A. Видільний алкалоз

B. Видільний ацидоз

C. Газовий ацидоз

D. Газовий алкалоз

E. Екзогенний ацидоз


323. Після адреналектомії у тварини виникла зупинка серця в діастолі і вона загинула. Яке порушення викликало зупинку серця?

A. Гіпернатріемія

B. Гіперкаліемія

C. Гіпермагніемія

D. Гіперкальціемія

E. Гіперфосфатемія


324. Після двостороннього видалення наднирників у собаки з'явилась м’язова слабкість, адинамія, зниження температури тіла, гіпоглікемія. Які ще можуть бути відмічені прояви наднирникової недостатності?

A. Лімфопенія

B. Артеріальна гіпотонія

C. Гіпопротеїнемія

D. Гіпернатріємія

E. Ожиріння


325. У хворого Аддісонова або бронзова хвороба. Недостатність якого гормону має місце при цьому патологічному процесі?

A. Інсуліну

B. Альдостерону

C. Адреналіну

D. Тироксину

E. Вазопресину


326. У хворого, що протягом року лікувався преднізолоном, при поступовому зменшенні дози з’явилися швидка фізична і психічна втомлюваність, артеріальна гіпотензія, схуднення, прогресуюча гіперпігментація шкіри. Що лежить в основі цього симптомокомплексу?

A. Хронічна гіпофункція кори наднирників

B. Гостра гіпофункція кори наднирників

C. Хронічна гіперфункція кори наднирників

D. Гостра гіперфункція кори наднирників

E. Дисфункція кори наднирників


327. Після перенесеного сепсису у хворого з'явився бронзовий колір шкіри, характерний для Аддісонової хвороби. Механізм гіперпігментації полягає у підвищенні секреції гормону:

A. Меланоцитстимулюючого

B. Соматотропного

C. Гонадотропного

D. Лактотропного

E. Тиреотропного


328. Хворий з Аддісоновою хворобою має виражену схильність до гіпоглікемії. Чим спричинена гіпоглікемія у хворого?

A. Активацією глікогенолізу

B. Пригніченням синтезу глікогену

C. Пригніченням глюконеогенезу

D. Пригніченням гліколізу

E. Активацією ліполізу


329. У хворого з частими ортостатичними колапсами діагностовано хвороба Аддісона. Зменшення чутливості до якого гормону є головною ланкою в патогенезі цих проявів при гіпокортицизмі?

A. Соматотропіну

B. Соматостатину

C. Адреналіну

D. Альдостерону

E. ТТГ


330. У хворого на ожиріння виявили синдром Іценка – Кушинга. Яке порушення метаболізму є головною ланкою в патогенезі ожиріння в хворих на кортизолпродукуючі пухлини?

A. Гіпераміноацидемія

B. Гіперглікемія

C. Гіперβ-ліпопротеінемія

D. Гіпоα-ліпопротеінемія

E. Гіперхолестеринемія


331. У хворої виявлені ожиріння обличчя, шиї і верхньої частини тулуба, стрії на животі, гірсутизм, гіперглікемія і гіпертензія. Гіперпродукція якого гормону може бути причиною таких змін?

A. Альдостерон

B. Адреналін

C. Тестостерон

D. Кортизол

E. Естрадіол


332. Хвора з діагнозом хвороба Іценка - Кушінга скаржиться на м’язову слабкість, гіпертензію, ожиріння верхньої частини тулуба, шиї та обличчя. Первинна гіперпродукція якого гормону зумовлює дану хворобу?

A. Кортикотропіну

B. Кортизолу

C. Катехоламінів

D. Інсуліну

E. Тироксину


333. У хворої 44 років виявлені ознаки гіперкортицизму. Що допоможе диференціювати хворобу Іценка – Кушінга від синдрому Іценка – Кушінга?

A. Кількість еозинофілів у крові

B. Рівень кортикотропіну в крові

C. Рівень кортизолу в крові

D. Вміст 17–оксикетостероїдів у сечі

E. Рівень глюкози у крові


334. У хворого виявлено при гормон-продукуючій пухлині пучкової зони кори наднирників. Вміст у крові якого гормону зменшується в цьому випадку?

A. ТТГ

B. СТГ

C. АКТГ

D. ФСГ

E. ЛГ


335. У хворого відмічається періодичні напади головного болю з серцебиттям і сильним потовиділенням. При обстеженні виявлені гіпертензія, гіперглікемія, підвищення основного обміну, тахікардія. Для якої патології наднирників характерні ці симптоми?

A. Гіпофункції мозкової речовини наднирників

B. Гіперфункції мозкової речовини наднирників

C. Гіперфункції кори наднирників

D. Гіпофункції кори наднирників

E. Первинному гіперальдостеронізмі


336. У хворої з феохромоцитомою після психічного навантаження виникає тахікардія, підвищується артеріальний тиск, з’являється різкий біль у надчеревній ділянці. Ці приступи зумовлені:

A. Збільшенням секреції тиреоїдних гормонів

B. Звільненням норадреналіну симпатичними нервами

C. Активацією вегетативних ядер гіпоталамуса

D. Масивним викидом катехоламінів наднирниками

E. Підвищеним синтезом адренокортикотропного гормону


337. У дівчинки діагностований адреногенітальний синдром з ознаками вірилізації. Надмірна секреція яких гормонів наднирників зумовила вірилізацію?

A. Глюкокортикоїдів

B. Естрогенів

C. Мінералокортикоїдів

D. Андрогенів

E. Катехоламінів


338. У дівчинки діагностований адреногенітальний синдром. Гіперпродукція якого гормону стала причиною надлишку андрогенів?

A. ТТГ

B. СТГ

C. АКТГ

D. ФСГ

E. ЛГ


339. У дівчинки діагностований адреногенітальний синдром. Первинний генетичний дефект синтезу яких гормонів наднирників зумовив дану патологію?

A. Глюкокортикоїдів

B. Естрогенів

C. Мінералокортикоїдів

D. Андрогенів

E. Катехоламінів


340. У чоловіка 42 років, що довгий час перебував у стані стресу, у сечі значно збільшений вміст 17-кетостероїдів, що в першу чергу свідчить про підвищення секреції:

A. Адреналіну

B. Альдостерону

C. АКТГ

D. Норадреналіну

E. Естрадіолу


341. Чоловік 25 років надійшов до лікарні через годину після автомобільної аварії. Спостерігається картина гострого стресу без виражених ушкоджень. Яке явище в периферійній крові буде характерним для цього стану?

A. Еозинопенія

B. Анемія

C. Моноцитопенія

D. Лейкопенія

E. Нейтрофілія


342. У спортсмена, який тривалий час вживав анаболічні стероїди, виник гіпогонадизм. Чим зумовлений цей стан?

A. Гіпопродукцією реніну

B. Гіпопродукцією гонадотропіну

C. Гіпопродукцією соматотропіну

D. Гіпопродукцією кортикотропіну

E. Гіпопродукцією адреналіну


343. У жінки після тяжкого психоемоційного стресу виникла аменорея. Функції якого органу порушені?

A. Середньої частини гіпофізу

B. Задньої частини гіпофізу

C. Надниркових залоз

D. Щитоподібної залози

E. Гіпоталамуса


344. Через 1–2 доби після видалення у собаки прищитоподібних залоз спостерігалось: в’ялість, спрага, різке підвищення нервово-м’язової збудливості з розвитком тетанії. Яке порушення обміну електролітів виникло?

A. Гіпокальціємія

B. Гіперкальціємія

C. Гіпомагніємія

D. Гіпермагніємія

E. Гіпонатріємія


345. Хворому 45-ти років при оперативному втручанні на щитоподібній залозі випадково видалили прищитоподібні залози. Це призвело до:

A. Тетанії

B. Підвищення рівня кальцію в крові і резорбції кісток

C. Підвищення рівня кальцію, натрію і калію в крові

D. Зниження артеріального тиску

E. Підвищення артеріального тиску


346. У пацієнта 20 років спостерігається гіпоплазія емалі, порушення утворення дентину. Гіперпродукція якого гормону стала причиною вказаної патології?

A. Паратгормону

B. Тироксину

C. Тиреотропіну

D. Соматотропіну

E. Тиреокальцитоніну


347. Жінка, що хворіє на цукровий діабет 1 типу, знаходиться у стані крайнього збудження; агресивна, відмовляється від обстеження. Шкіра бліда, волога. Руки тремтять. Пульс - 90/хв, AT - 135/85 мм рт.ст., ЧД 18/хв. Запаху ацетону або алкоголю немає. Що найбільш імовірно наявне у хворої?

A. Алкогольна інтоксикація

B. Гіперкетонемія

C. Гіперлактацидемія

D. Гіпоглікемія

E. Гіперглікемія


348. У дівчини 14 років, яка скаржиться на значне схуднення, спрагу, поліурію, виявлено гіперглікемію натщесерце та зниження толерантності до глюкози. При нестачі якого гормону може розвинутись такий стан?

A. Інсуліну

B. Тироксину

C. Глюкокортикоїдів

D. Альдостерону

E. Соматотропного гормону


349. У хворого діагностовано цукровий діабет 1 типу. Яка ендокринопатія за патогенезом виникла у хворого?

A. Дисрегуляторна

B. Залозиста

C. Периферична

D. Гіперфункція

E. Дисфункція


350. У хворого вперше виявили цукровий діабет 2 типу. У крові гіперглікемія натщесерце 8 ммоль/л, гіперінсулінемія. Яка ендокринопатія за патогенезом виникла у хворого?

A. Дисрегуляторна

B. Залозиста

C. Периферична

D. Гіперфункція

E. Дисфункція


Патофізіологія нервової системи 351. Відомо, що при раптовому припиненні надходження кисню в головний мозок, непритомність настає через 6-7 секунд. Яке патологічне явище лежить в основі порушення функцій клітин мозку в даному випадку?

A. Токсичне ураження

B. Енергодефіцит

C. Іонний дисбаланс

D. Гіпергідратація клітин мозку (набухання)

E. Гіпогідратація клітин мозку


352. Відомо, що при зниженні рівня глюкози в крові дуже швидко виникає гіпоглікемічна кома. Яке патологічне явище лежить в основі порушення функцій клітин мозку в даному випадку?

A. Токсичне ураження

B. Енергодефіцит

C. Іонний дисбаланс

D. Гіпергідратація клітин мозку (набухання)

E. Гіпогідратація клітин мозку


353. У хворого з хворобою Крейтцфельда-Якоба розвинулася спонгіоформна губчаста енцефалопатія. До якої групи належить інфекційний агент, що викликав це захворювання?

A. РНК-вмісні віруси

B. ДНК-вмісні віруси

C. Бактеріі

D. Пріони

E. Гриби


354. З нейронів ніколи не утворюються пухлини, оскільки вони не діляться і в них неможлива одна з стадій канцерогенезу. Яка це стадія?:

A. Ініціація

B. Трансформація

C. Промоція

D. Прогресія

E. Малігнізація


355. У головному мозку ніколи не буває фізіологічної артеріальної гіперемії. Це явище зумовлене тим, що мозковий кровообіг підпорядкований:

A. Добовим змінам активності мозку

B. Метаболічним потребам нейронів

C. Центральній нервовій регуляції

D. Центральній гуморальній регуляції

E. Інтенсивності розумової праці


356. У хворого діагностовано автоімунний енцефаліт. Чим може бути зумовлене це захворювання?

A. Пошкодженням гематоенцефалічного бар’єру

B. Зняттям імунологічної толерантності

C. Недостатністю гуморального імунітету

D. Недостатністю клітинного імунітету

E. Недостатністю фагоцитозу


357. У хворого з черепно-мозковою травмою виник енцефаліт. Який механізм призвів до автоімунного ушкодження мозку?

A. Трансформація лімфоцита

B. Втрата толерантності

C. Демаскування антигену

D. Гіперпродукція IgE

E. Гіпопродукція IgG


358. У дитини 6 місяців з уповільненим моторним і психічним розвитком відзначається збліднення шкірних покривів, волосся, райдужної оболонки ока, позитивна проба Фелінга (з 3% розчином трихлороцтового заліза). Яке з зазначених спадкових захворювань виявлено в дитини?

A. Галактоземія

B. Алкаптонурія

C. Хвороба Дауна

D. Альбінізм

E. Фенілкетонурія


359. Після автомобільної катастрофи у хворого діагностована травма середньої третини плеча з неповним розривом серединного нерва. Окрім рухових і сенсорних порушень нижче місця травми, хворий скаржиться на пекучий, нестерпний біль. Який вид болю у хворого?

A. Відбитий.

B. Соматичний.

C. Фантомний.

D. Каузалгія.

E. Проекційний.


360. Хворий з анемією Аддісона – Бірмера скаржиться на відчуття „повзання мурашок” по тілу. Як називається це порушення чутливості?

A. Анестезія

B. Гіпоестезія

C. Парестезія

D. Анальгезія

E. Гіперестезія


361. Після травматичного видалення зуба хворий скаржиться на тупий, не чітко локалізований, сильний біль у яснах, підвищення температури тіла до 37,5 0 С. Діагностовано альвеоліт. Який вид болю у даного хворого?

A. Епікритичний

B. Протопатичний

C. Вісцеральний

D. Відбитий

E. Фантомний


362. Хворий 50 років з цукровим діабетом 1-го типу скаржиться на повну втрату чутливості пальців ніг. Як називається таке порушення чутливості?

A. Анальгезія

B. Анестезія

C. Гіпоестезія

D. Парестезія

E. Гіперестезія


363. Хворому 4 місяці тому ампутували ногу. Зараз його дуже турбує відчуття сильного болю у відсутній кінцівці. Як можна визначити цей вид болю?

A. Патологічний

B. Фізіологічний

C. Соматичний

D. Вісцеральний

E. Відбитий


364. Дитина випадково доторкнулася до гарячої праски, скрикнула та відсмикнула руку. Який вид болю виник у цьому випадку?

A. Вісцеральний

B. Відбитий

C. Фантомний

D. Епікритичний

E. Протопатичний


365. У хворого порушена координація рухів; рухи розмашисті, непропорційні, хода «півняча», «п'яна», пальцем важко попасти в ціль. З ураженням яких нервових структур пов'язано це порушення?

A. Пірамідної системи

B. Лімбічної системи

C. Кори мозку

D. Мозочка

E. Стовбура мозку


366. У тварини після видалення одного з відділів головного мозку рухи втратили плавність, точність. Як називається описане явище?

A. Тремор

B. Гіпокінез

C. Атаксія

D. Ригідність

E. Атетоз


367. У тварини після перерізання n. oftalmicus з'явилися екзофтальм, звуження зіниці, в'ялість повік. Через деякий час рогівка помутніла, утворилася виразка. Порушення якої функції нервового волокна виникло в даному випадку?

A. Чутливої

B. Больової

C. Трофічної

D. Рухової

E. Вегетативної


368. У хворого відсутні довільні і мимовільні рухи нижніх кінцівок, що супроводжуються атонією і атрофією м'язів. Який параліч має місце в даному випадку?

A. Спастичний

B. Периферичний

C. Центральний

D. Геміплегія

E. Моноплегія


369. У хворого відмічені швидкі мимовільні стереотипні скорочення м’язів обличчя, що імітують мигання і примружування. До якої форми розладів рухової функції нервової системи слід віднести описане порушення?

A. Гіпокінез

B. Акінезія

C. Гіперкінез

D. Атаксія

E. Арефлексія


370. 78-річному хворому поставлений діагноз «паркінсонізм». Нестача якого медіатора в хвостатому ядрі головного мозку є головною ланкою в патогенезі цієї хвороби?

A. Дофаміну

B. Ацетилхоліну

C. Адреналіну

D. Гістаміну

E. Норадреналіну


371. У пацієнта під час гіпертонічного кризу стали неможливими довільні рухи правою ногою, пасивні рухи збереглися в повному обсязі. При пальпації визначається високий тонус м'язів кінцівки. Яка форма порушення рухової функції має місце у хворого?

A. Периферичний парез

B. Атаксія мозочка

C. Центральний параліч

D. Периферичний параліч

E. Тетанія


372. У дитини 12 років спостерігаються швидкі, несподівані, розмашисті, позбавлені стереотипності рухи. У них беруть участь м'язи обличчя і кінцівок. М'язовий тонус понижений. Яка форма порушень рухової функції нервової системи виникла?

A. Хорея

B. Тремор

C. Атетоз

D. Тик

E. Абазія


373. У хворого виникли мимовільні „червоподібні” рухи в пальцях рук і ніг (згинання і розгинання пальців, що настають один за одним; перерозгинання середніх і кінцевих фаланг), які посилюються при довільних рухах. Як називається така форма порушення рухової функції нервової системи?

A. Хорея

B. Тремор

C. Атетоз

D. Тик

E. Абазія


374. У жінки після інсульту відсутні довільні рухи у верхній і нижній кінцівках справа. Тонус м'язів і рефлекси в них підвищені. Спостерігаються патологічні рефлекси. Які клітини мозку пошкоджені у хворої?

A. Пірамідні клітини кори

B. Глія

C. Альфа-мотонейрони рухових ядер

D. Чутливі нейрони

E. Нейрони мозочка


375. У хворого на поліомієліт відсутні довільні і мимовільні рухи правої ноги, що супроводжуються атонією і атрофією м'язів. Які клітини мозку пошкоджені у хворого?

A. Пірамідні клітини кори

B. Глія

C. Альфа-мотонейрони рухових ядер

D. Чутливі нейрони

E. Нейрони мозочка


Екстремальні стани 376. У хворого з печінковою недостатністю з’явилася сонливість, а потім він знепритомнів. Накопичення якої речовини порушує енергозабезпечення нейронів при печінковій енцефалопатії?

A. Глюкоза

B. ГАМК

C. Метилмеркаптан

D. Аміак

E. Прямий білірубін


377. У хворого з печінковою недостатністю з’явилася сонливість, а потім він знепритомнів. Яка речовина гальмує синаптичну передачу в мозку при печінковій енцефалопатії?

A. Глюкоза

B. ГАМК

C. Метилмеркаптан

D. Аміак

E. Прямий білірубін


378. Хворого на цукровий діабет 1 типу доставили у клініку в коматозному стані. Об’єктивно: дихання шумне, часте, глибоке, запах ацетону з рота. Вміст глюкози в крові 15,2 ммоль/л, кетонових тіл – 10,6 ммоль/л. Для якого ускладнення даного захворювання характерні такі показники?

A. Печінкова кома

B. Кетоацидотична кома

C. Лактацидемічна кома

D. Гіпоглікемічна кома

E. Гіперосмолярна кома


379. Хворому на цукровий діабет І типу ввели інсулін. Через деякий час у хворого з’явилися загальна слабкість, гостре почуття голоду, дратівливість, запаморочення, кома. Який основний механізм виникнення коми у пацієнта?

A. Зменшення глюконеогенезу.

B. Вуглеводне голодування головного мозку.

C. Посилення кетогенезу.

D. Посилення глікогенолізу.

E. Посилення ліпогенезу.


380. У хворого на цукровий діабет розвинулась гіперглікемічна кома (рівень глюкози 54 ммоль/л). Яке патологічне явище лежить в основі порушення функцій клітин мозку в даному випадку?

A. Токсичне ураження

B. Енергодефіцит

C. Іонний дисбаланс

D. Гіпергідратація клітин мозку (набухання)

E. Гіпогідратація клітин мозку


381. Хворий 35 років доставлений у стаціонар непритомним, дихання часте, глибоке (типу Куссмауля), АТ 90/45 мм рт. ст., з рота запах ацетону. Глюкоза крові – 18 ммоль/л, у сечі – глюкозурія, кетонурія. Яка кома виникла у даного хворого?

A. Гіпоглікемічна

B. Гіперосмолярна

C. Кетоацидотична

D. Печінкова

E. Лактацидотична


382. Хворий 55 років доставлений у стаціонар непритомним, дихання часте, поверхневе, АТ 90/45 мм рт. ст., шкіра в'яла, суха. Лабораторно: глюкоза крові – 55 ммоль/л, Na – 150 ммоль/л, Cl – 15 ммоль/л Яка кома виникла у даного хворого?

A. Гіпоглікемічна

B. Гіперосмолярна

C. Кетоацидотична

D. Печінкова

E. Лактацидотична


383. Хворий 68 років доставлений у стаціонар непритомним, дихання часте, глибоке, АТ 90/45 мм рт. ст., шкіра в'яла, суха. Лабораторно: глюкоза крові – 15 ммоль/л, молочна кислота крові – 12 ммоль/л. Яка кома виникла у даного хворого?

A. Гіпоглікемічна

B. Гіперосмолярна

C. Кетоацидотична

D. Печінкова

E. Лактацидотична


384. У хворого на хронічну ниркову недостатність з’явилися анорексія, нудота, блювота, порушення ритму серця, свербіння шкіри, психічні порушення аж до коми. Який механізм розвитку уремічної коми є головним?

A. Нирковий алкалоз

B. Порушення ліпідного обміну

C. Зміни вуглеводного обміну

D. Гіперазотемія

E. Зневоднення


385. У хворого, що раптово знепритомнів, у плазмі крові Na + – 180 ммоль/л, осмолярність – 340 мосмоль/л, глюкоза – 5 ммоль/л. Чим, найімовірніше, зумовлене виникнення коми у хворого?

A. Вуглеводним голодуванням

B. Гіпоосмолярним станом

C. Дегідратацією нейронів

D. Гіперглікемією

E. Набуханням нейронів


386. У хворого на хворобу Аддісона внутрішньовенна інфузія глюкози призвела до «водного отруєння» та гіпоосмолярної коми. Чим зумовлене виникнення коми у хворого?

A. Вуглеводним голодуванням

B. Алкалозом

C. Дегідратацією нейронів

D. Гіперкаліємією

E. Набуханням нейронів


387. Хворого доставлено до медичної установи в коматозному стані. Зі слів супровідних удалося встановити, що хворий знепритомнів під час тренування на завершальному етапі марафонської дистанції. Яка кома найбільш імовірна в цього хворого?

A. Гіперглікемічна

B. Печінкова

C. Уремічна

D. Гіпоглікемічна

E. Гіпотиреоїдна


388. Хворий сонливий, свідомість затуманена, реакція на сильні подразники загальмована. М'язові фібрилярні посмикування. Дихання типу Чейна-Стокса з аміачним запахом. При аускультації серця - шум тертя перикарда. У крові знижена кількість сечовини. Який вид коми розвинувся у хворого?

A. Кетоацидотична

B. Ниркова

C. Гіперосмолярна

D. Апоплексична

E. Печінкова


389. У лікарню швидкої допомоги поступив чоловік із закритим переломом стегна. Постраждалий у збудженому стані. Що вимагає негайної корекції в патогенезі травматичного шоку у пацієнта?

A. Ацидоз.

B. Біль.

C. Інтоксикація.

D. Гіпоксія.

E. Аритмія серця.


390. Після введення анестетика в пацієнта розвинувся анафілактичний шок. Який механізм визначає розвиток гострої недостатності кровообігу при цьому?

A. Гіперволемія

B. Зниження судинного опору

C. Біль

D. Активація симпато-адреналової системи

E. Зниження скоротливої функції серця


391. У патогенезі гострої недостатності кровообігу при анафілактичному шоку є розширення артеріол та підвищення проникності капілярів. Який з медіаторів зумовлює розвиток даних порушень?

A. Гістамін

B. Гепарин

C. Тромбоксани

D. Лімфокіни

E. Комплемент


392. У хворого після введення лідокаїну розвинувся анафілактичний шок. Які медіатори анафілаксії спричинюють безпосередньо обструкцію нижніх дихальних шляхів і гостру недостатність дихання?

A. Хемотаксичні фактори

B. Гістамін, лейкотрієни

C. Цитокіни

D. Комплемент

E. Катехоламіни


393. У хворого на інфаркт міокарда з’явилися клінічні ознаки кардіогенного шоку. Який із вказаних нижче патогенетичних механізмів є первинним у розвитку шоку?

A. Депонування крові у венах

B. Зниження насосної функції серця

C. Зниження діастолічного притоку крові до серця

D. Збільшення судинного тонусу

E. Зниження судинного тонусу


394. Хворий, 52 років, був доставлений в клініку з симптомами гострого панкреатиту та явищами панкреатичного шоку. Що є основним механізмом розвитку панкреатичного шоку?

A. Надходження активованих протеаз у кров

B. Стимулювання панкреатичної секреції

C. Порушення відтоку панкреатичного соку

D. Зменшення інкреції інсуліну

E. Підвищення артеріального тиску


395. Хворий, 52 років, був доставлений в клініку з симптомами гострого панкреатиту та явищами панкреатичного шоку. Активація якої системи є причиною гіпотензії при панкреатичному шоці?

A. Калікреїн-кінінової

B. Ренін-ангіотензинової

C. Фібринолізу

D. Антикоагулянтної

E. Глутатіонової


396. Під час легеневої кровотечі у хворого на туберкульоз пацієнт втратив 45 % загального об’єму крові. Яке ускладнення, найбільш вірогідно, може виникнути у хворого?

A. Постгеморагічний шок

B. Анафілактичний шок

C. Больовий шок

D. Ішемія мозку

E. Інфаркт міокарду


397. Під час легеневої кровотечі у хворого на туберкульоз пацієнт втратив 45 % загального об’єму крові. Який із вказаних нижче патогенетичних механізмів є первинним у розвитку постгеморагічного шоку?

A. Депонування крові у венах

B. Зниження насосної функції серця

C. Гіповолемія

D. Збільшення судинного тонусу

E. Зниження судинного тонусу


398. У хворого з опіками 40 % поверхні тіла розвинувся опіковий шок. Який механізм розвитку шоку домінує на його початку?

A. Больовий

B. Порушення білкового обміну

C. Аутоімунізація

D. Порушення мінерального обміну

E. Зневоднення


399. Потерпілого з джутом на стегні привезли через 3 години після травми в задовільному стані. Після зняття джгута стан хворого різко погіршився: з'явився різкий набряк тканини стегна, частий пульс, холодний піт, артеріальний тиск знизився до 80/60 мм рт.ст. Яке ускладнення розвинулось?

A. Анафілактичний шок

B. Токсемічний шок

C. Кардіогенний шок

D. Геморагічний шок

E. Колапс


400. Масивне тривале розчавлювання м'яких тканин потерпілого в результаті шахтного вибуху призвело до появи у хворого олігурії, гіпоізостенурії, протеїнурії, міоглобінурії, гіперкаліємії, гіпонатріємії. Який провідний механізм порушення функції нирок у цьому випадку?

A. Больове подразнення

B. Порушення симпатичної нервової системи

C. Викид катехоламінів

D. Втрата білка

E. Розвиток токсемії


 

1. A 26 year old pregnant woman is under treatment at an in-patient hospital.

After a continuous attack of vomiting she was found to have reduced volume of circulating blood. What kind of change in general blood volume is the case?

A. Polycythemic hypovolemia

B. Simple hypovolemia

C. Oligocythemic hypovolemia

D. Polycythemic hypervolemia

E. Oligocythemic hypervolemia

2. A blood drop has been put into a test tube with 0,3% solution of NaCl.

What will happen to erythrocytes?

A. Osmotic haemolysis

B. Shrinkage

C. Mechanical haemolysis

D. Any changes will be observed

E. Biological haemolysis

3. In a dysentery patient undergoing treatment in the contagious isolation ward, a significant increase in packed cell volume has been observed (60%).

What other value will be affected by this change?

A. Increasing blood viscosity

B. Increasing volume of blood circulation

C. Leukopenia

D. Thrombocytopenia

E. Increasing erythrocyte sedimentation rate (ESR)

4. During postembryonal haemopoiesis in the red bone marrow the cells of one of the cellular differons demonstrate a gradual decrease in cytoplasmic basophilia as well as an increase in oxyphilia, the nucleus is being forced out. Such morphological changes are typical for the following haemopoiesis type:

A. Erythropoiesis

B. Lymphopoiesis

C. Neutrophil cytopoiesis

D. Eosinophil cytopoiesis

E. Basophil cytopoiesis

5. In a car accident a man got injured and lost a lot of blood. What changes in peripheral blood are most likely to occur on the 2nd day after the injury?

A. Erythropenia

B. Hypochromia

C. Anisocytosis

D. Microplania

E. Significant reticulocytosis

6. Examination of a 52-year-old female patient has revealed a decrease in the amount of red blood cells and an increase in free hemoglobin in the blood plasma (hemoglobinemia).Color index is 0,85. What type of anemia is being observed in the patient?

A. Acquired hemolytic

B. Hereditary hemolytic

C. Acute hemorrhagic

D. Chronic hemorrhagic

E. Anemia due to diminished erythropoiesis

7. A 37-year-old female patient complains of headache, vertigo, troubled sleep, numbness of limbs. For the last 6 years she has been working at the gas-discharge lamp-producing factory in the lead-processing shop.

Blood test findings: low hemoglobin and RBC level, serum iron concentration exceeds the norm by several times.Specify the type of anemia:

A. Iron refractory anemia

B. Iron-deficiency anemia

C. Minkowsky-Shauffard disease

D. Hypoplastic anemia

E. Metaplastic anemia

8. A patient is diagnosed with irondeficiency sideroachrestic anemia, progression of which is characterized by skin hyperpigmentation, pigmentary cirrhosis, heart and pancreas affection. Iron level in the blood serum is increased. What disorder of iron metabolism causes this disease?

A. Failure to assimilate iron leading to iron accumulation in tissues

B. Excessive iron intake with food

C. Disorder of iron absorption in bowels

D. Increased iron assimilation by body

E. -

9. Degenerative changes in posterior and lateral columns of spinal cord (funicular myelosis) caused by methylmalonic acid accumulation occur in patients with B12-deficiency anemia. This results in synthesis disruption of the following substance:

A. Myelin

B. Acetylcholine

C. Norepinephrine

D. Dopamine

E. Serotonin

10. Blood test of a patient suffering from atrophic gastritis gave the following results: RBCs - 2, 0 • 1012 /l, Hb- 87 g/l, colour index - 1,3, WBCs - 4, 0 • 109/l, thrombocytes - 180 • 109/l. Anaemia migh have been caused by the following substance deficiency:

A. Vitamin B12

B. Vitamin A

C. Vitamin K

D. Iron

E. Zinc

11. A patient suffering from chronic myeloleukemia has got the following symptoms of anemia: decreased number of erythrocytes and low haemoglobin concentration, oxyphilic and polychromatophilic normocytes, microcytes. What is the leading pathogenetic mechanism of anemia development?

A. Substitution of haemoblast

B. Intravascular hemolysis of erythrocytes

C. Deficiency of vitamin B12

D. Reduced synthesis of erythropoietin

E. Chronic haemorrhage

12. On the fifth day after the acute blood loss a patient has been diagnosed with hypochromic anemia. What is the main mechanism of hypochromia development?

A. Release of immature red blood cells from the bone marrow

B. Impaired iron absorption in the intestines

C. Increased destruction of red blood cells in the spleen

D. Impaired globin synthesis

E. Increased excretion of body iron

13. A 19-year-old female patient has had low haemoglobin rate of 90-95 g/l since

childhood. Blood count results obtained after hospitalisation are as follows:erythrocytes - 3, 2 • 1012/l, Hb- 85 g/l, colour index - 0,78; leukocytes - 5, 6 • 109/l,

platelets - 210 • 109/l. Smear examination revealed anisocytosis, poikilocytosis and target cells. Reticulocyte rate is 6%. Iron therapy was ineffective. What blood

pathology corresponds with the described

clinical presentations?

A. Thalassemia

B. Enzymopathy

C. Membranopathy

D. Sickle-cell anemia

E. Favism

14. Along with normal hemoglobin types there can be pathological ones in the organism of an adult. Name one of them:

A. HbS

B. HbF

C. HbA1

D. HbA2

E. HbO2

Hemostasis disorder.

14. A patient is diagnosed with hereditary coagulopathy that is characterized by factor VIII deficiency. Specify the phase of blood clotting during which coagulation will be disrupted in the given case:

A. Thromboplastin formation

B. Thrombin formation

C. Fibrin formation

D. Clot retraction

E. -

15. After a tourniquet application a patient was found to have petechial haemorrhages. The reason for it is the dysfunction of the following cells:

A. Platelets

B. Eosinophils

C. Monocytes

D. Lymphocytes

E. Neutrophils

16. After implantation of a cardiac valve a young man constantly takes indirect anticoagulants. His state was complicated by hemorrhage. What substance content has

decreased in blood?

A. Prothrombin

B. Haptoglobin

C. Heparin

D. Creatin

E. Ceruloplasmin

17. A 3-year-old boy with pronounced hemorrhagic syndrome doesn’t have antihemophilic globulin A (factor VIII) in the blood plasma. Hemostasis has been impaired at the following stage:

A. Internal mechanism of prothrombinase activation

B. External mechanism of prothrombinase activation

C. Conversion of prothrombin to thrombin

D. Conversion of fibrinogen to fibrin

E. Blood clot retraction

18. A 12-year-old patient has been admitted to a hospital for hemarthrosis of the knee joint. From early childhood he suffers from frequent bleedings. Diagnose the boy’s disease:

A. Hemophilia

B. Hemorrhagic vasculitis

C. Hemolytic anemia

D. B12 (folic acid)-deficiency anemia

E. Thrombocytopenic purpura

19. After pancreatic surgery the patient developed hemorrhagic syndrome with disturbed 3rd stage of blood clotting. What will be the most likely mechanism of the hemostatic disorder?

A. Fibrinolysis activation

B. Decrease of prothrombin synthesis

C. Decrease of fibrinogen synthesis

D. Qualitative abnormalities of fibrinogenesis

E. Fibrin-stabilizing factor deficiency

Leukocytosis and leukemia.

1. A 3-year-old child had eaten some strawberries. Soon he developed a rash and itching. What was found in the child’s leukogram?

A. Eosinophilia

B. Hypolymphemia

C. Neutrophilic leukocytosis

D. Monocytosis

E. Lymphocytosis

2. After an attack of bronchial asthma a patient had his peripheral blood tested.

What changes can be expected?

A. Eosinophilia

B. Leukopenia

C. Lymphocytosis

D. Thrombocytopenia

E. Erythrocytosis

3. As a result of a road accident a 26- year-old man is in the torpid phase of shock. Blood count: leukocytes - 3, 2 • 109/l. What is the leading mechanism of leukopenia development?

A. Leukocyte redistribution in the bloodstream

B. Leukopoiesis inhibition

C. Faulty release of mature leukocytes from the bone marrow into the blood

D. Leukocyte destruction in the hematopietic organs

E. Increased excretion of the leukocytes from the organism

4. A 26-year-old man is in the torpid shock phase as a result of a car accident.

In blood: 3, 2 • 109/l. What is the leading mechanism of leukopenia development?

A. Redistribution of leukocytes in bloodstream

B. Leikopoiesis inhibition

C. Disturbed going out of mature leukocytes from the marrow into the blood

D. Lysis of leukocytes in the blood-forming organs

E. Intensified elimination of leukocytes from the organism

5. A 59-year-old woman has been hospialized in a surgical ward due to exacerbation of chronic osteomyelitis of the left shin. Blood test: leukocytes - 15, 0 • 109/l. Leukogram: myelocytes - 0%, metamyelocytes - 8%, stab neutrophils - 28%, segmented neutrophils - 32%, lymphocytes - 29%, monocytes - 3%. Such blood count would be called:

A. Regenerative left shift

B. Right shift

C. Hyperregenerative left shift

D. Degenerative left shift

E. Regenerative-degenerative left shift

Pathophysiology of the systemic blood circulation

Cardiac insufficiency.

20. A patient suffering from stenocardia was taking nitroglycerine which caused restoration of blood supply of myocardium and relieved pain in the cardiac area. What intracellular mechanism provides restoration of energy supply of insulted cells?

A. Intensification of ATP resynthesis

B. Reduction of ATP resynthesis

C. Increased permeability of membranes

D. Intensification of oxygen transporting into the cell

E. Intensification of RNA generation

21. Experimental stimulation of the sympathetic nerve branches that innervate the heart caused an increase in force of heart contractions because the membrane of typical cardiomyocytes permitted an increase in:

A. Calcium ion entry

B. Calcium ion exit

C. Potassium ion exit

D. Potassium ion entry

E. Calcium and potassium ion exit

22. For biochemical diagnostics of myocardial infarction it is necessary to measure activity of a number of enzymes and their isoenzymes. What enzymatic test is considered to be the best to prove or disprove the diagnosis of infarction in the early period after the chest pain is detected?

A. Creatine kinase isoenzyme CK-MB

B. Creatine kinase isoenzyme CK-MM

C. LDH1 lactate dehydrogenase isoenzyme

D. LDH2 lactate dehydrogenase isoenzyme

E. Aspartate aminotransferase cytoplasmic isoenzyme

23. A 50 year old patient suffers from essential hypertension. After a physical stress he experienced muscle weakness, breathlessness, cyanosis of lips, skin and face. Respiration was accompanied by distinctly heard bubbling rales. What mechanism underlies the development of this syndrome?

A. Acute left-ventricular failure

B. Chronic right-ventricular failure

C. Chronic left-ventricular failure

D. Collapse

E. Cardiac tamponade

24. After a serious psychoemotional stress a 48 year old patient suddenly developed acute heart ache irradiating to the left arm. Nitroglycerine relieved pain after 10minutes. What is the leading pathogenetic mechanism of this process development?

A. Spasm of coronary arteries

B. Dilatation of peripheral vessels

C. Obstruction of coronary vessels

D. Compression of coronary vessels

E. Increase in myocardial oxygen consumption

25. The patient with acute myocardial infarction was given intravenously different solutions during 8 hours with medical dropper 1500 ml and oxygen intranasally. He died because of pulmonary edema. What caused the pulmonary edema?

A. Volume overload of the left ventricular

B. Decreased oncotic pressure due to hemodilution

C. Allergic reaction

D. Neurogenic reaction

E. Inhalation of the oxygen

26. A 35-year-old male developed acute heart failure while running for a long time.

What changes in the ionic composition can be observed in the cardiac muscle?

A. Accumulation of Na+ and Ca2+ ions in the myocardium cells

B. Accumulation of K+ and Mg2+ ions in the myocardium cells

C. Reduction of Na+ and Ca2+ ions in the myocardium cells

D. Reduction of K+ and Mg2+ ions in the extracellular space

E. Reduction of Na+ and Ca2+ ions in the extracellular space

27. A patient with extensive myocardial infarction has developed heart failure.

What pathogenetic mechanism contributed to the development of heart failure in the patient?

A. Reduction in the mass of functioning myocardiocytes

B. Pressure overload

C. Volume overload

D. Acute cardiac tamponade

E. Myocardial reperfusion injury

28. ECG of a 44-year-old patient shows of hypertrophy of both ventricles and the right atrium. The patient was diagnosed with the tricuspid valve insufficiency.

What pathogenetic variant of cardiac dysfunction is usually observed in case of such insufficiency?

A. Heart overload by volume

B. Heart overload by resistance

C. Primary myocardial insufficiency

D. Coronary insufficiency

E. Cardiac tamponade

29. A patient who had been continuously taking drugs blocking the production of angiotensin II developed bradycardia and arrhythmia. A likely cause of these disorders is:

A. Hyperkalemia

B. Hypokalemia

C. Hypernatremia

D. Hypocalcemia

E. Hypercalcemia

30. An isolated cell of human heart automatically generates excitement impulses with frequency of 60 times per minute. This cell was taken from the following heart structure:

A. Sinoatrial node

B. Atrium

C. Ventricle

D. Atrioventricular node

E. His’ bundle

31. Heart rate of a 30-year-old man under emotional stress reached 112 bpm. The reason for the heart rate increase is the altered condition of the following conducting system of heart:

A. Sinoatrial node

B. Purkinje’s fibers

C. His’ bundle branches

D. Atrioventricular node

E. His’ bundle

32. Since a patient has had myocardial infarction, his atria and ventricles contract independently from each other with a frequency of 60-70 and 35-40 per minute. Specify the type of heart block in this case:

A. Complete atrioventricular

B. Partial atrioventricular

C. Sino-atrial

D. Intra-atrial

E. Intraventricular

33. A patient complains of palpitation after stress. The pulse is 104 bpm, P-Q=0,12 seconds, there are no changes of QRS complex. What type of arrhythmia does the patient have?

A. Sinus tachycardia

B. Sinus bradycardia

C. Sinus arrhythmia

D. Ciliary arrhythmia

E. Extrasystole

34. A 67 year old patient complains of periodic heart ache, dyspnea during light physical activities. ECG reveals extraordinary contractions of heart ventricles. Such arrhythmia is called:

A. Extrasystole

B. Bradycardia

C. Tachycardia

D. Flutter

E. Fibrillation

35. A 45-year-old patient was admitted to the cardiological department. ECG data:

negative P wave overlaps QRS complex, diastolic interval is prolonged after

extrasystole. What type of extrasystole is it?

A. Atrioventricular

B. Sinus

C. Atrial

D. Ventricular

E. Bundle-branch

36. Analysis of the ECG revealed the missing of several PQRST cycles. The remaining waves and complexes are not changed. Specify the type of arrhythmia:

A. Sinoatrial block

B. Atrial fibrillation

C. Atrioventricular block

D. Atrial premature beat

E. Intra-atrial block

37. ECG of a patient displays an abnormally long R wave (up to 0,18s). This is caused by a decrease in the conduction velocity of the following heart structures:

A. Ventricles

B. Atria

C. Atrio-ventricular node

D. Right ventricle

E. Left ventricle

38. During ventricular systole, the cardiac muscle does not respond to additional stimulation because it is in the phase of:

A. Absolute refractoriness

B. Relational refractoriness

C. Hyperexcitability

D. Subnormal excitability

E. There is no correct answer

39. A patient complaining of pain in the left shoulder-blade region has been diagnosed with miocardial infarction. What kind of pain does the patient have?

A. Radiating

B. Visceral

C. Phantom

D. Protopathic

E. Epicritic

39/. A woman, who has been suffering from marked hypertension for 15 years, has lately developed dyspnea, palpitations, slightly decreased systolic pressure, while diastolic pressure remains the same What is the main mechanism of heart failure development in this case?

A. Cardiac overload due to increased vascular resistance

B. Cardiac overload due to increased blood volume

C. Damage to the myocardium

D. Disorder of impulse conduction in the myocardium

E. Dysregulation of cardiac function

Pathophysiology of vessels.

1. As a result of a home injury, a patient suffered a significant blood loss, which led

to a fall in blood pressure. Rapid blood pressure recovery after the blood loss is

provided by the following hormones:

A. Adrenaline, vasopressin

B. Cortisol

C. Sex hormones

D. Oxytocin

E. Aldosterone

2. A patient with constant headaches, pain in the occipital region, tinnitus, dizziness has been admitted to the cardiology department. Objectively:AP- 180/110mm

Hg, heart rate - 95/min. Radiographically, there is a stenosis of one of the renal arteries. Hypertensive condition in this patient has been caused by the activation of the following system:

A. Renin-angiotensin

B. Hemostatic

C. Sympathoadrenal

D. Kinin

E. Immune

3. A 43-year-old-patient has arterial hypertension caused by an increase in cardiac output and general peripheral resistance. Specify the variant of hemodynamic development of arterial hypertension in the given case:

A. Eukinetic

B. Hyperkinetic

C. Hypokinetic

D. Combined

E. -

4. A 16-year-old female patient has fainted after quickly changing her body position from horizontal to vertical one. Which process from the ones listed below has caused the loss of consciousness in the first place?

A. Decreasing venous return

B. Increasing venous return

C. Increasing central venous pressure

D. Decreasing oncotic pressure of blood plasma

E. Increasing arterial pressure

5. A patient has insufficient blood supply to the kidneys, which has caused the development of pressor effect due to the constriction of arterial resistance vessels. This is the result of the vessels being greately affected by the following substance:

A. Angiotensin II

B. Angiotensinogen

C. Renin

D. Catecholamines

E. Norepinephrine

6. A month after surgical constriction of rabbit’s renal artery the considerable increase of systematic arterial pressure was observed. What of the following regulation mechanisms caused the animal’s pressure change?

A. Angiotensin-II

B. Vasopressin

C. Adrenaline

D. Noradrenaline

E. Serotonin

Pathophysiology of the respiratory system.

1. A 23-year-old patient has been admitted to a hospital with a craniocerebral injury. The patient is in a grave condition.Respiration is characterized by prolonged convulsive inspiration followed by a short expiration. What kind of respiration is it typical for?

A. Apneustic

B. Gasping breath

C. Kussmaul’s

D. Cheyne-Stokes

E. Biot’s

2. A patient has increased thickness of alveolar-capillary membrane caused by a pathologic process. The direct consequence will be reduction of the following value:

A. Diffusing lung capacity

B. Oxygen capacity of blood

C. Respiratory minute volume

D. Alveolar ventilation of lungs

E. Expiratory reserve volume

3. A patient has a traumatic injury of sternocleidomastoid muscle. This has resulted in a decrease in the following value:

A. Inspiratory reserve volume

B. Expiratory reserve volume

C. Respiratory volume

D. Residual volume

E. Functional residual lung capacity

4. When studying the signs of pulmonary ventilation, reduction of forced expiratory volume has been detected.What is the likely cause of this phenomenon?

A. Obstructive pulmonary disease

B. Increase of respiratory volume

C. Increase of inspiratory reserve volume

D. Increase of pulmonary residual volume

E. Increase of functional residual lung capacity

5. A patient with bronchial asthma has developed acute respiratory failure. What kind of respiratory failure occurs in this case?

A. Obstructive disturbance of alveolar ventilation

B. Restrictive ventilatory defect

C. Perfusion

D. Diffusion

E. Dysregulation of alveolar ventilation

6. Analysis of the experimental spirogram of a 55-year-old person revealed a decrease in tidal volume and respiratory amplitude compared to the situation of ten years ago. The change in these indicators is caused by:

A. Decreased force of respiratory muscle contraction

B. Gas composition of the air

C. Physical build of a person

D. Height of a person

E. Body mass of a person

7. A female patient, having visited the factory premises with lots of dust in the air for the first time, has got cough and burning pain in the throat. What respiratory receptors, when irritated, cause this kind of reaction?

A. Irritant receptors

B. Juxtacapillary (J) receptors

C. Stretch receptors of lungs

D. Proprioceptors of respiratory muscles

E. Thermoreceptors

8. A patient under test was subjected to a moderate physical stress. His minute blood volume amounted 10 l/min. What blood volume was pumped through his lung vessels every minute?

A. 10 l/min

B. 5 l/min

C. 4 l/min

D. 6 l/min

E. 7 l/min

9. To assess the effectiveness of breathing in patients, the indicator of functional residual capacity is used. It includes the following volumes:

A. Expiratory reserve volume and residual volume

B. Inspiratory reserve volume and residual volume

C. Inspiratory reserve volume, tidal volume, residual volume

D. Expiratory reserve volume and tidal volume

E. Inspiratory reserve volume and tidal volume

10. Lung ventilation in a person is increased as a result of physical activity.

Which of the following indices of the external respiration is much higher than

in a state of rest?

A. Respiratory volume

B. Vital capacity of lungs

C. Inspiratory reserve volume

D. Expiratory reserve volume

E. Total lung capacity

11. A patient demonstrates sharp decrease of pulmonary surfactant activity. This condition can result in:

A. Alveolar tendency to recede

B. Decreased airways resistance

C. Decresed work of expiratory muscles

D. Increased pulmonary ventilation

E. Hyperoxemia

Pathophysiology of the liver and digestive system.

1. A patient has a critical impairment of protein, fat and hydrocarbon digestion.

Most likely it has been caused by low secretion of the following digestive juice:

A. Pancreatic juice

B. Saliva

C. Gastric juice

D. Bile

E. Intestinal juice

2. A 42 year old patient complains of pain in the epigastral area, vomiting; vomit masses have the colour of "coffee-grounds", the patient has also melena. Anamnesis records gastric ulcer. Blood formula: erythrocytes - 2, 8 • 1012/l, leukocytes - 8 • 109/l, Hb- 90 g/l. What complication is it?

A. Haemorrhage

B. Penetration

C. Perforation

D. Canceration

E. Pyloric stenosis

3. A patient takes cholagogues. What other process besides biliary excretion will be stimulated?

A. Intestinal motility

B. Gastric juice secretion

C. Pancreatic juice secretion

D. Gastric motor activity

E. Water absorption

4. A 49-year-old male patient with acute pancreatitis was likely to develop pancreatic necrosis, while active pancreatic proteases were absorbed into the blood stream and tissue proteins broke up. What protective factors of the body can inhibit these processes?

A. α2-macroglobulin, α1-antitrypsin

B. Immunoglobulin

C. Cryoglobulin, interferon

D. Ceruloplasmin, transferrin

E. Hemoplexin, haptoglobin

5. A 30 year old woman has subnormalconcentration of enzymes in the pancreatic juice. This might be caused by the hyposecretion of the following gastrointestinal hormone:

A. Cholecystokinin-pancreozymin

B. Somatostatin

C. Secretin

D. Gastro-inhibiting peptide

E. Vaso-intestinal peptide

6. A 60 year old patient was found to have a dysfunction of main digestive enzyme of saliva. This causes the disturbance of primary hydrolysis of:

A. Carbohydrates

B. Fats

C. Proteins

D. Cellulose

E. Lactose

7. A 30-year-old male patient with acute pancreatitis has been found to have a disorder of cavitary protein digestion. The reason for such condition can be the hyposynthesis and hyposecretion of the following enzyme:

A. Tripsin

B. Pepsin

C. Lipase

D. Dipeptidase

E. Amylase

8. A specimen of a parenchymal organ shows poorly delineated hexagonal lobules surrounding a central vein, and the interlobular connective tissue contains embedded triads (an artery, a vein and an excretory duct). What organ is it?

A. Liver

B. Pancreas

C. Thymus

D. Spleen

E. Thyroid

9. A 53-year-old male patient complains of acute pain in the right hypochondrium. Objective examination revealed scleral icterus. Laboratory tests revealed increased ALT activity, and stercobilin was not detected in the stool. What disease is characterized by these symptoms?

A. Cholelithiasis

B. Hemolytic jaundice

C. Hepatitis

D. Chronic colitis

E. Chronic gastritis

10. A patient has normally coloured stool including a large amount of free fatty acids.

The reason for this is a disturbance of the following process:

A. Fat absorption

B. Fat hydrolysis

C. Biliary excretion

D. Choleresis

E. Lipase secretion

11. A patient has been admitted to the contagious isolation ward with signs of jaundice caused by hepatitis virus. Which of the symptoms given below is strictly specific for hepatocellular jaundice?

A. Increase of ALT, AST level

B. Hyperbilirubinemia

C. Bilirubinuria

D. Cholemia

E. Urobilinuria

12. An infectious disease unit admitted a patient with signs of jaundice caused by

hepatitis virus. Select an indicator that is specific only for parenchymatous jaundice:

A. Increase in ALT and AST rate

B. Hyperbilirubinemia

C. Bilirubinuria

D. Cholaemia

E. Urobilinuria

13. A tooth extraction in a patient with chronic persistent hepatitis was complicated with prolonged hemorrhage. What is the reason for the haemorrhagic syndrome?

A. Decrease in thrombin production

B. Increase in thromboplastin production

C. Decrease in fibrin production

D. Increase in fibrinogen synthesis

E. Fibrinolysis intensification

14. After severe viral hepatitis a 4 year-old boy presents with vommiting, occasional loss of consciousness, convulsions. Blood test revealed hyperammoniemia. Such condition is caused by a disorder of the following biochemical hepatic process:

A. Disorder of ammonia neutralization

B. Disorder of biogenic amines neutralization

C. Protein synthesis inhibition

D. Activation of amino acid decarboxylation

E. Inhibition of transamination enzymes

15. Blood analysis of a patient with jaundice reveals conjugated bilirubinemia, increased concentration of bile acids. There is no stercobilinogen in urine.

What type of jaundice is it?

A. Obstructive jaundice

B. Hepatocellular jaundice

C. Parenchymatous jaundice

D. Hemolytic jaundice

E. Cythemolytic jaundice

16. A patient with jaundice has high total bilirubin that is mainly indirect (unconjugated), high concentration of stercobilin in the stool and urine. The level of direct (conjugated) bilirubin in the blood plasma is normal. What kind of jaundice can you think of?

A. Hemolytic

B. Parenchymal (hepatic)

C. Mechanical

D. Neonatal jaundice

E. Gilbert’s disease

17. A patient presents with icteritiousness of skin, scleras and mucous membranes. Blood plasma the total bilirubin is increased, stercobilin is increased in feces, urobilin is increased in urine. What type of jaundice is it?

A. Haemolytic

B. Gilbert’s disease

C. Parenchymatous

D. Obturational

E. Cholestatic

18. Enzymatic jaundices are accompanied by abnormal activity of UDPglucuronyl transferase. What compound is accumulated in blood serum in case of these pathologies?

A. Unconjugated bilirubin

B. Conjugated bilrubin

C. Dehydrobilirubin

D. Hydrobilirubin

E. Choleglobin

19. A 35-year-old man with peptic ulcer disease has undergone antrectomy. After the surgery secretion of the following gastrointestinal hormone will be disrupted the most:

A. Gastrin

B. Histamine

C. Secretin

D. Cholecystokinin

E. Neurotensin

20. A patient presents with steatorrhea. This disorder can be linked to disturbed supply of the intestine with the following substances:

A. Bile acids

B. Carbohydrates

C. Tripsin

D. Chymotrypsin

E. Amylase

Pathophysiology of the kidneys

1. Urine analysis has shown high levels of protein and erythrocytes in urine. This can be caused by the following:

A. Renal filter permeability

B. Effective filter pressure

C. Hydrostatic blood pressure in glomerular capillaries

D. Hydrostatic primary urine pressure in capsule

E. Oncotic pressure of blood plasma

2. Diabetic nephropathy with uremia has developed in a patient with pancreatic diabetes. The velocity of glomerular filtration is 9 ml/min. What mechanism of a decrease in glomerular filtration velocity and chronic renal failure development is most likely in the case of this patient?

A. Reduction of active nephron mass

B. Decrease in systemic arterial pressure

C. Obstruction of nephron tubules with hyaline casts

D. Tissue acidosis

E. Arteriolar spasm

3. According to the results of glucose tolerance test, the patient has no disorder of carbohydrate tolerance. Despite that, glucose is detected in the patients’ urine (5 mmol/l). The patient has been diagnosed with renal diabetes. What renal changes cause glucosuria in this case?

A. Decreased activity of glucose reabsorption enzymes

B. Increased activity of glucose reabsorption enzymes

C. Exceeded glucose reabsorption threshold

D. Increased glucose secretion

E. Increased glucose filtration

4. Due to the use of poor quality measles vaccine for preventive vaccination, a 1-year-old child has developed an autoimmune renal injury. The urine was found to contain macromolecular proteins. What process of urine formation has been disturbed?

A. Filtration

B. Reabsorption

C. Secretion

D. Reabsorption and secretion

E. Secretion and filtration

5. A child has an acute renal failure. What biochemical factor found in saliva can confirm this diagnosis?

A. Increase in urea concentration

B. Increase in glucose concentration

C. Decrease in glucose concentration

D. Increase in concentration of higher fatty acids

E. Decrease in nucleic acid concentration

6. A patient with a history of chronic glomerulonephritis presents with azotemia, oliguria, hypo- and isosthenuria, proteinuria. What is the leading factor in the pathogenesis of these symptoms development under chronic renal failure?

A. Mass decrease of active nephrons

B. Intensification of glomerular filtration

C. Tubular hyposecretion

D. Disturbed permeability of glomerular membranes

E. Intensification of sodium reabsorption

7. A histological specimen of kidney shows a structure consisting of a glomerulus of fenestrated capillaries and a bilayer epithelial capsule. Specify this structure:

A. Renal corpuscle

B. Proximal tubule

C. Distal tubule

D. Henle’s loop

E. Receiving tube

8. A 4 year old child with hereditary renal lesion has signs of rickets, vitamin D concentration in blood is normal. What is the most probable cause of rickets development?

A. Impaired synthesis of calcitriol

B. Increased excretion of calcium

C. Hyperfunction of parathyroid glands

D. Hypofunction of parathyroid glands

E. Lack of calcium in food

9. 14 days after quinsy a 15-year-old child presented with morning facial swelling, high blood pressure, "meat slops"urine. Immunohistological study of a renal biopsy sample revealed deposition of immune complexes on the basement membranes of the capillaries and in the glomerular mesangium. What disease developed in the patient?

A. Acute glomerulonephritis

B. Acute interstitial nephritis

C. Lipoid nephrosis

D. Acute pyelonephritis

E. Necrotizing nephrosis

10. A patient with massive burns developed acute renal insufficiency characterized by a significant and rapid deceleration of glomerular filtration.What is the mechanism of its

development?

A. Reduction of renal blood flow

B. Damage of glomerular filter

C. Reduction of functioning nephron number

D. Rise of pressure of tubular fluid

E. Renal artery embolism

11. A male patient has been diagnosed with acute post-streptococcal glomerulonephritis. It is most likely that the lesion of the basement membrane of renal corpuscles was caused by the following allergic reaction:

A. Immune complex

B. Anaphylactic

C. Cytotoxic

D. Delayed

E. Stimulating

12. As a result of continuous starvation the glomerular filtration rate has increased by 20%. The most probable cause of the glomerular filtration alteration under the mentioned conditions is:

A. Decrease in the oncotic pressure of blood plasma

B. Increase in the systemic arterial pressure

C. Increase in the permeability of the renal filter

D. Increase of the filtartion quotient

E. Increase of the renal blood flow

13. A 12-year-old child developed nephritic syndrome (proteinuria, hematuria, cylindruria) 2 weeks after a case of tonsillitis, which is a sign of affected glomerular basement membrane in the kidneys. What mechanism is the most likely to cause the basement membrane damage?

A. Immune complex

B. Granulomatous

C. Antibody-mediated

D. Reaginic

E. Cytotoxic

15. Poisoning caused by mercury (II) chloride (corrosive sublimate) occurred in the result of safety rules violation. In 2 days the patient’s diurnal diuresis was 620 ml. The patient developed headache, vomiting, convulsions, dyspnea; moist crackles were observed in the lungs. Name this pathology:

A. Acute renal failure

B. Chronic renal failure

C. Uremic coma

D. Glomerulonephritis

E. Pyelonephritis

16. A man presents with glomerular fi- ltration rate of 180 ml/min., while norm is 125±25 ml/min. The likely cause of it is the decreased:

A. Plasma oncotic pressure

B. Effective filtration pressure

C. Hydrostatic blood pressure in the glomerular capillaries

D. Renal blood flow

E. Permeability of the renal filter

Pathophysiology of the endocrine system

1. Parodontitis is treated with calcium preparations and a hormone that stimulates tooth mineralization and inhibits tissue resorption. What hormone is it?

A. Calcitonin

B. Parathormone

C. Adrenalin

D. Aldosterone

E. Thyroxine

2. A 41-year-old male patient has a history of recurrent attacks of heartbeats (paroxysms), profuse sweating, and headaches. Examination revealed hypertension, hyperglycemia, increased basal metabolic rate, and tachycardia. These clinical presentations are typical for the following adrenal pathology:

A. Hyperfunction of the medulla

B. Hypofunction of the medulla

C. Hyperfunction of the adrenal cortex

D. Hypofunction of the adrenal cortex

E. Primary aldosteronism

3. In the course of an experiment adenohypophysis of an animal has been removed. The resulting atrophy of thyroid gland and adrenal cortex has been caused by deficiency of the following hormone:

A. Tropic hormones

B. Thyroid hormones

C. Somatotropin

D. Cortisol

E. Thyroxin

4. A patient with signs of osteoporosis and urolithiasis has been admitted to the endocrinology department. Blood test has revealed hypercalcemia and hypophosphatemia. These changes are associated with abnormal synthesis of the following hormone:

A. Parathyroid hormone

B. Calcitonin

C. Cortisol

D. Aldosterone

E. Calcitriol

5. A child has abnormal formation of tooth enamel and dentin as a result of low concentration of calcium ions in blood. Such abnormalities might be caused by deficiency of the following hormone:

A. Parathormone

B. Thyrocalcitonin

C. Thyroxin

D. Somatotropic hormone

E. Triiodothyronine

6. Atria of an experimental animal were superdistended by blood that resulted in decreased reabsorption of Na+ and water in renal tubules. This can be explained by the influence of the following factor upon kidneys:

A. Natriuretic hormone

B. Aldosterone

C. Renin

D. Angiotensin

E. Vasopressin

7. A 44 year old woman complains of general weakness, heart pain, significant increase of body weight. Objectively: moon face, hirsutism, AP is 165/100 mm Hg, height - 164 cm, weight - 103 kg; the fat is mostly accumulated on her neck, thoracic girdle, belly. What is the main pathogenetic mechanism of obesity?

A. Increased production of glucocorticoids

B. Reduced production of thyroid hormones

C. Increased insulin production

D. Reduced glucagon production

E. Increased mineralocorticoid production

8. Examination of a 42 year old patient revealed a tumour of adenohypophysis. Objectively: the patient’s weight is 117 kg, he has moon-like hyperemic face, redblue striae of skin distension on his belly. Osteoporosis and muscle dystrophy are present. AP is 210/140 mm Hg. What is the most probable diagnosis?

A. Cushing’s disease

B. Cushing’s syndrome

C. Conn’s disease

D. Diabetes mellitus

E. Essential hypertension

9. A 38-year-old female patient complains of general weakness, cardiac pain, increased appetite, no menstruation. Objectively: the height is 166 cm, weight 108 kg, the patient has moon-shaped face, subcutaneous fat is deposited mainly in the upper body, torso and hips. There are also blood-red streaks. Ps- 62/min, AP-160/105 mm Hg. Which of the following diseases is the described pattern of obesity most typical for?

A. Cushing pituitary basophilism

B. Alimentary obesity

C. Myxedema

D. Insulinoma

E. Babinski-Frohlich syndrome

10. A 19-year-old male was found to have an elevated level of potassium in the secondary urine. These changes might have been caused by the increase in the following hormone level:

A. Aldosterone

B. Oxytocin

C. Adrenaline

D. Glucagon

E. Testosterone

11. The patient with complaints of permanent thirst applied to the doctor.

Hyperglycemia, polyuria and increased concentration of 17-ketosteroids in the urine were revealed. What disease is the most likely?

A. Steroid diabetes

B. Insulin-dependent diabetes mellitus

C. Myxoedema

D. Type I glycogenosis

E. Addison’s disease

12. A 20 year old patient complains of morbid thirst and huperdiuresis (up to 10 l daily). Glucose concentration in blood is normal but it is absent in urine. The patient has been diagnosed with diabetes insipidus. What hormonal drug is the most appropriate for management of this disorder?

A. Vasopressin

B. Cortisol

C. Thyroxin

D. Oxytocin

E. Insulin

13. A 30-year-old female exhibits signs of virilism (growth of body hair, balding temples, menstrual disorders). This condition can be caused by the overproduction of the following hormone:

A. Testosterone

B. Oestriol

C. Relaxin

D. Oxytocin

E. Prolactin

14. A 46-year-old patient suffering from the diffuse toxic goiter underwent resection of the thyroid gland. After the surgery the patient presents with appetite loss, dyspepsia, increased neuromuscular excitement. The body weight remained unchanged. Body temperature is normal. Which of the following has caused such a condition in this patient?

A. Reduced production of parathormone

B. Increased production of thyroxin

C. Increased production of calcitonin

D. Increased production of thyroliberin

E. Reduced production of thyroxin

15. The secretion of which hypophysial hormones will be inhibited after taking the oral contraceptives containing sex hormones?

A. Gonadotropic hormone

B. Vasopressin

C. Thyrotrophic hormone

D. Somatotropic hormone

E. Ocytocin

16. A female patient with bronchial asthma had taken prednisolone tablets (1 tablet 3 times a day) for 2 months. Due to a significant improvement of her condition the patient suddenly stopped taking it. What complication is likely to develop in this case?

A. Withdrawal syndrome

B. Cushing’s syndrome

C. Gastrorrhagia

D. Upper body obesity

E. Hypotension

17. A patient complains of polyuria (7 liters per day) and polydipsia. Examination reveals no disorders of carbohydrate metabolism. These abnormalities might be caused by the dysfunction of the following endocrine gland:

F. Neurohypophysis

G. Adenohypophysis

H. Islets of Langerhans (pancreatic islets)

I. Adrenal cortex

J. Adrenal medulla

18. For people adapted to high external temperatures profuse sweating is not accompanied by loss of large volumes of sodium chloride. This is caused by the effect the following hormone has on perspiratory glands:

A. Aldosterone

B. Vasopressin

C. Cortisol

D. Tgyroxin

E. Natriuretic

19. On examination the patient presents with hirsutism, moon-shaped face, stretch marks on the abdomen. BP is 190/100 mm Hg, blood glucose is 17,6 mmol/l. What pathology is such clinical presentation characteristic of?

A. Adrenocortical Hyperfunction

B. Hyperthyroidism

C. Hypothyroidism

D. Gonadal hypofunction

E. Hyperfunction of the insular apparatus

20. On examination the patient is found to have low production of adrenocorticotropic hormone. How would this affect production of the other hormones?

A. Decrease adrenocorticotropic hormones synthesis

B. Decrease hormone synthesis in the adrenal medulla

C. Decrease insulin synthesis

D. Increase sex hormones synthesis

E. Increase thyroid hormones synthesis

Pathophysiology of the nervous system

1. A patient complains that at the bare mention of the tragic events that once occurred in his life he experiences tachycardia, dyspnea and an abrupt rise in blood pressure. What structures of the CNS are responsible for these cardiorespiratory reactions in this patient?

A. Cerebral cortex

B. Cerebellum

C. Lateral hypothalamic nuclei

D. Specific thalamic nuclei

E. Quadrigemina of mesencephalon

2. As a result of a continuous chronic encephalopathy, a patient has developed spontaneous motions and a disorder of torso muscle tone. These are the symptoms of the disorder of the following conduction tract:

A. Tractus rubrospinalis

B. Tractus corticospinalis

C. Tractus corticonuclearis

D. Tractus spinothalamicus

E. Tractus tectospinalis

3. A patient complaining of pain in the left shoulder-blade region has been diagnosed with miocardial infarction. What kind of pain does the patient have?

A. Radiating

B. Visceral

C. Phantom

D. Protopathic

E. Epicritic

4. After the traumatic tooth extraction a patient is complaining of acute, dull, poorly-localized pain in gingiva, body temperature rise up to 37, 5oC. The patient has been diagnosed with alveolitis. Specify the kind of pain in this patient:

A. Protopathic

B. Epicritic

C. Visceral

D. Heterotopic

E. Phantom

5. A patient got a gunshot wound of hip which damaged the sciatic nerve. Any impact on the affected limb causes severe, excruciating pain. What mechanism of pain is most likely in this case?

A. Causalgic

B. Reflex

C. Phantom

D. Endorphin hypofunction

E. Enkephalin hypofunction

6. An animal has an increased tonus of extensor muscles. This is the result of enhanced information transmission to the motoneurons of the spinal cord through the following descending pathways:

A. Vestibulospinal

B. Medial corticospinal

C. Reticulospinal

D. Rubrospinal

E. Lateral corticospinal

7. During an animal experiment, surgical damage of certain brain structures has caused deep prolonged sleep. What structure is most likely to cause such condition, if damaged?

A. Reticular formation

B. Basal ganglion

C. Red nuclei

D. Hippocampus

E. Cerebral cortex

8. In a cat with decerebrate rigidity themuscle tone is to be decreased. This can be achieved by:

A. Destruction of the vestibular nuclei of Deiters

B. Stimulation of the otolithic vestibular receptors

C. Stimulation of the vestibular nuclei of Deiters

D. Stimulation of the vestibulocochlear nerve

E. Stimulation of the ampullar vestibular receptors

9. As a result of an injury, the integrity of the anterior spinal cord root was broken. Specify the neurons and their processes that had been damaged:

A. Axons of motor neurons

B. Motor neuron dendrites

C. Axons of sensory neurons

D. Dendrites of sensory neurons

E. Dendrites of association neurons

10. A patient got an injury of spinal marrow in a road accident that caused loss of tactile sensation, posture sense, vibration sense. What conduction tracts are damaged?

A. Fascicle of Goll and cuneate fascicle

B. Anterior spinocerebellar tract

C. Rubrospinal tract

D. Reticulospinal tract

E. Tectospinal tract

11. A patient presents with the following motor activity disturbances: tremor, ataxia and asynergia movements, dysarthria. The disturbances are most likely to be localized in:

A. Cerebellum

B. Basal ganglions

C. Limbic system

D. Brainstem

E. Medulla oblongata

12. A male with a lesion of one of the CNS parts has asthenia, muscular dystonia, balance disorder. Which CNS part has been affected?

A. Cerebellum

B. Black substance

C. Reticular formation

D. Red nuclei

E. Vestibular nuclei

13. Vegetative abnormalities in the sleep, heat regulation, all kinds of metabolism, diabetes insipidus are developing in the patient due to grouth of the tumour in the III ventricle of brain. Irritation of the nucleus of what part of the brain can cause these symptoms?

A. Hypothalamus

B. Cerebral peduncles (cruces cerebri)

C. Mesencephalic tegmentum

D. Pons cerebelli

E. Medulla

14. A 5-month-old boy was hospitalized for tonic convulsions. He has a lifetime history of this disease. Examination revealed coarse hair, thinned and fragile nails, pale and dry skin. In blood: calcium - 1,5 millimole/l, phosphor - 1,9 millimole/l. These changes are associated with:

A. Hypoparathyroidism

B. Hyperparathyroidism

C. Hyperaldosteronism

D. Hypoaldosteronism

E. Hypothyroidism

15. A 49-year old female patient has limitation of left limbs arbitrary movements.

Muscular tonus of left hand and leg is overstrained and spasmodic, local tendon reflexes are strong, pathological reflexes are presented. What is the most likely development mechanism of hypertension and hyperreflexia?

A. Reduction of descending inhibitory

B. Motoneuron activation induced by stroke

C. Activation of excitatory influence from the focus of stroke

D. Activation of synaptic transmission

E. Inhibition of cerebral cortex motoneurons

16. A patient underwent an extraction of a part of a CNS structures by medical indications. As a result of the extraction the patient developed atony, astasia, intention tremor, ataxy and adiadochokinesis. Which part of CNS structure had been extracted?

A. Cerebellum

B. Amygdaloid corpus

C. Hippocamp

D. Basal ganglions

E. Limbic system

17. A patient with hypertension has developed headache, tinnitus, vomiting, high BP up to 220/160 mm Hg. On examination: facial asymmetry on the right, volitional mobility is absent, increased tendon reflexes and muscle tone of extremities on the right. What motor disorder of nervous system occurred in this case?

A. Hemiplegia

B. Paraplegia

C.Tetraplegia

D. Hyperkinesis

E. Monoplegia

18. Parkinson’s disease is caused by disturbance of dopamine synthesis. What brain structure synthesizes this neurotransmitter?

A. Substantia nigra

B. Globus pallidus

C. Corpora quadrigemina

D. Red nuclei

E. Hypothalamus

Pathophysiology of extreme situation.

40. After a road accident a victim has tachycardia, arterial blood pressure 130/90 mm Hg, tachypnoe, the skin is pale and dry, excitation of central nervous system is observed. What shock stage is the patient most likely in?

A. Erectile

B. Terminal

C. Torpid

D. Preshock (compensation stage)

E. Agony

41. As a result of a trauma a patient has developed traumatic shock that led to the following disorders: AP is 140/90 mm Hg, Ps is 120 bpm. The patient is fussy, talkative, pale. Such state relates to the following shock phase:

A. Erectile

B. Latent period

C. Terminal

D. Torpid

E. -

42. A man has a considerable decrease in diuresis as a result of 1,5 l blood loss.

The primary cause of such diuresis disorder is the hypersecretion of the following

hormone:

A. Vasopressin

B. Corticotropin

C. Natriuretic

D. Cortisol

E. Parathormone

43. A 63 year old patient with collapse presentations was delivered to the emergency hospital. A physician has chosen noradrenalin against hypotension. What is its mechanism of action?

A. Activation of α1-adrenoreceptors

B. Activation of serotonin receptors

C. Activation of β-adrenoreceptors

D. Activation of dopamine receptors

E. Block of M-cholinoreceptors

44. A nurse accidentally injected a nearly double dose of insulin to a patient with diabetes mellitus. The patient lapsed into a hypoglycemic coma. What drug should be injected in order to help him out of coma?

A. Glucose

B. Lidase

C. Insulin

D. Somatotropin

E. Noradrenaline

45. A comatose patient was taken to the hospital. He has a history of diabetes mellitus. Objectively: Kussmaul breathing, low blood pressure, acetone odor of breath. After the emergency treatment the patient’s condition improved. What drug had been administered to the patient?

A. Insulin

B. Adrenaline

C. Isadrinum

D. Glibenclamide

E. Furosemide

46. An unconscious patient was taken by ambulance to the hospital. On objective examination the patient was found to have no reflexes, periodical convulsions, irregular breathing. After laboratory examination the patient was diagnosed with hepatic coma. Disorders of the central nervous system develop due to the accumulation of the following metabolite:

A. Ammonia

B. Urea

C. Glutamine

D. Bilirubin

E. Histamine

47. A patient has been diagnosed with influenza. His condition became drastically worse after taking antipyretic drugs. His consciousness is confused, AP is 80/50 mm Hg, Ps is 140/m, body temperature droped down to 35, 8oC.What complication developed in this patient?

A. Collapse

B. Hyperthermia

C. Hypovolemia

D. Acidosis

E. Alkalosis

48. An adult man presents with systemic arterial pressure drop from 120/70 to 90/50 mm Hg. This resulted in reflex vasoconstriction. Vasoconstriction will be minimal in the following organ:

A. Heart

B. Skin

C. Bowels

D. Skeletal muscles

E. Liver

 

1. У хворого на первинний гіперальдостеронізм під-вищений артеріальний тиск крові. Кількість гемо-глобіну, еритроцитів, лейкоцитів і тромбоцитів у крові в нормі, гематокрит – 0,32. Яке порушення загального об’єму крові має місце в пацієнта?

A. @Гіперволемія олігоцитемічна

B. Гіперволемія поліцитемічна

C. Гіповолемія поліцитемічна

D. Гіповолемія нормоцитемічна

E. Гіперволемія нормоцитемічна

2. У дитини з опіковою хворобою (площа опіків 20 % поверхні тіла), гематокрит складає 0,65. Яке по-рушення загального об’єму крові має місце в да-ному випадку?

A. @Гіповолемія поліцитемічна

B. Гіповолемія олігоцитемічна

C. Гіперволемія олігоцитемічна

D. Гіповолемія нормоцитемічна

E. Гіперволемія нормоцитемічна

3. У хворого на залізодефіцитну анемію гематокрит складає 0,31. Як можна охарактеризувати загаль-ний об’єм крові в даному випадку?

A. @Ізоволемія олігоцитемічна

B. Гіповолемія поліцитемічна

C. Гіперволемія поліцитемічна

D. Гіповолемія нормоцитемічна

E. Гіперволемія нормоцитемічна

4. При пораненні пацієнт втратив 15 % об’єму крові. Який з нижченазваних механізмів швидко компен-сує крововтрату?

A. @Надходження тканинної рідини до судин

B. Посилення еритропоезу

C. Перехід на мегалобластичний еритропоез

D. Вихід еритробластів з кісткового мозку

E. Посилення лейкопоезу

5. При пораненні пацієнт втратив 15 % об’єму крові. Який з нижченазваних механізмів забезпечує від-новлення кисневої ємності крові?

A. @Нормобластичний еритропоез

B. Мегалобластичний еритропоез

C. Синтез гемоглобіну S

D. Тахікардія

E. Надходження тканинної рідини до судин

6. При пораненні пацієнт втратив 10 % об’єму крові. Який механізм може швидко усунути (зменшити) ступінь гіповолемії при цьому?

A. @Надходження тканинної рідини до судин

B. Активація еритропоезу

C. Перехід на мегалобластичний еритропоез

D. Збільшення частоти дихання

E. Збільшення в крові гаптоглобіну

7. При пораненні пацієнт втратив 15 % об’єму крові. Який з нижченазваних механізмів швидко норма-лізує артеріальний тиск?

A. @Спазм кровоносних судин

B. Посилення еритропоезу

C. Викид еритроцитів з депо

D. Брадикардія

E. Збільшення ШОЕ

8. При пораненні пацієнт втратив 15 % об’єму крові. Який з нижченаведених механізмів швидко ком-пенсує гіпоксію при цьому?

A. @Збільшення хвилинного об’єму крові

B. Зниження артеріального тиску крові

C. Посилення еритропоезу

D. Надходження тканинної рідини до судин

E. Збільшення синтезу альбумінів в печінці

9. На 7 добу після гострої шлункової кровотечі кіль-кість ретикулоцитів в крові хворого складає 8%. Це свідчить про:

A. @Збільшенні еритропоезу в кістковому мозку

B. Запалення в шлунку

C. Пухлинну трансформацію еритробластів

D. Дефіцит вітаміну В12

E. Алергію

10. Хворий скаржится на біль в животі, стул з доміш-ками крові, загальну слабкість. При обстеженні виявлено карциному кишки і анемію: Hb – 4,4 ммоль/л, Er – 2,8 Т/л, КП – 0,7. У мазку крові: анізоцитоз, пойкілоцитоз, 2% поліхроматофільних еритроцитів. Яка анемія в хворого?

A. @Хронічна постгеморагічна

B. Фолієво дефіцитна

C. В12 – дефіцитна

D. Спадкова гемолітична

E. Набута гемолітична

11. У пацієнта аналіз крові: еритроцити 7 Т/л, гемо-глобін 12 ммоль/л, лейкоцити 7,0 Г/л, тромбоцити 200 Г/л. Охарактеризуйте зміни в крові пацієнта:

A. @Еритроцитоз

B. Анемія

C. Лейкопенія

D. Лейкоцитоз

E. Тромбоцитоз

12. У хворого на цироз печінки діагностовано гіперх-ромну анемію. Які клітини периферійної крові є маркерами патологічного еритропоезу при цьому?

A. @Мегалоцити

B. Нормобласти

C. Ретикулоцити

D. Анулоцити

E. Мікроцити

13. На шостому місяці вагітності в жінки розвинулася залізодефіцитна анемія. Які еритроцити перифе-ричної крові вказують про нестачу гемоглобіну?

A. @Гіпохромні

B. Нормохромні

C. Гіперхромні

D. Мегалоцити

E. Мегалобласти

14. У вагітної жінки виявлено залізодефіцитну ане-мію. Які еритроцити свідчать про малий вміст ге-моглобіну при цьому?

A. @Анулоцити

B. Ретикулоцити

C. Мегалоцити

D. Мегалобласти

E. Нормобласти

15. У хворого на анемію в крові збільшена кількість клітин, що свідчать про активність фізіологічних механізмів регенерації «червоної» крові. Які це клітини?

A. @Поліхроматофіли

B. Еритробласти

C. Пойкілоцити

D. Мікроцити

E. Мегалоцити

16. Через тиждень після втрати 15 % об’єму крові у пацієнта зменшилися вміст еритроцитів, гемогло-біну, гематокрит, кольоровий показник; ретику-лоцити складають 10%. Яка анемія виникла у хворого?:

A. @Регенераторна

B. Апластична

C. Мегалобластна

D. Гіперхромна

E. Гіпорегенераторна

17. При анеміях в периферичній крові виявляються регенеративні, дегенеративні і патологічні форми еритроцитів. Назвіть дегенеративну форму ерит-роцитів.

A. @Анулоцити

B. Ретикулоцити

C. Нормобласти ацидофільні

D. Поліхроматофіли

E. Нормобласти поліхроматофільні

18. При анеміях в периферичній крові виявляються регенеративні, дегенеративні і патологічні форми еритроцитів. Назвіть регенеративну форму ерит-роцитів.

A. @Поліхроматофільні еритроцити

B. Пойкілоцити

C. Анізоцити

D. Анулоцити

E. Сфероцити

19. Після хіміотерапії у пацієнта: гемоглобін 5,0 ммоль/л, еритроцити 2,5 Т/л, КП 1,0, ретикулоци-ти 0,01%. Визначте анемію в хворої:

A. @Арегенераторна

B. Гіпохромна

C. Гіперхромна

D. Регенераторна

E. Залізодефіцитна

20. Для з’ясування стану кісткового мозку при анемі-ях бажано оцінити його регенераторну здатність. Визначення кількості яких клітин в крові дозволяє це зробити?

A. @Ретикулоцити, нормобласти

B. Мікроцити, макроцити

C. Овалоцити, ехіноцити

D. Анулоцити

E. Дрепаноцити

21. Привертає увагу лимонний відтінок шкіри у паціє-нтки. У крові хворої: регенеративна анемія; вміст непрямого білірубіну втричі перевищує норму; «біла кров» не змінена. Для яких захворювань си-стеми крові така картина є типовою:

A. @Гемолітична анемія

B. Залізодефіцитна анемія

C. Мегалобластні анемії

D. Апластична анемія

E. Хронічний лейкоз

22. Пацієнт скаржиться на задишку та жовтуватий колір шкіри. У крові хворого зменшено вміст ери-троцитів, гемоглобіну, збільшені вміст непрямого білірубіну та кількість ретикулоцитів. Зазначте механізм анемії в хворого:

A. @Гемоліз еритроцитів

B. Крововтрата гостра

C. Крововтрата хронічна

D. Порушення мітозу

E. Порушення синтезу гемоглобіну

23. Пацієнт страждає на задишку, його склери та шкіра мають жовтий відтінок. У крові зменшено вміст гемоглобіну, еритроцитів, КП 0,9, ретикуло-цитів 4%; кількість лейкоцитів і тромбоцитів в межах норми. Що може бути причиною такого стану крові пацієнта?

A. @Гемоліз

B. Опромінення

C. Дефіцит фолієвої кислоти

D. Дефіцит заліза

E. Вживання штучних андрогенів

24. У новонародженої дитини виражена жовтяниця, в крові зменшено вміст гемоглобіну і еритроцитів. Імунна реакція якого типу (за Кумбсом і Джелом) є головною ланкою в патогенезі гемолітичної хво-роби новонароджених?

A. @II типу (цитотоксична гуморальна)

B. I типу (анафілактична, локальна)

C. I типу (анафілактична, системна)

D. III типу (імунокомплексна)

E. V типу (стимулююча)

25. Пацієнтка скаржиться на напади болю в спині, які завершуються появою сечі бурого кольору; об-стеження виявило пароксизмальну нічну гемогло-бінурію. Надайте патогенетичну характеристику анемії в хворої:

A. @Гемолітична набута

B. В12–дефіцитна

C. Постгеморагічна

D. Фолієводефіцитна

E. Залізодефіцитна

26. Точкова мутація гена β–ланцюга гемоглобіну є причиною анемії. Вкажіть типову для даної анемії патологічну форму гемоглобіну.

A. @Hb S

B. Hb A2

C. Hb A1

D. Hb F

E. Барт–Hb

27. У хворої на системний червоний вовчак в крові визначаються антиеритроцитарні антитіла, змен-шення кількості еритроцитів і гемоглобіну. Дайте характеристику цієї анемії:

A. @Гемолітична, імунна

B. Гемолітична, токсична

C. Гемолітична, спадкова

D. Постгеморагічна, набута

E. Дисеритропоетична, дисрегуляторна

28. Лікування гіпохромної анемії препаратами заліза було неефективним у пацієнта. З дефіцитом якого вітаміну може бути пов'язаний розвиток залізо-рефратерної анемії?

A. @В6

B. В12

C. Фолієвої кислоти

D. В1

E. А

29. Через деякий час після резекції пухлини тонкої кишки в хворого виявлено фолієводефіцитну ане-мію. Які клітини є типовими в периферичній крові при цьому?

A. @Мегалобласти

B. Нормобласти

C. Еритробласти

D. Мікроцити

E. Анулоцити

30. У хворого лікування гіпохромної анемії препара-тами заліза не було ефективним; у подальшому було діагностовано залізорефрактерну анемію. Порушення синтезу яких речовин є ланкою в па-тогенезі цієї анемії?

A. @Порфіринів

B. РНК

C. ДНК

D. Еритропоетинів

E. Білків

31. Жінка скаржиться на тривалі менорагії, погане загальне самопочуття. Об’єктивно: фіброміома матки, анемія (гіпохромна, мікроцитарна, гіпоре-генеративна). Що може бути причиною анемії в хворої?

A. @Дефіцит заліза

B. Дефіцит В6

C. Дефіцит В12

D. Імунний гемоліз

E. Токсичний гемоліз

32. Чоловік, що хворіє на атрофічний гастрит, скар-житься на швидку втомлюваність, задишку, від-чуття оніміння в кінцівках. При обстеженні вияв-лено гіперхромну анемію, лейкопенію. Що може бути причиною анемії та лейкопенії у хворого?

A. @Дефіцит вітаміну В12

B. Дефіцит фолієвої кислоти

C. Дефіцит заліза

D. Гемоліз еритроцитів

E. Хронічна крововтрата

33. У жінки, що хворіє на атрофічний гастрит, вияв-лено мегалобластну анемію, лейкопенію. Пору-шення синтезу якої речовини є ланкою в патоге-незі змін крові у хворої?

A. @ДНК

B. РНК

C. Порфіринів

D. Гема

E. Гемоглобіну

34. Жінка скаржиться на зниження працездатності, сонливість, задишку. Обстеження виявило мікро-цитарну анемію з КП 0,5. Що може бути причиною такого стану крові?

A. @Дефіцит заліза

B. Дефіцит фолієвої кислоти

C. Дефіцит вітаміну В12

D. Дія гемолітичної отрути

E. Імунні реакції проти еритроцитів

35. У хворого, якому 5 років тому зробили резекцію шлунка, діагностовано гіперхромну анемію. Що може бути причиною анемії у хворого?

A. @Дефіцит фактора Касла

B. Дефіцит заліза

C. Аліментарна нестача віт. В12

D. Дефіцит фактора Хагемана

E. Гіповітаміноз К

36. Жінка скаржиться на зниження працездатності, сонливість, задишку при швидкій ході. В крові: еритроцити – 4,0 Т/л, гемоглобін – 82 г/л, колір-ний показник – 0,6. В мазку крові – велика кіль-кість мікроцитів, анулоцитів. Для якої анемії це характерно?

A. @Залізодефіцитної

B. Гострої постгеморагічної

C. В12 – дефіцитної

D. Фолієводефіцитної

E. Серповидноклітинної

37. В хворої діагностовано регенеративну анемію. Які клітини можуть бути в периферичній крові в да-ному випадку?

A. @Нормобласти

B. Мієлобласти

C. Мегакаріобласти

D. Лімфобласти

E. Монобласти

38. Для з’ясування стану кісткового мозку при анемі-ях бажано оцінити його регенераторну здібність. Визначення кількості яких клітин в крові дозволяє це зробити?

A. @Ретикулоцити і нормобласти

B. Мікроцити і макроцити

C. Мегалоцити

D. Мегалобласти

E. Пойкілоцити

39. У чоловіка з анацидним гастритом при обстеженні виявлено таке: еритроцити 3,5 Т/л, гемоглобін 4,2 ммоль/л, КП 0,6, мікроцитоз, анулоцитоз. Дефіцит якої речовини є причиною виникнення і розвитку анемії в хворого?

A. @Заліза

B. Фолієвої кислоти

C. Вітаміна В12

D. Вітаміна В1

E. Вітаміна С

40. У вагітної жінки виявлено при обстеженні: ерит-роцити – 3,0 Т/л, гемоглобін – 4,4 ммоль/л, КП 0,7, в периферичній крові – мікроцитоз, анулоци-тоз, ретікулоцитоз. Яка анемія є у пацієнтки?

A. @Залізодефіцитна

B. Фолієводефіцитна

C. В12 дефіцитна

D. Гемолітична спадкова

E. Мегалобластна

41. При обстеженні хворого в крові: гемоглобін 5,6 ммоль/л, еритроцити 2,0 Т/л, КП 1,4, в великої кі-лькості – мегалоцити і мегалобласти. Про яку анемію це свідчить?

A. @В12–, фолієводефіцитну

B. Залізодефіцитну

C. Набуту гемолітичну

D. Спадкову гемолітичну

E. Хронічну постгеморагічну

42. При обстеженні хворого із жовтяницею виявлено: білірубін непрямий 128 мкмоль/л, гемоглобін 5,0 ммоль/л, еритроцити 2,5 Т/л, КП 1,0, ретикулоци-ти 7%, пойкілоцитоз. Охарактеризуйте анемію у хворого за патогенезом:

A. @Гемолітична

B. Нормохромна

C. Регенераторна

D. Гіпопластична

E. Дисрегуляторна

43. У жінки 55 років, яка тривалий час страждає на залізодефіцитну анемію, виявлені дистрофічні зміни в міокарді. Недостатня активність яких фе-рментів кардіоміоцитів може бути причиною дис-трофії міокарда у хворої?

A. @Тканинного дихання

B. Фосфоліпаз

C. Кіназ

D. Пероксидаз

E. Фосфорилаз

44. Стоматолог звернув увагу на «лакований», «ма-линовий» язик у пацієнта. У крові – низький вміст еритроцитів і гемоглобіну, КП 1,4, багато мегало-цитів і мегалобластів. Для якої патології крові такі зміни є типовими?

A. @В12–, фолієводефіцитна анемія

B. Залізодефицитна анемія

C. Гемолітична анемія

D. Еритроцитоз

E. Гемофілія

45. Хворий скаржиться на неприємні больові відчуття в язику. Язик гладенький, яскраво–червоного ко-льору. У крові пацієнта: Er – 2,2 Т/л, Hb – 6,5 ммоль/л, КП – 1,47. Для якої анемії характерні такі зміни?

A. @Мегалобластна

B. Нормобластна

C. Таласемія

D. Залізодефіцитна

E. Залізорефрактерна

46. Жінку, яка хворіє на атрофічний гастрит, турбує загальна слабкість, запаморочення, задишка, біль і печія в ділянці язика та відчуття оніміння в кін-цівках. При обстеженні у неї виявлено мегалобла-стну анемію (КП 1,45), лейкопенію. Що є причи-ною мегалобластичної анемії у хворої?

A. @Дефіцит вітаміну В12

B. Дефіцит фолієвої кислоти

C. Дефіцит заліза

D. Гемоліз внутрішньосудинний

E. Гемоліз позасудинний

47. У хворого 35 років розвинулася імунна гемолітич-на анемія. Який показник сироватки крові зростає при цьому найбільше?

A. @Непрямий білірубін

B. Прямий білірубін

C. Стеркобіліноген

D. Холестерин

E. Жовчні кислоти

48. У пацієнта 34 р діагностовано серпоподібноклі-тинну анемію. Назвіть тип успадкування дефект-ного гену, що визначає розвиток цієї анемії.

A. @Аутосомний з неповним домінуванням

B. Аутосомно–рецесивний

C. Домінантний з повним домінуванням

D. Зчеплений з Х–хромосомою, рецесивний

E. Зчеплений з Х–хромосомою, домінантний

49. У дівчинки, 5 років, виявлено нормохромну, гіпо-регенераторну анемію, пойкілоцитоз та мікросфе-роцитоз еритроцитів. Для яких анемій характерні зазначені порушення?

A. @Гемолітична спадкова

B. Гемолітична набута

C. Постгеморагічна

D. Залізодефіцитна

E. Мегалобластна

50. У пацієнта с хроничним мієлолейкозом знижено вміст еритроцитів і гемоглобіну в крові. Який ме-ханізм визначає розвиток анемії в зазначеному випадку?

A. @Витіснення еритроцитарного ростка

B. Дефіцит заліза

C. Дефіцит еритропоетинів

D. Гемоліз

E. Крововтрата

51. Раптово, після застосування амідопірину, у чоло-віка сеча набула бурий колір. При обстеженні ви-явлено зниження кількості еритроцитів, вільний гемоглобін у крові. Надайте патогенетичну харак-теристику анемії у цього хворого.

A. @Гемолітична імунна

B. Хронічна постгеморагічна

C. Залізодефіцитна

D. Дисрегуляторна

E. Гостра постгеморагічна

52. У хворого на хронічний гломерулонефрит діагнос-товано нормохромну анемію. Що найімовірніше за все спричинює розвиток анемії при хронічному нефриті?

A. @Дефіцит еритропоетинів

B. Дефіцит фолієвої кислоти

C. Дефіцит внутрішнього факторау Кастла

D. Імунний гемоліз еритроцитів

E. Дефіцит зовнішнього фактора Кастла

53. Хворий на хронічний лімфолейкоз скаржится на задишку, жовтушність шкіри і склер. При обсте-женні: Er – 2,5 Т/л, Yb – 4,5 ммоль/л, ретикулоци-ти – 5%, L – 64 Г/л (лімфоцитоз), Tr – 200 Г/л. У крові пацієнта багато антиеритроцитарних анти-тіл. Яка це форма анемії за (механізмом)?

A. @Гемолітична імунна

B. Гемолітична токсична

C. Гіпорегенераторна

D. Постгеморагічна

E. Дисеритропоетична

54. У хворого з гіпохромною анемією в еритроцитах знаходиться 45 % HbS та 55 % HbА1. Яка форма анемії у хворого?

A. @Гемоглобінопатія, серпоподібноклітинна

B. Гемоглобінопатія, таласемія

C. Перніціозна

D. Ферментопатія

E. Мембранопатія

55. У людини до травми гематокритний показник 40%. Яким він буде через добу після втрати 750 мл крові?

A. @30%

B. 55%

C. 40%

D. 45%

E. 50%

56. Хворий 2 роки тому переніс резекцію пілоричного відділу шлунка. Спостерігається слабкість, періо-дична поява темних кіл під очима, задишка. У крові: Hb – 70 г/л, Er. – 3,0 Т/л, КП – 0,7. Які змі-ни еритроцитів у мазках крові найбільш характер-ні для цього стану?

A. @Мікроцити

B. Шизоцити

C. Овалоцити

D. Макроцити

E. Мегалоцити

57. Чоловік 56–ти років потрапив до клініки зі скар-гами на загальну слабкість, біль і печіння в язиці, відчуття оніміння в кінцівках. У минулому переніс резекцію кардіального відділу шлунка. У крові: Hb – 80 г/л; Er. – 2,0 Т/л; КП – 1,2, L. – 3,5 Г/л. Який вид анемії у цього хворого?

A. @B12–фолієводефіцитна

B. Гемолітична

C. Залізодефіцитна

D. Апластична

E. Постгеморагічна

58. Після оперативного видалення частини шлунка у хворого порушилося всмоктування вітаміну B12, він виводиться з калом. Розвинулася анемія. Який фактор необхідний для всмоктування цього віта-міну?

A. @Гастромукопротеїн

B. Гастрин

C. Фолієва кислота

D. Соляна кислота

E. Пепсин

59. Жінка 55–ти років звернулася зі скаргами на три-валі циклічні маткові кровотечі протягом року, слабкість, запаморочення. Об'єктивно: блідість шкіри. У крові: Hb– 70 г/л, Er.– 3,2 T/л, КП– 0,6, L.– 6,0 Г/л, ретикулоцити – 1%; гіпохромія ерит-роцитів. Яка анемія у хворої (за патогенезом)?

A. @Хронічна постгеморагічна

B. B12–фолієводефіцитна

C. Апластична

D. Гемолітична

E. Залізодефіцитна

60. У хворого з хронічною постгеморагічною анемією у крові знижена концентрація сироваткового залі-за, гіпохромія еритроцитів, пойкіло– та анізоцитоз. Яка величина кольорового показника може бути при цьому?

A. @0,7

B. 0,8

C. 0,9

D. 1,0

E. 1,1

61. У людей, які живуть в умовах високогір’я, вміст еритроцитів становить більше 6 Т/л. Який голов-ний механізм розвитку еритроцитозу в цих умо-вах?

A. @Активація кровотворення

B. Збільшення частоти серцевих скорочень

C. Зменшення обсягу депо крові

D. Зменшення частоти дихання

E. Активація парасимпатичної нервової системи

62. У дитини, яка знаходилась на вигодовуванні ко-зячим молоком, розвинулась В12–дефіцитна ане-мія. Який колірний показник характерний для цієї анемії?

A. @1,5

B. 1,1

C. 1,0

D. 0,8

E. 0,6

63. У хворої К., 37 років, яка останні 6 років працює на заводі газорозрядних ламп у свинцевому цеху, в аналізі крові знайдено: кількість еритроцитів та гемоглобіну знижена, бато сидероцитів, мікроци-тоз, гіпохромія; вміст сироваткового заліза підви-щений. Назвіть вид анемії.

A. @Залізорефрактерна

B. Залізодефіцитна

C. Анемія Мінковського–Шоффара

D. Гіпопластична

E. Метапластична

64. У хворого в аналізі крові виявлено: Er. – 7,4 Т/л, Hb – 10,8 ммоль/л, КП – 0,69. L – 15 Г/л, б – 2%, е – 11%, ю – 1%, п/я – 15%, с/я – 56%, л – 12%, мон. – 3%, Tr – 550 Г/л, ретикулоцити – 2%. Нас-лідком якої патології є ці зміни крові?

A. @Еритремія.

B. Гіпоксична гіпоксія.

C. Еритропоетинпродукуюча пухлина.

D. Локальна гіпоксія нирок.

E. Компенсаторна реакція на крововтрату.

65. У хворого діагностована сидероахрестична анемія. Вміст заліза у сироватці крові підвищений. Яким порушенням обміну заліза для синтезу гемоглобі-ну?

A. @Невикористання заліза для синтезу гемоглобі-ну.

B. Надмірне поступлення заліза з їжею.

C. Порушення всмоктування заліза в кишечника.

D. Підвищене використання заліза організмом.

E. Дефіцит заліза

66. У хворого через тиждень після масивної крововт-рати в крові виявляється велика кількість реге-нераторних форм еритроцитів. Яких саме?

A. @Ретикулоцитів

B. Мегалобластів

C. Мегалоцитів

D. Мікроцитів

E. Сфероцитів

67. Внаслідок поранення плеча робітник з кровоте-чею потрапив до лікарні. Через 5 днів у крові в нього виявлено збільшену кількість регенератив-них форм еритроцитів. Назвіть їх:

A. @Ретикулоцити

B. Мегалоцити

C. Еритробласти

D. Мегалобласти

E. Лімфобласти

68. У 5–тирічної дитини з підвищеною температурою тіла після прийому аспірину спостерігається поси-лений гемоліз еритроцитів. Спадкова недостат-ність якого ферменту змогла спричинити у дитини гемолітичну анемію?

A. @Глюкозо–6–фосфат–дегідрогенази

B. Глюкозо–6–фосфатази

C. Глікогенфосфорилази

D. Гліцерофосфатдегідрогенази

E. Гамаглутамілтрасферази

69. У хворого на хронічний неспецифічний виразковий коліт постійно виявляється крові у калі. Для ко-рекції лікування потрібно вирішити питання про тип гіпоксії при хронічній кровотечі:

A. @Гемічний.

B. Серцево–судинний.

C. Дихальний.

D. Тканинний

E. Гіпоксичний

70. У хлопчика 12 років на фоні інвазії гельмінтом Лентецем широким (Diphyllobothrium latum) спо-стерігається макроцитарна, мегалобластична ане-мія (еритроцити 2 Т/л при збільшенні їх об’єму і зміні форми клітини), порушення з боку перифе-ричної нервової системи (фунікулярний мієлоз), хейліт та глосит. Який авітаміноз спостерігається?

A. @В12

B. В6

C. В2

D. В1

E. В5

71. У хворого на пневмонію гемолітичний криз із роз-витком анемії. У крові знайдено дрепаноцити. Що є безпосередньою причиною гемолітичного кризу?

A. @Гіпоксія, викликана пневмонією

B. Зміна осмолярності крові

C. Гіпероксія

D. Гетерозиготність за Нb S

E. Мутація структурного гена

72. У хворого на В12–дефіцитну анемію розвинулись дегенеративні процеси в задніх і бокових стовпах спинного мозку (фунікулярний мієлоз), пов’язані з нагромадженням метилмалонової кислоти. Вона порушує синтез

A. @Мієліну

B. Ацетилхоліну

C. Норадреналіну

D. Дофаміну

E. Серотоніну

73. У хворого після тривалого курсу протитуберку-льозного лікування, яке включало ізоніазид, ви-никла анемія, стійка до препаратів заліза. Провід-ною ланкою її патогенезу вважається дефіцит

A. @Вітаміну В6

B. Вітаміну К

C. Вітаміну В12

D. Вітаміну С

E. Вітаміну РР

74. У пацієнта 42 років з виробничим шкідливим кон-тактом зі свинцем еритропенія та низький вміст гемоглобіну. Підвищений рівень сироваткового заліза у крові. Яка анемія у хворого ?

A. @Залізорефрактерна

B. В12–фолієводефіцитна

C. Гемолітична

D. Гіпопластична

E. Залізодефіцитна

75. При обстеженні хворого на атрофічний гастрит виявлено мегалобластну анемію. Дефіцит якої ре-човини є причиною виникнення анемії у цього хворого?

A. @Гастромукопротеїну

B. Вітаміну В6

C. Вітаміну В1

D. Заліза

E. Еритропоетинів

76. У хворого після операції під наркозом виник гемо-ліз. Лабораторно виявлені у крові HbS та HbA1. Який тип успадкування має серпоподібноклітинна анемія, діагностована у хворого?

A. @Неповне домінування

B. Повне домінування

C. Рецесивний

D. Зчеплений з Х–хромосомою

E. Зчеплений з У–хромосомою

Патологія лейкоцитів

77. У хворого виявлено недостатність гранулоцитопо-езу та тромбоцитопоезу. У якому з перелічених органів найбільш імовірно відбувається патологіч-ний процес?

A. @Червоний кістковий мозок

B. Печінка

C. Лімфатичний вузол

D. Вилочкова залоза

E. Селезінка

78. Робітник, що потрапив під дію іонізуючого випро-мінювання під час аварії на атомній станції (доза 5 Гр), був терміново госпіталізований. Як зміню-ється клітинний склад крові в перші години після опромінення?

A. @Лімфопенія

B. Нейтропенія

C. Лейкопенія

D. Тромбоцитопенія

E. Анемія

79. Молоду людину бентежить розповсюдження ви-сипів на шкірі тулуба і періанально. Попередній діагноз: „Герпетична інфекція. Імунодефіцитний стан?” Занадто мала кількість яких лейкоцитів в крові хворого дозволить підтвердити СНІД?

A. @CD4+ лімфоцити

B. CD8+ лімфоцити

C. Макрофаги

D. В–лімфоцити

E. Нейтрофіли

80. У приймальне відділення лікарні поступила дівчи-нка з болем у животі. Діагностовано гострий апе-ндицит. Як змінюється клітинний склад перифери-чної крові при гострому нормергічному запаленні?

A. @Нейтрофілія

B. Нейтропенія

C. Анемія

D. Лімфопенія

E. Агранулоцитоз

81. У хворого гострий апендицит. Яке ядерне зрушен-ня нейтрофілів периферичної крові є типовим для гострого нормергічного запалення?

A. @Ядерне зрушення вліво регенераторне

B. Ядерне зрушення вліво гіперрегенераторне

C. Ядерне зрушення вліво дегенеративне

D. Ядерне зрушення вліво регенераторно–дегенеративне

E. Ядерне зрушення вправо

82. Матір хлопчика 4–х років скаржиться на те, що дитина погано їсть, дратівлива. Обстеження пока-зало, що хлопчик інфікований аскаридами. Які зміни з боку периферичної крові є типовими для гельмінтозів?

A. @Еозинофілія

B. Лімфоцитоз

C. Моноцитоз

D. Нейтрофілія

E. Нейтропенія

83. У хворого на гостру пневмонію: лейкоцити 16 Г/л, лімфоцити 26%, нейтрофіли: мієлоцити – 4%, юні – 4%, п/я – 12%, с/я – 50%. Яке ядерне зрушен-ня нейтрофілів периферичної крові має місце в цьому випадку?

A. @Вліво гіперрегенераторне

B. Вліво регенераторне

C. Вліво дегенеративне

D. Вліво регенераторно–дегенеративне

E. Вправо

84. До кардіологічного відділення лікарні поступив чоловік із стенокардією. Аналіз ЕКГ, зміни складу периферичної крові пацієнта показали інфаркт мі-окарда. Які зміни клітинного складу периферичної крові індуковані некротичними змінами в міокар-ді?

A. @Нейтрофільний лейкоцитоз

B. Еозинофільний лейкоцитоз

C. Анемія

D. Лімфопенія

E. Моноцитоз

85. Через 2 години після вживання фенацетину у па-цієнта з’явився гострий біль у горлі, підвищилася температура тіла. Об’єктивно: некротична ангіни, у крові – агранулоцитоз. Що може бути синонімом агранулоцитозу?

A. @Нейтропенія

B. Нейтрофілія

C. Лімфоцитоз

D. Лімфопенія

E. Моноцитоз

86. При черговому обстеженні у пацієнтки з тиреоїди-том виявлені мегалобластна анемія та лейкопенія. Лейкоцити 3,0 Г/л, серед них: лімфоцити 64%, моноцити 4%, еозинофіли 2 %, нейтрофіли 30% (лише сегментоядерні). Яке ядерне зрушення нейтрофілів периферичної крові спостерігається в даному випадку?

A. @Вправо

B. Вліво регенеративне

C. Вліво гіперрегенеративне

D. Вліво дегенеративне

E. Вліво регенеративно–дегенеративне

87. Охарактеризуйте ядерне зрушення нейтрофілів периферичної крові у хворого на гостру бактеріа-льну пневмонію: Лейкоцити – 11 Г/л, лімфоцити – 25%, нейтрофіли: ю – 5%, п/я – 10%, с/я – 60%.

A. @Вліво регенераторне

B. Вліво гіперрегенераторне

C. Вліво дегенеративне

D. Вліво регенераторно–дегенеративне

E. Вправо

88. У крові хворого загальна кількість лейкоцитів складає 10 Г/л, при цьому нейтрофіли складають 35%, лімфоцити – 60%. Що може бути причиною таких зрушень "білої" крові?

A. @Хронічне запалення

B. Опромінення організму

C. Застосування глюкокортикоїдів

D. Гострій лейкоз

E. Крововтрата

89. У лікарню поступила жінка з опіками. Як в типо-вих випадках при гострому пошкодженні зміню-ється клітинний склад периферичної крові?

A. @Нейтрофілія

B. Нейтропенія

C. Еритроцитоз

D. Лімфопенія

E. Лімфоцитоз

90. Після курсу хіміотерапії у хворої: лейкоцити 3,0 Г/л, з яких лімфоцити 70%, нейтрофіли: п/я – 14%, с/я – 16%. Охарактеризуйте стан „білої” крові в пацієнтки:

A. @Нейтропенія

B. Гранулоцитоз

C. Лімфопенія

D. Лімфоцитоз

E. Анемія

91. У неврологічне відділення лікарні поступив чоло-вік 67 років з інсультом. Як у типових випадках при інсульті змінюється клітинний склад перифе-ричної крові?

A. @Нейтрофільний лейкоцитоз

B. Лімфопенія

C. Еозинофільний лейкоцитоз

D. Моноцитоз

E. Еритропенія

92. У хворого на інфекційний ендокардит у крові: лейкоцити 10 Г/л, лімфоцити 30%, моноцити 18%, нейтрофіли 50%, еозинофіли 2%. Визначте зміни "білої" крові в пацієнта.

A. @Моноцитоз

B. Гранулоцитоз

C. Агранулоцитоз

D. Лімфоцитоз

E. Еритроцитоз

93. Обстеження показало, що загальна кількість лей-коцитів в крові пацієнта складає 10 Г/л; при цьо-му нейтрофіли складають 82%. Охарактеризуйте зміни клітинного складу "білої" крові в даному ви-падку:

A. @Нейтрофільний лейкоцитоз

B. Моноцитоз

C. Лімфоцитоз

D. Агранулоцитоз

E. Еритроцитоз

94. Обстеження показало, що загальна кількість лей-коцитів в крові пацієнта складає 10 Г/л; при цьо-му нейтрофіли складають 82%. Вкажіть можливу причину таких змін складу "білої" крові:

A. @Травма, опіки

B. Вірусна інфекція

C. Гельмінти

D. Хронічне запалення

E. Хронічний лімфолейкоз

95. Обстеження показало, що загальна кількість лей-коцитів у крові пацієнта складає 8 Г/л; при цьому еозинофіли складають 14%. Вкажіть можливу причину змін складу "білої" крові у хворого:

A. @Глистна інвазія

B. Гостре запалення / вірусна інфекція

C. Гостре запалення / травма / опіки

D. Гостре запалення / бактеріальная інфекція

E. ВІЛ – інфекція

96. Обстеження показало, що загальна кількість лей-коцитів в крові пацієнта складає 10 Г/л; при цьо-му лімфоцити складають 57%. Охарактеризуйте зміни клітинного складу "білої" крові в цьому ви-падку:

A. @Лімфоцитоз

B. Моноцитоз

C. Нейтрофільний лейкоцитоз

D. Еозинофілія

E. Базофілія

97. Обстеження показало, що загальна кількість лей-коцитів у крові пацієнта складає 10 Г/л; при цьо-му лімфоцити складають 57%. Вкажіть можливу причину таких змін складу "білої" крові:

A. @Хронічне запалення

B. Гостре запалення / травма / опіки

C. Гостре запалення / бактеріальна інфекція

D. Опромінення

E. Застосування глюкокортикоїдів

98. Дослідження показало, що загальна кількість лейкоцитів в крові пацієнта складає 11 Г/л; при цьому нейтрофіли складають 70%, з них 9% – паличкоядерні. Охарактеризуте зміни клітинного складу "білої" крові в зазначеному випадку:

A. @Ядерний зсув нейтрофілів вліво

B. Ядерний зсув нейтрофілів вправо

C. Нейтропенія

D. Агранулоцитоз

E. Лімфоцитоз

99. Дослідження показало, що загальна кількість лейкоцитів в крові пацієнта складає 11 Г/л; при цьому нейтрофіли складають 70%, з них 9% – паличкоядерні. Вкажить можливу причину таких змін складу "білої" крові:

A. @Гостре запалення / травма / опіки / бактеріа-льна інфекція

B. Гостре запалення / вірусна інфекція

C. Гостре запалення / глистна інфекція

D. ВІЛ –інфекція, фінальний період

E. Хронічне запалення

100. Дослідження показало, що загальна кількість лейкоцитів в крові пацієнта складає 4 Г/л; при цьому лімфоцити складають 80%. Охарактеризу-те зміни клітинного складу "білої" крові в зазна-ченому випадку:

A. @Нейтропенія

B. Гранулоцитоз

C. Моноцитоз

D. Лімфопенія

E. Еритропенія

101. Загальна кількість лейкоцитів в крові пацієнта складає 3,5 Г/л, при цьому лімфоцити складають 10%. Що може бути причиною змін клітинного складу "білої" крові в зазначеному випадку:

A. @Опромінення

B. Опіки

C. Травма

D. Інфекція

E. Хронічний лейкоз

102. У пацієнта з мегалобластною анемією вміст лей-коцитів в крові 4 Г/л, лімфоцити 40%, нейтрофіли 40%; ядра нейтрофілів містять 5–7 сегментів, па-личкоядерні клітини відсутні. Зазначте зміни "бі-лої" крові у пацієнта.

A. @Ядерний нейтрофільний зсув вправо

B. Ядерний нейтрофільний зсув вліво

C. Лейкоцитоз

D. Лімфоцитоз

E. Агранулоцитоз

103. Загальна кількість лейкоцитів у крові пацієнта 3 Г/л; при цьому нейтрофілів 35%, з яких 10% – паличкоядерні з токсичною зернистістю, сегмен-тоядерних – 25%. Як можна визначити ядерне зрушення нейтрофілів у пацієнта?

A. @Вліво дегенеративне

B. Вправо

C. Вліво регенеративне

D. Вліво гіперрегенеративне

E. Вліво регенеративно–дегенеративне

104. Пацієнт часто хворіє на інфекції. У нього виявлено нейтропенію, при цьому більшість нейтрофілів мають ядра з 4 – 7 сегментів. Як можна визначити ядерне зрушення нейтрофілів у пацієнта?

A. @Вправо

B. Вліво регенеративне

C. Вліво гіперрегенеративне

D. Вліво регенеративно–дегенеративне

E. Вліво дегенеративне

105. Аналіз крові у дитини з гострим болем в животі показав нейтрофільний лейкоцитоз, при цьому нейтрофілів 84% (юні 5%, паличкоядерні 9%, се-гментоядерні 70%). Яке зрушення нейтрофілів периферичної крові у дитини?

A. @Вліво регенеративне

B. Вліво гіперрегенеративне

C. Вліво дегенеративне

D. Вліво регенеративно–дегенеративне

E. Вправо

106. У юнака з ангіною: лейкоцити 11 Г/л, з них лім-фоцити 20%, нейтрофіли 70% (юні 3%, паличко-ядерні 10%, сегментоядерні 57%). Яке зрушення нейтрофілів периферичної крові є типовим для го-строго нормергічного запалення?

A. @Вліво регенеративне

B. Вліво гіперрегенеративне

C. Вліво дегенеративне

D. Вліво регенеративно–дегенеративне

E. Вправо

107. У крові хворого на гострий бронхіт: лейкоцити 11 Г/л, лімфоцити 25%, нейтрофіли 70%: юні 4%, п/я 8%, с/я 58%, з токсичною зернистістю. Яке зрушення нейтрофілів периферичної крові спо-стерігається в даному випадку?

A. @Вліво регенеративно–дегенеративне

B. Вліво регенеративне

C. Вліво гіперрегенеративне

D. Вліво дегенеративне

E. Вправо

108. Батьки для профілактики кишкових інфекцій у дитини 3–х років тривало застосовували антибіо-тики. Через місяць стан дитини погіршився. У крові – виражена лейкопенія і гранулоцитопенія. Який найбільш вірогідний механізм виявлених змін у крові?

A. @Мієлотоксичний

B. Віковий

C. Цитолітичний

D. Аутоімунний

E. Перерозподільний

109. У студента через 2 години після іспиту в аналізі крові виявлено лейкоцитоз без істотних змін у лейкоцитарній формулі. Який найбільш вірогідний механізм розвитку лейкоцитозу?

A. @Перерозподіл лейкоцитів в організмі

B. Посилення лейкопоезу та зменшення руйнування лейкоцитів

C. Посилення лейкопоезу

D. Уповільнення руйнування лейкоцитів

E. Уповільнення міграції лейкоцитів у тканини

110. Чоловік 26–ти років перебуває в торпідній стадії шоку внаслідок автомобільної аварії. В крові: лейк.– 3,2 Г/л. Який головний механізм в розвитку лейкопенії?

A. @Перерозподіл лейкоцитів у судинному руслі

B. Пригнічення лейкопоезу

C. Порушення виходу зрілих лейкоцитів з кісткового мозку в кров

D. Руйнування лейкоцитів у кровотворних органах

E. Підвищення виділення лейкоцитів з організму

111. У хворого через добу після апендектомії у крові визначається нейтрофільний лейкоцитоз із реге-неративним зсувом. Який найбільш вірогідний ме-ханізм розвитку лейкоцитозу в даному випадку?

A. @Посилення лейкопоезу

B. Перерозподіл лейкоцитів у організмі

C. Уповільнення виходу лейкоцитів з кіскового моз-ку

D. Уповільнення руйнування лейкоцитів

E. Уповільнення міграції лейкоцитів у тканини

112. У спортсмена внаслідок значного фізичного нава-нтаження у крові визначається лейкоцитоз (12 Г/л). Який механізм лейкоцитозу в даному випад-ку?

A. @Перерозподіл лейкоцитів у судинах

B. Прискорене дозрівання лейкоцитів

C. Зменшений вихід L у тканини

D. Активація лейкопоезу в кістковому мозку

E. Вихід лейкоцитів з тканин у кров

113. У пацієнта, який хворіє на бронхіальну астму, що має інфекційно–алергічну природу, лаборант ви-явив порушення в лейкоцитарній формулі. Кіль-кість яких форм лейкоцитів при цьому змінилася?

A. @Еозинофілів

B. Нейтрофілів

C. Лімфоцитів

D. Моноцитів

E. Базофілів

114. У людини на фоні впливу іонізуючого опромінення в крові визначено зменшення кількості грануло-цитів. Чим обумовлений агранулоцитоз?

A. @Пригніченням лейкопоезу

B. Збільшенням переходу гранулоцитів ув тканини

C. Розвитком аутоімунного процесу

D. Підвищеним руйнуванням лейкоцитів

E. Порушенням виходу зрілих лейкоцитів з кістково-го мозку

115. У хворого на рак сечового міхура під час прове-дення цитостатичної хіміотерапії в аналізі крові встановлено: лейкоцити – 0,8 Г/л, гранулоцити – 0,6 Г/л. Яке ураження білої крові має місце у хво-рого?

A. @Мієлотоксичний агранулоцитоз.

B. Імунний агранулоцитоз.

C. Алейкія.

D. Лейкопенія.

E. Лейкоцитоз.

116. У хворого з анемією виявлено лейкопенію, в маз-ку крові – атипові нейтрофіли гігантських розмірів з гіперсегментованими ядрами. Вони характерні для

A. @В12 – дефіцитної анемії

B. Залізодефіцитної анамії

C. Серповидноклітинної анемії

D. Токсико–гемолітичної анемії

E. Анемії Мінковського–Шоффара

Лейкози

117. У крові пацієнта: знижено вміст гемоглобіну та еритроцитів, лейкоцитів 27 Г/л, гранулоцитоз, ядерне зрушення нейтрофілів вліво до промієло-цитів. Що може бути причиною вищезазначених змін?

A. @Хронічний мієлолейкоз

B. Хронічний лімфолейкоз

C. Анемія

D. Гострий лейкоз

E. Еритроцитоз

118. Хворий 67 р. скаржиться на втомлюваність, схуд-нення, жовтушність шкіри і склер. У нього в кро-ві: Hb 5,0 ммоль/л, Er 2,5 Т/л, L – 30 Г/л, лімфо-цити – 25 Г/л. Що може бути причиною такого стану?

A. @Хронічний лімфолейкоз

B. Хронічний мієлолейкоз

C. Гострий лейкоз

D. Дефіцит фолієвої кислоти

E. Надлишок кортизолу

119. В абсолютній більшості випадків хронічного мієло-лейкозу в клітинах пухлинного клону є "Філаде-льфійська" хромосома. Яка подія відбувається із кровотворною клітиною кісткового мозку внаслі-док зазначеної хромосомної аберації?

A. @Трансформація

B. Прогресія

C. Припинення поділу

D. Порушення дозрівання

E. Вихід у кров

120. Хворий на гострий лейкоз після курсу хіміотерапії почуває себе задовільно. Що можна трактувати як головний показник стану ремісії при гострому лейкозі?

A. @Бласти в кістковому мозку менше 5%

B. Лейкозні інфільтрати на слизовій оболонці рото-вої порожнини

C. Тромбоцитів 160 Г/л

D. Вміст гемоглобіну в крові більше 5 ммоль/л

E. Нейтрофіли в крові менше 10 Г/л

121. Хвора скаржиться на часті інфекційні захворю-вання. При обстеженні: анемія, тромбоцитопенія, лейкоцитів 8 Г/л, нейтрофіли складають 70%, з них: 45% – промієлоцити, 25% – сегментоядерні. Що може бути причиною такого?

A. @Гострий лейкоз

B. Хронічний мієлолейкоз

C. Хронічний лімфолейкоз

D. Гостра бактеріальна інфекція

E. Хронічна інфекція

122. Хвора скаржиться на втомлюваність, носові кро-вотечі, біль у кістках. При обстеженні: еритроци-ти – 2,7 Т/л, гемоглобін – 4,0 ммоль/л, лейкоцити – 14 Г/л, тромбоцити – 70 Г/л. Серед лейкоцитів крові 7% складають бласти. Що може бути при-чиною такого?

A. @Гострий лейкоз

B. Хронічний лейкоз

C. Еритроцитоз

D. Анемія

E. Нейтропенія

123. При черговому обстеженні в крові пацієнтки ви-явлено панцитопенію. Дослідження клітинного складу її кісткового мозку показало гострий міє-лолейкоз. Що є специфічною ознакою гострих лейкозів?

A. @Збільшення кількості незрілих клітин в кістко-вому мозку

B. Велика кількість зрілих та дозріваючих клітин в кістковому мозку

C. Велика кількість зрілих гранулоцитів в крові

D. Збільшення кількості дозріваючих гранулоцитів в крові

E. Зменшення незрілих клітин в крові

124. У хворого виявлені анемія, тромбоцитопенія, гра-нулоцитоз. Загальна кількість лейкоцитів у крові складає 57 Г/л, є ядерний зсув нейтрофілів вліво до промієлоцитів. У кістковому мозку – грануло-цитоз. Яка патологія наявна в хворого?

A. @Хронічний мієлолейкоз

B. Хронічний лімфолейкоз

C. Гострий лейкоз

D. Агранулоцитоз

E. Еритроцитоз

125. Пацієнту, який скаржиться на загальну слабкість та біль в кістках, встановлено діагноз "Хронічний мієлолейкоз". Назвіть хромосомну аномалію, ти-пову для більшості випадків цього захворювання.

A. @Транслокація 9 на 22

B. Транслокація 21 на 15

C. Делеція Х

D. Делеція 12

E. Трисомія 21

126. У крові пацієнта: анемія, тромбоцитопенія, зага-льна кількість лейкоцитів – 3,0 Г/л, лімфоцити –60%, промієлоцити – 15%. Що може бути причи-ною вищеописаної зміни складу "білої" крові:

A. @Гострий лейкоз

B. Хронічний лейкоз

C. Гостре запалення / травма / бактеріальна інфек-ція

D. Глистяна інвазія

E. Дефіцит заліза

127. Чоловік звернувся до лікаря з приводу збільшен-ня лімфатичного вузла на шиї. Будь–які неприємні відчуття в цьому місці в нього відсутні. За резуль-татами біопсії діагностовано злоякісну пухлину з лімфоїдної клітини лімфатичного вузла. Як звуть-ся такі пухлини системі крові?

A. @Лімфома

B. Гострий лейкоз

C. Хронічний лейкоз

D. Карцинома

E. Гемобластоз

128. Останнім часом перебіг хронічного мієлоїдного лейкозу (ХМЛ) у хворого змінився, в його крові зникають дозріваючі гранулоцити, з’являються бластні клітини. Як можна зазначити такі зміни периферичної крові в термінальну стадію розвит-ку ХМЛ?

A. @Лейкемічний провал

B. Ядерний зсув вліво регенеративний

C. Ядерний зсув вліво гіперрегенеративний

D. Дегенерація лейкоцитів

E. Регенерація лейкоцитів

129. Після третього курсу хіміотерапії у хворого на хронічний мієлолейкоз відсутні будь–які позитивні зміни. Яка властивість пухлин пов’язана з розвит-ком їх резистентності до хіміотерапії?

A. @Прогресія

B. Промоція

C. Автономний ріст

D. Інвазивний ріст

E. Експансивний ріст

130. У хворого гострий Т–клітинний лейкоз. Стосовно якого онковірусу доведено, що він здатен викли-кати лейкоз у людини?

A. @HTLV

B. HPV

C. Герпесу

D. Аденовіруси

E. Поліоми

131. У хворого на хронічний мієлолейкоз визначається гіперплазія ясен. Дослідження біоптату слизової оболонки ротової порожнини виявило її лейкозну інфільтрацію. З якою властивістю пухлини пов’язане ураження ротової порожнини у хворо-го?

A. @Метастазування

B. Анаплазія

C. Метаплазія

D. Автономність росту

E. Імморталізація

132. У постраждалого після опромінення виник гострий лейкоз. Який механізм трансформації характерний для фізичних канцерогенів?

A. @Мутаційний

B. Епігеномний

C. Трансплантаційний

D. Індукційний

E. Продукційний

133. У хворого на хронічний мієлолейкоз на фоні рези-стентності пухлини до хіміотерапії виник бластний криз, що було розцінено лікарем як прояв пух-линної прогресії. Що складає суть пухлинної про-гресії?

A. @Малігнізація

B. Розповсюдження по організму

C. Здатність до безмежного поділу

D. Антигенне спрощення клітин

E. Експансивний ріст

134. Елерман і Банг в експерименті викликали лейкоз у курей шляхом введення їм безклітинного фільт-рату, отриманого з крові курки. Який механізм канцерогенезу вмикають лише онкогенні віруси?

A. @Епігеномний

B. Мутаційний

C. Трансплантаційний

D. Індукційний

E. Продукційний

135. Стан хворого на хронічний мієлолейкоз раптово значно погіршився, звичайні протипухлинні пре-парати перестали діяти. Що можна трактувати як гематологічний прояв пухлинної прогресії хроніч-ного мієлолейкозу?

A. @Лейкемічний провал

B. Анемія

C. Еритроцитоз

D. Тромбоцитоз

E. Гранулоцитоз

136. У крові хворого лейкоцити – 27 Г/л. Лейкоцитарна формула: базофіли – 1 %, еозинофіли – 4 %, лі-мфоцити – 12 %, моноцити 3 %; нейтрофіли 80% (промієлоцити – 5 %, мієлоцити –5 %, юні– 7 %, паличкоядерні – 13 %, сегментоядерні – 50 %). Яка патологія крові найбільш вірогідно відповідає зазначеним показникам?

A. @Хронічний мієлолейкоз

B. Хронічний лімфолейкоз

C. Гострий лейкоз

D. Анемія

E. Еритроцитоз

137. У пацієнта в крові: еритроцити – 2,0 Т/л, гемогло-бін – 5,0 ммоль/л, лейкоцити – 10 Г/л, тромбоцити – 100 Г/л. Лейкоцитарна формула: еозинофіли – 10 %, мієлобласти – 10 %, нейтрофіли сегментоя-дерні – 40 %, лімфоцити – 40 %. Яка патологія крові відповідає вказаним змінам периферічної крові?

A. @Гострий мієлолейкоз

B. Гострий лімфолейкоз

C. Хронічний лімфолейкоз

D. Хронічний мієлолейкоз

E. Анемія

138. У хворого виявлені тромбоцитопенія і гранулоци-тоз. Загальна кількість лейкоцитів в крові – 80 Г/л, є гіперрегенеративний зсув нейтрофілів вліво. У кістковому мозку – гранулоцитоз. Яка па-тологія наявна в хворого?

A. @Хронічний мієлолейкоз

B. Хронічний лімфолейкоз

C. Гострий лейкоз

D. Агранулоцитоз

E. Еритроцитоз

139. Хворий скаржится на втомлюваність, схуднення, жовтушність шкіри і склер. При обстеженні: ерит-роцити – 2,5 Т/л, гемоглобін – 4,5 ммоль/л, рети-кулоцити – 2%, лейкоцити – 54 Г/л, тромбоцити – 200 Г/л. Серед лейкоцитів 90% складають малі та середні лімфоцити. Охарактеризуйте зміни "білої" крові в пацієнта.

A. @Хронічний лімфолейкоз

B. Хронічний мієлолейкоз

C. Гострий лейкоз

D. Анемія

E. Лімфопенія

140. Хвора скаржится на втомлюваність, носові крово-течі, біль у кістках. При обстеженні: еритроцити – 2,7 Т/л, гемоглобін – 4,0 ммоль/л, ретикулоцити – 0,05%, лейкоцити – 14 Г/л, тромбоцити – 70 Г/л. Серед лейкоцитів 7% складають бласти. Що може бути причиною вищезазначених змін?

A. @Гострий лейкоз

B. Хронічний лейкоз

C. Еритроцитоз

D. Анемія

E. Нейтропенія

141. Пацієнтку С., 55 років, турбує біль у кістках, осо-бливо в нічні часи. Привертає увагу гіперплазія її ясен. У крові: гемоглобін 6,0 ммоль/л, еритроцити 3,0 Т/л, тромбоцити 60 Г/л, лейкоцити 7 Г/л: лім-фоцити 40%; нейтрофіли 40%, з них паличкояде-рні 5%, сегментоядерні 35%; бласти 5%. Що мо-же бути причиною такого стану в системі крові?

A. @Гострий лейкоз

B. Інфекція вірусна

C. Інфекція бактеріальна

D. Хронічний мієлолейкоз

E. Хронічний лімфолейкоз

142. Після лікування від лімфогранулематозу пацієнтка була в стані повної клініко–гематологічної ремісії. Тепер не виявлено змін кількості еритроцитів, ге-моглобіну, тромбоцитів і лейкоцитів в крові, але 7% усіх лейкоцитів представлені бластними кліти-нами. Що може бути причиною таких змін крові?

A. @Гострий лейкоз

B. Хронічний лейкоз

C. Інфекція

D. СНІД

E. Анемія

143. Які серед нижченазваних канцерогенних речовин можуть бути визначені як лейкозогенні більшою мірою, ніж інші?

A. @Бензол

B. ПАВ

C. Аміноазосполуки

D. Аденовіруси

E. Вірус папіломи людини HPV

144. У хворого на шиї виявлено пакет спаяних між со-бою лімфовузлів щільної консистенції. При гісто-логічному обстеженні видаленого лімфовузла від-мічається проліферація ретикулярних клітин, ная-вність клітин Березовського–Штернберга. Для якого захворювання така картина є типовою?

A. @Лімфогранулематоз

B. Мієлоцитарний лейкоз

C. Лімфобластний лейкоз

D. Лімфоцитарний лейкоз

E. Мієлобластний лейкоз

145. Хворий 23–х років скаржиться на слабкість, під-вищення температури до 38–400C. Об'єктивно: печінка і селезінка збільшені. У крові: Hb– 100 г/л, Er – 2,9 Т/л, L. – 4,4 Г/л, Tr– 48 Г/л, ней-трофіли сегментоядерні – 17%, лімфоцити – 15%, бластні клітини – 68%. Всі цитохімічні реакції не-гативні. Дайте гематологічний висновок:

A. @Недиференційований лейкоз

B. Хронічний мієлолейкоз

C. Гострий еритромієлоз

D. Гострий лімфобластний лейкоз

E. Гострий мієлобластний лейкоз

146. При генетичному обстеженні хворих хронічним мієлолейкозом виявлена специфічна хромосома, яка є генетичним маркером хвороби. Який вид хромосомної аберації має місце при цьому?

A. @Делеція 22 хромосоми з транслокацією на 9–у.

B. Інверсія короткого плеча 21–ї хромосом

C. Делеція 23–ї хромосоми

D. Транслокація короткого плеча 21 хромосоми на 15–у

E. Дуплікація довгого плеча 22–ї хромосом.

147. У клініку поступив хворий 65 років зі скаргами на відчуття важкості в підреберних ділянках, збіль-шення лімфатичних вузлів, загальну слабість. Об’єктивно: гепатоспленомегалія, Нb – 4,8 ммоль/л, лейкоцити – 90 Г/л, лімфоцити 75%, ШОЕ 35 мм/год., в мозку та у периферичній крові багато тіней Гумпрехта. Для якого захворювання характерна така клінічна картина?

A. @Хронічний лімфолейкоз

B. Гострий лімфолейкоз

C. Гострий мієлолейкоз

D. Хронічний мієлолейкоз

E. Залізодефіцитна анемія

148. Хворий А., 64 роки, блідий, геморагічний висип на тілі та кінцівках, всі групи лімфовузлів збільшені. У крові: Hb – 3,9 ммоль/л, Er – 2,0 Т/л, L – 26 Г/л, Tr – 120 Г/л, лейкоцитарна формула: сегментоя-дерних нейтрофілів – 9 %, лімфоцитів – 8 %, мо-ноцитів – 3 %, бластних клітин – 80 %. Цитохіміч-ні дослідження бластних клітин: позитивна реак-ція на глікоген та кислу фосфатазу, негативна – на пероксидазу. Який росток кісткового мозку за-знав пухлинної трансформації?

A. @Лімфоїдний

B. Мієлоїдний

C. Моноцитарний

D. Мегакаріоцитарний

E. Еритроцитарний

149. У пацієнта визначаються дифузна лімфаденопатія і жовтяниця. При обстеженні: анемія, лейкоцитоз 35 Г/л, лімфоцитоз (лімфоцитів 90%), антиерит-роцитарні антитіла. Що є причиною розвитку аутоімунної анемії у пацієнта з хронічним лімфо-лейкозом?

A. @Зняття імунологічної толерантності

B. Недостатність гуморального імунітету

C. Недостатність клітинного імунітету

D. Недостатність фагоцитозу

E. Недостатність комплементу

150. Хворий звернувся до лікаря зі скаргами на зага-льну слабкість, гематоми на тілі. У крові: лейко-цитів 9,0 Г/л. Лейкоцитарна формула: Б–0%, Е–1%, мієлобласти–5%, П–2%, С–55%, Л–20%, М–4%. Про яку патологію з нижченаведених можна думати?

A. @Гострий мієлолейкоз

B. Хронічний мієлолейкоз

C. Гострий лімфолейкоз

D. Хронічний лімфолейкоз

E. Лейкемоїдна реакція

151. У хворого відмічається різке збільшення печінки і селезінки. У периферичній крові 12 Г/л лейкоци-тів, основну масу яких складають промієлоцити і мієлоцити. Про яке захворювання йде мова?

A. @Мієлоцитарний лейкоз

B. Мієлобластний лейкоз

C. Лімфоцитарний лейкоз

D. Лімфобластний лейкоз

E. Лімфосаркома

152. У хворого виявлена спленомегалія, геморагічний діатез. Периферична кров не змінена. У кістково-му мозку велика кількість мієлобластів. Про яке захворювання йде мова?

A. @Мієлобластний лейкоз

B. Лімфобластний лейкоз

C. Лімфосаркома

D. Мієлоцитарний лейкоз

E. Лімфоцитарний лейкоз

153. Хвора Р., 47 р., скаржиться на втомлюваність, носові кровотечі, біль у кістках. При обстеженні: еритроцити – 2,7 Г/л, гемоглобін – 4,0 ммоль/л, ретикулоцити – 0,5 %, лейкоцити – 14 Г/л, тром-боцити – 70 Г/л. Серед лейкоцитів 7 % складають бласти. Що може бути причиною вищезазначених змін?

A. @Гострий лейкоз

B. Хронічний лейкоз

C. Нейтропенія

D. Еритроцитоз

E. Анемія

Порушення гемостазу

154. У хворого на цингу спостерігаються кровоточи-вість ясен та петехії на шкірі. Що спричинює пору-шення гемостазу при цьому захворюванні?

A. @Порушення синтезу колагену

B. Тромбоцитопенія

C. Надлишок антикоагулянтів

D. Активація фібринолізу

E. Дефіцит прокоагулянтів

155. У дівчинки після перенесеної ангіни з’явився пе-техіальний висип на шкірі кінцівок та тулубі. Об’єктивно: кількість тромбоцитів 80 Г/л, антит-ромбоцитарні антитіла. Алергічні реакції якого ти-пу (за Кумбсом і Джелом) лежать в основі цього захворювання?

A. @II типу (гуморальні цитотоксичні )

B. I типу (анафілактичні)

C. III типу (імунокомплексні)

D. IV типу (клітинні цитотоксичні)

E. V типу (стимулюючі)

156. У хворого порушена адгезія тромбоцитів до кола-гену та спостерігаються кровотечі з дрібних су-дин. Порушення якої ланки гемостазу можна при-пустити у хворого?

A. @Судинно–тромбоцитарної

B. Коагуляційної, І фази

C. Коагуляційної, ІІІ фази

D. Фібринолізу

E. Коагуляційної, ІІ фази

157. У хворого діагностовано тромбоцитопенію. Які клінічні прояви типові для порушень тромбоцита-рно – судинного гемостазу?

A. @Петехії, екхімози (синці)

B. Гемартрози

C. Гематоми

D. Зменшення часу кровотечі

E. Збільшення часу згортання крові

158. У хворої при обстеженні виявили тромбоцитопа-тію. Вкажіть, яка зміна відіграє важливу роль в патогенезі тромбоцитопатій?

A. @Продукція патологічних тромбоцитів кістковим мозком

B. Зниження активності антикоагулянтів

C. Гіперактивація тромбоцитопоезу

D. Підвищення концентрації в крові прокоагулянтів

E. Пригнічення фібринолізу.

159. Перед проведенням оперативного втручання ви-явлено, що в людини час кровотечі збільшений до 10 хв. Дефіцит яких формених елементів у складі крові може бути причиною таких змін?

A. @Тромбоцитів

B. Еритроцитів

C. Моноцитів

D. Лімфоцитів

E. Лейкоцитів

160. У пацієнтки тривале вживання аспірину виклика-ло крововиливи. Об’єктивно: тромбоцитопенія з порушенням функціональної активності тромбоци-тів. Тромбоцитопатія в даному випадку зумовлена пригніченням активності:

A. @Циклооксигенази

B. Цитохромоксидази

C. Ліпооксигенази

D. Супероксиддисмутази

E. Фосфоліпази А2.

161. У хворого діагностовано зменшення продукції ен-дотелієм судин фактора Віллебранда. Яке пору-шення судинно–тромбоцитарного гемостазу спо-стерігається при цьому?

A. @Порушення адгезії тромбоцитів

B. Порушення агрегації тромбоцитів

C. Гіперкоагуляція

D. Порушення полімеризації фібрину

E. Посилений фібриноліз

162. У дитини з геморагічним висипом, що виник після ГРВІ, діагностовано геморагічний васкуліт (хворо-ба Шенляйн – Геноха). Алергічні реакції якого ти-пу (за Кумбсом і Джелом) лежать в основі цього захворювання?

A. @III типу (імунокомплексні)

B. I типу (анафілактичні)

C. II типу (гуморальні цитотоксичні )

D. IV типу (клітинні цитотоксичні)

E. V типу (стимулюючі)

163. У дитини з геморагічним висипом, що виник після ГРВІ, діагностовано геморагічний васкуліт (хворо-ба Шенляйн – Геноха). Що спричинює порушення гемостазу при цьому захворюванні?

A. @Пошкодження судинної стінки

B. Спадковий дефект сполучної тканини судинної сті-нки

C. Спадковий дефіцит антикоагулянтів

D. Пригнічення фібринолізу

E. Спадковий дефіцит прокоагулянтів

164. У жінки, що страждає на жовчно–кам’яну хворобу виявлено геморрагічний синдром, зумовлений де-фіцитом вітаміну К. Який з перелічених факторів є неповноцінним при гіповітамінозі К?

A. @Стюарта–Прауера (ф. X)

B. Антигемофільний глобулін А (ф. VIII)

C. Фактор Віллебранда

D. Фібринстабілізуючий (ф. XIII)

E. Фібриноген (ф. I)

165. У пацієнта при обстеженні було виявлено тро-мбофілію (прискорення процесу зсідання крові). Що могло стати причиною порушення?

A. @Дефіцит інгібіторів протеолітичних ферментів

B. Підвищення концентрації простацикліну

C. Зниження концентрації тромбіну в крові

D. Підвищення концентрації гепарину в крові

E. Підвищення концентрації фібринолізину в крові

166. У хворого із захворюванням печінки виявлено зниження вмісту протромбіну в крові. Це призве-де до порушення перш за все:

A. @Другої фази коагуляційного гемостазу

B. Фібринолізу

C. Третьої фази коагуляційного гемостазу

D. Судинно–тромбоцитарного гемостазу

E. Першої фази коагуляційного гемостазу

167. У хлопчика 7 років, після падіння з велосипеда виник гемартроз колінного суглоба. Введення крі-опреципітату та відкачування крові з суглоба призвело до значного поліпшення стану дитини. Про яке захворювання слід думати?

A. @Гемофілія А

B. Геморагічний васкуліт.

C. Тромбоцитопатія

D. Тромбоцитопенія

E. Ревматоїдний артрит

168. У хлопчика з вираженим геморагічним синдромом в плазмі крові відсутній антигемофильній глобулін А (фактор VIII). Яка фаза гемостазу первинно порушена у цієї дитини?

A. @Внутрішній шлях активації протромбінази

B. Ретракція кров’яного згустка

C. Перетворення фібриногену на фібрин

D. Перетворення протромбіну на тромбін

E. Зовнішній шлях активації протромбінази

169. Хлопчик страждає на гемофілію. Якими клінічними ознаками проявляються порушення коагуляційно-го гемостазу?

A. Петехіальними крововиливами

B. Мікрогематурією

C. Екхімозами (синцями)

D. Порушенням гостроти зору

E. @Гематомами, тривалими кровотечами

170. У постраждалого внаслідок ДТП через деякий час після перенесеної політравми розвинувся синдром дисемінованого внутрішньосудинного зсідання крові (ДВЗ). Який фактор був ініціатором цього ускладнення?

A. @Тканинний тромбопластин (ф. III)

B. Фібриноген (ф. I)

C. Антигемофільний глобулін А (ф. VIII)

D. Фактор Стюарта–Прауера (ф. X)

E. Антигемофільний глобулін В (ф. IX)

171. У хворого на гострий панкреатит розвинувся син-дром дисемінованого внутрішньосудинного зсідан-ня крові (ДВЗ). Яка речовина була ініціатором цього ускладнення?

A. @Трипсин

B. Фібриноген (ф. I)

C. Антигемофільний глобулін А (ф. VIII)

D. Фактор Стюарта–Прауера (ф. X)

E. Антигемофільний глобулін В (ф. IX)

172. У хворого на політравму з гострою нирковою не-достатністю, стан ускладнився внутрішньою кро-вотечею. Що є головною ланкою патогенезу цієї стадії ДВЗ – синдрому?

A. @Споживання факторів згортання крові

B. Викид лейкоцитів з депо

C. Тромбоцитоз

D. Активація протромбінази

E. Пригнічення фібринолізу

173. У хворого з опіковою хворобою як ускладнення розвинувся ДВЗ – синдром. Яку стадію ДВЗ – син-дрому можна запідозрити, якщо відомо, що кров хворого згортається менше ніж за 3 хв.?

A. Гіпокоагуляції

B. Відновлення

C. @Гіперкоагуляції

D. Латентну

E. Термінальну

174. У хворого з травмою розвинувся ДВЗ – синдром. Які зміни гемостазу спостерігаються в ІІ фазу ДВЗ –синдрому?

A. @Гіпокоагуляція

B. Гіперкоагуляція.

C. Фібриноліз

D. Тромбоцитопенія

E. Тромбоцитопатія

175. У пацієнта після переливання несумісної крові виник ДВЗ – синдром. Що є головною ланкою в патогенезі цього ускладнення при гемолізі?

A. @Надходження у кров внутрішньоклітинних про-теаз

B. Накопичення білірубіну в крові

C. Надлишок у крові тромбопластину

D. Надлишок у крові протромбіну

E. Збільшення вмісту плазміногену

176. У хворого на тлі хронічної ниркової недостатності розвився ДВЗ–синдром. При обстеженні виявлено збільшення часу згортання крові, тромбоцитопе-нію, підвищення рівня фібрин–мономерних ком-плексів та продуктів деградації фібрину. Про яку стадію ДВЗ–синдрому слід думати?

A. @Гіпокоагуляції

B. Гіперкоагуляції

C. Відновлення

D. Латентну

E. Нестабільну

177. У хворого на хронічний лімфолейкоз виникли ге-морагії внаслідок розвитку ДВЗ – синдрому. Які зміни показників периферичної крові будуть спос-терігатися при цьому?

A. @Гіпокоагуляція, тромбоцитопенія

B. Еритроцитоз, підвищення в’язкості крові

C. Тромбоцитоз, зменшення часу згортання крові

D. Гіперкоагуляція, збільшення агрегації тромбоцитів

E. Підвищена активність прокоагулянтів

178. У жінки з сепсисом з’явилися петехіальні крово-виливи, у крові зменшився вміст тромбоцитів і фі-бриногену, з’явилися продукти деградації фібри-ну. Що може бути причиною зазначених змін?

A. @ДВЗ синдром

B. Лейкопенія

C. Лімфопенія

D. Тромбоцитоз

E. Анемія

179. У хлопчика 3–х років, що страждає на геморагії, у крові відсутній антигемофільний глобулін А (фак-тор VIII). Недостатність якого механізму гемокоа-гуляції має місце у цього хворого?

A. @Внутрішній механізм формування протромбінази

B. Перетворення фібриногену в фібрин

C. Зовнішній механізм формування протромбінази

D. Перетворення протромбіну в тромбін

E. Ретракція кров’яного згустка

180. Чоловік Т., 46 р., який протягом двох років хворіє на хронічний мієлоїдний лейкоз, поступив до ліка-рні в стані гострої ниркової недостатності. Що мо-же бути причиною гострої ниркової недостатності в цього хворого?

A. @ДВЗ – синдром

B. Анемія

C. Лімфопенія

D. Нейтропенія

E. Тромбоцитопенія

181. Недостатня активність яких факторів зсідання крові зумовлює розвиток геморагічного синдрому в хворих на гіповітаміноз К?

A. @X, IX, VII, II

B. VIII, IX, XI

C. X, XI, XIII

D. V, I

E. Фактор Віллебранда

182. У пацієнтки, яка систематично вживає ацетилса-ліцилову кислоту, з’явилися крововиливи. Із зме-ншенням активності яких ферментів тромбоцитів пов'язаний розвиток тромбоцитопатії в даному випадку?

A. @Циклооксигенази

B. Ліпоксигенази

C. Пероксидази

D. Цитохромоксидази

E. Глюкозо–6–фосфатдегідрогенази

183. У дівчинки 10 років періодично виникають крово-течі з носа, з’являються невеликі геморагічні ви-сипання на шкірі. При обстеженні виявлено: час кровотечі – 10 хвилин, знижена адгезивнаї здат-ність тромбоцитів та низька активність ф. VIII (VIII:C). Яке захворювання у дитини?

A. @Хвороба Віллебранда

B. Гемофілія А

C. Гемофілія В

D. Спадкова дисфібриногенемія

E. Тромбоцитопенії

184. Після перенесеної ангіни в дівчинки 5 років спос-терігаються петехіальні висипи на шкірі тулуба і кінцівок, кровотечі з ясен. При обстеженні вияв-лено зменшення кількості тромбоцитів у крові. При якому рівні тромбоцитопенії (Г/л) з’являються її клінічні ознаки ?

A. @50

B. 150

C. 100

D. 20

E. 10

185. Що може бути головною ланкою в патогенезі тромбофілії в пацієнтки Т., 17 років, яка страждає на тромбофлебіт вен нижніх кінцівок?

A. @Недостатність антикоагулянтів

B. Анемія

C. Тромбоцитопенія

D. Тромбоцитопатія

E. Недостатність прокоагулянтів

186. З метою діагностики якого зрушення в системі гемокоагуляції призначають дослідження рівня продуктів деградації фібрину в крові?

A. @ДВЗ

B. Тромбоцитопатія

C. Тромбоцитопенія

D. Вазопатія геморагічна

E. Тромбоз.

187. У хворого діагностовано генетичний дефект мем-бранного рецептора до фактора Віллебранда – глікопротеїду (ГП) 1b, який відповідає за почат-ковий етап адгезії тромбоцитів до колагену. Як називається ця хвороба?

A. @Бернара–Сульє

B. Віллебранда

C. Аддісона–Бірмера

D. Вільсона–Коновалова

E. Гланцмана–Негелі

188. У хворого діагностовано спадковий дефект ГП IIb–IIIa – мембранного рецептора, що забезпечує поєднання фібрину з мембраною тромбоцитів і по-трібного для їх агрегації. Про яку хворобу йде мо-ва?

A. @Гланцмана–Негелі

B. Віллебранда

C. Аддісона–Бірмера

D. Вільсона–Коновалова

E. Бернара–Сульє

189. У дитини з комбінованим імунодефіцитом при об-стеженні крові виявлено зменшення адгезії тром-боцитів до колагену, їх агрегації, ослаблення коа-гуляції крові та ретракції кровяного згусткa. При якому імунодефіциті спостерігаються подібні змі-ни?

A. @Віскотта–Олдріча

B. Луї–Барр

C. Ді–Джорджі

D. Незелофа

E. Брутона

190. У хворої дівчинки діагностовано тромбастенію Гланцманa. Яке порушення в системі гемостазу має місце при цьому?

A. @Дисагрегаційна тромбоцитопатія

B. Абсолютна тромбоцитопенія

C. Дисадгезійна тромбоцитопатія

D. Дефіцитна тромбоцитопатія

E. Дисдегрануляційна тромбоцитопатія

191. Учасники експедиції, які перебували на Півночі, скаржаться на кровоточивість ясен та петехіальні крововиливи на шкірі. З анамнезу відомо – у раці-оні не вистачало аскорбінової кислоти, і це приз-вело до ламкості судинної стінки. Яка це за пато-генезом вазопатія?

A. @Диспластична

B. Запальна

C. Метапластична

D. Дистрофічна

E. Імунна

192. У хворого після тривалої операції на підшлунковій залозі, післяопераційна рана довго кровоточила. За даними коагулограми виявлено суттєве підви-щення рівня плазміну. Яка за патогенезом коагу-лопатія спостерігається в даному випадку?

A. @Фібринопатія

B. Протромбіназопатія

C. Тромбопластинопатія

D. Тромбінопатія

E. Вазопатія

193. У хворого діагностовано генетичний дефект V фа-ктора, який стає нечутливим до інактивації антит-ромботичним комплексом тромбомодулін–протеїн С, що знижує здатність судинної стінки обмежува-ти утворення фібрину. Яка патологія згортання крові виникне при цій аномалії?

A. @Тромбофілія

B. Тромбоцитопенія

C. Тромбоцитопатія

D. Геморрагічний синдром

E. ДВЗ–синдром

Серцева недостатність

194. У жінки, 22 років, діагностовано міокардит, є ознаки серцевої недостатності. Який вид серцевої недостатності за патогенезом має місце у хворої?

A. @Від ушкодження міокарда.

B. Від перевантаження серця підвищеним опором відтоку крові.

C. Від зниження об’єму циркулюючої крові.

D. Від підвищення опору в судинах малого кола.

E. Від перевантаження міокарда збільшеним об’ємом крові.

195. У хворого наслідком гіпертонічного кризу стала гостра серцева недостатність. Який головний ме-ханізм виникнення серцевої недостатності у цьому випадку?

A. @Перевантаження серця опором.

B. Абсолютна коронарна недостатність.

C. Порушення ритму серця.

D. Перевантаження серця об’ємом.

E. Ушкодження міокарда.

196. У хворого з недостатністю мітрального клапана виникла гостра серцева недостатність. Який па-тофізіологічний варіант недостатності серця спо-стерігається в цьому випадку?

A. @Від перевантаження серця.

B. Внаслідок гіпоксичного ушкодження міокарда

C. Внаслідок коронарогенного ушкодження серця

D. Внаслідок нейрогенного ушкодження серця

E. Внаслідок гострої тампонади серця

197. У хворого 65 років діагностовано повну атріо–вентрикулярну блокаду, яка ускладнилась серце-вою недостатністю. Який патогенетичний варіант серцевої недостатності у хворого?

A. @Міокардіальна, аритмічна.

B. Перевантаження серця опором.

C. Позаміокардіальна.

D. Перевантаження серця об’ємом.

E. Міокардіальна, внаслідок ушкодження міокар-да.

198. У хворого на артеріальну гіпертензію з’явилися задишка, кашель, запаморочення, акроціаноз. Яка недостатність кровообігу найбільш вірогідно розвинулася в пацієнта?

A. @Серцева, лівошлуночкова

B. Серцева, правошлуночкова

C. Коронарна

D. Серцева, від перевантаження об’ємом

E. Судинна

199. Хворий, 44 років, скаржиться на задишку, серце-биття, біль у правому підребер’ї, набряки на но-гах. При обстеженні діагностовано недостатність тристулкового клапана. Який патогенетичний ва-ріант серцевої недостатності спостерігається у хворого?

A. @Перевантаження серця об’ємом.

B. Перевантаження серця опором.

C. Позаміокардіальна.

D. Аритмічна.

E. Від ушкодження міокарду.

200. У хворого спостерігається підвищення опору ви-гнанню крові з лівого шлуночка. При якому із пе-рерахованих патологічних процесів може виник-нути така ситуація?

A. @Артеріальна гіпертензія

B. Гіпертензія малого кола кровообігу

C. Мітральна недостатність

D. Недостатність аортального клапана

E. Поліцитемія

201. У хворого спостерігається перевантаження об’ємом лівого шлуночка. При якому із перерахо-ваних патологічних процесів може виникнути така ситуація?

A. @Недостатність аортального клапана

B. Емболія легеневої артерії

C. Аортальний стеноз

D. Мітральний стеноз

E. Артеріальна гіпотензія

202. У хворого спостерігається різке підвищення арте-ріального тиску за рахунок зміни тонусу судин. Який компенсаторний механізм забезпечує збіль-шення сили скорочень міокарду у цьому випадку?

A. @Гомеометричний.

B. Пригнічення симпатичної нервової системи.

C. Гетерометричний.

D. Активація парасимпатичної нервової системи.

E. Активація ренін–ангіотензинової системи.

203. У хворого діагностована недостатність мітрально-го клапану, але ознак серцевої недостатності не виявлено. Який негайний механізм компенсації за-безпечує в такому випадку гіперфункцію серця?

A. @Гетерометричний механізм

B. Гіпертрофія лівого передсердя

C. Гіпертрофія лівого шлуночка

D. Гомеометричний механізм

E. Тахікардія

204. У хворого виявлена недостатність аортального клапана. Який головний негайний внутрішньосер-цевий механізм компенсації включається при цьо-му?

A. @Гетерометричний.

B. Гіпертрофія міокарда.

C. Хроноінотропний.

D. Гомеометричний.

E. Гіпотрофія міокарда.

205. У чоловіка, 25 років, виявлено недостатність міт-рального клапана без порушення кровообігу. Який довготривалий механізм забезпечує компен-сацію?

A. @Гіпертрофія міокарда.

B. Міогенна дилатація.

C. Гомеометричний.

D. Механізм Боудича.

E. Гетерометричний.

206. У хворого з хронічною серцевою недостатністю виявлена гіпертрофія міокарда. Які зміни харак-терні для гіпертрофованого серця?

A. @Зменшення енергозабезпечення міокардіоци-тів

B. Зменшення маси серця

C. Збільшення кількості міокардіоцитів

D. Зменшення маси міокардіоцитів

E. Зменшення залишкового діастолічного об'єму крові

207. Через місяць після виникнення експериментальної артеріальної гіпертензії в собаки товщина стінки лівого шлуночка серця збільшилась в 1,7 рази. Надалі маса серця не збільшувалась, хвилинний об’єм серця нормалізувався. Яка стадія гіпертро-фії міокарда спостерігається у тварини?

A. @Завершеної гіпертрофії.

B. Декомпенсації.

C. Прогресуючого кардіосклерозу.

D. Початкова.

E. Аварійна.

208. У тварини з недостатністю аортальних клапанів розвинулась гіпертрофія лівого шлуночка серця. В окремих його ділянках визначаються локальні контрактури. Накопичення якої речовини в карді-оміоцитах зумовило контрактури?

A. @Кальцію.

B. Молочної кислоти.

C. Натрію.

D. Вуглекислого газу.

E. Калію.

209. У хворого з недостатністю мітрального клапана виникла гіпертрофія лівого шлуночка серця. Який механізм розвитку гіпертрофії є провідним?

A. @Активація генетичного аппарату

B. Активація гліколізу.

C. Збільшення споживання жирних кислот.

D. Збільшення надходження іонів калію в клітину.

E. Збільшення клітинного дихання.

210. Кролю накладено кільце на аорту, що звузило її просвіт. Після цього маса міокарда почала швидко збільшуватись. Що є найбільш характерним для цієї стадії розвитку гіпертрофії міокарду?

A. @Збільшення навантаження на одиницю м’язової маси серця (ІФС)

B. Порушення кровопостачання міокарду

C. Зменшення симпатичної іннервації серця

D. Погіршення умов для утилізації енергії

E. Розвиток аритмій

211. У хворого 25 років після перенесеного гострого ендокардиту виникла недостатність клапанів аор-ти, що викликала гіпертрофію лівого шлуночка серця. Через вісім тижнів маса серця збільшилась на 65 % і ріст її зупинився. Що викликало припи-нення подальшої гіпертрофії?

A. @Нормалізація навантаження на одиницю м’язової маси серця

B. Погіршення пластичного забезпечення кардіо-міоцитів

C. Порушення судинного забезпечення серця

D. Погіршення енергетичного забезпечення міо-кардіоцитів

E. Порушення регуляторного забезпечення серця

212. Через 1 годину після накладання кільця, що зву-жує аорту, у собаки зросла сила та частота сер-цевих скорочень, а об’єм циркулюючої крові та товщина стінки лівого шлуночка не відрізнялися від вихідних показників. Яка стадія гіпертрофії мі-окарда спостерігається у тварини?

A. @Аварійна.

B. Завершеної гіпертрофії.

C. Відносно стійкої гіперфункції.

D. Декомпенсації.

E. Прогресуючого кардіосклерозу.

213. У дитини 14 років з відкритою артеріальною про-токою маса обох половин серця в 3 рази переви-щує вікову норму. Спостерігаються стійкий ціаноз, задишка, періодичні запаморочення, значна за-тримка фізичного розвитку. Стан дитини прогре-сивно погіршується. Яка особливість гіпертрофо-ваного міокарда сприяє виникненню декомпенса-ції?

A. @Зниження енергозабезпечення міокардіоци-тів.

B. Активація гліколізу.

C. Активація генетичного апарату.

D. Активація кальцієвих насосів.

E. Збільшення кількості судин і нервів

214. У дитини 14 років з відкритою артеріальною про-токою маса обох половин серця в 3 рази переви-щує вікову норму. Спостерігаються стійкий ціаноз, задишка, періодичні запаморочення, значна за-тримка фізичного розвитку. Стан дитини прогре-сивно погіршується. Яка стадія гіпертрофії міока-рда спостерігається у дитини?

A. @Прогресуючого кардіосклерозу.

B. Завершеної гіпертрофії.

C. Відносно стійкої гіперфункції.

D. Компенсації.

E. Аварійна.

215. У тварини з мітральним стенозом розвинулась гіпертрофія лівого передсердя. Який показник свідчить про перехід гіпертрофії у другу стадію?

A. @Нормалізація інтенсивності функціонування структур.

B. Збільшення інтенсивності функціонування структур.

C. Зменшення інтенсивності функціонування структур.

D. Збільшення маси серця на 70%.

E. Нормалізація хвилинного об’єму серця.

216. У собаки з аортальним стенозом розвинулась гі-пертрофія лівого шлуночка серця. Який показник свідчить про наявність першої стадії гіпертрофії?

A. @Збільшення інтенсивності функціонування структур.

B. Нормалізація інтенсивності функціонування структур.

C. Зменшення інтенсивності функціонування структур.

D. Збільшення маси серця на 100%.

E. Нормалізація хвилинного об’єму серця.

217. У хворого розвинулася гостра лівошлуночкова недостатність, яка швидко ускладнилася набря-ком легень. Що є головною ланкою в патогенезі кардіогенного набряку легень?

A. @Збільшення гідростатичного тиску крові

B. Збільшення онкотичного тиску тканинної рідини

C. Зменшення гідростатичного тиску крові

D. Зменшення онкотичного тиску крові

E. Зменшення онкотичного тиску тканинної рідини

218. У хворого на гіпертонічну хворобу під час фізич-ного навантаження з’явились відчуття м’язової слабкості, нестачі повітря, ціаноз, дистанційні во-логі хрипи. Що лежить в основі виникнення цих порушень?

A. @Гостра лівошлуночкова недостатність

B. Синдром реперфузії

C. Рефлекс Китаєва

D. Гостра правошлуночкова недостатність

E. Колапс

219. У хворого на гіпертонічну хворобу під час фізич-ного навантаження з’явились відчуття м’язової слабкості, нестачі повітря, ціаноз, дистанційні во-логі хрипи. Що є найтяжчим проявом лівошлуноч-кової серцевої недостатності?

A. @Набряк легень

B. Асцит

C. Стенокардія

D. Тахікардія

E. Набряк на ногах

220. У хворого на гіпертонічну хворобу під час фізич-ного навантаження з’явились відчуття м’язової слабкості, нестачі повітря, ціаноз, дистанційні во-логі хрипи. Який рефлекс запобігає розвитку на-бряку легень при лівошлуночковій недостатності серця?

A. @Китаєва

B. Пульмо–коронарний

C. Пульмо–кардіальний

D. Крачмера

E. Бейнбріджа

221. У хворого на правошлуночкову серцеву недостат-ність розвинулися асцит та набряки. Який основ-ний патогенетичний механізм розвитку набряків у цього хворого?

A. @Збільшення гідростатичного тиску крові у венах

B. Збільшення проникності стінок судин

C. Збільшення онкотичного тиску крові

D. Збільшення онкотичного тиску міжклітинної рі-дини

E. Зменшення осмотичного тиску крові

222. У хворого на ревматизм розвинувся міокардит з проявами недостатності кровообігу. Яке порушен-ня гемодинаміки спостерігається при цьому?

A. @Зниження систолічного артеріального тиску.

B. Зниження венозного тиску.

C. Підвищення систолічного артеріального тиску.

D. Зростання швидкості кровообігу.

E. Зниження діастолічного артеріального тиску.

223. Хворий, 39 років, з хронічною серцевою недоста-тністю скаржиться на задишку при невеликому фізичному навантаженні, набряки на ногах. Що може бути свідченням декомпенсації функції сер-ця у хворого?

A. @Тахікардія

B. Збільшення швидкості кровотоку

C. Анемія

D. Підвищення артеріального тиску

E. Зменшення венозного тиску

224. Хвора 45 років скаржиться на задишку при неве-ликому фізичному навантажені, набряки на ногах. Діагностовано недостатність кровообігу. Який ге-модинамічний показник декомпенсації серця спо-стерігається в даному випадку?

A. @Зменшення хвилинного об’єму серця

B. Зменшення об’єму циркулюючої крові

C. Зниження венозного тиску

D. Підвищення артеріального тиску

E. Задишка

225. Хворий 65 років скаржиться на загальну слабість, серцебиття й задишку при помірному фізичному навантаженні, набряки та ціаноз нижніх кінцівок. Діагностовано серцеву недостатність. Чим зумов-лений ціаноз у хворого?

A. @Збільшенням кількості відновленого гемогло-біну в крові

B. Збільшенням напруги кисню в артеріальній крові

C. Зменшенням напруги вуглекислого газу в ар-теріальній крові

D. Зменшенням напруги кисню в артеріальній крові

E. Зменшенням кількості гемоглобіну в крові

226. У хворого з міокардіодистрофією розширені поро-жнини серця, знижена сила серцевих скорочень, збільшилися кінцевий систолічний об’єм шлуноч-ків та кровонаповнення вен. Про який функціона-льний стан серця свідчать ці порушення?

A. @Міогенна дилатація

B. Гіпертрофія серця, аварійна стадія

C. Гіпертрофія серця, стадія завершеної гіперт-рофії

D. Гіпертрофія серця, стадія гіперфункції

E. Тоногенна дилатація

227. У хворого розвинулась серцева недостатність за лівошлуночковим типом. Які компенсаторні реак-ції в організмі хворого будуть спрямовані на запо-бігання набряку легень?

A. @Спазм артеріол та зменшення тиску в капі-лярах малого кола.

B. Розширення артеріол великого кола кровообі-гу.

C. Накопичення крові у венах великого кола кро-вообігу.

D. Підвищення проникності легеневих капілярів.

E. Підвищення центрального венозного тиску.

228. У хворого з патологією серцево–судинної системи розвинулись набряки на нижніх кінцівках. Який механізм розвитку серцевого набряку?

A. @Підвищення гідростатичного тиску на веноз-ному кінці капіляра

B. Підвищення онкотичного тиску

C. Підвищення гідростатичного тиску на артеріа-льному кінці капіляра

D. Зниження осмотичного тиску крові

E. Порушення лімфовідтоку

229. У чоловіка, віком 35 років, під час тривалого бігу сталась гостра серцева недостатність. Які зміни іонного складу спостерігаються у серцевому м’язі при цьому становищі?

A. @Накопичення в клітинах міокарду іонів Na+ і Ca2+.

B. Накопичення в клітинах міокарду іонів K+ і Mg2+.

C. Зменшення в клітинах міокарду іонів Na+ і Ca2+.

D. Зменшення в позаклітинному просторі іонів K+ і Mg2+.

E. Збільшення в позаклітинному просторі іонів Na+ і Ca2+.

230. У хворого швидко наростали симптоми недостат-ності кровообігу. Лікарі швидкої допомоги встано-вили гостру тампонаду серця. Ця патологія є ре-зультатом:

A. @Гострого ексудативного перикардиту

B. Інфаркту міокарда

C. Тромбозу легеневої артерії

D. Фібриляції шлуночків

E. Тромбозу лівого атріо – вентрикулярного отвору.

231. У хворого, що страждає на серцеву недостатність, спостерігаються збільшення печінки, набряки ни-жніх кінцівок, асцит. Який механізм є провідним в утворенні даного набряку?

A. @Застійний

B. Лімфогенний

C. Колоїдно–осмотичний

D. Мембраногенний

E. Гіперволемічний

232. У підлітка після перенесеного інфекційного захво-рювання спостерігається коливання частоти сер-цевих скорочень залежно від фази дихання. Який механізм виникнення дихальної аритмії у хворого?

A. @Коливання тонусу блукаючого нерва

B. Порушення збудливості серця

C. Рефлекс Бейнбріджа

D. Порушення скоротливої функції серця

E. Порушення провідності серця

233. Дані ЕКГ пацієнта з гіпертонічною хворобою: ритм синусовий, правильний, частота серцевих скоро-чень – 92 за 1 хв., тривалість інтервалу Р–Q 0,19 с., комплекс QRS не змінений. Яка функція серця порушена?

A. @Автоматизм

B. Провідність

C. Збудливість

D. Скоротливість

E. Збудливість і провідність

234. Після тяжкої інфекції у хворого розвинувся міока-рдит з ушкодженням провідної системи серця і виникненням синдрому Морганьї – Адамса – Стокса. Яке порушення ритму у пацієнта?

A. @Перехід неповної атріовентрикулярної бло-кади в повну.

B. Блокада правої ніжки пучка Гіса.

C. Пароксизмальна тахікардія.

D. Блокада лівої ніжки пучка Гіса.

E. Передчасне збудження шлуночків.

235. У хворого виявлено екстрасистолію. Дані ЕКГ: відсутній зубець Р, комплекс QRS деформований і розширений, наявна повна компенсаторна пауза. Вкажіть локалізацію ектопічного вогнища збу-дження в цьому випадку?

A. @Шлуночки.

B. Передсердно–шлуночковий вузол.

C. Пучок Гіса.

D. Передсердя.

E. Синусовий вузол.

236. У хворого на дифузний токсичний зоб часто вини-кають серцебиття, слабкість, дефіцит пульсу. На ЕКГ виявлено відсутність зубців Р, замість них чи-сленні хвилі f, неоднакові інтервали R–R. Яке по-рушення серцевого ритму у хворого?

A. @Миготіння передсердь

B. Пароксизмальна тахікардія

C. Синусова екстрасистолія

D. Атріовентрикулярна блокада

E. Фібриляція шлуночків

237. Нормальний ритм серця у хворої раптово перер-вався нападом скорочень частотою 200 уд/хв. тривалістю до 5 хв. Назвіть можливий наслідок тривалої пароксизмальної тахікардії.

A. @Зменшення серцевого викиду

B. Збільшення серцевого викиду

C. Збільшення ударного об’єму серця

D. Підвищення систолічного тиску

E. Збільшення коронарного кровотоку

238. На ЕКГ лікар виявив у окремих серцевих циклах скорочення інтервалу Т–Р, двофазний зубець Р, комплекс QRS без порушень, неповна компенса-торна пауза. Для якої аритмії характерні такі змі-ни на ЕКГ?

A. @Передсердної екстрасистолії

B. Синусової аритмії

C. Передсердно–шлуночкової екстрасистолії

D. Шлуночкової екстрасистолії

E. Пароксизмальної тахікардії

239. У хворого, що страждає на важку форму пору-шення водно–сольового обміну, настала зупинка серця в діастолі. Що є найбільш імовірною причи-ною зупинки серця в діастолі?

A. @Гіперкаліємія

B. Гіперкальціємія

C. Дегідратація організму

D. Гіпомагніємія

E. Гіпонатріємія

240. На ЕКГ у хворої на міокардит виявлено подов-ження PQ інтервалу до 0,4 сек. Яка властивість провідної системи порушена?

A. @Провідність

B. Рефрактерність

C. Автоматизм

D. Скоротність

E. Збудливість

241. Під час операційного втручання трапилося реф-лекторне посилення впливу блукаючого нерва на серце. Що при цьому може виникнути?

A. @Зупинка серця

B. Збільшення провідності атріовентрикулярного вузла

C. Збільшення збудливості міокарда

D. Посилення скорочень міокарда

E. Збільшення частоти серцевих скорочень

242. Електрик, випадково доторкнувся до оголеного електропроводу обома руками і загинув. Що спричинило смерть у даному випадку?

A. @Фібриляція серця

B. Синусова брадикардія

C. Повна блокада серця

D. Синусова тахікардія

E. Тампонада серця

243. У хворої, 45 років, виявлено такі зміни ЕКГ: інтер-вал Р–Q подовжений, при цьому випадає кожен другий комплекс QRST. Яке саме порушення ритму спостерігається?

A. @Неповна атріовентрикулярна блокада ІІІ ступеня.

B. Синоаурикулярна блокада.

C. Атріовентрикулярна блокада повна.

D. Внутрішньошлуночкова блокада.

E. Неповна атріовентрикулярна блокада І ступе-ня.

244. У чоловіка 50 років раптово виникли сильне сер-цебиття, біль у серці, різка слабкість, підвищення АТ; пульс аритмічний, з дефіцитом. На ЕКГ вияв-лено відсутність зубців Р і різна тривалість інтер-валів R–R. Порушення якої функції серцевого м’яза спостерігається в цьому випадку?

A. @Збудливості та провідності

B. Збудливості

C. Автоматизму

D. Провідності

E. Скоротливості

245. На електрокардіограмі виявлено екстрасистолу. Яка зміна ЕКГ характерна для усіх екстрасистол?

A. @Вкорочений Т – Р перед екстрасистолою

B. Вкорочений Т – Р після екстрасистоли

C. Подовжений Т – Р перед екстрасистолою

D. Подовжений Т – Р після екстрасистоли

E. Вкорочений Р – Q перед екстрасистолою

246. На електрокардіограмі виявлено передсердну екстрасистолу. Яка зміна ЕКГ характерна лише для передсердної екстрасистоли?

A. @Деформований і розширений зубець Р

B. Вкорочений Т – Р перед екстрасистолою

C. Деформований і розширений шлуночковий комплекс

D. Повна компенсаторна пауза

E. Відсутність зубця Р

247. Дані ЕКГ хворого з порушенням серцевого ритму: частота серцевих скорочень – 50 за 1 хв., ритм синусовий, неправильний; інтервал P–Q подовже-ний; періодичне випадання комплексу QRS (пері-оди Самойлова – Венкебаха). Яке порушення сер-цевого ритму має місце?

A. @Неповна атріовентрикулярна блокада ІІ ст.

B. Неповна атріовентрикулярна блокада І ст.

C. Повна атріовентрикулярна блокада.

D. Синдром слабкості синусового вузла.

E. Блокада правої ніжки пучка Гіса.

248. У хворої на ішемічну хворобу серця за даними ЕКГ частота серцевих скорочень – 230 за 1 хв., зубці Р деформовані, шлуночкові комплекси без змін. Яке порушення серцевого ритму має місце?

A. Фібриляція шлуночків.

B. Тремтіння передсердь.

C. @Передсердна пароксизмальна тахікардія.

D. Шлуночкова екстрасистолія.

E. Миготлива аритмія.

249. Чоловік, 57 років, скаржиться на біль у ділянці серця, який виник після негативних емоцій. Нітро-гліцерин достатньо швидко усунув біль. Який ме-ханізм ішемії найбільш імовірний?

A. @Ангіоспастичний.

B. Механічний.

C. Обтураційний.

D. Странгуляційний

E. Компресійний.

250. У хворого, що скаржиться на стенокардію у спо-кої, рівень холестерину у крові становить 8 ммоль/л. Що, найімовірніше, є причиною вінцевої недостатності у хворого?

A. @Атеросклероз вінцевих артерій

B. Фізичне навантаження

C. Стрес

D. Артеріосклероз Менкеберга

E. Міокардит

251. При ішемії у міокардіоцитах виникає контрактура міофібрил, активація фосфоліпази А2 та роз’єднання окиснення з фосфоруванням. Який механізм пошкодження клітин домінує у цьому випадку?

A. @Кальцієвий

B. Ліпідний

C. Ацидотичний

D. Електолітно–осмотичний

E. Протеїновий

252. Емоційний стрес у пацієнта похилого віку був при-чиною виникнення гострого болю за грудиною. Лікар швидкої медичної допомоги встановив іше-мічну хворобу серця. Що є причиною розвитку стенокардії у хворого?

A. @Гостра вінцева недостатність

B. Інфаркт міокарда

C. Хронічна дихальна недостатність

D. Хронічна серцева недостатність

E. Асфіксія

253. У експерименті на коронарну артерію наклали лігатуру, а через 60 хв. зняли, що призвело до збільшення зони некрозу порівняно з очікуваною. Що лежить в основі даного явища?

A. @Реперфузійний синдром

B. Резорбційно–некротичний синдром

C. Больовий синдром

D. Синдром Дресслера

E. Астенічний синдром

254. У хворого, який раптово помер, патологоанатомі-чно виявили обтуруючий тромб у лівій коронарній артерії, але не виявили некрозу. Що, найімовірні-ше, стало причиною ранньої серцевої смерті у хворого?

A. @Фібриляція шлуночків

B. Реперфузійний синдром

C. Пароксизмальна тахікардія

D. Больовий шок

E. Синдром Дресслера

255. Відновлення коронарного кровообігу у хворого на інфаркт міокарда супроводжувалося зниженням скорочувальної здатності серця і погіршенням за-гального стану пацієнта. Які реакції визначають збільшення пошкодження кардіоміоцитів в умовах реперфузії?

A. @Вільнорадикальні

B. Глікозилювання

C. Фосфорилювання

D. Трансамінування

E. Дефосфорилювання

256. Через 3 тижні після ангіни у хворого виник осере-дковий некроз міокарда. У крові виявили міокар-діотоксичні антитіла. Алергічні реакції якого типу викликали некроз серця у даному випадку?

A. @II типу (гуморальні цитотоксичні )

B. I типу (анафілактичні)

C. V типу (стимулюючі)

D. III типу (імунокомплексні)

E. IV типу (клітинні цитотоксичні)

257. Після перев’язки однієї з гілок коронарних артерій у собаки розвинувся інфаркт міокарда, який су-проводжувався проявами резорбційно–некро тичного синдрому. Яка характерна ознака розвитку цього синдрому?

A. @Збільшення вмісту креатинфосфокінази в крові.

B. Біль за грудиною.

C. Зниження хвилинного об’єму крові.

D. Підвищення рівня катехоламінів у крові.

E. Фібриляція шлуночків.

258. Хворий з інфарктом міокарда скаржиться на га-рячку, лабораторно виявлені нейтрофільний лей-коцитоз, підвищення ШОЕ. Яка речовина викли-кала ці зміни?

A. @Інтерлейкін 1

B. Гістамін

C. Інтерферон

D. Калідин

E. Лізоцим

259. У пацієнта, у якого 1,5 міс тому виник інфаркт міокарда, діагностовано синдром Дресслера з ха-рактерною тріадою: перикардитом, плевритом, пневмонією. Який головний механізм цього ускла-днення?

A. @Автоімунне ураження.

B. Зниження резистентності організму до інфекції.

C. Викид у кров міокардіальних ферментів.

D. Інтоксикація організму продуктами некрозу.

E. Активація сапрофітної мікрофлори.

260. У тварини, якій у порожнину перикарда вводили фізіологічний розчин виникла гостра тампонада серця. Який процес первинно порушується при цьому виді недостатності серця?

A. @Діастолічне розширення серця

B. Скорочення міокардіоцитів

C. Автоматизм серця

D. Проведення імпульсів

E. Збудження міокардіоцитів

261. У тварини, якій у порожнину перикарда вводили фізіологічний розчин виникла гостра тампонада серця. Який компенсаторний механізм спрацьовує при цьому виді недостатності серця?

A. @Тахікардія

B. Гомеометричний

C. Гетерометричний

D. Гіпертрофія міокарда

E. Інотропна дія катехоламінів

262. У щура під час відтворення гемічної гіпоксії вве-денням нітрату натрію на електрокардіограмі ви-никли зміщення сегменту ST, зміни комплексу QRS і зубця T. Який механізм виникнення некрозу сер-ця у даному випадку?

A. @Гіпоксичний

B. Імунний

C. Електролітно–стероїдний

D. Токсичний

E. Ішемічний

263. У експерименті після тривалої ішемії у частини міокардіоцитів були відсутні скорочення. Через деякий час після відновлення кровообігу ці кліти-ни почали скорочуватись. Як називається це явище?

A. @«Засинання» (гібернація)

B. «Приголомшеність» (stunning)

C. Реперфузійний синдром

D. Прекондиціювання

E. Контрактура

264. У експерименті перетиснули вінцеву артерію на 5 хвилин. При повторній ішемії патологічні зміни в цій зоні міокарда були виражені менше, ніж в ін-тактних. Яке явище виникло?

A. @Прекондиціювання

B. «Приголомшеність» (stunning)

C. «Засинання» (гібернація)

D. Реперфузійний синдром

E. Контрактура

265. У експерименті перетиснули вінцеву артерію на 5 хвилин. При повторній ішемії патологічні зміни в цій зоні міокарда були виражені менше, ніж в ін-тактних. Що лежить в основі цього явища?

A. @Метаболічна адаптація

B. Активація антиоксидантних ферментів

C. Активація лізосомних ферментів

D. Активація каспаз

E. Контрактура

266. У експерименті відновлення кровообігу після ко-роткочасної ішемії призвело до значного пошко-дження міокардіоцитів. Що лежить в основі репе-рфузійного пошкодження клітин?

A. @Активація каспаз

B. Активація антиоксидантних ферментів

C. Активація лізосомних ферментів

D. Метаболічна адаптація

E. Контрактура

267. У експерименті відновлення кровообігу після ко-роткочасної ішемії призвело до значного пошко-дження міокардіоцитів. Що лежить в основі репе-рфузійного пошкодження клітин?

A. @Апоптоз

B. Склероз

C. Некроз

D. Адаптація

E. Контрактура

268. В експерименті подразнюють гілочки блукаючого нерва, які іннервують серце. Це призвело до того, що припинилося проведення збудження від пе-редсердь до шлуночків. Електрофізіологічні зміни в яких структурах серця є причиною цього?

A. @Атріовентрикулярний вузол

B. Синоатріальний вузол

C. Пучок Гіса

D. Шлуночки

E. Передсердя

269. Під час реєстрації ЕКГ хворого з гіперфункцією щитоподібної залози зареєстровано збільшення частоти серцевих скорочень. Вкорочення якого елементу ЕКГ про це свідчить?

A. @Інтервал R–R

B. Інтервал P–Q

C. Сегмент P–Q

D. Інтервал P–T

E. Комплекс QRS

270. У хворого 59–ти років, директора підприємства, після перевірки податкової інспекції з'явився інте-нсивний пекучий біль, локалізований за груди-ною, який іррадіює в ліву руку. Через 15 хвилин стан хворого нормалізувався. Який можливий ме-ханізм стенокардії є провідним у цього хворого?

A. @Підвищення у крові рівня катехоламінів

B. Тромбоз коронарних судин

C. Атеросклероз коронарних судин

D. Функціональне перевантаження серця

E. Внутрішньосудинна агрегація формених елеме-нтів

271. Жінка 49–ти років звернулася до лікаря зі скар-гами на підвищену втомлюваність та появу зади-шки під час фізичного навантаження. На ЕКГ: ЧСС– 50/хв, PQ– подовжений, QRS– не змінений, кількість зубців P перевищує кількість комплексів QRS. Який вид аритмії у пацієнтки?

A. @Атріовентрикулярна блокада

B. Синоатріальна блокада

C. Екстрасистолія

D. Миготлива аритмія

E. Синусова брадикардія

Патофізіологія судин

272. Хворий 67 років страждає на атеросклероз судин головного мозку. При обстеженні виявлено гіпер-ліпопротеїнемію IIб типу. Який клас ліпопротеїнів плазми крові значно підвищений у хворого?

A. @Ліпопротеїни низької щільності

B. Ліпопротеїни високої щільності

C. Альфа–ліпопротеїни

D. Хіломікрони

E. Комплекси жирних кислот з альбумінами

273. Після тривалого введення великих доз ергокаль-циферолу у тварини виникли склерозування та кальциноз середньої оболонки артерій. Який па-тологічний процес моделювали в експерименті?

A. @Артеріосклероз Менкеберга

B. Атеросклероз Маршана

C. Артеріолосклероз

D. Вікові зміни артерій

E. Ендартеріїт

274. У дитини 10 років з сухожильними ксантомами та тяжкими нападами стенокардії виявлено значне збільшення вмісту ліпопротеїнів низької щільності у плазмі крові. Який дефект лежить в основі ви-никнення первинної гіперліпопротеїнемії у хворо-го?

A. @Дефект рецептора апоВ/Е

B. Дефіцит ліпопротеїнліпази

C. Підвищена продукція апоВ

D. Аномальний апоЕ

E. Глікерування апоВ/Е

275. У хворого при ангіографії виявили значне атеро-склеротичне ураження ниркових артерій. До яких наслідків призводить ця патологія?

A. @Артеріальна гіпертензія

B. Ішемічна хвороба серця

C. Печінкова недостатність

D. Геморагічний інсульт

E. Дихальна недостатність

276. У хворого на цукровий діабет виявлено інтенсив-ний прогресуючий розвиток атеросклерозу, що проявлялося розвитком ішемічної хвороби серця, стенозуванням ниркових артерій, порушеннями мозкового кровообігу. Що є найбільш імовірною причиною розвитку атеросклерозу ?

A. @Глікерування апоВ/Е

B. Надмірне вживання холестерину з їжею

C. Ушкодження судин токсичними речовинами

D. Дефекти ліпопротеїдліпази

E. Спадкові дефекти рецепторів апоВ/Е

277. У тварини змоделювали артеріосклероз Менке-берга. Що є його основними проявами?

A. @Кальциноз та склероз медії

B. Кальциноз та склероз інтими

C. Ліпідоз інтими

D. Ліпідоз медії

E. Інфільтрація інтими

278. Хворий 67 років страждає на атеросклероз судин головного мозку. При обстеженні виявлено гіпер-холестеринемію. Який клас ліпопротеїнів плазми крові найбільш імовірно значно знижений у хворо-го?

A. @Ліпопротеїни високої щільності

B. Ліпопротеїни низької щільності

C. Ліпопротеїни дуже низької щільності

D. Хіломікрони

E. Комплекси жирних кислот з альбумінами

279. У кроля, якого тримали на раціоні з великим вміс-том тригліцеридів, виникло атеросклеротичне ураження артерій. Що було головною причиною розвитку атеросклерозу в даному випадку?

A. @Ендогенна гіперхолестеринемія

B. Екзогенна гіперхолестеринемія

C. Первинне пошкодження судинної стінки

D. Генетичний дефект рецепторів апоВ/Е

E. Генетичний дефект ліпопротеїнліпази

280. У хворого на цукровий діабет виявлено інтенсив-ний прогресуючий розвиток атеросклерозу, що проявлялося розвитком ішемічної хвороби серця, стенозуванням ниркових артерій, порушеннями мозкового кровообігу. Що є найбільш імовірною причиною розвитку атеросклерозу?

A. @Поява модифікованих ліпопротеїнів

B. Надмірне вживання холестерину з їжею

C. Ушкодження судин токсичними речовинами

D. Дефекти ліпопротеїдліпази

E. Спадкові дефекти рецепторів апоВ/Е

281. У хворого при ангіографії виявили значне атеро-склеротичне ураження внутрішніх сонних артерій. До яких наслідків може призвести ця патологія?

A. @Ішемічний інсульт

B. Ішемічна хвороба серця

C. Артеріальна гіпотензія

D. Хронічна ниркова недостатність

E. Інфаркт міокарда

282. У хворого, 43 років, артеріальна гіпертензія є на-слідком помірного збільшення серцевого викиду і загального периферичного опору. Який гемодина-мічний варіант артеріальної гіпертензії має місце в цьому випадку?

A. @Еукінетичний.

B. Дискінетичний.

C. Гіпокінетичний.

D. Змішаний.

E. Гіперкінетичний.

283. Дівчина астенічної конституції скаржиться на за-гальну слабкість, швидку втомлюваність, змен-шення працездатності. Об’єктивно: пульс – 98 за 1 хв., АТ – 90/60 мм. рт. ст. Який механізм зумо-вив гіпотензію у хворої?

A. @Зниження периферійного судинного опору

B. Депонування крові в портальній системі

C. Збільшення серцевого викиду

D. Збільшення хвилинного об’єму крові

E. Активація симпатичної нервової системи

284. У хворого виявлено артеріальну гіпертензію з гі-покінетичним типом кровообігу. Який фактор най-більш істотно впливає на артеріальний тиск?

A. Ударний об’єм серця

B. @Периферичний судинний опір

C. Частота серцевих скорочень

D. Венозне повернення до серця

E. Хвилинний об’єм серця

285. У хворого з гіпертонічною хворобою різко підви-щився артеріальний тиск до 180/105 мм рт. ст. Яке ускладнення, найбільш вірогідно, може вини-кнути у хворого в цьому випадку?

A. @Гостра серцева недостатність

B. Гостра ниркова недостатність

C. Хронічна ниркова недостатність

D. Хронічна серцева недостатність

E. Хронічна дихальна недостатність

286. При критичному зниженні температури тіла в тре-тьому періоді лихоманки на фоні тахікардії АТ = 80 / 60 мм рт. ст. До якої форми порушення судинного тонусу належить це явище?

A. @Гострої гіпотензії

B. Хронічної гіпертензії

C. Гострої гіпертензії

D. Хронічної гіпотензії

E. Гіпотонічної хвороби

287. У хворого з гіпертонічною хворобою різко підви-щився артеріальний тиск до 210/150 мм рт. ст. Яке ускладнення, найбільш вірогідно, може вини-кнути у хворого в цьому випадку?

A. @Геморагічний інсульт

B. Гостра ниркова недостатність

C. Хронічна серцева недостатність

D. Гіпотрофія лівого шлуночка серця

E. Гіпотрофія міокарда

288. У чоловіка, що тривалий час страждає на артері-альну гіпертензію, останнім часом систолічний тиск почав знижуватися, а діастолічний залиша-ється підвищеним. Який гемодинамічний тип арте-ріальної гіпертензії у хворого?

A. @Гіпокінетичний.

B. Дискінетичний.

C. Еукінетичний.

D. Гіперкінетичний.

E. Нормокінетичний.

289. Хворий страждає на гіпертонічну хворобу 5 років. Яке ускладнення, найбільш вірогідно, може вини-кнути в цьому випадку?

A. @Гіпертрофія лівого шлуночка серця

B. Гіпертрофія правого шлуночка серця

C. Гіпотрофія лівого шлуночка серця

D. Гостра ниркова недостатність

E. Жовтяниця

290. У чоловіка, що 15 років хворіє на гіпертонічну хворобу, виявлено гіпокінетичний тип кровообігу. Що є причиною стійкого збільшення судинного опору при цьому?

A. @Артеріолосклероз

B. Атеросклероз

C. Артеріосклероз Менкеберга

D. Розширення судин

E. Підвищена в’язкість крові

291. У хворого, 18 років, артеріальна гіпертензія є на-слідком значного збільшення серцевого викиду при нормальному периферичному опорі судин. Який гемодинамічний варіант артеріальної гіпер-тензії має місце в цьому випадку?

A. @Гіперкінетичний.

B. Дискінетичний.

C. Гіпокінетичний.

D. Змішаний.

E. Еукінетичний.

292. У хворого на стійку артеріальну гіпертензію знач-но погіршився стан під час лікування артриту ін-дометацином. Що призвело до посилення артеріа-льної гіпертензії?

A. @Гіпопродукція простагландинів.

B. Гіперпродукція реніну.

C. Гіпопродукція кінінів.

D. Гіперпродукція кортизолу.

E. Гіперпродукція альдостерону.

293. У тварини, якій видалили обидві нирки, на фоні регулярного гемодіалізу виникла стійка артеріа-льна гіпертензія. Що призвело до підвищення ар-теріального тиску?

A. @Гіпопродукція простагландинів і кінінів.

B. Гіперпродукція реніну.

C. Гіперпродукція кінінів і простагландинів.

D. Гіперпродукція кортизолу.

E. Гіперпродукція альдостерону.

294. Артеріальна гіпертензія у хворого зумовлена ви-раженим атеросклерозом ниркових артерій. Гіпе-рактивність якої системи є головною ланкою в па-тогенезі цієї форми гіпертензії?

A. @Ренін–ангіотензинової

B. Симпато–адреналової

C. Калікреїн–кінінової

D. Парасимпатичної

E. Гіпоталамо–гіпофізарної

295. У хворого на феохромоцитому (пухлину мозкової речовини наднирників) розвинулася артеріальна гіпертензія. Що стало причиною підвищення арте-ріального тиску?

A. @Збільшення хвилинного об’єму серця

B. Гіпопродукція простагландинів

C. Вазодилатація

D. Гіпернатріємія

E. Гіповолемія

296. У хворого на мікседему розвинулася артеріальна гіпотензія. Що стало причиною зниження артеріа-льного тиску?

A. @Зниження хвилинного об’єму серця

B. Спазм судин

C. Тахікардія

D. Гіпонатріємія

E. Гіперволемія

297. У жінки 25 років, що скаржиться на болі в шиї та голові, виявлені при обстеженні остеохондроз шийного відділу хребта та нестійка артеріальна гіпертензія. Який різновид симптоматичної гіпер-тензії у хворої?

A. @Центрально–ішемічна.

B. Невротична.

C. Рефлексогенна.

D. Реноваскулярна.

E. Ренопривна.

298. У хворого на синдром Конна (первинний гіпераль-достеронізм) розвинулася артеріальна гіпертензія. Що стало причиною підвищення артеріального ти-ску?

A. @Гіпернатріємія

B. Гіперкаліємія

C. Гіперкальціємія

D. Ацидоз

E. Гіповолемія

299. У хворого на синдром Іценка – Кушинга розвину-лася артеріальна гіпертензія. Що є механізмом підвищення артеріального тиску?

A. @Гіперволемія

B. Гіповолемія

C. Зменшення опору судин

D. Депонування крові у портальній системі

E. Гіпогідрія

300. У чоловіка з хворобою Аддісона травма виклика-ла значне погіршення стану. Які зміни в системі кровообігу характерні для таких хворих?

A. @Гіпотензія

B. Гіпертензія

C. Гіперволемія

D. Гіперглікемія

E. Гіпертрофія міокарда

301. У хворого на Базедову хворобу розвинулася ар-теріальна гіпертензія. Що стало причиною підви-щення артеріального тиску?

A. @Збільшення хвилинного об’єму серця

B. Зменшення опору судин

C. Гіповолемія

D. Активація калікреїн–кінінової системи

E. Системна дегрануляція тканинних базофілів

302. За концепцією Р. Росса атеросклероз є проявом хронічного запалення. Що є пусковим механізмом його розвитку?

A. @Ушкодження ендотеліоцитів

B. Гіподинамія

C. Надлишкове вживання жирів

D. Надлишкове вживання вуглеводів

E. Стрес

303. При атеросклерозі в інтимі уражених артерій міс-тяться «пінисті» клітини. Які клітини є їх поперед-никами?

A. @Макрофаги

B. Ендотеліоцити

C. Тромбоцити

D. Еритроцити

E. Лімфоцити

304. Дефіцит ліпопротеїнів високої щільності (ЛПВЩ) сприяє атеросклерозу. Який процес безпосеред-ньо пригнічується при зниженні ЛПВЩ у крові?

A. @Транспорт холестерину з тканин у печінку

B. Транспорт холестерину з печінки у тканини

C. Рецептор–опосередковане захоплення ліпоп-ротеїнів

D. Транспорт екзогенного холестерину у тканини

E. Транспорт тригліцеридів у тканини

305. Цукровий діабет сприяє атеросклерозу, оскільки при ньому з’являються модифіковані ліпопротеїни низької щільності. Який процес безпосередньо пригнічується при цьому?

A. @Рецептор–опосередкований транспорт ліпо-протеїнів

B. Транспорт холестерину з тканин у печінку

C. Нерегульований ендоцитоз ліпопротеїнів

D. Транспорт екзогенного холестерину у тканини

E. Транспорт тригліцеридів у тканини

306. У кролів Ватанабе внаслідок спадкового дефекту апоВ/Е–рецепторів виникає значна гіперхолесте-ринемія та швидко прогресує атеросклероз. Який процес первинно пригнічується при цьому, сприя-ючи розвитку атеросклерозу?

A. @Рецептор–опосередкований транспорт ліпо-протеїнів

B. Транспорт холестерину з тканин у печінку

C. Нерегульований ендоцитоз ліпопротеїнів

D. Транспорт екзогенного холестерину у тканини

E. Транспорт тригліцеридів у тканини

307. У кролів при згодовуванні холестерину (за М.М. Аничковим та С.С. Халатовим) виникає значна гі-перхолестеринемія та швидко прогресує атеро-склероз. Який процес, що сприяє розвитку атеро-склерозу, значно активується при цьому?

A. @Нерегульований ендоцитоз ліпопротеїнів

B. Транспорт холестерину з тканин у печінку

C. Рецептор–опосередкований транспорт ліпоп-ротеїнів

D. Синтез холестерину у клітинах

E. Транспорт тригліцеридів у тканини

308. У хворого на бронхіальну астму виявлено легене-ву гіпертензію. Яку зміну в організмі спричинює підвищення тиску в легеневій артерії?

A. @Правошлуночкову серцеву недостатність

B. Лівошлуночкову серцеву недостатність

C. Вінцеву недостатність

D. Ниркову недостатність

E. Печінкову недостатність

309. У хворого виявлено первинну легеневу гіпертен-зію. Який фактор є провідним у її розвитку?

A. @Зниження рО2 у альвеолах

B. Збільшення рО2 у альвеолах

C. Зниження рСО2 у альвеолах

D. Збільшення рН

E. Алкалоз

310. Пацієнтка, дівчина астенічної конституції, скар-житься на загальну слабкість, швидку втомлюва-ність, зменшення працездатності. При обстеженні: пульс – 98 за 1 хв., артеріальний тиск – 90/50 мм. рт. ст. Який механізм зумовив розвиток первинної артеріальної гіпотензії у хворої?

A. @Пригнічення судинорухового центру

B. Збудження судинорухового центру

C. Зменшення серцевого викиду

D. Зменшення загального об’єму крові

E. Збільшення опору судин

311. Пацієнтка, хвора на гастроентерит, скаржиться на загальну слабкість, повторну блювоту, діарею. При обстеженні: пульс – 100 за 1 хв., артеріаль-ний тиск – 90/70 мм. рт. ст. Який механізм є про-відним у зниженні артеріального тиску в хворої?

A. @Зменшення загального об’єму крові

B. Збудження судинорухового центру

C. Пригнічення судинорухового центру

D. Зменшення насосної функції серця

E. Збільшення опору судин

Патофізіологія дихання

312. У недоношеної дитини при народженні діагносту-вали рестриктивну дихальну недостатність. Яка причина лежить в основі недостатності дихання в цьому випадку?

A. @Дефіцит сурфактанта

B. Набряк легенів

C. Внутрішньоутробна гіперкапнія

D. Потрапляння навколоплідних вод

E. Токсоплазмоз

313. При обстеженні у хворого виявили обструктивну форму дихальної недостатності. Яка причина мог-ла б її викликати?

A. @Запалення бронхів

B. Запалення легенів

C. Порушення цілісності плевральної порожнини

D. Зниження утворення сурфактанта

E. Пневмосклероз

314. У непритомного юнака, що отруївся морфіном, спостерігається рідке поверхневе дихання. Яка дихальна недостатність виникла при цьому?

A. @Вентиляційна дисрегуляторна

B. Вентиляційна рестриктивна

C. Вентиляційна обструктивна

D. Перфузійна

E. Дифузійна

315. У хворого з поліомієлітом діагностовано значні патологічні зміни альвеолярної вентиляції. Пору-шення функції якої ланки системи зовнішнього дихання спостерігається в цьому випадку?

A. @Мотонейронів спинного мозку

B. Дихального центру

C. Нервово–м’язового синапсу

D. Дихальних м’язів

E. Легеневої паренхіми

316. Хворий з мітральним стенозом скаржиться на ва-жку задишку, ціаноз, кашель. Яка дихальна недо-статність спостерігається при цьому?

A. @Перфузійна

B. Вентиляційна рестриктивна

C. Вентиляційна обструктивна

D. Вентиляційна дисрегуляторна

E. Дифузійна

317. При обстеженні хворого виявили перфузійну фо-рму дихальної недостатності. Яка причина могла її викликати?

A. @Емболія легеневої артерії

B. Порушення цілісності плевральної порожнини

C. Зниження утворення сурфактанту

D. Запалення бронхів

E. Емфізема

318. У шахтаря виявлений фіброз легенів, який супро-воджується порушенням альвеолярної вентиляції. Який провідний механізм виникнення вентиляцій-ної недостатності в цьому випадку?

A. @Зменшення розтяжності легенів

B. Порушення нервової регуляції дихання

C. Обмеження рухливості грудної клітки

D. Спазм бронхів

E. Звуження верхніх дихальних шляхів

319. У підлітка, 12 років, виник важкий напад бронхіа-льної астми: виражена експіраторна задишка, блідість шкіри. Який вид порушення альвеолярної вентиляції має місце?

A. @Обструктивний

B. Центральний

C. Нервово–м’язовий

D. Торакодіафрагмальний

E. Рестриктивний

320. У хворого на цукровий діабет виникла діабетична кома, яка супроводжувалася глибоким гучним ди-ханням. Як називається такий тип дихання?

A. @Гіперпное

B. Поліпное

C. Агональне дихання

D. Періодичне дихання

E. Брадипное

321. У хворого, 62 років, після мозкового крововиливу спостерігається наростання глибини і частоти ди-хання, а потім зменшення до повного припинення, після чого цикл дихальних рухів відновлюється. Який тип дихання виник у хворого?

A. @Чейна–Стокса

B. Апнейстичне

C. Куссмауля

D. Гаспінг–дихання

E. Біотта

322. У результаті виробничої травми у хворого вияв-лено перелом декількох ребер. Який характер ди-хання за звичай спостерігається в таких випад-ках?

A. @Тахіпное

B. Гіперпное

C. Еупное

D. Брадипное

E. Апное

323. Анафілактичний шок у морської свинки зумовив зміну характеру дихання. Який тип дихання спо-стерігається в цьому випадку?

A. @Експіраторна задишка

B. Інспіраторна задишка

C. Стенотичне

D. Кусмауля

E. Брадипное

324. Хворий з черепно–мозковою травмою госпіталізо-ваний у важкому стані. Його дихання характери-зується судомним подовженим вдихом, який пе-реривається рідкими короткими видихами. Для якого типу дихання це характерно?

A. @Апнейстичного

B. Чейна–Стокса

C. Гаспінг–дихання

D. Біотта

E. Куссмауля

325. У хворого з двостороннім запаленням легенів з’явилась задишка. Назвіть найбільш суттєву ознаку для визначення диспное.

A. @Відчуття нестачі повітря

B. Збільшення глибини дихання

C. Зміна частоти дихання

D. Зміна ритму дихання

E. Збільшення хвилинного об’єму дихання

326. У тварин було перерізано обидва блукаючих нер-ва. Яким стане дихання в цьому випадку?

A. @Рідким та глибоким

B. Періодичним

C. Рідким та поверхневим

D. Частим та поверхневим

E. Частим та глибоким

327. При обстеженні у хворого діагностували обструк-цію нижніх дихальних шляхів. Яке порушення ди-хання виникло?

A. @Експіраторна задишка

B. Інспіраторна задишка

C. Тахіпное

D. Апнейстичне

E. Гаспінг

328. Хворий був госпіталізований в стані діабетичної коми. Дихання шумне, часте, після глибокого вди-ху йде посилений видих. Який тип дихання спо-стерігається у цього хворого?

A. @Куссмауля

B. Біотта

C. Апнейстичне

D. Чейна–Стокса

E. Гаспінг

329. У хворого в стані агонії після короткочасної зупи-нки дихання почали реєструватися поодинокі зіт-хання, сила і частота яких поступово зменшува-лися. Якому типу дихання відповідають подібні прояви?

A. @Гаспінг–диханню

B. Апнейстичному

C. Чейна–Стокса

D. Біотта

E. Куссмауля

330. У хворого на дифтерію виник набряк гортані. На-звіть характерний тип дихання при цьому.

A. @Рідке та глибоке (стенотичне)

B. Часте та глибоке

C. Часте та поверхневе

D. Рідке та поверхневе

E. Біотта

331. Під час операції неадекватно висока вентиляція легенів ускладнилась розвитком судом у пацієнта. Який розлад кислотно–основного стану може бути причиною їх розвитку?

A. @Алкалоз газовий

B. Ацидоз газовий

C. Ацидоз метаболічний

D. Ацидоз видільний

E. Алкалоз метаболічний

332. У хворого 76 років виявлено значне збільшення залишкового об’єму легенів. Яке захворювання може викликати ці зміни?

A. @Емфізема

B. Пневмонія

C. Плеврит

D. Бронхіт

E. Туберкульоз

333. Під час аускультації хворого попросили глибоко дихати. Після 5 хвилин інтенсивних дихальних ру-хів хворий відчув запаморочення. Це викликано:

A. @Гіпокапнією

B. Гіпероксією

C. Розширенням судин мозку

D. Перерозподілом крові в нижні кінцівки

E. Гіпоксією

334. У травмпункт доставили потерпілого з травматич-ним ураженням грудної клітки. На рентгенограмі виявлено газ у плевральній порожнині. Назвіть характерну для пневмотораксу ознаку.

A. @Зниження транспульмонального тиску

B. Періодичне дихання

C. Пригнічення дихального центру

D. Порушення прохідності трахеї

E. Підвищення транспульмонального тиску

335. У хворого внаслідок патологічного процесу збіль-шилась товщина альвеолокапілярної мембрани. Безпосереднім наслідком цього буде зменшення:

A. @Дифузійної здатності легенів

B. Альвеолярної вентиляції

C. Кисневої ємності крові

D. Резервного об’єму видиху

E. Хвилинного об’єму дихання

336. Хворий на хронічний бронхіт скаржиться на зади-шку під час фізичного навантаження, постійний кашель з виділенням мокроти. При обстеженні ді-агностовано ускладнення – емфізема легенів. Во-на зумовлена зменшенням:

A. @Еластичних властивостей легенів

B. Розтяжності легенів

C. Залишкового об’єму легенів

D. Дифузійної здатності легенів

E. Перфузії легенів

337. У хворого на туберкульоз легенів спостерігається значний дефіцит сурфактанту. Які патологічні змі-ни в легеневій тканині можуть виникнути в цьому випадку?

A. @Ателектаз

B. Бронхоспазм

C. Емфізема

D. Набряк

E. Лімфостаз

338. У хворого внаслідок хронічного захворювання ор-ганів дихання на тлі задишки, тахікардії та ціанозу під час дослідження газового складу крові вияв-лено розвиток гіпоксемії та гіперкапнії. Яке пору-шення зовнішнього дихання спостерігається у цього хворого?

A. @Гіповентиляція

B. Гіпервентиляція

C. Гіпердифузія

D. Гіперперфузія

E. Гіпоперфузія

339. У хворого в стані спокою значно збільшена робо-та дихальних м’язів. Що з нижче перерахованого може бути причиною цього явища?

A. @Обструкція дихальних шляхів

B. Пригнічення дихального центру

C. Негативний плевральний тиск

D. Гіпокапнія

E. Алкалоз

340. У робітника цементного заводу, що хворіє на пне-вмоконіоз, спостерігається задишка, акроціаноз. Що є причиної цього?

A. @Зниження дифузійної здатності легенів

B. Обструкція великих бронхів

C. Еритроцитоз

D. Токсична дія SiO2 на дихальний центр

E. Параліч діафрагми

341. До відділення професійної патології поступив шах-тар з діагнозом пневмоконіоз. Яке порушення не-респіраторних функцій легень передувало пнев-моконіозу?

A. @Захисної.

B. Фільтраційної.

C. Екскреторної.

D. Всмоктувальної.

E. Метаболічної.

342. Хворий 30 років страждає на бронхіальну астму. Який типовий патологічний процес є головним при цій хворобі?

A. @Алергія.

B. Парабіоз.

C. Запалення.

D. Патологічна домінанта.

E. Стрес.

343. У досліді на собаці вивчали вплив різних гумора-льних факторів на дихальний центр. Який із чин-ників крові найбільше збуджує безпосередньо ре-цептори дихального центру?

A. @Підвищення концентраціі Н+

B. Зниження напруги СО2

C. Зниження напруги О2

D. Підвищення концентрації N2

E. Підвищення напруги O2

344. У хворого з фіброзом легень знижена легенева вентиляція. Яка зміна показника зовнішнього ди-хання є причиною цього явища?

A. @Зменшення життєвої ємкості легень

B. Збільшення резервного об’єму вдиху

C. Збільшення резервного об’єму видиху

D. Збільшення залишкового об’єму

E. Збільшення об’єму мертвого простору

345. У хворого із вираженим пневмосклерозом після перенесеного інфільтративного туберкульозу ле-гень розвинулась дихальна недостатність. До яко-го патогенетичного типу вона відноситься?

A. @Рестриктивного

B. Обструктивного

C. Дисрегуляційного

D. Дифузійного

E. Перфузійного

346. У хворого на енцефаліт спостерігаються дихальні рухи постійної амплітуди, які чергуються з періо-дами апное. Для якого типу дихання це притаман-не?

A. @Біота

B. Чейн–Стокса

C. Куссмауля

D. Апнейстичного

E. Стенотичне

347. У чоловіка внаслідок ДТП пошкоджена грудна клітка. Він відчуває інтенсивний біль при диханні. Яке порушення дихання буде у хворого?

A. @Тахіпное.

B. Брадіпное.

C. Гіпериное.

D. Інспіраторна задишка.

E. Експіраторна задишка.

348. У хворого на інсулінзалежний цукровий діабет є прояви кетоацидозу. Які зміни зовнішнього дихан-ня найімовірніше будуть у хворого?

A. @Гіперпное.

B. Тахіпное.

C. Брадіпное.

D. Дихання Чейна–Стокса.

E. Дихання Біота.

349. У хворого з емболією легеневої артерії розвинувся респіраторний алкалоз. Що є безпосередньою причиною цього?

A. @Гіпервентиляція

B. Гіповентиляція

C. Гіперкапнія

D. Артеріальна гіпоксемія

E. Брадипное

350. У хворого на цукровий діабет 1 типу на тлі арте-ріальної гіпотензії розвинулося патологічне гіпер-пное. Що спричинило розвиток дихання Куссмау-ля у хворого?

A. @Ацидоз метаболічний

B. Гіперглікемія

C. Дефіцит утворення енергії

D. Дефіцит інсуліну

E. Ацидоз газовий

351. У підлітка 12–ти років, який хворіє на бронхіальну астму, виник тяжкий напад астми: виражена екс-піраторна задишка, блідість шкірних покривів. Який вид порушення альвеолярної вентиляції має місце?

A. @Обструктивний

B. Рестриктивний

C. Торако–діафрагмальний

D. Нервово–м'язовий

E. Центральний

Патофізіологія травлення

352. Після отруєння фосфорорганічними речовинами у хворого виникло тривале підвищення слиновиді-лення. До якого порушення в організмі може при-звести гіперсалівація?

A. @Нейтралізації шлункового соку

B. Підсилення травлення у шлунку

C. Гіпоосмолярної дегідратації

D. Гіпоосмолярної гіпергідратації

E. Пригнічення пристінкового травлення

353. У чоловіка, якому 5 років тому видалили шлунок, стоматолог виявив атрофічно–запальні процеси слизової ротової порожнини: яскраво–червоний, гладенький («лакований») язик, гінгівіт. Чим зу-мовлені ці зміни?

A. @Дефіцитом вітаміну В12.

B. Психоемоційним перенапруженням.

C. Дефіцитом вітаміну В6.

D. Дефіцитом вітаміну С.

E. Гіперсалівацією.

354. У тварини в експерименті індукували розвиток карієсу. Який найважливіший компенсаторний ме-ханізм включається при розвитку цього захворю-вання?

A. @Утворення вторинного дентину

B. Пригнічення фагоцитозу

C. Гіпотрофія слинних залоз

D. Гіперфункція паращитоподібних залоз

E. Новоутворення емалі

355. Хворий звернувся до лікаря зі скаргами на гнійні виділення з ясенних кишень і розхитування зубів. Що є причиною гнійного пародонтиту у пацієнта?

A. @Бактеріальна інфекція

B. Вірусна інфекція

C. Гиперсалівація

D. Місцева анафілаксія

E. Генералізована анафілаксія

356. До гастроентерологічного відділення потрапив хворий 57 років з підозрою на синдром Золлінгера – Еллісона, про що свідчило різке збільшення рів-ня гастрину у сироватці крові. Яке порушення се-креторної функції шлунка найбільш імовірне в цього хворого?

A. @Гіперсекреція гіперацидна

B. Ахілія

C. Гіпосекреція гіперацидна

D. Гіпосекреція гіпоацидна

E. Гіперсекреція гіпоацидна

357. У хворого, який отримав тривалий курс лікування глюкокортикоїдами, виявлені виразки шлунка. Який механізм є головним у їх розвитку?

A. @Зменшення регенерації епітелію шлунка

B. Зменшення секреції шлункового соку

C. Зменшення секреції гастрину

D. Збільшення секреції простагландинів Е1, Е2

E. Підвищення тонусу парасимпатичної нервової системи

358. Хворий скаржиться на нудоту, печію, біль у епіга-стральній області, закріп, схуднення. При обсте-женні загальна НСl – 96 ммоль/л, вільна – 60 ммоль/л. Який стан кислотоутворюючої функції у даного хворого?

A. @Гіперацидний

B. Ахілія

C. Гіпоацидний

D. Анацидний

E. Нормацидний

359. Для моделювання виразки шлунку тварині ввели в шлункові артерії атофан, який викликає їх скле-розування. Який механізм пошкодження слизової оболонки шлунка є провідним у даному випадку?

A. @Гіпоксичний

B. Механічний

C. Нейродистрофічний

D. Дисрегуляторний

E. Нейрогуморальний

360. У пацієнта 35 років, хворого на невроз, виявили виразкову хворобу. Який чинник, найімовірніше, міг стати причинним у виникненні цього захворю-вання?

A. @Порушення кірково–підкіркових взаємовідно-син

B. Зменшення концентрації адреналіну в крові

C. Зниження продукції глюкокортикоїдів

D. Порушення режиму харчування

E. Зниження впливу парасимпатичної нервової си-стеми

361. У хворого з синдромом Zollinger–Ellison (пухлина з G–клітин з локалізацією в підшлунковій залозі) відзначається збільшення секреції HCl та пептичні виразки. Яка з перерахованих речовин викликає цей комплекс симптомів?

A. @Гастрин

B. Вазоактивний інтестинальний пептид

C. Пепсин

D. Трипсин

E. Секретин

362. У хворої В., яка довгий час приймала саліцилати для лікування ревматизму, під час фіброгастрос-копії знайдена дуоденальна виразка. Який про-відний механізм розвитку виразкової хвороби в даному випадку?

A. @Зниження синтезу простагландинів

B. Зниження продукції муцину

C. Зниження секреції пепсину

D. Зниження секреції соляної кислоти

E. Зниження моторики шлунка

363. У жінки 52 років з хворобою Іценка – Кушинга підвищена кислотність шлункового соку. Який ме-ханізм забезпечив підвищення секреції в шлунку?

A. @Зниження синтезу простагландинів

B. Посилення секреції секретину

C. Гіпосекреція гастрину

D. Збільшення вмісту гістаміну

E. Зменшення виділення гастрину

364. Аналіз матеріалу біопсії слизової оболонки шлунка людини, хворої на хронічний гастрит, показав різ-ке зменшення кількості парієнтальних клітин. Як це відбивається на показниках аналізу шлунково-го соку?

A. @Зменшення кислотності

B. Збільшення кислотності

C. Зменшення продукції слизу

D. Збільшення вільної хлористоводневої кислоти

E. Зменшення продукції ферментів

365. У хворого зі скаргами на оперізуючий біль в епіга-стральній області, повторну блювоту різко знизи-вся системний артеріальний тиск. При лаборатор-ному обстеженні знайдено підвищений вміст діас-тази в сечі. Про яке ускладнення гострого пан-креатиту повинен подумати лікар у першу чергу?

A. @Панкреатичний шок

B. Гострий апендицит

C. Виразку шлунка

D. Панкреатичну недостатність

E. Діарею

366. У хворої зі скаргами на оперізуючий біль в епігас-тральній області при лабораторному обстеженні знайдено підвищений вміст діастази в сечі, а та-кож стеаторея. Для якої форми патології травного каналу найбільш характерні такі явища?

A. @Гострий панкреатит

B. Гострий апендицит

C. Виразкова хвороба шлунка

D. Гострий коліт

E. Інфекційний гепатит

367. У жінки віком 67 років, яка тривалий час страж-дала на холецистит, після їжі раптово виник різ-кий біль у верхній частині живота, нудота, блю-вання. Встановлено діагноз – гострий панкреатит. Що є основною ланкою патогенезу цього захво-рювання?

A. @Передчасна активація ферментів підшлунко-вої залози

B. Зниження рівня ферментів у панкреатичному соку

C. Підвищення активації ферментів у дванадцяти-палій кишці

D. Зниження секреції панкреатичного поліпептиду

E. Підвищення рівня холецистокініну

368. У хворого виявлений синдром подразненої кишки, який характеризується значним посиленням мото-рики. Виділення якого інкрету найбільш імовірно збільшується при цьому?

A. @Мотиліну

B. Інтестинального пептиду

C. Глюкагону

D. Урогастрону

E. Секретину

369. До дитячої лікарні потрапила 8–місячна дитина з діареєю, здуттям живота, гіпотрофією. Симптоми з¢явилися і почали наростати після введення в ра-ціон харчування борошняних виробів. Діагносто-вано целіакія. Який процес у травному каналі по-рушений?

A. @Всмоктування в кишках

B. Секреція шлункового соку

C. Евакуація шлункового вмісту

D. Секреція жовчі

E. Секреція підшлункового соку

370. Хворий, що лікувався антибіотиками з приводу пневмонії, скаржиться на прискорене виділення рідких випорожнень із специфічним запахом, біль і бурчання в животі. Що, найімовірніше, стало причиною розвитку проносу?

A. @Дисбактеріоз

B. Механічна стимуляція моторики

C. Імунне пошкодження кишок

D. Зниження моторики кишок

E. Гіперосмолярність химуса

371. У новонародженої дитини діагностована спадкова недостатність лактази. Який процес при цьому був порушений у дитини?

A. @Мембранне травлення

B. Порожнинне травлення

C. Екскреторна функція кишок

D. Секреторна функція підшлункової залози

E. Секреція шлункового соку

372. У хворого, що тривалий час страждає на хроніч-ний ентероколіт, після вживання молока виникли метеоризм, діарея, коліки. З нестачею якого фер-менту в кишечнику це пов'язано?

A. @Лактаза

B. Мальтаза

C. Глікогенсинтетаза

D. Амілаза

E. Сахараза

373. У хворого після апендектомії припинилося просу-вання химуса. Який вид кишкової непрохідності за механізмом виник у даному випадку?

A. @Паралітична

B. Странгуляційна

C. Обтураційна

D. Низька

E. Висока

374. У хворого з ущемленою паховою грижею поруше-не просування химуса. Який вид кишкової непро-хідності має місце в даному випадку?

A. @Странгуляційна

B. Обтураційна

C. Паралітична

D. Спастична

E. Атонічна

375. Хворий з високою кишковою непрохідністю скар-житься на біль у животі, багаторазову блювоту з жовчю, задишку. Об'єктивно: ЧСС – 110 за 1 хв., АТ – 90/50 мм рт. ст., гематокрит – 0,52, гіпонат-ріємія. Що є провідним механізмом порушень кро-вообігу у хворого?

A. @Дегідратація, гіповолемія

B. Негазовий алкалоз

C. Артеріальна гіпертензія

D. Газовий ацидоз

E. Кишкова автоінтоксикація

376. Хворий з високою кишковою непрохідністю скар-житься на біль у животі, багаторазову блювоту з жовчю, задишку. Об'єктивно: ЧСС – 110 за 1 хв., АТ – 90/50 мм рт. ст., гематокрит – 0,52, гіпонат-ріємія. Яке порушення кислотно–основного гомео-стазу в хворого?

A. @Видільний ацидоз

B. Негазовий алкалоз

C. Газовий ацидоз

D. Газовий алкалоз

E. Екзогенний ацидоз

377. Хворий скаржиться на часті нудоти, які нерідко завершуються блювотою. Порушення якої із фун-кцій шлунка найвірогідніше запідозрити у даного хворого?

A. @Евакуаторної.

B. Єкскреторної

C. Всмоктувальної

D. Інкреторної.

E. Секреторної.

378. У хворого на запалення трійчастого нерва остан-німи роками прогресує пародонтит. Який фактор є головним у розвитку пародонтиту?

A. @Нейродистрофічні порушення в пародонті

B. Зменшення активності лейкоцитарної еластази

C. Послаблення утворення імуноглобулінів

D. Підвищення тонуса вагуса

E. Зниження активності калікреїн–кінінової систе-ми

379. Хворий з Прикарпаття, що страждає на єндемічниї зоб, звернувся до лікаря із скаргами на гноєтечі з ясенних закутків та розхитування зубів. Що в да-ному випадку є основним фактором розвитку па-родонтиту?

A. @Ендокринні порушення

B. Стресорні впливи

C. Гіперсалівація

D. Порушення ковтання

E. Неповноцінне харчування

380. У немовляти внаслідок неправильного годування виникла виражена діарея. Одним з основних нас-лідків діареї є екскреція великої кількості бікар-бонату натрію. Яка форма порушення кислотно–лужного балансу має місце в цьому випадку?

A. @Видільний ацидоз

B. Видільний алкалоз

C. Респіраторний алкалоз

D. Респіраторний ацидоз

E. Метаболічний ацидоз

381. У хворого на ентерит, що супроводжувався знач-ною діареєю, спостерігається зменшення кількості води в позаклітинному просторі, збільшення її всередині клітин та зниження осмолярності крові. Як називають таке порушення водно–електролітного обміну?

A. @Гіпоосмолярна гіпогідратація

B. Гіперосмолярна гіпогідратація

C. Гіпоосмолярна гіпергідратація

D. Осмолярна гіпогідратація

E. Гіперосмолярна гіпергідратація

382. У новонародженої дитини з пілоростенозом часте блювання, що супроводжується апатією, слабкіс-тю, підвищенням тонусу м'язів, інколи судомами. Яка форма порушення кислотно–основного стану розвинулася в хворого?

A. @Негазовий алкалоз

B. Газовий алкалоз

C. Газовий ацидоз

D. Метаболічний ацидоз

E. Видільний ацидоз

383. У чоловіка віком 50 років, який страждає на ви-разкову хворобу шлунку, встановлено посилення секреції та підвищення кислотності шлункового соку. Який механізм може спричинити це явище?

A. @Підвищення активності блукаючого нерва.

B. Підвищення активності симпатичних нервів.

C. Зниження активності блукаючого нерва.

D. Зниження активності симпатичних нервів.

E. Зниження рівня гастрину в крові.

384. У людини під час обстеження, виявлена гіперсек-реція шлункового соку. Лікар рекомендував ви-ключити з дієти насичені бульйони і овочеві від-вари, тому що вони містять речовини, стимулюючі шлункову секрецію:

A. @Екстрактивні речовини і гістамін

B. Соляну кислоту

C. Велику кількість вуглеводів

D. Велику кількість жирів

E. Гастрин

385. Внаслідок тривалої діареї у хворого виникла ізоо-смолярна дегідратація. До якого ускладнення во-на може призвести, якщо її компенсувати введен-ням води?

A. @Набухання клітин

B. Зневоднення клітин

C. Набряк

D. Асцит

E. Артеріальна гіпертензія

386. У чоловіка 40 років після тяжкого емоційного стресу виявлена виразка шлунку. Який механізм є головним в її розвитку?

A. @Переважання факторів „агресії” над факторами „захисту” слизової оболонки

B. Гіперсекреція слизу

C. Інфікуванян слизової оболонки

D. Ахлоргідрія

E. Збільшення продукції простагландинів E1, E2

387. В ургентну лікарню надійшов хворий чоловік 50 років з сильним болем у животі, блювотою. При обстеженні був встановлений діагноз: гострий панкреатит. Вкажіть головну ланку патогенеза у розвитку панкреатиту.

A. @Активація ферментів в протоках підшлункової залози.

B. Розвиток гіповолемії.

C. Білкове подразнення.

D. Метаболічний ацидоз.

E. Набряк підшлункової залози.

388. У хворого після операції резекції шлунка розвину-вся демпінг–синдром, що проявляється раптовою слабістю, серцебиттям, посиленим потовиділен-ням, тремтінням. Таке погіршення самопочуття з’являється безпосередньо після або під час при-йому їжі. Що лежить в основі даної патології?

A. @Швидке наповнення тонкої кишки їжею.

B. Відсутність шлункового соку.

C. Гіпоглікемія.

D. Посилення моторики кишечника.

E. Перерізка гілок блукаючого нерва.

389. У хворого з синдромом подразненої кишки в сли-зовій оболонці порожньої кишки виявляють під-вищений вміст мотилину. Що є причиною розвитку проносу в даному випадку?

A. @Посилення перистальтики кишки

B. Посилення гниття їжі в шлунку

C. Гіперсекреція шлункового соку

D. Гіперсекреція панкреатичного соку

E. Дисбактеріоз

390. У чоловіка віком 30 років після струсу головного мозку виникла рвота. Який патогенетичний варі-ант рвоти спостерігається у цьому випадку?

A. @Центральна, під впливом механічної травми.

B. Центральна, під впливом активації рецепторів ла-біринту.

C. Вісцеральна, під впливом подразнення печінки і нирок.

D. Гематогенно–токсична, під впливом токсинів із крові.

E. Рефлекторна, під впливом підразнення блукаючо-го нерва шлунка.

391. Хворого 50 років доставлено в лікарню з підозрою на непрохідність верхніх відділів кишок. Які зміни в організмі з перелічених є найхарактернішими для цього виду кишкової непрохідності?

A. @Надмірна втрата води.

B. Недостатність травлення.

C. Перитоніт.

D. Кишкова інтоксикація.

E. Біль.

Патофізіологія печінки

392. У хворого з жовтяницею виявлено: збільшення в крові вмісту прямого білірубіну та холестерину, білірубінурія, ахолія, поява в крові жовчних кис-лот. Для якого виду жовтяниці це характерне?

A. @Механічної

B. Печінковоклітинної

C. Гемолітичної

D. Фізіологічної

E. Ядерної

393. У жінки після нападу печінкової коліки з'явилася жовтяниця. УЗД показало закупорку загальної жовчної протоки каменем. Підвищення кількості якого жовчного пігменту в крові свідчить про ме-ханічну жовтяницю?

A. @Прямого білірубіну

B. Непрямого білірубіну

C. Білівердину

D. Вердоглобіну

E. Стеркобіліну

394. Хворий відзначає часті проноси, особливо після вживання жирної їжі, втрату маси тіла. Лаборато-рні дослідження показали наявність стеатореї; кал ахолічний. Що може бути причиною такого стану?

A. @Обтурація жовчних шляхів

B. Дефіцит панкреатичної амілази

C. Запалення слизової оболонки тонкої кишки

D. Дефіцит панкреатичної ліпази

E. Незбалансована дієта

395. У хворого з жовтяницею виявлені гіпербілірубіне-мія за рахунок прямого і непрямого білірубіну, у сечі – прямий білірубін, уробілиноген. Є також гі-похолестеринемія, гіпопротеїнемія, уповільнення згортання крові. Для якого виду жовтяниці харак-терні такі зміни?

A. @Паренхіматозної

B. Механічної

C. Гемолітичної

D. Фізіологічної

E. Ядерної

396. Чоловік скаржиться на гарячку, нудоту, біль у правому підребер'ї. Об'єктивно: жовті шкіра та склери, печінка збільшена, сеча темна, кал зне-барвлений, гіпербілірубінемія (білірубін прямий та непрямий), білірубінурія, уробілінурія, гіпопротеї-немія. Для якого з названих нижче станів харак-терні такі зміни?

A. @Паренхіматозної жовтяниці

B. Підпечінкової жовтяниці

C. Надпечінкової гемолітичної жовтяниці

D. Гострого холециститу

E. Гострого панкреатиту

397. У хворого з жовтяницею діагностовано синдром Криглера – Найара. Яке порушення обміну жовч-них пігментів призвело до жовтяниці у хворого?

A. @Знижена коньюгація білірубіну гепатоцитами

B. Посилений гемоліз

C. Знижене захоплення білірубіну гепатоцитами

D. Знижена екскреція білірубіну гепатоцитами

E. Обтурація жовчного протока

398. У хворого з жовтяницею діагностовано синдром Жильбера. Яке порушення обміну жовчних пігме-нтів призвело до жовтяниці у хворого?

A. @Знижене захоплення білірубіну гепатоцитами

B. Посилений гемоліз

C. Знижена кон’югація білірубіну гепатоцитами

D. Знижена екскреція білірубіну гепатоцитами

E. Обтурація жовчного протока

399. У хворого розвинулась імунна гемолітична анемія. Який показник сироватки крові збільшиться най-більше?

A. @Непрямий білірубін

B. Прямий білірубін

C. Стеркобіліноген

D. Мезобіліноген

E. Протопорфірин

400. До лікаря звернувся пацієнт зі скаргами на пожо-втіння склер і шкіри, потемніння сечі і калу. Ана-ліз крові: еритроцити – 2,5 • 1012/л, Нв – 80 г/л, кольоровий показник – 0,9, ретикулоцитоз. Яка жовтяниця розвинулася у цього пацієнта?

A. @Гемолітична

B. Шунтова

C. Ядерна

D. Паренхіматозна

E. Механічна

401. У новонародженого з гемолітичною хворобою ро-звинулася енцефалопатія. Збільшення якої речо-вини в крові викликало ураження ЦНС?

A. @Білірубіну, не зв'язаного з альбуміном

B. Жовчних кислот

C. Комплексу білірубін–альбумін

D. Вердоглобіну

E. Білірубін–глюкуроніду

402. В експерименті після дії жовчі на серце жаби спо-стерігається брадикардія. Який механізм лежить в основі цього ефекту?

A. @Активація n. vagus жовчними кислотами

B. Блокада адренорецепторів білірубіном

C. Активація n. vagus прямим білірубіном.

D. Кардіодепресорний вплив холестерину

E. Активація холінестерази під дією жовчі

403. У хворого з жовтяницею спостерігається артеріа-льна гіпотензія, брадикардія, свербіння шкіри, дратівливість, сонливість вдень і безсоння вночі. Як називається такий синдром?

A. @Холемічний

B. Ахолічний

C. Холестатичний

D. Гіпохолічний

E. Дисхолічний.

404. Жінка, 55 років, скаржиться на швидку стомлю-ваність, дратівливість, безсоння вночі і сонливий стан вдень, свербіння шкіри. Попередній діагноз: жовчнокам'яна хвороба із закупоркою каменем загальної жовчної протоки. Чим зумовлені зміни з боку нервової системи у хворої?

A. @Холалемією

B. Гипербілірубінемією

C. Гіперхолестеринемією

D. Порушенням всмоктування жирів

E. Порушенням всмоктування жиророзчинних ві-тамінів

405. У хворого з жовтяницею з’явилися порушення травлення, стеаторея, знебарвлення калу, пору-шення згортання крові, ознаки авітамінозу. Про розвиток якого синдрому свідчать дані зміни?

A. @Ахолічного

B. Холемічного

C. Астенічного

D. Гіперхолічного

E. Холестатичного

406. У хворого, який страждає на жовчнокам’яну хво-робу, внаслідок обтурації жовчовивідних шляхів спостерігається знебарвлений жирний кал. Відсу-тність якого компонента жовчі зумовлює стеато-рею?

A. @Жовчних кислот

B. Жирних кислот

C. Холестерину

D. Жовчних пігментів

E. Лужної фосфатази

407. У хворого через наявність каменю у загальній жовчній протоці припинилося надходження жовчі в кишечник. Порушення якого з процесів спосте-рігається при цьому?

A. @Перетравлення жирів

B. Всмоктування білків

C. Перетравлення білків

D. Перетравлення вуглеводів

E. Всмоктування вуглеводів

408. У хворого з патологією печінки є ознаки геморагі-чного синдрому. Який можливий механізм його виникнення?

A. @Порушення утворення протромбіну

B. Підвищення судинної проникності під дією жов-чних пігментів

C. Гемоліз еритроцитів під дією жовчних кислот

D. Зниження утворення фактора Віллебранда

E. Зниження здатності тромбоцитів до адгезії і аг-регації

409. У хворого з алкогольним цирозом печінки скарги на загальну слабкість, задишку. Встановлено зниження артеріального тиску, асцит, розширення поверхневих вен передньої стінки живота, спле-номегалію. Яке порушення гемодинаміки спостері-гається у хворого?

A. @Синдром портальної гіпертензії

B. Недостатність лівого шлуночку серця

C. Недостатність правого шлуночку серця

D. Колапс

E. Тотальна серцева недостатність

410. У хворого, що страждає на цироз печінки, спосте-рігається зниження артеріального тиску, розши-рення поверхневих вен передньої стінки живота, спленомегалія, асцит. Який механізм є провідним у виникненні асциту?

A. @Застійний

B. Мембраногенний

C. Лімфогенний

D. Гіперпротеїнемічний

E. Гіперволемічний

411. У хворого 35 років, що переніс гепатит і продов-жував приймати алкоголь, розвинулися ознаки цирозу печінки з асцитом і набряками на нижніх кінцівках. Які зміни складу крові стали вирішаль-ними в розвитку набряку?

A. @Гіпоальбумінемія

B. Гіпокаліємія

C. Гіпоглобулінемія

D. Гіпохолестеринемія

E. Гіпоглікемія

412. Хвора на хронічний гепатит скаржиться на підви-щену чутливість до барбітуратів, які раніше вона переносила без симптомів інтоксикації. З пору-шенням якої функції печінки це пов’язане найбі-льше?

A. @Метаболічної

B. Фагоцитарної

C. Екскреторної

D. Гемодинамічної

E. Гемопоетичної

413. Скарги і об'єктивні дані дозволяють припустити наявність у хворого запального процесу в жовч-ному міхурі, порушення колоїдних властивостей жовчі, вірогідність утворення жовчних каменів. Яка речовина, головним чином, може зумовити утворення каменів?

A. @Холестерин

B. Хлориди

C. Фосфати

D. Урати

E. Оксалати

414. У хворого на хронічний гепатит виявлено дрібні крововиливи на шкірі, кровоточивість ясен. По-рушенням якої функції печінки зумовлений гемо-рагічний синдром?

A. @Білковосинтетичної

B. Пігментоутворюючої

C. Глікогенсинтезуючої

D. Детоксикуючої

E. Депонуючої

415. У людини унаслідок хронічного захворювання пе-чінки істотно порушена її білковосинтезуюча фун-кція. Який параметр гомеостазу зменшиться най-більш вірогідно?

A. @Онкотичний тиск плазми крові

B. рН

C. Осмотичний тиск плазми крові

D. Гематокрит

E. Об’єм крові

416. У хворого на цироз печінки низький артеріальний тиск. Який механізм порушення гемодинаміки при цьому?

A. @Депонування крові в портальному руслі

B. Збільшення опору судин

C. Збільшення ударного об'єму серця

D. Збільшення діастолічного об'єму серця

E. Розширення вен передньої стінки живота

417. Тварині після попередньої підготовки (фістула Екка – Павлова) видалили печінку. Що стане при-чиною загибелі тварини?

A. @Гіпоглікемічна кома

B. Гіперглікемічна кома

C. Портальна гіпертензія

D. Ахолія

E. Холемія

418. Після вживання жирної їжі у хворого з'являються нудота та печія, має місце стеаторея. Причиною такого стану може бути:

A. @Нестача жовчних кислот

B. Підвищене виділення ліпази

C. Порушення синтезу фосфоліпази

D. Порушення синтезу трипсину

E. Нестача амілази

419. Хворий відзначає часті проноси, особливо після вживання жирної їжі, схуднення. Лабораторні до-слідження показали наявність стеатореї; кал гіпо-холічний. Що може бути причиною такого стану?

A. Недостатність панкреатичної ліпази

B. Запалення слизової оболонки тонкої кишки

C. @Обтурація жовчних шляхів

D. Недостатність панкреатичної фосфоліпази

E. Незбалансована дієта

420. У юнака 16–ти років після перенесеного захворю-вання знижена функція синтезу білків у печінці внаслідок нестачі вітаміну K. Це може призвести до порушення:

A. @Зсідання крові

B. Утворення еритропоетинів

C. Утворення антикоагулянтів

D. Осмотичного тиску крові

E. Швидкості зсідання еритроцитів

421. У непритомного хворого рефлекси відсутні, періо-дично з’являються судоми, дихання нерівномірне. Після лабораторного обстеження було діагносто-вано печінкову кому. Нагромадження якого мета-боліту є суттєвим для появи розладів центральної нервової системи?

A. @Аміаку

B. Сечовини

C. Глутаміну

D. Білірубіну

E. Гістаміну

422. У пацієнта, що звернувся до лікаря, спостеріга-ється жовте забарвлення шкіри, сеча темна, кал темного кольору. Підвищення концентрації якої речовини буде спостерігатися в сироватці крові?

A. @Вільний білірубін

B. Мезобілірубін

C. Кон'югований білірубін

D. Вердоглобін

E. Білівердин

423. Хвора 48–ми років надійшла до клініки із скаргами на слабкість, дратівливість, порушення сну. Об'є-ктивно: шкіра та склери жовтого кольору. У кро-ві: підвищення рівня загального білірубіну з пере-важанням прямого. Кал – ахолічний. Сеча – тем-ного кольору (жовчні пігменти). Яка жовтяниця має місце в хворої?

A. @Механічна

B. Гемолітична

C. Синдром Кріглера–Найяра

D. Паренхіматозна

E. Синдром Жільбера

424. У хлопчика 4–х років після перенесеного важкого вірусного гепатиту мають місце блювання, втрата свідомості, судоми. У крові – гіперамоніємія. По-рушення якого біохімічного процесу викликало патологічний стан хворого?

A. @Порушення знешкодження аміаку в печінці

B. Посилення гниття білків у кишечнику

C. Пригнічення ферментів трансамінування

D. Порушення знешкодження біогенних амінів

E. Активація декарбоксилування амінокислот

425. У людини порушено всмоктування продуктів гід-ролізу жирів. Причиною цього може бути дефіцит у порожнині тонкої кишки наступних компонентів:

A. @Жовчні кислоти

B. Ліполітичні ферменти

C. Жовчні пігменти

D. Жиророзчинні вітаміни

E. Іони натрію

426. Хворий після вживання жирної їжі відчуває нудо-ту, млявість; з часом з'явилися ознаки стеатореї. У крові холестерин – 9,2 ммоль/л. Причиною та-кого стану є нестача у кишечнику:

A. @Жовчних кислот

B. Фосфоліпідів

C. Хіломікронів

D. Тригліцеридів

E. Жирних кислот

427. У чоловіка, віком 50 років, у якого спостерігалась гемолітична жовтяниця, було виявлено високий рівень непрямого білірубіну в крові. В той же час в сечі цього хворого непрямий білірубін був відсу-тній. Яка причина зумовила виникнення цього явища?

A. @Зв’язок білірубіну з альбуміном плазми.

B. Посилення екскреції білірубіну у дванадцятипа-лу кишку.

C. Дія білірубіну на нейрони головного мозку.

D. Порушення холестеринового індексу жовчі.

E. Підвищення здатності жовчі до утворення ка-менів.

428. Хворий 56 років скаржиться на слабкість, мля-вість, порушення нічного сну, іктеричність слизо-вих оболонок та шкіри. При дослідженні виявлено збільшену печінку та селезінку. Яке порушення виявили у хворого?

A. @Гепатолієнальний синдром.

B. Гепаторенальний синдром.

C. Регенераторна гіпертрофія печінки.

D. Синдром мальабсорбції.

E. Синдром портальної гіпертензії.

429. У хворого на цироз печінки встановлено значне зниження в крові вмісту альбумінів і глобулінів. Яке порушення в організмі найбільш вірогідно може бути наслідком такої гіпопротеїнемії?

A. @Печінковий набряк

B. Геморагічний синдром

C. Анемія

D. Енцефалопатія

E. Аміноацидурія

430. Хворий у непритомному стані доставлений швид-кою допомогою у лікарню. Об’єктивно: рефлекси відсутні, періодично з’являються судоми, дихан-нянерівномірне. Після лабораторного обстеження було діагностовано печінковукому. Нагромаджен-ня якого метаболіту є суттєвим для появи розла-дів центральної нервової системи?

A. @Аміаку

B. Сечовини

C. Глутаміну

D. Білірубіну

E. Гістаміну

431. Дитина 10–ти років потрапила до лікарні з озна-ками ураження печінки і нервової системи. При проведенні лабораторних досліджень виявлено підвищену екскрецію міді з сечею і дефіцит це-рулоплазміну в крові. Яке захворювання, найімо-вірніше, може бути у дитини?

A. @Хвороба Вільсона – Коновалова ;

B. Синдром Фанконі;

C. Цистінурія;

D. Ліпідоз;

E. Фенілкетонурія.

Патофізіологія нирок

432. Хворий після автомобільної катастрофи неприто-мний, АТ 70/40 мм рт. ст. За добу виділяє біля 300 мл сечі. Який механізм порушення сечоутворення у даному випадку?

A. @Зменшення клубочкової фільтрації

B. Підсилення клубочкової фільтрації

C. Послаблення канальцевої реабсорбції

D. Підсилення канальцевої реабсорбції

E. Зменшення канальцевої секреції

433. У хворого з гострою нирковою недостатністю в стадії поліурії азотемія не тільки не зменшилась, але й продовжувала підвищуватись. Що в даному випадку викликало поліурію?

A. @Зменшення реабсорбції

B. Збільшення фільтрації

C. Збільшення секреції

D. Збільшення реабсорбції

E. Зменшення фільтрації

434. У хворого А., 38 років, на 3–му році захворювання на системний червоний вовчак виявилось дифуз-не ураження нирок, що супроводжувалося масив-ними набряками, вираженою протеїнурією, гіпер-ліпідемією, диспротеїнемією. Який найбільш імові-рний механізм розвитку протеїнурії в даній клініч-ній ситуації?

A. @Автоімунне ушкодження клубочків нефронів

B. Запальне пошкодження канальців нефронів

C. Ішемічне ушкодження канальців

D. Збільшення рівня протеїнів у крові

E. Ураження сечовидільних шляхів

435. Хворий доставлений у лікарню із шлунковою кро-вотечею в тяжкому стані. АТ – 80/60 мм рт. ст. Діурез 60–80 мл сечі за добу з відносною щільніс-тю 1,028–1,036. Який патогенетичний механізм найімовірніше зумовив зменшення добового діу-резу?

A. @Зниження гідростатичного тиску в капілярах клубочків

B. Підвищення онкотичного тиску крові

C. Підвищений рівень залишкового азоту в крові

D. Підвищення колоїдно–осмотичного тиску в крові

E. Підвищення гідростатичного тиску в капсулі Шум-лянського – Боумена

436. 43–річний хворий скаржиться на масивні набряки та артеріальну гіпертензію. У сечі: відносна щіль-ність 1,017, білок 4,0 г/л, Ер – 15–18 в полі зору (вилужені). Який нирковий процес переважно по-рушений у хворого?

A. @Фільтрація

B. Реабсорбція

C. Секреція

D. Інкреція

E. Екскреція

437. У 20–річного хлопця через 2 тижня після перене-сеної лакунарної ангіни зявились скарги на зага-льну слабкість, набряки під очима. Після обсте-ження хворому поставлений діагноз: гострий гло-мерулонефрит. Які патологічні зміни в складі сечі найбільш імовірні?

A. @Протеїнурія

B. Наявність свіжих еритроцитів

C. Циліндрурія

D. Лейкоцитурія

E. Натрійурія

438. У хворого на аденому простати знизилась швид-кість клубочкової фільтрації. Який механізм цього ускладнення?

A. @Підвищення гідростатичного тиску в капсулі клубочка

B. Зниження гідростатичного тиску крові

C. Підвищення онкотичного тиску крові

D. Зменшення кількості функціонуючих нефронів

E. Зменшення площі фільтруючої поверхні

439. У хворого з хронічною патологією нирок після проведення проби Зимницького виявлено гіпосте-нурію. Які зміни сечовиділення будуть спостеріга-тися при цьому?

A. @Поліурія

B. Анурія

C. Олігурія

D. Полакіурія

E. Ніктурія

440. У хворого діагностовано хронічний гломерулонеф-рит. Внаслідок значних склеротичних змін маса функціонуючих нефронів зменшилася до 10%. Яке з перерахованих нижче порушень зумовлює уремічний синдром?

A. @Азотемія

B. Артеріальна гіпертензія

C. Порушення водного гемостазу

D. Порушення осмотичного гемостазу

E. Ниркова остеодистрофія

441. При обстеженні хворого встановлено, що кліренс ендогенного креатиніну після 24–х годинного збо-ру сечі становить 50 мл/хв (при нормі – 110–150 мл/хв.). Про порушення якого ниркового процесу свідчить наявність такої ознаки?

A. @Клубочкової фільтрації

B. Канальцевої реабсорбції

C. Ниркової інкреції

D. Канальцевої секреції

E. Екскреції сечі

442. У хворого, який був госпіталізований після того, як перебував під завалом на протязі 4 годин, спо-стерігається різке зниження діурезу, гіперазоте-мія, ознаки набряку головного мозку. В якій стадії гострої ниркової недостатності перебуває хворий?

A. @Олігоануричній

B. Початковій

C. Поліуричній

D. Одужання

E. Термінальній

443. В експерименті кролю ввели нефроцитотоксичну сироватку морської свинки. Яке захворювання нирок моделювалося в цьому досліді?

A. @Гострий дифузний гломерулонефрит

B. Нефротичний синдром

C. Гострий пієлонефрит

D. Хронічна ниркова недостатність

E. Хронічний пієлонефрит

444. У хворого після тяжкої травми грудної клітки роз-винувся шок та з’явилися ознаки гострої ниркової недостатності (ГНН). Що є провідним механізмом розвитку ГНН в даному випадку?

A. @Зменшення артеріального тиску

B. Порушення відтоку сечі

C. Підвищення тиску в капсулі клубочка

D. Підвищення тиску в ниркових артеріях

E. Зменшення онкотичного тиску крові

445. Білому щуру ввели під шкіру сулему в дозі 5 мг/кг маси тіла. Через 24 години в плазмі крові концен-трація креатиніну збільшилася в декілька разів. Який механізм ретенційної азотемії в даному ви-падку?

A. @Зниження клубочкової фільтрації

B. Зростання клубочкової фільтрації

C. Збільшення утворення креатиніну в м`язах

D. Зростання реабсорбції креатиніну

E. Збільшення секреції креатиніну в канальцях

446. У хворого, що тривалий час страждає на цукровий діабет І типу, виникли азотемія, олігурія, гіпо–, ізостенурія, протеїнурія. Що є головною ланкою в патогенезі цих симптомів при хронічній нирковій недостатності?

A. @Зменшення маси діючих нефронів

B. Порушення проникності клубочкових мембран

C. Підвищення клубочкової фільтрації

D. Зниження канальцевої секреції

E. Посилення реабсорбції натрію

447. У хворого з хронічною нирковою недостатністю встановлено зменшення кліренсу за інуліном до 60 мл/хв. З порушенням якої функції нирок це пов’язано?

A. @Клубочкової фільтрації

B. Канальцевої секреції

C. Реабсорбції в проксимальному відділі нефрону

D. Реабсорбції в дистальному відділі нефрону

E. Реабсорбції в збиральних ниркових трубочках

448. У хворого на хронічну ниркову недостатність з’явилися анорексія, диспепсія, порушення ритму серця, свербіння шкіри. Який механізм розвитку цих порушень є головним?

A. @Гіперазотемія

B. Розвиток ниркового ацидозу

C. Порушення ліпідного обміну

D. Зміни вуглеводного обміну

E. Порушення водно–електролітного обміну

449. У хворого з гломерулонефритом виявлено: АТ – 185/105 мм рт.ст., анемія, лейкоцитоз, гіперазо-темія, гіпопротеїнемія. Який показник свідчить про ускладнення гломерулонефриту нефротичним синдромом?

A. @Гіпопротеїнемія

B. Анемія

C. Лейкоцитоз

D. Гіперазотемія

E. Артеріальна гіпертензія

450. В результаті порушення техніки безпеки відбуло-ся отруєння сулемою (хлорид ртуті). Через 2 дні добовий діурез становив 200 мл. У хворого зявилися головний біль, блювання, судоми, за-дишка, у легенях – вологі хрипи. Як називається така патологія?

A. @Гостра ниркова недостатність

B. Хронічна ниркова недостатність

C. Уремічна кома

D. Гломерулонефрит

E. Пієлонефрит

451. У результаті гострої ниркової недостатності у хво-рого виникла олігурія. Яка добова кількість сечі відповідає даному симптому?

A. @100–500 мл

B. 1500–2000 мл

C. 1000–1500 мл

D. 500–1000 мл

E. 50–100 мл

452. У хворого, віком 58 років, з гострою серцевою недостатністю, спостерігалось зменшення добової кількості сечі – олігурія. Який механізм цього явища?

A. @Зниження гідростатичного тиску крові в клубоч-кових капілярах

B. Підвищення гідростатичного тиску крові в клубоч-кових капілярах

C. Зниження кількості функціонуючих нефронів

D. Збільшення онкотичного тиску крові

E. Зниження проникності клубочкової мембрани

453. У хворого на гострий гломерулонефрит спостері-гається протеїнурія з молекулярною масою білка понад 70000 D. Яке порушення спричинило це явище?

A. @Ушкодження клубочкової мембрани

B. Порушення реабсорбції білка

C. Порушення реабсорбції білка

D. Зміна заряду поліаніонів клубочкової мембрани

E. Ушкодження канальців

454. У хворого, що тривалий час страждає на хроніч-ний гломерулонефрит, виникла азотемія при помі-рній поліурії. Що є головною ланкою в патогенезі цих симптомів при хронічній нирковій недостатно-сті?

A. @Зменшення маси діючих нефронів

B. Підвищення клубочкової фільтрації

C. Зниження канальцевої секреції

D. Порушення проникності клубочкових мембран

E. Недостатність вазопресину

455. У хворого на первинний нефротичний синдром встановлений вміст загального білку крові 40 г/л. Яка причина зумовила гіпопротеїнемію?

A. @Протеїнурія

B. Порушення всмоктування білка у кишечнику

C. Вихід білка з судин у тканини

D. Зниження синтезу білка у печінці

E. Підвищений протеоліз

456. У дитини 5 років через 2 тижні після перенесеної ангіни виник гострий дифузний гломерулонефрит, що характеризувався олігурією, протеїнурією, ге-матурією, збільшенням ОЦК. Порушення якого з ниркових процесів найбільш суттєве для виник-нення цих симптомів?

A. @Клубочкової фільтрації

B. Ниркової інкреції

C. Канальцевої реабсорбції

D. Канальцевої секреції

E. Концентрування сечі

457. В результаті гострої ниркової недостатності у хво-рого виникла анурія. Яка добова кількість сечі ві-дповідає даному симптому?

A. @50–100 мл

B. 100–500 мл

C. 1500–2000 мл

D. 1000–1500 мл

E. 500–1000 мл

458. У хворого на мієломну хворобу знизилась швид-кість клубочкової фільтрації (ШКФ). Який фактор обумовив зменшення ШКФ в даному випадку?

A. @Підвищення онкотичного тиску крові

B. Зниження гідростатичного тиску крові

C. Підвищення гідростатичного тиску в капсулі клу-бочка

D. Зменшення маси діючих нефронів

E. Зменшення площі фільтруючої поверхні

459. Хворій Н. 3 роки тому був поставлений діагноз хронічний гломерулонефрит. Останні 6 місяців зявились набряки. Що головним чином спричинює їх розвиток?

A. @Протеїнурія

B. Гіперальдостеронізм

C. Гіперпродукція вазопресину

D. Порушення білковоутворюючої функції печінки

E. Гіперосмолярність плазми

460. У чоловіка 48 років, що давно хворіє на хронічний гломерулонефрит, обстеження виявило наявність хронічної недостатності нирок. Чим пояснюється гіперазотемія при хронічній нирковій недостатнос-ті?

A. @Зниженням клубочкової фільтрації

B. Зниженням канальцевої реабсорбції

C. Зниженням канальцевої екскреції

D. Порушенням білкового обміну

E. Порушенням водно–електролітного балансу

461. У хворого на хронічний пієлонефрит розвинулась ізостенурія. Яка питома вага сечі має бути у цього хворого?

A. @1,010

B. 1,000

C. 1,020

D. 1,030

E. 0,900

462. У жінки 30–ти років виникли набряки обличчя. При обстеженні виявлені протеїнурія (5,87 г/л), гіпопротеїнемія, диспротеїнемія, гіперліпідемія. Для якого стану характерно таке поєднання симп-томів?

A. @Нефротичний синдром

B. Нефритичний синдром

C. Хронічна ниркова недостатність

D. Хронічний пієлонефрит

E. Гостра ниркова недостатність

463. У хворого з нефротичним синдромом спостеріга-ються масивні набряки обличчя та кінцівок. Який патогенетичний механізм є провідним в розвитку набряків?

A. @Зниження онкотичного тиску крові

B. Підвищення гідродинамічного тиску крові

C. Підвищення судинної проникності

D. Підвищення лімфовідтоку

E. Лімфостаз

464. У водія, який потрапив у ДТП, отримав травму та знаходиться у стані шоку, спостерігається змен-шення добової кількості сечі до 300 мл. Який ос-новний патогенетичний фактор цієї зміни діурезу?

A. @Падіння артеріального тиску

B. Зниження онкотичного тиску крові

C. Підвищення проникності судин

D. Зменшення кількості функціонуючих клубочків

E. Вторинний гіперальдостеронізм

465. У чоловіка 43–х років з видаленою ниркою були виявлені симптоми анемії. Що зумовило появу цих симптомів?

A. @Зниження синтезу еритропоетинів

B. Нестача вітаміну B12

C. Нестача фолієвої кислоти

D. Нестача заліза

E. Підвищене руйнування еритроцитів

466. У хворого з гострою нирковою недостатністю на 6–й день проведення терапевтичних заходів ви-никла поліурія. Чим зумовлене зростання діурезу на початку поліуричної стадії гострої ниркової не-достатності?

A. @Відновленням фільтрації в нефронах

B. Збільшенням натрійуретичного фактора

C. Зменшенням альдостерону в плазмі

D. Зменшенням вазопресину в плазмі

E. Збільшенням об'єму циркулюючої крові

467. У результаті порушення техніки безпеки відбуло-ся отруєння сулемою (хлористою ртуттю). Через 2 дні добовий діурез склав 620 мл. У хворого з'яви-лися головний біль, блювання, судоми, задишка, у легенях – вологі хрипи. Яка патологія має місце?

A. @Гостра ниркова недостатність

B. Гломерулонефрит

C. Хронічна ниркова недостатність

D. Уремічна кома

E. Пієлонефрит

468. У хворого виявлено порушення реабсорбції іонів натрію, глюкози, амінокислот, гідрокарбонатів, фосфатів, бета–2–мікроглобуліну. Для ушко-дження якого відділу нефрону це характерно?

A. @Проксимального

B. Дистального

C. Петлі нефрону

D. Збірних канальців

E. Сполучних сегментів

469. У пацієнта 20 років при лабораторному обстежен-ні виявлений підвищений вміст глюкози у сечі при нормальному її вмісту у крові. Найбільш вірогід-ною причиною цього явища є порушення процесу:

A. @Канальцевої реабсорбції

B. Секреції інсуліну

C. Клуб очкової фільтрації

D. Секреції глюкокортикоїдів

E. Канальцевої секреції

470. У хворого з хронічною нирковою недостатністю виникли диспептичні явища, нестерпне свербіння шкіри, різка загальна слабкість, запах аміаку та сіро–землистий відтінок шкіри. Ці симптоми є нас-лідком

A. @Зниження екскреції продуктів азотистого обміну

B. Порушення водно–електролітного обміну

C. Порушення кислотно–основної рівноваги

D. Порушення вуглеводного обміну

E. Порушення ліпідного обміну

471. Хворий тривалий час страждає на гіпертонічну хворобу. Який аналіз крові потрібно час від часу робити, щоб вчасно виявити початок розвитку ниркового ускладнення?

A. @Визначення вмісту креатиніну.

B. Визначення вмісту білка.

C. Визначення вмісту натрію.

D. Визначення вмісту калію.

E. Визначення вмісту холестерину.

Патофізіологія ендокринної системи

472. У хворого Н., 25 років, після перенесеної інфекції розвинувся нецукровий діабет. Дефіцит якого го-рмону призвів до розвитку даної патології?

A. @Вазопресину

B. Альдостерону

C. Кортизолу

D. Реніну

E. Інсуліну.

473. Хворий скаржиться на постійну спрагу, знижений апетит. Випиває за добу 9 л. Поліурія, сеча зне-барвлена, відносна щільність – 1,005. Найбільш імовірною причиною розвитку даної патології у хворого є пошкодження:

A. @Гіпоталамічних ядер

B. Наднирників

C. Аденогіпофізу

D. Епіфізу

E. Базальної мембрани капілярів клубочків

474. У дитини 2,5 років спостерігається затримка у фізичному розвитку, поганий сон та апетит, спра-га, поліурія. Цукор у сечі не виявляється. Яка ен-докринна патологія є причиною порушень водно–мінерального обміну?

A. @Гіпосекреція антидіуретичного гормону

B. Гіперсекреція антидіуретичного гормону

C. Вторинний гіперальдостеронізм

D. Гіпосекреція соматотропного гормону

E. Гіпосекреція адренокортикотропного гормону

475. До лікаря звернувся чоловік 27 років. При огляді було виявлено збільшення кистей рук, ступнів та нижньої щелепи. Крім того є імпотенція, атрофія яєчок. Функції якої залози порушені?

A. @Передньої частини гіпофізу

B. Задньої частини гіпофізу

C. Надниркових залоз

D. Щитоподібної залози

E. Прищитоподібних залоз

476. При руйнуванні 95 % маси залози у жінки розви-нулися гіпоглікемія, аменорея, остеопороз, атро-фія м’язів, випадіння зубів і волосся, кахексія. Аб-солютна недостатність якої залози спричинює дані зрушення?

A. @Аденогіпофізу

B. Нейрогіпофізу

C. Острівців підшлункової залози

D. Тимусу

E. Мозкової речовини наднирників

477. Батьки хлопчика 15 років звернулися до лікаря зі скаргами на значне відставання сина в рості. Об’єктивно: статура правильна, зріст 98 см, інте-лект не порушений. При нестачі якого гормону з'являються такі зміни?

A. @Соматотропного

B. Тиреотропного

C. Адренокортикотропного

D. Гонадотропного

E. Пролактину

478. У хворого В., 14 років, виявлено пропорційне збі-льшення усіх частин тіла. Зріст 198 см. Лаборато-рно – гіперглікемія, глюкозурія, порушення тесту толерантності до глюкози. Причиною цукрового діабету є:

A. @Гіперсекреція соматотропного гормону

B. Гіпосекреція інсуліну

C. Гіперсекреція кортикотропіну

D. Гіпосекреція вазопресину

E. Гіперсекреція глюкокортикоїдів

479. У хворої, що страждає на гіпертиреоз, підвищені основний обмін, температура тіла, стійкість до гі-потермії. Що можна зазначити як головну ланку в патогенезі цих проявів?

A. @Збільшення споживання О2 мітохондріями

B. Зменшення окисного фосфорування

C. Активація глікогенезу

D. Активація гліколізу

E. Інактивація ферментів дихального ланцюга

480. Пацієнт скаржиться на напади серцебиття, пітли-вість і дратівливість. Обстеження показало наяв-ність у нього дифузного токсичного зобу. Імунні реакції якого типу (за Кумбсом і Джеллом) є голо-вною ланкою в патогенезі Базедової хвороби?

A. @V типу (стимулюючі)

B. I типу (анафілактичні)

C. II типу (гуморальні цитотоксичні )

D. III типу (імунокомплексні)

E. IV типу (клітинні цитотоксичні)

481. При хворобі Базедова тироцити продукують над-мірну кількість гормонів Т3 і Т4. Що спричинює дисфункцію тироцитів при цьому автоімунному захворюванні?

A. @IgG

B. IgA

C. IgE

D. Імунні комплекси

E. Т–лімфоцити

482. У ліквідатора аварії на ЧАЕС через деякий час з¢явились скарги на підвищену збудливість, нер-возність, серцебиття, зниження маси тіла, постій-ну слабкість, тремтіння тіла, підвищення темпера-тури. Гіперфункція якої залози може бути причи-ною вказаних змін:

A. @Щитоподібної залози

B. Аденогіпофізу

C. Кіркової речовини наднирників

D. Мозкової речовини наднирників

E. Паращитоподібної залози

483. У хворого з базедовою хворобою виявлено тахі-кардію та ізольовану систолічну артеріальну гіпе-ртензію. Збільшення чутливості до якого гормону є головною ланкою в патогенезі цих проявів при гіпертиреозі?

A. @Адреналіну

B. Соматотропіну

C. Соматостатину

D. Альдостерону

E. ТТГ

484. Жінка 36 років звернулася до лікаря з приводу схуднення. При загальному дослідженні звертає на себе увагу потовщення шиї, екзофтальм. Пульс 120 уд/хв. Які додаткові дослідження доці-льно було б провести в цієї жінки ?

A. @Дослідження рівнів Т3 і Т4

B. Ультразвукове дослідження яєчників

C. Дослідження рівня катехоламінів крові

D. Томографічне дослідження зони турецької сід-ловини

E. Тест навантаження глюкозою

485. У пацієнта 20 років після хірургічного видалення щитоподібної залози спостерігається гіпоплазія емалі, порушення утворення дентину. Недостат-ність якого гормону стала причиною вказаної па-тології?

A. @Тиреокальцитоніну

B. Паратгормону

C. Тироксину

D. Тиреотропіну

E. Трийодтироніну

486. Хворій Д., 50 років, був поставлений діагноз мік-седема. Порушення утворення яких гормонів при-зводить до розвитку даної патології?

A. @Тироксину і трийодтироніну

B. Кортизолу і альдостерону

C. АКТГ і СТГ

D. Окситоцину і вазопресину

E. Інсуліну і глюкагону

487. У жительки Закарпаття з’явились слабкість, сон-ливість, апатія, зниження пам’яті, набряки. Після обстеження встановлений діагноз “ендемічний зоб”. Брак якої речовини у воді та їжі може приз-вести до цієї хвороби?

A. @Йоду

B. Цинку

C. Фтору

D. Заліза

E. Магнію

488. У жительки Закарпаття діагностовано гіпотиреої-дний зоб. Вміст якого гормону в крові збільшуєть-ся при ендемічному зобі?

A. @ТТГ

B. АКТГ

C. СТГ

D. МСГ

E. АДГ

489. Дитина 3–х років млява, малорухлива, різко відс-тає від однолітків у фізичному і психічному розви-тку, понижені основний обмін і температура тіла. Яке захворювання у дитини?

A. @Кретинізм

B. Базедова хвороба

C. Тиреотоксикоз

D. Ендемічний зоб

E. Спорадичний зоб

490. Дитина, хвора на кретинізм, млява, малорухлива, різко відстає від однолітків у фізичному і психіч-ному розвитку, понижені основний обмін і темпе-ратура тіла. Що лежить в основі цього захворю-вання?

A. @Вроджений гіпотиреоз

B. Аутоімунний гіпертиреоз

C. Вторинний гіпертиреоз

D. Дефіцит йоду

E. Надлишок йоду

491. Жінка 53 років скаржиться на зниження пам’яті та розумової працездатності. Зріст 163 см, вага ті-ла 92 кг, рівномірне відкладання жиру, лице оду-тле, малорухома, апатична. Зниженням функції якої залози зумовлена хвороба?

A. @Щитовидної.

B. Гіпофізу.

C. Надниркових.

D. Статевих.

E. Прищитовидних.

492. У хворого виявлено гіперкаліємію та гіпонатріє-мію. Знижена секреція якого гормону може спри-чинити такі зміни?

A. @Альдостерону

B. Кортизолу

C. Вазопресину

D. Паратгормону

E. Натрійуретичного

493. У хворого на правошлуночкову серцеву недостат-ність є асцит та набряки. Яке ендокринне пору-шення має значення в патогенезі серцевих набря-ків?

A. @Вторинний гіперальдостеронізм

B. Первинний гіперальдостеронізм (синдром Кон-на)

C. Вторинний гіперкортицизм (хвороба Іценка – Кушинга)

D. Первинний гіперкортицизм (синдром Іценка – Кушинга)

E. Первинний гіпокортицизм (хвороба Адісона)

494. У хворого виявлена аденома клубочкової зони кори наднирників. Внаслідок цього розвинувся первинний гіперальдостеронізм або хвороба Кон-на. На обмін якого іону впливає ця пухлина?

A. @Натрію

B. Хлору

C. Магнію

D. Кальцію

E. Заліза

495. У хворого на аденому клубочкової зони кори над-нирників (хворобу Конна) спостерігаються м’язова слабкість, атонія кишок, аритмії серця, парестезії. Дефіцит якого катіону є головною ланкою в пато-генезі цих розладів?

A. @Калію

B. Натрію

C. Кальцію

D. Магнію

E. Заліза

496. У хворого на аденому клубочкової зони кори над-нирників (хворобу Конна) спостерігаються артері-альна гіпертензія, м’язова слабкість, аритмії сер-ця, судоми. Яке порушення кислотно–основного гомеостазу в хворого?

A. @Видільний алкалоз

B. Видільний ацидоз

C. Газовий ацидоз

D. Газовий алкалоз

E. Екзогенний ацидоз

497. У хворого з печінковою комою при вірусному ге-патиті в крові підвищилась концентрація Na+ та знизилась концентрація К+, тобто почались про-яви гіперальдостеронізму. Чим обумовлено підви-щення дії альдостерону у хворого?

A. @Зменьшенням його метаболізму.

B. Підвищенням його сінтезу альдостерону.

C. Збільшенням кіоткості рецепторів до нього.

D. Порушенням його транспорту.

E. Зменьшенням його виведення.

498. Після адреналектомії у тварини виникла зупин-ка серця в діастолі і вона загинула. Яке порушення викликало зупинку серця?

A. @Гіперкаліемія

B. Гіпернатріемія

C. Гіпермагніемія

D. Гіперкальціемія

E. Гіперфосфатемія

499. Після двостороннього видалення наднирників у собаки з¢явилась м’язова слабкість, адинамія, зниження температури тіла, гіпоглікемія. Якийі ще може бути відмічений прояв наднирникової недо-статності?

A. @Артеріальна гіпотонія

B. Лімфопенія

C. Гіпопротеїнемія

D. Гіпернатріємія

E. Ожиріння

500. Хвора, 28 р., скаржиться на в’ялість, швидку ро-зумову та фізичну втомлюваність, диспептичні порушення. При обстеженні виявлено: позитивні туберкулінові проби, гіпоглікемія, АТ – 90/60 мм.рт.ст.,гіпонатріємію, пігментацію шкіри. При якій патології наднирників спостерігаються подібні явища?

A. @Хвороба Аддісона.

B. Синдром Іценка–Кушинга.

C. Гостра недостатність кори наднирників.

D. Гіпофункція мозкового шару наднирників.

E. Синдром Конна.

501. У хворого хронічна недостатність кори наднирко-вих залоз (Аддісонова або бронзова хвороба). Не-достатність якого гормону має місце при цьому патологічному процесі?

A. @Кортизолу

B. Інсуліну

C. Адреналіну

D. Тироксину

E. Вазопресину

502. У хворого Аддісонова або бронзова хвороба. Не-достатність якого гормону має місце при цьому патологічному процесі?

A. @Альдостерону

B. Інсуліну

C. Адреналіну

D. Тироксину

E. Вазопресину

503. У хворого, що протягом року лікувався преднізо-лоном, при поступовому зменшенні дози з’явилися швидка фізична і психічна втомлюваність, артері-альна гіпотензія, схуднення, прогресуюча гіперпі-гментація шкіри. Що лежить в основі цього симп-томокомплексу?

A. @Хронічна гіпофункція кори наднирників

B. Гостра гіпофункція кори наднирників

C. Хронічна гіперфункція кори наднирників

D. Гостра гіперфункція кори наднирників

E. Дисфункція кори наднирників

504. Після перенесеного сепсису у хворого з¢явився бронзовий колір шкіри, характерний для Аддісо-нової хвороби. Механізм гіперпігментації полягає у підвищенні секреції гормону:

A. @Меланоцитстимулюючого

B. Соматотропного

C. Гонадотропного

D. Лактотропного

E. Тиреотропного

505. Хворий з Аддісоновою хворобою має виражену схильність до гіпоглікемії. Чим спричинена гіпоглі-кемія у хворого?

A. @Пригніченням глюконеогенезу

B. Активацією глікогенолізу

C. Пригніченням синтезу глікогену

D. Пригніченням гліколізу

E. Активацією ліполізу

506. У хворого з частими ортостатичними колапсами діагностовано хвороба Аддісона. Зменшення чут-ливості до якого гормону є головною ланкою в патогенезі цих проявів при гіпокортицизмі?

A. @Адреналіну

B. Соматотропіну

C. Соматостатину

D. Альдостерону

E. ТТГ

507. При обстеженні у хворого виявлено гіпоглікемію, м’язову слабість, зниження артеріального тиску. Що може бути причиною цих проявів?

A. @Недостатня продукція глюкокортикоїдів

B. Гіпофункція щитовидної залози

C. Гіпофункція статевих залоз

D. Гіпофункція мозкового шару наднирників

E. Гіпофункція прищитоподібних залоз

508. У хворого виявили гіперплазію кори наднирників. АТ – 190/90 мм.рт.ст.; у крові – вміст глюкози – 20 ммоль/л, збільшення кортікотропину; у сечі – глюкозурія. Спостерігається ожиріння, гірсутизм. Для якої патології характерні виявлені зміни?

A. @Хвороби Іценка–Кушинга.

B. Хвороби Аддісона.

C. Синдрому Іценка–Кушинга.

D. Адипозогенітальної дистрофії.

E. Хвороби Барракера– Сіммондса.

509. У хворого на ожиріння виявили синдром Іценка – Кушинга. Яке порушення метаболізму є головною ланкою в патогенезі ожиріння в хворих на корти-золпродукуючі пухлини?

A. @Гіперглікемія

B. Гіпераміноацидемія

C. Гіперβ–ліпопротеінемія

D. Гіпоα–ліпопротеінемія

E. Гіперхолестеринемія

510. У хворої виявлені ожиріння обличчя, шиї і верх-ньої частини тулуба, стрії на животі, гірсутизм, гі-перглікемія і гіпертензія. Гіперпродукція якого го-рмону може бути причиною таких змін?

A. @Кортизолу

B. Альдостерону

C. Адреналіну

D. Тестостерону

E. Естрадіолу

511. Хвора з діагнозом хвороба Іценка – Кушінга ска-ржиться на м’язову слабкість, гіпертензію, ожи-ріння верхньої частини тулуба, шиї та обличчя. Первинна гіперпродукція якого гормону зумовлює дану хворобу?

A. @Кортикотропіну

B. Кортизолу

C. Катехоламінів

D. Інсуліну

E. Тироксину

512. У хворої 44 років виявлені ознаки гіперкортициз-му. Що допоможе диференціювати хворобу Іцен-ка – Кушінга від синдрому Іценка – Кушінга?

A. @Рівень кортикотропіну в крові

B. Кількість еозинофілів у крові

C. Рівень кортизолу в крові

D. Вміст 17–оксикетостероїдів у сечі

E. Рівень глюкози у крові

513. У хворого виявлена гормон–продукуюча пухлина пучкової зони кори наднирників. Вміст у крові якого гормону зменшується в цьому випадку?

A. @АКТГ

B. ТТГ

C. СТГ

D. ФСГ

E. ЛГ

514. У хворого відмічається періодичні напади головно-го болю з серцебиттям і сильним потовиділенням. При обстеженні виявлені гіпертензія, гіпергліке-мія, підвищення основного обміну, тахікардія. Для якої патології наднирників характерні ці симпто-ми?

A. @Гіперфункції мозкової речовини наднирників

B. Гіпофункції мозкової речовини наднирників

C. Гіперфункції кори наднирників

D. Гіпофункції кори наднирників

E. Первинному гіперальдостеронізмі

515. Хворий 42–х років висуває скарги на сильне сер-цебиття, пітливість, нудоту, порушення зору, тре-мор рук, підвищення артеріального тиску. З анам-незу: 2 роки тому було встановлено діагноз фео-хромоцитома. Гіперпродукція яких гормонів зумо-влює цю патологію?

A. @Катехоламінів

B. АКТГ

C. Альдостерону

D. Глюкокортикоїдів

E. Тиреоїдних гормонів

516. У хворої з феохромоцитомою після психічного на-вантаження виникає тахікардія, підвищується ар-теріальний тиск, з’являється різкий біль у надче-ревній ділянці. Ці приступи зумовлені:

A. @Масивним викидом катехоламінів наднирни-ками

B. Збільшенням секреції тиреоїдних гормонів

C. Звільненням норадреналіну симпатичними нер-вами

D. Активацією вегетативних ядер гіпоталамуса

E. Підвищеним синтезом адренокортикотропного гормону

517. У дівчинки діагностований адреногенітальний си-ндром з ознаками вірилізації. Надмірна секреція яких гормонів наднирників зумовила вірилізацію?

A. @Андрогенів

B. Глюкокортикоїдів

C. Естрогенів

D. Мінералокортикоїдів

E. Катехоламінів

518. У дівчинки діагностований адреногенітальний си-ндром. Гіперпродукція якого гормону стала при-чиною надлишку андрогенів?

A. @АКТГ

B. ТТГ

C. СТГ

D. ФСГ

E. ЛГ

519. У дівчинки діагностований адреногенітальний си-ндром. Первинний генетичний дефект синтезу яких гормонів наднирників зумовив дану патоло-гію?

A. @Глюкокортикоїдів

B. Естрогенів

C. Адреналіну

D. Андрогенів

E. Норадреналіну

520. У чоловіка 42 років, що довгий час перебував у стані стресу, у сечі значно збільшений вміст глю-кокортикостероїдів, що в першу чергу свідчить про підвищення секреції:

A. @АКТГ

B. Адреналіну

C. Альдостерону

D. Норадреналіну

E. Естрадіолу

521. Чоловік 25 років надійшов до лікарні через годину після автомобільної аварії. Спостерігається карти-на гострого стресу без виражених ушкоджень. Яке явище в периферійній крові буде характер-ним для цього стану?

A. @Еозинопенія

B. Анемія

C. Моноцитопенія

D. Лейкопенія

E. Нейтрофілія

522. У тварини, що знаходиться в стані стресу, спосте-рігаються клінічні прояви загального адаптаційно-го синдрому (гіпотонія м'язів і артеріальна гіпоте-нзія, гіпотермія, гіпоглікемія, згущення крові, ео-зинопенія). У якій із стадій стресу, що послідовно розвиваються, знаходиться тварина?

A. @Фазі шоку

B. Фазі протівошоку

C. Стадії виснаження

D. Стадії резистентності

E. Торпідній стадії

523. У жінки після тяжкого психоемоційного стресу виникла аменорея. Функції якого органу поруше-ні?

A. @Гіпоталамуса

B. Середньої частини гіпофізу

C. Задньої частини гіпофізу

D. Надниркових залоз

E. Щитоподібної залози

524. У експерименті на щурах викликали електротрав-му, періодично пропускаючи електричний струм через організм тварин, унаслідок чого виникли певні пошкодження органів і систем організму. Які з них характерні для прояву загального адапта-ційного синдрому?

A. @Виразки слизової оболонки шлунка і 12–типалої кишки

B. Фібриляція серцевого м'яза

C. Артеріальна гіпотензія

D. Тонічні судоми

E. Зупинка дихання

525. У дослідах на щурах, що перенесли гравітаційний стрес, виявлені однотипні зміни. Яка з виявлених змін є специфічною для стресу?

A. @Інволюція тіміко–лімфатичного апарату

B. Атрофія кори надниркових

C. Гіперемія селезінки

D. Гранулоцитопенія

E. Гіпотрофія кори надниркових

526. Для якої стадії загального адаптаційного синдрому характерні гіпертрофія кори наднирників і секре-ція великої кількості гормонів, активація анаболі-чних процесів, посилення гліконеогенезу?

A. @Резистентності

B. Тривоги

C. Виснаження

D. Аварійної

E. Завершальної

527. У спортсмена, який тривалий час вживав анаболі-чні стероїди, виник гіпогонадизм. Чим зумовлений цей стан?

A. @Гіпопродукцією гонадотропіну

B. Гіпопродукцією реніну

C. Гіпопродукцією соматотропіну

D. Гіпопродукцією кортикотропіну

E. Гіпопродукцією адреналіну

528. До лікаря звернулися батьки хлопчика 10–ти ро-ків, у якого відзначалося збільшення волосяного покриву на тілі, ріст бороди і вусів, низький голос. Збільшення секреції якого гормону можна припус-тити?

A. @Тестостерону

B. Кортизолу

C. Прогестерону

D. Соматотропіну

E. Естрогену

529. До лікаря звернулася жінка 58 років зі скарга–ми на прогресуюче збільшення маси тіла, підвищену стомлюваність. З анамнезу відомо клімакс насту-пив в 55 років, характер харчування помірний. Що може зумовити появу ожиріння у жінки?

A. @Гіпофункція статевих залоз

B. Хвороба Іценко–кушинга

C. Аліментарний дисбаланс

D. Пухлина яєчника

E. Гіпофункція щитовидної залози

530. У дитини 2–х років виникли судоми внаслідок зниження концентрації іонів кальцію в плазмі крові. Функція якого ендокринного органу зниже-на?

A. @Прищитоподібних залоз

B. Шишкоподібної залози

C. Кори наднирників

D. Гіпофіза

E. Тимуса

531. Через 1–2 доби після видалення у собаки прищи-топодібних залоз спостерігалось: в’ялість, спрага, різке підвищення нервово–м’язової збудливості з розвитком тетанії. Яке порушення обміну елект-ролітів виникло?

A. @Гіпокальціємія

B. Гіперкальціємія

C. Гіпомагніємія

D. Гіпермагніємія

E. Гіпонатріємія

532. Хворому 45–ти років при оперативному втручанні на щитоподібній залозі випадково видалили при-щитоподібні залози. Це призвело до:

A. @Тетанії

B. Підвищення рівня кальцію в крові і резорбції кі-сток

C. Підвищення рівня кальцію, натрію і калію в крові

D. Зниження артеріального тиску

E. Підвищення артеріального тиску

533. У пацієнта 20 років спостерігається гіпоплазія емалі, порушення утворення дентину. Гіперпро-дукція якого гормону стала причиною вказаної патології?

A. @Паратгормону

B. Тироксину

C. Тиреотропіну

D. Соматотропіну

E. Тиреокальцитоніну

534. У жінки 46 років після операції на щитоподібній залозі в невдовзі з’явилися фібрилярні посмику-вання м’язів рук, ніг, обличчя. Ці порушення мож-на усунути шляхом введення

A. @Паратгормону

B. Трийодтироніну

C. Тиреотропіну

D. Тироксину

E. Тіреотропного гормону

535. У щура з хронічною нирковою недостатністю ви-явлено остеопороз, патологічна кальцифікація внутрішніх органів, артеріальна гіпертензія. З по-силенням дії якого гормону пов’язані ці порушен-ня?

A. @Паратгормон

B. Тироксин

C. Трийодтиронін

D. Кальцитонін

E. Адреналін

536. У дівчини 14 років, яка скаржиться на значне схуднення, спрагу, поліурію, виявлено гіпергліке-мію натщесерце та зниження толерантності до глюкози. При нестачі якого гормону може розви-нутись такий стан?

A. @Інсуліну

B. Тироксину

C. Глюкокортикоїдів

D. Альдостерону

E. Соматотропного гормону

537. У хворого діагностовано цукровий діабет 1 типу. Яка ендокринопатія за патогенезом виникла у хворого?

A. @Залозиста

B. Дисрегуляторна

C. Периферична

D. Гіпоталамічна

E. Пострецепторна

538. У хворого вперше виявили цукровий діабет 2 ти-пу. У крові гіперглікемія натщесерце 8 ммоль/л, гіперінсулінемія. Яка ендокринопатія за патогене-зом виникла у хворого?

A. @Периферична

B. Дисрегуляторна

C. Залозиста

D. Гіпоталамічна

E. Пострецепторна

Патофізіологія нервової системи

539. Відомо, що при раптовому припиненні надходжен-ня кисню в головний мозок, втрата свідомості на-стає вже через 6–7 секунд. Яке патологічне яви-ще лежить в основі порушення функцій клітин го-ловного мозку в даному випадку?

A. @Енергодефіцит

B. Токсичне ураження

C. Іонний дисбаланс

D. Гіпергідратація клітин мозку

E. Гіпогідратація клітин мозку

540. Відомо, що при зниженні рівня глюкози в крові дуже швидко виникає гіпоглікемічна кома. Що лежить в основі порушення функцій клітин голов-ного мозку в даному випадку?

A. @Енергодефіцит

B. Токсичне ураження

C. Іонний дисбаланс

D. Гіпогідратація клітин мозку

E. Гіпергідратація клітин мозку

541. У хворого з хворобою Крейтцфельда–Якоба роз-винулася губчаста енцефалопатія. До якої групи належить інфекційний агент, що викликав це за-хворювання?

A. @Пріони

B. РНК–вмісні віруси

C. ДНК–вмісні віруси

D. Бактеріі

E. Гриби

542. З нейронів ніколи не утворюються пухлини, оскі-льки вони не діляться і в них неможлива одна з стадій канцерогенезу. Яка це стадія?

A. @Промоція

B. Ініціація

C. Трансформація

D. Прогресія

E. Малігнізація

543. У хворого з черепно–мозковою травмою діагнос-товано автоімунний енцефаліт. Чим може бути зумовлене це захворювання?

A. @Пошкодженням гематоенцефалічного бар’єру

B. Зняттям імунологічної толерантності

C. Недостатністю гуморального імунітету

D. Недостатністю клітинного імунітету

E. Недостатністю фагоцитозу

544. У хворого, що переніс черепно–мозковоу травму, виник енцефаліт. Який механізм призвів до автоі-мунного ушкодження головного мозку?

A. @Демаскування антигену

B. Трансформація лімфоцита

C. Втрата толерантності

D. Гіперпродукція IgE

E. Гіпопродукція IgG

545. У дитини 6 місяців з уповільненим моторним і пси-хічним розвитком відзначається збліднення шкір-них покривів, волосся, райдужної оболонки ока, позитивна проба Фелінга (з 3% розчином трихло-роцтового заліза). Яке з зазначених спадкових за-хворювань виявлено в дитини?

A. @Фенілкетонурія

B. Галактоземія

C. Алкаптонурія

D. Хвороба Дауна

E. Альбінізм

546. У тварини після перетину n. ophthalmicus з'явили-ся енофтальм, звуження зіниці та в'ялість повік. Через деякий час рогівка помутніла, утворилася виразка. Порушення якої функції нервового воло-кна призвело до утворення виразки?

A. @Трофічної

B. Чутливої

C. Больової

D. Рухової

E. Вегетативної

547. Після проведення блокади за Вишневським у хво-рого з'явилися енофтальм, звуження зіниці та в'ялість верхньої повіки. З порушенням якої фун-кції нервового волокна пов'язана описана симп-томатика?

A. @Вегетативної

B. Чутливої

C. Больової

D. Трофічної

E. Рухової

548. У хворого після травматичної перерізки сідничного нерва виникли трофічні зміни шкіри. Основним механізмом їх появи є:

A. @Припинення аксоплазматичного току

B. Втрата нервом збудливості

C. Фагоцитоз нервових закінчень

D. Руйнування мієлінової оболонки

E. Пошкодження перехватів Ранв’є

549. В реанімаційне відділення доставили водія, який потрапив в автомобільну аварію. Він не зразу ре-агує на запитання, байдужий до всього, блідий, дихання поверхневе, рідке, АТ дорівнює 75/50 мм рт. ст. Назвіть головну ланку патогенезу проявів з боку центральної нервової системи (ЦНС).

A. @Гальмування ЦНС.

B. Збудження ЦНС.

C. Крововтрата.

D. Токсемія.

E. Перерозподіл крові.

550. Що лежить в основі механізму формування пато-логічної домінанти?

A. @Формування генератора патологічно посиленого збудження.

B. Феномен випадіння.

C. Позамежове гальмування.

D. Перезбудження.

E. Парабіоз.

551. У хворого через 7 років після резекції шлунка ви-явлена гіперхромна В12 дефіцитна анемія з синд-ромом фунікулярного мієлозу. Який патогенез ураження спинного мозку?

A. @Накопичення метилмалонової кислоти.

B. Гіпоксичне пошкодження речовини мозку

C. Порушення синтезу ДНК

D. Дефіцит фолієвої кислоти

E. Гіпотрансферинемія

552. В єксперименті у тварини в результаті проведеної перерізки депресорного нерва та руйнування ка-ротидних клубочків виникла стійка артеріальна гіпертензія. З порушенням якої функції централь-ної нервової системи пов’язане це явище?

A. @Вегетативної

B. Вищої нервової діяльності

C. Сенсорної

D. Моторної

E. Трофічної

553. В результаті перенесеної нейроінфекції хворий втратив здатність впізнавати знайомий предмет навпомацки. Як називається порушений вид скла-дної чутливості?

A. @Стереогноз

B. Відчуття дискримінації

C. Двомірно–просторове відчуття

D. Відчуття локалізації

E. М’язово–суглобове відчуття

554. У хворих з В12 – дефіцитною анемією виникають дегенеративні процеси в задніх та бічних стовпах спинного мозку (фунікулярний мієлоз). Ураження аксонів пов’язане з порушенням утворення:

A. @Мієліну

B. Ацетилхоліну

C. Дофаміну

D. Серотоніну

E. Норадреналіну

555. Під час операції на легенях у хворого виникла зупинка серця. Регулярний серцевий ритм вдало-ся відновити лише через 10 хв. Найглибші зміни внаслідок гіпоксії сталися в

A. @Корі головного мозку

B. Серці

C. Печінці

D. Нирках

E. Селезінці

556. Після автомобільної катастрофи у хворого діагно-стована травма середньої третини плеча з непов-ним розривом серединного нерва. Окрім рухових і сенсорних порушень нижче місця травми, хворий скаржиться на нестерпний, пекучий біль. Який вид болю у хворого?

A. @Каузалгія.

B. Відбитий.

C. Соматичний.

D. Фантомний.

E. Проекційний.

557. У хворого в результаті вогнепального поранення стегна пошкоджений сідничний нерв. Будь–які впливи на хвору кінцівку спричиняють нестерпний біль. Який механізм формування больових відчут-тів найбільш імовірний при цьому?

A. @Каузалгічний

B. Рефлекторний

C. Фантомний

D. Гіпофункція ендорфіну

E. Гіпофункція енкефаліну

558. Хворий з анемією Аддісона – Бірмера скаржиться на відчуття „повзання мурашок” по тілу. Як нази-вається це порушення чутливості?

A. @Парестезія

B. Анестезія

C. Гіпестезія

D. Анальгезія

E. Гіперестезія

559. Після травматичного видалення зуба хворий ска-ржиться на тупий, не чітко локалізований, силь-ний біль у яснах, підвищення температури тіла до 37,50С. Діагностовано альвеоліт. Який вид болю у даного хворого?

A. @Протопатичний

B. Епікритичний

C. Вісцеральний

D. Відбитий

E. Фантомний

560. Хворий 50 років з цукровим діабетом 1–го типу скаржиться на повну втрату чутливості пальців ніг. Як називається таке порушення чутливості?

A. @Анестезія

B. Анальгезія

C. Гіпестезія

D. Парестезія

E. Гіперестезія

561. Хворий 43 років, що чотири місяці тому переніс травматичну ампутацію лівої нижньої кінцівки, скаржиться на відчуття присутності ампутованої кінцівки та постійний, іноді нестерпний біль у ній. Яка форма хронічного болю у хворого?

A. @Фантомний біль

B. Рефлекторний біль

C. Таламічний біль

D. Невралгія

E. Каузалгія

562. У хворого, що надійшов до стаціонару зі скаргами на загальне нездужання, нудоту та біль справа внизу живота, після огляду діагностовано гострий апендицит. Який різновид болю у хворого?

A. @Вісцеральний біль

B. Соматичний глибокий біль

C. Соматичний поверхневий ранній біль

D. Соматичний поверхневий пізній біль

E. Фантомний біль

563. У хворого 65 років періодичні больові відчуття під лівою лопаткою і в лівому плечі різко посилилися після значного фізичного навантаження. Методом ЕКГ діагностовано гострий інфаркт міокарда. Який вид болю мав місце в даному випадку?

A. @Рефлекторний

B. Соматичний

C. Фантомний

D. Каузалгія

E. Невралгія міжреберного нерва

564. У чоловіка 68 років через тиждень після перене-сеного інсульту відсутні рухи у верхній і нижній лівих кінцівках. Тонус м'язів цих кінцівок і рефле-кси в них підвищені, наявні патологічні рефлекси. Яка форма паралічу у хворого?

A. @Геміплегія

B. Моноплегія

C. Диплегія

D. Параплегія

E. Тетраплегія

565. У хворого відсутні довільні і мимовільні рухи ниж-ніх кінцівок, що супроводжуються атонією та ат-рофією м'язів. Який параліч має місце в даному випадку?

A. @Периферичний

B. Спастичний

C. Центральний

D. Геміплегія

E. Моноплегія

566. У хворого після перенесеного ішемічного інсульту стали неможливими довільні рухи у правих кінців-ках, пасивні рухи збереглися в повному обсязі. При пальпації визначається підвищений тонус м'я-зів кінцівок. Яка форма порушення рухової функ-ції має місце у хворого?

A. @Центральний параліч

B. Периферичний парез

C. Мозочкова атаксія

D. Периферичний параліч

E. Тетанія

567. У постраждалого внаслідок ДТП встановлено пе-релом хребта на рівні VII шийного хребця. Нижче рівня ураження справа відмічається порушення рухової функції та випадіння глибокої чутливості, а зліва – випадіння поверхневої чутливості. Який варіант ушкодження спинного мозку найбільш імовірний?

A. @Синдром Броун–Секара

B. Порушення задніх рогів спинного мозку

C. Ушкодження передніх рогів спинного мозку

D. Тотальне ушкодження спинного мозку

E. Ушкодження задніх канатиків спинного мозку

568. У хворого внаслідок травми поперекового відділу хребта відбувся половинний розрив спинного моз-ку. При обстеженні на боці ураження спостеріга-ється параліч кінцівки, втрата тактильної та гли-бокої суглобово–м’язової чутливості, на протиле-жному боці – втрата больової і температурної чу-тливості. Для якого синдрому характерний такий симптомокомплекс?

A. @Синдром Броун–Секара

B. Синдром Верніке–Корсакова

C. Синдром Джуліана–Барра

D. Синдром Ламберта–Ітона

E. Синдром Паркінсона

569. У жінки після інсульту відсутні довільні рухи у верхній і нижній кінцівках справа. Тонус м'язів і рефлекси в них підвищені. Спостерігаються пато-логічні рефлекси. Пошкодженням яких клітин мо-зку можна пояснити виявлену у хворої симптома-тику?

A. @Пірамідних клітин кори

B. Клітин нейроглії

C. Альфа–мотонейронів рухових ядер

D. Чутливих нейронів

E. Нейронів мозочка

570. У хворого на поліомієліт відсутні довільні та мимо-вільні рухи правої ноги, що супроводжуються атонією і атрофією м'язів. Які клітини мозку пош-коджені у хворого?

A. @Альфа–мотонейрони рухових ядер

B. Пірамідні клітини кори

C. Клітини нейроглії

D. Чутливі нейрони

E. Нейрони мозочка

571. У хворий Л., 40 років, через місяць після автомо-більної травми відсутні активні рухи у всіх сугло-бах правої ноги. Об'єм м'язів в ділянці гомілки правої ноги на 2 см менше, ніж на лівій. Ахіллів і колінний рефлекси справа відсутні. Визначається термогіпестезия та гіпальгезія на зовнішній пове-рхні правої ноги, втрата проприоцептивной чутли-вості в ділянці стопи. Порушенням якого відділу рухового аналізатора обумовлені наявні розлади?

A. @Периферичних нервів

B. Спінальних провідних шляхів

C. Пірамідних нейронів

D. Нервово–м'язових синапсів

E. Екстрапірамідних нейронів

572. У хворого порушена координація рухів: рухи роз-машисті, непропорційні, хода «півняча», «п'яна», пальцем важко потрапити в ціль. Вказані симпто-ми виникають навіть з відкритими очима. З ура-женням яких нервових структур пов'язано це по-рушення?

A. @Мозочка

B. Пірамідної системи

C. Лімбічної системи

D. Кори мозку

E. Стовбура мозку

573. У тварини після видалення мозочка рухи втратили плавність та точність, виникла дисметрія. Як на-зивається описане явище?

A. @Атаксія

B. Тремор

C. Гіпокінез

D. Ригідність

E. Атетоз

574. При повороті голови вбік у хворого з'являються відчуття «провалювання», нудота, блювання, та-хікардія та блідість обличчя. Хода хитка. Уражен-ня якої структури найбільш імовірне у хворого?

A. @Вестибулярного апарату

B. Мозочка

C. Середнього мозку

D. Кори лобної частки

E. Внутрішньої капсули

575. У хворого відмічені швидкі мимовільні стереотипні скорочення м’язів обличчя, що імітують мигання і примружування. До якої форми розладів рухової функції нервової системи слід віднести описане порушення?

A. @Гіперкінез

B. Гіпокінез

C. Акінезія

D. Атаксія

E. Арефлексія

576. 78–річному хворому встановлено діагноз «паркін-сонізм». Недостатнє утворення якого медіатора в чорній субстанції та блакитній плямі головного мо-зку є головною ланкою в патогенезі цієї хвороби?

A. @Дофаміну

B. Ацетилхоліну

C. Адреналіну

D. Гістаміну

E. Норадреналіну

577. У дівчинки 12 років спостерігаються швидкі, не-сподівані, розмашисті, позбавлені стереотипності рухи. У них беруть участь м'язи обличчя і кінці-вок. М'язовий тонус знижений. Яка форма пору-шень рухової функції нервової системи виникла у хворої?

A. @Хорея

B. Тремор

C. Атетоз

D. Тик

E. Абазія

578. У хворого виникли мимовільні повільні „червопо-дібні” рухи в пальцях рук і ніг (згинання і розги-нання пальців, що настають один за одним, пере-розгинання середніх і кінцевих фаланг), що поси-люються при довільних рухах. Як називається ця форма порушення рухової функції нервової сис-теми?

A. @Атетоз

B. Хорея

C. Тремор

D. Тик

E. Абазія

Екстремальні стани

579. У хворого з печінковою недостатністю з’явилася сонливість, а потім він знепритомнів. Накопичення якої речовини порушує енергозабезпечення ней-ронів при печінковій енцефалопатії?

A. @Аміак

B. Глюкоза

C. ГАМК

D. Метилмеркаптан

E. Прямий білірубін

580. У хворого з печінковою недостатністю з’явилася сонливість, а потім він знепритомнів. Яка речови-на гальмує синаптичну передачу в мозку при пе-чінковій енцефалопатії?

A. @ГАМК

B. Глюкоза

C. Метилмеркаптан

D. Аміак

E. Прямий білірубін

581. Хворого доставили у клініку в коматозному стані. В анамнезі – захворювання на цукровий діабет 1 типу протягом 5 років. Об’єктивно: дихання шум-не, часте, глибоке, у повітрі, що видихається, від-чутний запах ацетону. Вміст глюкози в крові 15,2 ммоль/л, кетонових тіл – 10,6 ммоль/л. Для якого ускладнення цукрового діабету характерні такі показники?

A. @Кетоацидотична кома

B. Печінкова кома

C. Лактацидемічна кома

D. Гіпоглікемічна кома

E. Гіперосмолярна кома

582. Хворому на цукровий діабет І типу ввели інсулін. Через деякий час у хворого з’явилися загальна слабкість, гостре відчуття голоду, дратівливість, запаморочення, кома. Який основний механізм виникнення коми у пацієнта?

A. @Вуглеводне голодування головного мозку.

B. Зменшення глюконеогенезу.

C. Посилення кетогенезу.

D. Посилення глікогенолізу.

E. Посилення ліпогенезу.

583. У хворого на цукровий діабет розвинулась гіпе-росмолярна кома (рівень глюкози 54 ммоль/л). Яке патологічне явище лежить в основі порушен-ня функцій клітин мозку в даному випадку?

A. @Гіпогідратація клітин мозку

B. Токсичне ураження

C. Енергодефіцит

D. Алкалоз

E. Гіпергідратація клітин мозку (набухання)

584. Хворий 35 років доставлений у стаціонар непри-томним, дихання часте, глибоке (типу Куссмауля), АТ 90/45 мм рт. ст., з рота запах ацетону. Глюко-за крові – 18 ммоль/л, у сечі – глюкозурія, кето-нурія. Яка кома виникла у даного хворого?

A. @Кетоацидотична

B. Гіпоглікемічна

C. Гіперосмолярна

D. Печінкова

E. Лактацидотична

585. Хворий 55 років доставлений у стаціонар непри-томним, дихання часте, поверхневе, АТ 90/45 мм рт. ст., шкіра в'яла, суха. Лабораторно: глюкоза крові – 55 ммоль/л, Na – 150 ммоль/л, Cl – 15 ммоль/л Яка кома виникла у даного хворого?

A. @Гіперосмолярна

B. Гіпоглікемічна

C. Кетоацидотична

D. Печінкова

E. Лактацидотична

586. Хворий 68 років доставлений у стаціонар непри-томним, дихання часте, глибоке, АТ 90/45 мм рт. ст., шкіра в'яла, суха. Лабораторно: глюкоза кро-ві – 15 ммоль/л, молочна кислота крові – 12 ммоль/л. Яка кома виникла у даного хворого?

A. @Лактацидотична

B. Гіпоглікемічна

C. Гіперосмолярна

D. Кетоацидотична

E. Печінкова

587. У хворого на хронічну ниркову недостатність з’явилися анорексія, нудота, блювота, порушення ритму серця, свербіння шкіри, психічні порушення аж до коми. Який механізм розвитку уремічної коми є головним?

A. @Гіперазотемія

B. Нирковий алкалоз

C. Порушення ліпідного обміну

D. Зміни вуглеводного обміну

E. Зневоднення

588. У хворого, що раптово знепритомнів, у плазмі крові вміст Na+ 180 ммоль/л, осмолярність – 340 мосмоль/л. Чим, найімовірніше, зумовлене виник-нення коми у хворого?

A. @Дегідратацією нейронів

B. Вуглеводним голодуванням

C. Алкалозом

D. Гіперкаліємією

E. Набуханням нейронів

589. У хворого на хворобу Аддісона внутрішньовенна інфузія глюкози призвела до «водного отруєння» та гіпоосмолярної коми. Чим зумовлене виникнен-ня коми у хворого?

A. @Набуханням нейронів

B. Вуглеводним голодуванням

C. Алкалозом

D. Дегідратацією нейронів

E. Гіперкаліємією

590. Хворого доставлено до медичної установи в кома-тозному стані. Зі слів супровідних удалося встано-вити, що хворий знепритомнів під час тренування на завершальному етапі марафонської дистанції. Яка кома найбільш імовірна в цього хворого?

A. @Гіпоглікемічна

B. Гіперглікемічна

C. Печінкова

D. Ацидотична

E. Гіпотиреоїдна

591. У лікарню швидкої допомоги поступив чоловік із закритим переломом стегна. Постраждалий у збу-дженому стані. Що вимагає негайної корекції в патогенезі травматичного шоку у пацієнта?

A. @Біль.

B. Ацидоз.

C. Інтоксикація.

D. Гіпоксія.

E. Аритмія серця.

592. Після введення анестетика в пацієнта розвинувся анафілактичний шок. Який механізм визначає ро-звиток гострої недостатності кровообігу при цьо-му?

A. @Зниження судинного опору

B. Гіперволемія

C. Біль

D. Активація симпато–адреналової системи

E. Зниження скоротливої функції серця

593. У патогенезі гострої недостатності кровообігу при анафілактичному шоку є розширення артеріол та підвищення проникності капілярів. Який з медіа-торів зумовлює розвиток даних порушень?

A. @Гістамін

B. Гепарин

C. Тромбоксани

D. Лімфокіни

E. Комплемент

594. У хворого після введення лідокаїну розвинувся анафілактичний шок. Які медіатори анафілаксії спричинюють безпосередньо обструкцію нижніх дихальних шляхів і гостру недостатність дихання?

A. @Гістамін, лейкотрієни

B. Хемотаксичні фактори

C. Кініни

D. Комплемент

E. Катехоламіни

595. У хворого на інфаркт міокарда з’явилися клінічні ознаки кардіогенного шоку. Який із вказаних ни-жче патогенетичних механізмів є первинним у ро-звитку шоку?

A. @Зниження насосної функції серця

B. Депонування крові у венах

C. Зниження діастолічного притоку крові до серця

D. Збільшення судинного тонусу

E. Зниження судинного тонусу

596. Хворий, 52 років, був доставлений в клініку з симптомами гострого панкреатиту та явищами панкреатичного шоку. Що є основним механізмом розвитку панкреатичного шоку?

A. @Надходження активованих протеаз у кров

B. Стимулювання панкреатичної секреції

C. Порушення відтоку панкреатичного соку

D. Зменшення інкреції інсуліну

E. Підвищення артеріального тиску

597. Хворий був доставлений у клініку з симптомами гострого панкреатиту та явищами панкреатичного шоку. Активація якої системи є причиною гіпотен-зії при панкреатичному шоку?

A. @Калікреїн–кінінової

B. Ренін–ангіотензинової

C. Фібринолізу

D. Антикоагулянтної

E. Глутатіонової

598. Під час легеневої кровотечі у хворого на туберку-льоз пацієнт втратив 45 % загального об’єму кро-ві. Яке ускладнення, найбільш вірогідно, може виникнути у хворого?

A. @Постгеморагічний шок

B. Анафілактичний шок

C. Больовий шок

D. Ішемія мозку

E. Інфаркт міокарду

599. Під час легеневої кровотечі у хворого на туберку-льоз пацієнт втратив 45 % загального об’єму кро-ві. Який із вказаних нижче патогенетичних меха-нізмів є первинним у розвитку постгеморагічного шоку?

A. @Гіповолемія

B. Депонування крові у венах

C. Зниження насосної функції серця

D. Збільшення судинного тонусу

E. Зниження судинного тонусу

600. У хворого з опіками 40 % поверхні тіла розвинув-ся опіковий шок. Який механізм розвитку шоку домінує на його початку?

A. @Больовий

B. Порушення білкового обміну

C. Аутоімунізація

D. Порушення мінерального обміну

E. Зневоднення

601. Потерпілого з джутом на стегні привезли через 3 години після травми в задовільному стані. Після зняття джгута стан хворого різко погіршився: з'явився різкий набряк тканини стегна, частий пульс, холодний піт, артеріальний тиск знизився до 80/60 мм рт.ст. Яке ускладнення розвинулось?

A. @Токсемічний шок

B. Анафілактичний шок

C. Кардіогенний шок

D. Геморагічний шок

E. Колапс

602. У хворого з стафілакоковою інфекцією виник сеп-тичний шок зі значним зменшенням артеріального тиску. Який провідний механізм викликав у хворо-го зниження артеріального тиску?

A. @Відкриття артеріовенозних шунтів.

B. Збільшення мікроциркуляторного русла.

C. Збільшення об’єму крові.

D. Пригнічення діяльності судинно–рухового цен-тру.

E. Збільшення роботи серця.

603. У хворого, доставленого в клініку каретою невід-кладної допомоги, перебіг геморогічного шоку ускладнився розвитком гострої ниркової недоста-тності. Назвіть ініціальну ланку в механізмі розви-тку цього ускладненя.

A. @Централізація кровообігу з виникненням ішемії нирок

B. Підвищення проникності стінки капілярів.

C. Вторинний гіперальдостеронізм

D. Викид у кров вазопресину

E. Активація парасимпатичної системи.

604. Чоловіка доставили з місця автомобільної аварії в лікарню в непритомному стані. Шкірні покриви бліді, пульс частий і поверхневий. Переломів кіс-ток і пошкодження головного мозку не виявлено. При пункції черевної порожнини отримано значну кількість крові. Первинною причиною тяжкого стану потерпілого є

A. @Гіповолемія

B. Еритропенія

C. Гіпонатріємія

D. Гіпопротеїнемія

E. Гіпоінсулінемія

605. Кардіогенний шок у хворого на інфаркт ускладни-вся розвитком метаболічного ацидозу. Який ком-пенсаторний механізм може вмикатися при цьому?

A. @Гіпервентиляція альвеол

B. Гіповентиляція альвеол

C. Зменшення амоніогенезу в нирках

D. Зменшення реабсорбції гідрокарбонату в нир-ках

E. Зменшення ацидогенезу в нирках

606. При колаптоїдному зниженні артеріального тиску у хворого виник певний тип гіпоксії. Який тип гі-поксії розвивається при коллапсі ?

A. @Циркуляторний.

B. Дихальний.

C. Гемічний.

D. Тканинний

E. Гіпоксичний

607. У потерпілого після травми развинувся травмати-чний шок. При огляді: АТ = 140/90 мм. рт. ст., пульс 120 уд/хв., збуджений, багатослівний, блі-дий. Яка стадія шоку у хворого?

A. @Еректильна

B. Латентна

C. Термінальна

D. Торпідна

E. Кінцева

608. Експериментальній тварині викликали септичний шок. Що є головним у розвитку септичного шоку?

A. @Відкриття артеріовенозних анастомозів

B. Зменшення тонусу судин

C. Збільшення тонусу судин

D. Збільшення проникності судинної стінки

E. Зменшення серцевого викиду

609. Після перенесенного ДТП у потерпілого: ниткопо-дібний пульс, стійка гіпотонія, часте та поверхне-ве дихання, блідість шкіри, холодний піт, сопор. Яка стадія шоку наймовірніше розвинулась у хво-рого?

A. @Торпідна.

B. Термінальна

C. Еректильна

D. Предшок

E. Агонія

610. В клинику доставлен пострадавший в автоаварии пациент: сознание сохранено, жалоб не предъяв-ляет, пульс 120 уд./мин, нитевидный, АД–60/40 мм рт. ст., кожные покровы бледные. В какой стадии шока находится пострадавший?

A. @Торпидной

B. Эректильной

C. Терминальной

D. Агональной

E. Метаболической

611. У робітника мартенівського цеху під кінець зміни різко підвищилась температура тіла до 39,80С. Скарги на слабість, шум у вухах , озноб. Відміча-ється загальмованість, розширення зіниць, тремор пальців рук. АТ 85/55 мм рт. ст. Який екстрема-льний стан у хворого?

A. @Тепловий шок

B. Колапс

C. Постгеморагічний шок

D. Опіковий шок.

E. Кардіогенний шок

612. Хвора доставлена бригадою швидкої допомоги. Об'єктивно: стан важкий, непритомність, адина-мія. Шкірні покриви сухі, запалі очі, ціаноз облич-чя, тахікардія, запах ацетону з рота. Результати аналізів: глюкоза крові – 20,1 ммоль/л, у сечі – 3,5%. Який найбільш вірогідний діагноз?

A. @Гіперглікемічна кома

B. Анафілактичний шок

C. Гостра серцева недостатність

D. Гіпоглікемічна кома

E. Гостре алкогольне отруєння

613. У хворого на гострий міокардит з'явилися клінічні ознаки кардіогенного шоку. Який із вказаних ни-жче патогенетичних механізмів є провідним в роз-витку шоку?

A. @Зниження насосної функції серця

B. Збільшення периферичного опору судин

C. Зниження судинного тонусу

D. Зниження діастолічного притоку до серця

E. Депонування крові в органах

 

У трирiчної дитини з пiдвищеною температурою тiла пiсля прийому аспiрину спостерiгається посилений гемолiз еритроцитiв. Вроджена недостатнiсть якого ферменту могла викликати у дитини гемолiтичну анемiю?

+Глюкозо-6-фосфатдегiдрогеназа

-Глюкозо-6-фосфатаза

-Глiкогенфосфорилаза

-Глiцеролфосфатдегiдрогеназа

-γ-глутамiлтрансфераза

У хворого пiсля оперативного видалення кiсти пiдшлункової залози виник геморагiчний синдром з вираженим порушенням зсiдання кровi. Розвиток цього ускладнення пояснюється:

A. Активацiєю фiбринолiтичної системи

B. Недостатнiм утворенням фiбрину

C. Зменшенням кiлькостi тромбоцитiв

D. Активацiєю протизгортальної системи

E. Активацiєю фактору Крисмаса

У хворого 37-ми рокiв на фонi тривалого застосування антибiотикiв спостерiгається пiдвищена кровоточивiсть при невеликих пошкодженнях. У кровi зниження активностi факторiв згортання кровi II, VII, IX, X, подовження часу згортання кровi. Недостатнiстю якого вiтамiну обумовленi зазначенi змiни?

A. Вiтамiн K

B. Вiтамiн C

C. Вiтамiн D

D. Вiтамiн E

E. Вiтамiн B6

У хлопчика 3-х рокiв з вираженим геморагiчним синдромом вiдсутнiй антигемофiльний глобулiн А (фактор VIII) у плазмi кровi. Яка фаза гемостазу первинно порушена у цього хворого?

A. Внутрiшнiй механiзм активацiї протромбiнази

B. Зовнiшнiй механiзм активацiї протромбiнази

C. Перетворення протромбiну в тромбiн

D. Перетворення фiбриногену в фiбрин

E. Ретракцiя кров’яного згустку

У хворого опiкова хвороба ускладнилася ДВЗ-синдромом. Яку стадiю ДВЗ-синдрому можна запiдозрити, якщо вiдомо, що кров хворого згортається менше нiж за 3 хвилини?

A. Гiперкоагуляцiї

B. Перехiдна

C. Гiпокоагуляцiї

D. Фiбринолiз

E. Термiнальна

У хворого iз захворюванням печiнки виявлено зниження вмiсту протромбiну в кровi. Це призведе, перш за все, до порушення:

A. Другої фази коагуляцiйного гемостазу

B. Першої фази коагуляцiйного гемостазу

C. Судинно-тромбоцитарного гемостазу

D. Фiбринолiзу

E. Антикоагулянтних властивостей кровi

Видалення зуба у пацiєнта з хронiчним персистуючим гепатитом ускладнилось тривалою кровотечею. Яка причина геморагiчного синдрому?

A. Зменшення утворення тромбiну

B. Збiльшення утворення тромбопластину

C. Зменшення утворення фiбрину

D. Збiльшення синтезу фiбриногену

E. Посилення фiбринолiзу

У лiквiдатора наслiдкiв аварiї на АЕС пiд час перебiгу гострої променевої хвороби виник геморагiчний синдром. Що має найбiльше значення в патогенезi цього синдрому?

A. Тромбоцитопенiя

B. Порушення структури стiнки судин

C. Пiдвищення активностi факторiв фiбринолiзу

D. Пiдвищення активностi факторiв систем протизсiдання кровi

E. Зменшення активностi факторiв зсiдання кровi

У людини до травми гематокритний показник 40%. Яким вiн буде через добу пiсля втрати 750 мл кровi?

A. 0.3

B. 0.4

C. 0.55

D. 0.45

E. 0.5

У чоловiка 30-ти рокiв перед операцiєю визначили групову належнiсть кровi. Кров резус-позитивна. Реакцiю аглютинацiї еритроцитiв не викликали стандартнi сироватки груп 0 αβ (I), Аβ (II), Вα (III). Дослiджувана кров належить до групи:

A. 0 αβ (I)

B. Аβ (II)

C. Вα (III)

D. АВ (IV)

E. -

У чоловiка 30-ти рокiв перед операцiєю визначили групову належнiсть кровi. Кров резус-позитивна. Реакцiю аглютинацiї еритроцитiв не викликали стандартнi сироватки груп 0αβ (I), Аβ (II), Вα (III). Дослiджувана кров належить до групи:

A. 0αβ (I)

B. Аβ (II)

C. Вα (III)

D. АВ (IV)

E. -

На останньому мiсяцi вагiтностi вмiст фiбриногену в плазмi кровi в 2 рази вище за норму. Яку швидкiсть осiдання еритроцитiв слiд при цьому очiкувати?

A. 40-50 мм/годину

B. 0-5 мм/годину

C. 10-15 мм/годину

D. 5-10 мм/годину

E. 3-12 мм/годину

У вагiтної жiнки визначили групу кровi. Реакцiя аглютинацiї еритроцитiв вiдбулася зi стандартними сироватками груп 0αβ (I), Bα (III) i не вiдбулася зi стандартною сироваткою групи Aβ (II). Дослiджувана кров належить до групи:

A. Aβ (II)

B. 0αβ (I)

C. Bα (III)

D. AB (IV )

E. -

Встановлено, що аглютинацiя еритроцитiв кровi реципiєнта викликали стандартнi сироватки I та II груп i не викликали - сироватка III групи I антирезусна сироватка. Кров якої групи за системами AB0 i резус можна переливати реципiєнту?

A. B, α (III) Rh−

B. A, β (II) Rh−

C. 0, α, β, (I) Rh+

D. AB (IV ), Rh+

E. AB (IV ), Rh−

Чоловiк 56-ти рокiв потрапив до клiнiки зi скаргами на загальну слабкiсть, бiль i печiння в язицi, вiдчуття онiмiння в кiнцiвках. У минулому перенiс резекцiю кардiального вiддiлу шлунка. У кровi: Hb- 80 г/л; ер.- 2, 0 • 10 12 /л; КП1,2, лейк.- 3, 5 • 10 9 /л. Який вид анемiї у цього хворого?

A. B12 -фолiєводефiцитна

B. Гемолiтична

C. Постгеморагiчна

D. Апластична

E. Залiзодефiцитна

Дiвчинка 10-ти рокiв часто хворiє на гострi респiраторнi iнфекцiї, пiсля яких спостерiгаються множиннi точковi крововиливи в мiсцях тертя одягу. Який гiповiтамiноз має мiсце в дiвчинки:

A. C

B. B6

C. B1

D. A

E. B2

Пацiєнт страждає на геморагiчний синдром, що проявляється частими носовими кровотечами, посттравматичними та спонтанними внутрiшньошкiрними та внутрiшньосуглобовими крововиливами. Пiсля лабораторного обстеження було дiагностовано гемофiлiю В. Дефiцит якого фактора згортання кровi обумовлює дане захворювання?

A. IX

B. VIII

C. XI

D. V

E. VII

У хворого спостерiгаються геморагiї, в кровi знижена концентрацiя протромбiну. Недостатнiсть якого вiтамiну призвела до порушення синтезу цього фактору згортання кровi?

A. K

B. A

C. D

D. C

E. E

У хворих з непрохiднiстю жовчовивiдних шляхiв пригнiчується зсiдання кровi, виникають кровотечi, що є наслiдком недостатнього засвоєння такого вiтамiну:

A. K

B. A

C. D

D. E

E. C

У вагiтної жiнки визначили групову належнiсть кровi. Реакцiю аглютинацiї еритроцитiв викликали стандартнi сироватки I, III груп, та не викликала сироватка II групи. Якою є група дослiджуваної кровi за системою АВ0?

A. А(II), β

B. В(III), α

C. О(I), α, β

D. АВ (IV)

E. -

Пiсля прийому сульфанiламiдiв у хворого виникли лихоманка, блювання i стул з кров’ю. У кровi: лейк. 0, 9•109/л (гранул. - 0,7•109/л), лейкоаглютинiни. Який з термiнiв найбiльш точно характеризує виявленi змiни у кровi?

A. Агранулоцитоз

B. Лейкопенiя

C. Лейкоз

D. Гемодилюцiя

E. -

У хворого, що перенiс 5 рокiв тому субтотальну резекцiю шлунка, розвинулась B12-фолiєводефiцитна анемiя. Який механiзм є провiдним у розвитку такої анемiї?

A. Вiдсутнiсть внутрiшнього фактора Касла

B. Вiдсутнiсть зовнiшнього фактора Касла

C. Порушення всмоктування вiтамiну B12 в тонкiй кишцi

D. Дефiцит фолiєвої кислоти

E. Дефiцит транскобаламiну

У хворої 36-ти рокiв, яка лiкувалася сульфанiламiдами з приводу респiраторної вiрусної iнфекцiї, в кровi гiпорегенераторна нормохромна анемiя, лейкопенiя, тромбоцитопенiя. В кiстковому мозку - зменшення кiлькостi мiєлокарiоцитiв. Яка це анемiя?

A. Гiпопластична

B. Гемолiтична

C. Постгеморагiчна

D. B12-фолiєводефiцитна

E. Залiзодефiцитна

Пiсля оперативного видалення частини шлунка у хворого порушилося всмоктування вiтамiну B 12 , вiн виводиться з калом. Розвинулася анемiя. Який фактор необхiдний для всмоктування цього вiтамiну?

A. Гастромукопротеїн

B. Гастрин

C. Соляна кислота

D. Пепсин

E. Фолiєва кислота

У районах Пiвденної Африки у людей розповсюджена серпоподiбноклiтинна анемiя, при якiй еритроцити мають форму серпа внаслiдок замiни в молекулi гемоглобiну амiнокислоти глутамiну на валiн. Чим викликана ця хвороба?

A. Генна мутацiя

B. Порушення механiзмiв реалiзацiї генетичної iнформацiї

C. Кросинговер

D. Геномнi мутацiї

E. Трансдукцiя

У хворого на мiкросфероцитарну гемолiтичну анемiю (хворобу Мiнковського-Шоффара), внаслiдок пiдвищення проникливостi мембрани еритроцитiв, у клiтину надходять iони натрiю та вода. Еритроцити набувають форму сфероцитiв i легко руйнуються. Який провiдний механiзм пошкодження еритроцитiв має мiсце в даному випадку?

A. Електролiтно-осмотичний

B. Кальцiєвий

C. Ацидотичний

D. Протеїновий

E. Нуклеїновий

Дитина 6-ти рокiв знаходиться на стацiонарному лiкуваннi з дiагнозом алергiчного ринiту. В кровi: змiни в лейкоцитарнiй формулi. Кiлькiсть яких клiтин лейкоцитарного ряду може бути збiльшена?

A. Еозинофiли

B. Т-лiмфоцити

C. В-лiмфоцити

D. Базофiли

E. Нейтрофiли

При аналiзi кровi у спортсмена виявлено: ер.- 5, 5 • 10 12 /л, Hb- 180 г/л, лейк.- 7 • 10 9 /л, н.- 64%, б.- 0,5%, е.- 0,5%, м.- 8%, л.- 27%. Такi показники свiдчать про стимуляцiю, перш за все:

A. Еритропоезу

B. Лейкопоезу

C. Лiмфопоезу

D. Гранулоцитопоезу

E. Iмуногенезу

В пробiрку, що мiстить розчин NaCl 0,9%, додана крапля кровi. Що вiдбудеться з еритроцитами?

A. Залишаться без змiн

B. Осмотичний гемолiз

C. Бiологiчний гемолiз

D. Зморшкування

E. Набухання

До приймального вiддiлення доставлено жiнку 38-ми рокiв з матковою кровотечею, що триває другу добу. Що з наведеного буде виявлено при аналiзi кровi хворої?

A. Зменшення гематокритного показника

B. Еозинофiлiя

C. Уповiльнення ШОЕ

D. Лейкоцитоз

E. Збiльшення колiрного показника

У приймально-дiагностичне вiддiлення доставили жiнку 38-ми рокiв з шлунковою кровотечею. Якi змiни найбiльш iмовiрнi з боку кровi через добу?

A. Зменшення гематокритного числа

B. Лейкоцитоз

C. Еритроцитоз

D. Лейкопенiя

E. Збiльшення гематокритного числа

У чоловiка 43-х рокiв з видаленою ниркою були виявленi симптоми анемiї. Що зумовило появу цих симптомiв?

A. Зниження синтезу еритропоетинiв

B. Пiдвищене руйнування еритроцитiв

C. Нестача залiза

D. Нестача вiтамiну B12

E. Нестача фолiєвої кислоти

У хворого в лейкограмi: лейкоцити - 14 • 10 9 /л; мiєлобласти - 71%, промiєлоцити, мiєлоцити, метамiєлоцити - 0%, паличкоядернi нейтрофiли - 6%, сегментоядернi - 13%; лiмфоцити - 7%, моноцити - 3%. Яка патологiя у хворого?

A. Мiєлобластний лейкоз

B. Нейтрофiльний лейкоцитоз

C. Хронiчний мiєлолейкоз

D. Лiмфобластний лейкоз

E. Хронiчний лiмфолейкоз

У хворого в обох щелепах рентгенологiчно виявлено численнi дефекти у виглядi гладкостiнних округлих отворiв. При гiстологiчному дослiдженнi - явища остеолiзису i остеопорозу при явищах слабкого кiсткоутворення. В сечi хворого знайдено бiлок БенсДжонса. Назвiть захворювання:

A. Мiєломна хвороба

B. Хронiчний мiєлолейкоз

C. Хронiчний еритромiєлоз

D. Гострий мiєлолейкоз

E. Гострий недиференцiйований лейкоз

Батьки для профiлактики кишкових iнфекцiй у дитини 3-х рокiв тривало застосовували антибiотики. Через мiсяць стан дитини погiршився. У кровi виражена лейкопенiя i гранулоцитопенiя. Який найбiльш вiрогiдний механiзм виявлених змiн у кровi?

A. Мiєлотоксичний

B. Аутоiмунний

C. Перерозподiльний

D. Вiковий

E. Гемолiтичний

Лiкар при дослiдженнi мазку кровi у пацiєнта з анемiєю встановив дiагноз - спадкова гемолiтична анемiя Мiнковського-Шофара. Виявлення у кровi яких характерних клiтин надало можливiсть лiкарю встановити дiагноз?

A. Мiкросфероцити

B. Полiхроматофiли

C. Мегалоцити

D. Пойкiлоцити

E. Анiзоцити

Хворий 2 роки тому перенiс операцiю резекцiї пiлоричного вiддiлу шлунка. Спостерiгається слабкiсть, перiодична поява темних кiл пiд очима, задишка. У кровi: Hb- 70 г/л, ер.- 3, 0 • 1012 /л, КП- 0,7. Якi змiни еритроцитiв у мазках кровi найбiльш характернi для даного стану?

A. Мiкроцити

B. Мегалоцити

C. Шизоцити

D. Овалоцити

E. Макроцити

У результатi радiацiйного випромiнювання ушкодженi стовбуровi гемопоетичнi клiтини. Утворення яких клiтин сполучної тканини буде порушено?

A. Макрофаги

B. Фiбробласти

C. Адiпоцити

D. Меланоцити

E. Перiцити

У чоловiка 50-ти рокiв при обстеженнi було виявлено зниження кiлькостi еритроцитiв у кровi та пiдвищення рiвня вiльного гемоглобiну в плазмi кровi (гемоглобiнемiя). КП становив 0,85. Який вид анемiї спостерiгається у хворого?

A. Набута гемолiтична

B. Спадкова гемолiтична

C. Гостра постгеморагiчна

D. Хронiчна постгеморагiчна

E. Анемiя внаслiдок порушення еритропоезу

Ураження хворого одноразовою дозою iонiзуючого випромiнювання спричинило розвиток кiстковомозкової форми променевої хвороби. Якi патологiчнi прояви з боку кровi будуть характерними в перiод удаваного благополуччя?

A. Наростаюча лiмфопенiя, лейкопенiя

B. Перерозподiльчий лейкоцитоз, лiмфоцитоз

C. Анемiя, лейкопенiя

D. Тромбоцитопенiя, анемiя

E. Тромбоцитопенiя, лейкоцитоз

Хворий 23-х рокiв скаржиться на слабкiсть, пiдвищення температури до 38 − 40℃. Об’єктивно: печiнка i селезiнка збiльшенi. У кровi: Hb- 100 г/л, ер.- 2, 9 • 1012 /л, лейк.- 4, 4 • 109/л, тромб.48 • 109/л, нейтрофiли сегментоядернi 17%, лiмфоцити - 15%, бластнi клiтини - 68%. Всi цитохiмiчнi реакцiї негативнi. Дайте гематологiчний висновок:

A. Недиференцiйований лейкоз

B. Хронiчний мiєлолейкоз

C. Гострий мiєлобластний лейкоз

D. Гострий лiмфобластний лейкоз

E. Гострий еритромiєлоз

У хворого, що надiйшов до хiрургiчного вiддiлення з ознаками гострого апендициту, виявленi наступнi змiни бiлої кровi: загальна кiлькiсть лейкоцитiв - 16 • 10 9 /л. Лейкоцитарна формула: б.- 0, е.- 2%, ю.- 2%, п.- 8%, с.- 59%, л.25%, м.- 4%. Як класифiкуються зазначенi змiни?

A. Нейтрофiлiя з регенеративним зсувом влiво

B. Нейтрофiлiя з зсувом вправо

C. Нейтрофiлiя з дегенеративним зсувом влiво

D. Лейкемоїдна реакцiя за нейтрофiльним типом

E. Нейтрофiлiя з гiперрегенеративним зсувом влiво

Пiд час роботи щодо лiквiдацiї наслiдкiв аварiї на АЕС, робiтник одержав дозу опромiнення 500 рентген. Скаржиться на головний бiль, нудоту, запаморочення. Якi змiни кiлькостi лейкоцитiв можна очiкувати в хворого через 10 годин пiсля опромiнення?

A. Нейтрофiльний лейкоцитоз

B. Лiмфоцитоз

C. Лейкопенiя

D. Агранулоцитоз

E. Лейкемiя

Жiнка 62-х рокiв скаржиться на частий бiль у дiлянцi грудної клiтки та хребта, переломи ребер. Лiкар припустив мiєломну хворобу (плазмоцитому). Який з перерахованих нижче лабораторних показникiв буде мати найбiльше дiагностичне значення?

A. Парапротеїнемiя

B. Гiперальбумiнемiя

C. Протеїнурiя

D. Гiпоглобулiнемiя

E. Гiпопротеїнемiя

У студента через 2 години пiсля iспиту в аналiзi кровi виявлено лейкоцитоз без iстотних змiн у лейкоцитарнiй формулi. Який найбiльш вiрогiдний механiзм розвитку лейкоцитозу?

A. Перерозподiл лейкоцитiв в органiзмi

B. Посилення лейкопоезу

C. Уповiльнення руйнування лейкоцитiв

D. Уповiльнення мiграцiї лейкоцитiв у тканини

E. Посилення лейкопоезу та зменшення руйнування лейкоцитiв

Чоловiк 26-ти рокiв перебуває в торпiднiй стадiї шоку внаслiдок автомобiльної аварiї. В кровi: лейк.- 3, 2 10 9 /л. Який головний механiзм в розвитку лейкопенiї?

A. Перерозподiл лейкоцитiв у судинному руслi

B. Пригнiчення лейкопоезу

C. Порушення виходу зрiлих лейкоцитiв з кiсткового мозку в кров

D. Руйнування лейкоцитiв у кровотворних органах

E. Пiдвищення видiлення лейкоцитiв з органiзму

З метою оцiнки адаптацiї до фiзичного навантаження лiкар провiв обстеження робiтникiв пiсля виконання важкої працi. Якi змiни в загальному аналiзi кровi можна виявити?

A. Перерозподiльчий лейкоцитоз

B. Лейкопенiя

C. Анемiя

D. Гiпоальбумiнемiя

E. Зсув лейкоцитарної формули влiво

У вагiтної жiнки 24-х рокiв пiсля тривалого блювання було зареєстровано зниження об’єму циркулюючої кровi. Про яку змiну загальної кiлькостi кровi може йти мова?

A. Полiцитемiчна гiповолемiя

B. Проста гiповолемiя

C. Олiгоцитемiчна гiповолемiя

D. Полiцитемiчна гiперволемiя

E. Олiгоцитемiчна гiперволемiя

У хворого через добу пiсля апендектомiї при аналiзi кровi виявили нейтрофiльний лейкоцитоз з регенеративним зсувом влiво. Який найбiльш iмовiрний механiзм розвитку абсолютного лейкоцитозу у периферичнiй кровi хворого?

A. Посилення лейкопоезу

B. Перерозподiл лейкоцитiв в органiзмi

C. Зменшення руйнування лейкоцитiв

D. Уповiльнення емiграцiї лейкоцитiв у тканинi

E. Активацiя iмунiтету

У пацiєнта встановлено гiповiтамiноз фолiєвої кислоти, що може призвести до порушення синтезу:

A. Пуринових та тимiдилових нуклеотидiв

B. Пуринових нуклеотидiв та холестерину

C. Тимiдилових нуклеотидiв та жирних кислот

D. Гема та креатину

E. Цитрату та кетонових тiл

У хворого скарги на загальну слабкiсть, пiдвищену втому, зниження апетиту i маси тiла. В анамнезi частi пневмонiї. На пiдставi клiнiчних даних та результатiв дослiдження периферiйної кровi у нього дiагностовано хронiчний лiмфолейкоз. Якi дегенеративнi змiни лейкоцитiв характернi для даного захворювання?

A. Тiнi Боткiна-Гумпрехта

B. Тiльця Князьковi-Деле

C. Зерна Амато

D. Палички Ауера

E. Токсична зернистiсть

У хворого в анамнезi: з дитинства вiдмiчався знижений рiвень гемоглобiну. Лiкування препаратами залiза не дає ефекту. У кровi: ер.- 3, 1 • 10 12 /л, ретик.16%, Hb- 85 г/л, КП- 0,75; в мазку кровi анiзоцити, пойкiлоцити, мiшенеподiбнi еритроцити, еритроцити з базофiльною зернистiстю, рiвень залiза у сироватцi 30 мкмоль/л. Для якої патологiї системи кровi характернi такi данi?

A. Таласемiя

B. Залiзодефiцитна анемiя

C. B 12 -дефiцитна анемiя

D. Фолiєводефiцитна анемiя

E. Гiпопластична анемiя

У хворої 19-ти рокiв з дитинства вiдмiчалося зниження гемоглобiну до 90-95 г/л. У кровi пiд час госпiталiзацiї: ер.- 3, 2 • 1012/л, Hb- 85 г/л, КП- 0,78; лейк.- 5, 6 • 109/л, тромб.- 210 • 109/л. В мазку: анiзоцитоз, пойкiлоцитоз, мiшенеподiбнi еритроцити. Ретикулоцити 6%. Лiкування препаратами залiза було неефективне. Яку патологiю системи кровi можна запiдозрити в даному випадку?

A. Таласемiя

B. Ферментопатiя

C. Мембранопатiя

D. Серпоподiбноклiтинна анемiя

E. Фавiзм

До клiнiки поступив чоловiк 40-ка рокiв, якого укусила гадюка. Де переважно буде проходити гемолiз еритроцитiв у цьому випадку?

A. У кровоносному руслi

B. У клiтинах печiнки

C. У клiтинах селезiнки

D. У кiстковому мозку

E. У паренхiмi нирок

У недоношеного немовляти спостерiгається жовтяниця. З нестачею у нього якого ферменту це пов’язано?

A. УДФ-трансглюкуронiдаза

B. Лужна фосфатаза

C. Кисла фосфатаза

D. Каталаза

E. НАД+ - дегiдрогеназа

У кровi чоловiка 26-ти рокiв виявлено 18% еритроцитiв сферичної, сплощеної, кулястої та остистої форм. Iншi еритроцити були у формi двоввiгнутих дискiв. Як називається таке явище?

A. Фiзiологiчний пойкiлоцитоз

B. Патологiчний пойкiлоцитоз

C. Фiзiологiчний анiзоцитоз

D. Патологiчний анiзоцитоз

E. Еритроцитоз

Через рiк пiсля субтотальної резекцiї шлунка з приводу виразки малої кривизни виявленi змiни в аналiзi кровi - анемiя, лейко- i тромбоцитопенiя, КП-1,3, наявнiсть мегалобластiв та мегалоцитiв. Дефiцит якого фактору обумовив розвиток цiєї анемiї?

A. Фактор Касла

B. Хлороводнева кислота

C. Муцин

D. Пепсин

E. Гастрин

У чоловiка 40-ка рокiв внаслiдок посиленого гемолiзу еритроцитiв пiдвищився вмiст залiза в плазмi кровi. Який бiлок забезпечує його депонування в тканинах?

A. Феритин

B. Гаптоглобiн

C. Трансферин

D. Транскортин

E. Альбумiн

У синтезi пуринових нуклеотидiв беруть участь деякi амiнокислоти, похiднi вiтамiнiв, фосфорнi ефiри рибози. Коферментна форма якого вiтамiну є переносником одновуглецевих фрагментiв в цьому синтезi?

A. Фолiєва кислота

B. Пантотенова кислота

C. Нiкотинова кислота

D. Рибофлавiн

E. Пiридоксин

Жiнка 55-ти рокiв звернулася зi скаргами на тривалi циклiчнi матковi кровотечi протягом року, слабкiсть, запаморочення. Об’єктивно: блiдiсть шкiри. У кровi: Hb- 70 г/л, ер.- 3, 2 1012/л, КП- 0,6, лейк.- 6, 0 • 109/л, ретикулоцити - 1%; гiпохромiя еритроцитiв. Яка анемiя у хворої?

A. Хронiчна постгеморагiчна

B. Гемолiтична

C. Апластична

D. B12 -фолiєводефiцитна

E. Залiзодефiцитна

При обстеженнi в аналiзi кровi пацiєнта виявлено лейкоцитоз, лiмфоцитоз, клiтини Боткiна-Гумпрехта на тлi анемiї. Про яку хворобу слiд думати лiкарю?

A. Хронiчний лiмфолейкоз

B. Гострий мiєлолейкоз

C. Лiмфогранулематоз

D. Iнфекцiйний мононуклеоз

E. Мiєломна хвороба

При розтинi тiла померлого виявлена гiперплазiя кiсткового мозку плоских i трубчастих кiсток (пiоїдний кiстковий мозок), спленомегалiя (6 кг), гепатомегалiя (5 кг), збiльшення всiх груп лiмфатичних вузлiв. Якому захворюванню вiдповiдають виявленi змiни?

A. Хронiчний мiєлолейкоз

B. Хронiчний лiмфолейкоз

C. Мiєломна хвороба

D. Справжня полiцитемiя

E. Лiмфогрануломатоз

У хворого в кровi: ер.- 3, 0 • 10 12 /л; Hb90г/л; ретикул.- 0,5%. В мазку: пойкiлоцити, гiпохромнi еритроцити. Залiзо сироватки кровi - 80 мкмоль/л. Для якої патологiї це характерно?

A. Залiзорефрактерна анемiя

B. Хвороба Мiнковського-Шоффара

C. Залiзодефiцитна анемiя

D. B 12 -дефiцитна анемiя

E. Серпоподiбноклiтинна анемiя

Чоловiк 23-х рокiв пiсля ДТП надiйшов до лiкарнi у важкому станi iз черепно-мозковою травмою. Дихання характеризується судомним тривалим вдихом, який переривається коротким видихом. Для якого типу дихання це характерно?

A. Апнейстичне

B. Гаспiнг-дихання

C. Кусмауля

D. Чейн-Стокса

E. Бiота

До приймального вiддiлення лiкарнi надiйшов непритомний юнак з ознаками отруєння морфiном. Вiдзначається поверхневе та рiдке дихання, яке обумовлене пригнiченням дихального центру. Який тип недостатностi дихання виник при цьому?

A. Вентиляцiйна дисрегуляторна

B. Вентиляцiйна обструктивна

C. Вентиляцiйна рестриктивна

D. Перфузiйна

E. Дифузiйна

У групи альпiнiстiв на висотi 3000 метрiв було зроблено аналiз кровi. Виявлене зниження HCO 3 до 15 ммоль/л (норма 22-26 ммоль/л). Який механiзм зниження HCO 3 кровi?

A. Гiпервентиляцiя

B. Посилення ацидогенезу

C. Гiповентиляцiя

D. Зниження амонiогенезу

E. Зниження реабсорбцiї бiкарбонатiв у нирках

У хворого внаслiдок хронiчного обструктивного бронхiту на тлi задишки, тахiкардiї та цiанозу пiд час дослiдження газового складу кровi виявлено розвиток гiпоксемiї та гiперкапнiї. Яке порушення зовнiшнього дихання спостерiгається у хворого?

A. Гiповентиляцiя

B. Гiпоперфузiя

C. Гiперперфузiя

D. Гiпердифузiя

E. Гiпервентиляцiя

У людини в результатi патологiчного процесу збiльшена товщина гематоальвеолярного бар’єру. Безпосереднiм наслiдком цього буде зменшення:

A. Дифузiйної здатностi легень

B. Резервного об’єму видиху

C. Альвеолярної вентиляцiї легень

D. Кисневої ємностi кровi

E. Хвилинного об’єму дихання

Лiкар записав в iсторiї хвороби, що у хворого дихання поверхневе (знижена глибина дихання). Це означає, що зменшеним є такий показник зовнiшнього дихання:

A. Дихальний об’єм

B. Життєва ємнiсть легень

C. Функцiональна залишкова ємнiсть

D. Ємнiсть вдиху

E. Хвилинний об’єм дихання

У людини збiльшена вентиляцiя легень внаслiдок фiзичного навантаження. Який з наведених показникiв зовнiшнього дихання у неї значно бiльший, нiж у станi спокою?

A. Дихальний об’єм

B. Життєва ємнiсть легень

C. Резервний об’єм вдиху

D. Резервний об’єм видиху

E. Загальна ємнiсть легень

Пiд час обiду дитина поперхнулася i аспiрувала їжу. Почався сильний кашель, шкiра i слизовi цiанотичнi, пульс частий, дихання рiдке, видих подовжений. Яке порушення зовнiшнього дихання розвинулося у дитини?

A. Експiраторна задишка при асфiксiї

B. Iнспiраторна задишка при асфiксiї

C. Стенотичне дихання

D. Альтернуюче дихання

E. Дихання Бiота

При дослiдженнi зовнiшнього дихання лiкар попросив пацiєнта здiйснити максимально глибокий видих пiсля максимально глибокого вдиху для визначення такого показника:

A. Життєва ємнiсть легень

B. Загальна ємнiсть легень

C. Функцiональна залишкова ємнiсть

D. Резервний об’єм видиху

E. Киснева ємнiсть кровi

Людина зробила максимально глибокий видих. Як називається об’єм повiтря, що знаходиться в її легенях пiсля цього?

A. Залишковий об’єм

B. Функцiональна залишкова ємнiсть легень

C. Ємнiсть вдиху

D. Резервний об’єм видиху

E. Альвеолярний об’єм

Чоловiк 50-ти рокiв хворiє на хронiчний бронхiт, скаржиться на задишку пiд час фiзичного навантаження, постiйний кашель з вiдходженням харкотиння. При обстеженнi дiагностовано ускладнення - емфiзема легень. Чим вона зумовлена?

A. Зниження еластичних властивостей легень

B. Зменшення альвеолярної вентиляцiї

C. Зменшення розтяжностi легень

D. Зменшення перфузiї легень

E. Порушення вентиляцiйноперфузiйного спiввiдношення в легенях

Чоловiк 38-ми рокiв загинув при спробi пiдйому вантажу. Розвинувся колаптоїдний стан. На аутопсiї виявлений розрив обширної аневризми грудного вiддiлу аорти. За життя страждав на вiсцеральний сифiлiс. Який патологiчний процес в даному випадку обумовив зменшення мiцностi стiнки аорти, її розширення i розрив?

A. Зникнення еластичних волокон

B. Зникнення колагенових волокон

C. Атрофiя м’язового шару

D. Змiни iнтими за типом "шагреневої шкiри"

E. Новоутворення судин

У результатi поранення у чоловiка 35-ти рокiв настав повний розрив спинного мозку на рiвнi першого шийного сегменту. Як змiниться характер дихання?

A. Зупиниться

B. Не змiниться

C. Збережеться дiафрагмальне, щезне грудне

D. Збережеться грудне, щезне дiафрагмальне

E. Стане рiдким та глибоким

Анатомiчний мертвий простiр – це частина повiтря, яка залишається в повiтроносних шляхах пiсля видиху. В якiй iз наведених нижче ситуацiй вiдбудеться зменшення анатомiчного мертвого простору?

A. Накладання трахеостоми

B. Нахил голови вперед

C. Поворот лежачого пацiєнта на лiвий бiк

D. Поворот лежачого пацiєнта на правий бiк

E. Дихання через рот

У шахтаря виявлено фiброз легень, що супроводжувався порушенням альвеолярної вентиляцiї. Який механiзм виникнення цього порушення є провiдним?

A. Обмеження дихальної поверхнi легень

B. Звуження верхнiх дихальних шляхiв

C. Порушення нервової регуляцiї дихання

D. Обмеження рухомостi грудної клiтки

E. Спазм бронхiв

У пiдлiтка 12-ти рокiв, який хворiє на бронхiальну астму, виник тяжкий напад астми: виражена експiраторна задишка, блiдiсть шкiрних покривiв. Який вид порушення альвеолярної вентиляцiї має мiсце?

A. Обструктивний

B. Рестриктивний

C. Торако-дiафрагмальний

D. Центральний

E. Нервово-м’язовий

У людини пiсля довiльної тривалої затримки дихання збiльшилися частота й глибина дихання. Якi змiни у складi кровi стали причиною цього?

A. Пiдвищення pCO2

B. Зниження pCO2

C. Зниження pO2

D. Пiдвищення pO2

E. Пiдвищення pH

Щуру в плевральну порожнину введено 0,5 мл повiтря. Який тип недостатностi дихання виникає в даному випадку?

A. Рестриктивне порушення альвеолярної вентиляцiї

B. Обструктивне порушення альвеолярної вентиляцiї

C. Перфузiйний

D. Дифузiйний

E. Дисрегуляторне порушення альвеолярної вентиляцiї

У хворого iз вираженим пневмосклерозом пiсля перенесеного iнфiльтративного туберкульозу легень розвинулась дихальна недостатнiсть. До якого патогенетичного типу вона вiдноситься?

A. Рестриктивний

B. Обструктивний

C. Дисрегуляцiйний

D. Рефлекторний

E. Апнеїстичний

В експериментi на тваринi здiйснили перерiзку блукаючих нервiв з обох бокiв. Як при цьому змiниться характер дихання?

A. Стане глибоким i рiдким

B. Стане поверхневим та частим

C. Стане глибоким i частим

D. Стане поверхневим та рiдким

E. Дихання не змiниться

Внаслiдок дiї на органiзм електричного струму мiської електромережi впродовж 0,1 сек у напрямку "права рука-голова"у постраждалого спостерiгалась зупинка дихання. Вкажiть найбiльш iмовiрний механiзм цього ускладнення:

A. Тотальний паралiч дихального центру

B. Рефлекторна зупинка дихання (больовий шок)

C. Паралiч дихальних м’язiв

D. Емоцiйний стрес

E. Паралiч центрiв вдиху

Людина зробила спокiйних видих. Як називається об’єм повiтря, який мiститься у неї в легенях при цьому?

A. Функцiональна залишкова ємнiсть легень

B. Залишковий об’єм

C. Резервний об’єм видиху

D. Дихальний об’єм

E. Життєва ємнiсть легень

У неврологiчне вiддiлення з приводу мозкового крововиливу поступив хворий 62-х рокiв. Об’єктивно: стан важкий. Спостерiгається наростання глибини i частоти дихання, а потiм його зменшення до апное, пiсля чого цикл дихальних рухiв вiдновлюється. Який тип дихання у хворого?

A. Чейна-Стокса

B. Кусмауля

C. Бiота

D. Гаспiнг-дихання

E. Апнейстичне

При обстеженнi людини встановлено, що хвилинний об’єм серця дорiвнює 3500 мл, систолiчний об’єм - 50 мл. Якою є частота серцевих скорочень за хвилину у людини?

A. 70

B. 60

C. 50

D. 80

E. 90

У людини визначили частоту серцевих скорочень за пульсом. Вона дорiвнює 120/хв. Якою при цьому є тривалiсть серцевого циклу?

A. 0,5 с

B. 0,7 с

C. 0,8 с

D. 0,9 с

E. 1,0 с

У жiнки 30-ти рокiв хвилинний об’єм кровi у станi спокою становить 5 л/хв. Який об’єм кровi проходить у неї через судини легень за 1 хвилину?

A. 5 л

B. 3,75 л

C. 2,5 л

D. 2,0 л

E. 1,5 л

Визначте пульсовий i середньодинамiчний артерiальний тиск (мм рт.ст.) у обстежуваного, якщо вимiряний у нього артерiальний тиск становить 130/70 мм рт.ст.:

A. 60, 90

B. 60, 80

C. 50, 90

D. 60, 100

E. 50, 70

Подразнення правого блукаючого нерва спричинило рiзке сповiльнення атрiовентрикулярного проведення. На ЕКГ при цьому буде подовжений:

A. Iнтервал P − Q

B. Комплекс QRST

C. Зубець T

D. Зубець P

E. Iнтервал R − R

Пiд час реєстрацiї ЕКГ хворого з гiперфункцiєю щитоподiбної залози зареєстровано збiльшення частоти серцевих скорочень. Вкорочення якого елементу ЕКГ про це свiдчить?

A. Iнтервал R − R

B. Сегмент P − Q

C. Iнтервал P − Q

D. Iнтервал P − T

E. Комплекс QRS

У хворого через 12 годин пiсля гострого нападу загруднинного болю знайдено рiзке пiдвищення активностi АсАТ у сироватцi кровi. Вкажiть патологiю, для якої характерне це змiщення:

A. Iнфаркт мiокарда

B. Вiрусний гепатит

C. Колагеноз

D. Цукровий дiабет

E. Нецукровий дiабет

Пiд час хiрургiчного втручання на тонкiй кишцi у людини можлива рефлекторна зупинка серця. Якi рецептори в мiокардi необхiдно заблокувати, щоб попередити зупинку?

A. M-холiнорецептори

B. H-холiнорецептори

C. Пуриновi рецептори

D. α-адренорецептори

E. β-адренорецептори

У результатi побутової травми у пацiєнта виникла значна крововтрата, що супроводжувалося зниженням артерiального тиску. Дiя яких гормонiв забезпечує швидке вiдновлення кров’яного тиску, викликаного крововтратою?

A. Адреналiн, вазопресин

B. Кортизол

C. Статевi

D. Окситоцин

E. Альдостерон

У хiрурга пiсля проведення тривалої операцiї пiдвищився артерiальний тиск до 140/110 мм рт.ст. Якi змiни гуморальної регуляцiї можуть бути причиною пiдвищення артерiального тиску в даному випадку?

A. Активацiя симпатоадреналової системи

B. Активацiя утворення i видiлення альдостерону

C. Активацiя ренiн-ангiотензинової системи

D. Активацiя калiкреїн-кiнiнової системи

E. Гальмування симпатоадреналової системи

У тварини через 2 тижнi пiсля експериментального звуження ниркової артерiї пiдвищився артерiальний тиск. Зi збiльшенням дiї на судини якого фактора гуморальної регуляцiї це пов’язано?

A. Ангiотензин II

B. Кортизол

C. Альдостерон

D. Вазопресин

E. Дофамiн

Жiнка 49-ти рокiв звернулася до лiкаря зi скаргами на пiдвищену втомлюванiсть та появу задишки пiд час фiзичного навантаження. На ЕКГ: ЧСС50/хв, iнтервал P Q- подовжений, комплекс QRS- не змiнений, кiлькiсть зубцiв P перевищує кiлькiсть комплексiв QRS. Який вид аритмiї у пацiєнтки?

A. Атрiовентрикулярна блокада

B. Екстрасистолiя

C. Синусова брадикардiя

D. Миготлива аритмiя

E. Синоатрiальна блокада

У хворого 45-ти рокiв при аналiзi ЕКГ встановлено: ритм синусовий, число передсердних комплексiв бiльше числа шлуночкових комплексiв; прогресуюче подовження iнтервалу P – Q вiд комплексу до комплексу; випадiння окремих шлуночкових комплексiв; зубцi P та комплекси QRST без змiн. Назвiть тип порушення серцевого ритму:

A. Атрiовентрикулярна блокада II ступеня

B. Синоаурiкулярна блокада

C. Атрiовентрикулярна блокада I ступеня

D. Внутрiшньопередсердна блокада

E. Повна атрiовентрикулярна блокада

У хворої 45-ти рокiв при електрокардiографiчному обстеженнi виявлено такi змiни: iнтервал P − Q подовжений, при цьому випадає кожен другий або третiй комплекс QRST. Яке саме порушення провiдностi серця спостерiгається?

A. Атрiовентрикулярна блокада III ступеня

B. Атрiовентрикулярна блокада повна

C. Синоаурiкулярна блокада

D. Внутрiшлуночкова блокада

E. Атрiовентрикулярна блокада I ступеня

В експериментi на ссавцi зруйнували певну структуру серця, що призвело до припинення проведення збудження вiд передсердь до шлуночкiв. Що саме зруйнували?

A. Атрiовентрикулярний вузол

B. Синоатрiальний вузол

C. Пучок Гiса

D. Нiжки пучка Гiса

E. Волокна Пуркiн’є

На iзольованому серцi вивчалась швидкiсть проведення збудження у рiзних його дiлянках. Де була виявлена найменша швидкiсть?

A. Атрiовентрикулярний вузол

B. Пучок Гiса

C. Волокна Пуркiн’є

D. Мiокард передсердь

E. Мiокард шлуночкiв

В експериментi подразнюють гiлочки блукаючого нерва, якi iннервують серце. Це призвело до того, що припинилося проведення збудження вiд передсердь до шлуночкiв. Електрофiзiологiчнi змiни в яких структурах серця є причиною цього?

A. Атрiовентрикулярний вузол

B. Пучок Гiса

C. Синоатрiальний вузол

D. Шлуночки

E. Передсердя

У людини частота серцевих скорочень постiйно утримується на рiвнi 40 разiв за хвилину. Що є водiєм ритму серця у неї?

A. Атрiовентрикулярний вузол

B. Синоатрiальний вузол

C. Пучок Гiса

D. Нiжки пучка Гiса

E. Волокна Пуркiн’є

У пацiєнта тривалiсть iнтервалу P − Q на ЕКГ перевищує норму при нормальнiй тривалостi зубця P . Причиною цього є зменшення швидкостi проведення збудження:

A. Атрiо-вентрикулярним вузлом

B. Сино-атрiальним вузлом

C. Пучком Гiса

D. Нiжками пучка Гiса

E. Волокнами Пуркiн’є

Пiд час аналiзу електрокардiограми встановлено збiльшення тривалостi i амплiтуди зубця S. Деполяризацiя якої дiлянки серця порушена у хворого?

A. Базальнi вiддiли шлуночкiв

B. Передсердя

C. Верхiвка серця

D. Боковi стiнки шлуночкiв

E. Середня i нижня третина мiжшлуночкової перегородки

Пiд час роботи лiкарю–стоматологу доводиться довго стояти на ногах, що може призвести до застою кровi у венах нижнiх кiнцiвок та їх варикозного розширення. З порушенням якого механiзму венозного припливу кровi до серця це пов’язано?

A. Вiдсутнiсть скорочення скелетних м’язiв

B. Градiєнт тиску

C. Присмоктувальний ефект грудної клiтки

D. Залишкова рушiйна сила серця

E. Присмоктувально-тисковий помповий ефект дiафрагми на органи черевної порожнини

У хворого температура протягом 5-ти днiв коливалася в межах 39,5℃ 40,2℃. На 6-й день на тлi рiзкого зниження температури до 35,2℃ розвинувся колапс. Який головний механiзм колапсу?

A. Вазодилатацiя

B. Гiпервентиляцiя

C. Посилене потовидiлення

D. Тахiкардiя

E. Полiурiя

В експериментi у тварини в результатi проведеного перетинання депресорного нерва та руйнування каротидних клубочкiв розвинулась стiйка гiпертензiя. З порушенням якої функцiї нервової системи пов’язане це явище?

A. Вегетативна

B. Вища нервова дiяльнiсть

C. Рухова

D. Сенсорна

E. Трофiчна

В експериментi подразнюють гiлочки симпатичного нерва, якi iннервують серце. Це призвело до збiльшення сили серцевих скорочень, тому що через мембрану типових кардiомiоцитiв збiльшився:

A. Вхiд iонiв кальцiю

B. Вихiд iонiв кальц iю

C. Вихiд iонiв калiю

D. Вхiд iонiв калiю

E. Вхiд iонiв кальцiю та калiю

У хворого, що страждає на важку форму порушення водно-сольового обмiну, настала зупинка серця в дiастолi. Який найбiльш вiрогiдний механiзм зупинки серця в дiастолi?

A. Гiперкалiємiя

B. Гiпернатрiємiя

C. Дегiдратацiя органiзму

D. Гiпокалiємiя

E. Гiпонатрiємiя

Обстеження пацiєнта з високим артерiальним тиском показало в нього вторинну артерiальну гiпертензiю. Причиною такого стану є ренiнпродукуюча пухлина нирки. Що є головною ланкою в патогенезi вторинної артерiальної гiпертензiї в хворого?

A. Гiперпродукцiя ангiотензину 2, альдостерону

B. Гiперпродукцiя кортизолу

C. Гiперпродукцiя iнсулiну

D. Недостатня продукцiя вазопресину

E. Недостатня продукцiя катехоламiнiв

При проходженнi профiлактичного огляду у шахтаря лiкар встановив змiни функцiонального стану серця, що свiдчать про серцеву недостатнiсть в стадiї компенсацiї. Що з нижче перерахованого є головним пiдтвердженням компенсацiї дiяльностi серця?

A. Гiпертрофiя мiокарда

B. Тахiкардiя

C. Збiльшення артерiального тиску

D. Задишка

E. Цiаноз

У чоловiка 65-ти рокiв впродовж 15-ти рокiв була виражена артерiальна гiпертензiя. Останнiм часом систолiчний тиск почав знижуватися, а дiастолiчний залишився пiдвищеним. Який гемодинамiчний тип артерiальної гiпертензiї у хворого?

A. Гiпокiнетичний

B. Нормокiнетичний

C. Гiперкiнетичний

D. Еукiнетичний

E. -

Хвора на ревматоїдний артрит пiсля трьохтижневого лiкування преднiзолоном почала скаржитись на перебої в роботi серця. З чим пов’язаний розвиток даного небажаного ефекту препарату?

A. Гiпокалiємiя

B. Гiперкалiємiя

C. Гiперурiкемiя

D. Гiперглiкемiя

E. Гiпоглiкемiя

Пiд час аналiзу ЕКГ людини з’ясовано, що у другому стандартному вiдведеннi вiд кiнцiвок зубцi P позитивнi, їхня амплiтуда 0,1 mV (норма - 0,05-0,25 mV), тривалiсть - 0,1 с (норма - 0,07-0,10 с). Вiрним є висновок, що у передсердях нормально вiдбувається процес:

A. Деполяризацiї

B. Реполяризацiї

C. Збудження

D. Скорочення

E. Розслаблення

У хворого внаслiдок травми розвинувся травматичний шок, у перебiгу якого мали мiсце наступнi порушення: АТ- 140/90 мм рт.ст., Ps- 120/хв. Хворий метушливий, багатослiвний, блiдий. Якiй стадiї шоку вiдповiдає цей стан?

A. Еректильна

B. Латентний перiод

C. Термiнальна

D. Торпiдна

E. Кiнцева

У хворого на ЕКГ виявлено збiльшення тривалостi комплексу QRS. Наслiдком чого це може бути?

A. Збiльшення часу охоплення збудженням шлуночкiв

B. Порушення провiдностi у атрiовентрикулярному вузлi

C. Збiльшення збудливостi передсердь

D. Збiльшення збудливостi шлуночкiв та передсердь

E. Збiльшення часу охоплення збудженням передсердь

При гострому iнфарктi в мiокардi виникає декiлька зон в осередку iнфаркту: зона некрозу, зона iшемiчного пошкодження i зона iшемiї. Зонi пошкодження на ЕКГ вiдповiдає:

A. Змiщення сегменту RS - T вище iзолiнiї

B. Негативний зубець T

C. Глибокий зубець Q

D. Комплекс QRS типу QS

E. Зниження зубця R

У собаки в дослiдi подразнювали на шиї периферичний вiдрiзок блукаючого нерва. При цьому спостерiгали такi змiни серцевої дiяльностi:

A. Зменшення частоти скорочень

B. Збiльшення сили скорочень

C. Збiльшення швидкостi атрiовентрикулярного проведення

D. Збiльшення частоти та сили скорочень

E. Збiльшення збудливостi мiокарда

У хворого на гострий мiокардит з’явилися клiнiчнi ознаки кардiогенного шоку. Який iз вказаних нижче патогенетичних механiзмiв є провiдним в розвитку шоку?

A. Зниження насосної функцiї серця

B. Депонування кровi в органах

C. Зниження дiастолiчного притоку до серця

D. Зниження судинного тонусу

E. Збiльшення периферичного опору судин

У тварини з недостатнiстю аортальних клапанiв розвинулась гiпертрофiя лiвого шлуночка серця. В окремих його дiлянках визначаються локальнi контрактури. Накопичення якої речовини в мiокардiоцитах обумовило контрактури?

A. Кальцiй

B. Калiй

C. Молочна кислота

D. Вуглекислий газ

E. Натрiй

У хворого з пересадженим серцем при фiзичному навантаженнi збiльшився хвилинний об’єм кровi. Який механiзм регуляцiї забезпечує цi змiни?

A. Катехоламiни

B. Симпатичнi безумовнi рефлекси

C. Парасимпатичнi безумовнi рефлекси

D. Симпатичнi умовнi рефлекси

E. Парасимпатичнi умовнi рефлекси

Жiнцi 54-х рокiв поставили попереднiй дiагноз: iнфаркт мiокарда. Характерною ознакою даного захворювання є суттєве пiдвищення в кровi активностi такого ферменту:

A. Креатинфосфокiназа

B. Каталаза

C. Г-6-ФДГ

D. Альфа-амiлаза

E. Аргiназа

Чоловiк 58-ми рокiв хворiє на атеросклероз судин головного мозку. При обстеженнi виявлена гiперлiпiдемiя. Вмiст якого класу лiпопротеїдiв у сироватцi кровi даного чоловiка найбiльш вiрогiдно буде пiдвищений?

A. Лiпопротеїди низької щiльностi

B. Лiпопротеїди високої щiльностi

C. Комплекси жирних кислот з альбумiнами

D. Хiломiкрони

E. Холестерин

У хлопчика 11-ти рокiв вмiст холестерину в сироватцi кровi до 25 ммоль/л. В анамнезi - спадкова сiмейна гiперхолестеринемiя, причиною якої є порушення синтезу бiлкiв-рецепторiв до:

A. Лiпопротеїнiв низької щiльностi

B. Лiпопротеїнiв високої щiльностi

C. Хiломiкронiв

D. Лiпопротеїнiв дуже низької щiльностi

E. Лiпопротеїнiв промiжної щiльностi

У вiддiлення реанiмацiї надiйшов чоловiк 47-ми рокiв з дiагнозом iнфаркт мiокарда. Яка з фракцiй лактатдегiдрогенази (ЛДГ) буде переважати в сироватцi кровi впродовж перших двох дiб захворювання?

A. ЛДГ1

B. ЛДГ2

C. ЛДГ3

D. ЛДГ4

E. ЛДГ5

У чоловiка 72-х рокiв довготривала хронiчна патологiя легень призвела до недостатностi клапанiв легеневої артерiї i трикуспiдального клапану, недостатностi кровообiгу за правошлуночковим типом. Який тип артерiальної гiпертензiї є причиною перевантаження серця об’ємом?

A. Легенева гiпертензiя

B. Центрально-iшемiчна гiпертензiя

C. Есенцiальна гiпертензiя

D. Рефлексогенна гiпертензiя

E. Сольова гiпертензiя

Пiд час об’єктивного обстеження хворого з дiагнозом: атеросклеротичний мiокардiосклероз, лiкар встановив феномен дефiциту пульсу. При якiй формi порушення серцевого ритму спостерiгається такий феномен?

A. Миготлива аритмiя

B. Iдiовентрикулярний ритм

C. Передсердно-шлуночковий ритм

D. Брадикардiя

E. Синусова екстрасистолiя

Пiд час тривалого бiгу у чоловiка 35-ти рокiв виникла гостра серцева недостатнiсть. Якi змiни iонного складу спостерiгаються у серцевому м’язi при цьому станi?

A. Накопичення в клiтинах мiокарда iонiв Na+ i Ca2+

B. Накопичення в клiтинах мiокарда iонiв K+ i Mg2+

C. Зменшення в клiтинах мiокарда iонiв Na+ i Ca2+

D. Зменшення в позаклiтинному просторi iонiв K+ i Mg2+

E. Збiльшення в позаклiтинному просторi iонiв Na+ i Ca2+

Пiд час диспансерного огляду у чоловiка 36-ти рокiв, водiя за професiєю, артерiальний тиск склав 150/90 мм рт.ст. Скарги на шум у вухах наприкiнцi робочого дня та загальне нездужання, якi зникають пiсля вiдпочинку. Дiагностована гiпертонiчна хвороба. Який провiдний патогенетичний механiзм у цьому випадку?

A. Нейрогенний

B. Нирковий

C. Гуморальний

D. Ендокринний

E. Рефлексогенний

У хворого 59-ти рокiв, директора пiдприємства, пiсля перевiрки податкової iнспекцiї з’явився iнтенсивний пекучий бiль, локалiзований за грудниною, який iррадiює в лiву руку. Через 15 хвилин стан хворого нормалiзувався. Який можливий механiзм стенокардiї є провiдним у цього хворого?

A. Пiдвищення у кровi рiвня катехоламiнiв

B. Атеросклероз коронарних судин

C. Внутрiшньосудинна агрегацiя формених елементiв

D. Тромбоз коронарних судин

E. Функцiональне перевантаження серця

У водiя, який потрапив у ДТП, отримав травму та знаходиться у станi шоку, спостерiгається зменшення добової кiлькостi сечi до 300 мл. Який основний патогенетичний фактор цiєї змiни дiурезу?

A. Падiння артерiального тиску

B. Зниження онкотичного тиску кровi

C. Пiдвищення проникностi судин

D. Зменшення кiлькостi функцiонуючих клубочкiв

E. Вторинний гiперальдостеронiзм

Хворого на трансмуральний iнфаркт мiокарда лiвого шлуночка переведено до вiддiлення реанiмацiї у важкому станi. АТ70/50 мм рт.ст., ЧСС- 56/хв., ЧД- 32/хв. Зазначте головну ланку в патогенезi кардiогенного шоку:

A. Падiння серцевого викиду

B. Падiння периферичного судинного опору

C. Втрата води

D. Крововтрата

E. Втрата електролiтiв

Пiд час бiйки у чоловiка виникла зупинка серця внаслiдок сильного удару у верхню дiлянку передньої черевної стiнки. Який iз зазначених механiзмiв спричинив зупинку серця?

A. Парасимпатичнi безумовнi рефлекси

B. Симпатичнi безумовнi рефлекси

C. Парасимпатичнi умовнi рефлекси

D. Симпатичнi умовнi рефлекси

E. Периферичнi рефлекси

Жiнка 25-ти рокiв скаржиться на постiйний бiль у дiлянцi серця, задишку пiд час рухiв, загальну слабкiсть. Об’єктивно: шкiра блiда та холодна, акроцiаноз. Ps- 96/хв., АТ- 105/70 мм рт.ст. Межа серця змiщена на 2 см влiво. Перший тон над верхiвкою серця послаблений, систолiчний шум над верхiвкою. Дiагностовано недостатнiсть мiтрального клапана серця. Чим обумовлене порушення кровообiгу?

A. Перевантаження мiокарда збiльшеним об’ємом кровi

B. Перевантаження мiокарда пiдвищеним опором вiдтоку кровi

C. Пошкодження мiокарда

D. Зниження об’єму циркулюючої кровi

E. Збiльшення об’єму судинного русла

У спортсмена легкоатлета (бiгуна на довгi дистанцiї) пiд час змагань розвинулась гостра серцева недостатнiсть. В результатi чого виникла ця патологiя?

A. Перевантаження серця об’ємом

B. Порушення вiнцевого кровообiгу

C. Прямого пошкодження мiокарда

D. Патологiя перикарда

E. Перевантаження серця опором

У хворого 44-х рокiв на ЕКГ виявленi ознаки гiпертрофiї обох шлуночкiв та правого передсердя. Дiагностовано недостатнiсть тристулкового клапана. Який патогенетичний варiант порушення функцiї серця має мiсце при цiй недостатностi?

A. Перевантаження серця об’ємом

B. Перевантаження серця опором

C. Первинна мiокардiальна недостатнiсть

D. Коронарна недостатнiсть

E. Тампонада серця

У хворого на ессенцiальну артерiальну гiпертензiю розвинувся гiпертонiчний криз, що призвело до нападу серцевої астми. Який механiзм серцевої недостатностi є провiдним в даному випадку?

A. Перевантаження серця пiдвищеним опором

B. Перевантаження серця збiльшеним об’ємом кровi

C. Абсолютна коронарна недостатнiсть

D. Пошкодження мiокарда

E. Порушення надходження кровi до серця

Хворий 21-го року надiйшов до стацiонару з загостренням хронiчного тонзилiту. Скаржиться на слабкiсть, задуху при помiрному фiзичному навантаженнi. Температура 37,5℃. ЧСС- 110/хв. ЕКГ: ритм синусовий, iнтервал P Q подовжений. Яка аритмiя у хворого?

A. Передсердно-шлуночкова блокада I ст.

B. Передсердно-шлуночкова блокада II ст.

C. Внутрiшньопередсердна блокада

D. Порушення внутрiшньошлуночкової провiдностi

E. Передсердно-шлуночкова екстрасистолiя

Аналiз ЕКГ хворого виявив вiдсутнiсть зубця Р. Тривалiсть та амплiтуда QRS комплексу та зубця Т вiдповiдають нормi. Що є водiєм ритму серця даного пацiєнта?

A. Передсердно-шлуночковий вузол

B. Синусовий вузол

C. Пучок Гiса

D. Волокна Пуркiньє

E. Мiокард шлуночкiв

У здорової дорослої людини швидкiсть проведення збудження через атрiовентрикулярний вузол дорiвнює 0,02-0,05 м/с. Атрiовентрикулярна затримка забезпечує:

A. Послiдовнiсть скорочення передсердь та шлуночкiв

B. Одночаснiсть скорочення обох передсердь

C. Одночаснiсть скорочення обох шлуночкiв

D. Достатню силу скорочення передсердь

E. Достатню силу скорочення шлуночкiв

У хворого на iшемiчну хворобу серця вiдзначається гiпертрофiя мiокарда, тахiкардiя, зниження ХОК. Який з механiзмiв є провiдним в ушкодженнi кардiомiоцитiв у даному випадку?

A. Пошкодження специфiчних мембранних насосiв

B. Збiльшення числа α та βадренорецепторiв

C. Втрата Mg 2+ кардiомiоцитами

D. Втрата Ca 2+ кардiомiоцитами

E. Дегiдратацiя кардiомiоцитiв

У жiнки обмежений кровотiк у нирках, пiдвищений артерiальний тиск. Гiперсекрецiя якого гормону зумовила пiдвищений тиск?

A. Ренiн

B. Адреналiн

C. Норадреналiн

D. Еритропоетин

E. Вазопресин

До серцево-судинного вiддiлення надiйшов хворий зi скаргами на постiйний головний бiль у потиличнiй дiлянцi, шум у вухах, запаморочення. При обстеженнi: АТ180/110 мм рт.ст., ЧСС- 95/хв. Рентгенологiчно визначено звуження однiєї з ниркових артерiй. Активацiя якої з перерахованих систем викликала гiпертензивний стан хворого?

A. Ренiн-ангiотензинова

B. Гемостатична

C. Симпатоадреналова

D. Кiнiнова

E. Iмунна

У пацiєнта, який пiвтора мiсяця тому перенiс iнфаркт мiокарда, дiагностовано синдром Дреслера з характерною трiадою: перикардит, плеврит, пневмонiя. Який головний механiзм цього ускладнення?

A. Сенсибiлiзацiя органiзму антигенами мiокарда

B. Зниження резистентностi до iнфекцiйних агентiв

C. Активацiя сапрофiтної мiкрофлори

D. Iнтоксикацiя органiзму продуктами некрозу

E. Викидання у кров мiокардiальних ферментiв

У дорослої людини системний артерiальний тиск знизився з 120/70 до 90/50 мм рт.ст., що викликало рефлекторне звуження судин. У якому з зазначених органiв звуження судин буде найменшим?

A. Серце

B. Шкiра

C. Кишечник

D. Скелетнi м’язи

E. Печiнка

Внаслiдок стресу у похилої людини пiдвищився артерiальний тиск. Причиною цього є активацiя:

A. Симпато-адреналової системи

B. Парасимпатичного ядра блукаючого нерва

C. Функцiї щитоподiбної залози

D. Функцiї кори наднирникiв

E. Функцiї гiпофiзу

На ЕКГ пацiєнта мають мiсце такi змiни: зубець P - нормальний, iнтервал P − Q - вкорочений, шлуночковий комплекс QRST - розширений, зубець R двогорбий або двофазний. Яка iз форм аритмiї має мiсце у даного пацiєнта?

A. Синдром WPW (Вольфа-ПаркiнсонаУайта)

B. Синдром Фредерiка (трiпотiння передсердь)

C. Атрiовентрикулярна блокада

D. Миготiння шлуночкiв

E. Миготлива аритмiя

Хворий 39-ти рокiв з алкогольним цирозом печiнки скаржиться на задишку, загальну слабкiсть. Встановлено зниження артерiального тиску, розширення поверхневих вен передньої стiнки живота, спленомегалiю. Яке порушення гемодинамiки спостерiгається у хворого?

A. Синдром портальної гiпертензiї

B. Недостатнiсть лiвого шлуночка серця

C. Недостатнiсть правого шлуночка серця

D. Колапс

E. Тотальна серцева недостатнiсть

При аналiзi ЕКГ виявлено випадiння деяких серцевих циклiв PQRST. Наявнi зубцi та комплекси не змiненi. Назвiть вид аритмiї:

A. Синоатрiальна блокада

B. Миготлива аритмiя

C. Атрiовентрикулярна блокада

D. Передсердна екстрасистола

E. Внутрiшньопередсердна блокада

Пiд час емоцiйного збудження частота серцевих скорочень (ЧСС) у людини 30-ти рокiв досягла 112/хв. Змiна стану якої структури провiдної системи серця є причиною збiльшення ЧСС?

A. Синоатрiальний вузол

B. Волокна Пуркiн’є

C. Нiжки пучка Гiса

D. Атрiовентрикулярний вузол

E. Пучок Гiса

Хворому чоловiку 75-ти рокiв, у якого частота серцевих скорочень була 40/хвилину, iмплантували серцевий електростимулятор. Пiсля цього частота серцевих скорочень зросла до 70/хв. Функцiю якого вiддiлу серця взяв на себе електростимулятор?

A. Синоатрiальний вузол

B. Атрiовентрикулярний вузол

C. Нiжки Гiса

D. Волокна пучка Гiса

E. Волокна Пуркiн’є

На перехiд iз горизонтального положення у вертикальне система кровообiгу вiдповiдає розвитком рефлекторної пресорної реакцiї. Що з наведеного є її обов’язковим компонентом?

A. Системне звуження венозних судин ємностi

B. Системне розширення артерiальних судин опору

C. Зменшення об’єму циркулюючої кровi

D. Зменшення частоти серцевих скорочень

E. Зменшення насосної функцiї серця

У студента 18-ти рокiв пiд час фiзичного навантаження реографiчно зареєстровано перерозподiл кровотоку органiв. У яких судинах кровотiк пiдвищився найбiльшою мiрою?

A. Скелетнi м’язи

B. Печiнка

C. Головний мозок

D. Нирки

E. Шлунково-кишковий тракт

У хворого пiсля тривалого психоемоцiйного напруження спостерiгається пiдвищення артерiального тиску, що супроводжується серцебиттям, кардiалгiями, головним болем, запамороченням. Домiнуючим у формуваннi артерiальної гiпертензiї у даному випадку є збiльшення:

A. Тонусу артерiол

B. Тонусу венул

C. Об’єму циркулюючої кровi

D. Частоти серцевих скорочень

E. Серцевого викиду

При аналiзi ЕКГ необхiдно визначити, що є водiєм ритму серця. Зробити це можна на пiдставi вимiрювання:

A. Тривалостi iнтервалу R − R

B. Амплiтуди зубцiв

C. Напрямку зубцiв

D. Тривалостi зубцiв

E. Тривалостi комплексу QRST

У щурiв, що знаходяться у станi стресу, пiдвищенi м’язовий тонус та артерiальний тиск, збiльшений вмiст глюкози у кровi, посилена секрецiя кортикотропiну i кортикостероїдiв. У якiй фазi стресу знаходяться цi тварини?

A. Фаза протишоку

B. Виснаження

C. Фаза шоку

D. Еректильна

E. Термiнальна

У хворого виявлено екстрасистолiю. На ЕКГ при екстрасистолiчному скороченнi вiдсутнiй зубець P , комплекс QRS деформований, є повна компенсаторна пауза. Якi це екстрасистоли?

A. Шлуночковi

B. Передсерднi

C. Передсердно-шлуночковi

D. Синуснi

E. -

Хворий був доставлений до лiкарнi в коматозному станi. В анамнезi цукровий дiабет. Об’єктивно: дихання Кусмауля, зниження артерiального тиску, у видихуваному повiтрi запах ацетону. Пiсля проведеної невiдкладної терапiї стан покращився. Який препарат було введено хворому?

A. Iнсулiн

B. Адреналiн

C. Iзадрин

D. Букаркам

E. Глiбенкламiд

У хворої встановлено порушення видiлення тиреотропного гормону гiпофiза. Зi зниженням функцiй якої частки гiпофiза це пов’язано?

A. Lobus anterior

B. Infundibulum

C. Lobus posterior

D. Pars intermedia

E. -

У пацiєнта з пiдвищеним артерiальним тиском, тремором, тахiкардiєю, була дiагностовано доброякiсна пухлина мозкової речовини наднирникiв. Гiперсекрецiя якого гормону викликає таку симптоматику?

A. Адреналiн

B. Глюкагон

C. Iнсулiн

D. Тироксин

E. Соматотропiн

Недбалий студент раптово зустрiвся з деканом. Концентрацiя якого гормону найшвидше збiльшиться в кровi студента?

A. Адреналiн

B. Тиреолiберин

C. Кортикотропiн

D. Кортизол

E. Соматотропiн

У дiвчинки дiагностований адреногенiтальний синдром (псевдогермафродитизм). Надмiрна секрецiя яких гормонiв наднирникiв обумовила дану патологiю?

A. Андрогени

B. Естрогени

C. Мiнералокортикоїди

D. Глюкокортикоїди

E. Катехоламiни

У хворого вiдзначаються перiодичнi напади серцебиття (пароксизми), сильне потовидiлення, напади головного болю. При обстеженнi виявлена гiпертензiя, гiперглiкемiя, пiдвищення основного обмiну, тахiкардiя. При якiй патологiї наднирникiв спостерiгається подiбна картина?

A. Гiперфункцiя мозкового шару

B. Гiпофункцiя мозкового шару

C. Гiперфункцiя кори наднирникiв

D. Гiпофункцiя кори наднирникiв

E. Первинний альдостеронiзм

Хлопчик 5-ти мiсяцiв госпiталiзований з приводу тонiчних судом. Хворiє з народження. Об’єктивно: волосся жорстке, нiгтi витонченi та ламкi, шкiрнi покриви блiдi та сухi. В бiохiмiчному аналiзi кровi: кальцiй - 0,5 ммоль/л (норма - 0,75-2,5 ммоль/л), фосфор - 1,9 ммоль/л (норма - 0,646-1,292 ммоль/л). З чим пов’язанi цi змiни?

A. Гiпопаратиреоз

B. Гiперпаратиреоз

C. Гiперальдостеронiзм

D. Гiпоальдостеронiзм

E. Гiпотиреоз

У хворої внаслiдок запалення порушена ендокринна функцiя фолiкулярних клiтин фолiкулiв яєчника. Синтез яких гормонiв буде пригнiчений?

A. Естрогени

B. Прогестерон

C. Лютропiн

D. Фолiкулостимулюючий гормон

E. Фолiстатин

При оглядi пацiєнта виявлене надмiрне розростання кiсток i м’яких тканин обличчя, збiльшенi розмiри язика, розширенi мiжзубнi промiжки в збiльшенiй зубнiй дузi. Якi змiни секрецiї гормонiв у нього найбiльш вiрогiднi?

A. Збiльшена секрецiя соматотропного гормону

B. Зменшена секрецiя соматотропного гормону

C. Збiльшена секрецiя iнсулiну

D. Зменшена секрецiя тироксину

E. Зменшена секрецiя iнсулiну

Чоловiку 46-ти рокiв, що хворiє на дифузний токсичний зоб, була проведена операцiя резекцiї щитоподiбної залози. Пiсля операцiї вiдмiчаються вiдсутнiсть апетиту, диспепсiя, пiдвищена нервово-м’язова збудливiсть. Маса тiла не збiльшилася. Температура тiла у нормi. Чим, iз нижче перелiченого, обумовлений стан хворого?

A. Зниженням продукцiї паратгормону

B. Зниженням продукцiї тироксину

C. Пiдвищенням продукцiї кальцитонiну

D. Пiдвищенням продукцiї тиреолiберину

E. Пiдвищенням продукцiї тироксину

Хворий 42-х рокiв висуває скарги на сильне серцебиття, пiтливiсть, нудоту, порушення зору, тремор рук, пiдвищення артерiального тиску. З анамнезу: 2 роки тому було встановлено дiагноз феохромоцитома. Гiперпродукцiя яких гормонiв зумовлює цю патологiю?

A. Катехоламiни

B. Альдостерон

C. Глюкокортикоїди

D. АКТГ

E. Тиреоїднi гормони

У хворого 41-го року вiдзначається гiпонатрiємiя, гiперкалiємiя, дегiдратацiя, зниження артерiального тиску, м’язова слабкiсть, брадикардiя, аритмiя. З порушенням функцiй яких гормонiв це пов’язано?

A. Кортикостероїди

B. Тиреоїднi

C. Гормони пiдшлункової залози

D. Статевi гормони

E. Гормони мозкової речовини наднирникiв

Пiсля перенесеного сепсису у хворої 27-ми рокiв з’явився бронзовий колiр шкiри, характерний для аддiсонової хвороби. Механiзм гiперпiгментацiї полягає в пiдвищеннi секрецiї такого гормону:

A. Меланоцитстимулюючий

B. Соматотропний

C. Гонадотропний

D. B-лiпотропний

E. Тиреотропний

Пацiєнт 16-ти рокiв, що страждає на хворобу Iценко-Кушiнга, консультований з приводу надмiрної ваги тiла. При опитуваннi з’ясувалося, що енергетична цiннiсть спожитої їжi складає 1700-1900 ккал/добу. Яка провiдна причина ожирiння у даному випадку?

A. Надлишок глюкокортикоїдiв

B. Нестача iнсулiну

C. Надлишок iнсулiну

D. Нестача глюкокортикоїдiв

E. Гiподинамiя

Чоловiк середнього вiку виїхав до iншої країни на обiцяну йому роботу, але працевлаштуватися тривалий час йому не вдавалося. Якi з ендокринних залоз були виснаженi у цiєї людини найбiльше?

A. Наднирники

B. Прищитоподiбнi

C. Сiм’яники

D. Пiдгрудинна

E. Щитоподiбна

Пацiєнта турбують полiурiя (7 л на добу) i полiдипсiя. При обстеженнi не виявлено нiяких розладiв вуглеводного обмiну. Дисфункцiя якої ендокринної залози може бути причиною даних порушень?

A. Нейрогiпофiз

B. Аденогiпофiз

C. Острiвцi пiдшлункової залози

D. Кора наднирникiв

E. Мозкова речовина наднирникiв

Жiнка 44-х рокiв скаржиться на загальну слабкiсть, бiль у дiлянцi серця, значне збiльшення маси тiла. Об’єктивно: обличчя мiсяцеподiбне, гiрсутизм, АТ- 165/100 мм рт.ст., зрiст - 164 см, вага - 103 кг; переважно накопичення жиру на шиї, верхньому плечовому поясi, животi. Що є основним патогенетичним механiзмом ожирiння у жiнки?

A. Пiдвищення продукцiї глюкокортикоїдiв

B. Зниження продукцiї тиреоїдних гормонiв

C. Пiдвищення продукцiї iнсулiну

D. Зниження продукцiї глюкагону

E. Пiдвищення продукцiї мiнералокортикоїдiв

Спецiальний режим харчування призвiв до зменшення iонiв Ca2+ в кровi. До збiльшення секрецiї якого гормону це призведе?

A. Паратгормон

B. Тирокальцитонiн

C. Вазопресин

D. Соматотропiн

E. Тироксин

У людини збiльшений вмiст iонiв кальцiю в плазмi кровi, зменшений – у кiстках. Надмiрна секрецiя якого гормону може спричинити такi змiни?

A. Паратгормон

B. Тироксин

C. Трийодтиронiн

D. Тиреокальцитонiн

E. Альдостерон

Внаслiдок вираженого зниження концентрацiї кальцiю в плазмi кровi у дитини 2-х рокiв виникли тетанiчнi скорочення дихальних i глоткових м’язiв. Зниження секрецiї якого гормону може бути причиною цього?

A. Паратгормон

B. Тиреокальцитонiн

C. Альдостерон

D. Соматотропiн

E. Кортизол

До лiкаря звернувся чоловiк 27-ми рокiв. При оглядi було виявлено збiльшення кистей, стоп та нижньої щелепи. Крiм того спостерiгалися деформацiя суглобiв (kiphosis), гормональнi порушення (iмпотенцiя, атрофiя яєчок). Функцiя якої залози порушена?

A. Передня частка гiпофiзу

B. Наднирковi залози

C. Шишкоподiбне тiло

D. Щитоподiбна залоза

E. Прищитоподiбнi залози

У дитини 2-х рокiв виникли судоми внаслiдок зниження концентрацiї iонiв кальцiю в плазмi кровi. Функцiя якого ендокринного органу знижена?

A. Прищитоподiбнi залози

B. Гiпофiз

C. Кора наднирникiв

D. Шишкоподiбна залоза

E. Тимус

До лiкаря звернулася жiнка 32-х рокiв зi скаргами на вiдсутнiсть лактацiї пiсля народження дитини. Дефiцитом якого гормону можна пояснити дане порушення?

A. Пролактин

B. Соматотропiн

C. Вазопресин

D. Тиреокальцитонiн

E. Глюкагон

Хворому тривалий час вводили високi дози гiдрокортизону, внаслiдок чого настала атрофiя однiєї з зон кори наднирникiв. Яка це зона?

A. Пучкова

B. Клубочкова

C. Сiтчаста

D. Клубочкова i сiтчаста

E. -

При тиреотоксикозi пiдвищується продукцiя тиреоїдних гормонiв Т3 та Т4, розвиваються схуднення, тахiкардiя, психiчне збудження та iнше. Як саме впливають тиреоїднi гормони на енергетичний обмiн в мiтохондрiях клiтин?

A. Роз‘єднують окислення та окисне фосфорилювання

B. Активують субстратне фосфорилювання

C. Блокують субстратне фосфорилювання

D. Блокують дихальний ланцюг

E. Активують окисне фосфорилювання

Хворий знаходиться на облiку в ендокринологiчному диспансерi з приводу гiпертиреозу. До схуднення, тахiкардiї, тремтiння пальцiв рук, приєдналися симптоми гiпоксiї – головний бiль, втомлюванiсть, мерехтiння "мушок"перед очима. Який механiзм дiї тиреоїдних гормонiв лежить в основi розвитку гiпоксiї?

A. Роз’єднання окиснення та фосфорилування

B. Гальмування синтезу дихальних ферментiв

C. Конкурентне гальмування дихальних ферментiв

D. Посилення синтезу дихальних ферментiв

E. Специфiчне зв’язування активних центрiв дихальних ферментiв

Стресовий стан i больове вiдчуття у пацiєнта перед вiзитом до стоматолога супроводжуються анурiєю (вiдсутнiстю сечовидiлення). Це явище зумовлене збiльшенням:

A. Секрецiї вазопресину та адреналiну

B. Активностi парасимпатичної нервової системи

C. Активностi антиноцiцептивної системи

D. Секрецiї вазопресину та зменшенням адреналiну

E. Секрецiї адреналiну та зменшенням вазопресину

Хворий помилково прийняв надмiрну дозу тироксину. До яких змiн секрецiї тиреолiберину та тиреотропiну це призведе?

A. Секрецiя гормонiв зменшиться

B. Секрецiя гормонiв збiльшиться

C. Змiн секрецiї гормонiв не буде

D. Секрецiя тиреолiберину збiльшиться, тиреотропiну - зменшиться

E. Секрецiя тиреотропiну збiльшиться, тиреолiберину - зменшиться

У хворих на тиреотоксикоз спостерiгаються гiпертермiя, булiмiя, зменшення маси тiла, що пов’язане з порушенням:

A. Спряження окислення i фосфорилювання

B. Розпаду АТФ

C. Синтезу жирiв

D. Циклу лимонної кислоти

E. β-окиснення жирних кислот

У чоловiка 35-ти рокiв феохромоцитома. В кровi спостерiгається пiдвищений рiвень адреналiну та норадреналiну, концентрацiя вiльних жирних кислот зросла в 11 разiв. Активацiя якого ферменту пiд впливом адреналiну пiдвищує лiполiз?

A. ТАГ-лiпаза

B. Лiпопротеїдлiпаза

C. Фосфолiпаза

D. Фосфолiпаза С

E. Холестеролестераза

До лiкаря звернулися батьки хлопчика 10-ти рокiв, у якого вiдзначалося збiльшення волосяного покриву на тiлi, рiст бороди i вус, низький голос. Збiльшення секрецiї якого гормону можна припустити?

A. Тестостерон

B. Соматотропiн

C. Естроген

D. Прогестерон

E. Кортизол

Жiнка 38-ми рокiв звернулася до ендокринологiчної клiнiки з виразним тремором кiнцiвок. Гiперпродукцiя, якого гормону здатна викликати такi порушення?

A. Тироксин

B. АКТГ

C. Iнсулiн

D. Адреналiн

E. Соматостатин

У мешканцiв територiй з холодним клiматом в кровi збiльшений вмiст гормону, що має пристосувальне терморегуляторне значення. Про який гормон йдеться?

A. Тироксин

B. Iнсулiн

C. Глюкагон

D. Соматотропiн

E. Кортизол

У хворого з верхнiм типом ожирiння тривало вiдзначалися артерiальна гiпертонiя, гiперглiкемiя, глюкозурiя. Смерть настала вiд крововиливу у головний мозок. Пiд час патоморфологiчного дослiдження виявленi базофiльна аденома гiпофiзу, гiперплазiя кори наднирникiв. Який найбiльш вiрогiдний дiагноз?

A. Хвороба Iценка-Кушiнга

B. Цукровий дiабет

C. Акромегалiя

D. Гiпофiзарний нанiзм

E. Адипозогенiтальна дистрофiя

У пацiєнта, що звернувся до лiкаря, спостерiгається жовте забарвлення шкiри, сеча темна, кал темно-жовтого кольору. Пiдвищення концентрацiї якої речовини буде спостерiгатися в сироватцi кровi?

A. Вiльний бiлiрубiн

B. Кон’югований бiлiрубiн

C. Мезобiлiрубiн

D. Вердоглобiн

E. Бiлiвердин

У хворого з жовтяницею встановлено: пiдвищення у плазмi кровi вмiсту загального бiлiрубiну за рахунок непрямого (вiльного), в калi i сечi - високий вмiст стеркобiлiну, рiвень прямого (зв’язаного) бiлiрубiну у плазмi кровi в межах норми. Який вид жовтяницi має мiсце у хворого?

A. Гемолiтична

B. Паренхiматозна

C. Механiчна

D. Хвороба Жильбера

E. -

У хворого з’явилися жовтушнiсть шкiри, склер та слизових оболонок. У плазмi кровi пiдвищений рiвень загального бiлiрубiну, в калi - рiвень стеркобiлiну, в сечi - уробiлiну. Який вид жовтяницi у хворого?

A. Гемолiтична

B. Хвороба Жiльбера

C. Паренхiматозна

D. Обтурацiйна

E. Холестатична

У 70-тi роки вченi встановили, що причиною важкої жовтяницi новонароджених є порушення зв’язування бiлiрубiну в гепатоцитах. Яка речовина використовується для утворення кон’югату?

A. Глюкуронова кислота

B. Сечова кислота

C. Сiрчана кислота

D. Молочна кислота

E. Пiровиноградна кислота

У хворого на жовтяницю у кровi пiдвищений вмiст прямого бiлiрубiну та жовчних кислот; у сечi вiдсутнiй стеркобiлiноген. При якiй жовтяницi можлива наявнiсть цих ознак?

A. Механiчна

B. Печiнкова

C. Паренхiматозна

D. Гемолiтична

E. Надпечiнкова

Хвора 48-ми рокiв надiйшла до клiнiки iз скаргами на слабкiсть, дратiвливiсть, порушення сну. Об’єктивно: шкiра та склери жовтого кольору. У кровi: пiдвищення рiвня загального бiлiрубiну з переважанням прямого. Кал - ахолiчний. Сеча - темного кольору (жовчнi пiгменти). Яка жовтяниця має мiсце в хворої?

A. Механiчна

B. Гемолiтична

C. Паренхiматозна

D. Синдром Жiльбера

E. Синдром Крiглера-Найяра

Хворий надiйшов до клiнiки зi скаргами на загальну слабкiсть, порушення сну. Шкiра має жовтий колiр. У кровi: збiльшена кiлькiсть прямого бiлiрубiну, жовчних кислот. Кал ахолiчний. Для якого стану характернi цi змiни?

A. Механiчна жовтяниця

B. Гемолiтична жовтяниця

C. Надпечiнкова жовтяниця

D. Синдром Жiльбера

E. Хронiчний холецистит

У хворого на пiдгострий септичний ендокардит при оглядi лiкар вiдзначив загальну слабкiсть i iктеричнiсть шкiри, склер i видимих слизових оболонок. У кровi виявлена збiльшена кiлькiсть непрямого бiлiрубiну. Що зумовлює жовтяничнiсть шкiри i слизових?

A. Надпечiнкова жовтяниця

B. Жирова дистрофiя

C. Гемосидероз

D. Печiнкова жовтяниця

E. Пiдпечiнкова жовтяниця

У юнака 20-ти рокiв дiагностовано спадковий дефiцит УДФ-глюкуронiлтрансферази. Пiдвищення якого показника кровi пiдтверджує дiагноз?

A. Непрямий (некон’югований) бiлiрубiн

B. Прямий (кон’югований) бiлiрубiн

C. Уробiлiн

D. Стеркобiлiноген

E. Тваринний iндикан

Хвора 28-ми рокiв потрапила до iнфекцiйної лiкарнi з приводу пожовтiння шкiри, склер, слизових оболонок. Лабораторно встановлене пiдвищення рiвня прямого бiлiрубiну у кровi. В сечi виявлений уробiлiноген i бiлiрубiн. Для якого з перелiчених захворювань характернi такi змiни?

A. Паренхiматозна жовтяниця

B. Гемолiтична жовтяниця

C. Iнфаркт нирки

D. Туберкульоз нирки

E. Механiчна жовтяниця

Для лiкування жовтяниць показано призначення барбiтуратiв, якi iндукують синтез УДФглюкуронiлтрансферази. Лiкувальний ефект при цьому обумовлений утворенням:

A. Прямого (кон’югованого) бiлiрубiну

B. Непрямого (некон’югованого) бiлiрубiну

C. Бiлiвердину

D. Протопорфирину

E. Гему

У хворого, який скаржився на бiль у дiлянцi лiвої лопатки, був дiагностований iнфаркт мiокарду. Назвiть вид болю у хворого?

A. Iррадiюючий (вiдбитий)

B. Вiсцеральний

C. Фантомний

D. Перший (протопатичний)

E. Другий (епiкритичний)

У людини нормальна чутливiсть шкiри пальця, але вiн не вiдчуває наявностi на ньому обручки. Який процес, спричинений впливом обручки, є причиною цього?

A. Адаптацiя рецепторiв

B. Розвиток фiброзної тканини

C. Порушення структури епiдермiсу

D. Порушення кровообiгу

E. Порушення структури рецепторiв

В результатi травми порушено цiлiснiсть переднього корiнця спинного мозку. Якi вiдростки яких нейронiв при цьому пошкодженi?

A. Аксони рухових нейронiв

B. Дендрити рухових нейронiв

C. Аксони чутливих нейронiв

D. Дендрити чутливих нейронiв

E. Дендрити вставних нейронiв

У пiддослiдного щура з паралiчем кiнцiвки спостерiгається зникнення сухожилкових i шкiрних рефлексiв, зниження м’язового тонусу, при цьому зберiгається здатнiсть м’язiв ураженої кiнцiвки вiдповiдати збудженням на пряму дiю постiйного струму. Який тип паралiчу вiдзначається у тварини?

A. В’ялий периферичний

B. В’ялий центральний

C. Спастичний периферичний

D. Спастичний центральний

E. Екстрапiрамiдний

Пiсля обстеження пацiєнта в клiнiцi нервових хвороб встановлена вiдсутнiсть звуження зiницi при дiї свiтла. З ураженням яких структур головного мозку це пов’язано?

A. Вегетативнi ядра 3 пари черепномозкових нервiв

B. Червонi ядра середнього мозку

C. Ретикулярнi ядра середнього мозку

D. Ядра гiпоталамуса

E. Ретикулярнi ядра довгастого мозку

У тварини збiльшений тонус м’язiв-розгиначiв. Це є наслiдком посиленої передачi iнформацiї до мотонейронiв спинного мозку такими низхiдними шляхами:

A. Вестибулоспiнальнi

B. Медiальнi кортикоспiнальнi

C. Ретикулоспiнальнi

D. Руброспiнальнi

E. Латеральнi кортикоспiнальнi

У чоловiка 60-ти рокiв пiсля iнсульту настав тривалий сон. Ураження яких структур ЦНС найбiльш iмовiрно призвело до цього стану?

A. Висхiдна частина РФ

B. Мозочок

C. Чорна субстанцiя

D. Прецентральна звивина

E. V-IX пари черепних нервiв

Внаслiдок травми у чоловiка 40-ка рокiв зруйнованi заднi корiнцi спинного мозку. Якi розлади будуть спостерiгатися в зонi iннервацiї цих корiнцiв?

A. Втрата всiх видiв чутливостi

B. Порушення функцiї посмугованих скелетних м’язiв

C. Порушення функцiї гладеньких м’язiв

D. Втрата температурної та вiбрацiйної чутливостi

E. Втрата больової чутливостi

У тварини в експериментi перерiзали заднi корiнцi спинного мозку. Якi змiнi вiдбуватимуться в зонi iннервацiї?

A. Втрата чутливостi

B. Втрата рухових функцiй

C. Зниження тонусу м’язiв

D. Пiдвищення тонусу м’язiв

E. Втрата чутливостi i рухових функцiй

Хворий 50-ти рокiв звернувся до клiнiки зi скаргами на загальну слабкiсть, втрату апетиту, аритмiю серця. Спостерiгається гiпотонiя м’язiв, млявi паралiчi, послаблення перистальтики кишечнику. Причиною такого стану може бути:

A. Гiпокалiємiя

B. Гiпопротеїнемiя

C. Гiперкалiємiя

D. Гiпофосфатемiя

E. Гiпонатрiємiя

Дитина 9-ми мiсяцiв харчується штучними сумiшами, якi не збалансованi за вмiстом вiтамiну B6 . У дитини спостерiгається пелагроподiбний дерматит, судоми, анемiя. Розвиток судом може бути пов’язаний з порушенням утворення:

A. ГАМК

B. Гiстамiну

C. Серотонiну

D. ДОФА

E. Дофамiну

Жiнка 68-ми рокiв скаржиться на вiдсутнiсть рухiв у правих руцi i нозi. Чотири мiсяцi тому перенесла iнсульт. Об’єктивно: рухи в правих кiнцiвках вiдсутнi, тонус м’язiв їх пiдвищений. Який стан спостерiгається у хворої?

A. Гемiплегiя

B. Моноплегiя

C. Параплегiя

D. Тетраплегiя

E. -

У хворого пiсля перенесеної черепно-мозкової травми порушений акт ковтання. Який вiддiл мозку постраждав?

A. Довгастий мозок

B. Середнiй мозок

C. Промiжний мозок

D. Кiнцевий мозок

E. Таламус

В хронiчному експериментi на щурах стимулювали електричним струмом паравентрикулярнi та супраоптичнi ядра гiпоталамуса. Яку поведiнкову реакцiю це спричинило у тварин?

A. Збiльшення споживання води

B. Зменшення споживання води

C. Збiльшення споживання їжi

D. Зменшення споживання їжi

E. Вiдмова вiд їжi та рiдини

У пiддослiдної тварини пiд час експерименту подразнюють периферичний вiдрiзок блукаючого нерва. Якi з наведених змiн будуть спостерiгатися при цьому?

A. Зменшення частоти серцевих скорочень

B. Збiльшення частоти серцевих скорочень

C. Розширення зiниць

D. Збiльшення частоти дихання

E. Розширення бронхiв

У хворої 49-ти рокiв вiдзначається обмеження довiльних рухiв у лiвих кiнцiвках. Тонус м’язiв у лiвих руцi та нозi пiдвищений за спастичним типом, посиленi мiсцевi сухожилковi рефлекси, виявляються патологiчнi рефлекси. Який найбiльш iмовiрний механiзм призвiв до розвитку м’язової гiпертонiї та гiперрефлексiї?

A. Зниження гальмiвних низхiдних впливiв

B. Активацiя мотонейронiв внаслiдок iнсульту

C. Активацiя збуджуючих впливiв з вогнища iнсульту

D. Активацiя синаптичної передачi iмпульсiв

E. Гальмування мотонейронiв кори головного мозку

У чоловiка вiдмiчається випадiння функцiї медiальних половин сiткiвки. Який вiддiл провiдного шляху зорового аналiзатора уражений?

A. Зорове перехрестя

B. Лiвий зоровий тракт

C. Правий зоровий тракт

D. Лiвий зоровий нерв

E. Правий зоровий нерв

У людини звуженi зiницi. Чим це зумовлено?

A. Зростання тонусу парасимпатичних центрiв

B. Зростання тонусу симпатичних центрiв

C. Збiльшення активностi симпатоадреналової системи

D. Дiя адреналiну

E. Дiя норадреналiну

Отруєння ботулiнiчним токсином, який блокує вхiд iонiв кальцiю до нервових закiнчень аксонiв мотонейронiв, небезпечно для життя, бо загрожує:

A. Зупинкою дихання

B. Зупинкою серця

C. Розладом тонусу судин

D. Розвитком блювання

E. Розвитком проносу

При дослiдженнi гостроти слуху в коваля виявили втрату слуху на 50% у дiапазонi низьких частот i майже нормальну гостроту слуху в дiапазонi високих частот. Порушення яких структур слухової системи призвело до такого стану?

A. Кортiєв орган - ближче до гелiкотреми

B. Кортiєв орган - ближче до овального вiконця

C. Середня частина кортiєвого органу

D. М’язи середнього вуха

E. Барабанна перетинка

Хвора 75-ти рокiв доставлена до офтальмологiчного вiддiлення лiкарнi зi скаргами на погiршення зору. При об’єктивному дослiдженнi встановлена наявнiсть пухлини мозку, що розташована в дiлянцi лiвого зорового тракту. При цьому у хворої спостерiгається випадiння поля зору в:

A. Лiвих половинах сiткiвки обох очей

B. Правих половинах сiткiвки обох очей

C. Правих i лiвих половинах сiткiвки лiвого ока

D. Правих i лiвих половинах сiткiвки правого ока

E. Правих i лiвих половинах сiткiвок обох очей

У хворої 18-ти рокiв з’явились постiйнi запаморочення, нiстагм очей, скандована мова, невпевнена хода. Це свiдчить про порушення функцiї:

A. Мозочка

B. Рухової кори

C. Базальних ганглiїв

D. Чорної субстанцiї

E. Вестибулярних ядер

Внаслiдок черепно-мозкової травми у хворого розвинулись наступнi симптоми: iнтенцiйний тремор, дисметрiя, адiадохокiнез, дизартрiя. Яка структура головного мозку ушкоджена?

A. Мозочок

B. Стрiатум

C. Рухова кора

D. Блiда куля

E. Чорна речовина

За медичним показанням пацiєнту було проведено видалення частини однiєї iз структур ЦНС. В результатi видалення у пацiєнта розвинулися атонiя, астазiя, iнтенцiйний тремор, атаксiя, адiадохокiнез. Частина якої структури ЦНС була вилучена?

A. Мозочок

B. Мигдалеподiбний комплекс

C. Гiпокамп

D. Базальнi ганглiї

E. Лiмбiчна система

Депресiї та емоцiйнi розлади є наслiдком нестачi у головному мозку норадреналiну, серотонiну та iнших бiогенних амiнiв. Збiльшення їх вмiсту у синапсах можна досягти за рахунок антидепресантiв, якi гальмують такий фермент:

A. Моноамiнооксидаза

B. Диамiнооксидаза

C. Оксидаза L-амiнокислот

D. Оксидаза D-амiнокислот

E. Фенiлаланiн-4-монооксигеназа

У лабораторному експериментi на собацi вивчали будову центральних вiддiлiв слухової сенсорної системи. Була зруйнована одна з структур середнього мозку. Собака втратив орiєнтувальний рефлекс на звуковi сигнали. Яка структура була зруйнована?

A. Нижнi горбики чотиригорбикового тiла

B. Верхнi горбики чотиригорбикового тiла

C. Чорна речовина

D. Ядра ретикулярної формацiї

E. Червоне ядро

У реанiмацiйному вiддiленнi знаходиться хворий у коматозному станi. При дослiдженнi кровi вiдзначено збiльшення концентрацiї iонiв K + I зменшення - Ca ++ , ацидоз, збiльшення рiвнiв сечовини, сечової кислоти. Який вид коми за етiологiєю найбiльш iмовiрний?

A. Ниркова

B. Печiнкова

C. Нейрогенна

D. Дiабетична

E. Гiпоглiкемiчна

Внаслiдок руйнування певних структур стовбуру мозку тварина втратила орiєнтувальнi рефлекси у вiдповiдь на сильнi свiтловi подразники. Якi структури було зруйновано?

A. Переднi горбки чотиригорбкового тiла

B. Заднi горбки чотиригорбкового тiла

C. Червонi ядра

D. Вестибулярнi ядра

E. Чорна речовина

Хворий звернувся до лiкаря зi скаргами на порушення вiдчуття рiвноваги, що з’явилося пiсля травми. Який нерв пошкоджено?

A. Присiнково-завитковий

B. Трiйчастий

C. Лицевий

D. Промiжний

E. Блукаючий

У кiшки з децеребрацiйною ригiднiстю потрiбно знизити тонус м’язiв. Цього можна досягти шляхом:

A. Руйнування вестибулярних ядер Дейтерса

B. Подразнення отолiтових вестибулорецепторiв

C. Подразнення вестибулярних ядер Дейтерса

D. Подразнення вестибулослухового нерва

E. Подразнення ампулярних вестибулорецепторiв

Пiд час футбольного матчу мiж вболiвальниками рiзних команд виникла сутичка. На фонi негативних емоцiй в одного учасника сутички були розширенi зiницi й пiдвищене серцебиття. Активацiя якої системи регуляцiї функцiй органiзму забезпечує такi вегетативнi змiни при негативних емоцiях?

A. Симпато-адреналова

B. Гiпоталамо-гiпофiзарно-тиреоїдна

C. Соматична нервова

D. Парасимпатична нервова

E. Метасимпатична нервова

В клiнiку доставлено чоловiка з травмою спини. Пiд час обстеження виявлено перелом хребцiв грудного вiддiлу. Пiд час об’єктивного огляду нейрохiрургом виявлено: нижче рiвня перелому з правого боку вiдсутня глибока чутливiсть, з лiвого боку – порушена температурна та тактильна чутливiсть. Яке ураження з боку спинного мозку є у хворого?

A. Синдром Броун Секара

B. Хвороба Паркiнсона

C. Судомний синдром

D. Анестезiя

E. Парастезiя

До вiддiлення нейрохiрургiї було доставлено чоловiка з втратою слуху внаслiдок травми голови. Порушення якої частки кори головного мозку може бути причиною цього?

A. Скронева

B. Постцентральна звивина

C. Тiм’яна

D. Потилична

E. Лобова

У чоловiка 33-х рокiв внаслiдок спинномозкової травми порушена больова та температурна чутливiсть, що обумовлено пошкодженням таких висхiдних шляхiв:

A. Спиноталамiчнi

B. Медiальний спинокортикальний

C. Заднiй спиномозочковий

D. Латеральний спинокортикальний

E. Переднiй спиномозочковий

У тварини зруйнували отолiтовi вестибулорецептори. Якi з наведених рефлексiв зникнуть внаслiдок цього у тварини?

A. Статокiнетичнi при рухах з лiнiйним прискоренням

B. Статокiнетичнi при рухах з кутовим прискоренням

C. Мiотатичнi

D. Випрямлення тулуба

E. Первиннi орiєнтувальнi

У хворого поперечний розрив спинного мозку нижче VI грудного сегменту. Як внаслiдок цього змiниться дихання?

A. Суттєво не змiниться

B. Зупиниться

C. Стане бiльш рiдким

D. Стане бiльш глибоким

E. Стане бiльш частим

До лiкарнi звернувся чоловiк 50-ти рокiв з розладами пам’ятi, болiсними вiдчуттями по ходу нервових стовбурiв, зниженням iнтелектуальних функцiй, порушеннями з боку серцево-судинної системи i явищами диспепсiї. В анамнезi хронiчний алкоголiзм. Дефiцит якого вiтамiну може викликати цi симптоми?

A. Тiамiн

B. Нiацин

C. Ретинол

D. Кальциферол

E. Рибофлавiн

У чоловiка 28-ми рокiв пiсля вогнепального поранення гомiлки розвинулася виразка на боцi пошкодження. Що є основним у патогенезi нейродистрофiї в даному випадку?

A. Травматизацiя периферичного нерва

B. Психiчний стрес

C. Порушення мiкроциркуляцiї

D. Iнфекцiя

E. Пошкодження тканини

При травмi периферичних нервiв виникає м’язова атрофiя, кiстки стають порозними i ламкими, на шкiрi i слизових виникають виразки. Яка функцiя нервових системи уражується у даному випадку?

A. Трофiчна

B. Рухова

C. Чутлива

D. Вегетативна

E. Вища нервова дiяльнiсть

У чоловiка пiсля гiпертонiчної кризи вiдзначається вiдсутнiсть довiльних рухiв в правих руцi та нозi, тонус м’язiв у цих кiнцiвках пiдвищений. Який вид розладу рухової функцiї спостерiгається у даному випадку?

A. Центральний паралiч

B. Периферичний паралiч

C. Периферичний парез

D. Рефлекторний парез

E. Центральний парез

Внаслiдок руйнування певних структур стовбуру мозку тварина втратила орiєнтувальнi рефлекси. Якi структури було зруйновано?

A. Чотиригорбкова структура

B. Медiальнi ядра ретикулярної формацiї

C. Червонi ядра

D. Вестибулярнi ядра

E. Чорна речовина

У хворого 48-ми рокiв на хронiчний гломерулонефрит наявнi набряки, АТ-210/100 мм рт.ст., ЧСС- 85/хв., межi серця розширенi. Який механiзм розвитку артерiальної гiпертензiї є головним?

A. Активацiя ренин-ангiотензинальдостеронової системи

B. Пiдвищення ОЦК

C. Пiдвищення продукцiї вазопресину

D. Пiдвищення активностi симпатичного вiддiлу нервової системи

E. Гiперфункцiя серця

У хворого 38-ми рокiв на 3-му роцi захворювання на системний червоний вiвчак виявлене дифузне ураження нирок, що супроводжується масивними набряками i вираженою протеїнурiєю. Що є найбiльш вiрогiдною причиною розвитку протеїнурiї у пацiєнта?

A. Аутоiмунне ушкодження нирок

B. Асептичне ураження нирок

C. Iшемiчне ушкодження нирок

D. Запальне ураження сечового мiхура

E. Запальне ураження сечовивiдних шляхiв

У хворого з гострою нирковою недостатнiстю на 6-й день проведення терапевтичних заходiв виникла полiурiя. Чим зумовлене зростання дiурезу на початку полiуричної стадiї гострої ниркової недостатностi?

A. Вiдновлення фiльтрацiї в нефронах

B. Збiльшенням об’єму циркулюючої кровi

C. Збiльшенням натрiйуретичного фактора

D. Зменшенням альдостерону в плазмi

E. Зменшенням вазопресину в плазмi

Хворий 55-ти рокiв хворiє на хронiчний гломерулонефрит протягом 15-ти рокiв. Якi змiни складу кровi або сечi найбiльш характерно свiдчать про обмеження секреторної функцiї нирок?

A. Гiперазотемiя

B. Гiперглiкемiя

C. Гiпопротеїнемiя

D. Протеїнурiя

E. Гiпо-, iзостенурiя

У пацiєнта з хронiчним захворюванням нирок розвинулась ниркова недостатнiсть. Який з показникiв найбiльш iмовiрно свiдчить про порушення реабсорбцiї в канальцях в даному випадку?

A. Гiпо- та iзостенурiя

B. Гiперазотемiя

C. Зниження клiренсу

D. Гематурiя

E. Лейкоцитурiя

Внаслiдок землетрусу чоловiк 50-ти рокiв двi доби перебував пiд завалом. Пiсля звiльнення з-пiд завалу рятiвниками у нього був встановлений синдром тривалого розчавлення. Виникнення якого ускладнення в подальшому найбiльш вiрогiдне?

A. Гостра ниркова недостатнiсть

B. Гостра печiнкова недостатнiсть

C. Гостра серцева недостатнiсть

D. Гостра судинна недостатнiсть

E. Гостра дихальна недостатнiсть

У результатi порушення технiки безпеки вiдбулося отруєння сулемою (хлористою ртуттю). Через 2 днi добовий дiурез склав 620 мл. У хворого з’явилися головний бiль, блювання, судоми, задишка, у легенях - вологi хрипи. Яка патологiя має мiсце?

A. Гостра ниркова недостатнiсть

B. Хронiчна ниркова недостатнiсть

C. Уремiчна кома

D. Гломерулонефрит

E. Пiєлонефрит

У пацiєнта встановлено порушення синтезу та видiлення вазопресину. В якому вiддiлi нефрона найбiльше порушиться процес сечоутворення?

A. Збiрна трубочка

B. Проксимальний звивистий каналець

C. Тонка частина петлi Генле

D. Товста частина петлi Генле

E. Клубочок

У хворого з масивними опiками розвинулась гостра недостатнiсть нирок, що характеризується значним i швидким зменшенням швидкостi клубочкової фiльтрацiї. Який механiзм її розвитку?

A. Зменшення ниркового кровотоку

B. Ушкодження клубочкового фiльтра

C. Зменшення кiлькостi функцiонуючих нефронiв

D. Збiльшення тиску канальцевої рiдини

E. Емболiя ниркової артерiї

У людини внаслiдок тривалого голодування швидкiсть клубочкової фiльтрацiї зросла на 20%. Найбiльш вiрогiдною причиною змiн фiльтрацiї в зазначених умовах є:

A. Зменшення онкотичного тиску плазми кровi

B. Збiльшення системного артерiального тиску

C. Збiльшення проникностi ниркового фiльтру

D. Збiльшення коефiцiєнта фiльтрацiї

E. Збiльшення ниркового кровотоку

Введення знеболюючого пацiєнту перед екстракцiєю зуба призвело до розвитку анафiлактичного шоку, який супроводжувався розвитком олiгурiї. Який патогенетичний механiзм зумовив зменшення дiурезу в данiй клiнiчнiй ситуацiї?

A. Зниження гiдростатичного тиску в капiлярах клубочкiв

B. Пiдвищення гiдростатичного тиску в капсулi Шумлянського-Боумена

C. Пошкодження клубочкового фiльтру

D. Збiльшення онкотичного тиску кровi

E. Зменшення кiлькостi функцiонуючих нефронiв

У хворого знижений синтез вазопресину, що призводить до полiурiї i, як наслiдок, до вираженої дегiдратацiї органiзму. У чому полягає механiзм розвитку полiурiї?

A. Зниження канальцевої реабсорбцiї води

B. Зниження канальцевої реабсорбцiї iонiв Na

C. Зниження канальцевої реабсорбцiї бiлку

D. Зниження реабсорбцiї глюкози

E. Збiльшення швидкостi клубочкової фiльтрацiї

У фiзично здорових молодих курсантiв пiсля важкого фiзичного навантаження при одноденному пiшому переходi на 50 км в сечi виявлено бiлок, рiвень якого в середньому не перевищував 1 г/л. Який рiзновид протеїнурiї мав мiсце?

A. Маршова

B. Дегiдратацiйна

C. Алiментарна

D. Органiчна

E. Несправжня

У жiнки 30-ти рокiв виникли набряки обличчя. При обстеженнi виявленi протеїнурiя (5,87 г/л), гiпопротеїнемiя, диспротеїнемiя, гiперлiпiдемiя. Для якого стану характерно таке поєднання симптомiв?

A. Нефротичний синдром

B. Нефритичний синдром

C. Хронiчний пiєлонефрит

D. Гостра ниркова недостатнiсть

E. Хронiчна ниркова недостатнiсть

У юнака 20-ти рокiв, через 2 тижнi пiсля перенесеної лакунарної ангiни, з’явилися скарги на загальну слабкiсть, набряки пiд очима. Пiсля обстеження хворому встановлено дiагноз: гострий гломерулонефрит. Якi патологiчнi змiни у складi сечi найбiльш вiрогiднi?

A. Протеїнурiя

B. Цилiндрурiя

C. Наявнiсть свiжих еритроцитiв

D. Пiурiя

E. Натрiйурiя

У населеному пунктi зареєстрований спалах гепатиту, який зв’язують з водним фактором. Який вiрус гепатиту мiг викликати спалах захворювань у цьому населеному пунктi?

A. E

B. C

C. D

D. G

E. B

У хворого 49-ти рокiв на гострий панкреатит виникала загроза некрозу пiдшлункової залози, що супроводжувалось надходженням у кров i тканини активних панкреатичних протеїназ i розщеплення тканинних бiлкiв. Якi захиснi фактори органiзму можуть iнгiбувати цi процеси?

A. α2 -макроглобулiн, α1 -антитрипсин

B. Iмуноглобулiни

C. Крiоглобулiн, iнтерферон

D. Церулоплазмiн, трансферин

E. Гемоплексин, гаптоглобiн

Хворому з гiперсекрецiєю шлункового соку лiкар рекомендував виключити з дiєти насиченi бульйони i овочевi вiдвари, тому що вони стимулюють шлункову секрецiю переважно через активацiю:

A. Вироблення гастрину

B. Смакових рецепторiв

C. Механорецепторiв ротової порожнини

D. Механорецепторiв шлунка

E. Вироблення секретину

У хворого нормально забарвлений кал, у складi якого є велика кiлькiсть вiльних жирних кислот. Причиною цього є порушення:

A. Всмоктування жирiв

B. Гiдролiзу жирiв

C. Жовчовидiлення

D. Жовчоутворення

E. Секрецiї лiпаз

Пацiєнт звернувся зi скаргами на гострий бiль у правому пiдребер’ї. При оглядi лiкар звернув увагу на пожовтiння склер хворого. Лабораторно: пiдвищена активнiсть АлАТ та негативна реакцiя на стеркобiлiн в калi. Для якого захворювання характернi такi симптоми?

A. Гепатит

B. Гемолiтична жовтяниця

C. Хронiчний гастродуоденiт

D. Хронiчний колiт

E. Хронiчний гастрит

Призначення доксициклiну гiдрохлориду викликало порушення симбiозу мiкробної флори в кишечнику. Визначити тип порушень при антибiотикотерапiї:

A. Дисбактерiоз

B. Сенсибiлiзацiя

C. Iдiосинкразiя

D. Суперiнфекцiя

E. Бактерiоз

У хворого на хронiчний гепатит виявлено значне зниження синтезу I секрецiї жовчних кислот. Який процес у найбiльшiй мiрi буде порушений у кишечнику цього хворого?

A. Емульгування жирiв

B. Травлення бiлкiв

C. Травлення вуглеводiв

D. Всмоктування глiцерину

E. Всмоктування амiнокислот

У людини порушено всмоктування продуктiв гiдролiзу жирiв. Причиною цього може бути дефiцит у порожнинi тонкої кишки:

A. Жовчних кислот

B. Жовчних пiгментiв

C. Лiполiтичних ферментiв

D. Iонiв натрiю

E. Жиророзчинних вiтамiнiв

Хворий пiсля вживання жирної їжi вiдчуває нудоту, млявiсть; з часом з’явилися ознаки стеатореї. У кровi холестерин - 9,2 ммоль/л. Причиною такого стану є нестача у кишечнику:

A. Жовчних кислот

B. Триглiцеридiв

C. Жирних кислот

D. Фосфолiпiдiв

E. Хiломiкронiв

Чоловiк 53-х рокiв звернувся зi скаргами на гострий бiль у правому пiдребер’ї. При оглядi лiкар звернув увагу на пожовтiлi склери хворого. Лабораторнi аналiзи показали пiдвищену активнiсть АЛТ та негативну реакцiю на стеркобiлiн у калi. Для якого захворювання характернi такi симптоми?

A. Жовчнокам’яна хвороба

B. Гемолiтична жовтяниця

C. Гепатит

D. Хронiчний колiт

E. Хронiчний гастрит

У юнака 16-ти рокiв пiсля перенесеного захворювання знижена функцiя синтезу бiлкiв у печiнцi внаслiдок нестачi вiтамiну K. Це може призвести до порушення:

A. Зсiдання кровi

B. Швидкостi осiдання еритроцитiв

C. Утворення антикоагулянтiв

D. Утворення еритропоетинiв

E. Осмотичного тиску кровi

При лабораторному дослiдженнi у хворого виявили стеаторею. Вкажiть фермент, недостатнiсть дiї якого призвела до виникнення цього симптому:

A. Лiпаза

B. Амiлаза

C. Пепсин

D. Лактаза

E. Хiмотрипсин

У новонародженого спостерiгається диспепсiя пiсля годування молоком. При замiнi молока розчином глюкози симптоми диспепсiї зникають. Недостатня активнiсть якого ферменту спостерiгається у новонародженого?

A. Лактаза

B. Сахараза

C. Мальтаза

D. Амiлаза

E. Iзомальтаза

При копрологiчному дослiдженнi встановлено, що кал знебарвлений, у ньому знайдено краплi нейтрального жиру. Найбiльш вiрогiдною причиною цього є порушення:

A. Надходження жовчi до кишечнику

B. Кислотностi шлункового соку

C. Секрецiї пiдшлункового соку

D. Секрецiї кишкового соку

E. Процесiв всмоктування в кишечнику

Пiсля вживання жирної їжi у хворого з’являються нудота та печiя, має мiсце стеаторея. Причиною такого стану може бути:

A. Нестача жовчних кислот

B. Пiдвищене видiлення лiпази

C. Порушення синтезу трипсину

D. Нестача амiлази

E. Порушення синтезу фосфолiпази

Хворий вiдзначає частi проноси, особливо пiсля вживання жирної їжi, схуднення. Лабораторнi дослiдження показали наявнiсть стеатореї; кал гiпохолiчний. Що може бути причиною такого стану?

A. Обтурацiя жовчних шляхiв

B. Запалення слизової оболонки тонкої кишки

C. Недостатнiсть панкреатичної лiпази

D. Недостатнiсть панкреатичної фосфолiпази

E. Незбалансована дiєта

Хворому на хронiчний гастрит зроблена внутрiшньошлункова рН-метрiя, за допомогою якої встановлено зниження кислотностi шлункового соку. Функцiя яких клiтин знижена?

A. Парiєтальнi екзокриноцити

B. Головнi екзокриноцити

C. Ендокриноцити

D. Шийковi клiтини

E. Додатковi клiтини

Пiсля переходу до змiшаного харчування у новонародженої дитини виникла диспепсiя з дiареєю, метеоризмом, вiдставанням у розвитку. Бiохiмiчна основа даної патологiї полягає у недостатностi:

A. Сахарази та iзомальтази

B. Лактази та целобiази

C. Трипсину та хiмотрипсину

D. Лiпази та креатинкiнази

E. Целюлази

До складу харчових рацiонiв обов’язково входять продукти, в яких є клiтковина. Вiдомо, що вона не перетравлюється ферментами травного тракту й не засвоюється органiзмом. Яку роль вiдiграє ця речовина?

A. Стимулює моторну функцiю травного каналу

B. Гальмує моторну функцiю травного каналу

C. Гальмує всмоктувальну функцiю травного каналу

D. Гальмує секреторну функцiю травного каналу

E. Гальмує процеси видiлення ферментiв травних сокiв

У хворого 30-ти рокiв iз гострим запаленням пiдшлункової залози (панкреатитом) виявлено порушення порожнинного травлення бiлкiв. Це може бути пов’язано iз недостатнiм синтезом та видiленням залозою такого ферменту:

A. Трипсин

B. Пепсин

C. Лiпаза

D. Дипептидаза

E. Амiлаза

Жiнка 45-ти рокiв декiлька рокiв хворiє на системний червоний вовчак у легкiй формi. При прогресуваннi захворювання (з’явився мiокардит) їй призначили преднiзолон як iмуносупресор. Через 2 мiсяцi прийому у хворої виникла шлункова кровотеча. Яка найбiльш iмовiрна її причина?

A. Ульцерогенна дiя

B. Зменшення згортання кровi

C. Пiдвищення артерiального тиску

D. Подальше прогресування захворювання

E. Збудження ЦНС

Проводять дуоденальне зондування. Що iз наведеного доцiльно ввести людинi пiд шкiру, щоб суттєво збiльшити надходження до дванадцятипалої кишки жовчi?

A. Холецистокiнiн-панкреозимiн

B. Гастрин

C. Секретин

D. Нейротензин

E. Соматостатин

 



#У лiкарню до кiнця робочого дня доставлений працiвник ”гарячого” цеху, який скаржиться на головний бiль, запаморочення, нудоту, загальну слабкiсть. Свiдомiсть збережена, шкiрнi покриви гiперемованi, сухi, гарячi на дотик. ЧСС- 130/хв. Дихання часте, поверхневе. Яке порушення процесiв регуляцiї тепла найiмовiрнiше виникло у людини у данiй ситуацiї?
+ Зниження тепловiддачi
- Посилення тепловiддачi i зниження теплопродукцiї
- Посилення тепловiддачi i теплопродукцiї
- Посилення теплопродукцiї без змiн тепловiддачi
- Зниження теплопродукцiї без змiни тепловiддачi


#Вiдомо, що типовi патологiчнi процеси розвиваються за однаковими закономiрностями в рiзних органах i тканинах та у рiзних видiв тварин. Яке з перерахованих явищ можна вiднести до типового патологiчного процесу?
+ Пухлина
- Туберкульоз
- Гiпертонiчна хвороба
- Непрохiднiсть кишкiвника
- Iнфаркт міокарда


#Людина, яка проживала в ендемiчному вогнищi, перехворiла триденною малярiєю. Через пiвтора року пiсля переїзду в iншу мiсцевiсть захворiла малярiєю знову. Яка найбiльш вiрогiдна форма цього захворювання?
+Рецидив
-Реiнфекцiя
-Суперiнфекцiя
-Персистуюча iнфекцiя
-Вторинна iнфекцiя


#У хворого лiкар дiагностував гостру гонорею. З анамнезу стало вiдомо, що ранiше вiн перенiс гонорею i вилiкування було повним. До якої категорiї iнфекцiй можна вiднести це нове захворювання?
+Реiнфекцiя
-Суперiнфекцiя
-Рецидив
-Вторинна iнфекцiя
-Автоiнфекцiя


#У хворого вiдзначається атрофiя альвеолярних вiдросткiв щелепи пiсля видалення зубiв. Це є прикладом:
+Патологiчного стану
-Патологiчної реакцiї
-Патологiчного процесу
-Структурного слiду адаптацiї
-Хвороби


#При термометрiї встановлено, що температура вiдкритих дiлянок шкiри на 1-1,5o нижче за температуру поруч розташованих дiлянок, закритих одягом з натуральних тканин. Причиною цього є те, що одяг, перш за все, зменшує тепловiддачу таким шляхом:
+Конвекцiя
-Радiацiя
-Проведення
-Випаровування
-Немає правильної відповіді


#Вiддачу тепла яким шляхом збiльшують люди, якi приймають прохолодний душ у спекотнi днi?
+Теплопроведення
-Конвекцiя
-Випромiнювання
-Випаровування поту
-Немає правильної відповіді


#У хворого виявлені такі зміни в крові:еритроцити - 2,8 Т/л, гемоглобін - 60 г/л, КП - 0,64, ретикулоцити - 0,1%, лейкоцити - 8,7 Г/л; в мазку: мікроцитоз і пойкілоцитоз. Залізо сироватки крові 4,5 мкмоль/л. Яка анемія у хворого?
+Залізодефіцитна
-Залізорефрактерна
-Метапластична
-Гіпопластична
-Гемолітична


#У робітника, який працював літом у щільному костюмі, різко підвищилась температура тіла, з’явились задишка, тахікардія, нудота, судоми, втрата свідомості. Що явилось причиною тяжкого стану робітника?
+ Зниження тепловіддачі
-Підвищення теплопродукції
-Підвищення тепловіддачі
-Зниження теплопродукції
-Тепловіддача дорівнює теплопродукції


#Охолодження тіла людини у воді виникає значно швидше, ніж на повітрі, тому, що у воді значно ефективнішою є віддача тепла шляхом:
+Теплопроведення
-Випаровування поту
-Конвекції
-Тепловипромінювання


#Людина стоїть у кімнаті в легкому одязі; температура повітря +14°C. Вікна і двері зачинені. Яким шляхом організм людини віддає найбільше тепла?
+Теплорадіація
-Випаровування
-Конвекція
-Перспірація
-Теплопроведення


#У холодну погоду з вітром люди мерзнуть швидше, ніж за відсутності вітру. Причиною цього є те, що вітер збільшує, насамперед, віддачу тепла таким шляхом:
+Конвекція
-Випаровування
-Радіація
-Теплопроведення


#Ураження хворого одноразовою дозою iонiзуючого випромiнювання спричинило розвиток кiстковомозкової форми променевої хвороби. Якi патологiчнi прояви з боку кровi будуть характерними в перiод удаваного благополуччя?
+Наростаюча лiмфопенiя, лейкопенiя
-Перерозподiльчий лейкоцитоз, лiмфоцитоз
-Анемiя, лейкопенiя
-Тромбоцитопенiя, анемiя
-Тромбоцитопенiя, лейкоцитоз


#У результаті радіаційного випромінювання ушкоджені стовбурові гемопоетичні клітини. Утворення яких клітин сполучної тканини буде порушено?
+Макрофаги
-Періцити
-Фібробласти
-Меланоцити
-Адіпоцити


#Пiд час роботи щодо лiквiдацiї наслiдкiв аварiї на АЕС, робiтник одержав дозу опромiнення 500 рентген. Скаржиться на головний бiль, нудоту, запаморочення. Якi змiни кiлькостi лейкоцитiв можна очiкувати в хворого через 10 годин пiсля опромiнення?
+Нейтрофiльний лейкоцитоз
-Лiмфоцитоз
-Лейкопенiя
-Агранулоцитоз
-Лейкемiя


#У лiквiдатора наслiдкiв аварiї на АЕС пiд час перебiгу гострої променевої хвороби виник геморагiчний синдром. Що має найбiльше значення в патогенезi цього синдрому?
+Тромбоцитопенiя
-Порушення структури стiнки судин
-Пiдвищення активностi факторiв фiбринолiзу
-Пiдвищення активностi факторiв систем протизсiдання кровi
-Зменшення активностi факторiв зсiдання кровi


#Для людини iснує суворе обмеження в часi перебування на висотi понад 800 метрiв над рiвнем моря без кисневих балонiв. Що є лiмiтуючим фактором для життя в даному випадку?
+Парцiальний тиск кисню в повiтрi
-Рiвень ультрафiолетового опромiнення
-Рiвень вологостi
-Температура
-Сила земного тяжiння


#У періоді розпалу гострої променевої хвороби у хворого спостерігалися лейкопенія, тромбоцитопенія, аутоінфекція, аутоинтоксикация, кровоточивість, підвищення температури тіла Для якої форми променевої хвороби характерна дана картина?
+Кістково-мозкової
-Кишкової
-Токсемічної
-Церебральної
-Геморагічної


#Рoбітника АЕС доставили у клініку після одноразового опромінення зі скаргами на слабкість, головний біль, підвищення температури, діарею. В аналізі крові – лейкоцитоз з лімфопеніею. Яка стадія променевої хвороб найбільш ймовірна у паціента?
+Період первинних реакцій
-Період вдаваного мнимого благополуччя
-Період разгорнутої кліничної картини
-Латентний період
-Продромальний період


#У альпініста під час сходження на висоті 6 тис. м над рівнем моря виникла ейфорія, неадекватна оцінка обстановки, спостерігались галюцинації. Яка головна причина у розвитку цих ознак гірської хвороби?
+Пониження парціального тиску кисню у повітрі.
-Фізичне навантаження.
-Пониження атмосферного тиску
-Снігова офтальмія.
-Розширення повітря в лобних пазухах.


#У кроля після опромінення спостерігається III період кістково-мозкової форми гострої променевої хвороби. Ураження якої тканини є провідним у патогенезі розладів при цьому?
+Кровотворної.
-Кісткової.
-Нервової.
-Епітелію статевих залоз.
-Залозистого епітелію.


#Під час ліквідації аварії на ЧАЕС робітник отримав дозу опромінення. При його обстеженні виявлені такі дані: еритроцитів - 2*1012/л, ретикулоцитів - немає, гемоглобін -50 г/л, кількість лейкоцитів - 3*109/л, в лейкоцитарній формулі виявлена лімофпенія число тромбоцитів 85*109/л. Для якої стадії променевої хвороби характерні такі зміни?
+Період розпалу.
-Період первинних реакцій.
-Прихований період.
-Латентний період.
-Кінець хвороби.


#Альпініст на протязі кількох діб підіймався в гору. На висоті 5 000 метрів його стали непокоїти тахіпное, тахікардія, головний біль розпираючого характеру. Вкажіть можливі причини вказаних симптомів?
+Зниження парціального тиску кисню в повітрі
-Зниження барометричного тиску повітря
-Недостатня вентиляція легень
-Газова емболія
-Зниження температури повітря


#При роботі з радіоактивними речовинами співробітник внаслідок аварії отримав дозу загального опромінення 4 Гр. Скаржиться на головний біль, нудоту, запаморочення. Які зміни в складі крові можна очікувати у хворого через 10 годин після опромінення?
+Нейтрофільний лейкоцитоз
-Лимфоцитоз
-Лейкопенія
-Агранулоцитоз
-Нейтропенія


#Після занурення водолаза на глибину 60 м у нього з’явилися симптоми порушення функцій центральної нервової системи – збудження, ейфорія, ослаблення уваги, професійні помилки. Ці симптоми пов’язані з токсичною дією на нейрони
+Азоту
-Кисню
-Вуглекислого газу
-Аміаку
-Лактату


#У водолаза, який проводив роботи на великій глибині, при швидкому поверненні його в умови нормального атмосферного тиску з'явився біль в суглобах, свербіж шкіри, порушення зору, втрата свідомості. Як називається описане явище?
+Хвороба декомпресії
-Стан невагомості
-Синдром вибухової декомпресії
-Баротравма
-Гіпероксія


#При пошкодженні клітини іонізуючим випромінюванням вмикаються механізми захисту і адаптації. Який механізм відновлення порушеного внутрішньоклітинного гомеостазу реалізується при цьому?
+Активація антиоксидантної системи
-Активація Са-опосередкованих клітинних функцій
-Гіпертрофія мітохондрій
-Накопичення Na
+ в клітинах
-Пригнічення аденілатциклази


#Цiанiстий калiй є отрутою, вiд якої смерть органiзму наступає миттєво. На якi ферменти в мiтохондрiях дiє цiанистий калiй?
+Цитохромоксидаза (аа3)
-Флавiновi ферменти
-Цитохром 5
-НАД
+ - залежнi дегiдрогенази
-Цитохром Р-450


#Виділяють декілька груп молекуляр­них механізмів, які мають важливе значен­ня в патогенезі ушкодження клітин, що сприяє розвитку патології. Які процеси за­безпечують протеїнові механізми ушко­дження?
+Пригнічення ферментів
-Перекисне окиснення ліпідів
-Осмотичне розтягнення мембран
-Активація фосфоліпаз
-Ацидоз


#Внаслiдок активацiї iонних каналiв зовнiшньої мембрани збудливої клiти-ни значно збiльшився її потенцiал спокою. Якi канали були активованi?
+Калiєвi
-Натрiєвi
-Швидкi кальцiєвi
-Повiльнi кальцiєвi
-Натрiєвi та кальцiєвi


#При пошкодженнi клiтини iонiзуючим випромiнюванням вмикаються механiзми захисту i адаптацiї. Який механiзм вiдновлення порушеного внутрiшньоклiтинного гомеостазу реалiзується при цьому?
+Активацiя антиоксидантної системи
-Активацiя С a-опосередкованих клiтинних функцiй
-Накопичення N a
+ в клiтинах
-Пригнiчення аденiлатциклази
-Гiпертрофiя мiтохондрiй


#Піддослідній тварині ввели блокатор цитохромоксидази, що призвело до її миттєвої загибелі. Яка із сполук калію може викликати вказані зміни:
+Цианід
-Нітрит
-Сульфат
-Фосфат
-Оксалат


#По приїзді групи експертів на місце злочину виявлено тіло без ознак життя. В ході дослідження крові загиблого виявлена велика концентрація іонів ціанової кислоти. Інгібування якого комплексу дихального ланцюга мітохондрій стало причиною смерті?
+IV
-II
-III
-V



#При патологічних процесах, які супроводжуються гіпоксією, відбувається неповне відновлення молекули кисню в дихальному ланцюзі і накопичення пероксиду водню. Вкажіть фермент, який забезпечує його руйнування:
+Каталаза
-Аконітаза
-Кетоглутаратдегідрогеназа
-Сукцинатдегідрогеназа
-Цитохромоксидаза


#При пошкодженні клітини іонізуючим випромінюванням вмикаються механізми захисту і адаптації. Який механізм відновлення порушеного внутрішньоклітинного гомеостазу реалізується при цьому?
+Активація антиоксидантної системи
-Активація Са-опосередкованих клітинних функцій
-Гіпертрофія мітохондрій
-Накопичення Na
+ в клітинах
-Пригнічення аденілатциклази


#У жiнки 37-ми рокiв протягом року перiодично виникали iнфекцiйнi захворюваннябактерiальногогенезу,їхперебiг був вкрай тривалим, ремiсiї - короткочасними. При обстеженнi виявлена гiпогамаглобулiнемiя. Порушення функцiї яких клiтин може бути прямою її причиною?
+Плазматичнi клiтини
-Фагоцити
-Нейтрофiли
-Макрофаги
-Лiмфоцити


#Хлопчик на другому роцi життя став часто хворiти на респiраторнi захворювання, стоматити, гнiйничковi ураження шкiри. Навiть невеликi пошкодження ясен і слизової оболонки ускладнюються запаленням, що протiкає тривало. Встановлено, що у кровi дитини практично вiдсутнi імуноглобуліни всіх класiв. Зниження функцiональної активностi якої клiтинної популяцiї лежить в основi описаного синдрому?
+В-лiмфоцити
-Т-лiмфоцити
-Нейтрофiли
-Макрофаги
-NK-лiмфоцити


#До приймального вiддiлення iнфекцiйної лiкарнi надiйшов чоловiк 25-ти рокiв. Дiагноз: СНIД. Ураження яких клiтин зумовлює станi мунодефiциту?
+Т-хелпери
-Т-кiллери
-Т-супресори
-Плазмоцити
-Гладкi клітини (тканиннi базофiли)


#Хворий 47-ми рокiв впродовж останнiх 3-х рокiв хворiє на туберкульоз легень, скаржиться на задишку, важкiсть в областi правого боку грудної стiнки, температуру тiла 37,7oC. Виявлено правобiчний ексудативний плеврит. Який тип клiтин передбачається у плевральному пунктатi? +Лiмфоцити
-Нейтрофiли
-Еритроцити
-Атиповiклiтини
-Еозинофiли


#У хворих із синдромом набутого іму­нодефіциту (СНІД) різко знижується іму­нологічна реактивність, що проявляється розвитком хронічних запальних процесів, інфекційних захворювань, пухлинного ро­сту. Клітини якого типу ушкоджує ВІЛ- інфекція, внаслідок чого знижується імун­ний захист?
+Т4-хелпери
-Т8-ефектори
-Т-супресори
-Природні кілери (НК)
-В-лімфоцити


#У пацієнта з бронхіальною астмою за допомогою шкірних алергічних проб встановлено сенсибілізацію алергеном тополиного пуху. Який фактор імунної системи відіграє вирішальну роль в розвитку цього імунопатологічного стану?
+IgE
-IgD
-IgG
-Сенсибілізовані Т-лімфоцити
-IgM


#При відборі для ревакцинації вакциною БЦЖ у школяра поставлено пробу Манту, яка виявилася негативною. Результат проби свідчить про такі особливості імунітету до туберкульозу:
+Відсутність клітинного імунітету
-Відсутність антитоксичного імунітету
-Відсутність гуморального імунітету
-Наявність гуморального імунітету
-Наявність клітинного імунітету


#У хворого, що надійшов до хірургічного відділення з ознаками гострого апендициту, виявлені наступні зміни білої крові: загальна кількість лейкоцитів – 16*109/л. Лейкоцитарна формула: б.- 0, е.- 2%, ю.- 2%, п.- 8%, с.- 59%, л.- 25%, м.- 4%. Як класифікуються зазначені зміни?
+Нейтрофілія з регенеративним зсувом вліво
-Нейтрофілія з дегенеративним зсувом вліво
-Нейтрофілія з гіперрегенеративним зсувом вліво
-Лейкемоїдна реакція за нейтрофільним типом
-Нейтрофілія з зсувом вправо


#Дитині після аналізу імунограми встано­вили діагноз первинний імунодефіцит гу­моральної ланки імунітету. Яка з причин може призвести до розвитку первинного імунодефіциту в організмі дитини?
+Спадкові порушення в імунній системі
-Порушення обміну речовин в організмі матері
-Порушення в процесі ембріонального розвитку
-Порушення реактивності та резистен­тності організму
-Токсичне пошкодження В-лімфоцитів


#У дитини 5-ти рокiв дiагностовано хворобу Брутона, яка проявляється у важкому перебiгу бактерiальних iнфекцiй, вiдсутностi В-лiмфоцитiв та плазматичних клiтин. Якi змiни вмiсту iмуноглобулiнiв будуть спостерiгатися в сироватцi кровi цiєї дитини?
+Зменшення IgA, IgM
-Зменшення IgD, IgE
-Збiльшення IgD, IgE
-Змiн не буде


#У хворого з клiнiчними ознаками iмунодефiциту проведено iмунологiчнi дослiдження. Виявлено значне зниження кiлькостi клiтин, що утворюють розетки з еритроцитами барана. Який висновок слiд зробити на основi даних аналiзу?
+Зниження рiвня T -лiмфоцитiв
-Зниження рiвня B-лiмфоцитiв
-Зниження рiвня натуральних кiлерiв
-Зниження рiвня системи комплементу
-Недостатнiсть клiтин-ефекторiв гуморального iмунiтет


#Досить часто причиною набутих iмунодефiцитiв є iнфекцiйне ураження органiзму, при якому збудники безпосередньо розмножуються в клiтинах iмунної системи i руйнують їх. Виберiть серед перерахованих тi захворювання, при яких має мiсце вищезгадане:
+Iнфекцiйний мононуклеоз, СНIД
-Туберкульоз, мiкобактерiоз
-Полiомiєлiт, гепатит А
-Дизентерiя, холера
-Ку-гарячка, висипний тиф


#У плазмi кровi здорової людини знаходиться декiлька десяткiв бiлкiв При захворюваннi органiзму з’являються новi бiлки, зокрема "бiлок гострої фази". Таким бiлком є
+С-реактивний бiлок
-Протромбiн
-Фiбриноген
-Iмуноглобулiн G
-Iмуноглобулiн А


#Хлопчик на другому роцi життя став часто хворiти на респiраторнi захворювання, стоматити, гнiйничковi ураження шкiри. Навiть невеликi пошкодження ясен i слизової ускладнюються запаленням, що протiкає три-вало. Встановлено, що у кровi дитини практично вiдсутнi iмуноглобулiни усiх класiв. Зниження функцiональної активностi якої клiтинної популяцiї лежить в основi описаного синдрому?
+В-лiмфоцити
-Т-лiмфоцити
-Нейтрофiли
-Макрофаги
-NK-лiмфоцити


#Недостатнiсть в органiзмi лiнолевої та лiноленової кислот призводить до ушкоджень шкiри, випадiння волосся, сповiльненого загоювання ран, тромбоцитопенiї, зниження опiрностi до iнфекцiйних захворювань. Порушення синтезу яких речовин найiмовiрнiше зумовлює вказанi симптоми?
+Ейкозаноїди
-Iнтерлейкiни
-Iнтерферони
-Катехоламiни
-Кортикостероїди


#При повторному введеннi алергену починається видiлення гiстамiну тучними клiтинами кровi. До якого рiвня реактивностi вiдноситься така вiдповiдь органiзму?
+Клiтинний
-Субклiтинний
-Молекулярний
-Органний
-Системний


#У хворого дiагностовано ГРВI. У сироватцi кровi знайдено iмуноглобулiни класу М. Який перiод iнфекцiйногопроцесу в даному випадку?
+Гострий
-Продромальний
-Iнкубацiйний
-Реконвалесценцiя
-Мiкробоносiйство


#У хворого спостерігається синдром Ді джорджі, в основі якого лежить гіпоплазія вилочкової залози До якої форми імунної патології належить це захворювання?
+Вроджений B дефіцит Т-лімфоцитів
-Вроджена дефіцит В-лімфоцитів
-Набутий дефіцит В-лімфоцитів
-Набутий дефіцит Т-лімфоцитів
-Iмунодепресія в системі Т-лімфоцитів


#Чоловік 25 років скаржиться на часто виникаючі запальні захворювання різної локалізації. Встановлено, що він - ін'єкційний наркоман. Проба на ВІЛ-інфекцію виявилась позитивною. Який з перелічених типів клітин імунної системи найбільш суттєво вражається ВІЛ?
+хелпери
-кілери
-нейтрофільні гранулоцити
-плазматичні клітини


#У хлопчика 5 міс., при дослідженні імунного статусу виявлено зменшення імуноглобулінів, особливо IgA та IgM. В крові та лімфатичних вузлах відсутні В-лімоцити та плазматичні клітини. Реакції Т-лімфоцитів збережені. Захворювання передається по спадковості як зчеплене із статтю. Яка патологія спостерігається у цієї дитини?
+Хвороба Бутона
-Синдром Луі-Барр
-Синдром Віскотта-Олдрича
-Імунодефіцит швейцарського типу
-Рання гіпогаммаглобулінемія


#У ВІЧ-інфікованого хворого спостерігається пригнічення активності імунної системи. Ураження яких клітин найбільшою мірою обумовлює стан імунодефіциту у цього хворого?
+Т-хелперів
-Т-супресорів
-Макрофагів
-В-лімфоцитів
-Т-кілерів


#При дослідженні стану імунної системи хворого із хронічними грибковими ураженнями шкіри виявлено порушення клітинного імунітету. Зниження яких показників найбільш характерні при цьому?
+Т-лімфоцитів
-Імуноглобулінів G
-Імуноглобулінів E
-В-лімфоцитів
-Плазмоцитів


#Хлопчик 1,5 років постійно хворіє на піодермію та тричі хворів на пневмонію. В крові знижена кількість имуноглобулінів G та A. Який вид імунодефіциту виник у дитини?
+Гіпогаммаглобулінемія Брутона
-Швейцарський тип
-Гіпоплазія вилочкової залози
-Синдром Віскотта –Олдрича
-Синдром Луи-Барр


З метою пригнічення аутоімунних реакцій після пересадки органів обов'язковим є проведення курсу гормонотерапії. Які гормони застосовують з цієї мети?
+Глюкокортикоїди.
-Мінералокортикоїди.
-Статеві гормони.
-Адреналін.
-Соматотропний гормон.


#У дитини двох років встановлено діагноз гіпоплазії тимуса. Який показник стану імунної системи є найбільш характерним для цього імунодефіцита?
+Зниження кількості Т-лімфоцитів
-Зниження кількості В-лімфоцитів
-Дефіцит Т и В-лімфоцитів
-Відсутність плазматичних клітин
-Зниження іммуноглобулінів М


#У новонароджених щурят в експерименті була видалена вилочкова залоза. При цьому розвинулася хвороба, яка характеризується різким зниженням в крові лімфоцитів, розвитком інфекцій, спленамегаліей, зупинкою росту і летальним результатом. Яке порушення функції імунної системи при цьому спостерігається?
+Недостатність системи Т - лімфоцитів
-Недостатність системи В - лімфоцитів
-Гіперфункція системи Т - лімфоцитів
-Гіперфункція системи В - лімфоцитів
-Комбінований дефект Т - і В – лімфоцитів


#У хворого 34-х років після перенесеної кишкової інфекції, викликаної сальмонелами, стали згасати симптоми захворювання. Імуноглобуліни якого класу будуть виявлені в крові хворого в період реконвалесценції?
+IgG
-IgA
-IgD
-IgE
-IgM


#У сироватці крові новонародженого виявлено антитіла до вірусу кору. Про наявність якого імунітету це може свідчити?
+Природний пасивний
-Природний активний
-Спадковий, видовий
-Штучний активний
-Штучний пасивний


#У людини під дією мутагенного фактору з’явилась велика кількість мутантних клітин. Але більшість з них були розпізнані і знищені клітинами:
+Т-лімфоцитами кілерами
-В -лімфоцитами
-Плазмобластами
-Стовбуровими
-Т-лімфоцитами супресорами


#В дитячому садку проведені планові щеплення вакциною проти кору. Яким методом можна перевірити формування післявакцинального імунітету?
+Серологічний
-Алергічний
-Бактеріологічний
-Бактеріоскопічний
-Вірусологічний


#Дитина 6-ти рокiв пiд час гри порiзала ногу осколком скла i була направлена у полiклiнiку для введення протиправцевої сироватки. З метою попередження розвитку анафiлактичного шоку лiкувальну сироватку вводили за методом Безредка. Який механiзм лежить в основi подiбного способу гiпосенсибiлiзацiї органiзму?
+Зв’язування фiксованих на тучних клiтинах IgE
-Блокування синтезу медiаторiв у тучних клiтинах
-Стимуляцiя iмунологiчної толерантностi до антигену
-Стимуляцiя синтезу антигенспецифiчних IgG
-Зв’язування рецепторiв до IgE на тучних клітинах


#У пацiєнтки 23-х рокiв пiсля використання нової губної помади з’явилися набряк i свербiння губ, а через 2 днi - кiрочки на червонiй облямiвцi губ. Який тип алергiчної реакцiї найбiльш iмовiрний?
+Анафiлактичний
-Цитотоксичний
-Iмунокомплексний
-Сповiльнений
-Стимулюючий


#У хворого переливання кровi ускладнилося розвитком гемотрансфузiйного шоку. Назвiть тип алергiчної реакцiї, що лежить в основi даної патологiї:
+Цитотоксичний
-Анафiлактичний
-Iмунокомплексний
-Гiперчутливiсть сповiльненого типу
-Рецептороопосередкований


#У чоловіка 36-ти років після перенесеної стрептококової інфекці діагностовано гострий гломерулонефрит. Найбільш імовірно, що ураження базальної мембрани ниркових тілець виникає внаслідок алергічної реакції такого типу:
+Імунокомплексна
-Стимулююча
-Сповільнена
-Анафілактична
-Цитотоксична


#У дівчинки 18-ти років через 5 годин пі­сля вживання морепродуктів на шкірі тулу­ба та дистальних відділів кінцівок з’явились маленькі сверблячі папули, які частиною зливаються між собою. Через добу виси­пка самовільно зникла. Назвіть механізм гіперчутливості, що полягає в основі даних змін:
+Атопія (місцева анафілаксія)
-Імунокомплексна гіперчутливість
-Системна анафілаксія
-Антитілоопосередкований клітинний цитоліз
-Клітинна цитотоксичність


#При підозрі на туберкульоз хворій ди­тині зробили пробу Манту. Через 24 години у місці введення алергену з’явились припу­хлість, гіперемія і болісність. Які основні компоненти визначають цю реакцію орга­нізму?
+Мононуклеари, Т-лімфоцити і лімфокіни
-В-лімфоцити, IgM
-Гранулоцити, Т-лімфоцити і IgG
-Плазматичні клітини, Т-лімфоцити і лімфокіни
-Макрофаги, В-лімфоцити і моноцити


#Який стан може розвинутися через 15- 30 хвилин після повторного введення ан­тигену внаслідок підвищеного рівня анти­тіл, переважно IgE, які адсорбуються на поверхні клітин-мішеней - тканинних базо­філів (тучних клітин) та базофілів крові?
+Анафілаксія
-Імунно-комплексна гіперчутливість
-Антитіло-залежна цитотоксичність
-Сироваткова хвороба
-Гіперчутливість уповільненого типу


#Медсестра зі стажем роботи 10 років захворіла на контактний дерматит верхніх кінцівок. До якого типу імунної патології відноситься це захворювання?
+Алергічна реакція сповільненого типу
-Алергічна реакція негайного типу
-Первинний імунодефіцит
-В-клітинний імунодефіцит
-Т-клітинний імунодефіцит


#Дитина 6-ти років знаходиться на стаціонарному лікуванні з діагнозом алергічного риніту. В крові: зміни в лейкоцитарній формулі. Кількість яких клітин лейкоцитарного ряду може бути збільшена?
+Еозинофіли
-В-лімфоцити
-Нейтрофіли
-Базофіли
-Т-лімфоцити


#Під час проведення хірургічних мані­пуляцій було використано новокаїн з ме­тою знеболення. Через 10 хвилин у хво­рого з’явилася блідість шкірних покривів, задишка, гіпотензія. Алергічну реакцію якого типу можна запідозрити?
+Aнафілактична
-Стимулююча
-Клітинно-опосередкована
-Імунокомплексна
-Цитотоксичнa


#У дитини 10-ти років поставлено пробу Манту (з туберкуліном). Через 48 годин на місці введення туберкуліну з'явилаcя папула розміром до 8 мм у діаметрі. Який тип реакції гіперчутливості розвинувся після введення туберкуліну?
+Реакція гіперчутливості IV типу
-Реакція типу сироваткової хвороби
-Реакція типу феномен Артюса
-Реакція гіперчутливості ІІ типу
-Атопічна реакція


#У хворого виявлена аутоімунна гемолітична анемія, що розвивається за цитотоксичним типом. Які речовини є антигенами при алергічних реакціях ІІ типу?
+Модифіковані рецептори клітинних мембран
-Модулятори запалення
-Сироваткові білки
-Гормони
-Антибіотики


#Пiсля проведення туберкулiнової проби (проба Манту) у дитини через 48 годин на мiсцi ведення туберкулiну утворилася папула до 10 мм у дiаметрi. Який механiзм гiперчутливостi лежить в основi розвитку вказаних змiн?
+Клiтинна цитотоксичнiсть
-Анафiлаксiя
-Антитiлозалежна цитотоксичнiсть
-Iмунокомплексна цитотоксичнiсть
-Гранулематоз


#На 8-й день пiсля введення протиправцевої сироватки з приводу брудної рани стопи у пацiєнта пiдвищилася температура тiла до 380 , з’явилися бiль у суглобах, висипка, свербiж. У кровi - лейкопенiя i тромбоцитопенiя. Який тип алергiчної реакцiї розвинувся?
+Iмунокомплексна
-Анафiлактична
-Цитотоксична
-Гiперчутливiсть уповiльненого типу
-Стимулююча


#До лiкаря звернулася хвора зi скаргами на нежить, який посилюється навеснi в перiцвiтiння рослин. Було встановлено дiагноз алергiйного ринiту. Якi змiни лейкоцитарної формули можна очiкувати в аналiзi кровi цiєї хворої?
+ Еозинофiлiя
-Зсув формули влiво
-Лiмфопенiя
-Еозинопенiя
-Лiмфоцитоз


#Хвора 27-ми рокiв закрапала в очi краплi, до складу яких входить пенiцилiн. Через декiлька хвилин з’явився свербiж та печiння тiла, набряк губ та повiк, свистячий кашель; став падати артерiальний тиск. Якi iмуноглобулiни беруть участь в розвитку даної алергiчної реакцiї?
+I gE та I gG
-I gM та I gG
-I gA та I gM
-I gM та I gD
-I gG та I gD


#Пiсля тижневого застосування нового косметичного засобу у жiнки розвинулося запалення повiк з гiперемiєю, iнфiльтрацiєю та болючiстю. Алергiчна реакцiя якого типу розвинулася у пацiєнтки?
+IV
-I
-II
-III
-V


#Експериментальнiй тваринi, пiсля попередньої сенсибiлiзацiї, пiдшкiрно введено дозу антигену. У мiсцi iн’єкцiї розвинулось фiбринозне запалення з альтерацiєю стiнок судин, основної речовини та волокнистих структур сполучної тканини у виглядi мукоїдного та фiбриноїдного набухання i некрозу. Яка iмунологiчна реакцiя має мiсце?
+Гiперчутливiсть негайного типу
-Гiперчутливiсть сповiльненого типу
-Реакцiя трансплантацiйного iмунiтету
-Нормергiчна реакцiя
-Гранульоматоз


#У хворої на дифтерiю дитини через10 днiв пiсля введення антитоксичної протидифтерiйної сироватки з’явилася висипка на шкiрi, яка супроводжува-лася сильним свербежем, пiдвищенням температура тiла до 38oC, появою бо-лю у суглобах. Яку причину цих явищ можна припустити?
+Сироваткова хвороба
-Анафiлактична реакцiя
-Атопiя
-Гiперчутливiсть уповiльненого типу
-Контактна алергiя


#У пацiєнта з бронхiальною астмою за допомогою шкiрних алергiчних проб встановлено сенсибiлiзацiю алергеном тополиного пуху. Який фактор iмунної системи вiдiграє вирiшальну роль в розвитку цього iмунопатологiчного стану?
+IgE
-IgD
-IgM
-Сенсибiлiзованi Т-лiмфоцити
-IgG


#У чоловiка 36-ти рокiв пiсля перенесеної стрептококової iнфекцiї дiагностовано гострий гломерулонефрит. Найбiльш iмовiрно, що ураження базальної мембрани ниркових тiлець виникає внаслiдок алергiчної реакцiї такого типу:
+Iмунокомплексна
-Анафiлактична
-Цитотоксична
-Сповiльнена
-Стимулююча


#При розтинi хворої 28-ми рокiв, що померла вiд уремiї, виявленi збiльшенi строкатi нирки з осередками крововиливiв. Патогiстологiчно в судинних клубочках виявленi гематоксилiновi тiльця, капiлярнi мембрани клубочкiв у виглядi дротяних петель, гiалiновi тромби та осередки фiбриноїдного некрозу. За патогенезом гiперчутливiсть якого типу лежить в основi описаної хвороби?
+Гiперчутливiсть III типу (iмунокомплексна)
-Гiперчутливiсть I типу (анафiлактична)
-Гiперчутливiсть II типу (антитiлозалежна)
-Гiперчутливiсть IV типу (клiтинна цитотоксичнiсть)
-Гiперчутливiсть V типу (гранулематоз)


#Хвора 37-ми рокiв померла пiд час нападу експiраторної задухи, що був спричинений контактом з екзогенним алергеном (пилок амброзiї). При гiстологiчному дослiдженнi в просвiтi бронхiв спостерiгаються скупчення слизу, в стiнцi бронхiв багато тучних клiтин (лаброцитiв), бiльшiсть з яких у станi дегрануляцiї, багато еозинофiлiв. До патогенезу якого типу реакцiй гiперчутливостi можна вiднести описанi змiни?
+I типу (анафiлактична)
-II типу (антитiлозалежна)
-II типу (iмунокомплексна)
-IV типу (клiтинна цитотоксичнiсть)
-V типу (гранулематоз)


#У хворого діагностовано тиротоксикоз. У крові знайдено антитиреоїдні антитіла. Який тип алергічної реакції за Кумбсом і Джелом спостерігається при розвитку цього захворювання?
+Стимулюючий
-Анафілактичний
-Цитотоксичний
-Імунокомплексний
-Гіперчутливість сповільненого типу


#У хворого через 9 діб після введення лікувальної сироватки зўявилася кропивўянка, зуд шкіри, набряк її та слизових оболонок, припухання лімфатичних вузлів. Яке захворювання розвинулось?
+Сироваткова хвороба.
-Феномен Швартцмана.
-Набряк Квінке.
-Феномен Овері.
-Поліноз.


#Жінка 27 років звернулась зі скаргами на свербіння та печію в очах, сльозотечію, чхання, виділення з носу. Симптоми з'явились після поїздки за місто влітку. Діагностовано поліноз. Якого типу алергічна реакція розвилась при наявному захворюванні?
+анафілактична
-цитотоксична
-реакція утворення імунних комплексів
-сповільненої чутливості
-стимулююча


#У жінки 45 років, в період цвітіння трав з'явилось гостре запальне захворювання верхніх дихальних шляхів та очей: гіперемія, набряк, слизове виділення. Який вид лейкоцитозу буде найбільш характерним при цьому?
+Еозінофілія.
-Базофілія.
-Нейтрофілія.
-Лімфоцитоз.
-Моноцитоз.


#У студентки, 20 років, третю весну підряд з початком цвітіння тополі починається свербіння і гіперемія очей, носа, ринорея, покашлювання, дрібне уртикарне висипання відкритих ділянок тіла. При обстеженні виявлено різке збільшення рівня Ig E. Алергологом призначена специфічна гіпосенсибілізуюча терапія. Вкажіть, яким чином проводять дане лікування?
+Багаторазовим введенням малих доз алергену
-Однократним введенням великої дози алергену
-Введенням антигістамінних препаратів
-Введенням гормонів наднирнииків
-Введенням хлористого кальцію.


#У хворого А. в серпні після праці на дачі развився стан, що характеризувався лікарем, як стан підвищенної і якісно зміненої реакції на надходження до організму сполук антигенної або гаптенної природи. Який з перерахованих станів найбільш підходить під описану лікарем характеристику?
+алергія
-анафілаксія
-параалергія
-тахіфілаксія
-імунологічна толерантність


#Хворому М. з метою введення лікувальної дози протиправцевої сироватки було зроблено пробу на чутливість, яка виявилась позитивною. Як найбільш правильно провести специфічну гіпосенсибілізацію хворому? Введенням:
+малих доз протиправцевої сироватки
-глюкокортикоїдів
-дозволеної дози протиправцевої сироватки
-антигістамінних препаратів
-імунодепресантів


#Через 5-8 днів після застосування значних кількостей лікувальної сироватки у хворого було виявлено шкірні висипання, свербіж, припухлість, біль в суглобах, підвищилася температура тіла, в сечі з'явився білок. Був поставлений діагноз сироваткова хвороба Що є важливим фактором у патогенезі цього синдрому?
+накопичення в крові циркулюючих імунних комплексів.
-дегрануляция тканинних базофілів.
-активація Т-кілерів.
-активація макрофагів
-цитолиз формених елементів крові


#Після прийому амідопірину у хворого виникла лейкопенія. В крові знайдені антилейкоцитарні антитіла. Який тип алергічної реакції за Кумбсом і Джеллом виник у даному випадку?
+Цитотоксичний.
-Стимулюючий.
-Анафілактичний.
-Гіперчутливість сповільненого типу.
-Імунокомплексний.


#У мишей з відсутнім волосяним покривом (тобто nude - голі) не було клітинних реакцій уповільненого типу. Для цієї патології найбільш вірогідним є:
+Відсутність вилочкової залози
-Відсутність гамаглобулінів у крові
-Порушення гемопоезу
-Дефект фагоцитозу
-Дефіцит компонентів системи комплементу


#Масугі викликав розвиток гломерулонефриту у щурів таким чином: гомогенат нирок щура вводив кролю. Через декілька тижнів сироватку сенсибілізованого кроля вводив щурам. Який тип алергічної реакциї за Джеллом та Кумбсом лежить в основі розвитку гломерулонефриту у щурів?
+Цитотоксичний
-Анафілактичний
-Імунокомплексний
-Гіперчутливість сповільненого типу
-Стимулюючий


#Чоловіку 30 років було травмовано праве яєчко. Через 6 тижнів, коли запальний процес яєчка закінчився, з’явився біль у лівому яєчку. Об’єктивно: шкіра над яєчком червоного кольору, припухла, яєчко болісне на дотик, щільне. Який процес лежить в основі цього пошкодження?
+Аутоалергія.
-Параалергія.
-Гетероалергія.
-Феномен Артюса.
-Феномен Оверї


#Жінку 44-х років втяла оса, внаслідок чого розвинувся шок. В анамнезі - тяжка алергічна реакція на укус оси. Об’єктивно: РS- 179/хв, слабкий, АТ- 80/40 мм рт.ст., ЧД-26/хв. Яка провідна ланка патогенезу анафілактичного шоку?
+Зниження периферійного опору судин
-Біль
-Зменшення об’єму циркулюючої крові
-Зменшення ударного об’єму серця
-Тахікардія


#В основі розвитку імунних і алергічних реакцій організмом застосовуються однакові механізми відповіді імунної системи на антиген. Визначте основну відмінність алергічних реакцій від імунних:
+Розвиток пошкодження тканин
-Кількість антигену, що потрапляє
-Особливість будови антигенів
-Спадкова схильність
-Шляхи потрапляння антигенів до організму


#При розтині хворої 28-ми років, що померла від уремії, виявлені збільшені строкаті нирки з осередками крововиливів. Патогістологічно в судинних клубочках виявлені гематоксилінові тільця, капілярні мембрани клубочків у вигляді дротяних петель, гіалінові тромби та осередки фібриноїдного некрозу. За патогенезом гіперчутливість якого типу лежить в основі описаної хвороби?
+Гіперчутливість III типу (імунокомплексна)
-Гіперчутливість I типу (анафілактична)
-Гіперчутливість II типу (антитілозалежна)
-Гіперчутливість IV типу (клітинна цитoтоксичність)
-Гіперчутливість V типу (гранулематоз)


#Хвора 37-ми років померла під час нападу експіраторної задухи, що був спричинений контактом з екзогенним алергеном (пилок амброзії). При гістологічному дослідженні в просвіті бронхів спостерігаються скупчення слизу, в стінці бронхів багато тучних клітин (ла-броцитів), більшість з яких у стані де-грануляції, багато еозинофілів. До патогенезу якого типу реакцій гіперчу-тливості можна віднести описані зміни?
+I типу (анафілактична)
-II типу (антитілозалежна)
-III типу (імунокомплексна)
-IV типу (клітинна цитотоксичність)
-V типу (гранулематоз)


#Ліквідатору наслідків аварії на Чорнобильській АЕС, що отримав велику дозу опромінення, проведено трансплантацію кісткового мозку. Через деякий час після проведеної операції у пацієнта діагностовано розвиток реакції "трансплантат проти хазяїна’. Які антигени стали пусковим механізмом виникнення цієї реакції?
+Антигени системи HLA-клітин організму ліквідатора
-Антигенами системи AB0 еритроцитів ліквідатора
-Антигени HBs, HBc, Hbe
-Антигени системи HLA-клітин організму донора
-Антигени системи Rh еритроцитів ліквідатора


#В пiдводному човнi пiд час занурення порушилася система подачi кисню. У пiдводникiв збiльшилися частота дихання i серцевих скорочень. Який вид гiпоксiї розвинувся у пiдводникiв?
+Гiпоксична
-Кров’яна
-Серцево-судинна
-Тканинна
-Дихальна


#Вагiтна жiнка пiд час пологiв втратила близько 800 мл кровi. Спостерiгається тахiкардiя, артерiальний тиск 100/70 мм рт.ст., тахiпное до 28/хв. Який тип гiпоксiї розвивається первинно в такiй клiнiчнiй ситуацiї?
+Кров’яна
-Серцево-судинна
-Змiшана
-Тканинна
-Дихальна


#Робочий комунальної служби спустився в каналiзацiйний колодязь без засобiв захисту i через деякий час знепритомнiв. Лiкарями швидкої допомоги дiагностовано отруєння сiрководнем. Який вид гiпоксiї при цьому розвинувся?
+Гемiчний
-Перевантажувальний
-Тканинний
-Циркуляторний
-Респiраторний


#Після ремонту автомобіля в закритому приміщенні при працюючому двигуні у чоловіка з'явилися задишка, запаморочення, акроціаноз, частота дихання 24-26/хв. Газовий склад крові: pO2- 60 мм рт.ст., pCO2- 30 мм рт.ст.; у крові наявний карбоксигемоглобін. Про який вид гіпоксії можна думати?
+Гемічна
-Тканинна
-Циркуляторна
-Респіраторна
-Гіпоксична


#Для людини існує суворе обмеження в часі перебування на висоті понад 800 метрів над рівнем моря без кисневих балонів. Що є лімітуючим фактором для життя в даному випадку?
+Парціальний тиск кисню в повітрі
-Рівень ультрафіолетового опромінення
-Рівень вологості
-Температура
-Сила земного тяжіння


#Чоловік 65 років страждає хронічною серцевою недостатністю по лівошлуночковому типу. Об'єктивно: ціаноз, задишка, кашель з харкотинням, періодичні напади задухи. Який тип гіпоксії первинно виник у хворого ?
+циркуляторна застійна
-циркуляторна ішемічна
-тканинна
-дихальна
-кров'яна


#Чоловік 36 років скаржиться на кашель з виділенням харкотиння, задишку, головний біль, загальну слабкість. Захворів після сильного переохолодження. При огляді: шкіра бліда, температура тіла 38оС. Пульс- 91/хв., АТ-125/6 мм рт.ст. В аналізі крові - нейтрофільний лейкоцитоз. Поставлений -діагноз: вогнищева пневмонія. Який тип гіпоксії має місце у хворого?
+дихальна
-гемічна
-тканинна
-циркуляторна застійна
-циркуляторна ішемічна


#Чоловік приблизно 50 років, винесений в непритомному стані з закритого помешкання, повного диму від пожежі. Який вид гіпоксії виник у постраждавшого?
+гемічна
-дихальна
-гіпоксична
-тканинна
-циркуляторна


#У лікарню був доставлений водій, який після роботи заснув в машині з


працюючим двигуном. Прокинувшись він відчув головний біль, почалася


блювота. Що стало причиною симптомів?
+карбоксигемоглобин
-метгемоглобін
-карбгемоглобин
-дезоксигемоглобин
-сульфгемоглобін


#При підйомі на "висоту" у барокамері у щура з'явилося часте дихання, тахікардія, зниження напруги рО2 в крові. Яка форма гіпоксії має місце в даному випадку?
+Гіпоксична
-Гемічна
-Циркуляторна
-Тканинна
-Дихальна


#Після аварії на хімічному виробництві відбулося забруднення навколишнього середовища нітроcполуками. У частини людей, які проживають в цій місцевості, з'явилась різка слабкість, головний біль, задишка, запаморочення. Який механізм розвитку даної форми гіпоксії?
+Збільшення утворення метгемоглобіну
-Зниження функції флавинових ферментів
-Утворення карбоксигемоглобіну
-Інактивація цитохромоксидази
-Пригнічення дегидрогеназ


#У хворого внаслідок отруєння бертолетовою сіллю розвинулася гемічна гіпоксія. Утворення якої речовини грае роль в патогенезі цієї гіпоксиї?
+Метгемоглобіну
-Оксиду азоту
-Сульфгемоглобіну
-Карбгемоглобіну
-Карбоксігемоглобіну


#Хворий В. 38 років, доставлений в приймальне відділення з ознаками гіпоксії розвилася після отруєння чадним газом. Стан середньої тяжкості, тахікардія, задишка, АТ 160/100. Який механізм токсичної дії окису вуглецю на організм?
+Утворення карбоксигемоглобіну
-Утворення метгемоглобіну
-Порушення дисоціації оксигемоглобіну
-Освіта карбгемоглобина
-Блокада кальцієвих каналів еритроцитів


#У хворого під час нападу бронхіальної астми при визначенні рСО2 в крові


виявлено наявність гіперкапнії, при визначенні РО2 гіпоксемії Який вид гіпоксії cпостерігається в даному випадку?
+Дихальна
-Гемічна
-Циркуляторна
-Тканинна
-гістотоксичнa


#У хворого з варикозним розширенням вен пiд час огляду нижнiх кiнцiвок вiдзначається: цiаноз, пастознiсть, зниження температури шкiри, поодинокi петехiї. Який розлад гемодинамiки має мiсце у хворого?
+Венозна гiперемiя
-Компресiйна iшемiя
-Обтурацiйна iшемiя
-Тромбоемболiя
-Артерiальна гіперемія


#У чоловiка 48-ми років виявлено порушення периферичного кровообiгу з обмеженням припливу артерiальної кровi, при цьому має мiсце зблiднiння даної дiлянки, зниження мiсцевої температури. Це порушення називається:
+Iшемiя
-Стаз
-Сладж
-Венозна гiперемiя
-Реперфузiйний синдром


#У жінки 60-ти років після емоційної реакції, яка була викликана гнівом, виник напад загрудинного болю. На ЕКГ були встановлені ознаки порушення коронарно­го кровообігу. Який вид порушень міг спри­чинити це явище?
+Ангіоспастична ішемія
-Венозний стаз
-Артеріальна гіперемія
-Венозна гіперемія
-Справжній стаз


#Студент на екзамені не зміг вірно від­повісти на питання екзаменаційного біле­ту, що супроводжувалося почервонінням шкіри обличчя, відчуттям жару і невпев­неністю поведінки. Який вид артеріальної гіперемії розвинувся у даному випадку?
+Нейротонічна
-Метаболічна
-Постішемічна
-Нейропаралітична
-Патологічна


#Під час гри у волейбол спортсмен після стрибка приземлився на зовнішній край стопи. Виник гострий біль у гомілковостопному суглобі, активні рухи в ньому обмежені, пасивні - в повному обсязі, але болісні. Потім розвинулася припухлість у ділянці зовнішньої щиколотки, шкіра почервоніла, стала теплішою на дотик. Який вид розладу периферичного кровообігу розвинувся в даному випадку?
+Артеріальна гіперемія
-Стаз
-Емболія
-Тромбоз
-Венозна гіперемія


#У кроля перерiзалі нерв, что iннервує праві вухо, i Відаль правий верхнiй шийно симпатичний вузол. Одразу пiсля операцiї провели вимірювання температури шкiри вух. Виявило, что температура шкiри вуха кролика на боцi денервацiї на 1,5 C вища, нiж на протилежних iнтактному боцi. Що з наведеного є найбiльш вiрогiдною причиною вказаних явищ?
+Артерiальна гiперемiя нейропаралiтічного типу
-Aртерiальна гiперемiя нейротонiчного типу
-Артерiальна гiперемiя, обумовлена ​​метаболiчнімі факторами
-Реактивна артерiальна гiперемiя
-Фiзiологiчна артерiальна гiперемiя


#У тварини через 2 тіжнi пiсля експериментально звуження ниркової артерiї підвищився артеріальний тиск. Зi збiльшення дiї на судину якого фактора гуморальної регуляцiї це пов'язано?
+ангiотензин II
-кортизол
-aльдостерон
-вазопресин
-дофамiн


#У пілота на висоті 14000 м трапилася аварійна розгерметизація кабіни. Який із видів емболій у нього розвинувся?
+Газова.
-Емболія стороннім тілом.
-Тромбоемболія.
-Повітряна.
-Жирова


#Після вимушеного швидкого підняття водолаза з глибини на поверхню у нього з’явилися ознаки кесонної хвороби – біль у суглобах, свербіння шкіри, мерехтіння в очах, затьмарення свідомості. Яким видом ємболії вони були зумовлені?
+Газовою
-Повітряною
-Жировою
-Тканинною
-Тромбоємболією


#У хворого з переломом гомілковостопного суглоба після зняття гіпсової пов'язки спостерігається набряк стопи, ціаноз, місцеве зниження температури, збільшення органу в обсязі. Який вид порушення кровообігу спостерігається при цьому?
+ Венозна гіперемія
-Робоча гіперемія
-Метаболічна артеріальна гіперемія
-Реактивна гіперемія
-Ішемія


#Під час роботи лікарю - стоматологу доводиться довго стояти на ногах, що може призвести до застою крові у венах нижніх кінцівок та їх варикозного розширення. З порушенням якого механізму венозного припливу крові до серця це пов’язано?
+Відсутність скорочення скелетних м’язів
-Градієнт тиску
-Залишкова рушійна сила серця
-Присмоктувальний ефект грудної клітки
-Присмоктувально-тисковий помповий ефект діафрагми на органи черевної порожнини


#У дитини 5-ти рокiв розвинулось гостре респiраторне захворювання, яке супроводжувалось кашлем, видiленням значної кiлькостi слизу iз носа. Який тип запалення у хворої дитини?
+Катаральне
-Фiбриноїдне
-Геморагiчне
-Гнiйне
-Гнилiсне


#Запалення характеризується розширенням кровоносних судин на ділянці пошкодження, зменшенням кровообігу, підвищенням проникливості стінки судин. Яким з нижче наведених клітин належить головна роль в цьому?
+Тканинні базофіли
-Фібробласти
-Плазмоцити
-Макрофаги
-Еозинофіли


#У плазмі крові здорової людини знахо­диться декілька десятків білків. При захворюванні організму з’являються нові білки, зокрема "білок гострої фази". Таким білком є:
+С-реактивний білок
-Протромбін
-Імуноглобулін А
-Фібриноген
-Імуноглобулін G


#При моделюванні запалення нижньої кінцівки у тварини підвищилася температура тіла, збільшився вміст антитіл та лейкоцитів у крові. Які речовини обумовили розвиток цих загальних реакцій організму при запаленні?
+Інтерлейкіни
-Глюкокортикоїди
-Мінералокортикоїди
-Лейкотриєни
-Соматомедіни


#Юнак 17-ти рокiв захворiв гостро, температура тiла пiдвищилася до 38, 50 , з’явилися кашель, нежить, сльозотеча, видiлення з носу. Яке запалення розвинулося у юнака?
+Катаральне
-Серозне
-Фiбринозне
-Гнiйне
-Геморагiчне


#При запаленнi ока у хворого вiдмiчалося накопичення мутної рiдини з високим вмiстом бiлку на днi передньої камери, яке отримало назву - гiпопiон. Який процес лежить в основi зазначених змiн?
+Порушення мiкроциркуляцiї
-Первинна альтерацiя
-Вторинна альтерацiя
-Пролiферацiя
-Немає правильної відповіді


#У вiдпочиваючого в санаторiї у результатi сонячного опiку на шкiрi спини утворилися мiхурцi, заповненi свiтлою рiдиною, оточенi зоною гiпермiї, болiснi. Який з перерахованих механiзмiв лежить в основi формування ексудацiї у вогнищi запалення?
+Збiльшення колоїдно-осмотичного тиску в тканинi
-Зменшення виведення рiдини з тканини
-Зменшення рiвня кейлонiв у тканинi
-Збiльшення кiлькостi лiзосомальних ферментiв
-Емiграцiя лейкоцитiв з судин


#Чоловік 38 років поступив у терапевтичне відділення з діагнозом: правосторонній ексудативний плевріт. Відкачана з плевральної порожнини грудної клітки рідина прозора, має відносну щільність 1.020; містить 55 г/л, білка альбуміно-глобуліновий коефіцієнт-1.6; загальна кількість клітин - 2.8. в 1 мкл; pH - 6.5. Який тип ексудату має місце у хворого?
+серозний
-фібринозний
-гнійний
-гнилісний
-геморагічний


#Відповідно до фізико-хімічної теорії Шаде в зоні запалення має місце: гіперосмія, гіперонкія, ацидоз. Розвиток гіперосмії, в деякій мірі, пов'язаний із зростанням концентрації К
+ в зоні запалення. Вказати причини гіперкалійіонії в запальному ексудаті
+Інтенсивна деструкція пошкоджених клітин
-Збільшення проникненості судинної стінки
-Активація проліферативних процесів
-Пригнічення глікогеноліза в зоні запалення
-Надлишок іонів Са++


#У хворого через добу після травми розпух колінний суглоб. При його пункції отримано 30 мл рідини рожевого кольору з питомою щільністю 1020. Загальний вміст білка в ній - 3%, альбуміну - 0,3%, глобулінів 2%, фібриногену - 0,7%, лейкоцитів - 1-3, еритроцитів - 15-20, місцями до 50 в полі зору. Якого характеру ексудат отриманий при пункції колінного суглоба у хворого?
+Геморагічний
-Серозний
-Гнійний
-Гнильний
-Фібринозний


#У хворого плевритом в плевральній порожнині виявлена смердюча рідина, яка містить в собі біогенні аміни, гази. Який різновид запалення в даному випадку?
+Гнилісне
-Альтеративне
-Катаральне.
-Гнійне
-Фібрінозне.


#При дослідженні запалення піддослідній тварині ввели смертельну дозу правцевого токсину в порожнину абсцесу, індукованого скипидаром. Але піддослідна тварина не загинула. Вкажіть найбільш ймовірну причину такого результату досліду?
+Формування бар'єру навколо запалення
-Активація синтезу антитіл при запаленні
-Стимуляція лейкопоезу при запаленні
-Посилення васкуляризації місця запалення
-Активація дезінтоксикаційнної функції фагоцитів


#У дитини 6 років розвинулася гіперергічна форма запалення верхніх дихальних шляхів. З’явилася загорза серйозного порушення дихання, а тому виникла необхідність застосувати протизапальні гормони. Серед гормонів протизапальний ефект проявляє
+Кортизол
-Адреналін
-Соматотропін
-Тестостерон
-Інсулін


#При моделюванні запалення на брижі жаби спостерігали крайове стояння лейкоцитів та їх еміграцію крізь судинну стінку. Який із перелічених факторів обумовлює цєй процес?
+Вплив хемотаксичних речовин
-Збільшення онкотичного тиску в осередку запалення
-Зниження онкотичного тиску в судинах
-Збільшення гідростатичного тиску в судинах
-Зменшення гідростатичного тиску в судинах


#Під час огляду шкіри лікар помітив у хворого нагнійний процес у вигляді круглих підвищень червонуватого коліру, оточених зоною гіперемії. Які медиатори запалення зумовили явища судинної гіперемії?
+Гістамін
-Інтерлейкін 1
-Фактор активації тромбоцитів .
-Тромбоксан
-Лізосомальні ферменти


#При мікроскопії препарату брижі жаби виявлено, що в деяких капілярах відзначається маятникоподібний рух крові, формені елементи при цьому (зокрема, лейкоцити) з осьового шару виходять в пристінковий, а деякі навіть випускають псевдоподии в стінку капілярів. Який стадії судинної реакції при запаленні відповідає описане явище?
+Престаз
-Стаз
-Короткочасний спазм судин
-Артеріальна гіперемія
-Венозна гіперемія


#Кухар в результаті необачності обпік руку парою. Підвищення концентрації якої речовини викликало почервоніння, набряклість та болючість ураженої ділянки шкіри?
+Гістамін
-Галактозамін
-Глутамін
-Лізин
-Тіамін


#У хворого з флегмоною передпліччя при мікробіологічному аналізі ексудату в зоні запалення визначена присутність стрептококів. Які клітини будуть переважати в ексудаті?
+Нейтрофільні гранулоцити
-Базофільні гранулоцити
-Еозинофільні гранулоцити
-Лімфоцити
-Моноцити


#Autopsy of the body shows that the soft meninges of the deceased individual are plethoric, thickened, opaque, and yellow-green colored. What type of exudative inflammation can be characterized by such changes in the soft meninges?
+Suppurative
-Catarrhal
-Fibrinous
-Hemorrhagic
-Serous


#У людини виявлена пухлина одного з вiддiлiв головного мозку, внаслiдок чого в неї порушена здатнiсть пiдтримувати нормальну температуру тiла. Яка структура головного мозку пошкоджена?
+Гiпоталамус
-Таламус
-Мозочок
-Стрiатум
-Чорна субстанція


#У хворого дiагностовано септичний ендокардит. Температура тiла протягом 5-ти днiв коливалася в межах 39,5oC - 40,2oC. На 6-й день на тлi рiзкого зниження температури до 35,2oC розвинувся колапс. Який головний механiзм колапсу?
+Вазодилатацiя
-Гiпервентиляцiя
-Посилене потовидiлення
-Тахiкардiя
-Полiурiя


#У хворого 26-ти років вдень раптово підвищилася температура до 39,5°С і через 6 годин повернулася до норми. На другу до­бу напад повторився і температура досягла 41,5°С. Період апірексії настав через 8 го­дин. Який тип температурної кривої?
+Переміжний
-Послаблюючий
-Септичний
-Постійний
-Виснажуючий


#В експерименті на кролику введення пірогеналу призвело до підвищення у тварини температури тіла. Яка з перерахованих речовин відіграє роль вторинного пірогену, що бере участь у механізмі виникнення лихоманкової реакції?
+Інтерлейкін-1
-Імуноглобулін
-Гістамін
-Піромен
-Брадикінін


#У хворих на поворотний тиф виникає лихоманка, яка характеризується кількаденними періодами високої гарячки, що чергується з періодами нормальної температури. Така температурна крива називається:
+Febris recurrns
-Febris atypica
-Febris hectica
-Febris continua
-Febris intermittens


#У пацієнта після переливання 200 мл крові підвищилася температура тіла до 37,9oC. Яка з наведених речовин найбільш вірогідно призвела до підвищення температури?
+Інтерлейкін-1
-Інтерлейкін-2
-Інтерлейкін-4
-Фактор некрозу пухлин
-Інтерлейкін-3


#У хворого вдень пiднялася температура тiла до 390 i через 6 годин повернулася до норми. На другу добу напад повторився: в перiод пароксизму температура досягла 410 , перiод апiрексiї настав через 8 годин. Як називається такий тип температурної кривої?
+Iнтермiтуючий
-Зворотнiй
-Гектичний
-Септичний
-Постiйний


#У хворого iз запаленням легень спостерiгається пiдвищення температури тiла. Яка бiологiчно активна речовина вiдiграє провiдну роль у виникненнi цього прояву?
+Iнтерлейкiн-I
-Гiстамiн
-Брадикiнiн
-Серотонiн
-Лейкотрiєни


#При обстеженнi хворого виявленi наступнi клiнiчнi прояви: шкiрнi покриви рожевi, теплi на дотик, сухi, ЧСС-92/хв.,ЧДР- 22/хв., температура тiла - 39, 2oC. Яке спiввiдношення процесiв утворення i вiддачi тепла в описаному перiодi лихоманки?
+Теплопродукцiя дорiвнює тепловiддачi
-Теплопродукцiя перевищує тепловiддачу
-Теплопродукцiя нижче за тепловiддачу
-Зниження тепловiддачi на фонi незмiненої теплопродукцiї
- Посилення теплопродукцiї без змiни тепловiддачі


#У людини у якої лихоманка спостерігається збліднення шкірних покривів, "гусяча шкіра",озноб, тахікардія. Який стадії лихоманки відповідає даний стан?
+Стадії підйому температури
-Стадії стояння температури
-Стадії падіння температури
-
-


#У хворого який довго лихоманить ранкова температура тіла була в межах 36,4-36,9оС. До вечора вона піднялася до 37,0-38,0оС, в деякі дні до 38,8оС. Хворий температурить більше 2-х місяців. Який тип лихоманки у хворого?
+постійний
-послаблюючий
-виснажливий
-хвилеподібний
-гектичний


#Чоловік 25 років скаржиться на загальну слабкість, озноб, біль у горлі. Об'єктивно: почервоніння в області мигдаликів. Температура тіла 38.6оС. Які з перелічених клітин є головним джерелом ендогенних пірогенів, що викликають гарячку у хворого?
+нейтрофіли
-еозинофіли
-в - лімфоцити
-базофіли
-тучні клітини


#При обстеженні температурить хворого виявлені наступні об'єктивні дані: шкірні покриви гіперемійовані, вологі на дотик, спостерігається поліурія, полидипсия, температура тіла-37,2oС. Який стадії лихоманки відповідає даний стан?
+Падіння температури
-Підйому температури
-Стояння температури
-
-


#У довго лихоманить хворого температура вранці була в межах 36,4-36,9oС, до вечора вона піднімалася до 37,0-38,0oC. Який тип лихоманки за ступенем підйому температури спостерігається у хворого?
+субфебрильна
-помірний
-гіперпіретичний
-високий
-


#У чоловіка 35-ти років розвинулася гарячка, яка супроводжувалася зміщенням установчої точки терморегуляційного центру на більш високий рівень, з послідовним чергуванням наступних стадій: lncrementi, fastigii, dесrеmenti. При якому захворюванні можуть спостерігатися подібні зміни?
+Гостра пневмонія
-Акромегалія
-Ренальний діабет
-Цукровий діабет
-Гіпертрофія міокарда


#У хворого Н. приступи гарячки виникають через день. Під час приступу температура різко підвищується і утримується на високому рівні до 2 год, а потім знижується до вихідного рівня. Цей тип гарячки характерний для
+Малярії
-Поворотного тифу
-Сепсису
-Бруцельозу
-Висипного тифу


#У хворого на пневмонію виникла гарячка. Що безпосередньо спричинює зміни установочної точки температури в нейронах гіпоталамуса цього хворого?
+Простагландини Е1, Е2
-Ендотоксин
-Екзотоксин
-Інтерлейкін-2
-Тромбоцитарний фактор росту


#У жінки встановлено діагноз - рак шийки матки. З яким вірусом може бути асоційована ця патологія?
+Вірус простого герпеса тип 2
-Varicella-Zoster вірус
-Папілома вірус
-Аренавірус
-Цитомегаловірус


#У чоловіка 63-х років рак стравоходу, метастази у лімфатичні вузли середостіння, ракова кахексія. Яка патогенетична стадія пухлинного процесу має місце?
+Прогресії
-Ініціації
-Промоції
-Трансформації


#Чоловік 58 років, страждає раком сечового міхура. У процесі трудової діяльності мав kонтакт з канцерогенними речовинами. Дія якого з нижче перелічених канцерогенів найбільш вірогідна у данному випадку?
+b – нафтіламін
-20 – метілхолантрен
-бензапірен
-діметіламіноазобензол
-ортоаміноазотолуол


#Проводиться медичний огляд робітників цеху по виробництву анілінових барвників. Наявність пухлини якої локалізації може бути оцінене як професійне захворювання, внаслідок контакту з бета-нафтіламіном?
+сечового міхура
-стравоходу
-печінки
-нирок
-легень


#Жінці поставлено діагноз ерозія шійки матки, яка є передпухлинною патологією. Який захисний механізм може попередити розвиток пухлини?
+Збільшення природних кілерів (NK-клітин)
-Високодозова імунологічна толерантність
-Збільшення активності лізосомальних ферментів
-Спрощення антигенного складу тканин
-Низькодозова імунологічна толерантність


#Жінка 67 років, страждає раком шлунку з метастазами у печінку. Яка властивість пухлинних клітин обумовлює їх здатність до метастазування?
+інфільтративний ріст
-біохімічний атипізм
-швидкий ріст
-автономність
-антигенна анаплазія


#Клінічне обстеження хворого дозволило встановити попередній діагноз рак печінки. Наявність якого білка в сироватці крові дозволить підтвердити діагноз?
+альфа-фетопротеїн
-Пропердин
-Парапротеїни
-С-реактивний протеїн
-гамма-глобуліни


#Встановлено, що при розвитку гепатоми в ній часто припиняється синтез жовчних кислот. Про який вид анаплазии це свідчить?
+Функціональної
-Енергетичної
-Морфологічної
-Біохімічної
-Фізико хімічної


#Хворому з ревматоїдним артритом тривалий час вводили гiдрокортизон. У нього з’явилися гiперглiкемiя, полiурiя, глюкозурiя, спрага. Цi ускладнення лiкування є наслiдком активацiї процесу:
+Глюконеогенез
-Глiкогенолiз
-Глiкогенез
-Глiколiз
-Лiполiз


#При диспансерному обстеженнi у хворого знайдено цукор в сечi. Який найбiльш iмовiрний механiзм виявлених змiн, якщо вмiст цукру в кровi нормальний?
+Порушення реабсорбцiї глюкози в канальцях нефрона
-Порушення фiльтрацiї глюкози в клубочковому вiддiлi нефрона
-Недостатня продукцiя iнсулiну пiдшлунковою залозою
-Iнсулiнорезистентнiсть рецепторiв клiтин
-Гiперпродукцiя глюкокортикоїдiв наднирниками


#У хворого 15-ти рокiв концентрацiя глюкози натще - 4,8 ммоль/л, через годину пiсля цукрового навантаження - 9,0 ммоль/л, через 2 години - 7,0 ммоль/л, через 3 години - 4,8 ммоль/л. Цi показники характернi для такого захворювання:
+Прихований цукровий дiабет
-Цукровий дiабет I типу
-Цукровий дiабет II типу
-Хвороба Iценко-Кушiнга


#У хлопчика 2-х рокiв спостерiгається збiльшення в розмiрах печiнки та селезiнки, катаракта. В кровi пiдвищена концентрацiя цукру, але тест толерантностi до глюкози в нормi. Спадкове порушення обмiну якої речовиниє причиною цього стану?
+Галактоза
-Фруктоза
-Глюкоза
-Мальтоза
-Сахароза


#У студента, який складає іспит, вміст глюкози у плазмі крові складає 8 ммоль/л. Збільшена секреція якого з наведених гормонів сприяє розвитку гіперглікемії у студента?
+Глюкагон
-Інсулін
-Трийодтиронін
-Тироксин
-Альдостерон


#Хвора 46-ти років скаржиться на сухість в роті, спрагу, почащений сечопуск, загальну слабкість. У крові: гіперглікемія, гіперкетонемія. У сечі: глюкоза, кетонові тіла. На ЕКГ: дифузні зміни в міокарді. Який найбільш імовірний діагноз?
+Цукровий діабет
-Нецукровий діабет
-Аліментарна гіперглікемія
-Гострий панкреатит
-Ішемічна хвороба серця


#Хвора 38-ми років надійшла до реанімаційного відділення в несвідомому стані. Рефлекси відсутні. Цукор крові - 2,1 ммоль/л. В анамнезі - цукровий діабет з 18-ти років. Яка кома має місце у хворої?
+Гіпоглікемічна
-Кетоацидотична
-Гіперглікемічна
-Лактацидемічна
-Гіперосмолярна


#У новонародженого спостерігається диспепсія після годування молоком. При заміні молока розчином глюкози симптоми диспепсії зникають. Недостатня активність якого ферменту спостерігається у новонародженого?
+Лактаза
-Амілаза
-Мальтаза
-Сахараза
-Ізомальтаза


#У жiнки 62-х рокiв розвинулася катаракта (помутнiння кришталика) на фонi цукрового дiабету. Який тип модифiкацiї бiлкiв має мiсце при дiабетичнiй катарактi?
+Глiкозилювання
-Фосфорилювання
-АДФ-рибозилювання
-Метилювання
-Обмежений протеолiз.


#Пiсля тривалого фiзичного навантаження пiд час заняття з фiзичної культури у студентiв розвинулась м’язова крепатура. Причиною її виникнення стало накопичення у скелетних м’язах молочної кислоти. Вона утворилась пiсля активацiї в органiзмi студентiв:
+Глiколiзу
-Глюконеогенезу
-Пентозофосфатного циклу
-Лiполiзу
-Глiкогенезу


#В експериментi на кролi встановлено, що об’єм кисню, який спожива-ється головним мозком за 1 хвилину, дорiвнює об’єму C O2, який видiляється клiтинами мозку в кров. Це свiдчить, що у клiтинах головного мозку має мiсце:
+Окислення вуглеводiв
-Окислення жирiв
-Гiпокапнiя
-Гiпоксiя
-Окислення бiлкiв


#При обстеженнi жiнки 56-ти рокiв, що хвора на цукровий дiабет 1-го типу, виявлене порушення бiлкового обмiну, що при лабораторному дослiдженнi кровi проявляється амiноацидемiєю, а клiнiчно - уповiльненням загоєння ран i зменшенням синтезу антитiл. Який з перерахованих механiзмiв викликає розвиток амiноацидемiї?
+Пiдвищення протеолiзу
-Гiперпротеїнемiя
-Зменшення концентрацiї амiнокислот у кровi
- Пiдвищення онкотичного тиску в плазмi кровi
- Збiльшення лiпопротеїдiв високої щiльностi


#Молода людина 25-ти рокiв споживає надмiрну кiлькiсть вуглеводiв (600 г на добу), що перевищує її енергетичнi потреби. Який процес буде активуватися в органiзмi людини у даному випадку?
+Лiпогенез
-Глiколiз
-Лiполiз
-Глюконеогенез
- Окиснення жирних кислот


#Чоловік 48 років, страждає на цукровий діабет з 6 років, госпіталізований у непритомному стані, який наступив після підвищеного фізичного навантаження. Об'єктивно: шкіра волога, тонус м'язів кінцівок підвищений сухожильні рефлекси підвищені, дихання поверхневе, пульс 78 уд/хв., АТ-95/60 мм. рт. ст., рівень глюкози у крові 1.88 ммоль/л. Для якого із перелічених станів найбільш характерні симптоми у чоловіка?
+гіпоглікемічної коми
-гіперкетонемічної
-лактацидемічної коми
-гіперосмолярної коми
-гіпотонічного стану


#У хворого визначається наявність гіперглікемії, поліурії, гіперстенурія і глюкозурія. Для якої нозологічної форми патології обміну характерно таке поєднання цих показників?
+Цукрового діабету
-Печінкового діабету
-нецукрового діабету
-глікогенозів
-діенцефального ожиріння


#Хворий А., 18 років, після перенесеної краснухи почав худнути, постійно відчував сухість в роті, спрагу, у нього підвищився апетит, почалось часте сечовиділення. Обўєктивно: добова кількість сечі 6 л, глюкоза крові 17 ммоль/л, в сечі виявлена глюкоза та ацетон.


Яке захворювання виникло у хворого?
+ІЗЦД (інсулінзалежний цукровий діабет).
-Симптоматичний цукровий діабет.
-Вторинний цукровий діабет.
-ІНЦД (інсуліннезалежний цукровий діабет).
-Стероїдний діабет.


#У хворого на цукровий діабет знижені процеси регенерації, довго не загоюються рани. З якими змінами в обміні речовин це пов язане?
+Пригніченням протеосинтезу.-Накопиченням кетонових тіл.
-Ацидозом.
-Зменшенням надходження глюкози в клітини.
-Порушенням жирового обміну.


#У хворого на цукровий діабет виникла діабетична кома, яка характеризувалася повною втратою свідомості, арефлексія, гіпотензією, падінням температури тіла, великим гучним диханням. Як називається такий тип дихання?
+Куссмауля
-Биота
-Чейн-Стокса
-Періодичне
-Апнейстичне


#У хворого А., 18 років після перенесеної краснухи виникла втрата маси тіла, постійне відчуття спраги, підвищення апетиту. Об'єктивно: добова кількість сечі 6 л, глюкоза крові 17,8 ммоль/л, в сечі виявлено глюкозу та ацетон. Яку патологію можливо запідозрити у хворого?
+Інсулінзалежний цукровий діабет
-Нецукровий діабет
-Стероїдний діабет
-Інсуліннезалежний цукровий діабет
-Діабетична нефропатія


#У вiддiлення реанiмацiї доставлено непритомного пацiєнта iз запахом ацетону з ротової порожнини. Методи експрес-аналiзу виявили у кровi 17,3 ммоль/л глюкози. Пiдвищення вмiсту яких речовин призвело до втрати свiдомостi?
+Кетоновi тiла
-Молочна кислота
-Глюкоза
-Жирнi кислоти
-Сечовина


#У хворий довго хворіє на цукровий діабет 2-го типу, після порушення дієти (вживання в їжу легкозасвоюваних вуглеводів) поступово наростала загальна слабкість, знизився артеріальний тиск, з'явилися галюцинації, судоми. Шкіра суха, чіткі прояви дегідратації організму. Рівень глюкози крові 40 ммоль / л.


Який вид коми є причиною погіршення стану хворої?
+Гіперосмолярна
-Ацидотична
-Лактатацідемічесна
-Kетонемічна
-Гіпоглікемічна


#У хворої А., 18 років, після перенесеної краснухи початку відзначатися втрата маси тіла, постійне відчуття сухості в роті, спрагу, підвищення апетиту, часте сечовиділення. Об'єктивно: добова кількість сечі 6 л, глюкоза крові 17,8 ммоль / л, в сечі виявлено глюкоза і ацетон. Який найбільш ймовірний патогенетичний механізм викликав підвищення рівня глюкози у хворої?
+Зменшення вироблення інсуліну
-Збільшення глюконеогенезу
-Підвищений руйнування інсуліну
-Пошкодження інсулінових рецепторів клітин
-Збільшення вироблення глюкокортикоїдів


#У хлопчика М., 2 років була діагностована хвороба Гірке, що супроводжується надмірним відкладанням глікогену в печінці і нирках, гіпоглікемією. При біохімічному дослідженні крові виявлено:
+Дефіцит глюкозо-6-фосфатази
-Зниження активності аміло-1,6-глюкозидази
-Дефіцит фруктозо-дифосфат альдолази
-Зниження активності фосфорилази
-Дефіцит кетогексокінази


#У піддослідної тварини (щури) шляхом внутрішньовенного введення аллоксона був викликаний експериментальний цукровий діабет. Який механізм дії даної речовини?
+Пошкодження beta - клітин панкреатиччних острівців
-Зв'язування цинку
-Утворення антитіл до інсуліну
-Активація інсулінази
-Активація вироблення контрінсулярнихгормонів


#У хворого на ранній стадії цукрового діабету спостерігається поліурія. Чим вона викликана?
+Гіперглікемією
-Кетонемією
-Гіпохолестеринемією
-Гіперхолестеринемією
-Гіперкалійемією


#Внаслідок тривалого голодування в організмі людини швидко зникають резерви вуглеводів. Який з процесів метаболізму за цих умов поновлює вміст глюкози в крові?
+Глюконеогенез
-Глікогеноліз
-Аеробний гліколіз
-Анаеробний гліколіз
-Пентозофосфатний шлях


#Аналіз крові хворого на цукровий діабет показав наявність молочної кислоти у концентрації 2,5 ммоль/л. Яка кома розвинулася у хворого?
+Лактацидемічна
-Гіперглікемічна
-Гіперкетонемічна
-Гіперосмолярна
-Гіпоглікемічна


#У хворого струс головного мозку,що супроводжується повторним блюванням i задишкою. При обстеженнi вiдзначено: рН - 7,62; pCO2 - 40 мм рт.ст. Яке порушення кислотно-основного стану є у хворого?
+Негазовий алкалоз
-Газовий алкалоз
-Негазовий ацидоз
-Газовий ацидоз


#У чоловiка 32-х рокiв, хворого на пневмонiю, спостерiгається закупорка харкотинням дихальних шляхiв. В органiзмi хворого при цьому буде розвиватися така змiна кислотно-лужної рiвноваги:
+Респiраторний ацидоз
-Метаболiчний ацидоз
-Респiраторний алкалоз
-Метаболiчний алкалоз
-Змiн не буде


#У альпiнiста, що пiднявся на висоту 5200м, розвинувся газовий алкалоз.Що є причиною його розвитку?
+Гiпервентиляцiя легенiв
-Гiповентиляцiя легенiв
-Гiпероксемiя
-Гiпоксемiя
-Зниження температури навколишнього середовища


#У хворого з дихальною недостатнiстю рН кровi 7,35. Визначення pCO2 показало наявнiсть гiперкапнiї. При дослiдженнi рН сечi вiдзначається пiдвищення її кислотностi. Яка форма порушення кислотно-основного стану в даному випадку?
+Ацидоз газовий, компенсований
-Ацидоз метаболiчний, компенсований
-Ацидоз метаболiчний, декомпенсований
-Алкалоз газовий, компенсований
-Алкалоз газовий, некомпенсований


#У пацієнта у результатi тривалого блювання вiдбувається значна втрата шлункового соку, що є причиною порушення кислотно-лужного стану в органiзмi. Яка з перерахованих форм порушення кислотно-лужного стану має мiсце?
+Негазовий алкалоз
-Газовий ацидоз
-Негазовий ацидоз
-Газовий алкалоз
-Метаболiчний ацидоз


#У хворого 40-ка рокiв ознаки гiрської хвороби: запаморочення, задишка, тахiкардiя, рН кровi - 7,50, pCO2- 30 мм рт.ст., зсув буферних основ +4 ммоль/л. Яке порушення кислотно-основногостану має мiсце?
+Газовий алкалоз
-Негазовий алкалоз
-Негазовий ацидоз
-Газовий ацидоз
-Видiльний ацидоз


#Чоловiк 53-х рокiв доставлений у стацiонар у непритомному станi. Об’єктивно: шкiра суха, дихання часте поверхневе, запах ацетону вiдсутнiй, Ps- 126/хв., АТ- 70/40 мм рт.ст. Вмiст глюкози у кровi 48 ммоль/л, реакцiя сечi на ацетон негативна. Для якого iз перелiчених станiв найбiльш характернi симптоми у хворого?
+Гiперосмолярна кома
-Гiперкетонемiчна кома
-Лактацидемiчна кома
-Токсична кома
-Колапс


#У хворого з дихальною недостатністю рН крові 7,35. Визначення рС02 показа­ло наявність гіперкапнїї. При дослідженні рН сечі відзначається підвищення її кисло­тності. Яка форма порушення кислотно- основного стану в даному випадку?
+Ацидоз газовий, компенсований
-Алкалоз газовий, декомпенсований
-Ацидоз метаболічний, декомпенсований
-Ацидоз метаболічний, компенсований
-Алкалоз газовий, компенсований


#У хворого після вживання недоброякісної їжі розвинулася діарея. На наступний день у нього знизився артеріальний тиск, з'явились тахікардія, екстрасистолія. pH крові складає 7,18. Ці порушення є наслідком розвитку:
+Негазового ацидозу
-Метаболічного алкалозу
-Негазового алкалозу
-Газового алкалозу
-Газового ацидозу


#У немовля з пілороспазмом внаслі­док блювання, що часто повторювалося, з’явилися слабкість, гіподинамія, іноді су­доми. Яка форма порушення кислотно-основного стану в нього спостерігається?
+Видільний алкалоз
-Метаболічний ацидоз
-Газовий алкалоз
-Видільний ацидоз
-Екзогенний негазовий ацидоз


#У немовляти внаслiдок неправильного годування виникла виражена дiарея. Одним з основних наслiдкiв дiареї є екскрецiя великої кiлькостi бiкарбонату натрiю. Яка форма порушення кислотно-лужного балансу має мiсце в цьому випадку?
+Метаболiчний ацидоз
-Метаболiчний алкалоз
-Респiраторний ацидоз
-Респiраторний алкалоз
-Не буде порушень кислотно-лужного балансу


#У клініку доставлений чоловік 30 років, з профузним поносом тривалістю 12 годин. Блювоти не було. Які зміни водно-електролітного балансу та кислотно-основної рівноваги спостерігаються у чоловіка?
+негазовий ацидоз з дегідратацією
-газовий ацидоз з дегідратацією
-газовий алкалоз з дегідратацією
-без змін pH крові
-негазовий алкалоз з дегідратацією


#У людини з хронічним гломерулонефритом наростає загальна слабкість, різка тахікардія з періодичної аритмією, загальмованість і сонливість. Який зрушення КОС супроводжує наближення уремічний коми?
+Негазовий видільної ацидоз
-Негазовий метаболічний ацидоз
-Газовий ацидоз
-Газовий алкалоз
-Негазовий алкалоз


#При обстеженні хворого визначається наявність гіперглікемії, кетонурія, поліурії, гіперстенурія і глюкозурії. Яка форма порушення кислотно-лужної рівноваги


має місце в даній ситуації?
+Метаболічний ацидоз
-Газовий ацидоз
-Газовий алкалоз
-Метаболічний алкалоз
-Негазовий алкалоз


#При підйомі в гори у альпініста розвинулась ейфорія, яка замінилася головним болем, запамороченням, серцебиттям, задишкою, що перейшла в апное. Яке порушення кислотно-лужної рівноваги розвинулось в даному випадку?
+Газовий алкалоз.
-Негазовий ацидоз.
-Газовий ацидоз.
-Негазовий алкалоз.
-Видільний алкалоз


#У хворого на цукровий діабет розвинулася діабетична кома внаслідок порушення кислотно-основного стану. Який вид порушення виник при цьому?
+Метаболічний ацидоз
-Метаболічний алкалоз
-Респіраторний ацидоз
-Газовий алкалоз
-Негазовий алкалоз


#У вагітної жінки розвинувся токсикоз з тяжким повторним блюванням шлунковим вмістом протягом доби. Наприкінці доби почали проявлятися тетанічні судоми та зневоднення організму. Який розлад кислотно-лужної рівноваги викликав дані зміни?
+Негазовий видільний алкалоз
-Газовий алкалоз
-Газовий ацидоз
-Негазовий видільний ацидоз
-Негазовий метаболічний ацидоз


#У вагiтної жiнки 24-х рокiв пiсля тривалого блювання було зареєстровано зниження об’єму циркулюючої кровi. Про яку змiну загальної кiлькостi кровi може йти мова?
+Полiцитемiчна гiповолемiя
-Проста гiповолемiя
-Олiгоцитемiчна гiповолемiя
-Полiцитемiчна гiперволемiя
-Олiгоцитемiчна гiперволемiя


#У хворого 41-го року вiдзначається гiпонатрiємiя, гiперкалiємiя, дегiдратацiя, зниження артерiального тиску, м’язова слабкiсть, брадикардiя, аритмiя. З порушенням функцiй яких гормонiв це пов’язано?
+Кортикостероїди
-Тиреоїднi
-Гормони пiдшлункової залози
-Статевi гормони
-Гормони мозкової речовини наднирникiв


#Хворий на гiпертонiчну хворобу разом з безсольовою дiєтою та з антигiпертензивними засобами, довгий час приймав гiдрохлортiазид, що зумовило порушення електролiтного балансу. Яке порушення внутрiшнього середовища виникло у хворого?
+Гiпохлоремiчний алкалоз
-Метаболiчний ацидоз
-Гiперкалiємiя
-Гiпермагнiємiя
-Збiльшення об’єму циркулюючої кровi


#При лабораторному дослiдженнi кровi пацієнта виявлено, що вмiст білків у плазмi становить 40г/л. Як це впливає на транс капілярний обмiн води в мiкроциркуляторному руслi?
+Збiльшується фiльтрацiя, зменшується реабсорбцiя
-Збiльшуються фiльтрацiя i реабсорбцiя
-Зменшується фiльтрацiя, збiльшується реабсорбцiя
-Зменшуються фiльтрацiя i реабсорбцiя
-Обмiн не змінюється


#У людини з масою 80 кг пiсля тривалого фiзичного навантаження об’єм циркулюючої кровi зменшився, гематокрит - 50%, загальний бiлок кровi - 80 г/л. Такi показники кровi є наслiдком, перш за все:
+Втрати води з потом
-Збiльшення кiлькостi еритроцитiв
-Збiльшення вмiсту бiлкiв у плазмi
-Збiльшення онкотичного тиску плазми
-Збiльшення дiурезу


#У дорослої людини за добу виділяється 20 л сечі з низькою відносною щільністю. Найбільш імовірною причиною цього є дефіцит в організмі:
+Вазопресину
-Натрійуретичного фактора
-Альдостерону
-Паратгормону
-Реніну


#У тварини через 2 тижні після експериментального звуження ниркової артерії підвищився артеріальний тиск. Зі збільшенням дії на судини якого фактора гуморальної регуляції це пов'язано?
+Ангіотензин ІІ
-Вазопресин
-Альдостерон
-Дофамін
-Кортизол


#У людини зменшений діурез, гіпернатріємія, гіпокаліємія. Гіперсекреція якого гормону може бути причиною таких змін?
+Альдостерон
-Адреналін
-Паратгормон
-Передсердний натрійуретичний фактор
-Вазопресин


#При токсичному ушкодженні клітин печінки з порушенням її функцій у хворого з'явилися набряки. Які зміни складу плазми крові є провідною причиною розвитку набряків?
+Зниження вмісту альбумінів
-Збільшення вмісту альбумінів
-Зменшення вмісту глобулінів
-Збільшення вмісту глобулінів
-Зменшення вмісту фібриногену


#У людини збільшений вміст іонів кальцію в плазмі крові, зменшений - у кістках. Надмірна секреція якого гормону може спричинити такі зміни?
+Паратгормон
-Тироксин
-Альдостерон
-Трийодтиронін
-Тиреокальцитонін


#Тварині внутрішньовенно ввели концентрований розчин хлориду натрію, що зумовило зниження реабсорбції іонів натрію у канальцях нирок. Внаслідок яких змін секреції гормонів це відбувається?
+Зменшення альдостерону
-Зменшення натрійуретичного фактора
-Збільшення альдостерону
-Збільшення вазопресину
-Зменшення вазопресину


#Внаслідок вираженого зниження концентрації кальцію в плазмі крові у дитини 2-х років виникли тетанічні скорочення дихальних і глоткових м'язів. Зниження секреції якого гормону може бути причиною цього?
+Паратгормон
-Тиреокальцитонін
-Кортизол
-Альдостерон
-Соматотропін


#У хворого на ентерит, що супрово­джувався значною діареєю, спостерігає­ться зменшення кількості води в позаклі­тинному просторі, збільшення її всереди­ні клітин та зниження осмолярності кро­ві. Як називають таке порушення водно- електролітного обміну?
+Гіпоосмолярна гіпогідратація
-Гіпоосмолярна гіпергідратація
-Гіперосмолярна гіпергідратація
-Осмолярна гіпогідратація
-Гіперосмолярна гіпогідратація


#Хворий 50-ти років звернувся до клініки зі скаргами на загальну слабкість, втрату апетиту, аритмію серця. Спостерігається гіпотонія м'язів, мляві паралічі, послаблення перистальтики кишечнику. Причиною такого стану може бути:
+Гіпокаліємія
-Гіперкаліємія
-Гіпофосфатемія
-Гіпопротеїнемія
-Гіпонатріємія


#Жінці 36-ти років після хірургічного втручання внутрішньовенно ввели концентрований розчин альбуміну. Це спричинило посилений рух води у такому напрямку:
+З міжклітинної рідини до капілярів
-З міжклітинної рідини до клітин
-Із клітин до міжклітинної рідини
-Із капілярів до міжклітинної рідини
-Змін руху води не відбуватиметься


#У хворого з масивними опіками розвинулась гостра недостатність нирок, що характеризується значним і швидким зменшенням швидкості клубочкової фільтрації. Який механізм її розвитку?
+Зменшення ниркового кровотоку
-Зменшення кількості функціонуючих нефронів
-Емболія ниркової артерії
-Збільшення тиску канальцевої рідини
-Ушкодження клубочкового фільтра


#Під час огляду дитини 11-ти місяців педіатр виявив викривлення кісток нижніх кінцівок і затримку мінералізації кісток черепа. Нестача якого вітаміну призводить до даної патології?
+Холекальциферол
-Біофлавоноїди
-Рибофлавін
-Пантотенова кислота
-Тіамін


#Чоловiк 32-х рокiв чотири роки страждає на хронiчний гломерулонефрит з нефротичним синдромом. Вiдзначаються явнi набряки на обличчi, в останнiй час з’явилися набряки на ногах та тулубi. Який iз механiзмiв розвитку набрякiв найбiльш вiрогiдний у цього хворого?
+Зниження онкотичного тиску кровi
-Пiдвищення гiдростатичного тиску кровi в капiлярах
-Пiдвищення онкотичного тиску тканинної рiдини
-Утруднення лiмфовiдтоку
-Пiдвищення проникливостi капiлярiв


#В експериментi збiльшили проникнiсть мембрани збудливої клiтини для iонiв калiю. До яких змiн мембранного потенцiалу це призведе?
+Гiперполяризацiя
-Деполяризацiя
-Потенцiал дiї
-Локальна вiдповiдь
-Змiн не буде


#У хворого 35-ти рокiв, який часто вживає алкоголь, на фонi лiкування сечогiнними засобами, виникли сильна м’язова i серцева слабкiсть, блювання, дiарея, АТ- 100/60 мм рт.ст., депресiя. Причиною такого стану є посилене видiлення з сечею:
+Калiю
-Натрiю
-Хлору
-Кальцiю
-Фосфатiв


#У хворого, що страждає на важку форму порушення водно-сольового обмiну, настала зупинка серця в дiастолi. Який найбiльш вiрогiдний механiзм зупинки серця в дiастолi
+Гiперкалiємiя
-Гiпернатрiємiя
-Дегiдратацiя органiзму
-Гiпокалiємiя
-Гiпонатрiємiя


#У хворого, що страждає на серцеву недостатнiсть, спостерiгаються збiльшення печiнки, набряки нижнiх кiнцiвок, асцит. Який механiзм є провiдним в утвореннi даного набряку?
+Гiдродинамiчний
-Колоїдно-осмотичний
-Лiмфогенний
-Мембраногенний
-Немає правильної відповіді


#У хворого з нефротичним синдромом спостерiгаються масивнi набряки обличчя та кiнцiвок. Який патогенетичний механiзм є провiдним в розвитку набрякiв?
+Зниження онкотичного тиску кровi
-Пiдвищення судинної проникностi
-Пiдвищення гiдродинамiчного тиску кровi
-Лiмфостаз
-Пiдвищення лiмфовiдтоку


#У людини осмотичний тиск плазми кровi 350 мосмоль/л (норма -300 мосмоль/л). Це спричинить, перш за все, посилену секрецiю такого гормону:
+Вазопресин
-Альдостерон
-Кортизол
-Адренокортикотропiн
-Натрiйуретичний


#Тваринi внутрiшньовенно ввели концентрований розчин хлориду натрiю, що зумовило зниження реабсорбцiї iонiв натрiю у канальцях нирок. Внаслiдок яких змiн секрецiї гормонiв це вiдбувається?
+Зменшення альдостерону
-Збiльшення альдостерону
-Зменшення вазопресину
-Збiльшення вазопресину
-Зменшення натрiйуретичного фактора


#У хворої людини посилений рух води з кровоносних капiлярiв до тканин, що викликало їх позаклiтинний набряк (збiльшенi розмiри м’яких тканин кiн-цiвок, печiнки тощо). Зменшення якого параметру гомеостазу є найбiльш iмо-вiрною причиною розвитку набряку?
+Онкотичний тиск плазми кровi
-Осмотичний тиск плазми кровi
-рН кровi
-В’язкiсть кровi
-Гематокрит


#При токсичному ушкодженнi клiтин печiнки з порушенням її функцiй у хворого з’явилися набряки. Якi змiни складу плазми кровi є провiдною причиною розвитку набрякiв?
+Зниження вмiсту альбумiнiв
-Збiльшення вмiсту глобулiнiв
-Зменшення вмiсту фiбриногену
-Збiльшення вмiсту альбумiнiв
-Зменшення вмiсту глобулiнiв


#Юнака 15-ти рокiв вжалила бджола. На мiсцi укусу визначається набряк, гiперемiя, пiдвищення температури. Назвiть iнiцiальний патогенетичний фактор запального набряку:
+Пiдвищення проникностi мiкросудин
-Пiдвищення осмотичного тиску у вогнищi запалення
-Зниження онкотичного тиску кровi
- Пiдвищення кров’яного тиску в капiлярах
- Немає правильної відповіді


#У туриста пiд час тривалого перебування на спекотi вiдбулася значна втрата води, що супроводжувалося рiзким зниженням дiурезу. Посилення секрецiї яких гормонiв вiдбувається при цьому?
+Вазопресин й альдостерон
-Адреналiн i норадреналiн
-Глюкокортикоїди й iнсулiн
-Тироксин i трийодтиронiн
-Серотонiн i дофамiн


#Пацiєнт 16-ти рокiв, що страждає на хворобу Iценко-Кушiнга, консультований з приводу надмiрної ваги тiла. При опитуваннi з’ясувалося, що енергетична цiннiсть спожитої їжi складає 1700-1900 ккал/добу. Яка провiдна причина ожирiння у даному випадку?
+Надлишок глюкокортикоїдiв
- Нестача iнсулiну
- Надлишок iнсулiну
- Нестача глюкокортикоїдiв
-Гiподинамiя


#Чоловік 64 років, скаржиться на задишку, часте серцебиття, швидку втомлюваність. Ввечері з'являються набряки на нижчих кінцівках.Що із нижче переліченого являється патогенетичним фактором цих набряків?
+підвищення гідростатичного тиску крові у венозних частинах капілярів
-зниження онкотичного тиску крові
-підвищення онкотичного тиску тканинної рідини
-підвищення проникливості капілярів
-порушення лімфовідтоку


#Жінка 55 років, яка проживає в гірській місцевості, діагностований ендемічний зоб. Об'єктивно: трохи підвищеної вгодованості, загальмована, апатична, збільшення щитовидної залози. Дефіцит якого з перелічених нижче елементів викликає цей стан?
+йод
-фтор
-марганець
-молібден
-натрій


#Внаслідок захворювання нирок у паціінта відмічаються набряки. В аналізах сечі асивна протеїнурія. Який механізм є основним у виникненні набряків у такого пацієнта?
+Зниження онкотичного тиску плазми крові
-Підвищення осмотичного тиску плазми крові
-Зниження онкотичного тиску лімфи
-Зниження онкотичного тиску тканин
-Зниження фільтраційного тиску в нирках


#У хворого на цукровий діабет виникла значна спрага, дисфагія та порушення психічної діяльності. Який тип розладів водно-електролітного балансу характеризує поява вказаних ознак?
+Дегідратація гіперосмотична.
-Дегідратація гіпоосмотична.
-Дегідратація ізоосмотична.
-Гідратація гіпоосмотична.
-Гідратація ізоосмотична.


#Внаслідок травмування у хворого видалили прищитовидні залози, що супроводжувалося: млявістю, спрагою, різким підвищенням нервово-м’язової збудливості. З порушенням обміну якої речовини це пов’язано:
+Кальцію
-Марганцю
-Хлору
-Молібдену
-Цинку


#При повному (з водою) аліментарному голодуванні розвинулись генералізовані набряки. Який із патогенетичних факторів у цьому випадку є ведучим?
+Зниження онкотичного тиску плазми крові.
-Зниження гідростатичного тиску міжклітинної рідини.
-Зниження осмотичного тиску плазми крові.
-Підвищення онкотичного тиску тканинної рідини.
-Підвищення осмотичного тиску міжклітинної рідини.


#У хворого важка нефропатія з масивним набряклим синдромом, яка ускладнила хронічну бронхоектатичну хворобу. Лабораторні дослідження виявляють рясну протеинурию, циліндрурію, значне зниження вміст білка в сироватці крові, Гіперліпемія, гіпокаліємію та ін. відхилення. Що є


первинним і найбільш суттєвою ланкою в патогенезі набряків у даного хворого?
+Зниження онкотичного тиску крові
-Підвищення гідростатичного тиску крові
-Підвищення тиску позаклітинної рідини
-Блокада лимфооттока
-Підвищення проникності мікросудин


#У хворого 40 років після перенесеної травми головного мозку стало спостерігатися підвищене виділення сечі (до 8 - 10 л / добу) і спрага. Аналіз сечі показав відсутність глюкозурії і низьку відносну щільність. Яка форма порушення водно-сольового обміну можлива в некомпенсовані період описаного захворювання?
+Гіперосмолярна гіпогідрадація
-Ізоосмолярна гіпогідрадація
-Гіпоосмолярна гипогидратация
-Ізоосмолярна гипергидратация
-Гіпоосмолярна гипергидратация


#Людині внутрішньовенно ввели 0,5 л ізотонічного розчину лікарської речовини. Які з рецепторів насамперед відреагують на зміну водно-сольового балансу організму?
+Волюморецептори порожнистих вен і передсердь
-Барорецептори дуги аорти
-Натрієві рецептори гіпоталамусу
-Осморецептори гіпоталамусу
-Осморецептори печінки


#У хворого з патологією серцево-судинної системи розвинулись набряки на нижніх кінцівках. Який механізм розвитку серцевого набряку?
+Підвищення гідростатичного тиску в венулах
-Зниження осмотичного тиску плазми крові
-Підвищення гідростатичного тиску в артеріолах
-Підвищення онкотичного тиску плазми крові
-Порушення лімфовідтоку


#У людини після гострої крововтрати виникло відчуття спраги. Зміна якого гомеостатичного параметру викликала це відчуття?
+Зменшення об’єму позаклітинної рідини
-Зниження онкотичного тиску рідин організму
-Зниження осмотичного тиску рідин організму
-Підвищення онкотичного тиску рідин організму
-Підвищення осмотичного тиску рідин організму


#В ході експерименту у білого щура моделювався набряк легені шляхом введення адреналіну. Який патогенетичний механізм розвитку набряку є провідним в даному випадку?
+Гідродинамічний
-Колоїдно-осмотичний
-Лімфогенний
-Мембраногенний
-Токсичний


#У зв’язку з крововтратою пацієнту введено 1 л розчину хлориду натрію з кон- центрацією 150 ммоль/л. Внаслідок цього, насамперед, зменшиться:
+Онкотичний тиск крові
-Онкотичний тиск міжклітинної рідини
-Осмотичний тиск внутрішньоклітинний
-Осмотичний тиск крові
-Осмотичний тиск міжклітинної рідини


#Після споживання солоної їжі у людини значно зменшилася кількість сечі. Підвищена секреція якого гормону призвела до зменшення діурезу?
+Вазопресин
-Альдостерон
-Ангіотензин-II
-Натрійуретичний
-Ренін


#Після видалення у пацієнта 2/3 шлунка у крові зменшився вміст гемоглобіну, кількість еритроцитів, збільшилися розміри цих клітин крові. Дефіцит якого вітаміну призводить до таких змін у крові?
+B12
-P
-PP
-C
-B6


#Дитина 9-ми місяців харчується штучними сумішами, які не збалансовані за вмістом вітаміну B6. У дитини спостерігається пелагроподібний дерматит, судоми, анемія. Розвиток судом може бути пов'язаний з порушенням утворення:
+ГАМК
-Серотоніну
-Дофаміну
-Гістаміну
-ДОФА


#У хворого спостерігається погіршення сутінкового зору. Який з вітамінних препаратів слід призначити пацієнту?
+Ретинолу ацетат
-Ціанокобаламін
-Піридоксину гідрохлорид
-Кислота нікотинова
-Кислота аскорбінова


#У хворого з частими кровотечами з внутрiшнiх органiв i слизових оболонок виявленi пролiн i лiзин у складi колагенових волокон. Через вiдсутнiсть якого вiтамiну порушено їх гiдроксилювання?
+Вiтамiн C
-Вiтамiн K
-Вiтамiн A
-Тiамiн
-Вiтамiн E


#У хворого відмічені такі зміни: порушення зору в сутінках, підсихання кон'юнктиви та рогової оболонки. Такі порушення можуть бути при недостачі вітаміну :



-D
-В12


#У хворого спостерігається гемералопія (куряча сліпота). Яка з перерахованих речовин матиме лікувальну дію?
+Каротин
-Карнітин
-Карнозин
-Кератин
-Креатин


#При голодуванні (у другому його періоді) у хворого може підвищуватися кількістьліпідів в крові і спостерігатися гипопротеинемия. Яка форма гіперліпемії має місце в даному випадку?
+ Транспортна
-Ретенційна
-Аліментарна
-Харчова
-Перерозподільча


#Пiсля хiмiчного опiку в хворого розвинувся стеноз стравоходу. Виникло рiзке схуднення вiд затрудненого прийому їжi. У кровi: ер.- 3, 0 • 1012/л, Hb-106 г/л, загальний бiлок - 57 г/л. Який вид голодування в хворого?
+Неповне
-Бiлкове
-Повне
-Водне
-Абсолютне


#При визначеннi енерговитрат органiзму людини методом непрямої калориметрiї встановлено, що за одну хвилину споживається 1000 мл кисню i видiляється 800 мл вуглекислого газу. Який дихальний коефiцiєнт у дослiджуваної людини?
+0,8
-1,25
-0,9
-0,84
-1,0


#У людини вимiрюють енерговитрати натщесерце, лежачи, в умовах фiзичного i психiчного спокою, при температурi комфорту. В який час енерговитрати будуть найбiльшими?
+17-18 годин вечора
-7-8 годин ранку
-10-12 годин дня
-14-16 годин дня
-3-4 години ранку


#Мати зауважила занадто темну сечу у її 5-рiчної дитини. Дитина скарг не висловлює. Жовчних пiгментiв у сечi не виявлено. Поставлено дiагноз алкаптонурiя. Дефiцит якого ферменту має мiсце у дитини?
+Оксидаза гомогентизинової кислоти
-Фенiлаланiнгiдроксилаза
-Тирозиназа
-Оксидаза оксифенiлпiрувату
-Декарбоксилаза фенiлпiрувату


# У людини порушено всмоктування продуктiв гiдролiзу жирiв. Причиною цього може бути дефiцит у порожнинi тонкої кишки:
+ Жовчних кислот
-Жовчних пiгментiв
-Лiполiтичних ферментiв
-Iонiв натрiю
-Жиророзчинних вітамінів


# У дитини виявлена схильнiсть до ожирiння, яка є результатом дiатезу. Назвiть вид дiатезу, при якому частiше може розвинутись ожирiння:
+ Нервово-артритичний
-Ексудативно-катаральний
-Лiмфатико-гiпопластичний
-Астенiчний


#Пацiєнт46-тирокiв звернувся до лiкаря зi скаргами на болi в дрiбних суглобах нiг та рук. Суглоби збiльшенi, мають вигляд потовщених вузлiв. У сироватцi встановлено пiдвищений вмiст уратiв. Це може бути спричинене:
+ Порушенням обмiну пуринiв
- Порушеннямобмiну вуглеводiв
- Порушенням обмiну лiпiдiв
- Порушенням обмiну пiримiдинiв
- Порушенням обмiну амінокислот


#У хворого нормально забарвлений кал, у складi якого є велика кiлькiсть вiльних жирних кислот. Причиною цього є порушення:
+ Всмоктуванняжирiв
- Гiдролiзужирiв
- Жовчовидiлення
- Жовчоутворення
- Секрецiїлiпаз


#Хворий 55-ти рокiв хворiє на хронiчний гломерулонефрит протягом 15-ти рокiв. Якi змiни складу кровi або сечi найбiльш характерно свідчать про обмеження секреторної функцiї нирок?
+ Гiперазотемiя
- Гiперглiкемiя
-Гiпопротеїнемiя
- Протеїнурiя
- Гiпо-, iзостенурiя


#При обстеженнi чоловiка 45-ти рокiв, що тривалий час перебував на рослиннiй дiєтi, виявлено негативний азотистий баланс. Яка особливiсть рацiону стала причиноюцього явища?
+ Недостатня кiлькiсть бiлкiв
- Надмiрна кiлькiсть води
- Надмiрна кiлькiсть вуглеводiв
-Недостатня кiлькiсть жирiв
- Недостатня кiлькiсть жирiв i білків


#Пацiєнт 16-ти рокiв, що страждає на хворобу Iценко-Кушiнга, консультований з приводу надмiрної ваги тiла. При опитуваннi з’ясувалося, що енергетична цiннiсть спожитої їжi складає 1700-1900 ккал/добу. Яка провiдна причина ожирiння у даному випадку?
+ Надлишок глюкокортикоїдiв
- Нестача iнсулiну
- Надлишок iнсулiну
- Нестача глюкокортикоїдiв
- Гiподинамiя


#При алкаптонурiї вiдбувається надмiрне видiлення гомогентизинової кислоти iз сечею. С порушенням метаболiзму якої амiнокислоти пов’язано виникнення цього захворювання?
+ Тирозин
- Фенiлаланiн
- Аланiн
- Метiонiн
- Аспарагiн


#Порушення процесів мієлінізації нервових волокон призводить до неврологічних розладів і розумової відсталості. Такі симптоми характерні для спадкових і набутих порушень обміну:
+ Сфінголіпідів
- Вищих жирних кислот
- Фосфатидної кислоти
- Холестерину
- Нейтральних жирів


#У хворого діагностовано алкаптонурію. Вкажіть фермент, дефект якого є причиною цієї патології:
+Оксидаза гомогентизинової кислоти
- ДОФА-декарбоксилаза
- Глутаматдегідрогеназа
- Фенілаланінгідроксилаза
- Піруватдегідрогеназа


#Батьки дитини 3-х років звернули увагу на потемніння кольору його сечі при відстоюванні. Об'єктивно: температура у нормі, шкірні покриви чисті, рожеві, печінка не збільшена. Назвіть імовірну причину даного стану:
+Алкаптонурія
- Гемоліз
- Синдром Іценка-Кушінга
- Фенілкетонурія
-Подагра


#Причиною захворювання на пелагру може бути переважне харчування кукурудзою і зниження у раціоні продуктів тваринного походження. Відсутність у раціоні якої амінокислоти призводить до даної патології?
+Триптофан
- Метіонін
- Ізолейцин
- Фенілаланін
- Гістидин


#У фізично здорових молодих курсантів після важкого фізичного навантаження при одноденному пішому переході на 50 км в сечі виявлено білок, рівень якого в середньому не перевищував 1 г/л. Який різновид протеїнурії мав місце?
+Маршова
- Аліментарна
- Несправжня
- Дегідратаційна
- Органічна


#При лабораторному дослідженні у хворого виявили стеаторею. Вкажіть фермент, недостатність дії якого призвела до виникнення цього симптому:
+Ліпаза
- Хімотрипсин
- Пепсин
- Лактаза
- Амілаза


#У хворого на хронічний гепатит виявлено значне зниження синтезу і секреції жовчних кислот. Який процес у найбільшій мірі буде порушений у кишечнику цього хворого?
+Емульгування жирів
- Всмоктування амінокислот
- Травлення вуглеводів
- Всмоктування гліцерину
- Травлення білків


#У дитини із розумовою відсталістю встановлено зелене забарвлення сечі після додавання 5% розчину FeCl3. Про порушення обміну якої амінокислоти свідчить позитивний результат цієї діагностичної проби?
+ Фенілаланіну
-Аргініну
-Глутаміну
-Тирозину
-Триптофану


#У клiнiчно здорових батькiв народилася дитина, хвора на фенiлкетонурiю (аутосомно-рецесивне спадкове захворювання). Якi генотипи батькiв?
+Аа х Аа
-АА х АА
-АА х Аа
-Аа х аа
-аа х аа


#У чоловiка 35-ти рокiв феохромоцитома. В кровi виявляється пiдвищений рiвень адреналiну та норадреналiну, концентрацiя вiльних жирних кислот зросла в 11 разiв. Активацiя якого ферменту пiд впливом адреналiну пiдвищує лiполiз?
+ТАГ-лiпаза
-Лiпопротеїдлiпаза
-Фосфолiпаза А2
-Фосфолiпаза С
-Холестеролестераза


#У 19-мiсячної дитини iз затримкою розвитку та проявами самоагресiї, вмiст сечової кислоти в кровi - 1,96 ммоль/л. При якому метаболiчному порушеннi це спостерiгається?
+|Синдром Леша-Нiхана
-Подагра
-Синдром набутого iмунодефiциту
-Хвороба Гiрке
-Хвороба Iценко-Кушiнга


#При обстеженнi хворого виявлена характерна клiнiка колагенозу. Вкажiть, збiльшення якого показника сечi характерне для цiєї патологiї:
+Гiдроксипролiн
-Аргiнiн
-Глюкоза
-Мiнеральнi солi
-Солi амонію


#Через кілька тижнів після народження у дитини почали відмічатися прояви ураження ЦНС, шкіра та волосся посвітлішали. При доливанні до свіжої сечі 5% розчину трихлороцтового заліза з'являється оливково-зелене забарвлення. Який найбільш вірогідний діагноз?
+Фенілкетонурія
-Алкаптонурія
-Фруктозурія
-Тирозиноз
-Альбінізм


#При обстеженні дитини з олігофренією виявлено в крові і спинномозковій рідині підвищений вміст фенілаланіну і фенілпіровиноградної кислоти. Реакція сечі з трихлороцтовим залізом позитивна. Виберіть – із порушення синтезу якого ферменту пов'язаний розвиток захворювання.
+Фенілаланінгідроксилаза.
-Оксидаза п-гідроксифенілпіровиноградної кислоти.
-Тирозиназа.
-Дофамінгідроксилаза.
-Оксидаза гомогентизинової кислоти.


#У дитини 3 років, яка страждає на квашіоркор, спостерігається порушення ороговіння епідермісу та збільшення його злущення, є жирова інфільтрація печінки. Який тип голодування спостерігається пр цьому?
+Білкове.
-Вуглеводне.
-Мінеральне
-Енергетичне.
-Жирове


#До лікаря звернувся чоловік 65 років з скаргами на гострий біль в великих пальцях ніг. Він любить та часто вживає пиво. Виникло підозріння на подагру. Для підтвердження діагнозу вміст якої із перелічених речовин необхідно визначити у крові?
+сечової кислоти
-сечовини
-лактату
-білірубіну
-кетонових тіл


#Чоловік 42 років, що страждає ожирінням за верхнім типом (плечовий пояс, лице місяцеподібне), АТ - 160/95 мм рт.ст., глюкоза крові 8.0 ммоль/л. Вміст кортизолу в крові підвищений, а адренокортикотропін знижений. Яка найбільш вірогідна причина розвитку гіперкортицизму?
+гормонопродуктуюча пухлина кори наднирників
-гормонопродуктуюча пухлина передньої ділянки гіпофізу
-зменшення продукції статинів
-збільшення продукції кортиколіберину
-зменшення вироблення статевих гормонів


#Відомо, що фенілкетонурія виникає внаслідок мутації гена, що відповідає за


перетворення амінокислоти фенілаланіну і розпаду її до кінцевих продуктів обміну – СО2 і Н2О. Вкажіть, який шлях обміну фенілаланіну приведе до розвитку фенілкетонурії.
+Фенілаланін - фенілпіруват - кетокислоти
-фенілаланін - тирозин - тироксин
-фенілаланін - тирозин - меланін
-фенілаланін - тирозин - норадреналін
-фенілаланін - тирозин - алкаптон


#Дитина, хвора на фенілкетонурію, страждає на розумову відсталість. Який механізм буде головним у розвитку пошкодження центральної нервової системи?
+Накопичення в крові фенілаланіну і фенілкетонів
-Підвищення синтезу тирозину
-Зниження синтезу меланіну
-Зниження синтезу тиреоїдних гормонів
-Збільшення екскреції з сечею фенілкетонових тіл


#Чоловік 67 років страждає на атеросклероз судин головного мозку. При обстеженні знайдена гіперліпідемія. Вміст якого класу ліпопротеїдів плазми крові найбільш вірогідно буде значно підвищений при біохімічному дослідженні?
+Ліпопротеїди низької щільності
-Хіломікрони
-Ліпопротеїди дуже низької щільності
-Ліпопротеїди високої щільності
-Комплекси жирних кислот з альбумінами


#Хворий скаржиться на періодичне ослаблення стула, яке пов’язує з прийомом багатої на жири їжі. При цьому він відмічає зменшення забарвленості калу. При лабораторному обстеженні встановлено нормальний вміст ліпідів в сироватці крові. Порушення якого із станів ліпідного обміну має місце у даного хворого?
+Всмоктування.
-Транспорту кров’ю.
-Проміжного обміну.
-Депонування в жировій тканині.
-Мобілізація із жирової тканини.


#У хворого діагностована пелагра. Порушення обміну якої амінокислоти лежить в основі цього захворювання?
+ Триптофану
-Тирозину
-Фенілаланіну
-Цистеїну
-Валіну


#Пацієнт 46-ти років звернувся до лікаря зі скаргами на болі в дрібних суглобах ніг та рук. Суглоби збільшені, мають вигляд потовщених вузлів. У сироватці встановлено підвищений вміст уратів. Це може бути спричинене:
+Порушенням обміну пуринів
-Порушенням обміну амінокислот
-Порушенням обміну вуглеводів
-Порушенням обміну ліпідів
-Порушенням обміну піримідинів


#У жінки 37-ми років протягом року періодично виникали інфекційні захворювання бактеріального генезу, їх перебіг був вкрай тривалим, ремісії - короткочасними. При обстеженні виявлена гіпогамаглобулінемія. Порушення функції яких клітин може бути прямою її причиною?
+Плазматичні клітини
-Лімфоцити
-Макрофаги
-Нейтрофіли
-Фагоцити


#Електрофоретичне дослідження сироватки крові хворого пневмонією показало збільшення одної з білкових фракцій. Вкажіть її:
+γ-глобуліни
-α1-глобуліни
-α2-глобуліни
-β-глобуліни
-Альбуміни


#Спадкова гіперліпопротеїнемія І типу обумовлена недостатністю ліпо-протеїнліпази. Підвищення рівня яких транспортних форм ліпідів в плазмі навіть натщесерце є характерним?
+Хіломікрони
-Ліпопротеїни високої густини
-Ліпопротеїни дуже низької густини
-Ліпопротеїни низької густини
-Модифіковані ліпопротеїни


# У кровi чоловiка 26-ти рокiв виявлено 18% еритроцитiв сферичної, сплощеної, кулястої та остистої форм. Iншi еритроцити були у формi двоввiгнутих дискiв. Як називається таке явище?
+Фiзiологiчний пойкiлоцитоз
-Патологiчний пойкiлоцитоз
-Фiзiологiчний анiзоцитоз
-Патологiчний анiзоцитоз
-Еритроцитоз


#Крива дисоцiацiї оксигемоглобiну змiщена вправо. Якi змiни в органiзмi людини можуть бути причиною цього?
+Гiпертермiя
-Збiльшення концентрацiї 2,3- дифосфоглiцерату в еритроцитах
-Алкалоз
-Гiпокапнiя
-Гiпоксемiя


#У хворого в крові: ер.- 3,0*1012/л; Hb- 90г/л; ретикул.- 0,5%. В мазку: пойкілоцити, гіпохромні еритроцити. Залізо сироватки крові - 80 мкмоль/л. Для якої патології це характерно?
+Залізорефрактерна анемія
-Серпоподібноклітинна анемія
-B12-дефіцитна анемія
-Залізодефіцитна анемія
-Хвороба Мінковського-Шоффара


#При аналізі крові у спортсмена вияв­лено: ер,- 5,5 • 1012/л, НЬ- 180 г/л, лейк,- 7 • 109/л, н,- 64%, б,- 0,5%, е,- 0,5%, м,- 8%, л,- 27%. Такі показники свідчать про сти­муляцію, перш за все:
+Еритропоезу
-Імуногенезу
-Гранулоцитопоезу
-Лімфопоезу
-Лейкопоезу


#Мати звернулася до лікаря з приводу того, що у дитини 5-ти років під дією сонячних променів на шкірі з'являються еритеми, везикулярний висип, свербіж шкіри. Лабораторні дослідження виявили зменшення вмісту заліза у сироватці крові, збільшення виділення з сечею уропорфіриногену І. Найбільш вірогідною спадковою патологією у дитини є:
+Еритропоетична порфірія
-Копропорфірія
-Печінкова порфірія
-Метгемоглобінемія
-Інтермітуюча порфірія


#До приймального відділення доставлено жінку 38-ми років з матковою кровотечею, що триває другу добу. Що з наведеного буде виявлено при аналізі крові хворої?
+Зменшення гематокритного показника
-Лейкоцитоз
-Еозинофілія
-Уповільнення ШОЕ
-Збільшення колірного показника


#Пацiєнтка 58-ми рокiв скаржиться на пiдвищену втомлюванiсть, зниження працездатностi, сонливiсть, задишку пiд час швидкої ходи. У кровi: ер.-4, 6 • 1012/л, Hb- 92 г/л, КП- 0,6. У мазку кровi - велика кiлькiсть анулоцитiв та мiкроцитiв. Для якої анемiї це характерно?
+Залiзодефiцитна
-Постгеморагiчна
-Гемолiтична
-Пернiцiозна
-Серповидноклiтинна


#У чоловiка 43-х рокiв з видаленою ниркою були виявленi симптоми анемiї. Що зумовило появу цих симптомiв?
+Зниження синтезу еритропоетинiв
-Пiдвищене руйнування еритроцитiв
-Нестача залiза
-Нестача вiтамiну B12
-Нестача фолiєвої кислоти


#Жiнка 55-ти рокiв звернулася зi скаргами на тривалi циклiчнi матковi кровотечi протягом року, слабкiсть, запаморочення. Об’єктивно: блiдiсть шкiри. У кровi: Hb- 70 г/л, ер.- 3, 2 • 1012/л, КП- 0,6, лейк.- 6, 0 • 109/л, ретикулоцити - 1%; гiпохромiя еритроцитiв. Яка анемiя у хворої?
+Хронiчна постгеморагiчна
-Гемолiтична
-Апластична
-B12-фолiєводефiцитна
-Залiзодефiцитна


#У людини до травми гематокритний показник 40%. Яким вiн буде через добу пiсля втрати 750 мл кровi?
+30%
-40%
-55%
-45%
-50%


#При аналiзi кровi у спортсмена виявлено: ер.- 5, 5 • 1012/л, Hb- 180 г/л, лейк.- 7•109/л, н.- 64%, б.- 0,5%, е.- 0,5%, м.- 8%, л.- 27%. Такi показники свiдчать про стимуляцiю, перш за все:
+Еритропоезу
-Лейкопоезу
-Лiмфопоезу
-Гранулоцитопоезу
-Iмуногенезу


#В пробiрку, що мiстить розчин NaCl 0,9%, додали краплю кровi. Що вiдбудеться з еритроцитами?
+Залишаться без змiн
-Осмотичний гемолiз
-Бiологiчний гемолiз
-Зморшкування
-Набряк


#Чоловік 47 років скаржиться на слабкість, запаморочення . Півроку назад переніс операцію з приводу резекції шлунку Аналіз крові: Hb-80г/л, еритроцити-3.5 Т/л, кольоровий показник-0.69, ШОЕ-15 мм/год. Сироваткове залізо-5.4 мкмоль/л. Гіпохромія еритроцитів. Яка анемія найбільш вірогідно має місце у хворого?
+залізодефіцитна анемія
-сидероахрестична анемія
-геморагічна анемія
-гемолітична анемія
-В12 -фолієводефіцитна анемія


#У хворого з анацидним гастритом при дослідженні крові отримані наступні


дані. Ер.- 3,0.Т / л; Нв-62 г / л; Ц.П-0,6; ретікулоц.-0,1%; тромбоц.-260 Г / л. Лейкоц.-5,6 Г / л. Б-0, Е-З, Мц-О, Ю-О, П-З, С-66, Л-25, Мо-З. Анізоцитоз-микроцитоз виражений, ШОЕ-10 мм / год. Про яку форму патології крові можна думати на підставі даних цього аналізу?
+Залізодефіцитна анемія
-В12 дефіцитна анемія
-мікросфероцітоз
-Гостра постгеморагічна анемія
-Апластична анемія


#Жінка 37 років скаржиться на загальну слабкість, часті запаморочення, утруднення ковтання їжі, бажання їсти крейду. Шкіра та видимі слизові оболонки бліді. В крові: eр.- 3,4*1012/л, Hb- 70 г/л, КП- 0,7, ретик.- 0,1%, лейк.- 4,7*109/л, е.- 2%, п.- 3%, с.- 64%, л.- 26%, м.- 5%. ШOЕ- 15 мм/г. Сироваткове залізо - 7,3 мкмоль/л. Дефіцит якої речовини обумовив виникнення захворювання?
+Заліза.
-Білка.
-Вітаміну В6.
-Вітаміну В12.
-Фолієвої кислоти.


#У дитини, що знаходиться на штучному вигодовуванні коров'ячим молоком,


розвинулася важка анемія: еритроцити - 3,4 * 1012 / л, Нb - 68 г / л, ретикулоцити - 0%. Яка анемія розвинулася у дитини?
+Залізодефіцитна
-B12-дефіцитна
-Вроджена гемолітична
-Гипопластична
-Cерповидно-клітинна


#Хворий переніс операцію з приводу резекції пілоричного відділу шлунку. Через рік скаржиться на слабкість, періодичну появу темних кіл під очима, задишку. В крові: Hb - 70г/л, ер. - 3,0*1012/л. Які зміни еритроцитів в мазках периферичної крові характерні для даного захворювання?
+Гіпохромні еритроцити
-Еритроцити з тільцями Кебо
-Еритроцити з тільцями Жоллі
-Гіперхромні еритроцити
-Макроцити


#На п"яту добу післі гострої крововтрати у хворого діагностована гіпохромна анемія. Який головний механізм у розвитку гіпохромії?
+Надходження з кісткового мозку незрілих еритроцитів
-Порушення всмоктування заліза у кишечнику
-Посилене руйнування еритроцитів у селезінці
-Порушення синтезу глобіну
-Підвищення виділення заліза з організму


#На шостому місяці вагітності в жінки з’явилася виражена залізодефіцитна анемія. Діагностичною ознакою її була поява в крові
+Анулоцитів
-Макроцитів
-Пойкілоцитів
-Ретикулоцитів
-Нормоцитів


#У дитини, що отримав в результаті необережного поводження з вогнем термічні опіки до 40% поверхні тіла, показник гематокриту виявляє порушення співвідношення плазми і формених елементів. Яка форма порушення загального обсягу крові спостерігається при цьому?


A * Поліцитемічна гіповолемія


B Поліцитемічна гіперволемія


C Олігоцитемічна гіповолемія


D Нормоцитемічна гіповолемія


E Олігоцитемічна гіперволемія


#На останньому місяці вагітності вміст фібриногену в плазмі крові в 2 рази вище за норму. Яку швидкість осідання еритроцитів слід при цьому очікувати?
+40-50 мм/годину
-0-5 мм/годину
-10-15 мм/годину
-3-12 мм/годину
-5-10 мм/годину


#У чоловіка 25-ти років на 4-й день після крововтрати в крові зросла кількість поліхроматофільних еритроцитів, ретикулоцитів і навіть з’явились поодинокі нормобласти. Про що свідчить такі гематологічні зміни?
+Процеси регенерації випереджають дозрівання клітин
-Зменшено об’єм плазми крові
-Знижена продукція еритропоезу в нирках
-Ослаблені процеси еритропоезу
-Підвищена спорідність гемоглобіну до кисню


#У хворої 36-ти рокiв, яка лiкувалася сульфанiламiдами з приводу респiраторної вiрусної iнфекцiї, в кровi гiпорегенераторна нормохромна анемiя, лейкопенiя, тромбоцитопенiя. В кiстковому мозку - зменшення кiлькостi мiєлокарiоцитiв. Яка це анемiя?
+Гiпопластична
-Гемолiтична
-Постгеморагiчна
-12-фолiєводефiцитна
-Залiзодефiцитна


#У хворого, що перенiс 5 рокiв тому субтотальну резекцiю шлунка, розвинулась В12-фолiєводефiцитна анемiя. Який механiзм є провiдним у розвитку такої анемiї?
+Вiдсутнiсть внутрiшнього фактора Касла
-Вiдсутнiсть зовнiшнього фактора Касла
-Порушення всмоктування вiтамiну В12 в тонкiй кишцi
-Дефiцит фолiєвої кислоти
-Дефiцит транскобаламiну


#У чоловiка 50-ти рокiв при обстеженнi було виявлено зниження кiлькостi еритроцитiв у кровi та пiдвищення рiвня вiльного гемоглобiну в плазмi кровi (гемоглобiнемiя). КП становив 0,85. Який вид анемiї спостерiгається у хворого?
+Набута гемолiтична
-Спадкова гемолiтична
-Гостра постгеморагiчна
-Хронiчна постгеморагiчна
-Анемiя внаслiдок порушення еритропоезу


#До клiнiки поступив чоловiк 40-ка рокiв, якого укусила гадюка. Де переважно буде проходити гемолiз еритроцитiв у цьому випадку?
+У кровоносному руслi
-У клiтинах печiнки
-У клiтинах селезiнки
-У кiстковому мозку
-У паренхiмi нирок


#У хворого в анамнезі: з дитинства відмічався знижений рівень гемоглобіну. Лікування препаратами заліза не дає ефекту. У крові: ер.- 3,1*1012/л, ретик.- 16%, Hb- 85 г/л, КП- 0,75; в мазку крові анізоцити, пойкілоцити, мішенеподібні еритроцити, еритроцити з базофільною зернистістю, рівень заліза у сироватці 30 мкмоль/л. Для якої патології системи крові характерні такі дані?
+Таласемія
-Гіпопластична анемія
-B12-дефіцитна анемія
-Залізодефіцитна анемія
-Фолієводефіцитна анемія


#У хворого спадкова гемолітична анемія Мінковського-Шофара. Виявлення у крові яких характерних клітин надало можливість лікарю встановити діагноз?
+Мікросфероцити
-Поліхроматофіли
-Анізоцити
-Пойкілоцити
-Мегалоцити


#Після видалення у пацієнта 2/3 шлунка у крові зменшився вміст гемоглобіну, кількість еритроцитів, збільшилися розміри цих клітин крові. Дефіцит якого вітаміну призводить до таких змін у крові?
+B12
-P
-PP
-C
-B6


#У хворої 19-ти років з дитинства від­мічалося зниження гемоглобіну до 90-95 г/л. У крові під час госпіталізації: ер,- 3,2 • 1012/л, НЬ- 85 г/л, КП- 0,78; лейк,-5,6• 109/л, тромб.- 210 • 109/л. В мазку: анізоцитоз, пойкілоцитоз, мішенеподібні еритроцити. Ретикулоцити - 6%. Лікуван­ня препаратами заліза було неефективне. Яку патологію системи крові можна запі­дозрити в даному випадку?
+Таласемія
-Ферментопатія
-Фавізм
-Серпоподібноклітинна анемія
-Мембранопатія


#Хворий 20-ти рокiв скаржиться на загальну слабкiсть, запаморочення, швидку втомлюванiсть. У кровi: Hb- 80 г/л. Мiкроскопiчно: еритроцити змiненої форми. Причиною цього стану може бути:
+Серпоподiбноклiтинна анемiя
-Паренхiматозна жовтяниця
-Гостра перемiжна порфiрiя
-Обтурацiйна жовтяниця
-Хвороба Аддiсона


#Чоловiк 56-ти рокiв потрапив до клiнiки зi скаргами на загальну слабкiсть, бiль i печiння в язицi, вiдчуття онiмiння в кiнцiвках. У минулому перенiс резекцiю кардiального вiддiлу шлунка. У кровi: Hb- 80 г/л; ер.- 2, 0 • 1012/л; КП-1,2, лейк.- 3, 5 • 109/л. Який вид анемiї у цього хворого?
+B12-фолiєводефiцитна
-Гемолiтична
-Постгеморагiчна
-Апластична
-Залiзодефiцитна


|#У чоловiка 30-ти рокiв перед операцiєю визначили групову належнiсть кровi. Кров резус-позитивна. Реакцiю аглютинацiї еритроцитiв не викликали стандартнi сироватки груп 0 ?? (I), А? (II), В? (III). Дослiджувана кров належить до групи:
+0 (αβ) (I)
-А (β) (II)
-В (α) (III)
-АВ (IV)
-Немає вірної відповіді


#Через рiк пiсля субтотальної резекцiї шлунка з приводу виразки малої кривизни виявленi змiни в аналiзi кровi - анемiя, лейко- i тромбоцитопенiя, КП-1,3, наявнiсть мегалобластiв та мегалоцитiв. Дефiцит якого фактору обумовив розвиток цiєї анемiї?
+Фактор Касла
-Хлороводнева кислота
-Муцин
-Пепсин
-Гастрин


#Хворий (28 років) прибув у стаціонар зі скаргами на біль у животі, нестійкий стул, слабкість, втому, віддишку. В анамнезі: 2 роки тому операція з приводу гострої кишкової непрохідності з резекцією 60 см тонкої кишки. При вступі в аналізі крові: кількість еритроцитів 2,4*1012/л, ретикулоцитів 0,4%, гемоглобіну 80 г/л, КП 1,25; в мазку крові макроанізоцити, пойкілоцити, шизоцити поодинокі мегалоцити, мегалобласти. Для якої патології системи крові характерні ці дані?
+В12-дефіцитна анемія
-Залізодефіцитна анемія
-Гіпопластична анемія
-Гемолітична анемія
-Хронична постгеморагична анемія


#У жінки на 6-му місяці вагітності при обстеженні виявлено знижену кількість еритроцитів і гемоглобіну, кольоровий показник – 1,4. В мазку з'явилися мегалоцити, поодинокі оксифільні мегалобласти. Який вид анемії за патогенезом найбільш імовірний в даному випадку?
+В12 і фолієводефіцитна анемія.
-Залізодефіцитна анемія.
-Мієлотоксична анемія.
-Апластична анемія.
-Метапластична анемія.


#Хворий 57 років скаржиться на слабкість, серцебиття, задишку при виконанні нетяжкої роботи. В крові: еритроцити - 0,79 Т/л, гемоглобін - 40 г/л, КП - 1,45, лейкоцити - 3,4 Г/л. В мазку: анізоцитоз і пойкілоцитоз еритроцитів, мегалобласти і мегалоцити. Яка анемія найбільш ймовірна у цього хворого?
+В12- фолієводефіцитна
-Постгеморагічна
-Залізодефіцитна
-Гемолітична
-Апластична


#У пацієнта, носія спадкової серповидної аномалії еритроцитів, захворювання пневмонією супроводжувалося гемолітичним кризом і розвитком анемії. Що є безпосередньо причиною гемолітичного кризу в даному випадку?
+Гіпоксія, викликана пневмонією
-Зміна осмолярності крові
-Гетерозиготність по Нb S
-Мутація структурного гена
-Гіпероксія


#У жінки на 7-му місяці вагітності стала швидко наростати анемія: еритроцити -2,7 * 1012 / л, Нb -90 г / л, анізоцитоз, кількості, поодинокі мегалобластні мегалоцити, ретикулоцити - 0%. Який вид анемії розвинувся в даному випадку?
+B12-дефіцитна
-Залізодефіцитна
-Гемолітична
-Постгеморагічна
-Талассемія


#Хвора звернулась в клініку зі скаргами на слабкість, задишку, швидку стомлюваність, запаморочення. В крові: ер. - 1,8*1012/л; Hb - 80 г/л; к.п. - 1,5; лейк. - 3,2*109/л. У мазку: анізоцитоз, пойкілоцитоз, мегалобласти мегалоцити. Який найбільш вірогідний діагноз?
+В12-дефіцитна анемія
-Залізодефіцитна анемія
-Постгеморагічна анемія
-Iмуногемолітична анемія
-Гострий лейкоз


#Хвора 3 р., поступила в дитячу клініку у важкому стані з гемоглобінопатією (серпоподібноклітинна анемія). Заміна якою аминокислотою глутамінової кислоти в бета–ланцюгу глобіну лежить в основї утворення патологічного гемоглобіну в даному випадку?
+валіном.
-серином
-тирозином
-фенілаланіном
-аргініном


#Хвора поступила в клініку на обстеження. З дитинства відмічалось зниження гемоглобіну до 90-95 г/л. Лікування препаратами заліза було неефективне. Аналіз крові при поступленні: Е–3,2х1012/л, Hb–85 г/л, к.п.–0,78. В мазку анізоцитоз, пойкілоцитоз, мішеневидні еритроцити, ретикулоцити –16%. Поставлений діагноз – таласемія. До якого виду гемолітичних анемій можна віднести дане захворювання?
+Спадкова гемоглобінопатія.
-Спадкова мембранопатія
-Набута мембранопатія
-Спадкова ферментопатія
-Набута ферментопатія


#Y хворого, що прибув із Тунісу, виявлена альфа-таласемія з гемолізом еритроцитів і жовтяницею. Хвороба була діагностована на основі наявності в крові
+Мішенеподібних еритроцитів
-Зернистих еритроцитів
-Поліхроматофільних еритроцитів
-Нормоцитів
-Ретикулоцитів


#У хворого з гемолітичною жовтяницею в мазку крові присутні еритроцити у вигляді мікросфероцітов 1 - 6 в полі зору. Яка можлива причина гемолізу еритроцитів, що викликає виникнення такої форми жовтяниці?
+Спадковий дефект розвитку їх мембран
-Спадковий дефект структури гемоглобіну
-Порушення ферментних систем еритроцитів
-Вплив на мембрану еритроцитів жовчних кислот
-Вплив на мембрану еритроцитів білірубіну


#В аналізі крові 35-річного хворого: Нв – 58 г/л, еритроцити – 1,3х1012/л, колірний показник – 1,3, лейкоцити – 2,8х109/л, тромбоцити – 1,1х109/л, ретикулоцити – 2%0, ШОЕ – 35 мм/час. Визначаються полісегментіровані нейтрофіли, а також тільця Жоллі, кільця Кебота, мегалоцити. Яка це анемія?
+В12 – фолієводефіцитна.
-Гіпопластична.
-Постгеморагічна.
-Гемолітична.
-Залізодефіцитна.


#У хворого після резекції шлунка розвинулася В-12 фолієводефіцитна анемія. Який із перерахованих кольорових показниівк характерний для цієї патології?
+1,4
-1,0
-0,8
-0,5
-0,2


#У хворого 35 років розвинулася імунна гемолітична анемія. Який показник сироватки крові зросте в найбільшій мірі?
+Непрямий білірубін
-Прямий білірубін
-Стеркобіліноген
-Мезобіліноген
-Протопорфірин


#У чоловіка 40-ка років було встановлено діагноз: серпоподібноклітинна анемія. Який механізм приводить до зменшення кількості еритроцитів в крові у цього хворого?
+Позасудинний гемоліз
-Внутрішньосудинний гемоліз
-Нестача білка
-Нестача вітаміну B12 і фолієвої кислоти
-Нестача заліза в організмі


#У хворого з гемолітичною анемією виявлено дефіцит піруваткінази в еритроцитах. За цих умов причиною розвитку гемолізу еритроцитів є:
+Зменшення активності Na+, К
+ -АТФ-ази
-Генетичні дефекти глікофорину А
-Дефіцит спектрину
-Надлишок К
+ в еритроцитах
-Нестача Na
+ в еритроцитах


#При обстеженнi в аналiзi кровi пацiєнтавиявленолейкоцитоз,лiмфоцитоз, клiтини Боткiна-Гумпрехта на тлi анемiї. Про яку хворобу слiд думати лiкарю?
+Хронiчний лiмфолейкоз
-Гострий мiєлолейкоз
-Лiмфогранулематоз
-Iнфекцiйний мононуклеоз
-Мiєломна хвороба


#У хворого скарги на загальну слабкiсть, пiдвищену втому, зниження апетиту i маси тiла. В анамнезi частi пневмонiї. На пiдставi клiнiчних даних та результатiв дослiдження периферiйної кровi у нього дiагностовано хронiчний лiмфолейкоз. Якi дегенеративнi змiни лейкоцитiв характернi для даного захворювання?
+Тiнi Боткiна-Гумпрехта
- Тiльця Князьковi-Деле
- Зерна Амато
- Палички Ауера
- Токсична зернистість


#Пiсля прийому сульфанiламiдiв у хворого виникли лихоманка, блювання i стул з кров’ю. У кровi: лейк.- 0, 9 · 109/л (гранул.- 0, 7 · 109/л), лейкоаглютинiни. Який з термiнiв найбiльш точно характеризує виявленi змiни у кровi?
+ Агранулоцитоз
- Лейкопенiя
- Лейкоз
- Гемодилюцiя
- –


#У хворого в лейкограмі: лейкоцити – 14-109/л; мієлобласти - 71%, промієлоцити, мієлоцити, метамієлоцити - 0%, паличкоядерні нейтрофіли - 6%, сегментоядерні - 13%; лімфоцити - 7%, моноцити - 3%. Яка патологія у хворого?
+Мієлобластний лейкоз
-Хронічний лімфолейкоз
- Хронічний мієлолейкоз
- Лімфобластний лейкоз
- Нейтрофільний лейкоцитоз


#У хворого через добу після апендектомії при аналізі крові виявили нейтрофільний лейкоцитоз з регенеративним зсувом вліво. Який найбільш імовірний механізм розвитку абсолютного лейкоцитозу у периферичній крові хворого?
+ Посилення лейкопоезу
- Зменшення руйнування лейкоцитів
- Активація імунітету
- Перерозподіл лейкоцитів в організмі
- Уповільнення еміграції лейкоцитів у тканині


#Чоловік 26-ти років перебуває в торпідній стадії шоку внаслідок автомобільної аварії. В крові: лейк,- 3,2 • 109/л. Який го­ловний механізм в розвитку лейкопенії?
+Перерозподіл лейкоцитів у судинному руслі
-Підвищення виділення лейкоцитів з організму
-Порушення виходу зрілих лейкоцитів з кісткового мозку в кров
-Пригнічення лейкопоезу
-Руйнування лейкоцитів у кровотворних органах


#У хворого 70-ти років атеросклероз ускладнився тромбозом судин нижніх кінцівок, виникла гангрена пальців лівої стопи. Початок тромбоутворення, найбільш вірогідно, пов'язаний з:
+Адгезією тромбоцитів
- Перетворенням фібриногену в фібрин
- Зниженням синтезу гепарину
- Активацією протромбінази
- Перетворенням протромбіну в тромбін


#Пацієнт звернувся до лікаря зі скаргами на задишку, що виникала після фізичного навантаження. Клінічне обстеження виявило анемію та наявність парапротеїну в зоні гамма-глобулінів. Який показник у сечі необхідно визначити для підтвердження діагнозу мієломи?
+ Білок Бенс-Джонса
- Антитрипсин
- Церулоплазмін
- Гемоглобін
- Білірубін


#У студента через 2 години пiсля iспиту в аналiзi кровi виявлено лейкоцитоз без iстотних змiн у лейкоцитарнiй формулi. Який найбiльш вiрогiдний механiзм розвитку лейкоцитозу?
+Перерозподiл лейкоцитiв в органiзмi
-Посилення лейкопоезу
-Уповiльнення руйнування лейкоцитiв
-Уповiльнення мiграцiї лейкоцитiв у тканини
-Посилення лейкопоезу та зменшення руйнування лейкоцитiв


#Хворий 23-х рокiв скаржиться на слабкiсть, пiдвищення температури до 38 - 400C. Об’єктивно: печiнка i селезiнка збiльшенi. У кровi: Hb- 100 г/л, ер.- 2, 9 • 1012/л, лейк.- 4, 4 • 109/л, тромб.-48 • 109/л, нейтрофiли сегментоядернi - 17%, лiмфоцити - 15%, бластнi клiтини - 68%. Всi цитохiмiчнi реакцiї негативнi. Дайте гематологiчний висновок:
+Недиференцiйований лейкоз
-Хронiчний мiєлолейкоз
-Гострий мiєлобластний лейкоз
-Гострий лiмфобластний лейкоз
-Гострий еритромiєлоз


#У хворого в лейкограмi: лейкоци-ти - 14 • 109/л; мiєлобласти - 71%, про-мiєлоцити, мiєлоцити, метамiєлоцити - 0%, паличкоядернi нейтрофiли - 6%, сегментоядернi - 13%; лiмфоцити - 7%, моноцити - 3%. Яка патологiя у хворого?
+Мiєлобластний лейкоз
-Нейтрофiльний лейкоцитоз
-Хронiчний мiєлолейкоз
-Лiмфобластний лейкоз
-Хронiчний лiмфолейкоз


#При обстеженнi в аналiзi кровi пацiєнта виявлено лейкоцитоз, лiмфоцитоз, клiтини Боткiна-Гумпрехта на тлi анемiї. Про яку хворобу слiд думати лiкарю?
+Хронiчний лiмфолейкоз
-Гострий мiєлолейкоз
-Лiмфогранулематоз
-Iнфекцiйний мононуклеоз
-Мiєломна хвороба


#У хворого 42 років при дослідженні периферичної крові виявлено: гемоглобін 80 г/л, еритроцитів 3,2 Т/л, лейкоцитів 25 Г/л; лейкоцитарна формула: базофілів - 5%, еозинофілів - 9%, міелобластів - 3%, проміелоцитів - 8%; нейтрофілів: міелоцитів - 11%, метаміелоцитів - 22%, паличкоядерних - 17%, сегментоядерних - 19%, лімфоцитів - 3%, моноцитів - 3%. Яка патологія крові найбільш вірогідна у хворого:
+Хронічний міелолейкоз
-Міелобластний лейкоз
-Еритроміелоз
-Проміелоцитарний лейкоз
-Панміелофтіз


#Ер.-3,1 Т / л Нв-90 г / л; K.П.-0,88; Тромбоцит. -110 Г / л; лейкоцит. -51 Г / л. Б-О, Е-I, Мц-О, Ю-О, П-I, С-24, Л-70, Мо-2, лімфобластів-2% .Тені Боткіна-Гумпрехта. ШОЕ 27 мм / год. Який вид патології супроводжується такими змінами в крові?
+Хронічний лімфолейкоз
-Гострий лімфолейкоз
-Туберкульоз
- Бруцельоз
-Сифіліс


#Ер .-3,5 Т / л Нв-110 г / л; K.П.-0,9; Тромбоцит. -100 Г / л; лейкоцит. -80 Г / л. Б-О, Е-О, Мц-О, Ю-0, П-0, С-30, Л-2, Мо-0, мієлобласти-68%. ШОЕ 25 мм / год. Для якого виду лейкозу характерний представлений аналіз?
+ Мієлобластний
-Хронічний мієлоїдний
-недіфференціруемого
-Хронічний лімфоїдний
-Моноцитарний


#У хворого на атрофічний гастрит виник дефіцит вітаміу В12. Яка зміна лейкоцитарної формули є найбільш типовою для гіповітамінозу В12?
+Ядерний зсув вправо
-Дегенеративний зсув вліво
-Гіперрегенеративний зсув вліво
-Регенеративно-дегенеративний ядерний зсув вліво
-Регенеративний ядерний зсув вліво


#У хворого С., виявлено такі зміни в периферичної крові: Ер. 3,2x1012 / л, Гем.80 г / л, Лейк. 25x109 / л. Лейкоцитарна формула: базофіли - 5%, еозинофіли - 9%, мієлобласти - 3%, проміелоціти - 8%; міелоціти - 11%, метамієлоцити - 22%, паличкоядерні - 17%, сегментоядерні - 19%, лімфоцити - 3%, моноцити - 3%. Визначте найбільш ймовірну патологію яка відповідає певному опису картини крові:
+хронічний мієлолейкоз
-гострий мієлобластний лейкоз
- ерітромієлоз
-лейкемоїдна реакція
-недеферінціруемий лейкоз


#У хворого П., виявлені такі зміни в периферичної крові: Ер. 3,0x1012 / л, Гем 80 г / л, Лейк. 21x109 / л. Лейкоцитарна формула: базофіли - 0%, еозинофіли - 0%/ мієлобласти - 54%, проміелоціти - 1%; міелоціти - 0%, метамієлоцити - 0%, паличкоядерні - 1%, сегментоядерні - 28%, лімфоцити - 13%, моноцити - 3%. Визначте найбільш ймовірну патологію яка відповідає певному опису картини крові:
+ гострий мієлобластний лейкоз
-хронічний мієлолейкоз
-ерітромієлоз
-лейкемоїдна реакція
-недеферінцірованний лейкоз


#Хворий на протязі останнього року став відмічати під_вищену втомлюваність, загальну слабість. Аналіз крові: Е– 4.1х1012/л, Hb–119 г/л, к.п.–0.87, лейкоцити – 57х109/л, лейкоформула: Ю–0, П–0, С–9%, Е–0, Б–0, лімфобласти–2%, пролімфоцити–5%, лімфоцити–81%, М–3%, тромбоцити – 160х109/л. В мазку: нормохромія, велика кількість тіней Боткіна–Гумпрехта. Про яку патологію системи крові свідчить дана гемограма?
+ Хронічний лімфолейкоз.
-Хронічний мієлолейкоз
-Гострий лімфобластний лейкоз
-Гострий мієлобластний лейкоз
-Хронічний монолейкоз


#У хворого в обох щелепах рентгенологічно виявлено численні дефекти у вигляді гладкостінних округлих отворів. При гістологічному дослідженні - явища остеолізису і остеопорозу при явищах слабкого кісткоутворення. В сечі хворого знайдено білок Бенс-Джонса. Назвіть захворювання:
+Мієломна хвороба
-Гострий мієлолейкоз
-Гострий недиференційований лейкоз
-Хронічний еритромієлоз
-Хронічний мієлолейкоз


#Хворий 62-х років блідий, всі групи лімфовузлів збільшені. В крові: Hb- 60 г/л, еритроцити - 1,9 Т/л, лейкоцити - 29 Г/л, тромбоцити - 110 Г/л. Лейкоцитарна формула: сегментоядерні лейкоцити - 10%, лімфоцити - 8%, моноцити - 2%, бластних клітин - 80%. Цитохімічні дослідження бластних клітин: позитивна реакція на глікоген, негативна - на ліпіди і пероксидазу. Дайте заключення про патологію:
+Гострий лімфобластний лейкоз
-Гострий мієлобластний лейкоз
-Гострий мегакаріоцитарний лейкоз
-Гострий монобластний лейкоз
-Гострий промієлоцитарний лейкоз


#Autopsy of the body of a 48-year-old man shows that the bone marrow in the flat bones, as well as in the cylindrical bone diaphyses and epiphyses, is moist, colored gray-red or gray-yellow, and puri-form (pyoid bone marrow). The spleen weight is 7 kg; it is dark red on section, with signs of ischemic infarctions. All the lymph nodes are enlarged, soft, and gray-red in color. In the liver there are signs of fatty degeneration and leukemic infiltrates. What is the most likely diagnosis?
+Chronic myeloid leukemia
-Acute lymphoid leukemia
-Acute myeloid leukemia
-Lymphogranulomatosis
-Multiple myeloma


#У хворого iз захворюванням печiнки виявлено зниження вмiсту протромбiну вкровi. Це призведе, першзавсе, допорушення:
+ Другої фази коагуляцiйного гемостазу
- Першої фази коагуляцiйного гемостазу
- Судинно-тромбоцитарного гемостазу
- Фiбринолiзу
- Антикоагулянтних властивостей кровi


#У хворого 37-ми рокiв на фонi тривалого застосування антибiотикiв спостерiгається пiдвищена кровоточивiсть при невеликих пошкодженнях. У кровi зниження активностi факторiв згортання кровi II, VII, IX, X, подовження часу згортання кровi. Недостатнiстю якого вiтамiну обумовленi зазначенi змiни?
+ Вiтамiн К
- Вiтамiн С
- Вiтамiн Е
- Вiтамiн D
- Вiтамiн В12


# У хлопчика 3-х рокiв з вираженим геморагiчним синдромом вiдсутнiй антигемофiльний глобулін А (факторVIII) у плазмi кровi. Яка фаза гемостазу первинно порушена у цього хворого?
+ Внутрiшнiй механiзм активацiї протромбiнази
- Зовнiшнiй механізм активації протромбiнази
- Перетворення протромбiнув тромбiн
- Перетворенняфiбриногенув фiбрин
- Ретракцiякров’яногозгустку


#При обтурацiйнiй жовтяницi i жовчних норицях часто спостерiгається протромбiнова недостатнiсть. З дефiцитом в органiзмi якого вiтамiну це пов’язано?
+ K
- B6
- A
- C
- E


#У результаті побутової травми у пацієнта виникла значна крововтрата, що супроводжувалося зниженням артеріального тиску. Дія яких гормонів забезпечує швидке відновлення кров’яного тиску, викликаного крововтратою?
+Адреналін, вазопресин
-Кортизол
-Окситоцин
-Альдостерон
-Статеві


#Видалення зуба у пацієнта з хроні­чним персистуючим гепатитом ускладни­лось тривалою кровотечею. Яка причина геморагічного синдрому?
+Зменшення утворення тромбіну
-Зменшення утворення фібрину
-Посилення фібринолізу
-Збільшення утворення тромбопластину
-Збільшення синтезу фібриногену


#Пацієнт страждає на геморагічний синдром, що проявляється частими носовими кровотечами, посттравматичними та спонтанними внутрішньошкірними та внутрішньосуглобовими крововиливами. Після лабораторного обстеження було діагностовано гемофілію В. Дефіцит якого фактора згортання крові обумовлює дане захворювання?
+ IX
- VII
- VIII
- XI
- V


#У хворого після оперативного видалення кісти підшлункової залози виник геморагічний синдром з вираженим порушенням зсідання крові. Розвиток цього ускладнення пояснюється:
+Активацією фібринолітичної системи
- Активацією протизгортальної системи
- Зменшенням кількості тромбоцитів
- Недостатнім утворенням фібрину
- Активацією фактору Крисмаса


#У хворої 43-х рокiв на фонi септичного шоку вiдзначається тромбоцитопенiя, зниження фiбриногену, поява в кровi продуктiв дегенерацiї фiбрину, поява петехiальних крововиливiв. Вкажiть причину виникнення даних змiн:
+ДВЗ-синдром
-Автоiмунна тромбоцитопенiя
-Геморагiчний дiатез
-Порушення утворення тромбоцитiв
-Екзогенна iнтоксикацiя


#У хворого опiкова хвороба ускладнилася ДВЗ-синдромом. Яку стадiю ДВЗ-синдрому можна запiдозрити, якщо вiдомо, що кров хворого згортається менше нiж за 3 хвилини?
+Гiперкоагуляцiї
-Перехiдна
-Гiпокоагуляцiї
-Фiбринолiз
-Термiнальна


#При обстеженні хворого з гемофілією виявлено зміна деяких показників крові. Який з перерахованих ознак відповідає цьому захворюванню?
+Час згортання крові загальмовано
-Тромбоцитопенія
-Еритроцитоз
-Еозинофілія
-Афібриногенемія


#У хворого виявлені множинні синяки на тілі, тривалість кровотечі за Дуке 25 хвилин, число тромбоцитів крові 25*109/л. Для якого захворювання характерні такі ознаки?
+Спадковий дефект утворення тромбоцитів.
-Гемофілія А.
-Гемофілія В.
-Хвороба Віллебранда.
-Авітаміноз С.


#Хворий 12-ти років поступив в клініку з гемартрозом колінного суглоба, з раннього дитинства страждає кровоточивістю. Яка хвороба у хлопчика?
+Гемофілія
-B12 фолієво-дефіцитна анемія
-Гемолітична анемія
-Геморагічний васкуліт
-Тромбоцитопенічна пурпура


#У спортсмена легкоатлета (бiгуна на довгi дистанцiї) пiд час змагань розвинулась гостра серцева недостатнiсть. В результатi чого виникла ця патологiя?
+ Перевантаження серця об’ємом
- Порушення вiнцевого кровообiгу
- Прямого пошкодження мiокарда
- Патологiя перикарда
- Перевантаження серця опором


#При дослiдженнi кровi хворого виявлено значне збiльшення активностi МВ-форм КФК (креатинфосфокiнази) та ЛДГ-1. Яку патологію можна припустити?
+ Iнфаркт мiокарда
- Гепатит
- Ревматизм
- Панкреатит
- Холецистит


#У чоловіка 35-ти років під час тривалого бігу виникла гостра серцева недостатність. Які зміни іонного складу спостерігаються у серцевому м'язі при цьому стані?
+ Накопичення в клітинах міокарда іонів Na
+ і Ca2+
- Збільшення в позаклітинному просторі іонів Na
+ і Ca2+
- Накопичення в клітинах міокарда іонів K
+ і Mg2+
- Зменшення в клітинах міокарда іонів Na
+ і Ca2+
- Зменшення в позаклітинному просторі іонів K
+ і Mg2


# Жінка 25-ти років скаржиться на по­стійний біль у ділянці серця, задишку під час рухів, загальну слабкість. Об’єктив­но: шкіра бліда та холодна, акроціаноз. Рв- 96/хв., АТ-105/70 мм рт.ст. Межа серця змі­щена на 2 см вліво. Перший тон над вер­хівкою серця послаблений, систолічний шум над верхівкою. Діагностовано недо­статність мітрального клапана серця. Чим обумовлене порушення кровообігу?
+Перевантаження міокарда збільшеним об’ємом крові
-Перевантаження міокарда підвищеним опором відтоку крові
-Збільшення об’єму судинного русла
-Зниження об’єму циркулюючої крові
-Пошкодження міокарда


#У хворого 44-х років на ЕКГ виявлені ознаки гіпертрофії обох шлуночків та правого передсердя. Діагностовано недостатність тристулкового клапана. Який патогенетичний варіант порушення функції серця має місце при цій недостатності?
+ Перевантаження серця об'ємом
- Коронарна недостатність
- Первинна міокардіальна недостатність
- Тампонада серця
- Перевантаження серця опором


#До вiддiлення реанiмацiї надiйшов чоловiк 47-ми рокiв з дiагнозом iнфаркт мiокарду. Яка з фракцiй лактат-дегiдрогенази (ЛДГ) буде переважати в сироватцi кровi протягом перших двох дiб?
+1
-2
-3
-4
-5


#Хворий 50-ти рокiв страждає на гiпертонiчну хворобу. Пiд час фiзичного навантаження у нього з’явилися вiдчуття м’язової слабкостi, нестачi повiтря, синюшнiсть губ, шкiри, обличчя; дихання супроводжувалося вiдчутними на вiдстанi вологими хрипами. Який механiзм лежить в основi виникнення такого синдрому?
+Гостра правошлуночкова недостатнiсть
-Хронiчна правошлуночкова недостатнiсть
-Хронiчна лiвошлуночкова недостатнiсть
-Колапс
-Тампонада серця


#З метою відтворення серцевої недостатності серце жаби перфузували розчином бромістого кадмію - блокатору сульфгідрильних груп. Який варіант серцевої недостатності при цьому виникає?
+Від токсичного пошкодження міокарду
-Змішана форма
-Від перевантаження об'ємом
-Спричинене порушенням вінцевого кровообігу
-Від перевантаження опором


#Пацієнт 64 років з гострою серцевою недостатністю, артеріальним тиском 80/60мм рт ст, добовим діурезом 530 мл, істотно збільшена концентрація сечовини і креатиніну в крові. Назвіть патогенетичний механізм розвитку азотемії і олігурії:
+ Зменшення фільтраційного тиску.
-Спазм приносять артеріол клубочка
-Збільшення вироблення вазопресину
-Зменшення об'єму циркулюючої крові
-Гіпернатріємія


#В експерименті подразнюють гілочки симпатичного нерва, які іннервують серце. Це призвело до збільшення сили серцевих скорочень, тому що через мембрану типових кардіоміоцитів збільшився:
+Вхід іонів кальцію
-Вихід іонів калію
-Вихід іонів кальцію
-Вхід іонів калію
-Вхід іонів кальцію та калію


#У хворого спостерігається атонія м’язів. Назвіть фермент м’язової тканини, активність якого може бути знижена при такому стані:
+Креатинфосфокіназа
-Амілаза
-Глутамінтрансфераза
-Каталаза
-Транскетолаза


#Дитина 10-ти років перенесла кілька атак ревматізма. При його клінічному


обстеженні було встановлено, що мали місце запальні явища в суглобах


і виявилися ознаки недостатності мітрального клапана. Яке з патологічних явищ у даного хворого можна віднести до поняття "хвороба"?
+Ревматизм
-Артрит
-Порок мітрального клапана
-Запалення суглобів
-Недостатність мітрального клапана


#У хворого з ознаками недостатності мітрального клапана в намнезе відзначалися атаки ревматизму, що супроводжувалися запальними явищами в суглобах. Яке з патологічних явищ у даного больногo відноситься до категорії "патологічного стану "?
+ Недостатність мітрального клапана
-Ревматизм
-Артрит
-Запалення суглобів
-Ревмокардит


#У хворого з недостатністю мітрального клапану виникла гіпертрофія лівого шлуночка серця. Який механізм є пусковим у розвитку гіпертрофії?
+Активація генетичного апарату.
-Збільшення споживання жирних кислот.
-Збільшення інтенсивності клітинного дихання.
-Активація гліколізу.
-Збільшення надходження Ca2
+ в клітину.


|#Через 1 годину після накладання кільця, що звужує аорту, в собаки різко зросла сила та частота серцевих скорочень, а об’єм циркулюючої крові та товщина стінки лівого шлуночка не відрізнялися від вихідних показників. Яка стадія гіпертрофії міокарда спостерігається у тварини?
+Аварійна
-Декомпенсації
-Прогресуючого кардіосклерозу
- Відносно стійкої гіперфункції
-Завершеної гіпертрофії


#У п’ятимісячної дівчинки виявлено застійні явища у легенях. При обстеженні виявлено зв’язок між висхідною аортою та легеневою артерією, що в нормі спостерігається у деяких земноводних і плазунів. Назвіть цю природжену ваду розвитку:
+Незрощення боталової протоки
-Дефект міжпередсердної перегородки
-Дефект міжшлуночкової перегородки
-Розвиток правої дуги аорти
-Транспозиція магістральних судин


#В експериментi на ссавцi зруйнували певну структуру серця, що призвело до припинення проведення збудження вiд передсердь до шлуночкiв. Що саме зруйнували?
+ Атрiовентрикулярний вузол
- Синоатрiальний вузол
- Пучок Гiса
- Нiжки пучка Гiса
- Волокна Пуркiн’є4


#В експериментi подразнюють гiлочки симпатичного нерва, якi iннервують серце. Це призвело до збiльшення сили серцевих скорочень, тому що через мембрану типових кардiомiоцитiв збiльшився:
+ Вхiд iонiв кальцiю
- Вихiд iонiв кальцiю
- Вихiд iонiв калiю
- Вхiд iонiв калiю
- Вхiд iонiв кальцiю та калію


# При аналiзi ЕКГ виявлено випадiння деяких серцевих циклiв PQRST. Наявнi зубцi та комплекси не змiненi. Назвiть вид аритмiї:
+ Синоатрiальна блокада
- Миготлива аритмiя
- Атрiовентрикулярна блокада
- Передсердна екстрасистола
- Внутрiшньопередсердна блокада


#Пiд час об’єктивного обстеження хворого з дiагнозом: атеросклеротичний мiокардiосклероз, лiкар встановив феномен дефiциту пульсу. При якiй формi порушення серцевого ритму спостерiгається такий феномен?
+ Миготлива аритмiя
- Iдiовентрикулярний ритм
- Передсердно-шлуночковий ритм
- Брадикардiя
- Синусова екстрасистолiя


#У хворої 45-ти років при електрокардіографічному обстеженні виявлено такі зміни: інтервал P-Q подовжений, при цьому випадає кожен другий або третій комплекс QRST. Яке саме порушення провідності серця спостерігається?
+Атріовентрикулярна блокада ІІІ ступеня
- Внутрішлуночкова блокада
- Атріовентрикулярна блокада повна
- Синоаурікулярна блокада
- Атріовентрикулярна блокада І ступеня


#У хворого 45-ти років при аналізі ЕКГ встановлено: ритм синусовий, число передсердних комплексів більше числа шлуночкових комплексів; прогресуюче подовження інтервалу P-Q від комплексу до комплексу; випадіння окремих шлуночкових комплексів; зубці P та комплекси QRST без змін. Назвіть тип порушення серцевого ритму:
+ Атріовентрикулярна блокада II ступеня
- Внутрішньопередсердна блокада
- Синоаурікулярна блокада
- Атріовентрикулярна блокада I ступеня
- Повна атріовентрикулярна блокада


#Під час об'єктивного обстеження хворого з діагнозом: атеросклеротичний міокардіосклероз, лікар встановив феномен дефіциту пульсу. При якій формі порушення серцевого ритму спостерігається такий феномен?
+Миготлива аритмія
- Брадикардія
- Передсердно-шлуночковий ритм
- Ідіовентрикулярний ритм
- Синусова екстрасистолія


#На ЕКГ пацієнта мають місце такі зміни: зубець P - нормальний, інтервал P-Q - вкорочений, шлуночковий комплекс QRST - розширений, зубець R - двогорбий або двофазний. Яка із форм аритмії має місце у даного пацієнта?
+Синдром WPW (Вольфа-Паркінсона-Уайта)
- Миготлива аритмія
- Миготіння шлуночків
- Синдром Фредеріка (тріпотіння передсердь)
- Атріовентрикулярна блокада


#Хворий 21-го року надійшов до стаціо­нару з загостренням хронічного тонзилі­ту. Скаржиться на слабкість, задуху при помірному фізичному навантаженні. Тем­пература 37,5°С. ЧСС- 110/хв. ЕКГ: ритм синусовий, інтервал РQ подовжений. Яка аритмія у хворого?
+Передсердно-шлуночкова блокада І ст.
-Передсердно-шлуночкова блокада II ст.
-Порушення внутрішньошлуночкової провідності
- Передсердно-шлуночкова екстрасисто­лія
- Внутрішньопередсердна блокада


#Жінка 49-ти років звернулася до лікаря зі скаргами на підвищену втомлюваність та появу задишки під час фізичного навантаження. На ЕКГ: ЧСС- 50/хв, інтервал PQ- подовжений, комплекс QRS- не змінений, кількість зубців P перевищує кількість комплексів QRS. Який вид аритмії у пацієнтки?
+ Атріовентрикулярна блокада
- Миготлива аритмія
- Синусова брадикардія
- Синоатріальна блокада
- Екстрасистолія


#У чоловiка 50-ти рокiв раптово виникли сильне серцебиття, бiль у серцi, рiзка слабкiсть, пiдвищення артерiального тиску; пульс аритмiчний, з дефiцитом. На ЕКГ виявлено вiдсутнiсть зубцiв Р та рiзна тривалiсть iнтервалiв R-R. Яке порушення серцевого ритму в хворого?
+ Миготлива аритмiя
-Дихальна аритмiя
-Пароксизмальна тахiкардiя
-Поперечна блокада серця
-Синусова екстрасистолiя


#Хворий 65-ти рокiв, що страждає на атеросклероз, госпiталiзований до хiрургiчного вiддiлення з приводу розлитого гнiйного перитонiту. Пiд час операцiї дiагностовано тромбоз брижових артерiй. Яка найбiльш iмовiрна причина перитонiту?
+ Геморагiчний iнфаркт
- Iшемiя ангiоспастична
- Iшемiчний iнфаркт
-Стаз
-Iшемiя компресiйна


#Підліток 15 років, скаржиться на недостачу повітря, загальну слабкість, серцебиття. ЧСС 130 уд/хв, АТ-100/60 мм рт. ст.на ЕКГ, комплекс QRS нормальної форми та тривалості. Число зубців Р та шлункових комплексів однакове, зубець Т злитий з зубцем Р.Яка аритмія серця спостерігається у підлітка ?
+синусова тахікардія
-синусова екстрасистолія
-мерехтіння передсердь
-тремтіння передсердь
-передсердна пароксизмальна тахікардія


#Людина отримала електротравму. При цьому струм перейшов через серцевий м'яз. Які небезпечні порушення в роботі серця можуть виникнути у цііі ситуаціі, що вимагають термінового втручання?
+Фібриляція шлуночків
-Фібриляція передсердь
-Екстрасистолія
-Тахікардія
-Брадікардія


#При аналізі ЕКГ встановлено: ритм синусовий, число передсердних комплексів більше числа шлуночкових комплексів; прогресуюче подовження інтервалу P-Q від комплексу до комплексу; випадання окремих шлуночкових комплексів, тому після зубця Р йде довга пауза; зубці Р та комплекси QRST без змін. Назвіть тип порушення серцевого ритму.
+Неповна атріо-вентрикулярна блокада П ступеня
-Сино-атріальна блокада.
-Неповна атріо-вентрикулярна блокада Iступеня
-Неповна атріо-вентрикулярна блокада IIIступеня
-Повна атріо-вентрикулярна блокада.


#У хворого на ЕКГ виявлено зсув сегмента S-T вище ізоелектричної лінії


на 1 мм і збільшення тривалості зубця T до 0,25 с. З порушенням якого


процесу пов'язані зазначені зміни на ЕКГ?
+реполяризації шлуночків
-деполяризації шлуночків
-атріовентрикулярного проведення
-реполяризації передсердь
-деполяризації передсердь


#У чоловіка 63 років зі слів родичів тричі відзначалась втрата свідомості. ЧД 18 за 1 хв., ЧСС 45 за 1 хв., АТ 100/70 мм рт. ст. На ЕКГ: частота Р 80 за хвилину, частота R 42 за хвилину, ритм правильний. Яка найбільш вірогідна аритмія?
+ Повна AV блокада
-AV блокада II ступеню
-Синусова брадікардія.
-Синоаурикулярна блокада.


#Через 3 тижні після гострого інфаркту міокарда у хворого з'явилися болі в серці та суглобах, запалення легень. Який механізм є основним у розвитку постінфарктного синдрому Дресслера у цього хворого?
+Аутоімунне запалення
-Вторинна інфекція
-Iшемія міокарда
-Тромбоз судин
-Резорбція білків з некротизованої ділянки міокарда


#При обстеженні у юнака 16 років було виявлено прискорення серцебиття під час вдиху, сповільнене – під час видиху. На ЕКГ відмічалося: вкорочення інтервалу RR під час вдоху та подовження його під час видоху. Назвіть вид аритмії
+ Синусова аритмія
-Миготлива аритмія
-Синусова тахікардія
-Ідіовентрикулярний ритм
-Синусова брадикардія


#Електрик, працюючи з порушенням правил техніки безпеки, випадково торкнувся оголеного електропровода обома руками і загинув. Смерть настала внаслідок
+ Фібриляції передсердь і шлуночків
-Повної атріо-вентрикулярної блокади
-Пригнічення автоматизму сино-атріального вузла
-Зменшення скоротливої здатності міокарда
-Порушення вагусної регуляції серця


#Під час об’єктивного обстеження хворого з діагнозом: атеросклеротичний міокардіосклероз, лікар встановив феномен дефіциту пульсу. При якій формі порушення серцевого ритму спостерігається такий феномен?
+Миготлива аритмія
-Ідіовентрикулярний ритм
-Брадикардія
-Передсердно-шлуночковий ритм
-Синусова екстрасистолія


#При аналізі ЕКГ виявлено випадіння деяких серцевих циклів PQRST. Наявні зубці та комплекси не змінені. Назвіть вид аритмії:
+Синоатріальна блокада
-Атріовентрикулярна блокада
-Внутрішньопередсердна блокада
-Миготлива аритмія
-Передсердна екстрасистола


#У хворого на ЕКГ виявлено збільшення тривалості комплексу QRS. Наслідком чого це може бути?
+Збільшення часу охоплення збудженням шлуночків
-Збільшення збудливості передсердь
-Збільшення збудливості шлуночків та передсердь
-Збільшення часу охоплення збудженням передсердь
-Порушення провідності у атріовентрикулярному вузлі


#Аналіз ЕКГ хворого виявив відсутність зубця P. Тривалість та амплітуда QRS комплексу та зубця Т відповідають нормі. Що є водієм ритму серця даного пацієнта?
+Передсердно-шлуночковий вузол
-Волокна Пуркіньє
-Міокард шлуночків
-Пучок Гіса
-Синусовий вузол


#У хворого напад тахікардії. Які мембранні циторецептори кардіоміоцитів доцільно заблокувати, щоб припинити напад?
+Бета-адренорецептори
-Альфа-адренорецептори
-М- та Н-холінорецептори
-М-холінорецептори
-Н-холінорецептори


#У хворого на ЕКГ виявлено, що тривалість інтервалу RR дорівнює 1,5 с, частота серцевих скорочень - 40/хв. Що є водієм ритму серця?
+Атріовентрикулярний вузол
-Ліва ножка Гіса
-Права ножка Гіса
-Пучок Гіса
-Синусовий вузол


#Під час розтину померлого 43-х років, що страждав на іХС з розвитком інфаркту міокарда, патологоанатом виявив набряк легень. Які патологічні зміни могли зумовити набряк легень?
+Гостра лівошлуночкова недостатність
-Ішемія малого кола
-Гостра правошлуночкова недостатність
-Гостре загальне малокрів’я
-Стаз крові


#Xворого 45-ти років при аналізі ЕКГ встановлено: ритм синусовий, число передсердних комплексів більше числа шлуночкових комплексів; прогресуюче подовження інтервалу P-Q від комплексу до комплексу; випадіння окремих шлуно-чкових комплексів; зубці P та комплекси QRST без змін. Назвіть тип порушення серцевого ритму:
+Атріовентрикулярна блокада II ступеня
-Атріовентрикулярна блокада I ступеня
-Внутрішньопередсердна блокада
-Повна атріовентрикулярна блокада
-Синоаурікулярна блокада


#У студента перед екзаменом виникла тахікардія. Які зміни на ЕКГ будуть свідчити про її наявність?
+Укорочення інтервалу R-R
-Подовження інтервалу P-Q
-Подовження інтервалу R-R
-Подовження сегменту Q-T
-Розширення комплексу QRS


#У хворого інфаркт міокарда. Активність якого ферменту буде значно підвищена в сироватці крові хворого в перші години?
+Креатинфосфокіназа МВ
-ACT
-АЛТ
-ЛДГ4
-ЛДГ5


#На ізольованому серці шляхом охолодження припиняють функціонування окремих структур. Яку структуру охолодили, якщо серце внаслідок цього спочатку припинило скорочення, а далі відновило їх із частотою у 2 рази меншою за вихідну?
+Синоатріальний вузол
-Атріовентрикулярний вузол
-Волокна Пуркіньє
-Ніжки пучка Гіса
-Пучок Гіса


#Підвищення внутрішньочерепного тиску у хворого з церебральною гематомою обумовило надмірну активність блукаючого нерва (ваготонію) та зміну частоти серцевих скорочень. Який вид аритмії серця виникає при цьому?
+Синусова брадикардія
-Пароксизмальна тахікардія
-Передсердно-шлуночкова блокада
-Синусова тахікардія
-Шлуночкова екстрасистолія


#У хворого з серцевою недостатністю виникла аритмія у вигляді генерації позачергових імпульсів в пучку Гіса. Порушення якої функції серцевого м’язу спостерігається в даному випадку?
+Збудливість
-Автоматизм
-Збудливість та провідність
-Провідність
-Скоротливість


#Під час серцевого нападу чоловік в автобусі втратив свідомість, з’явилися судоми. Лікар швидкої допомоги виявив на ЕКГ, що частота скорочення передсердь перевищує частоту скорочення шлуночків. Що може бути причиною даного стану?
+Повна поперечна блокада проведення збудження
-Виникнення гетеротропних вогнищ збудження
-Порушення автоматії АВ-вузла
-Порушення автоматії СА-вузла
-Порушення проведення збудження між передсердями


#Хворий на трансмуральний iнфаркт мiокарда лiвого шлуночка переведений до вiддiлення реанiмацiї у важкому станi. АТ- 70/50 мм рт.ст., ЧСС- 56/хв., ЧД- 32/хв. Зазначте головну ланку в патогенезi кардiогенного шоку:
+ Падiння серцевого викиду
- Падiння периферичного судинного опору
- Втрата води
- Крововтрата
- Втрата електролітів


#У хворого, який скаржився на біль у ділянці лівої лопатки, був діагностований інфаркт міокарду. Назвіть вид болю у хворого?
+Іррадіюючий (відбитий)
- Вісцеральний
- Перший (протопатичний)
- Другий (епікритичний)
- Фантомний


#У пацієнта, який півтора місяця тому переніс інфаркт міокарда, діагностовано синдром Дреслера з характерною тріадою: перикардит, плеврит, пневмонія. Який головний механізм цього ускладнення?
+Сенсибілізація організму антигенами міокарда
- Інтоксикація організму продуктами некрозу
- Зниження резистентності до інфекційних агентів
- Викидання у кров міокардіальних ферментів
- Активація сапрофітної мікрофлори


#У хворого на ішемічну хворобу серця відзначається гіпертрофія міокарда, тахікардія, зниження ХОК. Який з механізмів є провідним в ушкодженні кардіоміоцитів у даному випадку?
+Пошкодження специфічних мембранних насосів
- Втрата Mg2
+ кардіоміоцитами
- Збільшення числа a та b-адренорецепторів
- Втрата Ca2
+ кардіоміоцитами
- Дегідратація кардіоміоцитів


#У чоловіка 45-ти років після значного психоемоційного навантаження раптово з'явився стискаючий біль в ділянці серця з іррадіацією в ліву руку, шию, під ліву лопатку. Обличчя стало блідим, вкрилося холодним потом. Нітрогліцерин усунув напад болю. Який процес розвинувся у хворого?
+Стенокардія
- Перфорація виразки шлунка
- Інфаркт міокарда
- Психогенний шок
- Інсульт


#Хворий 49-ти років, водій за професією, скаржиться на нестерпний стискаючий біль за грудниною, що "віддає" у ділянку шиї. Біль виник 2 години тому. Об'єктивно: стан важкий, блідість, тони серця послаблені. Лабораторне обстеження показало високу активність креатинкінази та ЛДГ1. Для якого захворювання характерні такі симптоми?
+Гострий інфаркт міокарда
- Стенокардія
- Гострий панкреатит
- Цукровий діабет
- Жовчнокам'яна хвороба


#Одним із самих небезпечних моментів в патогенезі некрозу міокарда є подальше наростання зон некрозу, дистрофії та ішемії. Важлива роль в цьому належить підвищенню споживання міокардом кисню. Які речовини сприяють даному процесу?
+Катехоламіни
-Ацетилхолін
-Аденозин
-Холестерин
-Iони хлору


#Хворий 59 років госпіталізований у кардіологічне відділення в тяжкому стані з діагнозом: гострий інфаркт міокарда в області задньої стінки лівого шлуночка та перегородки, початковий набряк легень. Який первинний механізм, що викликає розвиток набряку легень у пацієнта?
+Лівошлуночкова недостатність
-Легенева венозна гіпертензія
-Легенева артеріальна гіпертензія
-Гіпоксемія
-Зниження альвеоло-капілярної дифузії кисню


#Хворий А, 59 років, директор приватного підприємства. Після перевірки податковоїінспекцією ввечері з'явилися інтенсивні пекучі болі, локалізовані за грудиною, иррадиирующие в ліву руку. Через 15 хв стан хворого нормалізувався. Який з механізмів розвитку ішемії серцевого м'яза є провідним у даного хворого?
+ Підвищення в крові рівня катехоламінів
-Атеросклероз коронарних артерій
-Внутрішньосудинна агрегація формених елементів
-Здавлення коронарних артерій при дилятации порожнин серця
-Функціональна перевантаження серця


#У жiнки обмежений кровотiк у нирках, пiдвищений артерiальний тиск. Гiперсекрецiя якого гормону зумовила пiдвищений тиск?
+ Ренiн
- Адреналiн
- Норадреналiн
- Еритропоетин
- Вазопресин


# У хворого з тромбофлебiтом нижнiх кiнцiвок раптово пiсля навантаження виникли задишка, рiзкий бiль у грудях, цiаноз, набухання шийних вен. Яке найбiльш iмовiрне порушення кровообiгу виникло у хворого?
+Тромбоемболiя легеневої артерiї
- Тромбоемболiя вінцевих судин
- Тромбоемболiя судин головного мозку
- Тромбоемболiя мезентерiальних судин
- Тромбоемболiя ворiтної вени


#До серцево-судинного вiддiлення надiйшов хворий зi скаргами на постiйний головний біль у потиличнiй дiлянцi, шум у вухах, запаморочення. При обстеженнi: АТ 180/110мм рт.ст., ЧСС-95/хв. Рентгенологiчно визначено звуження однiєї з ниркових артерiй. Активацiя якої з перерахованих систем викликала гiпертензивний стан хворого?
+ Ренiн-ангiотензинова
- Гемостатична
- Симпатоадреналова
- Кiнiнова
- Iмунна


#У чоловiка 65-ти рокiв впродовж 15-ти рокiв була виражена артерiальна гiпертензiя. Останнiм часом систолiчний тиск почав знижуватися, а дiастолiчний залишився пiдвищеним. Який гемодинамiчний тип артерiальної гiпертензiї у хворого?
+ Гiпокiнетичний
- Нормокiнетичний
- Гiперкiнетичний
- Еукiнетичний
- –


#У чоловiка 72-х рокiв довготривала хронiчна патологiя легень призвела до недостатностi клапанiв легеневої артерiї i трикуспiдального клапану, недостатностi кровообiгу за правошлуночковим типом. Який тип артерiальної гiпертензiї є причиноюпе- ревантаження серця об’ємом?
+ Легенева гiпертензiя
- Центрально-iшемiчна гiпертензiя
- Есенцiальна гiпертензiя
- Рефлексогенна гiпертензiя
- Сольова гіпертензія


#Обстеження пацiєнта з високим артерiальним тиском показало в нього вторинну артерiальну гiпертензiю. Причиною такого стану є ренiнпродукуюча пухлина нирки. Що є головною ланкою в патогенезi вторинної артерiальної гiпертензiї в хворого?
+ Гiперпродукцiя ангiотензину 2, альдостерону
- Гiперпродукцiя кортизолу
- Гiперпродукцiя iнсулiну
- Недостатня продукцiя вазопресину
- Недостатня продукцiя катехоламiнiв


#У хворого 48-ми років на хронічний гло­мерулонефрит наявні набряки, АТ- 210/100 мм рт.ст., ЧСС- 85/хв., межі серця розши­рені. Який механізм розвитку артеріальної гіпертензії є головним?
+Активація ренин-ангіотензин- альдостеронової системи
-Підвищення продукції вазопресину
-Підвищення ОЦК
-Гіперфункція серця
-Підвищення активності симпатичного відділу нервової системи


#Хворий 39-ти років з алкогольним цирозом печінки скаржиться на задишку, загальну слабкість. Встановлено зниження артеріального тиску, розширення поверхневих вен передньої стінки живота, спленомегалію. Яке порушення гемодинаміки спостерігається у хворого?
+Синдром портальної гіпертензії
- Недостатність правого шлуночка серця
- Недостатність лівого шлуночка серця
- Колапс
- Тотальна серцева недостатність


#У хворого 70-ти років атеросклероз ускладнився тромбозом судин нижніх кінцівок, виникла гангрена пальців лівої стопи. Початок тромбоутворення, найбільш імовірно, пов'язаний з:
+Адгезією тромбоцитів
- Перетворенням фібриногену в фібрин
- Зниженням синтезу гепарину
- Перетворенням протромбіну в тромбін
- Активацією протромбінази


#Чоловік 58-ми років хворіє на атеросклероз судин головного мозку. При обстеженні виявлена гіперліпідемія. Вміст якого класу ліпопротеїдів у сироватці крові даного чоловіка найбільш вірогідно буде підвищений?
+Ліпопротеїди низької щільності
- Холестерин
- Хіломікрони
- Комплекси жирних кислот з альбумінами
- Ліпопротеїди високої щільності


#На перехiд iз горизонтального положення у вертикальне система кровообiгу вiдповiдає розвитком рефлекторної пресорної реакцiї. Що з наведеного є її обов’язковим компонентом?
+Системне звуження венозних судин
-Системне розширення артерiальних судин опору
-Зменшення об’єму циркулюючої кровi
-Зменшення частоти серцевих скорочень
-Зменшення насосної функцiї серця


#Визначте пульсовий i середньо-динамiчний артерiальний тиск (мм рт.ст.) у обстежуваного, якщо вимiряний у нього артерiальний тиск становить 130/70 мм рт.ст.:
+60, 90
-60, 80
-50, 90
-60, 100
-50, 70


#У дорослої людини системний артерiальний тиск знизився з 120/70 до 90/50 мм рт.ст., що викликало рефлекторне звуження судин. У якому з за-значених органiв звуження судин буде найменшим?
+Серце
-Шкiра
-Кишечник
-Скелетнi м’язи
-Печiнка


#Пiд час роботи лiкарю–стоматологу доводиться довго стояти на ногах, що може призвести до застою кровi у венах нижнiх кiнцiвок та їх варикозного розширення. З порушенням якого механiзму венозного припливу кровi до серця це пов’язано?
+ Вiдсутнiсть скорочення скелетних м’язiв
- Градiєнт тиску
- Присмоктувальний ефект грудної клiтки
- Залишкова рушiйна сила серця


#У хворого після тривалого психоемоційного напруження спостерігається підвищення артеріального тиску, що супроводжується серцебиттям, кар-діалгіями, головним болем, запамороченням. Домінуючим у формуванні артеріальної гіпертензії у даному випадку є збільшення:
+Тонусу артеріол
-Об’єму циркулюючої крові
-Серцевого викиду
-Тонусу венул
-Частоти серцевих скорочень


#Щуру в плевральну порожнину введено 0,5 мл повiтря. Який тип недостатностi дихання виникає в даному випадку?
+ Рестриктивне порушення альвеолярної вентиляцiї
- Обструктивне порушення альвеолярної вентиляцiї
- Перфузiйний
- Дифузiйний
- Дисрегуляторне порушення альвеолярної вентиляції


# У неврологiчне вiддiлення з приводу мозкового крововиливу поступив хворий 62-х рокiв. Об’єктивно: стан важкий. Спостерiгається наростання глибини i частоти дихання, а потiм його зменшення до апное, пiсля чого цикл дихальних рухiв вiдновлюється. Який тип дихання у хворого?
+ Чейна-Стокса
- Кусмауля
- Бiота
- Гаспiнг-дихання
- Апнейстичне


# Чоловiк 50-ти рокiв хворiє на хронiчний бронхiт, скаржиться на задишку пiд час фiзичного навантаження, постiйний кашель з вiдходженням харкотиння. При обстеженнi дiагностовано ускладнення - емфiзема легень. Чим вона зумовлена?
+ Зниження еластичних властивостей легень
- Зменшення альвеолярної вентиляцiї
- Зменшення розтяжностi легень
- Зменшення перфузiї легень
- Порушення вентиляцiйно-перфузiйного спiввiдношення в легенях


#У хворого внаслiдок хронiчного обструктивного бронхiту на тлi задишки, тахiкардiї та цiанозу пiд час дослiдження газового складу кровi виявлено розвиток гiпоксемiї та гiперкапнiї. Яке порушення зовнiшнього дихання спостерiгається у хворого?
+Гiповентиляцiя
- Гiпоперфузiя
-Гiперперфузiя
- Гiпердифузiя
- Гiпервентиляцiя


#Хворий 47-ми рокiв впродовж останнiх 3-х рокiв хворiє на туберкульоз легень, скаржиться на задишку, важкiсть в областi правого боку грудної стiнки, температуру тiла 37,7oC. Виявлено правобiчний ексудативний плеврит. Який тип клiтин передбачається у плевральному пунктатi?
+ Лiмфоцити
- Нейтрофiли
- Еритроцити
- Атиповi клiтини
- Еозинофiли


#Чоловік 27-ми років після ДТП надійшов до лікарні у важкому стані із черепно-мозковою травмою. Дихання характеризується судомним тривалим вдихом, який пeреривається коротким видихом. Для якого типу дихання це характерно?
+Апнейстичне
-Біота
-Кусмауля
-Чейн-Стокса
-Гаспінг-дихання


# У хворого із вираженим пневмосклерозом після перенесеного інфільтративного туберкульозу легень розвинулась дихальна недостатність. До якого патогенетичного типу вона відноситься?
+ Рестриктивний
- Обструктивний
- Дисрегуляційний
- Апнеїстичний
- Рефлекторний


#В експерименті на тварині здійснили перерізку блукаючих нервів з обох боків. Як при цьому зміниться характер дихання?
+Стане глибоким і рідким
-Стане глибоким і частим
- Дихання не зміниться
- Стане поверхневим та частим
-Стане поверхневим та рідким


#У підлітка 12-ти років, який хворіє на бронхіальну астму, виник тяжкий напад астми: виражена експіраторна задишка, блідість шкірних покривів. Який вид порушення альвеолярної вентиляції має місце?
+Обструктивний
- Нервово-м'язовий
- Торако-діафрагмальний
- Центральний
- Рестриктивний


#Під час обіду дитина поперхнулася і аспірувала їжу. Почався сильний кашель, шкіра і слизові ціанотичні, пульс частий, дихання рідке, видих подовжений. Яке порушення зовнішнього дихання розвинулося у дитини? +Експіраторна задишка при асфіксії
-Дихання Біота
- Альтернуюче дихання
- Інспіраторна задишка при асфіксії
- Стенотичне дихання


#У шахтаря виявлено фiброз легень, що супроводжувався порушенням альвеолярної вентиляцiї. Який механiзм виникнення цього порушення є провiдним?
+Обмеження дихальної поверхнi легень
-Звуження верхнiх дихальних шляхiв
-Порушення нервової регуляцiї дихання
-Обмеження рухомостi грудної клiтки
-Спазм бронхiв


#Людина зробила максимально глибокий видих. Як називається об’єм повiтря, що знаходиться в її легенях пiсля цього?
+Залишковий об’єм
-Функцiональна залишкова ємнiсть легень
-Ємнiсть вдиху
-Резервний об’єм видиху
-Альвеолярний об’єм


#У людини внаслiдок тривалого перебування у горах на висотi 3000 м над рiвнем моря збiльшилась киснева ємкiсть кровi. Безпосередньою причиною цього є посилене утворення в органiзмi:
+Еритропоетинiв
-Лейкопоетинiв
-Карбгемоглобiну
-Катехоламiнiв
- 2,3-дифосфоглiцерату


#Лiкар записав в iсторiї хвороби, що у хворого дихання поверхневе (знижена глибина дихання). Це означає, що зменшеним є такий показник зовнiшнього дихання:
+Дихальний об’єм
-Життєва ємнiсть легень
-Функцiональна залишкова ємнiсть
-Ємнiсть вдиху
-Хвилинний об’єм дихання


#В результаті виробничої травми у хворого виявлено перелом кількох ребер. Який характер дихання зазвичай спостерігається в таких випадках?
+ Поліпное
-Еупное
-Гіперпное
-Брадипное
-Апное


#У дівчинки 14 років, експіраторна задишка. Стан розвився після гри з собакою. В анамнезі хворіє бронхіальною астмою. Якою із перелічених нижче біологічно активною речовиною найбільш вірогідно викликаний спазм гладеньких м'язів бронхіол дівчинки?
+лейкотрієн Д4
-тромбоксан А2
-серотонін
-брадикінін
-ацетілхолін


#В отоларингологічне відділення госпіталізована жінка зі скаргами на попадання стороннього предмету [вишневої кістки] у дихальні шляхи. Які зміни зовнішнього дихання слід очікувати?
+ Глибоке рідке
-Глибоке часте
-Часте поверхневе
- Куссмауля
-Періодичне


#У хворого на правець виникла гостра недостатність дихання. Який тип недостатності дихання виникає в даному випадку?
+ Дисрегуляторне порушення альолярної вентиляції
-Рестриктивне порушення альолярної вентиляції
-Обструктивне порушення альолярної вентиляції
-Перфузійний тип
-Дифузійний тип


#У xворого на дифтерію розвинувся набряк гортані. При цьому спостерігається рідке та глибоке дихання з затрудненням вдоху. Який тип дихання спостерігається при цьому?
+ Стенотичне
-Куссмауля
-Чейна-Стокса
-Апнейстичне
-Гаспінг


#У чоловіка 27 років діагностований: правосторонній ексудативний плеврит з ознаками недостатності дихання. Назвіть механізм розвитку цієї недостатності .
+порушення рухливості грудної клітини
-рестриктивна недостатність дихання
-обструктивна недостатність дихання
-порушення дифузії газів в легенях
-зменшення перфузії легень кров'ю


#Хворий І., 38 років, поступив зі скаргами на біль в боці, що підсилюється при вдиху і кашлі. Больові відчуття зменшуються в положенні лежачи на ураженому боці. При обстеженні дихання часте поверхневе, помітно обмеження дихальної рухливості відповідної половини грудної клітки. Який механізм зміни характеру дихання у хворого?
+Обмеження корою мозку рефлексу Герінга-Брейера
-перераздражения легеневих рецепторів блукаючих нервів
-Інтоксикація дихального центру продуктами запалення
-Підвищення збудливості дихального центру
-Гальмування кори мозку внаслідок інтоксикації


#Під час обіду дитина поперхнувся і аспірувала їжу. Почався сильний кашель, шкіра і слизові ціанотичні, пульс прискорений, дихання рідке, Видих подовжений. Яке порушення зовнішнього дихання розвинулося у дитини?
+ Стадія експіраторної задишки при асфіксії
-Стадія інспираторної задишки при асфіксії
-Дихання Біота
-Дихання Куссмауля
-Дихання Чейна-Стокса


#Чоловік 30 років скаржиться на задуху, важкість в правій половині грудної клітки, загальну слабкість. Температура тіла 38,9оС. Об’єктивно: права половина грудної клітки відстає від лівої. Плевральна пункція дала ексудат. Що являється провідним чинником ексудації у хворого?
+Підвищення проникливості стінки судин.
-Підвищення кров’яного тиску.
-Гіпопротеїнемія.
-Агрегація еритроцитів.
-Зменшення резорбції плевральної рідини .


#У людини в результаті патологічного процесу збільшена товщина гематоальвеолярного бар’єру. Безпосереднім наслідком цього буде зменшення:
+Дифузійної здатності легень
-Альвеолярної вентиляції легень
-Кисневої ємності крові
-Резервного об’єму видиху
-Хвилинного об’єму дихання


#Робота шахтарів у забої часто спричинює антракоз. Який вид дихальної недостатності може розвинутися при цьому?
+Рестриктивний
-Діафрагмальний
-Дисрегуляторний
-Обструктивний
-Торакальний


#В ході обстеження людини необхідно визначити, яка частка альвеолярного повітря оновлюється під час кожного вдиху. Який з наведених показників необхідно розрахувати для цього?
+Коефіцієнт легеневої вентиляції
-Життєва ємність легень
-Функціональна залишкова ємність легень
-Хвилинна альвеолярна вентиляція
-Хвилинний об’єм дихання


#Чоловік 40-ка років скаржиться на загальну слабкість, головний біль, кашель з виділенням мокротиння, задишку. Після клінічного огляду й обстеження поставлено діагноз: пневмонія. Який тип гіпоксії має місце у хворого?
+Респіраторна
-Іемічна
-Гіпоксична
-Тканинна
-Циркуляторна


#У людини після довільної тривалої затримки дихання збільшилися частота й глибина дихання. Які зміни в крові, насамперед, стали причиною цього?
+Підвищення рСО2
-Зниження рО2
-Зниження рСО2
-Підвищення рН
-Підвищення рО2


#Через рік після субтотальної резекції шлунка з приводу виразки малої кривизни виявлені зміни в аналізі крові - анемія, лейко- і тромбоцитопенія, КП-1,3, на­явність мегалобластів та мегалоцитів. Дефіцит якого фактору обумовив розвиток цієї анемії?
+Фактор Касла
-Хлороводнева кислота
-Муцин
-Гастрин
-Пепсин


#Жінка 45-ти років декілька років хворіє на системний червоний вовчак у легкій формі. При прогресуванні захворювання (з'явився міокардит) їй призначили преднізолон як імуносупресор. Через 2 місяці прийому у хворої виникла шлункова кровотеча. Яка найбільш імовірна її причина?
+Ульцерогенна дія
-Подальше прогресування захворювання
- Збудження ЦНС
- Зменшення згортання крові
- Підвищення артеріального тиску


#У приймально-діагностичне відділення доставили жінку 38-ми років з шлунковою кровотечею. Які зміни найбільш імовірні з боку крові через добу?
+Зменшення гематокритного числа
- Лейкопенія
- Еритроцитоз
- Лейкоцитоз
- Збільшення гематокритного числа


#У хворого 30-ти років із гострим запаленням підшлункової залози (панкреатитом) виявлено порушення порожнинного травлення білків. Це може бути пов’язано із недостатнім синтезом та виділенням залозою такого ферменту:
+Трипсин
-Пепсин
-Дипептидаза
-Ліпаза
-Амілаза


#При копрологічному дослідженні встановлено, що кал знебарвлений, у ньо­му знайдено краплі нейтрального жиру. Найбільш вірогідною причиною цього є порушення:
+Надходження жовчі до кишечнику
-Секреції підшлункового соку
-Секреції кишкового соку
-Кислотності шлункового соку
-Процесів всмоктування в кишечнику


#До гастроентерологічного відділення потрапив хворий 57 років з підозрою на синдром Золінгера-Еллісона, про що свідчило різке збільшення рівню гастрину у сироватці крові. Яке порушення секреторної функції шлунку найбільш вірогідне?
+ Гіперсекреція гіперацидна
-Гіпосекреція гіперацидна
-Ахілія
-Гіпосекреція гіпоацидна
-Гіперсекреція гіпоацидна


#Чоловік 32 років , звернувся з приводу диспептичних розладів, при обстеженні знайдено виразковий дефект слизової оболонки шлунка та діагностується синдром Золінгера-Еллісона. Що являється основним патогенетични механізмом виникнення виразки у даному випадку ?
+підвищення продукції гастрину
-підвищення продукції соляної кислоти
-зниження захисних властивостей слизової шлунка
-підвищення продукції інсуліну
-рефлюкс дуоденального вмісту у шлунок


#Хворий скаржиться на відрижку, печію, часті запори. При титруванні шлункового соку одержали такі дані: загальна кислотність - 88 т.о., загальна HCl - 83 т.о., вільна HCl - 50 т.о., зв'язана HCl - 33 т.о., кислі фосфати та органічні кислоти - 5 т.о. Оцініть стан кислотності шлункового соку.
+Гіперацидний.
-Гіпоацидний.
-Ахілія.
-Нормацидний.
-Гіперсекреція.


#У хворої зі скаргами на болі в епігастральній ділянці оперізуючого характеру при лабораторному обстеженні виявлено підвищений вміст діастази в сечі, а також вміст у калі великої кількості неперетравленого жиру. Для якої форми патології шлунково-кишкового тракту найбільш характерні описані явища?
+ Гострий панкреатит
-Виразкова хвороба шлунка
-Гострий апендицит
-Запалення товстого кишечника
-Інфекційний гепатит


#Для моделювання виразки шлунка тварині ввели в гастральні артерії атофан, який спричинює їх склерозування. Який механізм пошкодження слизової оболонки шлунку є провідним в даному експерименті?
+ Гіпоксичний
-Нейродистрофічний
-Механічний
-Дисрегуляторний
-Нейрогуморальний


#Хворому поставили діагноз - синдром подразненої кишки. Віділення якого інкрету найбільш вірогідно збільшується при цьом?
+ Мотіліну
-Інтестінального пептиду
-глюкагону
-Урогастрону
-секретину


#У хворого з синдромом Zollinger-Ellison [ пухлина підшлункової залози] відзначається збільшення секреції, перистальтики шлунково-кишкового тракту, а також діарея та пептичні виразки. Яка з перерахованих речовин, що секретується вказаною пухлиною, викликає цей комплекс симптомів?
+Гастрин
-Вазоактивний інтестинальний пептид
-Пепсин.
-Трипсин
-Секретин


#До складу харчових раціонів обов’язково входять продукти, в яких є клітковина. Відомо, що вона не перетравлюється ферментами травного тракту й не засвоюється організмом. Яку роль відіграє ця речовина?
+Стимулює моторну функцію травного каналу
-Гальмує всмоктувальну функцію травного каналу
-Гальмує моторну функцію травного каналу
-Гальмує процеси виділення ферментів травних соків
-Гальмує секреторну функцію травного каналу


#У людини порушено всмоктування продуктів гідролізу жирів. Причиною цього може бути дефіцит у порожнині тонкої кишки:
+Жовчних кислот
-Іонів натрію
-Жиророзчинних вітамінів
-Жовчних пігментів
-Ліполітичних ферментів


#У молодого чоловіка внаслідок подразнення сонячного сплетення запальним процесом (солярит) підвищена функціональна активність залоз шлунка, що виражається, зокрема, у збільшенні продукції хлоридної кислоти. Яка з вказаних нижче речовин викликає гіперхлор-гідрію у даному випадку?
+Гастрин
-Гастроінгібуючий пептид
-Глюкагон
-Калікреїн
-Урогастрон


# Пацiєнт звернувся зi скаргами на гострий бiль у правому пiдребер’ї. При оглядi лiкар звернув увагу на пожовтiння склер хворого. Лабораторно: пiдвищена активнiсть АлАТ та негативна реакцiя на стеркобiлiн в калi. Для якого захворювання характернi такi симптоми?
+ Гепатит
- Гемолiтична жовтяниця
- Хронiчний гастродуоденiт
- Хронiчний колiт
- Хронiчний гастрит


# У хворого з жовтяницею встановлено: підвищення у плазмi кровi вмiсту загального бiлiрубiну за рахунок непрямого (вiльного), в калi та сечi - високий вмiст стеркобiлiну, рiвень прямого (зв’язаного) бiлiрубiну в плазмi кровi в межах норми. Про який вид жовтяницi можна думати?
+ Гемолiтична
- Паренхiматозна (печiнкова)
- Механiчна
- Жовтяниця немовлят
- Хвороба Жильбера


#Чоловiк 53-х рокiв звернувся зi скаргами на гострий бiль у правому пiдребер’ї. При оглядi лiкар звернув увагу на пожовтiлi склери хворого. Лабораторнi аналiзи показали пiдвищену активнiсть АЛТ та негативну реакцiю на стеркобiлiн у калi. Для якого захворювання характернi такi симптоми?
+ Жовчнокам’яна хвороба
- Гемолiтична жовтяниця
- Гепатит
- Хронiчний колiт
- Хронiчний гастрит


#У хворого 20-ти років з жовтяницею встановлено: підвищення у плазмі крові вмісту загального білірубіну за рахунок не­прямого (вільного), в калі та сечі - високий вміст стеркобіліну, рівень прямого (зв’я­заного) білірубіну в плазмі крові в межах норми. Про який вид жовтяниці можна ду­мати?
+Гемолітична
-Жовтяниця немовлят
-Хвороба Жильбера
-Паренхіматозна (печінкова)
-Механічна


#У хворого на підгострий септичний ендокардит при огляді лікар відзначив загальну слабкість і іктеричність шкіри, склер і видимих слизових оболонок. У крові виявлена збільшена кількість непрямого білірубіну. Що зумовлює жовтяничність шкіри і слизових?
+ Надпечінкова жовтяниця
- Печінкова жовтяниця
- Підпечінкова жовтяниця
- Гемосидероз
- Жирова дистрофія


#Хвора 28-ми років потрапила до інфекційної лікарні з приводу пожовтіння шкіри, склер, слизових оболонок. Лабораторно встановлене підвищення рівня прямого білірубіну у крові. В сечі виявлений уробіліноген і білірубін. Для якого з перелічених захворювань характерні такі зміни?
+Паренхіматозна жовтяниця
- Гемолітична жовтяниця
- Інфаркт нирки
- Туберкульоз нирки
- Механічна жовтяниця


#У хворого на жовтяницю у крові під­вищений вміст прямого білірубіну та жов­чних кислот; у сечі відсутній стеркобіліноген. При якій жовтяниці можлива наяв­ність цих ознак?
+Механічна
-Гемолітична
-Надпечінкова
-Печінкова
-Паренхіматозна


#Хвора 48-ми років надійшла до клініки із скаргами на слабкість, дратівливість, порушення сну. Об'єктивно: шкіра та склери жовтого кольору. У крові: підвищення рівня загального білірубіну з переважанням прямого. Кал - ахолічний. Сеча - темного кольору (жовчні пігменти). Яка жовтяниця має місце в хворої?
+Механічна
- Гемолітична
- Синдром Кріглера-Найяра
- Синдром Жільбера
- Паренхіматозна


#Чоловiк 28-ми рокiв надiйшов зi скаргами на нудоту, блювання, бiль у правому пiдребер’ї. Об’єктивно: жовтяничнiсть шкiри, склер; температура тiла пiдвищена, печiнка збiльшена, сеча темна, кал гiпохолiчний. Гiпербiлiрубiнемiя (бiлiрубiн прямий та непрямий), бiлiрубiнурiя, уробiлiнурiя, гiпопротеїнемiя, зниження зсiдання кровi. Для якого з перелiчених нижче станiв найбiльш характернi цi змiни?
+Клiтиннопаренхiматозна жовтяниця
-Пiдпечiнкова жовтяниця
-Надпечiнкова гемолiтична жовтяниця
-Гострий холецистит
-Гострий панкреатит


#У хворого з алкогольним цирозом печiнки скарги на загальну слабкiсть, задишку. Встановлено зниження артерiального тиску, асцит, розширення по-верхневих вен передньої стiнки живота, спленомегалiю. Яке порушення гемодинамiки спостерiгається у хворого?
+Синдром портальної гiпертензiї
- Недостатнiсть лiвого шлуночка серця
- Недостатнiсть правого шлуночка серця
- Колапс
- Тотальна серцева недостатнiсть


#Жінка 55 років, скаржиться на швидку втомлюваність, роздратованість, безсоння та сонливий стан в день, свербіння шкіри. Пульс 58 уд/хв., АТ-110/65 мм рт.ст. Кал безколірний, містить багато жиру. Підозрюється жовчнокам'яна хвороба з закупорюванням каменем жовчної протоки.Що з перелікованого нижче найбільш вірогідно обумовлює стан хворої?
+холемія
-гіпербілірубінемія
-гіперхолестеринемія
-порушення всмоктування жиророзчинних вітамінів
-порушення всмоктування жирів


#У хворого жовтяницею при лабораторному дослідженні отримано такі дані,


характеризують порушення пігментного обміну: загальний вміст в сироватці


крові білірубіну - 80 мкмоль / л; вміст прямого білірубіну в сироватці крові -63 мкмоль / л; в сечі присутній білірубін; уробіліноген і стеркобилин в сечі


відсутній. Яка причина відсутності в сечі уробіліногену при даному виді жовтяниці?
+Відсутність надходження в кишечник білірубіну
- Порушення всмоктування уробилину в кишечнику
- Порушення виділення уробіліну нирками
-Порушення зв'язування уробіліну транспортним білком
-Дефіцит ферментів, що перетворюють глюкоронідбілірубін в уробіліноген


#Хворий скаржиться на біль в правому підребір ї , зуд шкіри, головний біль, дратливість, швидку стомлюваність. При обстеженні встановлено: жовтяничний колір шкіри та слизових оболонок, печінка збільшена, болісна при пальпації. АТ - 80/40 мм рт.ст., ЧСС - 46/хв. В крові виявлено: вільний білірубін - 34,15 мкмоль/л, звўязаний - 35,2 мкмоль/л, жовчні кислоти; в сечі - жовчні кислоти, звўязаний білірубін, уробіліноген; вміст стеркобіліногену в калі зменшений. Який вид жовтяниці у хворого?
+Печінково-клітинна.
-Гемолітична.
-Підпечінкова.
-Обтураційна.
-Транспортне


#У хворого з пухлинної обтурацією жовчовивідних шляхів у складі калу міститься велика кількість жиру (стеаторрея). Недолік якого компонента жовчі обумовлює це явище?
+Жовчних кислот
-Жирних кислот
-Холестерину
-Жовчних пігментів
-Лужний фосфатази


#У хворого С. з вираженим жовтушним синдромом виявлено: в крові: рівень непрямого білірубіну - 34,5 мкмоль/л, прямого - 35,2 мкмоль/л; в сечі: жовчні кислоти, уробіліноген; в калі – кількість стеркобіліногену зменшено. Який найбільш ймовірний вид жовтяниці розвинувся у хворого?
+Печінкова
-Надпечінкова
-Підпечінкова
--


#У хворого після отруєння грибами з’явилося жовте забарвлення шкіри та склер, темний колір сечі. Який пігмент спричинює забарвлення сечі у хворого на гемолітичну жовтяницю?
+ Стеркобілін
-Моноглюкуронід білірубіну
-Некон’югований білірубін
-Вердоглобін
-Білівердин


#Хвора на хронічний гепатит скаржиться на підвищення чутливості до барбітуратів, які раніше вона переносила без симптомів інтоксикації. З порушенням якої функції печінки це пов’язане у найбільшій мірі?
+ Метаболічної
-Утворення жовчі
-Гемодинамічної
-Гемопоетичної
-Фагоцитарної


#У жінки 57 років після тривалого больового нападу у правому підребір’ї з’явилась жовтяниця, після чого хвора звернулась до лікаря. Виникла підозра на появність у хворої гострого калькульозного холециститу. Дослідження якого показника крові свідчить про непрохідність жовчних протоків?
+Вільного та зв’язаного білірубіну
-Білкових фракцій
-Загальних ліпідів
-Сечової кислоти
-Залишкового азоту


#Хлопчику 15 років встановлено діагноз гострого вірусного гепатиту. Дослідження якого показника крові необхідно провести для підтвердження гострого ураження печінкових клітин?
+Активність амінотрансфераз [ЛЛТ і АСТ]
-Вміст вільного та зв’язаного білірубіну
-Швидкість осідання еритроцитів [ШОЕ].
-Рівень холестерину
-Вміст білкових фракцій


#Хворому 25 років встановлено діагноз хронічного гепатиту. Хворий скаржиться на втрату маси тіла на 10 кг протягом 2 місяців. Об'єктивно: шкіра суха, лущиться, бліда з жовтуватим відтінком, мелкоточечние крововиливи на шкірі, кровоточивість ясен. Порушення якої функції печінки відображають мелкоточечние крововиливи і кровоточивість ясен:
+ білоксинтетичної
-пигментоформуючої
-Глікогенсінтетіческой
-детоксикаційної
-депонуючої


#У хворого 38 років, який переніс гепатит і продовжував вживати алкоголь, розвинулися ознаки цирозу печінки з асцитом і набряками на нижніх кінцівках. Які зміни складу крові стали вирішальними в розвитку набряків?
+ Гіпоальбумінемія
-Гіпоглобулінемія
-Гіпохолестеринемія
-Гіпокаліємія
-Гіпоглікемія


#У повної жінки 52 років встановлено цироз печінки. Лабораторно: гіпоальбумінемія, гіперглобулінемія. Візуально: набряк рук, повік, ніг. Найбільш імовірною причиною набряків є зміна:
+Онкотичного тиску крові
-Буферної емності крові
-Кислотно-лужної рівноваги
-Дезінтоксикаційної функції печінки
-Глікогенсинтезуючої функції печінки


#У пацієнта, що звернувся до лікаря, спостерігається жовте забарвлення шкіри, сеча - темна, кал темно-жовтого кольору. Підвищення концентрації якої речовини буде спостерігатися в сироватці крові?
+Вільний білірубін
-Білівердин
-Вердоглобін
-Кон’югований білірубін
-Мезобілірубін


#У хворого на жовчно-кам’яну хворобу спостерігаються ознаки холемічного синдрому. Який симптом з перерахованих зумовлений відсутністю надходження жовчі у кишечник?
+Стеаторея
-Астенія
-Брадикардія
-Гіпотонія
-Шкірний свербіж


# Введення знеболюючого пацiєнту перед екстракцiєю зуба призвело до розвитку анафiлактичного шоку, який супроводжувався розвитком олiгурiї. Який патогенетичний механiзм зумовив зменшення дiурезу в данiй клiнiчнiй ситуацiї?
+ Зниження гiдростатичного тиску в капiлярах клубочкiв
- Пiдвищення гiдростатичного тиску в капсулi Шумлянського-Боумена
-Пошкодження клубочкового фiльтру
- Збiльшення онкотичного тиску кровi
- Зменшення кiлькостi функцiонуючих нефронів


# У чоловiка 25-ти рокiв дiагностований гострий дифузний гломерулонефрит. З анамнезу: за 18 днiв до прояву хвороби перенiс ангiну. Який механiзм ураження ниркових клубочкiв буде спостерiгатися у цьому випадку?
+ Iмунний
- Нефротоксичний
- Iшемiчний
- Медикаментозний


#У дитини 10-ти років через 2 тижні після перенесеної ангіни розвинувся нефритичний синдром (протеїнурія, гемату­рія, циліндрурія), що свідчить про уражен­ня базальної мембрани клубочків нирок. Який найбільш імовірний механізм лежить в основі ушкодження базальної мембрани?
+Імунокомплексний
-Реагіновий
-Антитільний
-Гранулематозний
-Цитотоксичний


#У пацієнта з хронічним захворюванням нирок розвинулась ниркова недостатність. Який з показників найбільш імовірно свідчить про порушення реабсорбції в канальцях в даному випадку?
+Гіпо- та ізостенурія
- Лейкоцитурія
- Зниження кліренсу
- Гіперазотемія
- Гематурія


#У хворого 38-ми років на 3-му році захворювання на системний червоний вівчак виявлене дифузне ураження нирок, що супроводжується масивними набряками і вираженою протеїнурією. Що є найбільш вірогідною причиною розвитку протеїнурії у пацієнта?
+Аутоімунне ушкодження нирок
-Асептичне ураження нирок
-Запальне ураження сечового міхура
-Ішемічне ушкодження нирок
-Запальне ураження сечовивідних шляхів


#У хворого з масивними опіками розви­нулась гостра недостатність нирок, що ха­рактеризується значним і швидким змен­шенням швидкості клубочкової фільтра­ції. Який механізм її розвитку?
+Зменшення ниркового кровотоку
-Збільшення тиску канальцевої рідини
-Ушкодження клубочкового фільтра
-Зменшення кількості функціонуючих нефронів
-Емболія ниркової артерії


#У жiнки 30-ти рокiв виникли набряки обличчя. При обстеженнi виявленi протеїнурiя (5,87 г/л), гiпопротеїнемiя, диспротеїнемiя, гiперлiпiдемiя. Для якого стану характерно таке поєднання симптомiв?
+Нефротичний синдром
-Нефритичний синдром
-Хронiчний пiєлонефрит
-Гостра ниркова недостатнiсть
-Хронiчна ниркова недостатнiсть


#У результатi порушення технiки безпеки вiдбулося отруєння сулемою (хлористою ртуттю). Через 2 днi добовий дiурез склав 620 мл. У хворого з’явилися головний бiль, блювання,судоми, задишка, у легенях - вологi хрипи. Яка патологiя має мiсце?
+Гостра ниркова недостатнiсть
- Хронiчна ниркова недостатнiсть
- Уремiчна кома
- Гломерулонефрит
- Пiєлонефрит


#У хворого з гострою нирковою недостатнiстю на 6-й день проведення терапевтичних заходiв виникла полiурiя. Чим зумовлене зростання дiурезу на початку полiуричної стадiї гострої ниркової недостатностi?
+Вiдновлення фiльтрацiї в нефронах
-Збiльшенням об’єму циркулюючої кровi
-Збiльшенням натрiйуретичного фактора
-Зменшенням альдостерону в плазмi
-Зменшенням вазопресину в плазмi


#У людини внаслiдок тривалого голодування швидкiсть клубочкової фiльтрацiї зросла на 20%. Найбiльш вiрогiдною причиною змiн фiльтрацiї в зазначених умовах є:
+Зменшення онкотичного тиску плазми кровi
-Збiльшення системного артерiального тиску
-Збiльшення проникностi ниркового фiльтру
-Збiльшення коефiцiєнта фiльтрацiї
-Збiльшення ниркового плазмотоку


#Чоловiк 65-ти рокiв, який страждає на подагру, скаржиться на бiль в дiлянцi нирок. При ультразвуковому обстеженнi встановлена наявнiсть ниркових каменiв. Пiдвищення концентрацiї якої речовини є найбiльш вiрогiдною причиною утворення каменiв у даному випадку?
+Сечова кислота
-Холестерин
-Бiлiрубiн
-Сечовина
-Цистин


#На препаратi нирки розрiзняються нефрони, якi лежать на межi мiж кiр-ковою та мозковою речовиною, мають однаковий дiаметр приносних i виносних артерiол. Назвiть, яка функцiя буде порушена при їхньому пошкодженнi?
+ Шунтування кровi при iнтенсивному кровообiгу
- Синтез ренiну
- Синтез простагландинiв
- Синтез еритропоетину
- Активнiсть натрiєвого рецептора


#Хворий 55-ти рокiв хворiє на хронiчний гломерулонефрит протягом 15-ти рокiв. Якi змiни складу кровi або сечi найбiльш характерно свiдчать про обмеження секреторної функцiї нирок?
+ Гiперазотемiя
- Гiперглiкемiя
- Гiпопротеїнемiя
- Протеїнурiя
- Гiпо-, iзостенурiя


#У жінки з первинним гіперпаратиреоідизмом періодично повторюються напади ниркової коліки. Ультразвукове обстеження показало наявність дрібних каменів в нирках, найбільш імовірною причиною утворення яких є:
+Гіперкальциемія
-Гіперфосфатемія
-Гіперхолестеринемія
-Гіперурікемія
-Гіперкаліемія


#У хворого після автомобільної травми артеріальний тиск 70/40 мм рт.ст.


Хворий в несвідомому стані. В добу виділяє близько 550 мл сечі. Періодично виникають судоми, дихання за типом Куссмауля. Як називається таке порушення функції нирок? Хворий в несвідомому стані. В добу виділяє


близько 550 мл сечі. Періодично виникають судоми, дихання за типом Куссмауля. Як називається таке порушення функції нирок?
+ Гостра ниркова недостатність
-Гострий дифузний гломерулонефрит
-Тубулопатія
-Хронічна ниркова недостатність
-Пієлонефрит


#У хворого А. після травматичного шоку развинулись ознаки ниркової недостатності. Якими патогенетичними механізмами зумовлено цей стан?
+Зниженням объёму клубочкової фільтрації
-Обтурацією канальців нирок
-Пригніченням екскреції сечі в канальцях
-Блокування відтоку сечі
-Пошкодження клубочкового апарату нирок


#Чоловік 30 років, скаржиться на слабкість, жагу, головний біль та біль у попереку. Місяць тому перехворів ангіною. На обличчі набряки. Пульс – 84 уд/хв, АТ - 175/100 мм рт.ст. В сечі - еритроцити 40-52 в полі зору, лейкоцити - 1- 2 в полі зору, білок - 4 г/л. Встановлено діагноз гострого дифузного гломерулонефриту. Який основний механізм ураження нирок у данного хворого?
+імунне пошкодження клубочків
-пошкодження канальців
-порушення гемодинаміки в нирках
-порушення уродинаміки
-пряме пошкодження клубочків мікроорганізмами


#У хворого 24 років через півтора тижні після важкої стрептококової ангіни проявилася набряклість обличчя, підвищився артеріальний тиск. Гематурія і протеїнурія 1,2 г / л. У крові виявлені антистрептококові антитіла і зниження компонентів комплементу. В мікросудинах яких структур найбільш ймовірно локалізація скупчень імунних комплексів, що зумовили розвиток нефропатії?
+Клубочки
-Піраміди
-Проксимальний відділ канальців
-Петля Генле
-дистальний відділ канальців


#Чоловік 32 років впродовж 4 років хворіє на хронічний гломерулонефрит.


Госпіталізований з ознаками анасарки: АТ- 185/105 мм рт.ст. У крові: Hb- 110 г/л, ер.- 2,6*1012/л, лейк.- 9,5*109/л, залишковий азот - 32 ммоль/л, загальний білок - 50 г/л. Яка зміна з найбільшою вірогідністю вказує на гломерулонефрит з нефротичним синдромом?
+Гіпопротеїнемія
-Анемія
-Лейкоцитоз
-Артеріальна гіпертензія
-Гіперазотемія


#В експерименті кролю ввели нефроцитотоксичну сироватку морськоі свинки. Яке захворювання нирок моделювалося в цьому досліді?
+ Гострий дифузний гломерулонефрит
-Нефротичний синдром
-Гострий пієлонефрит
-Хронічна ниркова недостатність
-Хронічний пієлонефрит


#У хворого із хронічною нирковою недостатністю встановлення зменшення кліренсу за інуліном до 60 мл/хв. З порушенням якої функції нирок це пов’язано?
+ Клубочкової фільтрації
-Канальцевої секреції
-Реабсорбції в проксимальному відділі нефрону
-Реабсорбції в дистальному відділі нефрону
-Реабсорбції в збиральних ниркових трубочках


#У хворого на хронічну ниркову недостатність з’явилися анорексія, диспепсія, порушення ритму серця, свербіння шкіри. Який механізм розвитку цих порушень є головним?
+ Накопичення продуктів азотистого обміну в крові
-Порушення ліпідного обміну
-Зміни вуглеводного обміну
-Нирковий ацидоз
-Порушення водно-електролітного обміну


#Хворий чоловік на протязі 17 років страждає хронічним гломерулонефритом. Пульс 82 за хвилину. АТ 190/120 мм рт.ст. Що являється первинним механізмом підвищення артеріального тиску у хворого?
+Підвищення загального периферичного опору.
-Збільшення об’єму циркулюючої крові;
-Підвищення тонусу венозних судин;
-Збільшення ударного об’єму крові;
-Збільшення хвилинного об’єму крові;


#У жінки обмежений кровотік у нирках, підвищений артеріальний тиск. Гіперсекреція якого гормону зумовила підвищений тиск?
+Ренін
-Адреналін
-Вазопресин
-Еритропоетин
-Норадреналін


#При розтині хворої 28-ми років, що померла від уремії, виявлені збільшені строкаті нирки з осередками крововиливів. Патогістологічно в судинних клубочках виявлені гематоксилінові тільця, капілярні мембрани клубочків у вигляді дротяних петель, гіалінові тромби та осередки фібриноїдного некрозу. За патогенезом гіперчутливість якого типу лежить в основі описаної хвороби?
+Гіперчутливість III типу (імунокомплексна)
-Гіперчутливість I типу (анафілактична)
-Гіперчутливість II типу (антитілозалежна)
-Гіперчутливість IV типу (клітинна цитотоксичність)
-Гіперчутливість V типу (гранулематоз)


#У хворого на хронічний дифузний гломерулонефрит розвинулася хронічна недостатність нирок. В термінальній стадії ХНН розвивається оліго- та анурія, що спричиняється:
+Зниженням маси діючих нефронів
-Ішемією коркової речовини нирок внаслідок спазму судин
-Дисемінованим внутрішньосудинним зсіданням крові
-Збільшенням реабсорбції води в дистальних канальцях
-Зменшенням фільтраційного тиску та фільтрації


#До лiкаря звернувся чоловiк 27-ми рокiв. При оглядi було виявлено збiльшення кистей, стоп та нижньої щелепи. Крiм того спостерiгалися деформацiя суглобiв (kiphosis), гормональнi порушення (iмпотенцiя, атрофiя яєчок). Функцiя якої залози порушена?
+ Передня частка гiпофiзу
- Наднирковi залози
- Шишкоподiбне тiло
- Щитоподiбна залоза
- Прищитоподiбнi залози


#У чоловiка 41-го року вiдзначаються перiодичнi напади серцебиття (пароксизми), сильне потовидiлення, напади головного болю. При обстеженнi виявлена гiпертензiя, гiперглiкемiя, пiдвищення основного обмiну, тахiкардiя. При якiй патологiї наднирникiв спостерiгається подiбна картина?
+ Гiперфункцiя мозкового шару
- Гiпофункцiя мозкового шару
- Гiперфункцiя кори наднирникiв
- Гiпофункцiя кори наднирникiв
- Первинний альдостеронiзм


#Жінка 37-ми років звернулася до ендо­кринологічної клініки з виразним тремо­ром кінцівок. Гіперпродукція якого гормо­ну здатна викликати такі порушення?
+Тироксин
-Адреналін
-АКТГ
-Соматостатин
-Інсулін


#У пацієнта з підвищеним артеріальним тиском, тремором, тахікардією, була діагностовано доброякісна пухлина мозкової речовини наднирників. Гіперсекреція якого гормону викликає таку симптоматику?
+Адреналін
- Соматотропін
- Інсулін
- Тироксин
- Глюкагон


#Після перенесеного сепсису у хворої 27-ми років з'явився бронзовий колір шкіри, характерний для аддісонової хвороби. Механізм гіперпігментації полягає в підвищенні секреції такого гормону:
+ Меланоцитстимулюючий
- B-ліпотропний
- Соматотропний
- Гонадотропний
- Тиреотропний


#Пацієнта турбують поліурія (7 л на добу) і полідипсія. При обстеженні не виявлено ніяких розладів вуглеводного обміну. Дисфункція якої ендокринної залози може бути причиною даних порушень?
+Нейрогіпофіз
- Кора наднирників
- Острівці підшлункової залози
- Аденогіпофіз
- Мозкова речовина наднирників


# Хлопчик 5-ти місяців госпіталізова­ний з приводу тонічних судом. Хворіє з на­родження. Об’єктивно: волосся жорстке, нігті витончені та ламкі, шкірні покриви бліді та сухі. В біохімічному аналізі кро­ві: кальцій - 0,5 ммоль/л (норма - 0,75-2,5 ммоль/л), фосфор - 1,9 ммоль/л (норма - 0,646-1,292 ммоль/л). З чим пов’язані ці зміни?
+Гіпопаратиреоз
-Гіпоальдостеронізм
-Гіперальдостеронізм
-Гіперпаратиреоз
-Гіпотиреоз


#До лікаря звернулася жінка 32-х років зі скаргами на відсутність лактації після народження дитини. Дефіцитом якого гормону можна пояснити дане порушення?
+Пролактин
- Соматотропін
- Тиреокальцитонін
- Вазопресин
- Глюкагон


#Чоловік середнього віку виїхав до іншої країни на обіцяну йому роботу, але працевлаштуватися тривалий час йому не вдавалося. Які з ендокринних залоз були виснажені у цієї людини найбільше?
+Наднирники
- Сім'яники
- Підгрудинна
- Прищитоподібні
- Щитоподібна


#У дівчинки діагностований адреногенітальний синдром (псевдогермафродитизм). Надмірна секреція яких гормонів наднирників обумовила дану патологію?
+Андрогени
- Катехоламіни
- Естрогени
- Мінералокортикоїди
- Глюкокортикоїди


#При огляді пацієнта виявлене надмірне розростання кісток і м'яких тканин обличчя, збільшені розміри язика, розширені міжзубні проміжки в збільшеній зубній дузі. Які зміни секреції гормонів у нього найбільш вірогідні?
+ Збільшена секреція соматотропного гормону
- Зменшена секреція тироксину
-Зменшена секреція інсуліну
- Зменшена секреція соматотропного гормону
- Збільшена секреція інсуліну


#Чоловіку 46-ти років, що хворіє на дифузний токсичний зоб, була проведена операція резекції щитоподібної залози. Після операції відмічаються відсутність апетиту, диспепсія, підвищена нервово-м'язова збудливість. Маса тіла не збільшилася. Температура тіла у нормі. Чим, із нижче переліченого, обумовлений стан хворого?
+Зниженням продукції паратгормону
- Підвищенням продукції тироксину
- Підвищенням продукції кальцитоніну
- Підвищенням продукції тиреоліберину
- Зниженням продукції тироксину


#У пiдлiтка внаслiдок радiоактивного опромiнення значно постраждала лiмфоїдна система, вiдбувся розпад великої кiлькостi лiмфоцитiв. Вiдновлення нормальної формули кровi можливо завдяки дiяльностi залози:
+Тимус
-Щитоподiбна
-Печiнка
-Пiдшлункова
-Наднирники


#Жiнка 44-х рокiв скаржиться на загальну слабкiсть, бiль у дiлянцi серця, значне збiльшення маси тiла. Об’єктивно: обличчя мiсяцеподiбне, гiрсутизм, АТ- 165/100 мм рт.ст., зрiст - 164 см, вага - 103 кг; переважно накопичення жиру на шиї, верхньому плечовому поясi, животi. Що є основним патогенетичним механiзмом ожирiння у жiнки?
+Пiдвищення продукцiї глюкокортикоїдiв
-Зниження продукцiї тиреоїдних гормонiв
-Пiдвищення продукцiї iнсулiну
-Зниження продукцiї глюкагону
-Пiдвищення продукцiї мiнералокортикоїдiв


#Недбалий студент раптово зустрiвся з деканом. Концентрацiя якого гормону найшвидше збiльшиться в кровi студента?
+ Адреналiн
- Тиреолiберин
- Кортикотропiн
- Кортизол
- Соматотропiн


#Жінка 26 років через рік після важких пологів з кровотечею скаржиться на загальну слабкість, втрату маси тіла на 18 кг, відсутність менструацій. Об'єктивно: гіпоплазія молочних залоз. Діагностовано хвороба Сіммондса. Що являється основним механізмом втрати ваги у жінки?
+зниження продукції гормонів аденогіпофізу
-зниження функції статевих залоз
-зниження функції кіркового шару наднирників
-гіпотиреоз
-гіпопаратиреоз


#У хворого при обстеженні виявлено гірсутизм, "місяцеподібне обличчя", смуги розтягування на шкірі живота. Артеріальний тиск 190/100 мм рт.ст., глюкоза крові - 17,6 ммоль / л. При якому із зазначених видів патології зустрічається подібна картина?
+гіперфункції кори надниркових залоз
-гіпертиреоз
-гіпотиреоз
-гіпофункція статевих залоз
-гіперфункція інсулярного апарату


#Після двостороннього видалення надниркових залоз у собаки з'явилася м'язова слабкість, адинамія, зниження температури тіла, гіпоглікемія. Який з перерахованих ознак ще може спостерігатись при надниркової недостатності?
+ Артеріальна гіпотонія
-Лімфопенія
-Посилення синтезу глікогену
-Збільшення концентрації натрію і хлоридів в сироватці крові
-Підвищення резистентності до дії бактерій і токсинів


#У хворого з гіперфункцією щитовидної залози підвищена температура тіла. Яке порушення енергетичного обміну є головним у підвищенні температури при цьому?
+Роз єднання окиснення та окисного фосфорування.
-Збільшення розпаду глікогену.
-Посилення ліполізу.
-Активація ферментів в циклі Кребса.
-Активація ферментів дихального ланцюга.


#Жінка 55 років, яка проживає в гірській місцевості, діагностований ендемічний зоб. Об'єктивно: трохи підвищеної вгодованості, загальмована, апатична, збільшення щитовидної залози. Дефіцит якого з перелічених нижче елементів викликає цей стан?
+йод
-фтор
-марганець
-молібден
-натрій


#У хлопчиkа діагностовано ендемічний зоб. Який основний механізм розвитку гіпотиреозу у хлопця?
+зниження продукції тироксину та трийодтиранину
-зниження продукції тиреотропіну
-зниження чутливості рецепторів тканин до тироксину та трийодтиранину
-підвищення метаболізму тироксину та трийодтиранину
-порушення транспорту тироксину та трийодтиранину


#Методом непрямої калориметрії встановлено, що основний обмін досліджуваного на 40% нижче повинного. Порушення діяльності якої ендокринної залози можна припустити?
+Щитовидної залози.
-Тимуса.
-Підшлункової залози.
-Епіфіза
-Прищитовидних залоз


#У 50-річної хворої після перенесеного інфекційного захворювання головного мозку значно збільшився діурез до 12 л за добу. При аналізі крові кількість глюкози становила 4,1 ммоль/л. Якого гормону найвірогідніше недостатньо?
+Антидіуретичного
-Глюкагону
-Інсуліну
-Кортизону
-Альдостерону


#У клініці встановлено, що при вагітності тяжкість симптомів ревматоїдного артриту різко знижується. Прискорення секреції яких гормонів, що володіють протизапальну дію, спостерігається при цьому?
+Глюкокортикоїдів
-естрогенів
-иодированная щитовидної залози
-Катехоламінів
-гонадотропні


#Жінка 29 років скаржиться на загальну слабкість, втрату маси тіла на 22 кг, аменорею. Хворіє після пологів. Об’єктивно: ріст 162 см, маса 46 кг, гіпоплазія молочних залоз. Діагностована гіпофізарна кахексія. Зменшення продукції якого гормону явилось найбільш суттєвим у схудненні жінки?
+Соматотропіну.
-Адренокортикотропіну.
-Тиреотропіну.
-Меланотропіну.
-Пролактотропіну.


#У хворої після пологів развинулись такі ознаки: атрофія скелетних м(язів, дістрофія шкіри, випадання волосся, гіпотрофія внутрішніх органів, зниження температури тіла, артеріального тиску, рівня глюкози в крові, атрофія щитовидної, надниркових та статевих залоз. Для якої патології характерні ці ознаки?
+Атрофії гіпофіза
-Пошкодження статевих залоз
-Гіпофункції щитовидної залози
-Гіперфункції аденогіпофіза
-Пошкодження надниркових залоз


#У хворого встановлено стабільне підвищення температури тіла, тахікардія, емоційна лабільність, тремор. Зі зміною продукціі якого гормона пов’язане виникнення цього стану?
+ Тироксину.
-Вазопресину.
-Тестостерону.
-Альдостерону.
-Інсуліну.


#У ліквідатора аварії на Чорнобильській АЕС через деякий час з'явилися скарги на підвищену збудливість, нервозність, серцебиття, зниження маса тіла, постійну слабкість, тремтіння тіла, відчуття жару, погану переносимість тепла. Гіперфункція якої залози може бути причиною зазначених змін:
+щитовидної залози
-аденогіпофіза
-коркової речовини надниркових залоз
-мозкової речовини надниркових залоз
-паращитовидних залоз


#У собаки з ендокринною патологією було виявлено: зменшення споживання кисню у стані спокою, зниження толерантності до глюкози, зниження температури тіла. Недостатністю якого гормону можна пояснити виявлені зміни?
+Тироксину
-Інсуліну
-Соматотропного
-Адренокортикотропного
-Гонадотропного


#Хворий жаліється на підвищену дратливість, періодичний субфібрілітет. Частота пульсу -120/хв. В крові збільшена кількість гормонів Т3 і Т4. Яку ендокринну патологію найбільш логічно запідозрити?
+ Гіпертіреоз.
-Надниркову недостатність.
-Гіпопаратіреоз.
-Гіперпаратіреоз.
-Гіпотіреоз.


#У хворого спостерігаються напади артеріальної гіпертензії, що супроводжуються тахікардією, рясним потовиділенням, різким болем в надчеревній ділянці. Для якого з перечисленних пухлинних захворювань залоз внутрішньої секреції найбільш характерні дані симптоми?
+ Феохромоцитома
-Базофільна аденома гіпофіза
-Аденома клубочкової зони наднирників
-Аденома щитовидної залози
-Пухлина яєчників


#Жінка 53 років, ріст 163 см., вага тіла 92 кг, рівномірне відткладання жиру, лице одутле, малорухома, апатична. При натискуванні шкіри ноги залишається ямка. Порушенням функції якої залози обумовлений стан хвороби?
+Щитовидної.
-Гіпофізу.
-Надниркових.
-Статевих.
-Прищитовидних.


#У хворого 48 років спостерігається артеріальна гіпертензія, головний біль, м’язoва слабкість, судоми. В крові знижена концентрація К
+ і підвищена концентрація Na+, що є наслідком гіперсекреції:
+Альдостерону
-Адреналину
-Паратгормону
-Кортизолу
-Дигідрохолестеролу


#У чоловіка 35-ти років феохромоцитома. В крові спостерігається підвищений рівень адреналіну та норадреналіну, концентрація вільних жирних кислот зросла в 11 разів. Активація якого ферменту під впливом адреналіну підвищує ліполіз?
+ТАГ -ліпаза
-Ліпопротеїдліпаза
-Фосфоліпаза А2
-Фосфоліпаза С
-Холестеролестераза


#При гістологічному дослідженні щитоподібної залози визначається значна інфільтрація лімфоцитами з утворенням лімфоїдних фолікулів, руйнування паренхіматозних елементів, розростання волокон сполучної тканини. Для якого захворювання характерна така картина?
+Зоб Хашимото
-Дифузний токсичний зоб
-Ендемічний зоб
-Колоїдний зоб
-Паренхіматозний зоб


#Пацієнт 16-ти років, що страждає на хворобу Іценко-Кушінга, консультований з приводу надмірної ваги тіла. При опитуванні з’ясувалося, що енергетична цінність спожитої їжі складає 1700-1900 ккал/добу. Яка провідна причина ожиріння у даному випадку?
+Надлишок глюкокортикоїдів
-Гіподинамія
-Надлишок інсуліну
-Нестача інсуліну
-Нестача глюкокортикоїдів


#У чоловіка 25-ти років з переломом основи черепа виділяється великий об’єм сечі з низькою відносною щільністю. Причиною змін сечоутворення є порушення синтезу такого гормону:
+Вазопресин
-Адренокортикотропний гормон
-Окситоцин
-Соматотропний гормон
-Тиреотропний гормон


#An unconscious patient was brought into the hospital. The smell of acetone can be detected from the patient’s mouth. Blood glucose - 25 mmol/L, ketone bodies -0.57 mmol/L. What hormone deficiency can result in the development of this condition?
+Insulin
-Aldosterone
-Glucocorticoids
-Somatotropin
-Thyroxin


#У людини виявлена пухлина одного з вiддiлiв головного мозку, внаслiдок чого в неї порушена здатнiсть пiдтримувати нормальну температуру тiла. Яка структура головного мозку пошкоджена?
+ Гiпоталамус
- Таламус
- Мозочок
- Стрiатум
- Чорна субстанція


# В клiнiку доставлено чоловiка з травмою спини. Пiд час обстеження виявлено перелом хребцiв грудного вiддiлу. Пiд час об’єктивного огляду нейрохiрургом виявлено: нижче рiвня перелому з правого боку вiдсутня глибока чутливiсть, з лiвого боку - порушена температурна та тактильна чутливiсть. Яке ураження з боку спинного мозку є у хворого?
+ Синдром Броун Секара
- Хвороба Паркiнсона
- Судомний синдром
- Анестезiя
- Парастезiя


# У хворої 49-ти рокiв вiдзначається обмеження довiльних рухiв у лiвих кiнцiвках. Тонус м’язiв у лiвих руцi та нозi пiдвищений за спастичним типом, посиленi мiсцевi сухожилковi рефлекси, виявляються патологiчнi рефлекси. Який найбiльш імовірний механізм призвiвдо розвитку м’язової гiпертонiї та гiперрефлексiї?
+ Зниження гальмівних низхідних впливiв
- Активацiя мотонейронiв внаслiдок iнсульту
- Активацiя збуджувальних впливiв з вогнища iнсульту
- Активацiя синаптичної передачi імпульсів
- Гальмування мотонейронiв кори головного мозку


#В експерименті у тварини в результаті проведеного перетинання депресорного нерва та руйнування каротидних клубочків розвинулась стійка гіпертензія. З порушен¬ням якої функції нервової системи пов’язане це явище?
+ Вегетативна
- Вища нервова діяльність
- Сенсорна
- Трофічна
- Рухова


#У людини звуженi зiницi. Чим це зумовлено?
+Зростання тонусу парасимпатичних центрiв
-Зростання тонусу симпатичних цен-трiв
-Збiльшення активностi симпато-адреналової системи
-Дiя адреналiну
-Дiя норадреналiну


#У стоматолога на прийомi дуже неспокiйний пацiєнт, який нiяк не може зручно влаштуватися в крiслi, норовить схопити доктора за руку, заглядає на манiпуляцiйний стiл, цiкавиться в медсестри, чи стерильнi iнструменти. Який темперамент у цього пацiєнта?
+Холерик
-Сангвiнiк
-Флегматик
-Меланхолiк
-Немає правильної відповіді


#У жінки 68 років після інсульту відсутні рухи в верхній та нижній правій кінцівках. Тонус м’язів цих кінцівок і рефлекси в них підвищені. Є патологічні рефлекси. Яка це форма параліча?
+Геміплегія
-Параплегія
-Тетраплегія
-Моноплегія
-Дисоціація


#У жінки, що хворіє на міастенію, виникли розлади дихання, що вимагали застосування штучної вентиляції легень. Який вид недостатності розвинувся у даної хворої?
+Нервово-м'язовий.
-Центрогенний.
-Торакодіафрагмальний.
-Обструктивний.
-Рестриктивний


#Після крововиливу в мозок у хворого стали неможливими активні рухи лівої руки і ноги. Тонус м'язів цих кінцівок збільшений, їх спинальні рефлекси різко посилені, розширені зони рефлексів. Позитивний рефлекс Бабінського Назвіть вид розладу нервової системи у хворого.
+Центральний параліч.
-Периферичний параліч.
-Спинальний шок.
-Децеребраційна ригідність
-Рефлекторний параліч


#У хворого через 7 років після резекції шлунка виявлена ​​гіперхромна В12


дефіцитна анемія і фунікулярний міелоз. Який патогенез уражень спинного мозку?
+ Накопичення метилмалонової кислоти.
-Гіпоксичне пошкодження тканин мозку
-Порушення синтезу ДНК
-Дефіцит фолієвої кислоти
-Гіпотрансферинемія


#У хворого в результаті вогнепального поранення стегна пошкоджений сідничний нерв. Будь-який вплив на хвору кінцівку спричиняє жорсткий, нестерпний біль. Який механізм формування болісних відчуттів найбільш ймовірний у цьому випадку?
+Каузалгічний
-Рефлекторний
-Фантомний
-Гіпофункція ендорфіну
-Гіпофункція енкефаліну


#У тварини в експерименті проведена перерізка обох блукаючих нервів. Як зміниться характер дихання у тварини?
+ Рідке і глибоке
-Часте та глибоке
-Рідке та поверхове
-Часте та поверхове
-Періодичне


#У хворого після відкритої травми хребта виявлено розрив правої половини спинного мозку. Зникнення якого виду чутливості слід очікувати тількі з боку розриву?
+ Пропріоцептиної
-Температурної
-Больової
-Тактильної
--


#Під час бойових дій у госпіталь доставили солдата з тяжким осколочним пошкодженням хребта. У пораненого встановлено наявність перерізки правої половини спинного мозку (синдром Броун-Секара). Зникненням якого виду чутливости проявляеться цей синдром?
+ пропріоцептивної - справа
-температурної - справа
-пропріоцептивної - зліва
-больової - справа
-тактильноі - справа


#У хворої Л., 40 років, через місяць після поєднаної автомобільної травми відсутні активні рухи у всіх суглобах правої ноги. Обсяг м'язів в області гомілки правої ноги на 2 см менше, ніж на лівій. Ахилов і колінний рефлекси справа відсутні. Визначається термгіпестезія і гіпальгезія на зовнішній поверхні правої ноги, втрата пропріоцептивної чутливості в ділянці стопи. Порушенням якого відділу рухового аналізатора обумовлені наявні розлади?
+Периферичних нервів
-Спиномозкових провідних шляхів
-пірамідних нейронів
-Нервово-м'язової синаптичноїпередачі
-екстрапірамідного нейронів


#Внаслідок ДТП у потерпілої 37-ми років виникло неутримання сечі. Які сегменти спинного мозку пошкоджені?
+S2-S4
-L1 — L2
-Th1 — L1
-Th1 — Th5
-Th2 — Th5


#У хворого, який на тлі атеросклерозу переніс ішемічний інсульт, спостерігається порушення рухової функції у вигляді геміплегії. Яка з перерахованих ознак є характерною для уражених кінцівок при даній патології?
+Гіпертонус м’язів
-Гіпорефлексія
-Гіпотонус м’язів
-Трофічні розлади


#Жiнку 44-х рокiв вжалила оса внаслiдок чого розвинувся шок. В анамнезi вже була важка алергiчна реакцiя на жалення оси. Об’єктивно: пульс 179/хв., слабкий,АТ-80/40 ммрт.ст., ЧД26/хв. Яка провiдна ланка патогенезу анафiлактичного шоку?
+ Зниження периферiйного опору судин
- Тахiкардiя
- Бiль
- Зменшення ударного об’єму серця
- Зменшення об’єму циркулюючої кровi


# У хворого дiагностовано септичний ендокардит. Температура тiла протягом 5-ти днiв коливалася в межах 39,5oC 40,2oC. На 6-й день на тлi рiзкого зниженнятемпературидо35,2oC розвинувся колапс. Який головний механiзм колапсу?
+ Вазодилатацiя
- Гiпервентиляцiя
- Посилене потовидiлення
- Тахiкардiя
- Полiурiя


#В результатi травми пошкоджений спинний мозок (з повним розривом) на рiвнi першого шийного хребця. Що вiдбудеться з диханням?
+ Припиняється
- Не змiнюється
- Зростає частота
- Зростає глибина
- Зменшується частота


#Чоловiк 53-х рокiв доставлений у стацiонар у непритомному станi. Об’єктивно: шкiра суха, дихання часте поверхневе, запах ацетону вiдсутнiй, Ps- 126/хв., АТ- 70/40 мм рт.ст. Вмiст глюкози у кровi 48 ммоль/л, реакцiя сечi на ацетон негативна. Для якого iз перелiчених станiв найбiльш характернi симптоми у хворого?
+ Гiперосмолярна кома
- Гiперкетонемiчна кома
- Лактацидемiчна кома
- Токсична кома
- Колапс


#У реанімаційному відділенні знаходиться хворий у коматозному стані. При дослідженні крові відзначено збільшення концентрації іонів K
+ і зменшення - Ca++, ацидоз, збільшення рівнів сечовини, сечової кислоти. Який вид коми за етіологією найбільш імовірний?
+Ниркова
- Нейрогенна
- Печінкова
- Гіпоглікемічна
- Діабетична


#У хворого 23-х років в результаті черепно-мозкової травми виник набряк мозку. Який механізм пошкодження клітин безпосередньо призвів до набряку мозку?
+Електролітно-осмотичний
- Ліпідний
- Кальцієвий
- Ацидотичний
- Протеїновий


#Внаслідок дії на організм електричного струму міської електромережі впродовж 0,1 сек у напрямку "права рука-голова" у постраждалого спостерігалась зупинка дихання. Вкажіть найбільш імовірний механізм цього ускладнення:
+ Тотальний параліч дихального центру
- Параліч центрів вдиху
- Емоційний стрес
- Рефлекторна зупинка дихання (больовий шок)
- Параліч дихальних м'язів


#Хворого доставлено у медичний заклад в коматозному стані. Зі слів супроводжуючих вдалося з'ясувати, що він знепритомнів під час тренування на завершальному етапі марафонської дистанції. Який вид коми найімовірніше можна запідозрити у даного пацієнта?
+Гіпоглікемічна
- Гіпотиреоїдна
- Печінкова
- Гіперглікемічна
- Ацидотична


#У хворого діагностовано грип. Після прийому антипіретиків стан його різко по­гіршився: свідомість потьмарена, АТ- 80/50 мм рт.ст., РS- 140/хв., температура тіла різ­ко знизилась до 35,8°С. Яке ускладнення виникло у даного хворого?
+Колапс
-Ацидоз
-Алкалоз
-Гіпертермія
-Гіповолемія


#У підлітка 12-ти років, який впродовж 3-х місяців різко схуд, вміст глюкози у кро­ві 50 ммоль/л. У нього розвинулася кома. Який головний механізм її розвитку?
+Гіперосмолярний
-Кетонемічний
-Гіпоксичний
-Гіпоглікемічний
-Лактацидемічний


#Внаслідок землетрусу чоловік 50-ти років дві доби перебував під завалом. Пі­сля звільнення з-під завалу рятівниками у нього був встановлений синдром тривало­го розчавлення. Виникнення якого ускла­днення в подальшому найбільш вірогідне?
+Гостра ниркова недостатність
-Гостра серцева недостатність
-Гостра дихальна недостатність
-Гостра печінкова недостатність
-Гостра судинна недостатність


#У хворого внаслідок травми розвинувся травматичний шок, у перебігу якого мали місце наступні порушення: АТ- 140/90 мм рт.ст., Ps- 120/хв. Хворий метушливий, багатослівний, блідий. Якій стадії шоку відповідає цей стан?
+ Еректильна
- Латентний період
- Кінцева
- Термінальна
- Торпідна


#У водія, який потрапив у ДТП, отримав травму та знаходиться у стані шоку, спостерігається зменшення добової кількості сечі до 300 мл. Який основний патогенетичний фактор цієї зміни діурезу?
+ Падіння артеріального тиску
- Підвищення проникності судин
- Вторинний гіперальдостеронізм
- Зменшення кількості функціонуючих клубочків
- Зниження онкотичного тиску крові


#Хвора доставлена бригадою швидкої допомоги. Об’єктивно: стан важкий, свiдомiсть вiдсутня, адинамiя. Шкiрнi покриви сухi, запалi очi, цiаноз обличчя, тахiкардiя, запах ацетону з рота. Результати аналiзiв: глюкоза кровi - 20,1 ммоль/л (у нормi - 3,3-5,5 ммоль/л), у сечi - 3,5% (у нормi - 0). Який найбiльш вiрогiдний дiагноз?
+Гiперглiкемiчна кома
-Гiпоглiкемiчна кома
-Гостра серцева недостатнiсть
-Гостре алкогольне отруєння
-Анафiлактичний шок


#До клiнiки поступив чоловiк 40-ка рокiв, якого укусила гадюка. Де переважно буде проходити гемолiз еритроцитiв у цьому випадку?
+У кровоносному руслi
-У клiтинах печiнки
-У клiтинах селезiнки
-У кiстковому мозку
-У паренхiмi нирок


#Хворого на трансмуральний iнфаркт мiокарда лiвого шлуночка переведено до вiддiлення реанiмацiї у важкому станi. АТ-70/50 мм рт.ст., ЧСС- 56/хв., ЧД- 32/хв. Зазначте головну ланку в патогенезi кардiогенного шоку:
+Падiння серцевого викиду
-Падiння периферичного судинного опору
-Втрата води
-Крововтрата
-Втрата електролiтiв


#Жінка 22 років, поступила у стаціонар в коматозном стані. Об'єктивно: шкіра суха, бліда, запах ацетону з рота, дихання Куссмауля, пульс 130 уд/хв., АТ- 95/60 мм.рт.ст., вміст глюкози у крові - 37.7 ммоль/л. Для якого із перелічених станів найбільш характерні симптоми, що спостерігаються?
+гіперкетонемічної коми
-гіперосмолярної коми
-лактацидемічної коми
-тривалого голодування
-алкогольної інтоксикації


#Хворого доставили в клініку в коматозному стані. Дихання шумне, глибоке. Чути запах ацетону. Вміст глюкози в крові 15,2 ммоль/л, кетонові тіла - 100 мкмоль/л. Для якого виду коми характерні такі розлади?
+Кетоацидотичної
-Печінкової
-Гіперглікемічної
-Гіпоглікемічної
-Гіперосмолярної


#У хворого після важкої травми грудної клитки розвинувся шок та з’явилися ознаки гострої ниркової недостатності [ГНН]. Що є провідним механізмом розвитку ГНН в даному випадку?
+ Падіння артеріального тиску
-Порушення відтоку сечі
-Підвищення тиску в капсулі клубочка
-Підвищення тиску в ниркових артеріях
-Зменшення онкотичного тиску крові


#В реанімаційне відділення доставили водія, який попав в автомобільну аварію. Він не зразу реагує на питання, байдужий до всього, блідий, дихання поверхневе, рідке, АТ дорівнює 75/50 мм рт. Ст. Назвіть головну ланку патогенезу вказаної патології.
+Гальмування центральної нервової системи.
-Збудження ЦНС.
-Крововтрата.
-Токсемія.
-Перерозподіл крові.


#Хворий Ж., 48 років, знаходиться в реанімації після отруєнням чотирьоххлористим вуглеводнем. Стан важкий, хворий без свідомості, пульс ниткоподібний, слабкого наповнення, ЧСС 40 уд / хв АТ 75/40 мм.рт.ст., дихання періодичне типу Біота. В крові прямий білірубін - 155 мкмоль / л. У сечі високий вміст аміаку і білірубіну. Що стало основною причиною описаного стану у хворого?
+ Печінкова недостатність,
-Кардіогенний шок
-Дихальна недостатність
-Серцева недостатність
-Ниркова недостатність


#У хворого в третьому періоді лихоманки настав критичний падіння температури тіла. При цьому спостерігалася тахікардія і зниження артеріального тиску до 80/60 мм рт. ст. Який вид колапсу розвинувся при цьому?
+ Інфекційно-токсичний
-Ортостатичний
-Геморагічний
-Кардіогенний
-Панкреатичний


#У чоловіка 35 років через 30 хвилин після автомобільної аварії виявлена масивна травма нижніх кінцівок без значної зовнішньої крововтрати. Постраждалий знаходиться у збудженному стані . Який компонент патогенезу травматичного шоку є у пацієнта ведучим і потребує негайного корегування?
+Біль.
-Внутрішня крововтрата.
-Внутрішня плазмовтрата.
-Інтоксікація.
-Порушення функції органів


#Введення знеболюючого пацієнту перед екстракцією зуба призвело до розвитку анафілактичного шоку, який супроводжувався розвитком олігурії. Який патогенетичний механізм зумовив зменшення діурезу в даній клінічній ситуації?
+Зниження гідростатичного тиску в капілярах клубочків
-Збільшення онкотичного тиску крові
-Зменшення кількості функціонуючих нефронів
-Підвищення гідростатичного тиску в капсулі Шумлянського-Боумена
-Пошкодження клубочкового фільтру


#Після операції на кишечнику у хворого з’явились симптоми отруєння аміаком за типом печінкової коми. Який механізм дії аміаку на енергозабезпечення ЦНС?
+Гальмування ЦТК в результаті зв’язування альфа-кетоглутарату
-Інактивація ферментів дихального ланцюга
-Гальмування бета-окиснення жирних кислот
-Гальмування гліколізу
-Роз’єднування окисного фосфорилю-вання


#Під час автомобільної аварії людина отримала сильний удар в епігастральну ділянку, внаслідок чого виникла зупинка серця. Що могло стати причиною таких змін серцевої діяльності?
+Підвищення тонусу блукаючого нерва
-Збільшене виділення адреналіну
-Збільшене виділення альдостерону
-Збільшене виділення кортизолу
-Підвищення тонусу симпатичної нервової системи


#У хворого на гострий мiокардит з’явилися клiнiчнi ознаки кардiогенного шоку. Який iз вказаних нижче патогенетичних механiзмiв є провiдним в розвитку шоку?
+Зниження насосної функцiї серця
-Депонування кровi в органах
-Зниження дiастолiчного притоку до серця
-Зниження судинного тонусу
-Збiльшення периферичного опору судин


#Хворий 50-ти років страждає на гіпертонічну хворобу. Під час фізичного навантаження у нього з’явилося відчут- тя слабкості, нестачі повітря, синюшність слизової оболонки губ, шкіри обличчя. Дихання супроводжувалося чутними на відстані вологими хрипами. Який механізм лежить в основі виникнення такого синдрому?
+Гостра лівошлуночкова недостатність
-Колапс
-Тампонада серця
-Хронічна лівошлуночкова недостатність
-Хронічна правошлуночкова недостатність


#Чоловіку 44 років з гострою пневмонією призначили пеніцилін внутрішньом’язово. Після проведення ін’єкції стан хворого різко погіршився: з’явилась задишка, хворий покрився холодним потом. Пульс 140/хв. слабкого наповнення. АТ 90/40 мм.рт.ст. Яке ускладнення найбільш ймовірно виникло у хворого?
+Анафілактичний шок.
-Тромбоемболія легеневої артерії.
-Кардіогенний шок
-Iнфекційно-токсичний шок.


#Жінка 29-ти років поступила у полого­ве відділення в терміні вагітності 40 тижнів. Шийка матки розкрита, але скорочення матки відсутнє. Лікар дав засіб гормональ­ної природи для посилення пологової діяль­ності. Назвіть засіб:
+Окситоцин
-Тестостерон
-Гідрокортизон
-Естрон
-АКТГ


#У хворої внаслідок запалення порушена ендокринна функція фолікулярних клітин фолікулів яєчника. Синтез яких гормонів буде пригнічений?
+ Естрогени
- Фолістатин
- Лютропін
- Прогестерон
- Фолікулостимулюючий гормон


#Секреція яких гормонів гіпофізу галь­мується після прийому оральних контра­цептивів, які містять статеві гормони?
+Гонадотропні
-Соматотропний
-Тиреотропні
-Окситоцин
-Вазопресин


#Чоловіку 46-ти років, що хворіє на дифузний токсичний зоб, була проведе­на операція резекції щитоподібної залози. Після операції відмічаються відсутність апетиту, диспепсія, підвищена нервово- м’язова збудливість. Маса тіла не збіль­шилася. Температура тіла у нормі. Чим, із нижче переліченого, обумовлений стан хворого?
+Зниженням продукції паратгормону
-Підвищенням продукції тиреоліберину
-Зниженням продукції тироксину
-Підвищенням продукції тироксину
-Підвищенням продукції кальцитоніну


#У дитини 2-х рокiв виникли судоми внаслiдок зниження концентрацiї iонiв кальцiю в плазмi кровi. Функцiя якого ендокринного органу знижена?
+Прищитоподiбнi залози
-Гiпофiз
-Кора наднирникiв
-Шишкоподiбна залоза
-Тимус


#Пiсля перенесеного сепсису у хворої 27-ми рокiв з’явився бронзовий колiр шкiри, характерний для аддiсонової хвороби. Механiзм гiперпiгментацiї полягає в пiдвищеннi секрецiї такого гормону:
+Меланоцитстимулюючий
-Соматотропний
-Гонадотропний


B-лiпотропний
-Тиреотропний


#Хворому 45-ти років при оперативному втручанні на щитовидній залозі випадково видалили прищитовидні залози. Це призвело до:
+ Тетанії
-Підвищення рівня кальцію в крові і резорбції кісток
-Підвищення рівня кальцію, натрію і калію в крові
-Зниження артеріального тиску
-Підвищення артеріального тиску


#У мишей з відсутнім волосяним покривом (тобто nude - голі) не було клітинних реакцій уповільненого типу. Для цієї патології найбільш вірогідним є:
+Відсутність вилочкової залози
-Відсутність гамаглобулінів у крові
-Порушення гемопоезу
-Дефект фагоцитозу
-Дефіцит компонентів системи комплементу


#Внаслідок травмування у хворого видалили прищитовидні залози, що супроводжувалося: млявістю, спрагою, різким підвищенням нервово-м’язової збудливості. З порушенням обміну якої речовини це пов’язано:
+Кальцію
-Марганцю
-Хлору
-Молібдену
-Цинку


# Стресовий стан i больове вiдчуття у пацiєнта перед вiзитом до стоматолога супроводжуються анурiєю (вiдсутнiстю сечовидiлення). Це явище зумовлене збiльшенням:
+ Секрецiї вазопресину та адреналiну
- Активностi парасимпатичної нервової системи
- Активностi антиноцiцептивної системи
- Секрецiї вазопресину та зменшенням адреналiну
- Секрецiї адреналiну та зменшенням вазопресину


#З метою оцінки адаптації до фізично­го навантаження лікар провів обстеження робітників після виконання важкої праці. Які зміни в загальному аналізі крові можна виявити?
+Перерозподільчий лейкоцитоз
-Анемія
-Гіпоальбумінемія
-Зсув лейкоцитарної формули вліво
-Лейкопенія


#Недбалий студент раптово зустрiвся з деканом. Концентрацiя якого гормону найшвидше збiльшиться в кровi студента?
+ Адреналiн
- Тиреолiберин
- Кортикотропiн
- Кортизол
-Соматотропiн


#Пiд час футбольного матчу мiж вболiвальниками виникла сутичка. На фонi негативних емоцiй в одного учасника сутички були розширенi зiницi й пiдвищене серцебиття. Активацiя якої системи органiзму забезпечує такi вегетативнi змiни при негативних емоцiях?
+Симпато-адреналова
-Гiпоталамо-гiпофiзарно-тиреоїдна
-Соматична нервова
-Парасимпатична нервова
-Метасимпатична нервова


#Чоловік 25 років, потупив у лікарню через одну годину після автомобільної катастрофи. Картина гострого стресу без розповсюджених пошкоджень. Яка із перелічених змін формених елементів крові буде найбільш характерною для цього стану?
+еозинопенія
-нейтропенія
-лейкопенія
-еритропенія
-моноцитопенія


#Жiнка 30-ти рокiв народила хлопчика з розщепленням верхньої губи ("заяча губа"i "вовча паща"). При додатковому обстеженнi виявленi значнi порушення нервової, серцево-судинної систем та зору. При дослiдженнi карiотипу дiагностована трисомiя за 13-ю хромосомою. Який синдром наявнийу хлопчика?
+ Патау
- Шерешевського-Тернера
- Едвардса
- Дауна
- Клайнфельтера


#До медико-генетичної консультацiї звернулася жiнка 30-ти рокiв у якої в ядрах бiльшостi клiтин епiтелiю слизової оболонки щоки було виявлено по два тiльця Барра. Який попереднiй дiагноз можна встановити?
+ Трисомiя за X-хромосомою
- Трисомiя за 21-ю хромосомою
- Трисомiя за 13-ю хромосомою
- Трисомiя за 18-ю хромосомою
- Моносомiя за X-хромосомою


#У подружжя народився син, хворий на гемофiлiю. Батьки здоровi, а дiдусь за материнською лінією також хворий на гемофiлiю. Визначте тип успадкування ознаки.
+ Рецесивний, зчеплений зi статтю
- Аутосомно-рецесивний
- Домiнантний, зчеплений зi статтю
- Неповне домiнування
- Аутосомно-домiнантний


#Для дiагностування деяких хромосомних хвороб використовують визначення статевого хроматину. Назвiть хворобу, при якiй потрiбне це визначення:
+ Синдром Шерешевського-Тернера
- Хвороба Дауна
- Гемофiлiя
- Трисомiя Х
- Хвороба Брутона


#При проведеннi амнiоцентезу в клiтинах плоду виявлено по 2 тiльця статевого хроматину (тiльця Барра). Для якого захворювання характерна дана ознака?
+ Трисомiя X
- Синдром Клайнфельтера
- Синдром Шерешевського-Тернера
- Синдром Дауна
- Синдром Патау


#Виникнення нижчеперерахованих за¬хворювань пов’язане із генетичними факторами. Назвіть патологію із спадковою схильністю:
+Цукровий діабет
-Серпоподібноклітинна анемія
-Фенілкетонурія
-Хорея Гентінгтона
-Дальтонізм44


#Юнак 18-ти років звернувся до медико- генетичної консультації з приводу відхи­лень у фізичному і статевому розвитку. При мікроскопії клітин слизової оболонки рота виявлене одне тільце Барра. Вкажіть найбільш імовірний каріотип юнака:
+47, XXY
- 45, ХО
- 47, XYY
-47,18+
-47, 21+


#У чоловіка 22-х років високого росту та астенічної будови тіла з ознаками гіпогонадизму, гінекомастією та зменшеною продукцією сперми (азооспермія) виявле­но каріотип 47 ХХУ. Який спадковий син­дром супроводжується такою хромосом­ною аномалією?
+Клайнфельтера
-Віскотта-Олдрича
-Дауна
-Луї-Барра
-Тернера


#Жінка 27-ми років звернулася зі скар­гами на дисменорею та безпліддя. При об­стеженні виявлено: зріст жінки 145 см, не­дорозвинені вторинні статеві ознаки, на шиї крилоподібні складки. При цитологі­чному дослідженні в соматичних клітинах не виявлено тілець Барра. Який діагноз встановив лікар?
+Синдром Шерешевського-Тернера
-Синдром Клайнфельтера
-Синдром трисомїї X
-Синдром Морріса


#У новонародженої дитини спостерігаються: судоми, блювання, жовтяниця, специфічний запах сечі. Лікар-генетик висловив підозру про спадкову хворобу обміну речовин. Який метод дослідження необхідно використати для постановки точного діагнозу?
+Біохімічний
- Дерматогліфіка
- Цитогенетичний
- Близнюковий
- Популяційно-статистичний


#В родині зростає дочка 14-ти років, у якої спостерігаються деякі відхилення від норми: зріст нижче, ніж у однолітків, відсутні ознаки статевого дозрівання, шия дуже коротка, плечі широкі. Інтелект в нормі. Яке захворювання можна припустити?
+Синдром Шерешевського-Тернера
- Синдром Дауна
- Синдром Едвардса
- Синдром Клайнфельтера
- Синдром Патау


#Чоловік звернувся до лікаря з приводу безпліддя. Має високий зріст, зниження інтелекту, недорозвинення статевих залоз. У епітелії слизової оболонки порожнини рота виявлений статевий хроматин (1 тільце Барра). Про яку патологію можна думати?
+ Синдром Клайнфельтера
- Акромегалія
- Адреногенітальний синдром
- Синдром Ді Джорджи
- Синдром Іценка-Кушинга


#У здорових батьків, спадковість яких не обтяжена, народилась дитина з чисельними вадами розвитку. Цитогенетичний аналіз виявив в соматичних клітинах дитини трисомію за 13-ю хромосомою (синдром Патау). З яким явищем пов'язане народження такої дитини?
+ Порушення гаметогенезу
- Хромосомна мутація
- Рецесивна мутація
- Домінантна мутація
- Соматична мутація


#До гінеколога звернулася жінка 28-ми років з приводу безпліддя. При обстежен­ні знайдено: недорозвинені яєчники та ма­тка, нерегулярний менструальний цикл. При дослідженні статевого хроматину в більшості соматичних клітин виявлено два тільця Бара. Яка хромосомна хвороба най­більш вірогідна у жінки?
+Синдром трипло-Х
-Синдром Едвардса
-Синдром Патау
-Синдром Клайнфельтера
- Синдром Шерешевського-Тернера


#У новонародженого хлопчика спосте­рігається деформація мозкового та лицьо­вого черепа, мікрофтальмія, деформація вушної раковини, вовча паща, і т.ін. Каріо­тип дитини - 47, ХУ, 13+. Про яку хворобу йде мова?
+Синдром Патау
-Синдром Шерешевського-Тернера
-Синдром Едвардса
-Синдром Дауна
-Синдром Клайнфельтера


#До генетичної консультації звернулася сімейна пара, в якій чоловік хворіє на інсулінозалежний цукровий діабет, а жінка здорова. Яка вірогідність появи інсулінозалежного діабету у дитини цього подружжя?
+Більше, ніж в популяції
- 100%
- 50%
- Нижче, ніж в популяції
- Така сама, як в популяції


#При диспансерному обстеженні хлопчику 7-ми років встановлено діагноз - дальтонізм. Батьки здорові, кольоровий зір у них у нормі, але у дідуся по материнській лінії така ж аномалія. Який тип успадкування цієї аномалії?
+ Рецесивний, зчеплений зі статтю
- Домінантний, зчеплений зі статтю
-Аутосомно-рецесивний
- Аутосомно-домінантний
- Неповне домінування


#При медичному огляді у військкоматі був виявлений хлопчик 15-ти років, високого зросту, з євнухоїдними пропорціями тіла, гінекомастією, з ростом волосся на лобку за жіночим типом. Відмічається відкладання жиру на стегнах, відсутність росту волосся на обличчі, високий голос; коефіцієнт інтелекту знижений. Виберіть каріотип, що відповідає даному


захворюванню:
+ 47, XXY
- 47, XXX
- 45, XO
- 46, XY
- 46, XX


#Внаслiдок впливу альфа-випромiнювання дiлянка ланцюга ДНК повернулася на 180 градусiв. Яка з перелiчених видiв мутацiй вiдбулася в ланцюзi ДНК?
+Iнверсiя
-Делецiя
-Дуплiкацiя
-Транслокацiя
-Реплiкацiя


#У клiнiчно здорових батькiв народилася дитина, хвора на фенiлкетонурiю (аутосомно-рецесивне спадкове захворювання). Якi генотипи батькiв?
+Аа х Аа
-АА х АА
-АА х Аа
-Аа х аа
-аа х аа


#В медико-генетичнiй консультацiї при обстеженнi хворого хлопчика в кровi були виявленi нейтрофiльнi лейкоцити з однiєю "барабанною паличкою". Наявнiсть якого синдрому можна запiдозрити у хлопчика?
+Синдром Клайнфельтера
-Синдром Дауна
-Синдром Шерешевського-Тернера
-Синдром Едвардса
-Синдром трисомiї-Х


#У жiнки пiд час гаметогенезу (в мейозi) статевi хромосоми не розiйшлися до протилежних полюсiв клiтини. Яйцеклiтина була заплiднена нормальним сперматозоїдом. Яке хромосомне захворювання може бути у дитини?
+ Синдром Шерешевського-Тернера
- Синдром Дауна
- Синдром Патау
- Синдром Едвардса
-Синдром котячого крику


#При обстеженнi 2-мiсячного хлопчика педiатр звернув увагу, що плач дитини схожий на нявкання кiшки, вiдзначаються мiкроцефалiя i вада серця. За допомогою цитогенетичного методу був встановлений карiотип - 46 XY, 5р-. На якiй стадiї мiтозу дослiджували карiотип хворого?
+Метафаза
-Прометафаза
-Профаза
-Анафаза
-Телофаза


#Чоловiк, що страждає на спадкову хворобу, одружився iз здоровою жiнкою. У них було 5 дiтей, три дiвчинки i два хлопчика. Усi дiвчатка успадкували хворобу батька. Який тип спадкування цього захворювання?
+Домiнантний, зчеплений з X - хромосомою
-Аутосомно-рецесивний
-Аутосомно-домiнантний
-Зчеплений з Y -хромосомою
-Рецесивний, зчеплений з X - хромосомою


#При вивченнi родоводу сiм’ї, в якiй спостерiгається гiпертрихоз (надмiрне оволосiння вушних раковин), виявлена ознака трапляється в усiх поколiннях тiльки у чоловiкiв i успадковується вiд батька до сина. Визначте тип успадкування гiпертрихозу:
+Зчеплений з Y-хромосомою
-Аутосомно-рецесивний
-Аутосомно-домiнантний
-Зчеплений з Х-хромосомою рецесив-ний
-Зчеплений з Х-хромосомою домiнан-тний


#У хлопчика зi спадково обумовленими вадами зразу ж пiсля народжен-ня спостерiгався характерний синдром, який називають "крик кiшки". У ран-ньому дитинствi малюк мав "нявкаючий" тембр голосу. Пiд час дослiдження карiотипу цiєї дитини було виявлено:
+Делецiю короткого плеча 5-ї хромосоми
-Додаткову 21-у хромосому
-Додаткову Х-хромосому
-Нестачу Х-хромосоми
-Додаткову Y-хромосому


#У новонародженої дитини виявлено наступну патологiю: аномалiя розвитку нижньої щелепи та гортанi, що супроводжується характерними змiнами голосу, а також мiкроцефалiя, вада серця, чотирьохпалiсть. Найбiльш iмовiрною причиною таких аномалiй є делецiя:
+Короткого плеча 5-ої хромосоми
-Короткого плеча 7-ої хромосоми
-Короткого плеча 9-ої хромосоми
-Короткого плеча 11-ої хромосоми
-21-ої хромосоми


#У фібробластах шкіри дитини із хворобою Дауна виявлено 47 хромосом. Визначте тип аномалії:
+Трисомія 21
-Полісомія Y
-Трисомія 13
-Трисомія 18
-Трисомія Х


#У юнака 18-ти років діагностовано хворобу Марфана. При дослідженні встановлено: порушення розвитку сполучної тканини, будови кришталика ока, аномалії серцево-судинної системи, арахнодактилія. Яке генетичне явище зумовило розвиток цієї хвороби?
+Плейотропія
-Кодомінування
-Комплементарність
-Множинний алелiзм
-Неповне домiнування


#A victim of a traffic accident was received by the intensive care unit. The patient is in a grave condition that can be characterized as a severe pathologic process that leads to exhaustion of vital functions and puts the patients into the marginal state between life and death due to critical reduction of capillary circulation in the affected organs. The patient is in the state of:
+Agony
-Pre-agony
-Shock
-Coma
-Collapse


#Insufficiency of the mitral valve was founded at man who 12 years ago suffered from rheumatic myocarditis. Investigations have shown that the inflammatory process is absent, minute volume of circulation is sufficient. Name the notion of general nosology.
+Pathological state
-Pathological reaction
-Disease process
-Typical pathological process
-Compensating reaction


#Atrophy of bone marked in the patient after the removal of teeth. This is an example of:
+Pathological condition
-Pathological reaction
-Pathological process
-Structural adaptation
-Disease


#A 50-year –old man has a stomach ulcer. After the treatment digestion become to normal, pain disappeared, and his mood improved. However, in a couple of weeks the patient developed pain in epigastric region, heartburn and belching. How this situation might be interpreted?
+Disease relapse
-Remission period
-Terminal status
-Premonitory period
-Latent period


#A 12-year-old boy after coming home from the school began to complaint of headache, nausea, chill, periodic muscle pain, loss of appetite, and fatigue. What disease period can show such signs?
+Prodromal


-Latent


-Incubation


-Height of the disease


-End of the disease


#A student fell ill with acute respiratory disease at the end of winter following long period of mental overloading. What is the cause of the disease?
+Pathogenic microorganism
-Irrational diet
-Mental overloading
-Overcooling
-Hypovitaminosis


#A 39-years-old patient has been suffering from gastric ulcer for last 4 years. Pain in epigastric region, heartburn, nausea, and constipation appear mainly in autumn and spring. Name this condition.
+Relapse
-Remission
-Complication
-Pathologic condition
-Relapse


#Gasping respiration appears in a patient with severe lung pathology. What terminal condition is this characteristic for?
+Agony
-Pre-agony
-Clinical death
-Biological death
-Terminal pause


#A 10-yeaes-old child endured several rheumatic attacks. At examination of him it was established that he had inflammatory process in his joints and signs of mitral valve insufficiency. What pathological appearance in this patient may be attributed?


to "disease"?
+Rheumatism
-Mitral valve disease
-Mitral valve insufficiency
-Inflammation of joints
-Arthritis


#A 28-year-old patient with symptoms of acute appendicitis was admitted to the surgical department. Acute pains in his right epigastric area and near the umbilicus were registered during palpation on examination. Schetkin-Blumberg symptom was positive. Which period of disease was observed in that patient?
+Period of manifestation
-Latent period
-Prodromal period
-Final period
-Period of functional disorder


#Increase in pulse rate, respiratory rate, and Increase in BP were noticed on the height of 1000 m above the sea in a 25-year-old woman, coming for holyday in the mountainous resort. In some time, all those symptoms disappeared. Which process was noticed in a woman?
+Adaptation
-Compensation
-Decompensation
-Stress
-Parabiosis


#A man with the complex of deviation of his health is considered sick. What is the more typically for disease?
+Disorder of physiological regulation of functions
-Decrease in ability to work
-Disorder of immunity
-Psychological deviations
-Decrease in adaptation


#A 25-year-old man has spent a long time in the sun under high air humidity. As a result of it his body temperature rose up to 39oC. What pathological process is it?
+Hyperthermia
-Infectious fever
-Hypothermia
-Noninfectious fever
-Burn disease


#The temperature of the ambient environment is 38oC and relative air humidity is 50%. What ways of heat emission provide maintaining a constant temperature of the human body?
+Evaporation
-Radiation
-Heat conduction
-Convection
-Convection and conduction


#At the end of the working day a worker of a hot work shop has been delivered to a hospital. The patient complains of headache, dizziness, nausea, general weakness. Objectively: the patient is conscious, his skin is hyperemic, dry, hot to the touch. Heart rate is of 130/min. Respiration is rapid, superficial. What disorder of thermoregulation is most likely to have occurred in this patient?
+Reduced heat transfer
-Increased heat transfer and reduced heat production
-Increased heat transfer and heat production
-Increased heat production with no changes to the heat transfer
-Reduced heat production


#A patient with heatstroke was delivered to the admission room. What compensatory reactions develop in the patient’s body in such case?
+Peripheral vasodilatation
-Coronary vasospasm
-Increased heart rate
-Persistent hyperglycemia
-Peripheral vasoconstriction


#Acute general disorders are observed at long-term action of environmental high temperature on the organism. What is the base of this phenomenon?
+Dehydration
-Cardiac insufficiency
-Increase in body temperature
-Intoxication
-Hypoxia


#A patient due to long-term work in the apartment in the condition of hot microclimate the body temperature rose considerably. The state became worse till evening: temperature of body 40°C, there was stupor, pallor of the skin, bleeding from the nose. Arterial pressure was 80/50 mmHg. What extremal state is observed in a patient?
+Thermal shock
-Hemorrhagic shock
-Hyperthermal coma
-Cardiogenic shock
-Hyperthermal collapse


#Having helped to eliminate consequences of a failure at a nuclear power plant, a worker got an irradiation doze of 500 roentgens. He complains of headache, nausea, dizziness. What changes in leukocytes quantity can be expected 10 hours after irradiation?
+Neutrophilic leukocytosis
-Lymphocytosis
-Leukopenia
-Agranulocytosis
-Leukemia


#A woman who was infected with toxoplasmosis during the pregnancy has a child with multiple congenital defects. This is a result of:
+Teratogenesis
-Cancerogenesis
-Biological mutogenesis
-Chemical mutogenesis
-Recombination


#Medical examination at the military registration and enlistment office revealed that a 15-year-old boy was high, with eunuchoid body proportions gynecomastia, female pattern of pubic hair distribution. The boy had also fat deposits on the thighs, no facial hair, high voice, subnormal intelligence quotient. Which karyotype corresponds with this disease?
+47, XXY
-45, XO
-46, XX
-46, XY
-47, XXX


#One of the parents is suspected of having phenylketonuria recessive gene. What is the risk of giving birth to a child with inborn phenylketonuria?
+0%
-25%
-50%
-75%
-100%


#A 28-year-old female patient consulted a gynecologist about sterility. Examination revealed underdeveloped ovaries and uterus, irregular menstrual cycle. Analysis of the sex chromatin revealed 2 Barr’s bodies in most somatic cells. What chromosome disease is most likely?
+Triple X syndrome
-Edwards syndrome
-Patau syndrome
-Klinefelter syndrome
-Turner syndrome


#Sex chromosomes of a woman didn’t separate and move to the opposite poles of a cell during gametogenesis (meiosis). The ovum was impregnated with a normal spermatozoon. Which chromosomal disease can be found in her child?
+Turner syndrome
-Down syndrome
-Patau syndrome
-Edward syndrome
-Cat cry syndrome


#A couple has a son with hemophilia. The parents are healthy but the maternal grandfather also has hemophilia. Specify the type of inheritance:
+Recessive sex-linked
-Recessive autosomal
-Dominant sex-linked
-Semidominant
-Autosomal dominant


#During a prophylactic medical examination, a 7-year-old boy was diagnosed with daltonism. His parents are healthy and have normal color vision, but his grandfather on his mother’s side has the same abnormality. What is the type of the abnormality inheritance?
+Recessive, sex-linked
-Dominant, sex-linked
-Semidominance
-Autosomal recessive
-Autosomal dominant


#According to the phenotypic diagnosis a female patient has been provisionally diagnosed with X-chromosome polysomia. This diagnosis can be confirmed by a cytogenetic method. What karyotype will allow to confirm the diagnosis?
+47(ХХХ)
-48(XXXY)
-48(XXYY)
-47(XXY)
-46(XX)


#Examination of an 18-year-old girl revealed the following features: hypoplasia of the ovaries, broad shoulders, narrow pelvis, shortening of the lower extremities, "sphinx neck". Mental development is normal. The girl was diagnosed with Turner’s syndrome. What kind of chromosome abnormality is it?
+Monosomy X
-Trisomy X
-Trisomy 13
-Trisomy 18
-Nullisomy X


#A 35-year-old male patient has been referred by an andrologist for the genetic counselling for the deviations of physical and mental development. Objectively: the patient is tall, has asthenic constitution, gynecomastia, mental retardation. Microscopy of the oral mucosa cells revealed sex chromatin (single Barr body) in 30% of cells. What is the most likely diagnosis?
+Klinefelter syndrome
-DiGeorge syndrome
-Down syndrome
-Recklinghausen disease
-Cushing pituitary basophilism


#A 25-year-old patient consulted a doctor about dysmenorrhea and infertility. Examination revealed that the patient was 145 cm high and had underdeveloped secondary sex characteristics, alar folds on the neck. Cytological study didn’t reveal any Barr bodies in the somatic cells. What diagnosis was made?
+Turner syndrome
-Klinefelter syndrome
-Morris syndrome
-Trisomy X syndrome


#Amniocentesis revealed two sex chromatin bodies (Barr bodies) in each cell of the sample. What disease is this character typical for?
+Trisomy X
-Klinefelter syndrome
-Turner syndrome
-Down syndrome
-Patau syndrome


#A boy referred to a genetics clinic was found to have 1 drumstick in blood neutrophils. The boy is likely to have the following syndrome:
+Klinefelter
-Down
-Turner
-Edwards
-Trisomy X


#A female patient has medical-genetic consultation. Physical examination revealed pterygiumcolli deformity (webbed neck), broad chest, underdeveloped breasts. Study of buccal epithelium cells revealed no X-chromatin in the nuclei. This indicates that the patient has the following syndrome:
+Turner
-Klinefelter
-Patau
-Down
-Edward


#A 2-year-old boy is diagnosed with Down syndrome. What chromosomal changes may be the cause of this disease?
+Trisomy 21
-Trisomy 13
-Trisomy X
-Trisomy 18
-Monosomy X


#Sex chromatin was detected during examination of a man’s buccal epithelium. It is characteristic of the following chromosome disease:
+Klinefelter syndrome
-Down disease
-Turner syndrome
-Triple X syndrome
-Hypophosphatemic rickets


#A 45-year-old woman gave birth to a boy with cleft maxilla (cleft lip and palate). On additional examination there are significant disturbances of the boy’s nervous, cardiovascular and visual systems. Karyotype investigation allowed diagnosing the patient with trisomy 13. What syndrome is present in boy?
+Klinefelter
-Down
-Turner
-Di George
-Patau


#Clinical presentation of a woman allowed provisionally diagnosing her with X polysomy. Cytogenetic method is applied to clarify the diagnosis. The diagnosis will be confirmed if the patient’s karyotype is?
+47,ХХХ
-48,XXXY
-48,XXYY
-47,XXY
-46,XX


#A 40-year-old pregnant woman underwent amniocentesis. Examination determined the fetal karyotype to be 47, XY+21. What fetal pathology was detected?
+Turner syndrome
-Down syndrome
-Patau syndrome
-Klinefelter syndrome
-Phenylketonuria


#A 20-year-old young man with tall stature, asthenic body type, signs of hypogonadism and gynecomastia, and low sperm count (azoospermia) has karyotype 47XXY. What hereditary syndrome can be characterized by this chromosomal anomaly?
+Turner syndrome
-Down syndrome
-Wiskott-Aldrich syndrome
-Klinefelter syndrome
-Louis-Bar syndrome


#A child with a history of frequent tonsillitis and pharyngitis has been diagnosed with lymphadenopathy and splenomegaly. His appearance is characterized by pastosity and paleness, muscular tissue is poorly developed. Lymphocytosis is present. What kind of diathesis is it?
+Lymphohypoplastic diathesis
-Exudative diathesis
-Gouty diathesis
-Asthenic diathesis
-Hemorrhagic diathesis


#A child is pale, pastose, muscular tissue is bad developed, lymph nodes are enlarged. He often suffers from tonsillitis and pharyngitis; blood has signs of lymphocytosis. The child is also predisposed to autoallergy diseases. What type of diathesis can be presumed in this case?
+Lymphohypoplastic
-Exudative
-Gouty
-Asthenic
-Hemorrhagic


#On examination the patient was determined to have strong, balanced, inert type of higher nervous activity according to Pavlov’s classification. What temperament according to Hippocrates is it?
+Choleric
-Sanguine
-Phlegmatic
-Melancholic


#After an immunoassay a child was diagnosed with immunodeficiency of humoral immunity. What is the reason for the primary immunodeficiency development in the child?
+Hereditary abnormality of immune system
-Embrylogical development abnormalities
-Pathometabolism in mother’s organism
-Immune responsiveness and resistance disorders
-Toxic damage of B-lymphocytes


#The immunoblot detected gp120 protein in the blood serum. This protein is typical for the following disease:
+HIV-infection
-Hepatitis B virus
-Tuberculosis
-Syphilis
-Poliomyelitis


#HIV has gp41 and gp120 on its surface interacting with target cells of an organism. Which of the following human lymphocyte antigens is gp120 complementary bound with?
+CD 4
-CD 3
-CD 8
-CD 19
-CD 28


#Examination of a child who frequently suffers from infectious diseases revealed that IgG concentration in blood serum was 10 times less than normal, IgA and IgM concentration was also significantly reduced. Analysis showed also lack of B-lymphocytes and plasmocytes. What disease are these symptoms typical for?
+Bruton’s disease
-Swiss-type agammaglobulinemia
-Dysimmunoglobulinemia
-Louis-Bar syndrome
-Di George syndrome


#A 13-year-old boy presents with eczematous rashes on his shins and torso. Anamnesis states cases of otitis, pneumonia and furuncles in the patient. Blood test: platelets- 70×109/l, low activity of T helper and T suppressor cells, low IgM, with normal IgA and IgG. What immunodeficiency disease does this boy have?
+Louis-Barr syndrome
-Severe combined immunodeficiency (Swiss type)
-Chediak-Higashi syndrome
-Wiskott-Aldrich syndrome
-Di George syndrome


#Parents of a 5-year-old child report him to have frequent colds that develop into pneumonias, presence of purulent rashes on the skin. Laboratory tests have revealed the following: absence of immunoglobulins of any type; naked cells are absent from the lymph nodes punctate. What kind of immune disorder is it?
+X-linked hypogammaglobulinemia (Bruton type agammaglobulinemia)
-Autosomal recessive agammaglobulinaemia (Swiss type)
-Hypoplastic anemia
-Agranulocytosis
-Louis-Barr syndrome


#A 10-year-old child had the tuberculin test administered. 48 hours later a papule up to 8 mm in diameter appeared on the site of the injection. What type of hypersensitivity reaction developed after the tuberculin injection?
+Type IV hypersensitivity reaction
-Arthus phenomenon
-Seroreaction
-Atopic reaction
-Type II hypersensitivity reaction


#During surgical manipulations a patient has been given Novocain injection for anesthesia. 10 minutes later the patient developed paleness, dyspnea, hypotension. What type of allergic reaction is it?
+Anaphylactic immune reaction
-Cytotoxic reaction
-Aggregate immune reaction
-Stimulating immune reaction
-Cell-mediated immune reaction


#A man with a long-term history of bronchial asthma died from asphyxia. Histological examination of his lungs revealed that the lumens of bronchioles and minor bronchi contained a lot of mucus with some eosinophils. There was also sclerosis of intra-alveolar septa, dilatation of alveolar lumens. What mechanism accounts for the development of hypersensitivity reaction?
+Anaphylactic immune reaction
-Cytotoxic reaction
-Immune complex reaction
-Lymphocyte-mediated cytolysis
-Granulomatosis


#A patient has been diagnosed with acute glomerulonephritis that developed after he had had streptococcal infection. It is most likely that the affection of basal glomerular membrane is caused by an allergic reaction of the following type:
+Immune complex
-Anaphylactic
-Cytotoxic
-Delayed
-Stimulating


#A child with suspected tuberculosis was given Mantoux test. After 24 hours the site of the allergen injection got swollen, hyperemic and painful. What are the main components that determine such response of the body?
+Mononuclear cells, T-lymphocytes and lymphokines
-Granulocytes, T-lymphocytes and IgG
-Plasma cells, T-lymphocytes and lymphokines
-B-lymphocytes, IgM
-Macrophages, B-lymphocytes and monocytes


#A child entering the school for the first time was given Mantoux test in order to determine if there was a need for revaccination. The reaction was negative. What is the meaning of this test result?
+No cell-mediated immunity to tuberculosis
-Availability of cell-mediated immunity to tuberculosis
-No antibodies to the tuberculosis bacteria
-No anti-toxic immunity to tuberculosis
-Presence of antibodies to the tuberculosis bacteria


#A child cut his leg with a piece of glass while playing and was brought to the clinic for the injection of tetanus toxoid. In order to prevent the development of anaphylactic shock the serum was administered by Bezredka method. What mechanism underlies this method of desensitization of the body?
+Binding of IgE fixed to the mast cells
-Blocking the mediator synthesis in the mast cells
-Stimulation of immune tolerance to the antigen
-Stimulation of the synthesis of antigenspecific IgG
-Binding of IgE receptors to the mast cells


#10 days after having quinsy caused by beta-hemolytic streptococcus a 6- year-old child exhibited symptoms of glomerulonephritis. What mechanism of glomerular lesion is most likely in this case?
+Immunocomplex
-Cellular cytotoxicity
-Anaphylaxis
-Atopy
-Antibody-dependent cell-mediated cytolysis


#A 22-year-old woman ate some seafood. 5 hours later the trunk and the distal parts of limbs got covered with small itchy papules which were partially fused together. After one day, the rash disappeared spontaneously. Specify the hypersensitivity mechanism underlying these changes:
+Atopy (local anaphylaxis)
-Systemic anaphylaxis
-Cellular cytotoxicity
-Immune complex hypersensitivity
-Antibody-dependent cell-mediated cytolysis


#During blood transfusion a patient has developed intravascular erythrocyte hemolysis. What kind of hypersensitivity does the patient have?
+II type (antibody-dependent)
-I type (anaphylactic)
-III type (immune complex)
-IV type (cellular cytotoxicity)
-IV type (granulomatosis)


#A 30-year-old patient has dyspnea fits, mostly at night. He has been diagnosed with bronchial asthma. What type of allergic


reaction according to the Gell-Coombs classification is most likely in this case?
+Anaphylactic
-Cytotoxic
-Stimulating
-Immune complex
-Delayed-type hypersensitivity


#Several minutes after a dentist administered Novocain for local anesthesia of a patient’s tooth, the following symptoms sharply developed in the patient: fatigue, skin itching. Objectively the following can be observed: skin hyperemia, tachycardia, BP dropped down to 70/40 mm Hg. What kind of allergic reaction is this pathology?
+Anaphylactic
-Cytotoxic
-Stimulating
-Cell-mediated immune reaction
-Immune complex


#After sensitization a test animal received subcutaneously a dose of antigen. At the site of injection, a fibrinous inflammation developed with alteration of vessels wall, basal substance, and fibrous structures of connective tissue. The inflammation took form of mucoid and fibrinoid necrosis. What immune response occurred in the test animal?
+Delayed-type hypersensitivity
-Transplantation immune reaction
-Normogenic reaction
-Granulomatosis
-Immediate-type hypersensitivity


#A 5-year-old child is diagnosed with Bruton syndrome (X-linked agammaglobulinemia) that manifests itself as severe clinical course of bacterial infection and absence of B-lymphocytes and plasma cells. What changes of immunoglobulin content can be observed in blood serum of the child with immunodeficiency?
+No changes
-Increased IgD, IgE
-Decreased IgD, IgE
-Increased IgA, IgM
-Decreased IgA, IgM


#During ascent into mountains a person develops increased respiration rate and rapid heart rate. What is the cause of these changes?
+Decrease in O2 partial pressure
-Increase in blood pH
-Increase nitrogen content in air
-Increase in CO2 partial pressure
-Increase in air humidity


#When ascending to the top of Elbrus, a mountain climber experiences oxygen starvation, dyspnea, palpitations, and numbness of the extremities. What kind of hypoxia has developed in the mountain climber?
+Hypoxic
-Circulatory
-Hemic
-Tissue
-Cardiac


#Measurements of the arterial pCO2 and pO2 during an attack of bronchial asthma revealed hypercapnia and hypoxemia respectively. What kind of hypoxia occurred in this case?
+Respiratory
-Hemic
-Circulatory
-Tissue
-Histotoxic


#A man has suffered multiple bone fractures of his lower extremities during a traffic accident. During transportation to a hospital his condition was further aggravated: blood pressure decreased, there were signs of pulmonary artery embolism. What kind of embolism is the most likely in the given case?
+Air embolism
-Gas embolism
-Thromboembolism
-Fat embolism
-Tissue embolism


#During the intravenous transfusion of the saline the patient’s condition deteriorated dramatically, and the patient died from asphyxiation. Autopsy revealed acute venous congestion of internal organs with the dramatic right heart dilatation. When the right ventricle was punctured underwater, the bubbles escaped. What pathological process occurred in the patient?
+Air embolism
-Gaseous embolism
-Adipose embolism
-Tissue embolism
-Thromboembolism


#A student failed to answer all the questions of examination paper correctly. As a result, he blushed, felt hot and lost confidence. What type of arterial hyperemia has developed in this case?
+Neurotonic hyperemia
-Neuroparalytic hyperemia
-Metabolic hyperemia
-Pathologic hyperemia
-Postishemic hyperemia


#A patient with obliterating endarteritis underwent ganglionic sympathectomy. What type of arterial hyperemia should have developed as a result of the surgery?
+Neuroparalytic
-Neurotonic
-Metabolic
-Functional
-Reactive


#A 54-year-old female was brought to the casualty department after a car accident. A traumatologist diagnosed her with multiple fractures of the lower extremities. What kind of embolism is most likely to develop in this case?
+Fat
-Tissue
-Thromboembolism
-Gaseous
-Air


#A 25-year-old patient complains on increasing pain in his leg muscles occurring during walking and forcing him to make frequent stops. Objectively: skin of legs is pale, no hair-covering, toenails are with trophic changes, no pulsation of pedal arteries. The most probable cause of these changes is:
+Ischemia
-Venous hyperemia
-Arterial hyperemia
-Embolism


#A patient with chronic heart failure presents with increased blood viscosity. Capillaroscopy detected damage to the vessel walls of the microcirculation system. What disorder is possible in the given case?
+Blood “sludge” phenomenon
-Thrombosis
-Embolism
-Venous hyperemia
-Arterial hyperemia


#During the exam a student was unable to correctly answer all the questions in his question card, which was accompanied by the reddening of his face and hot sensation. What type of arterial hyperemia did the student develop in this case?
+Pathologic
-Neurotonic
-Metabolic
-Neuroparalytic
-Post-ischemic


#Microscopy of the puncture sample obtained from the inflammation focus of the patient with cutaneous abscess revealed numerous blood cells of different types. What cells are the first to transfer from vessels to tissues during inflammation?
+Neutrophils
-Monocytes
-Lymphocytes
-Eosinophilic granulocytes
-Basophils


#A 30-year-old man complains of suffocation, heaviness in the chest on the right, general weakness. Body temperature is 38,9oC. Objectively the right side of the chest lags behind the left side during respiration. Pleurocentesis yielded exudate. What is the leading factor of exudation in the patient?
+Increased permeability of vessel wall
-Increased blood pressure
-Hypoproteinemia
-Erythrocyte aggregation
-Decreased resorption of pleural fluid


#A 7-year-old child has acute onset of disease: temperature rise up to 38 C, rhinitis, cough, lacrimation, and large-spot rash on the skin. Pharyngeal mucosa is edematous, hyperemic, with whitish spots in the buccal area. What kind of inflammation caused the changes in the buccal mucosa?
+Catarrhal inflammation
-Serous inflammation
-Fibrinous inflammation
-Hemorrhagic inflammation
-Suppurative inflammation


#Blood plasma of a healthy man contains several dozens of proteins. During an illness new proteins can originate, namely the protein of "acute phase". Select such protein from the listed below:
+C-reactive protein
-Prothrombin
-Fibrinogen
-Immunoglobulin G
-Immunoglobulin A


#The cellular composition of exudate largely depends on the etiological factor of inflammation. What leukocytes are the first to get into the focus of inflammation caused by pyogenic bacteria?
+Neutrophil granulocytes
-Monocytes
-Myelocytes
-Eosinophilic granulocytes
-Basophils


#As a result of careless handling of an iron, a 34-year-old female patient has got acute pain, redness, swelling of her right index finger. A few minutes later, there appeared a blister filled with a transparent liquid of yellow color. The described changes are a manifestation of the following pathological process:
+Exudative inflammation
-Traumatic edema
-Alternative inflammation
-Proliferative inflammation
-Vacuolar degeneration


#Lymphocytes and other cells of our body synthesize universal antiviral agents as a response to viral invasion. Name these protein factors:
+Interferon
-Interleukin - 2
-Cytokines
-Interleukin - 4
-Tumor necrosis factor


#A patient, having suffered a thermal burn, developed painful boils filled with turbid liquid in the skin. What morphological type of inflammation has developed in the patient?
+Serous
-Proliferative
-Croupous
-Granulomatous
-Diphtheritic


#Necrotic focus has appeared on burn, swallowing and red skin. What is the main mechanism of necrobiosis improvement in inflammatory area?
+Secondary alteration
-Primary alteration
-Emigration of leucocytes
-Dyapedesis of erythrocytes
-Fibroblasts proliferation


#A cook burnt his arm with steam. What substance increased and led to development of redness, edema and painfulness of affected area of skin?
+Histamine
-Lysine
-Thiamine
-Galactosamine
-Glutamine


#Enlargement and deformation of joints were revealed in a patient with rheumatism. What type of inflammation underlies these changes?
+Alterative
-Proliferative
-Exudative
-Fibrinous
-Hemorrhagic


#Indicate inflammatory mediators which have to be inhibited for decrease in exudation:
+Histamine
-Catecholamine
-Heparin
-Thromboxane
-Interleukine-1


#A patient has high body temperature, redness, edema, painfulness on her right forearm. What biological active substances intensify inflammatory reaction?
+Kinins
-Prostacyclin
-Phospholipase D
-Proteolysis inhibitors
-Vasopressin


#What inflammatory mediator is formed due to limited proteolysis of plasma globulins?
+Bradykinin
-Leukotrienes
-Histamine
-Prostaglandins
-Lymphokines


#Fever and Increase in antibodies and leukocytes have appeared in animal under experimental modeling of inflammation. What substances conditioned to all these common reactions in inflammation?
+Interleukins
-Leukotrienes
-Mineralocorticoids
-Glucocorticoids
-Somatomedins


#Which of following inflammatory mediators are formed under the influence of lypooxygenase?
+Leukotrienes
-Prostaglandins E1, E2
-Prostacyclin
-Thromboxane
-Thrombocytes activation factor


#A tumor is detected in one of the regions of the patient’s brain, resulting in the patient’s inability to maintain normal body temperature. What brain structure is damaged?
+Hypothalamus
-Thalamus
-Substantia nigra
-Cerebellum
-Striatum


#After transfusion of 200 ml of blood a patient presented with body temperature rise up to 37, 9oC. Which of the following substances is the most likely cause of temperature rise?
+Interleukin-1
-Interleukin-2
-Tumor necrosis factor
-Interleukin-3
-Interleukin-4


#A patient has been diagnosed with influenza. His condition became drastically worse after taking antipyretic drugs. His consciousness is confused, AP is 80/50 mm Hg, Ps is 140/m, body temperature dropped down to 35, 8oC. What complication developed in this patient?
+Collapse
-Hyperthermia
-Hypovolemia
-Acidosis
-Alkalosis


#This year influenza epidemic is characterized by patients’ body temperature varying from 36,9to 37,9o C. Such fever is called:
+Subfebrile
-Hyperpyretic
-High
-Apyretic
-Moderate


#A patient with lobar pneumonia has had body temperature of 39oC with daily temperature fluctuation of no more than 1oC for 9 days. This fever can be characterized by the following temperature curve:
+Persistent
-Hectic
-Remittent
-Hyperpyretic
-Recurrent


#Body temperature at the patient with an infection disease was increased to a day 39,5-40,50C and has been kept at this altitude about an hour and then returned to baseline. At which disease does this type of temperature curve occur?
+Malaria
-Tuberculosis
-Influenza
-Peritonitis
-Brucellosis


#In a patient elevation of body temperature takes turns with drops down to normal levels during the day. The rise in temperature is observed periodically once in four days. Specify the type of temperature curve:
+Febris intermittens
-Febris continua
-Febris reccurens
-Febris hectica
-Febris remitens


#A patient has acute bronchitis. The fever up to 38,5oC had lasted for a week, presently there is a decrease in temperature down to 37,0oC. Specify the leading mechanism in the 3rd stage of fever:
+Peripheral vasodilation
-Increased heat production
-Development of chill
-Increased diuresis
-Increased respiratory rate


#A patient suffers from intermittent fevers and normalizations of body temperature that occur during the day. The temperature rise is observed regularly every fourth day. Specify the type of temperature curve:
+Febris internittens
-Febris continua
-Febris reccurens
-Febris hectica
-Febris remitens


#At simulation of inflammation of the lower extremity the animal experienced raise of the temperature, Increase in amount of antibodies and leucocytes in the blood. What substances caused this general reaction of the organism on inflammation?
+Interleukin
-Glucocorticoid
-Mineralocorticoid
-Leukotrienes
-Somatomedins


#A patient has relapsing fever that is characterized by periods of several days of high temperatures, alternating with periods of normal temperature. This temperature curve is called:
+Febris recurrens
-Febris hectica
-Febris intermittens
-Febris continua
-Febris atypica


#Body temperature at the patient with an infection disease was increased to a day 39,5-40,50C and has been kept at this altitude about an hour and then returned to baseline. What type of temperature curve described in this case?
+Intermittens
-Constant
-Remittent
-Hectica
-Atypical


#A female patient has been diagnosed with cervical erosion, which is a precancerous pathology. What defense mechanism can prevent the development of a tumor?
+Increase in natural killer level (NK cells)
-High-dose immunological tolerance
-Increase in the activity of lysosomal enzymes
-Simplification of the antigenic structure of tissues
-Low-dose immunological tolerance


#A patient died of cancerous cachexia with primary localization of cancer in the stomach. Autopsy revealed acutely enlarged liver with uneven surface and numerous protruding nodes; the nodes had clear margins in the section, rounded shape, gray-pink color, varying density, sometimes contained necrotic foci. Histologically: there are atypical cells in the nodes. What pathologic process occurred in the liver?
+Cancer metastases
-Abscesses
-Regeneratory nodes
-Infarction
-Hepatic cancer


#Lung cancer developed at the patient who smoked tobacco for a long time. What of the carcinogens contained in tobacco smoke and concern to PAH?
+Benzpiren
-Dymethylaminoazobenzol
-β - naphthylamin
-Diethylnitrozamin
-Ortoaminoazotoluol


#There is high stage of interaction between lung cancer and tobacco smoking. What chemical carcinogen is contained in tobacco smog?
+3,4-benspyren
-Orhtoaninotoluol
-Aphlatoxin
-Methylcholatren
-Dyethylnitrosamine


#A patient, 62 years old, a fireman, complains of general weakness, sudden weight loss, husky voice, shortness of breathing, dry cough. Laryngeal tumor that germinates vocal cords and epiglottis was discovered at laryngoscopy. Name the most likely cause of the tumor in this patient:
+Polycyclic aromatic hydrocarbons
-Nitrosamines
-Aromatic amine and amides
-Retrovirus
-Ionizing radiation


#In a patient with metastases of lung carcinoma introduction of cytostatics led to suspension of metastases growth at first but later metastases resumed spread. What is the most possible mechanism of secondary growth of metastases?
+Absence of contact braking
-Absence of Heiflik’s limit
-Rise of genetic heterogeneity of tumor cells
-Increased glucose consumption by tumor
-Increased amino acids consumption by tumor


#A 56-years-old patient, who had contact with diethylnitrozamine at his work place, complains onpain in right subcostal area, weakness, loss of appetite, and decreased workability. At examination of this patient: surface of his liver is rough, splenomegaly and ascites are present in him; his body temperature is 37.2oC; in his blood analysis ESR is 25 mm/hour, besides neutrophilic leukocytosis, and hypochromic anemia were found. What disease developed in the patient’s organism?
+Cancer of liver
-Hepatitis
-Cirrhosis of liver
-Gallstone disease
-Dyskinesia of bile ducts


#Malignant tumor of lung was diagnosed in a patient. What feature of tumour growth testifies its malignancy?
+Infiltrative growth
-Unregulated growth
-Unlimited growth
-Expansive growth
-Appearance from one cell


#What biological process augmentation is typical for tumor cells?
+Anaerobic glycolysis
-Decarboxilation
-Tissue respiration
-Lipolysis
-Gluconeogenesis


#In 1910 Raus managed to cause sarcoma in chickens by cell-free infiltrate inserting. What was the method of experimental modeling?
+Induction
-Explantation
-Isotransplantation
-Homotransplantation
-Heterotransplantation


#A patient with urinary bladder cancer was working in coke factory. What substance was the most probable reason of this pathological condition?
+Naphtylamine
-Dichlorethane
-Vinegar acid
-Alcohol
-Petroleum ether


#They got nitrogenous nitrite to experimental animals. A tumor was developed in 80% of animals. What was the group of cancerogens?
+Nitrosamines
-Aminoasosubstances
-Polycyclic carbohydrates
-Simple chemical substances
-Hormones


#A 40-year-old woman with Cushing disease presents with steroid diabetes. On biochemical examination she has hyperglycemia and hypochloremia. What process activates in the first place in such patients?
+Gluconeogenesis
-Glycogenolysis
-Glucose reabsorption
-Glucose transportation into a cell
-Glycolysis


#A woman complains of visual impairment. Examination revealed obesity in the patient and her fasting plasma glucose level is hyperglycemic. What diabetes complication can cause visual impairment/blindness?
+Microangiopathy
-Atherosclerosis
-Macroangiopathy
-Glomerulopathy-Neuropathy


#A patient with insulin-dependent diabetes mellitus has been administered insulin. After a certain period of time the patient developed fatigue, irritability, excessive sweating. What is the main mechanism of such presentations developing?
+Decreased glyconeogenesis
-Increased ketogenesis
-Increased glycogenolysis
-Increased lipogenesis
-Carbohydrate starvation of the brain


#A patient with diabetes mellitus suffer from persistently nonhealing surgical wound, which is a sign of disrupted tissue trophism. What is the cause of such disorder?
+Disruption of protein metabolism regulation
-Ketonemia
-Anemia
-Hypoglycemia
-Increased lipid catabolism


#A 12-year-old teenager has significantly put off weight within 3 months; glucose concentration rose up to 50 millimole/l. He fell into a coma. What is the main mechanism of its development?
+Hyperosmolar
-Hypoglycemic
-Ketonemic
-Lactacidemic
-Hypoxic


#The patient with complaints of permanent thirst applied to the doctor. Hyperglycemia, polyuria and increased concentration of 17-ketosteroids in the urine were revealed. What disease is the most likely?
+Steroid diabetes
-Insulin-dependent diabetes mellitus
-Myxedema
-Type I glycogenosis
-Addison’s disease


#Before the cells can utilize the glucose, it is first transported from the extracellular space through the plasmatic membrane inside them. This process is stimulated by the following hormone:
+Insulin
-Glucagon
-Thyroxin
-Aldosterone
-Adrenalin


#According to the results of glucose tolerance test, the patient has no disorder of carbohydrate tolerance. Despite that, glucose is detected in the patients’s urine (5 mmol/l). The patient has been diagnosed with renal diabetes. What renal changes cause glucosuria in this case?
+Decreased activity of glucose reabsorption enzymes
-Increased activity of glucose reabsorption enzymes
-Exceeded glucose reabsorption threshold
-Increased glucose secretion
-Increased glucose filtration


#Examination of a 56-year-old female patient with a history of type 1 diabetes revealed a disorder of protein metabolism that is manifested by aminoacidemia in the laboratory blood test values, and clinically by the delayed wound healing and decreased synthesis of antibodies. Which of the following mechanisms causes the development of aminoacidemia?
+Increased proteolysis
-Albuminosis
-Decrease in the concentration of amino acids in blood
-Increase in the oncotic pressure in the blood plasma
-Increase in low-density lipoprotein level


#Prior to glucose utilization in cells it is transported inside cells from extracellular space through plasmatic membrane. This process is stimulated by the following hormone:
+Insulin
-Glucagon
-Thyroxin
-Aldosterone
-Adrenalin


#A 15-year-old patient has fasting plasma glucose level 4,8 mmol/l, one hour after glucose challenge it becomes 9,0 mmol/l, in 2 hours it is 7,0 mmol/l, in 3 hours it is 4,8 mmol/l. Such parameters are characteristic of:
+Subclinical diabetes mellitus
-Diabetes mellitus type 1
-Diabetes mellitus type 2
-Healthy person
-Cushing’s disease


#A 50-year-old inpatient during examination presents with glycosuria and blood glucose of 3,0 mmol/l, which are the most likely to be caused by:
+Renal disorder
-Essential hypertension
-Myxedema
-Pellagra
-Diabetes insipidus


#Ketosis develops in the patients with diabetes mellitus as the result of activation of fatty acids oxidation processes. What acid-base imbalance can result from accumulation of excessive ketone bodies in the blood?
+Metabolic acidosis
-Metabolic alkalosis
-Respiratory acidosis
-Gaseous alkalosis
-No imbalance occurs


#A patient suffers from disrupted patency of the airways at the level of small and medium-sized bronchial tubes. What changes of acid-base balance can occur in the patient?
+Respiratory acidosis
-Metabolic alkalosis
-Acid-base balance remains unchanged
-Respiratory alkalosis
-Metabolic acidosis


#A newborn child with pylorostenosis has often repeating vomiting accompanied by apathy, weakness, hypertonicity, sometimes convulsions. What disorder form of acid-base balance is it?
+Nongaseous alkalosis
-Gaseous alkalosis
-Gaseous acidosis
-Metabolic acidosis
-Excretory acidosis


#An infant has pylorospasm, weakness, hypodynamia, convulsions as a result of frequent vomiting. What kind of acid-base misbalance is it?
+Excretory alkalosis
-Excretory acidosis
-Metabolic acidosis
-Exogenous nongaseous acidosis
-Gaseous alkalosis


#After taking poor-quality food a patient developed repeated episodes of diarrhea. On the next day he presented with decreased arterial pressure, tachycardia, extrasystole. Blood pH is 7,18. These abnormalities were caused by the development of:
+Nongaseous acidosis
-Gaseous acidosis
-Nongaseous alkalosis
-Gaseous alkalosis
-Metabolic alkalosis


#A patient with diabetes developed a diabetic coma due to the acid-base imbalance. Specify the kind of this imbalance:
+Metabolic acidosis
-Metabolic alkalosis
-Respiratory acidosis
-Gaseous alkalosis
-Non-gaseous alkalosis


#A patient with respiratory failure has blood pH of 7,35. pCO2 test revealed hypercapnia. Urine pH test revealed an increase in the urine acidity. What form of acid-base imbalance is the case?
+Compensated respiratory acidosis
-Compensated metabolic acidosis
-Decompensated metabolic acidosis
-Compensated respiratory alkalosis
-Decompensated respiratory alkalosis


#A hypertensive patient had been keeping to a salt-free diet and taking antihypertensive drugs together with hydrochlorothiazide for a long time. This resulted in electrolyte imbalance. What disorder of the internal environment occurred in the patient?
+Hypochloremic alkalosis
-Metabolic acidosis
-Hyperkalemia
-Hypermagnesemia
-Increase in circulating blood volume


#A 30-year-old man with diabetes mellitus type I was hospitalized. The patient is comatose. Laboratory tests revealed hyperglycemia and ketonemia. What metabolic disorder can be detected in this patient?
+Metabolic acidosis
-Metabolic alkalosis
-Respiratory acidosis
-Respiratory alkalosis
-Normal acid-base balance


#A woman with enteritis accompanied by severe diarrhea presents with loss water in the extracellular space, high water content in the cells and decreasing blood osmolality. Name this type of water-electrolyte imbalance?
+Hypoosmolarhypohydration
-Hyperosmolar hypohydration
-Osmolarhypohydration
-Hypoosmolarhyperhydration
-Hyperosmolar hyperhydration


#A woman with hypophyseal diabetes insipidus developed a water-mineral imbalance. What type is of water-mineral imbalance develops in such case?
+Hyperosmolar hypohydration
-Hypoosmolarhypohydration
-Osmolarhypohydration
-Hypoosmolarhyperhydration
-Hyperosmolar hyperhydration


#A concentrated solution of sodium chloride was intravenously injected to an animal. This caused decreased reabsorption of sodium ions in the renal tubules. It is the result of the following changes of hormonal secretion:
+Aldosterone reduction
-Aldosterone increase
-Vasopressin reduction
-Vasopressin increase
-Reduction of atrial natriuretic factor


#A patient is 44 years old. Laboratory examination of his blood revealed that content of proteins in plasma was 40 g/l. What influence will be exerted on the transcapillary water metabolism?
+Filtration will be increased, reabsorption- decreased
-Both filtration and reabsorption will be increased
-Both filtration and reabsorption will be decreased
-Filtration will be decreased, reabsorption- increased
-Metabolism will stay unchanged


#A patient with enteritis accompanied by massive diarrhea has low water rate in the extracellular space, high water rate inside the cells and low blood osmolality. What is such disturbance of water-electrolytic metabolism called?
+Hypo-osmolar hypohydration
-Hyperosmolar hypohydration
-Osmolarhypohydration
-Hypo-osmolarhyperhydration
-Hyperosmolar hyperhydration


#A 20 -year old patient complains of morbid thirst and increased diuresis (up to 10 l daily). Glucose concentration in blood is normal but it is absent in urine. The patient has been diagnosed with diabetes insipidus. What hormonal drug is the most appropriate for management of this disorder?
+Vasopressin
-Cortisol
-Thyroxin
-Oxytocin
-Insulin


#Atria of an experimental animal were overextended by blood that resulted in decreased reabsorption of Na
+ and water in renal tubules. This can be explained by the influence of the following factor upon kidneys:
+Natriuretic hormone
-Aldosterone
-Renin
-Angiotensin
-Vasopressin


#A 56 -year old patient suffering from cardiac insufficiency has edema of feet and shins, edematous skin is pale and cold. What is the leading mechanism of edema pathogenesis?
+Rise of hydrostatic pressure in venules
-Drop of oncotic pressure in capillaries
-Increase in capillary permeability
-Disorder of lymph outflow
-Positive water balance


#A 49-year old woman spent a lot of time standing. As a result of it she got leg edema. What is the most likely cause of the edema?
+Increase in hydrostatic pressure of blood in veins
-Decrease in hydrostatic pressure of blood in veins
-Decrease in hydrostatic pressure of blood in arteries
-Increase in oncotic pressure of blood plasma
-Increase in systemic arterial pressure


#A patient who had been continuously taking drugs blocking the production of angiotensin II developed bradycardia and arrhythmia. A likely cause of these disorders is:
+Hyperkalemia
-Hypokalemia
-Hypernatremia
-Hypocalcemia
-Hypercalcemia


#A patient with a pathology of the cardiovascular system developed edema of the lower extremities. What is the mechanism of cardiac edema development?
+Increased hydrostatic pressure at the venous end of the capillary
-Increased oncotic pressure
-Increased hydrostatic pressure at the arterial end of the capillary
-Reduced osmotic pressure
-Lymph efflux disorder


#30 minutes after drinking mango juice a child suddenly developed a local swelling in the area of the soft palate, which impeded swallowing and, eventually, respiration. Mucosa of the swollen area was hyperemic and painless. Blood test revealed moderate eosinophilia. Body temperature was normal. Anamnesis states that the elder sister of the child has been suffering from bronchial asthma attacks. What kind of edema has developed in the child?
+Allergic
-Inflammatory
-Alimentary
-Cardiac
-Hepatic


#A patient has severe blood loss caused by an injury. What kind of dehydration will be observed in this particular case?
+Iso-osmolar
-Hyposmolar
-Hyperosmolar
-Normosmolar


#A 50-year-old man, who has been suffering from chronic hepatic failure for several years, has developed ascites. What is the main mechanism of this disorder development?
+Increased pressure in portal vein system
-Decrease in albumin and globulin synthesis in liver
-Increased content of low-density and very low-density lipoproteins in blood
-Neurotoxins appearing in blood
-Increase in blood oncotic pressure


#Upon toxic damage of hepatic cells resulting in disruption of liver function the patient developed edemas. What changes of blood plasma are the main cause of edema development?
+Decrease in albumin content
-Increase in globulin content
-Decrease in fibrinogen content
-Increase in albumin content
-Decrease in globulin content


#Patient with bile duct obstruction typically presents with inhibited blood clotting and develop hemorrhages due to insufficient assimilation of vitamin:
+K
-A
-E
-D
-C


#An 8-year old girl presents with signs disturbed twilight vision. This condition is caused by the deficiency of vitamin:
+A
-K
-E
-D
-C


#A sick child presents with high content of phenyl pyruvate in urine (normally it is practically absent). Blood phenylalanine level is 350mg/L (norm 15mg/l). what disease are these symptoms characteristic of?
+Phenylketonuria
-Tyrosinosis
-Albinism
-Gout
-Alkaptonuria


#A woman has been limiting the amount of products in her diet to lose some weight. 3 months later she developed edemas and her diuresis increased. What dietary component deficiency is the case of this?
+Proteins
-Fats
-Carbohydrates
-Vitamins
-Minerals


#A 52-year-old man presents with fever and pain in the joints. Both of his first metatarsophalangeal articulations are deformed, swollen and reddened. Blood urea is high. The patient is diagnosed with gout. What is the main developmental factor in the pathogenesis of this disease?
+Hyperuricemy
-Hyperaminoacidemia
-Citrullinuria
-Hyperazotemia
-Argininosuccinic aciduria


#A 2-year-old child with mental and physical retardation has been delivered to a hospital. He presents with frequent vomiting after having meals. There is phenylpyruvic acid in urine. Which metabolism abnormality is the reason for this pathology?
+Amino-acid metabolism
-Lipid metabolism
-Carbohydrate metabolism
-Water-salt metabolism
-Phosphoric calcium metabolism


#A patient has been diagnosed with alkaptonuria. Choose an enzyme whose deficiency can be the reason for this pathology:
+Homogentisic acid oxidase
-Phenylalanine hydroxylase
-Glutamate dehydrogenase
-Pyruvate dehydrogenase
-Dioxyphenylalanine decarboxylase


#A patient complains about dyspnea provoked by the physical activity. Clinical examination revealed anemia and presence of the paraprotein in the zone of gamma-globulins. To confirm the myeloma diagnosis, it is necessary to determine the following index in the patient’s urine:
+Bence Jones protein
-Bilirubin
-Haemoglobin
-Ceruloplasmin
-Antitrypsin


#A 2 -year old child with mental and physical retardation has been delivered to a hospital. He presents with frequent vomiting after having meals. There is phenylpyruvic acid in urine. Which metabolism abnormality is the reason for this pathology?
+Amino-acid metabolism
-Lipidic metabolism
-Carbohydrate metabolism
-Water-salt metabolism
-Phosphoric calcium metabolism


#A 62- year old woman complains of frequent pain attacks in the area of her chest and backbone, rib fractures. Her doctor suspected myeloma (plasmocytoma). What of the following laboratory characteristics will be of the greatest diagnostic importance?
+Paraproteinemia
-Hyperalbuminemia
-Proteinuria
-Hypoglobulinemia
-Hypoproteinemia


#Toxic affection of liver results in dysfunction of protein synthesis. It is usually accompanied by the following kind of dysproteinemia:
+Absolute hypoproteinemia
-Relative hypoproteinemia
-Absolute hyperproteinemia
-Relative hyperproteinemia
-Paraproteinemia


#A 46-year-old female patient consulted a doctor about pain in the small joints of the upper and lower limbs. The joints are enlarged and shaped like thickened nodes. Serum test revealed an increase in urate concentration. This might be caused by a disorder in metabolism of:
+Purines
-Carbohydrates
-Lipids
-Pyrimidines
-Amino acids


#In case of alkaptonuria, homogentisic acid is excreted in urine in large amounts. The development of this disease is associated with a disorder of metabolism of the following aminoacid:
+Tyrosine
-Phenylalanine
-Alanine
-Methionine
-Asparagine


#A 49-year-old man complains of pain in his metatarsophalangeal joints and joint deformation. In blood hyperuricemy can be observed. X-ray has revealed metatarsophalangeal joint space narrowing, erosion, periarticular calcification of the both joints, osteoporosis. Microscopy has revealed inflammatory granulomatous reaction surrounding necrotizing masses in the area of the first metatarsophalangeal joint. Choose the most likely diagnosis:
+Gout
-Pyrophosphate arthropathy
-Rheumatoid arthritis
-Hyperparathyroidism
-Urolithiasis


#During examination of a teenager with xanthomatosis the family history of hypercholesterolemia is revealed. What transportable lipids are increased in concentration in case of such a disease?
+Low-density lipoproteins
-Very low-density lipoproteins
-Intermediate-density lipoproteins
-High-density lipoproteins
-Chylomicrons


#During regular check-up a child is determined to have interrupted mineralization of the bones. What vitamin deficiency can be the cause?
+Calciferol
-Folic acid
-Tocopherol
-Cobalamin
-Riboflavin


#A 12-year-old patient was found to have blood serum cholesterol at the rate of 25 mmol/l. The boy has a history of hereditary familial hypercholesterolemia, which is caused by the impaired synthesis of the following protein receptors:
+Low density lipoproteins
-High density lipoproteins
-Chylomicrons
-Very low density lipoproteins
-Intermediate density lipoproteins


#A patient with exacerbated burn has developed esophagus stenosis. The patient presents with acute weight loss due to problematic food intake. Blood test: erythrocyte -3,0х1012/l, Hb-106 g/l, total protein - 57 g/l. What type of starvation does patient suffer from?
+Incomplete starvation
-Protein starvation
-Water starvation
-Absolute starvation
-Complete starvation


#When measuring power inputs of a man by the method of indirect calorimetry the following results were obtained:1000 ml oxygen consumption and 800 ml carbon dioxide liberation per minute. The man under examination has the following respiratory coefficient:
+0,8
-1,25
-0,9
-0,84
-1,0


#There is only one source of water for the body at absolute starvation - a process of organic compounds oxidation. Which of the following substances in these conditions is the main source of endogenous water?
+Fats
-Proteins
-Carbohydrates
-Glycoproteins
-Lipoproteins


#Negative nitrogen balance, hypoproteinemia, violation of water-salt metabolism combined with normal function of digestive system were founded at the vegetarians. Name the reason of this state.
+Monotonous protein diet
-Monotonous carbohydrate diet
-Lack of unsaturated fatty acids
-Deficiency of phospholipids in the food
-Lack of vitamin E in the food


#On 10th day of medical starvation the patient suffers from excitation, deep, noisy breathing, blood pressure dropped to 90/60 mmHg, oliguria, urine with a smell of acetone. Name the reason of this state.
+Ketosis
-Non-gas alkalosis
-Hyperglycemia
-Hypoglycemia
-Gas acidosis


#Esophagus stenosis developed at a patient after a chemical burn. Sharp weight loss raised. In the blood: erythrocyte -3,0х1012/l, Hb-106 g/l, total protein - 57 g/l. What type of starvation developed at a patient?
+Incomplete
-Albumen
-Water
-Absolute
-Complete


АНЕМИЯ (29)


#One of the causes of pernicious anemia is disturbed synthesis of transferrin - Castle’s intrinsic factor - by the parietal cells of the stomach. What substance is called Castle’s extrinsic factor?
+Cobalamin
-Folic acid
-Biotin
-Riboflavin
-Pyridoxine


#A woman has come to the hospital with complaints of general weakness, dizziness and dyspnea. Recently she has been taking levomycetin (chloramphenicol) for prevention of enteric infection. Blood test: erythrocytes - 1.9 ・1012/L, hemoglobin - 58 g/L, color index - 0.9, leukocytes - 2.2 G/L, reticulocytes - 0.3%. What type of anemia is it indicative of?
+Hemolytic
-Aplastic
-Hypoplastic
-Metaplastic
-Iron-deficiency


#A 30-year-old patient’s blood test has revealed the following: erythrocyte count is 6x1012/l, hemoglobin is 10,55 mmol/l. Vaquez’s disease was diagnosed. Name the leading part of pathogenesis:
+Neoplastic erythroid hyperplasia
-Iron-deficiency
-B 12 -deficiency
-Hypoxia
-Acidosis


#A 42-year-old patient complains of pain in the epigastral area, vomiting; vomit masses have the color of coffee-grounds; the patient suffers from melena. Anamnesis records gastric ulcer disease. Blood formula: erythrocytes - 2, 8x10 12 /l, leukocytes – 8x109 /l, Hb- 90 g/l. What complication is it?
+Hemorrhage
-Penetration
-Perforation
-Malignisation
-Pyloric stenosis


#A 27-year-old patient with injury to the neck has lost approximately 30% of the blood volume. The patient’s condition is severe: blood pressure is 60/40 mm Hg, heart rate is 140/min., respiratory rate is 30/min., conscious. Characterize the condition of the patient’s circulatory system:
+Hypovolemic shock
-Cardiogenic shock
-Collapse
-Coma
-Arterial hypertension


#On the fifth day after the acute blood loss a patient has been diagnosed with hypochromic anemia. What is the main mechanism of hypochromia?
+Release of immature red blood cells from the bone marrow
-Impaired iron absorption in the intestines
-Increased destruction of red blood cells in the spleen
-Impaired globin synthesis
-Increased excretion of body iron


#In a car accident a man got injured and lost a lot of blood. What changes in peripheral blood are most likely to occur on the 2nd day after the injury?
+Erythropenia
-Hypochromia
-Anisocytosis
-Microplania
-Significant reticulocytosis


#A 26 -year old pregnant woman is under treatment at an in-patient hospital. After a continuous attack of vomiting she was found to have reduced volume of circulating blood. What kind of change in general blood volume is the case?
+Polycythemic hypovolemia
-Simple hypovolemia
-Oligocythemic hypovolemia
-Polycythemic hypervolemia
-Oligocythemic hypervolemia


#A 38-year-old patient with uterine hemorrhage lasting for 2 days was delivered to the admission ward. Which of the following will be revealed in the patient’s blood?
+Decrease in the hematocrit
-Eosinophilia
-Deceleration in ESR
-Leukocytosis
-Increase in the color index


#A 32-year-old patient was admitted to the hospital with gross blood loss due to auto accident trauma. Ps – 110 Bpm, RR- 22 pm, BP- 100/60mm Hg. What changes in the blood will occur in an hour after the blood loss?
+Hypovolemia
-Erythropenia
-Hypochromia of erythrocytes
-Leukopenia
-Hypoproteinemia


#Examination of a 43 y.o. anephric patient revealed anemia symptoms. What is the cause of these symptoms?
+Reduced synthesis of erythropoietin
-Enhanced destruction of erythrocytes
-Iron deficit
-Vitamin B12 deficit
-Folic acid deficit


#In the blood of a 26-year-old man 18% of erythrocytes of the spherical, ball-shaped, flat and spinous shape have been revealed. Other erythrocytes were in form of the concavo-concave disks. How is this phenomenon called?
+Physiological poikilocytosis
-Pathological poikilocytosis
-Physiological anisocytosis
-Pathological anisocytosis
-Erythrocytosis


#A patient’s blood was analyzed and the decreased erythrocyte’s sedimentation rate (ESR) was discovered. What disease from the listed below is accompanied with decreased ESR?
+Polycytemia
-Hepatitis
-Splenomegaly
-Vitamin B12 deficiency
-Myocardial infarction


#A 54-year-old patient, 5th day after surgical operation. Blood count: Erythrocytes 3,6x1012/l, Hemoglobin 95 g/l, Erythrocyte’s hemoglobin content (color index) 0,78; Leukocytes 16x109/l, Platelets 450x109/l Blood picture: anizocytosis, poikilocytosis, reticulocytes-3,8%. What anemia does this patient have?
+Acute posthemorragic anemia
-Acquired hemolytic anemia
-Anemia from iron deficiency
-Hypoplastic anemia
-Chronic posthemorragic anemia


#Examination of a 52-year-old woman has revealed a decrease in the amount of red blood cells and an increase in free hemoglobin in the blood plasma (hemoglobinemia). Color index is 0,85. What type of anemia is being observed in the patient?
+Acquired hemolytic
-Hereditary hemolytic
-Acute hemorrhagic
-Chronic hemorrhagic
-Anemia due to diminished erythropoiesis


#Along with normal hemoglobin types there can be pathological ones in the organism of an adult. Name one of them:
+HbS
-HbF
-HbA 1
-HbA 2
-HbO 2


#A patient is diagnosed with chronic atrophic gastritis attended by deficiency of Castle’s intrinsic factor. What type of anemia does the patient have?
+B12 -deficiency anemia
-Iron refractory anemia
-Hemolytic anemia
-Iron-deficiency anemia
-Protein-deficiency anemia


#Biochemical analysis of an infant’s erythrocytes revealed evident glutathione peroxidase deficiency and low concentration of reduced glutathione. What pathological condition can develop in this infant?
+Hemolytic anemia
-Pernicious anemia
-Megaloblastic anemia
-Sicklemia
-Iron-deficiency anemia


#Examination of a 52-year-old female patient has revealed a decrease in the amount of red blood cells and an increase in free hemoglobin in the blood plasma (hemoglobinemia). Color index is 0,85. What type of anemia is being observed in the patient?
+Acquired hemolytic
-Hereditary hemolytic
-Acute hemorrhagic
-Chronic hemorrhagic
-Anemia due to diminished erythropoiesis


#A 37-year-old female patient complains of headache, vertigo, troubled sleep, numbness of limbs. For the last 6 years she has been working at the gas-discharge lamp-producing factory in the lead-processing shop. Blood test findings: low haemoglobin and RBC level, serum iron concentration exceeds the norm by several times. Specify the type of anemia:
+Iron refractory anemia
-Iron-deficiency anemia
-Minkowsky-Shauffard disease
-Hypoplastic anemia
-Metaplastic anemia


#A 19-year-old female patient has had low haemoglobin rate of 90-95 g/l since childhood. Blood count results obtained after hospitalization are as follows: erythrocytes - 3,2 x 1012/l, Hb- 85 g/l, color index - 0,78; leukocytes - 5,6 x 109/l, platelets – 210x 109/l. Smear examination revealed anisocytosis, poikilocytosis and target cells. Reticulocyte rate is 6%. Iron therapy was ineffective. What blood pathology corresponds with the described clinical presentations?
+Thalassemia
-Enzymopathy
-Membranopathy
-Sickle-cell anemia
-Favism


#A 34-year old woman was diagnosed with hereditary microspherocytic hemolytic anemia (Minkowsky-Shauffard disease). What mechanism caused hemolysis of erythrocytes?
+Membranopathy
-Enzymopathy
-Hemoglobinopathy
-Autoimmune disorder
-Bone marrow hypoploasia


#A 15- year old girl has pale skin, glossitis, gingivitis. Blood count: erythrocytes - 3,3 x1012/l, hemoglobin - 70 g/l, color index - 0,5. Examination of blood smear revealed hypochromia, microcytosis, poikilocytosis. What type of anemia is it?
+Iron-deficient
-B12-folic acid-deficient
-Sickle-cell
-Hemolytic
-Thalassemia


#As a result of increased permeability of the erythrocyte membrane in a patient with microspherocytic anaemia (Minkowsky-Shauffard disease) cells receive sodium ions and water. Erythrocytes take form of spherocytes and can be easily broken down. What is the leading mechanism of erythrocyte damage in this case?
+Electrolytic osmotic
-Calcium
-Acidotic
-Protein
-Nucleic


#A 20- year old patient complains of general weakness, dizziness, quick fatigability. Blood analysis results: Hb- 80g/l. Microscopical examination results: erythrocytes are of modified form. This condition might be caused by:
+Sickle-cell anemia
-Hepatocellular jaundice
-Acute intermittent porphyria
-Obstructive jaundice
-Addison’s disease


#A 56 -year old patient came to a hospital with complaints about general weakness, tongue pain and burning, sensation of limb numbness. In the past he underwent resection of forestomach. In blood: Hb- 80 g/l; erythrocytes - 2, 0·1012/l; colour index - 1,2, leukocytes - 3, 5 109/l. What anemia type is it?
+B12-folate deficient
-Hemolytic
-Posthemorrhagic
-Aplastic
-Iron-deficient


#Surgical removal of a part of stomach resulted in disturbed absorption of vitamin В12, it is excreted with feces. The patient was diagnosed with anemia. What factor is necessary for absorption of this vitamin?
+Gastromucoprotein
-Gastrin
-Hydrochloric acid
-Pepsin
-Folic acid


#A patient with hypochromic anemia has splitting hair and loss of hair, increased nail brittling and taste alteration. What is the mechanism of the development of these symptoms?
+Deficiency of iron-containing enzymes
-Deficiency of vitamin В12
-Decreased production of parathyrin
-Deficiency of vitamin А
-Decreased production of thyroid hormones


#Examination of initial molecular structure revealed substitution of the glutamic acid by valine. What inherited pathology is it typical for?
+Sickle-cell anemia
-Thalassemia
-Minkowsky-Shauffard disease
-Favism
-Hemoglobinosis


ЛЕЙКОЦИТОЗЫ (24)


#24 hours after an appendectomy the patient’s blood test shows neutrophilic leukocytosis with a regenerative shift. What is the most likely mechanism of absolute leukocytosis development in the patient’s peripheral blood?
+Intensification of leukopoiesis
-Deceleration of leukocyte migration to the tissues
-Immunity activation
-Decreased leukocyte disintegration
-Leukocyte redistribution


#After a severe stress the patient presents with eosinopenia in the blood test. In this case the decreased number of eosinophils can explain changes in the level of the following hormones:
+Glucocorticoids
-Vasopressin
-Adrenaline
-Mineralocorticoids
-Insulin


#A 59-year-old woman has been hospitalized in a surgical ward due to exacerbation of chronic osteomyelitis of the left shin. Blood test: leukocytes - 15,0x109 /l. Leukogram: myelocytes - 0%, metamyelocytes - 8%, stab neutrophils - 28%, segmented neutrophils - 32%, lymphocytes - 29%, monocytes - 3%. Such blood count would be called:
+Regenerative left shift
-Right shift
-Hyperregenerative left shift
-Degenerative left shift
-Regenerative-degenerative left shift


#A 3-year-old child has eaten some strawberries. Soon he developed a rash and itching. What was found in the child’s leukogram?
+Eosinophilia
-Lymphopenia
-Neutrophilic leukocytosis
-Monocytosis
-Lymphocytosis


#A 23-year-old patient with acute pulpitis has elevated body temperature and an increase in the WBC count up to 14x109 /L. The leucogram is as follows: basophils - 0, eosinophils - 2, monocytes - 0, immature neutrophils - 4, stab neutrophils - 8, segmented neutrophils - 56, lymphocytes - 26, monocytes - 4. How can we interpret these changes in the white blood cells?
+Neutrophilia with a regenerative left shift
-Neutrophilia with a degenerative left shift
-Neutrophilia with a hyperregenerative left shift
-Lymphocytosis
-Neutrophilic leukocytosis with a right hift


#A 49-year-old male patient with myocardial infarction has been admitted to the cardiology department. What changes in the peripheral blood cells are induced by the necrotic changes in the myocardium?
+Neutrophilic leukocytosis
-Monocytosis
-Eosinophilia
-Thrombocytopenia
-Lymphopenia


#Having helped to eliminate consequences of a failure at a nuclear power plant, a worker got radiation dose of 500 Roentgen. He complains of headache, nausea, dizziness. What changes in leukocytes quantity can be expected 10 hours after irradiation?
+Neutrophilic leukocytosis
-Lymphocytosis
-Leukopenia
-Agranulocytosis
-Leukemia


#After an attack of bronchial asthma, a patient had his peripheral blood tested. What changes can be expected?
+Eosinophilia
-Leukopenia
-Lymphocytosis
-Thrombocytopenia
-Erythrocytosis


#As a result of a road accident a 26- year-old man is in the torpid phase of shock. Blood count: leukocytes - 3, 2 109/l. What is the leading mechanism of leukopenia development?
+Leukocyte redistribution in the bloodstream
-Leukopoiesis inhibition
-Faulty release of mature leukocytes from the bone marrow into the blood
-Leukocyte destruction in the hematopietic organs
-Increased excretion of the leukocytes from the organism


#Two hours after an exam a student had a blood count done and it was revealed that he had leukocytosis without significant leukogram modifications. What is the most probable mechanism of leukocytosis development?
+Redistribution of leukocytes in the organism
-Leukopoiesis intensification
-Deceleration of leukocyte lysis
-Deceleration of leukocyte migration to the tissues
-Leukopoiesis intensification and deceleration of leukocyte lysis


#A 47- year old man with myocardium infarction was admitted to the cardiological department. What changes of cellular composition of peripheral blood are induced by necrotic changes in the myocardium?
+Neutrophilic leukocytosis
-Monocytosis
-Eosinophilic leukocytosis
-Thrombocytopenia
-Lymphopenia


#A 5 -year old child is ill with measles. Blood analysis revealed Increase in total number of leukocytes up to 13 · 109/l. Leukogram: basophils - 0, eosinophils - 1, myelocytes - 0, juvenile neutrophils - 0, band neutrophils - 2, segmented neutrophils - 41, lymphocytes - 28, monocytes - 28. Name this phenomenon:
+Monocytosis
-Agranulocytosis
-Lymphocytosis
-Eosinopenia
-Neutropenia


#Examination of a patient admitted to the surgical department with symptoms of acute appendicitis revealed the following changes in the white blood cells: the total count of leukocytes is 16x109/l. Leukocyte formula: basophils - 0, eosinophils - 2%, juvenile forms - 2%, stab - 8%, segmentonuclear - 59%, lymphocytes - 25%, monocytes- 4%. The described changes can be classified as:
+Neutrophilia with regenerative left shift
-Neutrophilia with right shift
-Neutrophilia with degenerative left shift
-Neutrophilic leukemoid reaction
-Neutrophilia with hyperregenerative left shift


#A 26- year old man is in the torpid shock phase as a result of a car accident. In blood: 3, 2 109/l. What is the leading mechanism of leukopenia development?
+Redistribution of leukocytes in bloodstream
-Leikopoiesis inhibition
-Disturbed going out of mature leukocytes from the marrow into the blood
-Lysis of leukocytes in the blood-forming organs
-Intensified elimination of leukocytes from the organism


#A 16-year-old boy was performed an appendectomy. He has been hospitalized for right lower quadrant abdominal pain within 18 hours. The surgical specimen is edematous and erythematous. Infiltration by what of the following cells is the most typical for the process occuring here?
+Neutrophils
-Eosinophils
-Basophils
-Lymphocytes
-Monocytes


#As a result of the damage of one of the Atomic Power Plant reactors the runout of radioelements took place. People in the high-radiation area were radiated with approximately 250-300 r. They were immediately hospitalized. What changes in the blood count would be typical for the victims?
+Lymphopenia
-Leukopenia
-Anemia
-Thrombopenia
-Neutropenia


#Blood sampling for the haematology is recommended to carry out on an empty stomach and in the morning. What changes in blood formula are possible if blood sampling was carried out after food intake?
+Increase in leukocyte number
-Increase in erythrocyte number
-Increase in plasm proteins
-Decrease in thrombocyte number
-Decrease in erythrocyte number


#Blood sampling for bulk analysis is recommended to be performed on an empty stomack and in the morning. What changes in blood composition can occur if to perform blood sampling after food intake?
+Increased contents of leukocytes
-Increased contents of erythrocytes
-Increased plasma proteins
-Reduced contents of thrombocytes
-Reduced contents of erythrocytes


#A 54-year-old man complains of general weakness, frequent colds and bruises constantly appearing on his body. Blood test: erythrocytes - 2.5x1012/L; Hb - 80 g/l; Colour index - 0.9; reticulocytes - absent; platelets – 50x109/L; leukocytes – 58x109/L; leukogram: basocytes - 5%, eosinophils - 15%, myeloblasts - 6%, myelocytes - 10%, juvenile - 18%, stab neutrophils - 26%, segmented neutrophils - 10%, lymphocytes - 8%, monocytes - 2%, ESR - 40 mm/hour. What hematologic conclusion can be made?
+Chronic myelogenous leukemia
-Leukemoid response
-Basophilic eosinophilic leukocytosis
-Myeloblastic leukaemia
-Chronic lymphocytic leukemia


#A patient is 20 years old, an athlete. He addressed a doctor with complaints of fatigue, fever up to 38o C - 40o C. Objectively: the liver and spleen are enlarged, lymph nodes on palpation are slightly enlarged, dense, painless. Blood test: Нb – 100 g/l; erythrocytes – 2,9x1012/l; leukocytes – 4,4x109/l. Leukogram: 68% of blast cells. Cytochemical investigation of blast cells revealed negative reactions to glycogen, peroxidase, non-specific esterase, lipids. Name this disease:
+Acute undifferentiated leukemia
-Acute myeloid leukemia
-Acute monoblastic leukemia
-Acute lymphoblastic leukemia
-Acute megakaryoblastic leukemia


#A 39-year-old patient underwent hematologic tests. The following results were obtained: RBC-2,8·1012/L, Нb-80 g/L, color index - 0,85, reticulocytesm - 0,1%, platelets – 16х109/L, WBC – 60х109/L. Basophils - 2, eosinophils - 8, promyelocytes - 5, myelocytes - 5, immature neutrophils - 16, stab neutrophils - 20, segmented neutrophils - 34, lymphocytes - 5, monocytes -5. What form of blood pathology are these results indicative of?
+Chronic myeloid leukemia
-Acute myeloid leukemia
-Hypoplastic anemia
-Undifferentiated leukemia
-Hemolytic anemia


#Microscopical examination of an enlarged cervical lymph node revealed blurring of its structure, absence of lymphoid follicles; all the microscopic fields showed cells with roundish nuclei and thin limbus of basophil cytoplasm. It is known from the clinical data that other groups of lymph nodes are also enlarged as well as spleen and liver. What disease might be suspected?
+Lymphoid leukosis
-Lymphogranulomatosis
-Lymphosarcoma
-Myeloid leukosis
-Multiple myeloma


#A 23 y.o. patient complains of weakness, temperature rise up to 38 - 400C. Objectively: liver and spleen are enlarged. Hemogram: Hb- 100 g/l, erythrocytes - 2, 9 x1012/l, leukocytes - 4,4 x109/l, thrombocytes – 48x 109/l, segmentonuclear neutrophils - 17%, lymphocytes - 15%, blast cells - 68%. All cytochemical reactions are negative. Make a hematological conclusion:
+Undifferentiated leukosis
-Chronic myeloleukosis
-Acute myeloblastic leukosis
-Acute lymphoblastic leukosis
-Acute erythromyelosis


ГЕМОСТАЗ (19)


#After pancreatic surgery the patient developed hemorrhagic syndrome with disturbed 3rd stage of blood clotting. What will be the most likely mechanism of the hemostatic disorder?
+Fibrinolysis activation
-Fibrin-stabilizing factor deficiency
-Qualitative abnormalities of fibrinogenesis
-Decrease in fibrinogen synthesis
-Decrease in prothrombin synthesis


#A 3-year-old boy with pronounced hemorrhagic syndrome has no antihemophilic globulin A (factor VIII) in the blood plasma. Hemostasis has been impaired at the following stage:
+Conversion of prothrombin to thrombin
-External mechanism of prothrombinase activation
-Conversion of fibrinogen to fibrin
-Blood clot retraction
-Internal mechanism of prothrombinase activation


#A patient visited a dentist to extract a tooth. After the tooth had been extracted, bleeding from the tooth socket continued for 15 minutes. Anamnesis states that the patient suffers from active hepatitis. What phenomenon can extend the time of hemorrhage?
+Decrease in fibrinogen content in bloods
-Decrease in albumin content in blood
-Increased activity of anticoagulation system
-Hypocalcemia
-Thrombocytopenia


#A 60-year-old man suffering from chronic hepatitis frequently observes nasal and gingival hemorrhages, spontaneous hemorrhagic rashes on the skin and mucosa. Such presentations result from:
+Decreased synthesis of prothrombin and fibrinogen
-Increased blood content of aminotransferases
-Decreased synthesis of serum albumins
-Increased blood content of macroglobulins and cryoglobulins
-Decreased blood content of cholinesterase


#After implantation of a cardiac valve a young man systematically takes indirect anticoagulants. His state was complicated by hemorrhage. What substance content has decreased in blood?
+Prothrombin
-Haptoglobin
-Heparin
-Creatine
-Ceruloplasmin


#A patient is diagnosed with hereditary coagulopathy that is characterised by factor VIII deficiency. Specify the phase of blood clotting during which coagulation will be disrupted in the given case:
+Thromboplastin formation
-Thrombin formation
-Fibrin formation
-Clot retraction


#A 28-year-old patient complains of frequent gingival haemorrhages. Blood test revealed the clotting factor II (prothrombin) deficiency. What phase of blood coagulation is impaired in this patient?
+Thrombin generation
-Vascular-platelet haemostasis
-Fibrinolysis
-Clot retraction


#A patient, who has been suffering for a long time from intestine disbacteriosis, has increased hemorrhaging caused by disruption of posttranslational modification of blood-coagulation factors II, VII, IХ, and Х in the liver. What vitamin deficiency is the cause of this condition?

-B12
-B9




#A 12-year-old patient has been admitted to a hospital for hemarthrosis of the knee joint. From early childhood he suffers from frequent bleedings. Diagnose the boy’s disease:
+Hemophilia
-Hemorrhagic vasculitis
-Hemolytic anemia
-B12 (folic acid)-deficiency anemia
-Thrombocytopenic purpura


#After a tourniquet application a patient was found to have petechial hemorrhages. The reason for it is the dysfunction of the following cells:
+Platelets
-Eosinophils
-Monocytes
-Lymphocytes
-Neutrophils


#A disaster fighter at a nuclear power plant developed hemorrhagic syndrome on the background of acute radiation disease. What is the most important factor of syndrome pathogenesis?
+Thrombocytopenia
-Vascular wall damage
-Increased activity of fibrinolysis factors
-Increased activity of anticoagulative


system factors
-Decreased activity of coagulative factors


#A tooth extraction in a patient with chronic persistent hepatitis was complicated with prolonged hemorrhage. What is the reason for the haemorrhagic syndrome?
+Decrease in thrombin production
-Increase in thromboplastin production
-Decrease in fibrin production
-Increase in fibrinogen synthesis
-Fibrinolysis intensification


#After a disease a 16-year-old boy is presenting with decreased function of protein synthesis in the liver as a result of vitamin K deficiency. This may cause disorder of:
+Blood coagulation
-Erythrocyte sedimentation rate
-Anticoagulant production
-Erythropoietin production
-Osmotic blood pressure


#A patient suffers from the hemorrhagic syndrome that shows itself in frequent nasal bleedings, posttraumatic and spontaneous intra-cutaneous and intra-articular hemorrhages. After a laboratory study a patient was diagnosed with the type B hemophilia. This disease is provoked by the deficit of the following factor of blood coagulation:
+IX
-VIII
-XI
-V
-VII


#As a result of post-translation modifications some proteins taking part in blood coagulation, particularly prothrombin, become capable of calcium binding. The following vitamin takes part in this process:
+K
-C
-A
-B1
-B2


#Patients with bile ducts obstruction suffer from inhibition of blood coagulation, bleedings as a result of low level of vitamin assimilation. What vitamin is deficient?


-D



#A 2-year-old child has got intestinal dysbacteriosis, which resulted in hemorrhagic syndrome. What is the most likely cause of hemorrhage of the child?
+Vitamin K deficiency
-Activation of tissue thromboplastin
-PP hypovitaminosis
-Fibrinogen deficiency
-Hypocalcemia


#Punctate hemorrhage was found out in the patient after application of a tourniquet. With dysfunction of what blood cells is it connected?
+Platelets
-Eosinophils
-Monocytes
-Lymphocytes
-Neutrophils


#A patient underwent a surgery for excision of a cyst on pancreas. After this he developed hemorrhagic syndrome with apparent disorder of blood coagulation. Development of this complication can be explained by:
+Activation of fibrinolytic system
-Insufficient fibrin production
-Reduced number of thrombocytes
-Activation of anticoagulation system
-Activation of Christmas factor


СЕРДЦЕ и АРИТМИИ (14)


#An athlete (long-distance runner) during a contest developed a case of acute cardiac insufficiency. This pathology resulted from:
+Cardiac volume overload
-Disrupted coronary circulation
-Direct damage to myocardium
-Pericardium pathology
-Cardiac pressure overload


#After a severe psycho-emotional stress a 48-year old patient suddenly developed acute heart pain irradiating to the left arm. Nitroglycerine relieved pain after 10 minutes. What is the leading pathogenetic mechanism of this process development?
+Spasm of coronary arteries
-Dilatation of peripheral vessels
-Obstruction of coronary vessels
-Compression of coronary vessels
-Increase in myocardial oxygen сconsumption


#The patient with acute myocardial infarction was given intravenously different solutions during 8 hours with medical dropper 1500 ml and oxygen intranasally. He died because of pulmonary edema. What caused the pulmonary edema?
+Volume overload of the left ventricular
-Decreased oncotic pressure due to hemodilution
-Allergic reaction
-Neurogenic reaction
-Inhalation of the oxygen


#ECG of a 44-year-old patient shows signs of hypertrophy of both ventricles and the right atrium. The patient was diagnosed with the tricuspid valve insufficiency. What pathogenetic variant of cardiac dysfunction is usually observed in case of such insufficiency?
+Heart overload by volume
-Heart overload by resistance
-Primary myocardial insufficiency
-Coronary insufficiency
-Cardiac tamponade


#12 hours after an acute attack of retrosternal pain a patient presented an Increase in aspartate aminotransferase activity in blood serum. What pathology is this deviation typical for?
+Myocardium infarction
-Viral hepatitis
-Collagenosis
-Diabetes mellitus
-Diabetes insipidus


#A 48-year-old is unconscious. He has a history of several syncopal episodes with convulsions. ECG shows deformed QRS complexes unconnected with P waves, atrial contraction are approximately 70/min, ventricular contractions - 25-30/min. Name the type of arrhythmia in this case:
+Complete atrioventricular block
-Second-degree atrioventricular block
-Intra-atrial block
-Intraventricular block
-First-degree atrioventricular block


#ECG of the patient shows increased duration of the QRS complex. What is the most likely cause?
+Disturbed conduction in the atrioventricular node
-Increased atrial excitability
-Increased period of ventricular excitation
-Increased atrial and ventricular excitability
-Increased period of atrial excitation


#A 15-year-old teenager complains of lack of air, general weakness, palpitations. Heart rate is 130/min.. BP is 100/60 mm Hg. ECG: QRS complex has normal shape and duration. The number of waves and ventricular complexes is equal, T wave merges with P wave. What type of cardiac arrhythmia observed in the teenager?
+Sinus tachycardia
-Sinus extrasystole
-Paroxysmal atrial tachycardia
-Atrial thrill
-Atrial fibrillation


#A 67-year-old patient complains of periodic heartache, dyspnea during light physical activities. ECG reveals extraordinary contractions of heart ventricles. Such arrhythmia is called:
+Extrasystole
-Bradycardia
-Tachycardia
-Flutter
-Fibrillation


#A patient complains of palpitations after stress. Pulse is 104/min., P-Q=0,12 seconds, there are no changes in QRS complex. What type of arrhythmia does the patient have?
+Sinus tachycardia
-Sinus bradycardia
-Sinus arrhythmia
-Ciliary arrhythmia
-Extrasystole


#Since a patient has had myocardial infarction, atria and ventricles contract independently from each other with a frequency of 60-70 and 35-40 per minute. Specify the type of heart block in this case:
+Complete atrioventricular
-Partial atrioventricular
-Sino-atrial
-Intra-atrial
-Intraventricular


#A patient who had been continuously taking drugs blocking the production of angiotensin II developed bradycardia and arrhythmia. A likely cause of these disorders is:
+Hyperkalemia
-Hypokalemia
-Hypernatremia
-Hypocalcemia
-Hypercalcemia


#A 45-year-old patient was admitted to the cardiological department. ECG data: negative P wave overlaps QRS complex, diastolic interval is prolonged after extrasystole. What type of extrasystole is it?
+Atrioventricular
-Sinus
-Atrial
-Ventricular
-Bundle-branch


#A patient who had been continuously taking drugs blocking the production of angiotensin II developed bradycardia and arrhythmia. A likely cause of these disorders is:
+Hyperkalemia
-Hypokalemia
-Hypernatremia
-Hypocalcemia
-Hypercalcemia


ГИПЕРТЕНЗИИ (15)


#After a physic trauma a woman developed periodical increases in her blood pressure accompanied by headache, palpitations and general weakness. What mechanism of hypertension development does this woman have?
+Increased arteriolar tone
-Tachycardia
-Venoconstriction
-Increased circulating blood volume
-Decreased cardiac output


#A 59-year-old man, a business manager, developed intense burning retrosternal pain that irradiates to the left arm. The pain occurred in the evening after tax audit. 15 minutes later the patient’s condition normalized. What mechanism of angina pectoris development is leading in this patient?
+Increased level of blood catecholamines
-Coronary atherosclerosis
-Intravascular aggregation of blood cells
-Coronary artery thrombosis
-Functional cardiac overload


#A 16-year-old girl fainted when she tried to quickly change her position from horizontal to vertical. What caused the loss of consciousness in the girl?
+Decreased venous return
-Increased venous return
-Increased central venous pressure
-Increased arterial pressure
-Decreased oncotic plasma pressure





#A patient has insufficient blood supply to the kidneys, which caused the development of pressure effect due to the constriction of arterial resistance vessels. This is the result of the vessels being greatly affected by the following substance:
+Angiotensin II
-Angiotensinogen
-Renin
-Catecholamines
-Norepinephrine


#A 41-year-old man has a history of recurrent attacks of heartbeats (paroxysms), profuse sweating, headaches. Examination revealed hypertension, hyperglycemia, increased basal metabolic rate, and tachycardia. These clinical presentations are typical for the following adrenal pathology:
+Hyperfunction of the medulla
-Hypofunction of the medulla
-Hyperfunction of the adrenal cortex
-Hypofunction of the adrenal cortex
-Primary aldosteronism


#A 43-year-old-patient has arterial hypertension caused by increase in cardiac output and general peripheral resistance. Specify the variant of hemodynamic development of arterial hypertension in the given case:
+Eukinetic
-Hyperkinetic
-Hypokinetic
-Combined


#A patient has been diagnosed with influenza. His condition drastically worsened after taking antipyretic drugs. He is unconscious, AP is 80/50 mm Hg, Ps is 140/m, body temperature dropped down to 35,8oC. What complication developed in this patient?
+Collapse
-Hyperthermia
-Hypovolemia
-Acidosis
-Alkalosis


#Autopsy has revealed shrunken kidneys weighing 50 mg, with fine grained surface and uniformly thinned substance. Microscopic investigation has shown the thickening of arteriole walls due to accumulation of homogeneous pink-colored masses in them. Glomeruli were undersized, sclerotic, with atrophied tubules. What disease are these changes characteristic of?
+Essential hypertension
-Pyelonephritis with kidney shrinkage
-Renal amyloidosis
-Acute glomerulonephritis
-Membranous nephropathy


#A patient with constant headaches, pain in the occipital region, tinnitus, dizziness has been admitted to the cardiology department. Objectively: AP- 180/110 mm Hg, heart rate - 95/min. Radiographically, there is a stenosis of one of the renal arteries. Hypertensive condition in this patient has been caused by the activation of the following system:
+Renin-angiotensin
-Hemostatic
-Sympathoadrenal
-Kinin
-Immune


#A month after surgical constriction of rabbit’s renal artery the considerable Increase in systematic arterial pressure was observed. What of the following regulation mechanisms caused the animal’s pressure change?
+Angiotensin-II
-Vasopressin
-Adrenaline
-Noradrenaline
-Serotonin


#An aged man had raise of arterial pressure under a stress. It was caused by activation of:
+Sympathoadrenal system
-Parasympathetic nucleus of vagus
-Functions of thyroid gland
-Functions of adrenal cortex
-Hypophysis function


#Arterial hypertension is caused by the stenosis of the renal arteries in the patient. Activation of what system is the main link in the pathogenesis of this form of hypertension?
+Renin-angiotensin
-Sympathoadrenal
-Parasympathetic
-Kallikrein-kinin
-Hypothalamic-pituitary


#Prophylactic medical examination of a 36-year old driver revealed that his BP was 150/90 mm Hg. At the end of working day he usually hears ear noise, feels slight indisposition that passes after some rest. He was diagnosed with essential hypertension. What is the leading pathogenetic mechanism in this case?
+Neurogenic
-Nephric
-Humoral
-Endocrinal
-Reflexogenic


#Arterial pressure of a surgeon who performed a long operation raised up to 140/110 mm Hg. What changes of humoral regulation could have caused the rise of arterial pressure in this case?
+Activation of sympathoadrenal system
-Activation of formation and excretion of aldosterone
-Activation of renin angiotensive system
-Activation of kallikrein kinin system
-Inhibition of sympathoadrenal system


#A 70-year old man is ill with vascular atherosclerosis of lower extremities and coronary heart disease. Examination revealed disturbance of lipid blood composition. The main factor of atherosclerosis pathogenesis is the excess of the following lipoproteins:
+Low-density lipoproteins
-Cholesterol
-High-density lipoproteins
-Intermediate density lipoproteins
-Chylomicrons


ДЫХАНИЕ (7)


#When studying the pulmonary ventilation values, the reduction of forced expiration volume has been detected. What is the likely cause of this phenomenon?
+Obstructive pulmonary disease
-Increase in respiratory volume
-Increase in functional residual lung capacity
-Increase in inspiratory reserve volume
-Increase in pulmonary residual volume


#A patient, who has been suffering from bronchial asthma for a long time, developed acute respiratory failure. What is the main mechanism of pathology development in this case?
+Obstructive disorders of pulmonary ventilation
-Decreased elasticity of the pulmonary tissue
-Restrictive disorders of pulmonary ventilation
-Pulmonary enzyme system disturbance
-Pulmonary blood supply disturbance


#An unconscious young man in the state of morphine intoxication has been delivered into an admission room. The patient’s respiration is slow and shallow due to suppression of the respiratory center. What kind of respiratory failure occurred in this case?
+Ventilatory disregulation
-Ventilatory obstruction
-Ventilatory restriction
-Perfusion
-Diffusion


#A 28-year-old patient undergoing treatment in the pulmonological department has been diagnosed with pulmonary emphysema caused by splitting of alveolar septum by tissue trypsin. The disease is caused by the congenital deficiency of the following protein:
+α1-proteinase inhibitor
-α2-macroglobulin
-Cryoglobulin
-Haptoglobin
-Transferrin


#A patient with bronchial asthma has developed acute respiratory failure. What kind of respiratory failure occurs in this case?
+Obstructive disturbance of alveolar ventilation
-Restrictive ventilatory defect
-Perfusion
-Diffusion
-Dysregulation of alveolar ventilation


#A patient with marked pneumofibrosis that developed after infiltrating pulmonary tuberculosis has been diagnosed with respiratory failure. What is its pathogenetic type?
+Restrictive
-Obstructive
-Dysregulatory
-Reflex
-Apneistic


#A 23-year-old patient has been admitted to a hospital with a cranio-cerebral injury. The patient is in a grave condition. Respiration is characterized by prolonged convulsive inspiration followed by a short expiration. What kind of respiration is it typical for?
+Apneustic
-Gasping breath
-Kussmaul’s
-Cheyne-Stokes
-Biot’s


ЖКТ (6)


#Protective function of saliva is based on several mechanisms, including the presence of enzyme that has bactericidal action and causes lysis of complex capsular polysaccharides of staphylococci and streptococci. Name this enzyme:
+Lysozyme
-Beta-glucuronidase
-Alpha-amylase
-Collagenase
-Oligo-l,6-glucosidase


#A 35-year-old man with peptic ulcer disease has undergone antrectomy. After the surgery secretion of the following gastrointestinal hormone will be disrupted the most:
+Gastrin
-Histamine
-Secretin
-Cholecystokinin
-Neurotensin


#Administration of doxycycline hydrochloride has caused an imbalance of the symbiotic intestinal microflora. Specify the kind of imbalance caused by the antibiotic therapy:
+Dysbacteriosis
-Sensibilization
-Idiosyncrasy
-Superimposed infection
-Bacteriosis


#A hospital has admitted a patient complaining of abdominal bloating, diarrhea, flatulence after eating protein foods. These signs are indicative of the impaired digestion of proteins and their increased degradation. Which of the following compounds is the product of this process?
+Indole
-Bilirubin
-Cadaverine
-Agmatine
-Putrescine


#A 30-year-old male patient with acute pancreatitis has been found to have a disorder of cavitary protein digestion. The reason for such condition can be the hyposynthesis and hyposecretion of the following enzyme:
+Tripsin
-Pepsin
-Lipase
-Dipeptidase
-Amylase


#A male patient has been diagnosed with gastric ulcer. Bacteriological examination of biopsy material from the affected part of stomach revealed small colonies of gram-negative, oxide reductase-positive flexibacteria that grew on the chocolate agar on the fifth day. Which of the following microorganisms is the most likely causative agent?
+Helicobacter pylori
-Campilobacter jejuni
-Campilobacter fetus
-Mycoplasma hominis
-Chlamydia trachomatis


ПЕЧЕНЬ (25)


#A patient present with acute attack of cholelithiasis. Laboratory examination of the patient’s faces will show the following in this case:
+Negative reaction to stercobilin
-Connective tissue
-Partially digested cellulose
-Positive reaction to stercobilin
-Starch granules


#Patients with bile duct obstruction typically present with inhibited blood clotting and develop hemorrhages due to insufficient assimilation of vitamin:
+K
-C
-D
-E
-A


#Encephalopathy has developed in a child with hemolytic disease of the newborn. What substance had increased in the child’s blood, resulting in damage to the CNS?
+Unconjugated bilirubin
-Verdohemoglobin
-Bilirubin glucuronide
-Bilirubin-albumin complex
-Bile acids


#A 46-year-old woman suffering from cholelithiasis developed jaundice. Her urine became dark yellow, while feces are light-colored. What substance will be the most increased in concentration in the blood serum in this case?
+Conjugated bilirubin
-Unconjugated bilirubin
-Biliverdine
-Mesobilirubin
-Urobilinogen


#A patient with jaundice has high total bilirubin that is mainly indirect (unconjugated), high concentration of stercobilin in the feces and urine. The level of direct (conjugated) bilirubin in the blood plasma is normal. What type of jaundice can be suspected?
+Hemolytic
-Parenchymal (hepatic)
-Gilbert’s disease
-Neonatal
-Mechanical


#Upon toxic damage of hepatic cells resulting in disruption of liver function the patient developed edemas. What changes of blood plasma are the main cause of edema development?
+Decrease in albumin content
-Increase in globulin content
-Decrease in fibrinogen content
-Increase in albumin content
-Decrease in globulin content


#A 16-year-old adolescent is diagnosed with hereditary UDP (uridine diphosphate) glucuronyl transferase deficiency. Laboratory tests revealed hyperbilirubinemia caused mostly by increased blood content of the following substance:
+Unconjugated bilirubin
-Conjugated bilirubin
-Urobilinogen
-Stercobilinogen
-Biliverdine


#A 50-year-old man, who has been suffering from chronic hepatic failure for several years, has developed ascites. What is the main mechanism of this disorder development?
+Increased pressure in portal vein system
-Decrease in albumin and globulin synthesis in liver
-Increased content of low-density and very low-density lipoproteins in blood
-Neurotoxins appearing in blood
-Increase in blood oncotic pressure


#A 43-year-old patient suffers from acute pancreatitis with disrupted common bile duct patency. What condition can develop in this case?
+Mechanical jaundice
-Hemolytic jaundice
-Hepatocellular jaundice
-Hepatic coma
-Portal hypertension


#A patient has been admitted to the contagious isolation ward with signs of jaundice caused by hepatitis virus. Which of the symptoms given below is strictly specific for hepatocellular jaundice?
+Increase in ALT, AST level
-Hyperbilirubinemia
-Bilirubinuria
-Cholemia
-Urobilinuria


#A 53-year-old male patient complains of acute pain in the right hypochondrium. Objective examination revealed scleral icterus. Laboratory tests revealed increased ALT activity, and stercobilin was not detected in the stool. What disease is characterized by these symptoms?
+Cholelithiasis
-Hemolytic jaundice
-Hepatitis
-Chronic colitis
-Chronic gastritis


#A patient has normally colored stool including a large amount of free fatty acids. The reason for this is a disturbance of the following process:
+Fat absorption
-Fat hydrolysis
-Biliary excretion
-Choleresis
-Lipase secretion


#Enzymatic jaundices are accompanied by abnormal activity of UDP-glucuronyl transferase. What compound is accumulated in blood serum in case of these pathologies?
+Unconjugated bilirubin
-Conjugated bilirubin
-Dehydrobilirubin
-Hydrobilirubin
-Choleglobin


#An infectious disease unit admitted a patient with signs of jaundice caused by hepatitis virus. Select an indicator that is specific only for parenchymatous jaundice:
+Increase in ALT and AST level
-Hyperbilirubinemia
-Bilirubinuria
-Cholaemia
-Urobilinuri


#Toxic affection of liver results in dysfunction of protein synthesis. It is usually accompanied by the following kind of dysproteinemia:
+Absolute hypoproteinemia
-Relative hypoproteinemia
-Absolute hyperproteinemia
-Relative hyperproteinemia
-Paraproteinemia


#Blood analysis of a patient with jaundice reveals conjugated bilirubinemia, increased concentration of bile acids. There is no stercobilinogen in urine. What type of jaundice is it?
+Obstructive jaundice
-Hepatocellular jaundice
-Parenchymatous jaundice
-Hemolytic jaundice
-Cythemolytic jaundice


#A patient presents with icterus of skin, sclera and mucous membranes. Blood plasma the total bilirubin is increased, stercobilin is increased in feces, urobilin is increased in urine. What type of jaundice is it?
+Haemolytic
-Gilbert’s disease
-Parenchymatous
-Obturational
-Cholestatic


#Jaundice treatment involves administration of barbiturates inducing the synthesis of UDP-glucuronyl transferase. A medicinal effect is caused by the


production of
+Direct reacting (conjugated) bilirubin
-Indirect reacting (unconjugated) bilirubin
-Biliverdin
-Protoporphyrin
-Heme


#A coprological survey revealed lightcolored feces containing drops of neutral fat. The most likely reason for this condition is the disorder of:
+Bile inflow into the bowel
-Gastric juice acidity
-Pancreatic juice secretion
-Intestinal juice secretion
-Intestinal absorption


#A viral infection has damaged cells that form walls of bile capillaries. This stimulated conditions for inflow of bile into the blood of sinusoidal capillaries. What cells are damaged?
+Hepatocytes
-Kupffer’s cells
-Ito cells
-Pit-cells
-Endotheliocytes


#Hepatitis has led to the development of hepatic failure. Mechanism of edema formation is activated by the impairment of the following liver function:
+Protein-synthetic
-Barrier
-Chologenetic
-Antitoxic
-Glycogen-synthetic


#A patient suffers from hepatocirrhosis. State of antitoxic liver function can be characterized by examination of the following substance exreted by urine:
+Hippuric acid
-Ammonium salts
-Creatinine
-Uric acid
-Amino acids


#A patient complains of frequent diarrheas, especially after consumption of rich food, weight loss. Laboratory examination revealed steatorrhea; his feces were hypocholic. What might have caused such condition?
+Obturation of biliary tracts
-Inflammation of mucous membrane of small intestine
-Lack of pancreatic lipase
-Lack of pancreatic phospholipase
-Unbalanced diet


#After a disease a 16-year-old boy is presenting with decreased function of protein synthesis in the liver as a result of vitamin K deficiency. This may cause disorder of:
+Blood coagulation
-Erythrocyte sedimentation rate
-Anticoagulant production
-Erythropoietin production
-Osmotic blood pressure


#A 48-year-old patient was admitted to the hospital with complaints about weakness, irritability, sleep disturbance. Objectively: skin and sclera are of yellow color. In blood: increased concentration of total bilirubin with prevailing direct bilirubin. The feces are acholic. The urine is dark (contains bile pigments). What type of jaundice is it?
+Mechanic
-Haemolytic
-Parenchymatous
-Gilbert’s syndrome
-Crigler-Najjar syndrome


ПОЧКИ (30)


#A 50-year-old patient during examination presents with glucosuria and blood glucose of 3.0 mmol/l, which are the most likely to be caused by:
+Renal disorder
-Essential hypertension
-Myxedema
-Pellagra
-Diabetes insipidus


#A 38-year-old man, who has been suffering from systemic lupus erythematosus for 3 years, developed diffuse renal lesions accompanied by massive edemas, marked proteinuria, hyperlipidemia and dysproteinemia. What is the most likely mechanism of proteinuria development in this case?
+Autoimmune damage to the nephrons
-Ischemic damage to the tubules
-Morbid affection of the urinary tracts
-Inflammatory damage to the nephrons
-Increased blood proteins


#A lab rat has subcutaneously received mercury (II) chloride in the amount of 5 mg/kg. 24 hours later the plasma creatinine concentration increased several times. What mechanism of retention azotemia is observed in this case?
+Decreased glomerulus filtration
-Increased creatinine production in the renal tubules
-Increased glomerular filtration
-Increased creatinine production in the muscles
-Increased creatinine reabsorption


#A traumatology unit received a patient with crushed muscular tissue. What biochemical indicator of urine will be raised in this case?
+Creatinine
-Glucose
-Uric acid
-Total lipids
-Mineral salts


#A 40-year-old woman was diagnosed with glomerulonephritis based on her clinical symptoms and the results of urine analysis. Anamnesis states chronic tonsillitis. What microorganisms are the most likely cause for the kidney damage in this case?
+Streptococci
-Staphylococci
-Escherichia
-Mycoplasma
-Meningococci


#Glomerular filtration of a person, who has been starving for a long time, has increased by 20%. The most likely cause of filtration changes in the given conditions is:
+Decrease in blood plasma oncotic pressure
-Increase in systemic blood pressure
-Increase in renal filter permeability
-Increase in filtration factor
-Increase in renal plasma flow


#A 12-year-old child has developed nephritic syndrome (proteinuria, hematuria, cylindruria) 2 weeks after the case of tonsillitis, which is a sign of affected glomerular basement membrane in the kidneys. What mechanism is the most likely to cause the basement membrane damage?
+Immune complex
-Granulomatous
-Antibody-mediated
-Reaginic
-Cytotoxic


#Urine analysis has shown high levels of protein and erythrocytes in urine. This can be caused by the following:
+Renal filter permeability
-Effective filter pressure
-Hydrostatic blood pressure in glomerular capillaries
-Hydrostatic primary urine pressure in capsule
-Oncotic pressure of blood plasma


#Due to the use of poor-quality measles vaccine for preventive vaccination, a 1-year-old child developed an autoimmune renal injury. The urine was found to contain macromolecular proteins. What process of urine formation was disturbed?
+Filtration
-Reabsorption
-Secretion
-Reabsorption and secretion
-Secretion and filtration


#Urine analysis has shown high levels of protein and erythrocytes in urine. This can be caused by the following:
+Renal filter permeability
-Effective filter pressure
-Hydrostatic blood pressure in glomerular capillaries
-Hydrostatic primary urine pressure in capsule
-Oncotic pressure of blood plasma


#A patient has insufficient blood supply to the kidneys, which has caused the development of pressor effect due to the constriction of arterial resistance vessels. This is the result of the vessels being greately affected by the following substance:
+Angiotensin II
-Angiotensinogen
-Renin
-Catecholamines
-Norepinephrine


#14 days after quinsy a 15-year-old child presented with morning facial swelling, high blood pressure, "meat slops"urine. Immunohistological study of a renal biopsy sample revealed deposition of immune complexes on the basement membranes of the capillaries and in the glomerular mesangium. What disease developed in the patient?
+Acute glomerulonephritis
-Acute interstitial nephritis
-Lipoid nephrosis
-Acute pyelonephritis
-Necrotizing nephrosis


#A month after surgical constriction of rabbit’s renal artery the considerable Increase in systematic arterial pressure was observed. What of the following regulation mechanisms caused the animal’s pressure change?
+Angiotensin-II
-Vasopressin
-Adrenaline
-Noradrenaline
-Serotonin


#A child has an acute renal failure. What biochemical factor found in saliva can confirm this diagnosis?
+Increase in urea concentration
-Increase in glucose concentration
-Decrease in glucose concentration
-Increase in concentration of higher fatty acids
-Decrease in nucleic acid concentration


#A patient with a history of chronic glomerulonephritis presents with azotemia, oliguria, hypo- and isosthenuria, proteinuria. What is the leading factor in the pathogenesis of these symptoms development under chronic renal failure?
+Decreased mass of active nephrons
-Intensification of glomerular filtration
-Tubular hyposecretion
-Disturbed permeability of glomerular membranes
-Intensification of sodium reabsorption


#A patient has been diagnosed with acute glomerulonephritis that developed after he had had streptococcal infection. It is most likely that the affection of basal glomerular membrane is caused by an allergic reaction of the following type:
+Immune complex
-Anaphylactic
-Cytotoxic
-Delayed
-Stimulating


#A patient is 44 years old. Laboratory examination of his blood revealed that content of proteins in plasma was 40 g/l. What influence will be exerted on the transcapillary water metabolism?
+Filtration will be increased, reabsorption- decreased
-Both filtration and reabsorption will be increased
-Both filtration and reabsorption will be decreased
-Filtration will be decreased, reabsorption- increased
-Metabolism will stay unchanged


#A patient with massive burns developed acute renal insufficiency characterized by a significant and rapid deceleration of glomerular filtration. What is the mechanism of its development?
+Reduction of renal blood flow
-Damage of glomerular filter
-Reduction of functioning nephron number
-Rise of pressure of tubular fluid
-Renal artery embolism


#A 30- year old woman has face edema. Examination revealed proteinuria (5,87 g/l), hypoproteinemia, dysproteinemia, hyperlipidemia. What condition is the set of these symptoms typical for?
+Nephrotic syndrome
-Nephritic syndrome
-Chronic pyelonephritis
-Acute renal failure
-Chronic renal failure


#Two weeks after lacunar tonsillitis a 20- year-old man started complaining about general weakness, lower eyelid edemata. After examination the patient was diagnosed with acute glomerulonephritis. What are the most likely pathological changes in the urine formula?
+Proteinuria
-Cylindruria
-Presence of fresh erythrocytes
-Pyuria
-Natriuria


#As a result of continuous starvation the glomerular filtration rate has increased by 20%. The most probable cause of the glomerular filtration alteration under the mentioned conditions is:
+Decrease in the oncotic pressure of blood plasma
-Increase in the systemic arterial pressure
-Increase in the permeability of the renal filter
-Increase in the filtartion coefficient
-Increase in the renal blood flow


#A patient with massive burns developed acute renal insufficiency characterized by a significant and rapid deceleration of glomerular filtration. What is the mechanism of its development?
+Reduction of renal blood flow
-Damage of glomerular filter
-Reduction of functioning nephron number
-Rise of pressure of tubular fluid
-Renal artery embolism


#An electron micrograph of a kidney fragment presents an afferent arteriole. Under its endothelium some big cells can be seen that contain secretory granules. What type of cells is it?
+Juxtaglomerular
-Mesangeal
-Smooth muscle cells
-Juxtavascular
-Interstitial


#Examination of a 43 y.o. anechoic patient revealed anemia symptoms. What is the cause of these symptoms?
+Reduced synthesis of erythropoietins
-Enhanced destruction of erythrocytes
-Iron deficit
-Vitamin B12 deficit
-Folic acid deficit


#Violation of safety rules resulted in calomel intoxication. Two days later the daily diuresis was 620 ml. A patient experienced headache, vomiting, convulsions, dyspnea, moist rales in lungs. What pathology is it?
+Acute renal insufficiency
-Chronic renal insufficiency
-Uremic coma
-Glomerulonephritis
-Pyelonephritis


#On the 6th day of treatment a patient with acute renal insufficiency developed polyuria. Diuresis intensification at the beginning of polyuria stage of acute renal insufficiency is caused by:
+Renewal of filtration in nephrons
-Volume expansion of circulating blood
-Growth of natriuretic factor
-Reduction of aldosterone content in plasma
-Reduction of vasopressin content in plasma


#A 65- year old man suffering from gout complains of kidney pain. Ultrasound examination revealed renal calculi. The most probable cause of calculi formation is the strengthened concentration of the following substance:
+Uric acid
-Cholesterol
-Bilirubin
-Urea
-Cystine


#A 30 -year old woman has face edema. Examination revealed proteinuria (5,87 g/l), hypoproteinemia, dysproteinemia, hyperlipidemia. What condition is the set of these symptom typical for?
+Nephrotic syndrome
-Nephritic syndrome
-Chronic pyelonephritis
-Acute renal failure
-Chronic renal failure.


#A patient is 44 years old. Laboratory examination of his blood revealed that content of proteins in plasma was 40 g/l. What influence will be exerted on the trans-capillary water exchange?
+Filtration will be increased, reabsorption- decreased
-Both filtration and reabsorption will be increased
-Both filtration and reabsorption will be decreased
-Filtration will be decreased, reabsorption- increased
-Exchange will stay unchanged


#A driver who got a trauma in a road accident and is shocked has reduction of daily urinary output down to 300 ml. What is the main pathogenetic factor of such diuresis change?
+Drop of arterial pressure
-Drop of oncotic blood pressure
-Increased vascular permeability
-Decreased number of functioning glomerules
-Secondary hyperaldosteronism


ЭНДОКРИННАЯ СИСТЕМА (48)


#During removal of the hyperplastic thyroid gland of a 47-year-old woman, the parathyroid gland was damaged. One month after the surgery the patient developed signs of hypoparathyroidism: frequent convulsions, hyperreflexia, laryngospasm. What is the most likely cause of the patient’s condition?
+Hypocalcemia
-Hyponatremia
-Hypophosphatemia
-Hyperchlorhydria
-Hyperkalemia


#On examination the patient presents with hirsutism, moon-shaped face, stretch marks on the abdomen. BP is 190/100 mm Hg, blood glucose is 17.6 mmol/L. What pathology is such clinical presentation characteristic of?
+Adrenocortical hyperfunction
-Hyperthyroidism
-Hyperfunction of the insular apparatus
-Gonadal hypofunction
-Hypothyroidism


#A 40-year-old woman suffers from Cushing’s disease - steroid diabetes. On biochemical examination she has hyperglycemia and hypochloremia. What process activates in the first place in such patients?
+Gluconeogenesis
-Glycogenolysis
-Glucose reabsorption
-Glycolysis
-Glucose transport to cells


#Corticosteroid hormones regulate the adaptation processes of the body as a whole to environmental changes and ensure the maintenance of internal homeostasis. What hormone activates the hypothalamo-pituitary-adrenal axis?
+Corticoliberin
-Somatoliberin
-Corticostatin
-Somatostatin
-Thyroliberin


#Autopsy of a 40-year-old woman, who died of cerebral hemorrhage during hypertension crisis, revealed: upper body obesity, hypertrichosis, hirsutism, stretchmarks on the skin of thighs and abdomen. Pituitary basophil adenoma is detected in the anterior lobe. What diagnosis is the most likely?
+Cushing’s disease
-Essential hypertension
-Alimentary obesity
-Simmonds’ disease
-Hypothalamic obesity
-Prednisolone


#Examination of a 42-year-old patient revealed a tumor of adenohypophysis. Objectively: the patient’s weight is 117 kg, he has moon-like hyperemic face, red-blue striae of skin distension on his belly. Osteoporosis and muscle dystrophy are present. AP is 210/140 mm Hg. What is the most probable diagnosis?
+Cushing’s disease
-Cushing’s syndrome
-Conn’s disease
-Diabetes mellitus
-Essential hypertension


#In the course of an experiment adenohypophysis of an animal has been removed. The resulting atrophy of thyroid gland and adrenal cortex has been caused by deficiency of the following hormone:
+Tropic hormones
-Thyroid hormones
-Somatotropin
-Cortisol
-Thyroxin


#A man has a considerable decrease in diuresis as a result of 1,5 l blood loss. The primary cause of such diuresis disorder is the hypersecretion of the following hormone:
+Vasopressin
-Corticotropin
-Natriuretic
-Cortisol
-Parathormone


#A 20- year old patient complains of morbid thirst and increased diuresis (up to 10 l daily). Glucose concentration in blood is normal but it is absent in urine. The patient has been diagnosed with diabetes insipidus. What hormonal drug is the most appropriate for management of this disorder?
+Vasopressin
-Cortisol
-Thyroxin
-Oxytocin
-Insulin


#The secretion of which hypophysial hormones will be inhibited after taking the oral contraceptives containing sex hormones?
+Gonadotropic hormone
-Vasopressin
-Thyrotrophic hormone
-Somatotropic hormone
-Oxytocin


#A 32-year-old patient consulted a doctor about the absence of lactation after parturition. Such disorder might be explained by the deficit of the following hormone:
+Prolactin
-Somatotropin
-Vasopressin
-Thyrocalcitonin
-Glucagon


#A patient complains of polyruria (7 liters per day) and polydipsia. Examination reveals no disorders of carbohydrate metabolism. These abnormalities might be caused by the dysfunction of the following endocrine gland:
+Neurohypophysis
-Adenohypophysis
-Islets of Langerhans (pancreatic islets)
-Adrenal cortex
-Adrenal medulla


#Roentgenological examination of skull base bones revealed enlargement of sellar cavity, thinning of anterior clinoid processes, destruction of different parts, destruction of different parts of sella turcica. Such bone destruction might be caused by a tumour of the following wndocrinous gland:
+Hypophysis
-Epiphysis
-Thymus gland
-Adrenal glands
-Thyroid gland


#Examination of a patient revealed overgrowth of facial bones and soft tissues, tongue enlargement, wide interdental spaces in the enlarged dental arch. What changes of the hormonal secretion are the most likely?
+Hypersecretion of the somatotropic hormone
-Hyposecretion of the somatotropic hormone
-Hypersecretion of insulin
-Hyposecretion of thyroxin
-Hyposecretion of insulin


#A 32-year-old patient consulted a doctor about the absence of lactation after parturition. Such disorder might be explained by the deficit of the following hormone:
+Prolactin
-Somatotropin
-Vasopressin
-Thyrocalcitonin
-Glucagon


#A patient has a decreased vasopressin synthesis that causes polyuria and as a result of it evident organism dehydration. What is the mechanism of polyuria development?
+Reduced tubular reabsorption of water
-Reduced tubular reabsorption of Na ions
-Reduced tubular reabsorption of protein
-Reduced glucose reabsorption
-Acceleration of glomerular filtration


#Osmotic pressure of a man’s blood plasma is 350 mosmole/l (standard pressure is 300 mosmole/l). First of all it will result in high secretion of the following hormone:
+Vasopressin
-Aldosteron
-Cortisol
-Adrenocorticotropin
-Natriuretic


#Intake of oral contraceptives containing sex hormones inhibits secretion of the hypophysiae hormones. Secretion of which of the indicated hormones is inhibited while taking oral contraceptives with sex hormones?
+Follicle-stimulating
-Vasopressin
-Thyrotropic
-Somatotropic
-Oxytocin


#A 46-year-old patient has complained of headache, fatigue, thirst, pains in the spine and joints for the last 2 years. Clinically observed disproportional enlargement of hands, feet, nose, superciliary arches. He notes that he needed to buy bigger shoes three times. What is the main reason of such disproportional enlargement of different parts of the body?
+Cartilaginous tissue proliferation under growth hormone influence
-Increased sensitivity of the tissues to growth hormone
-Joints dystrophy development
-Increased sensitivity of the tissues to insulin
-Joints chronic inflammation development


#Prolonged treatment of hypothyroidism has caused general dystrophy, dental caries, tachycardia, tremor of extremities. What drug is the cause of these side effects?
+L-thyroxin
-Humulin (Human insulin)
-Parathyreoidinum
-Thyrocalcitonin


#A patient with signs of osteoporosis and urolithiasis has been admitted to an endocrinology department. Blood test revealed hypercalcemia and hypophosphatemia. These changes are associated with abnormal synthesis of the following hormone:
+Parathyroid hormone
-Calcitonin
-Cortisol
-Aldosterone
-Calcitriol


#A 4 -year old child with hereditary renal lesion has signs of rickets, vitamin D concentration in blood is normal. What is the most probable cause of rickets development?
+Impaired synthesis of calcitriol
-Increased excretion of calcium
-Hyperfunction of parathyroid glands
-Hypofunction of parathyroid glands
-Lack of calcium in food


#A 5-month-old boy was hospitalized for tonic convulsions. He has a lifetime history of this disease. Examination revealed coarse hair, thinned and fragile nails, pale and dry skin. In blood: calcium - 1,5 mmol/l, phosphor - 1,9 mmol/l. These changes are associated with:
+Hypoparathyroidism
-Hyperparathyroidism
-Hyperaldosteronism
-Hypoaldosteronism
-Hypothyroidism


#Parodontitis is treated with calcium preparations and a hormone that stimulates tooth mineralization and inhibits tissue resorption. What hormone is it?
+Calcitonin
-Parathormone
-Adrenalin
-Aldosterone
-Thyroxine


#A child has abnormal formation of tooth enamel and dentin as a result of low concentration of calcium ions in blood. Such abnormalities might be caused by deficiency of the following hormone:
+Parathormone
-Thyrocalcitonin
-Thyroxin
-Somatotropic hormone
-Triiodothyronine


#A 46-year-old patient suffering from the diffuse toxic goiter underwent resection of the thyroid gland. After the surgery the patient presents with appetite loss, dyspepsia, increased neuromuscular excitement. The body weight remained unchanged. Body temperature is normal. Which of the following has caused such a condition in this patient?
+Reduced production of parathormone
-Increased production of thyroxin
-Increased production of calcitonin
-Increased production of thyroliberin
-Reduced production of thyroxin


#A 2 y.o. child has convulsions as a result of lowered concentration of calcium ions in blood plasma. It is caused by reduced function of:
+Parathyroid glands
-Hypophysis
-Adrenal cortex
-Pineal gland
-Thymus


#Kidneys of a man under examination show increased resorbtion of calcium ions and decreased resorbtion of phosphate ions. What hormone causes this phenomenon?
+Parathormone
-Thyrocalcitonin
-Hormonal form D3
-Aldosterone
-Vasopressin


#A 41-year-old man has a history of recurrent attacks of heartbeats (paroxysms), profuse sweating, headaches. Examination revealed hypertension, hyperglycemia, increased basal metabolic rate, and tachycardia. These clinical presentations are typical for the following adrenal pathology:
+Hyperfunction of the medulla
-Hypofunction of the medulla
-Hyperfunction of the adrenal cortex
-Hypofunction of the adrenal cortex
-Primary aldosteronism


#There are cortical and medullary substances separated by connective tissue layer in the endocrine gland specimen. Parenchyma cells make up three zones in cortical substance, with rounded masses in the superficial zone, parallel chords in the middle one, reticular structure of cell chords in the deep one. What gland is it?
+Adrenal gland
-Thyroid gland
-Pituitary gland
-Epiphysis
-Hypothalamus


#A 19-year-old male was found to have an elevated level of potassium in the secondary urine. These changes might have been caused by the increase in the following hormone level:
+Aldosterone
-Oxytocin
-Adrenaline
-Glucagon
-Testosterone


#A 38-year-old female patient complains of general weakness, cardiac pain, increased appetite, no menstruation. Objectively: the height is 166 cm, weight 108 kg, the patient has moon-shaped face, subcutaneous fat is deposited mainly in the upper body, torso and hips. There are also blood-red streaks. Ps- 62/min, AP- 160/105 mm Hg. Which of the following diseases is the described pattern of obesity most typical for?
+Cushing pituitary basophilism
-Alimentary obesity
-Myxedema
-Insulinoma
-Babinski-Frohlich syndrome


#A 44- year old woman complains of general weakness, heart pain, significant Increase in body weight. Objectively: moon face, hirsutism, AP is 165/100 mm Hg, height - 164 cm, weight - 103 kg; the fat is mostly accumulated on her neck, thoracic girdle, belly. What is the main pathogenetic mechanism of obesity?
+Increased production of glucocorticoids
-Reduced production of thyroid hormones
-Increased insulin production
-Reduced glucagon production
-Increased mineralocorticoid production


#A concentrated solution of sodium chloride was intravenously injected to an animal. This caused decreased reabsorption of sodium ions in the renal tubules. It is the result of the following changes of hormonal secretion:
+Aldosterone reduction
-Aldosterone increase
-Vasopressin reduction
-Vasopressin increase
-Reduction of atrial natriuretic factor


#A patient ill with neurodermatitis has been taking prednisolone for a long time. Examination revealed high rate of sugar in his blood. This complication is caused by the drug influence upon the following link of carbohydrate metabolism:
+Gluconeogenesis activation
-Glycogenogenesis activation
-Intensification of glucose absorption in the bowels
-Inhibition of glycogen synthesis
-Activation of insulin decomposition


#A patient suffering from pheochromocytoma complains of thirst, dry mouth, hunger. Blood test for sugar revealed hyperglycemia. What type of hyperglycemia is it?
+Adrenal
-Hypercorticoid
-Alimentary
-Somatotropic
-Hypoinsulinemic


#A middle-aged man went to a foreign country because he had been offered a job there. However he had been unemployed for quite a long time. What endocrine glands were exhausted most of all in this man?
+Adrenal glands
-Parathyroid glands
-Seminal glands
-Substernal gland
-Thyroid gland


#A patient with android-type obesity had been suffering from arterial hypertension, hyperglycemia, glycosuria for a long time and died from the cerebral haemorrhage. Pathologic examination revealed pituitary basophil adenoma, adrenal cortex hyperplasia. What is the most likely diagnosis?
+Itsenko-Cushing’s syndrome
-Diabetes mellitus
-Acromegalia
-Pituitary nanism
-Adiposogenital dystrophy


#To prevent the transplant rejection after organ transplantation it is required to administer hormonotherapy for the purpose of immunosuppression. What hormones are used for this purpose?
+Glucocorticoids
-Mineralocorticoids
-Sexual hormones
-Catecholamines
-Thyroid


#A 59- year old patient is a plant manager. After the tax inspection of his plant he felt intense pain behind his breastbone irradiating to his left arm.15 minutes later his condition came to normal. Which of the possible mechanisms of stenocardia development is the leading in this case?
+High catecholamine concentration in blood
-Coronary atherosclerosis
-Intravascular aggregation of blood corpuscles
-Coronary thrombosis
-Functional heart overload


#Examination of a patient revealed hyperkaliemia and hyponatremia. Low secretion of which hormone may cause such changes?
+Aldosteron
-Vasopressin
-Cortisol
-Parathormone
-Natriuretic


#A 40 y.o. patient complains of intensive heartbeats, sweating, nausea, vision impairment, arm tremor, hypertension. From his anamnesis: 2 years ago he was diagnosed with pheochromocytoma. Hyperproduction of what hormones causes the given pathology?
+Catecholamines
-Aldosterone
-Glucocorticoids
-ACTH
-Thyroid hormones


#Continious taking of a drug can result in osteoporosis, erosion of stomach mucous membrane, hypokaliemia, retention of sodium and water, reduced content of corticotropin in blood. Name this drug:
+Prednisolone
-Hydrochlorothiazide
-Digoxin
-Indometacin
-Reserpine


#A 30-year-old woman developed the signs of virilism (body hair growth, balding temples, disturbed menstrual cycle). What hormone can cause this condition when hyperproduced?
+Relaxin
-Oxytocin
-Prolactin
-Estriol
-Testosterone


#Parents of a 10 y.o. boy consulted a doctor about extension of hair-covering, growth of beard and moustache, low voice. Intensified secretion of which hormone must be assumed?
+Of testosterone
-Of somatotropin
-Of oestrogen
-Of progesterone
-Of cortisol


#A girl is diagnosed with adrenogenital syndrome (pseudohermaphroditism). This pathology was caused by hypersecretion of the following adrenal hormone:
+Androgen
-Estrogen
-Aldosterone
-Cortisol
-Adrenalin


#A female patient presents with endocrine dysfunction of follicular cells of the ovarian follicles resulting from an inflammation. The synthesis of the following hormone will be inhibited:
+Estrogen
-Progesterone
-Lutropin
-Follicle stimulating hormone
-Follistatine


#A 30-year-old female exhibits signs of virilism (growth of body hair, balding temples, menstrual disorders). This condition can be caused by the overproduction of the following hormone:
+Testosterone
-Oestriol
-Relaxin
-Oxytocin
-Prolactin


НЕРВНАЯ СИСТЕМА (16)


#64-year-old woman presents with disturbed fine motor function of her fingers, marked muscle rigidity and tremor. The neurologist diagnosed her with Parkinson's disease. What brain structures are damaged resulting in this disease?
+Substantia niagra
-Red nuclei
-Reticular formation
-Thalamus
-Cerebellum


#A patient got a gunshot wound of hip which damaged the sciatic nerve. Any impact on the affected limb causes severe, excruciating pain. What mechanism of pain is most likely in this case?
+Causalgic
-Reflex
-Phantom
-Endorphin hypofunction
-Enkephalin hypofunction


#After the traumatic tooth extraction, a patient is complaining of acute, dull, poorly-localized pain in gingiva, body temperature rises up to 37, 5oC. The patient has been diagnosed with alveolitis. Specify the kind of pain in this patient:
+Protopathic
-Epicritic
-Visceral
-Heterotopic
-Phantom


#After amputating the upper extremity, a patient had a bad pain in it. Which mechanism of the pain feeling formation is more possible in this case?
+Phantom
-Reflex
-Hyposecretion of endorphin
-Hypersecretion of endorphin
-Hyposecretion of encephalin


#After a road accident a patient was diagnosed with a trauma of the brachium with incomplete rupture of the median nerve. Besides, disorders of the motor and sense functions, the patient complains of sharp, stinging, intolerable pain. What kind of pain is it?
+Causalgia
-Somatic
-Projective
-Reflected
-Phantom


#A patient, complaining of pain in the region of the left scapula, was diagnosed with myocardial infarction. What kind of pain is it?
+Irradiating (reflected)
-Phantom.
-Visceral
-Early (protopathic)
-Late (epycritical)


#A patient after hypertension stroke does not have voluntary movements in his right arm and leg with the increased muscle tone in these extremites. What type of disfunction of nervous system is it?
+Central paralysis
-Peripheral paralysis
-Peripheral paresis
-Reflex paresis
-Central paresis


#A 68-year-old woman cannot move her upper and lower right extremities after stroke. Muscle tone of these extremities and reflexes are increased. There are pathological reflexes. What form of the paralysis is it?
+Hemiplegia
-Paraplegia
-Tetraplegia
-Monoplegia
-Dissociation


#A 28 -year old man had a gunshot wound of shin that resulted in an ulcer from the side of the injury. What is the main factor of neurodystrophy pathogenesis in this case?
+Traumatization of peripheral nerve
-Psychical stress
-Microcirculation disturbance
-Infection
-Tissue damage


#An experimental rat with extremity paralysis has no tendon and cutaneous reflexes, muscle tone is decreased, but muscles of the affected extremity maintain their ability to react with excitation to the direct action of continious current. What type of paralysis is it?
+Flaccid peripheral
-Flaccid central
-Spastic peripheral
-Spastic central
-Extrapyramidal


#Increase in blood pressure and rapid pulse are noticed in a sportsman at the start before competitions. Influence of which part of the CNS can above-mentioned changes be explained?
+Cortex of hemispheares
-Medulla oblongata
-Mesencephalon
-Diencephalons
-Hypothalamus


#A patient had hemiplegia after insult. What disorder is observed in this case?
+Movement
-Taste
-Balance
-Vision
-Hearing


#In an experiment a part of the brain of an animal has been removed. As a result, asynergia, atonia, and dysmetria developed. What part of the brain was removed?
+CerebeIlum
-Mesencephalon
-Frontal part
-Parietal part
-Reticular formation


#Patients, suffering from epilepsy, have specific centers in brain cortex. They function as a pathological determinant. What is the underlying mechanism of the formation of these centers?
+Formation of generator of pathological exaltation
-Protective inhibition
-Phenomenon of fallout
-Overexcitation
-Parabiosis

-A 68-year-old woman had a stroke which resulted in the absence of voluntary movements of both upper and lower right extremities. The tonus of muscles and reflexes is increased. Pathological reflexes are observed. What kind of paralysis is it?
+Tetraplegic.
-Central
-Paraplegic
-Peripherie
-Reflex


#After a hypertonic crisis a patient presents with lacking spontaneous movements in his right arm and leg, muscle tone of these extremities is increased. What type of motor dysfunction has developed in this case?
+Central paresis
-Peripheral paralysis
-Central paralysis
-Peripheral paresis
-Reflectory paresis;


ЭКСТРЕМАЛЬНЫЕ СОСТОЯНИЯ (3)


#As a result of a trauma a patient has developed traumatic shock that led to the following disorders: AP is 140/90 mm Hg, Ps is 120 bpm. The patient is fussy, talkative, pale. Such state relates to the following shock phase:
+Erectile
-Latent period
-Terminal
-Torpid


#After a road accident a victim has tachycardia, arterial blood pressure 130/90 mm Hg, tachypnoe, the skin is pale and dry, excitation of central nervous system is observed. What shock stage is the patient most likely in?
+Erectile
-Terminal
-Torpid
-Preshock (compensation stage)
-Agony


#A 27-year-old patient with injury to the neck has lost approximately 30% of the blood volume. The patient’s condition is severe: blood pressure is 60/40 mm Hg, heart rate is 140/min., respiratory rate is 30/min., conscious. Characterize the condition of the patient’s circulatory system:
+Hypovolemic shock
-Cardiogenic shock
-Collapse
-Coma
-Arterial hypertension


У альпiнiста, що пiднявся на висоту 5200 м, розвинувся газовий алкалоз. Що є причиною його розвитку?
+Гiпервентиляцiя легенiв
-Гiповентиляцiя легенiв
-Гiпероксемiя
-Гiпоксемiя
-Зниження температури навколишнього середовища


Крива дисоцiацiї оксигемоглобiну змiщена вправо. Якi змiни в органiзмi людини можуть бути причиною цього?
+Гiпертермiя
-Збiльшення концентрацiї 2,3дифосфоглiцерату в еритроцитах
-Алкалоз
-Гiпокапнiя
-Гiпоксемiя


В пiдводному човнi пiд час занурення порушилася система подачi кисню. У пiдводникiв збiльшилися частота дихання I серцевих скорочень. Який вид гiпоксiї розвинувся у пiдводникiв?
+Гiпоксична
-Кров’яна
-Серцево-судинна
-Тканинна
-Дихальна


Вивчення органiзму мешканця Памiру виявило високий рiвень основного обмiну, розширення грудної клiтки, зростання кисневої ємностi кровi за рахунок збiльшення еритроцитiв, високий вмiст гемоглобiну. До якого адаптивного екологiчного типу слiд вiднести цього чоловiка?
+Гiрський
-Пустельний
-Арктичний
-Тропiчний
-Субтропiчний


Пiсля ремонту автомобiля в закритому примiщеннi при працюючому двигунi у чоловiка з’явилися задишка, запаморочення, акроцiаноз, частота дихання 24-26/хв. Газовий склад кровi: pO2 - 60 мм рт.ст., pCO2 - 30 мм рт.ст.; у кровi наявний карбоксигемоглобiн. Про який вид гiпоксiї можна думати?
+Гемiчна
-Гiпоксична
-Циркуляторна
-Респiраторна
-Тканинна


Робочий комунальної служби спустився в каналiзацiйний колодязь без засобiв захисту i через деякий час знепритомнiв. Лiкарями швидкої допомоги дiагностовано отруєння сiрководнем. Який вид гiпоксiї при цьому розвинувся?
+Гемiчний
-Перевантажувальний
-Тканинний
-Циркуляторний
-Респiраторний


У хворого 23-х рокiв в результатi черепно-мозкової травми виник набряк мозку. Який механiзм пошкодження клiтин безпосередньо призвiв до набряку мозку?
+Електролiтно-осмотичний
-Лiпiдний
-Кальцiєвий
-Ацидотичний
-Протеїновий


У людини внаслiдок тривалого перебування у горах на висотi 3000 м над рiвнем моря збiльшилась киснева ємкiсть кровi. Безпосередньою причиною цього є посилене утворення в органiзмi:
+Еритропоетинiв
-Лейкопоетинiв
-Карбгемоглобiну
-Катехоламiнiв
-2,3-дифосфоглiцерату


У дорослої людини системний артерiальний тиск знизився з 120/70 до 90/50 мм рт.ст., що викликало рефлекторне звуження судин. У якому з зазначених органiв звуження судин буде найбiльшим?
+Кишечник
-Серце
-Головний мозок
-Нирки
-Наднирники


Вагiтна жiнка пiд час пологiв втратила близько 800 мл кровi. Спостерiгається тахiкардiя, артерiальний тиск 100/70 мм рт.ст., тахiпное до 28/хв. Який тип гiпоксiї розвивається первинно в такiй клiнiчнiй ситуацiї ?
+Кров’яна
-Серцево-судинна
-Змiшана
-Тканинна
-Дихальна


Пiд час бiгу на короткi дистанцiї у нетренованої людини виникає м’язова гiпоксiя. До накопичення якого метаболiту в м’язах це призводить?
+Лактат
-Кетоновi тiла
-Ацетил-КоА
-Глюкозо-6-фосфат
-Оксалоацетат


До лiкарнi доставлено хворого з отруєнням iнсектицидом - ротеноном. Яка дiлянка мiтохондрiального ланцюга переносу електронiв блокується цiєю речовиною?
+НАДН-коензим Q-редуктаза
-Сукцинат-коензим Q-редуктаза
-Коензим Q-цитохром С-редуктаза
-Цитохром С-оксидаза
-АТФ-синтетаза


До реанiмацiйного вiддiлення в тяжкому станi, без свiдомостi надiйшов пацiєнт. Дiагностовано передозування барбiтуратiв, якi спричинили феномен тканинної гiпоксiї. На якому рiвнi вiдбулося блокування електронного транспорту?
+НАДН-коензим-Q-редуктаза
-Цитохромоксидаза
-Цитохром b - цитохром c 1
-Убiхiнон
-АТФ-синтаза


У людей, якi постiйно проживають в гiрськiй мiсцевостi, адаптацiя до "кисневого голодування"здiйснюється шляхом полегшеної вiддачi кисню гемоглобiном внаслiдок:
+Пiдвищеного утворення 2,3дифосфоглiцерату в еритроцитах
-Зниженого утворення 2,3дифосфоглiцерату в еритроцитах
-Зростання парцiального тиску CO 2
-Пiдвищення pH кровi
-Зниження температури кровi


Для людини iснує суворе обмеження в часi перебування на висотi понад 800 метрiв над рiвнем моря без кисневих балонiв. Що є лiмiтуючим фактором для життя в даному випадку?
+Парцiальний тиск кисню в повiтрi
-Рiвень ультрафiолетового опромiнення
-Рiвень вологостi
-Температура
-Сила земного тяжiння


Видiляють декiлька груп молекулярних механiзмiв, якi мають важливе значення в патогенезi ушкодження клiтин, що сприяє розвитку патологiї. Якi процеси забезпечують протеїновi механiзми ушкодження?
+Пригнiчення ферментiв
-Перекисне окиснення лiпiдiв
-Активацiя фосфолiпаз
-Осмотичне розтягнення мембран
-Ацидоз


Хвора 56-ти рокiв тривалий час хворiє на тиреотоксикоз. Який тип гiпоксiї може розвинутися у цiєї хворої?
+Тканинна
-Гемiчна
-Циркуляторна
-Дихальна
-Змiшана


Цiанiстий калiй є отрутою, вiд якої смерть органiзму наступає миттєво. На якi ферменти в мiтохондрiях дiє цiанистий калiй?
+Цитохромоксидаза (аа3)
-Флавiновi ферменти
-Цитохром
-НАД
+ - залежнi дегiдрогенази
-Цитохром Р-450


У вiдпочиваючого в санаторiї у результатi сонячного опiку на шкiрi спини утворилися мiхурцi, заповненi свiтлою рiдиною, оточенi зоною гiперемiї, болiснi. Який з перерахованих механiзмiв лежить в основi формування ексудацiї у вогнищi запалення?
+Збiльшення колоїдно-осмотичного тиску в тканинi
-Зменшення виведення рiдини з тканини
-Зменшення рiвня кейлонiв у тканинi
-Збiльшення кiлькостi лiзосомальних ферментiв
-Емiграцiя лейкоцитiв з судин


У дитини 5-ти рокiв розвинулось гостре респiраторне захворювання, яке супроводжувалось кашлем, видiленням значної кiлькостi слизу iз носа. Який тип запалення у хворої дитини?
+Катаральне
-Фiбриноїдне
-Геморагiчне
-Гнiйне
-Гнилiсне


Пiсля загоєння рани на її мiсцi утворився рубець. Яка речовина є основним компонентом цього рiзновиду сполучної тканини?
+Колаген
-Еластин
-Гiалуронова кислота
-Хондроiтин-сульфат
-Кератансульфат


Жiнка 30-ти рокiв хворiє близько року, коли вперше з’явився бiль у дiлянцi суглобiв, їх припухлiсть, почервонiння шкiри над ними. Попереднiй дiагноз - ревматоїдний артрит. Змiна якого компоненту в структурi бiлка сполучної тканини є однiєю з причин цього захворювання?
+Колаген
-Муцин
-Мiозин
-Овоальбумiн
-Тропонiн


При активацiї запального процесу, деяких аутоiмунних та iнфекцiйних захворюваннях у плазмi кровi рiзко зростає рiвень бiлкiв гострої фази. Який iз наведених нижче бiлкiв здатний утворювати гель при охолодженнi сироватки?
+Крiоглобулiн
-Гаптоглобiн
-Церулоплазмiн
-С-реактивний бiлок
-α 2 -макроглобiн


Юнака 15-ти рокiв вжалила бджола. На мiсцi укусу визначається набряк, гiперемiя, пiдвищення температури. Назвiть iнiцiальний патогенетичний фактор запального набряку:
+Пiдвищення проникностi мiкросудин
-Пiдвищення осмотичного тиску у вогнищi запалення
-Зниження онкотичного тиску кровi
-Пiдвищення кров’яного тиску в капiлярах
-Порушення лiмфовiдтоку


При запаленнi ока у хворого вiдмiчалося накопичення мутної рiдини з високим вмiстом бiлку на днi передньої камери, яке отримало назву - гiпопiон. Який процес лежить в основi зазначених змiн?
+Порушення мiкроциркуляцiї
-Первинна альтерацiя
-Вторинна альтерацiя
-Пролiферацiя
--


При утилiзацiї арахiдонової кислоти за циклооксигеназним шляхом утворюються бiологiчно активнi речовини. Вкажiть їх:
+Простагландини
-Тироксин
-Бiогеннi амiни
-Соматомедини
-Iнсулiноподiбнi фактори росту


Чоловiк 60-ти рокiв скаржиться на бiль у суглобах. У сироватцi кровi пацiєнта виявлено пiдвищення концентрацiї С-реактивного бiлку та оксипролiну. Для якого захворювання характернi цi симптоми?
+Ревматизм
-Подагра
-Гепатит
-Жовтяниця
-Цукровий дiабет


У сироватцi кровi пацiєнта встановлено пiдвищення активностi гiалуронiдази. Визначення якого бiохiмiчного показника сироватки кровi дозволить пiдтвердити припущення про патологiю сполучної тканини?
+Сiаловi кислоти
-Бiлiрубiн
-Сечова кислота
-Глюкоза
-Галактоза


У плазмi кровi здорової людини знаходиться декiлька десяткiв бiлкiв. При захворюваннi органiзму з’являються новi бiлки, зокрема "бiлок гострої фази". Таким бiлком є:
+С-реактивний бiлок
-Протромбiн
-Фiбриноген
-Iмуноглобулiн G
-Iмуноглобулiн А


У хворої 38-ми рокiв ревматизм в активнiй фазi. Визначення якого лабораторного показника сироватки кровi має дiагностичне значення при данiй патологiї?
+С-реактивний бiлок
-Сечова кислота
-Сечовина
-Креатинiн
-Трансферин


Запалення характеризується розширенням кровоносних судин на дiлянцi пошкодження, зменшенням кровообiгу, пiдвищенням проникливостi стiнки судин. Яким з нижче наведених клiтин належить головна роль в цьому?
+Тканиннi базофiли
-Фiбробласти
-Плазмоцити
-Еозинофiли
-Макрофаги


Пацiєнту, який знаходився в клiнiцi з приводу пневмонiї, ускладненої плевритом, у складi комплексної терапiї вводили преднiзолон. Протизапальна дiя цього синтетичного глюкокортикоїда пов’язана з блокуванням вивiльнення арахiдонової кислоти шляхом гальмування такого ферменту:
+Фосфолiпаза A2
-Циклооксигеназа
-Фосфолiпаза C
-Лiпоксигеназа
-Пероксидаза


У чоловiка 48-ми рокiв виявлено порушення периферичного кровообiгу з обмеженням припливу артерiальної кровi, при цьому має мiсце зблiднiння даної дiлянки, зниження мiсцевої температури. Це порушення називається:
+Iшемiя
-Стаз
-Сладж
-Венозна гiперемiя
-Реперфузiйний синдром


У хворого 70-ти рокiв атеросклероз ускладнився тромбозом судин нижнiх кiнцiвок, виникла гангрена пальцiв лiвої стопи. Початок тромбоутворення, найбiльш iмовiрно, пов’язаний з:
+Адгезiєю тромбоцитiв
-Активацiєю протромбiнази
-Перетворенням протромбiну в тромбiн
-Перетворенням фiбриногену в фiбрин
-Зниженням синтезу гепарину


У жiнки 60-ти рокiв пiсля емоцiйної реакцiї, яка була викликана гнiвом, виник напад загрудинного болю. На ЕКГ були встановленi ознаки порушення коронарного кровообiгу. Який вид порушень мiг спричинити це явище?
+Ангiоспастична iшемiя
-Артерiальна гiперемiя
-Венозна гiперемiя
-Справжнiй стаз
-Венозний стаз


Пiд час гри у волейбол спортсмен пiсля стрибка приземлився на зовнiшнiй край стопи. Виник гострий бiль у гомiлковостопному суглобi, активнi рухи в ньому обмеженi, пасивнi в повному обсязi, але болiснi. Потiм розвинулася припухлiсть у дiлянцi зовнiшньої щиколотки, шкiра почервонiла, стала теплiшою на дотик. Який вид розладу периферичного кровообiгу розвинувся в даному випадку?
+Артерiальна гiперемiя
-Стаз
-Емболiя
-Венозна гiперемiя
-Тромбоз


У кроля перерiзали нерв, що iннервує праве вухо, i видалили правий верхнiй шийний симпатичний вузол. Одразу пiсля операцiї провели вимiрювання температури шкiри вух. Виявилося, що температура шкiри вуха кролика на боцi денервацiї на 1,5℃ вища, нiж на протилежному iнтактному боцi. Що з наведеного є найбiльш вiрогiдною причиною вказаних явищ?
+Артерiальна гiперемiя нейропаралiтичного типу
-Артерiальна гiперемiя нейротонiчного типу
-Артерiальна гiперемiя, обумовлена метаболiчними факторами
-Реактивна артерiальна гiперемiя
-Фiзiологiчна артерiальна гiперемiя


У хворого з варикозним розширенням вен пiд час огляду нижнiх кiнцiвок вiдзначається: цiаноз, пастознiсть, зниження температури шкiри, поодинокi петехiї. Який розлад гемодинамiки має мiсце у хворого?
+Венозна гiперемiя
-Компресiйна iшемiя
-Обтурацiйна iшемiя
-Тромбоемболiя
-Артерiальна гiперемiя


Хворий 65-ти рокiв, що страждає на атеросклероз, госпiталiзований до хiрургiчного вiддiлення з приводу розлитого гнiйного перитонiту. Пiд час операцiї дiагностовано тромбоз брижових артерiй. Яка найбiльш iмовiрна причина перитонiту?
+Геморагiчний iнфаркт
-Iшемiя ангiоспастична
-Iшемiчний iнфаркт
-Стаз
-Iшемiя компресiйна


При лабораторному дослiдженнi кровi пацiєнта виявлено, що вмiст бiлкiв у плазмi становить 40 г/л. Як це впливає на транскапiлярний обмiн води в мiкроциркуляторному руслi?
+Збiльшується фiльтрацiя, зменшується реабсорбцiя
-Збiльшуються фiльтрацiя i реабсорбцiя
-Зменшується фiльтрацiя, збiльшується реабсорбцiя
-Зменшуються фiльтрацiя i реабсорбцiя
-Обмiн не змiнюється


У спортсмена пiсля iнтенсивного тренування вiдзначається значне зниження тонусу судин у дiлянцi працюючих м’язiв. Причиною розвитку такого ефекту є накопичення у працюючих тканинах:
+Метаболiтiв
-Ренiн-ангiотензину
-Гiстамiну
-Натрiйуретичного гормону
-Серотонiну


Жiнка 45-ти рокiв хворiє на рак лiвої молочної залози. На лiвiй руцi є ознаки недостатностi лiмфатичної системи - набряк кiнцiвки, збiльшення лiмфовузлiв. Яка форма недостатностi лiмфообмiну спостерiгається у хворої?
+Механiчна недостатнiсть
-Динамiчна недостатнiсть
-Резорбцiйна недостатнiсть
-Змiшана недостатнiсть
--


Студент на екзаменi не змiг вiрно вiдповiсти на питання екзаменацiйного бiлету, що супроводжувалося почервонiнням шкiри обличчя, вiдчуттям жару i невпевненiстю поведiнки. Який вид артерiальної гiперемiї розвинувся у даному випадку?
+Нейротонiчна
-Нейропаралiтична
-Метаболiчна
-Патологiчна
-Постiшемiчна


У хворого з тромбофлебiтом нижнiх кiнцiвок раптово пiсля навантаження виникли задишка, рiзкий бiль у грудях, цiаноз, набухання шийних вен. Яке найбiльш iмовiрне порушення кровообiгу виникло у хворого?
+Тромбоемболiя легеневої артерiї
-Тромбоемболiя вiнцевих судин
-Тромбоемболiя судин головного мозку
-Тромбоемболiя мезентерiальних судин
-Тромбоемболiя ворiтної вени


Пацiєнт звернувся до лiкаря зi скаргами на задишку, що виникала пiсля фiзичного навантаження. Клiнiчне обстеження виявило анемiю та наявнiсть парапротеїну в зонi гамма-глобулiнiв. Який показник у сечi необхiдно визначити для пiдтвердження дiагнозу мiєломи?
+ Бiлок Бенс-Джонса
- Бiлiрубiн
- Гемоглобiн
- Церулоплазмiн
- Антитрипсин


У жiнки встановлено дiагноз – рак шийки матки. З яким вiрусом може бути асоцiйована ця патологiя?
+ Папілома-вірус
- Varicella-Zoster вiрус
- Цитомегаловiрус
- Вiрус простого герпеса тип 2
- Аренавiрус


В експериментi показано, що при саркомi Iєнсена споживання глюкози з привiдної до пухлини артерiї значно збiльшується, має мiсце також прирiст вмiсту молочної кислоти у вiдвiднiй венi. Про що свiдчить дане явище?
+ Посилення анаеробного глiколiзу
- Посилення окисних процесiв
- Посилення окиснення бiлкiв
- Зменшення анаеробного глiколiзу
- Зменшення окисних процесiв


У чоловiка 63-х рокiв рак стравоходу, метастази у лiмфатичнi вузли середостiння, ракова кахексiя. Яка патогенетична стадiя пухлинного процесу має мiсце?
+ Прогресiї
- Промоцiї
- Трансформацiї
- Iнiцiацiї
-


При гiстологiчному дослiдженнi новоутворення шкiри виявлено: паренхiма сформована з покривного епiтелiю iз збiльшеним числом шарiв. Строма разом з розростаннями епiтелiю формує сосочки. Вкажiть вид атипiзму:
+ Тканинний
- Клiтинний
- Гiстохiмiчний
- Функцiональний
- Метаболiчний


До клiнiки надiйшла дитина 4-х рокiв з ознаками тривалого бiлкового голодування: затримка росту, анемiя, набряки, розумова вiдсталiсть. Причиною розвитку набрякiв у цiєї дитини є зниження синтезу:
+ Альбумiнiв
- Глобулiнiв
- Гемоглобiну
- Лiпопротеїнiв
- Глiкопротеїнiв


У людини зменшений дiурез, гiпернатрiємiя, гiпокалiємiя. Гiперсекрецiя якого гормону може бути причиною таких змiн?
+ Альдостерон
- Вазопресин
- Передсердний натрiйуретичний фактор
- Адреналiн
- Паратгормон


У хворого виявлено гiперкалiємiю та гiпонатрiємiю. Знижена секрецiя якого гормону може спричинити такi змiни?
+ Альдостерон
- Вазопресин
- Кортизол
- Паратгормон
- Натрiйуретичний


У хворого 28-ми рокiв тривале блювання призвело до зневоднення органiзму. Пiдвищена секрецiя якого гормону перш за все сприятиме збереженню води в органiзмi?
+ Вазопресин
- Кальцитонiн
- Тироксин
- Соматостатин
- Альдостерон


У людини осмотичний тиск плазми кровi 350 мосмоль/л (норма – 300 мосмоль/л). Це спричинить, перш за все, посилену секрецiю такого гормону:
+ Вазопресин
- Альдостерон
- Кортизол
- Адренокортикотропiн
- Натрiйуретичний


У туриста пiд час тривалого перебування на спекотi вiдбулася значна втрата води, що супроводжувалося рiзким зниженням дiурезу. Посилення секрецiї яких гормонiв вiдбувається при цьому?
+ Вазопресин й альдостерон
- Адреналiн i норадреналiн
- Глюкокортикоїди й iнсулiн
- Тироксин i трийодтиронiн
- Серотонiн i дофамiн


У дорослої людини за добу видiляється 20 л сечi з низькою вiдносною щiльнiстю. Найбiльш iмовiрною причиною цього є дефiцит в органiзмi:
+ Вазопресину
- Альдостерону
- Натрiйуретичного фактора
- Ренiну
- Паратгормону


Людинi внутрiшньовенно ввели 0,5 л iзотонiчного розчину лiкарської речовини. Якi з рецепторiв насамперед вiдреагують на змiну водно-сольового балансу органiзму?
+ Волюморецептори порожнистих вен I передсердь
- Осморецептори гiпоталамусу
- Осморецептори печiнки
- Натрiєвi рецептори гiпоталамусу
- Барорецептори дуги аорти


У людини з масою 80 кг пiсля тривалого фiзичного навантаження об’єм циркулюючої кровi зменшився, гематокрит - 50%, загальний бiлок кровi - 80 г/л. Такi показники кровi є наслiдком, перш за все:
+ Втрати води з потом
- Збiльшення кiлькостi еритроцитiв
- Збiльшення вмiсту бiлкiв у плазмi
- Збiльшення онкотичного тиску плазми
- Збiльшення дiурезу


У хворого, що страждає на серцеву недостатнiсть, спостерiгаються збiльшення печiнки, набряки нижнiх кiнцiвок, асцит. Який механiзм є провiдним в утвореннi даного набряку?
+ Гiдродинамiчний
- Колоїдно-осмотичний
- Лiмфогенний
- Мембраногенний
-


В експериментi збiльшили проникнiсть мембрани збудливої клiтини для iонiв калiю. Якi змiни електричного стану мембрани при цьому виникнуть?
+ Гiперполяризацiя
- Деполяризацiя
- Потенцiал дiї
- Локальна вiдповiдь
- Змiн не буде


У хворого на ентерит, що супроводжувався значною дiареєю, спостерiгається зменшення кiлькостi води в позаклiтинному просторi, збiльшення її всерединi клiтин та зниження осмолярностi кровi. Як називають таке порушення водно-електролiтного обмiну?
+ Гiпоосмолярна гiпогiдратацiя
- Гiперосмолярна гiпогiдратацiя
- Осмолярна гiпогiдратацiя
- Гiпоосмолярна гiпергiдратацiя
- Гiперосмолярна гiпергiдратацiя


Лiкар-цитогенетик при виготовленнi метафазної пластинки обробив культуру лейкоцитiв гiпотонiчним (0,56%) розчином хлориду калiю. Пiсля цього вiдбулося набухання клiтин i розрив клiтинної мембрани за рахунок надходження води до клiтини. Який механiзм транспорту має мiсце в цьому випадку?
+ Ендоосмос
- Полегшена дифузiя
- Дифузiя
- Пiноцитоз
- Фагоцитоз


Хворому внутрiшньовенно ввели гiпертонiчний розчин глюкози. Це пiдсилить рух води:
+ З клiтин до мiжклiтинної рiдини
- З мiжклiтинної рiдини до капiлярiв
- З мiжклiтинної рiдини до клiтин
- З капiлярiв до мiжклiтинної рiдини
- Змiн руху води не буде


Жiнцi 36-ти рокiв пiсля хiрургiчного втручання внутрiшньовенно ввели концентрований розчин альбумiну. Це спричинило посилений рух води у такому напрямку:
+ З мiжклiтинної рiдини до капiлярiв
- З мiжклiтинної рiдини до клiтин
- Iз клiтин до мiжклiтинної рiдини
- Iз капiлярiв до мiжклiтинної рiдини
- Змiн руху води не вiдбуватиметься


Тваринi внутрiшньовенно ввели концентрований розчин хлориду натрiю, що зумовило зниження реабсорбцiї iонiв натрiю у канальцях нирок. Внаслiдок яких змiн секрецiї гормонiв це вiдбувається?
+ Зменшення альдостерону
- Збiльшення альдостерону
- Зменшення вазопресину
- Збiльшення вазопресину
- Зменшення натрiйуретичного фактора


При токсичному ушкодженнi клiтин печiнки з порушенням її функцiй у хворого з’явилися набряки. Якi змiни складу плазми кровi є провiдною причиною розвитку набрякiв?
+ Зниження вмiсту альбумiнiв
- Збiльшення вмiсту глобулiнiв
- Зменшення вмiсту фiбриногену
- Збiльшення вмiсту альбумiнiв
- Зменшення вмiсту глобулiнiв


У хворого з нефротичним синдромом спостерiгаються масивнi набряки обличчя та кiнцiвок. Який патогенетичний механiзм є провiдним в розвитку набрякiв?
+ Зниження онкотичного тиску кровi
- Пiдвищення судинної проникностi
- Пiдвищення гiдродинамiчного тиску кровi
- Лiмфостаз
- Пiдвищення лiмфовiдтоку


У хворого 35-ти рокiв, який часто вживає алкоголь, на фонi лiкування сечогiнними засобами, виникли сильна м’язова i серцева слабкiсть, блювання, дiарея, АТ- 100/60 мм рт.ст., депресiя. Причиною такого стану є посилене видiлення з сечею:
+ Калiю
- Натрiю
- Хлору
- Кальцiю
- Фосфатiв


У хворої людини посилений рух води з кровоносних капiлярiв до тканин, що викликало їх позаклiтинний набряк (збiльшенi розмiри м’яких тканин кiнцiвок, печiнки тощо). Зменшення якого параметру гомеостазу є найбiльш iмовiрною причиною розвитку набряку?
+ Онкотичний тиск плазми кровi
- Осмотичний тиск плазми кровi
- рН кровi
- В’язкiсть кровi
- Гематокрит


Хворий пiсля перенесеного епiдемiчного паротиту схуднув, постiйно вiдчуває спрагу, п’є багато води, вiдмiчає часте сечовидiлення, пiдвищений апетит, шкiрний свербiж, слабкiсть, фурункульоз. У кровi: глюкоза - 16 ммоль/л, кетонових тiл - 100 мкмоль/л; глюкозурiя. Яке захворювання розвинулось у пацiєнта?
+ Iнсулiнозалежний цукровий дiабет
- Iнсулiнонезалежний цукровий дiабет
- Стероїдний дiабет
- Нецукровий дiабет
- Цукровий дiабет недостатнього харчування


Хвороба Андерсена належить до групи спадкових хвороб, що розвиваються внаслiдок уродженої недостатностi синтезу певних ферментiв глiкогенолiзу. Недостатнiсть якого ферменту є молекулярною основою цього глiкогенозу?
+ Амiло(1,4-1,6)трансглiкозидаза
- Глiкогенсинтаза
- Глюкозо-6-фосфатази
- Лiзосомальнi глiкозидази
- Фосфофруктокiназа


До клiнiки доставлено хвору на цукровий дiабет, госпiталiзовано у прекоматозному станi кетоацидотичного типу. Збiльшення вмiсту якого метаболiту до цього призвело?
+ Ацетоацетат
- Цитрат
- α-кетоглутарат
- Малонат
- Аспартат


При недостатностi кровообiгу у перiод iнтенсивної м’язової роботи у м’язi в результатi анаеробного глiколiзу накопичується молочна кислота. Яка її подальша доля?
+ Включається в глюконеогенез у печiнцi
- Видаляється через нирки з сечею
- Використовується у м’язi для синтезу амiнокислот
- Використовується тканинами для синтезу кетонових тiл
- Використовується у тканинах для синтезу жирних кислот


Хвора доставлена бригадою швидкої допомоги. Об’єктивно: стан важкий, свiдомiсть вiдсутня, адинамiя. Шкiрнi покриви сухi, запалi очi, цiаноз обличчя, тахiкардiя, запах ацетону з рота. Результати аналiзiв: глюкоза кровi 20,1 ммоль/л (у нормi - 3,3-5,5 ммоль/л), у сечi - 3,5% (у нормi - 0). Який найбiльш вiрогiдний дiагноз?
+ Гiперглiкемiчна кома
- Гiпоглiкемiчна кома
- Гостра серцева недостатнiсть
- Гостре алкогольне отруєння
- Анафiлактичний шок


Чоловiк 53-х рокiв доставлений у стацiонар у непритомному станi. Об’єктивно: шкiра суха, дихання часте поверхневе, запах ацетону вiдсутнiй, Ps- 126/хв., АТ- 70/40 мм рт.ст. Вмiст глюкози у кровi 48 ммоль/л, реакцiя сечi на ацетон негативна. Для якого iз перелiчених станiв найбiльш характернi симптоми у хворого?
+ Гiперосмолярна кома
- Гiперкетонемiчна кома
- Лактацидемiчна кома
- Токсична кома
- Колапс


У пiдлiтка 12-ти рокiв, який впродовж 3-х мiсяцiв рiзко схуд, вмiст глюкози у кровi 50 ммоль/л. У нього розвинулася кома. Який головний механiзм її розвитку?
+ Гiперосмолярний
- Гiпоглiкемiчний
- Кетонемiчний
- Лактацидемiчний
- Гiпоксичний


Хвора 38-ми рокiв надiйшла до реанiмацiйного вiддiлення в несвiдомому станi. Рефлекси вiдсутнi. Цукор кровi 2,1 ммоль/л. В анамнезi - цукровий дiабет з 18-ти рокiв. Яка кома має мiсце у хворої?
+ Гiпоглiкемiчна
- Кетоацидотична
- Лактацидемiчна
- Гiперосмолярна
- Гiперглiкемiчна


Хворого доставлено у медичний заклад в коматозному станi. Зi слiв супроводжуючих вдалося з’ясувати, що вiн знепритомнiв пiд час тренування на завершальному етапi марафонської дистанцiї. Який вид коми найiмовiрнiше можна запiдозрити у даного пацiєнта?
+ Гiпоглiкемiчна
- Гiперглiкемiчна
- Ацидотична
- Гiпотиреоїдна
- Печiнкова


У хлопчика 2-х рокiв спостерiгається збiльшення в розмiрах печiнки та селезiнки, катаракта. В кровi пiдвищена концентрацiя цукру, але тест толерантностi до глюкози в нормi. Спадкове порушення обмiну якої речовини є причиною цього стану?
+ Галактоза
- Фруктоза
- Глюкоза
- Мальтоза
- Сахароза


Дитина 1-го року вiдстає в розумовому розвитку вiд своїх однолiткiв. Ранком вiдзначаються блювання, судоми, непритомнiсть. У кровi – гiпоглiкемiя натще. З дефектом якого ферменту це пов’язане?
+ Глiкогенсинтетаза
- Фосфорилаза
- Аргiназа
- Сахараза
- Лактаза


У п’ятирiчного хлопчика спостерiгалися малий зрiст, розумове вiдставання, обмеженi рухи, грубi риси обличчя. Цi особливостi стали помiтними з 18мiсячного вiку. У нього виявили дефiцит L-iдуронiдази. Обмiн яких сполук порушено?
+ Глiкозамiноглiкани
- Бiлки
- Нуклеотиди
- Вiтамiни
- Фосфолiпiди


Людина хворiє на цукровий дiабет, що супроводжується гiперглiкемiєю натще понад 7,2 ммоль/л. Рiвень якого бiлку плазми кровi дозволяє ретроспективно (за попереднi 4-8 тижнi до обстеження) оцiнити рiвень глiкемiї?
+ Глiкозильований гемоглобiн
- Альбумiн
- Фiбриноген
- С-реактивний бiлок
- Церулоплазмiн


У жiнки 62-х рокiв розвинулася катаракта (помутнiння кришталика) на фонi цукрового дiабету. Який тип модифiкацiї бiлкiв має мiсце при дiабетичнiй катарактi?
+ Глiкозилювання
- Фосфорилювання
- АДФ-рибозилювання
- Метилювання
- Обмежений протеолiз


Пiсля тривалого фiзичного навантаження пiд час заняття з фiзичної культури у студентiв розвинулась м’язова крепатура. Причиною її виникнення стало накопичення у скелетних м’язах молочної кислоти. Вона утворилась пiсля активацiї в органiзмi студентiв:
+ Глiколiзу
- Глюконеогенезу
- Пентозофосфатного циклу
- Лiполiзу
- Глiкогенезу


У студента, який складає iспит, вмiст глюкози у плазмi кровi складає 8 ммоль/л. Збiльшена секрецiя якого з наведених гормонiв сприяє розвитку гiперглiкемiї у студента?
+ Глюкагон
- Iнсулiн
- Тироксин
- Трийодтиронiн
- Альдостерон


При глiкогенозi - хворобi Гiрке - порушується перетворення глюкозо-6-фосфату на глюкозу, що призводить до накопичення глiкогену в печiнцi та нирках. Дефiцит якого ферменту є причиною захворювання?
+ Глюкозо-6-фосфатаза
- Глiкогенсинтетаза
- Фосфорилаза
- Гексокiназа
- Альдолаза


У трирiчної дитини з пiдвищеною температурою тiла пiсля прийому аспiрину спостерiгається посилений гемолiз еритроцитiв. Вроджена недостатнiсть якого ферменту могла викликати у дитини гемолiтичну анемiю?
+ Глюкозо-6-фосфатдегiдрогеназа
- Глюкозо-6-фосфатаза
- Глiкогенфосфорилаза
- Глiцеролфосфатдегiдрогеназа
- γ-глутамiлтрансфераза


Хворому з ревматоїдним артритом тривалий час вводили гiдрокортизон. У нього з’явилися гiперглiкемiя, полiурiя, глюкозурiя, спрага. Цi ускладнення лiкування є наслiдком активацiї процесу:
+ Глюконеогенез
- Глiкогенолiз
- Глiкогенез
- Глiколiз
- Лiполiз


При хронiчному передозуваннi глюкокортикоїдiв розвивається гiперглiкемiя. Назвiть процес вуглеводного обмiну, за рахунок якого збiльшується концентрацiя глюкози у плазмi кровi:
+ Глюконеогенез
- Глiкогенолiз
- Аеробний глiколiз
- Пентозофосфатний цикл
- Глiкогенез


У жiнки 45-ти рокiв хвороба Iценко-Кушiнга - стероїдний дiабет. При бiохiмiчному обстеженнi: гiперглiкемiя, гiпохлоремiя. Який з перерахованих нижче процесiв активується у жiнки в першу чергу?
+ Глюконеогенез
- Глiкогенолiз
- Реабсорбцiя глюкози
- Транспорт глюкози в клiтину
- Глiколiз


Молода людина 25-ти рокiв споживає надмiрну кiлькiсть вуглеводiв (600 г на добу), що перевищує її енергетичнi потреби. Який процес буде активуватися в органiзмi людини у даному випадку?
+ Лiпогенез
- Глiколiз
- Лiполiз
- Глюконеогенез
- Окиснення жирних кислот


Дитина квола, апатична. Печiнка збiльшена, при її бiопсiї виявлено значний надлишок глiкогену. Концентрацiя глюкози в кровi нижче норми. У чому причина зниженої концентрацiї глюкози у кровi цiєї хворої?
+ Понижена (вiдсутня) активнiсть глiкоген-фосфорилази у печiнцi
- Понижена (вiдсутня) активнiсть гексокiнази у печiнцi
- Пiдвищена активнiсть глiкогенсинтетази у печiнцi
- Понижена (вiдсутня) активнiсть глюкозо-6-фосфатази у печiнцi
- Дефiцит гену, який вiдповiдає за синтез глюкозо-1фосфатуридинтрансферази


При диспансерному обстеженнi у хворого знайдено цукор в сечi. Який найбiльш iмовiрний механiзм виявлених змiн, якщо вмiст цукру в кровi нормальний?
+ Порушення реабсорбцiї глюкози в канальцях нефрона
- Порушення фiльтрацiї глюкози в клубочковому вiддiлi нефрона
- Недостатня продукцiя iнсулiну пiдшлунковою залозою
- Iнсулiнорезистентнiсть рецепторiв клiтин
- Гiперпродукцiя глюкокортикоїдiв наднирниками


Пiдшлункова залоза - орган змiшаної секрецiї. Ендокринно продукує бетаклiтинами гормон iнсулiн, який впливає на обмiн вуглеводiв. Як вiн впливає на активнiсть глiкогенфосфорилази (ГФ) i глiкогенсинтетази (ГС)?
+ Пригнiчує ГФ, активує ГС
- Активує ГФ i ГС
- Пригнiчує ГФ i ГС
- Активує ГФ, пригнiчує ГС
- Не впливає на активнiсть ГФ i ГС


У хворого 15-ти рокiв концентрацiя глюкози натще - 4,8 ммоль/л, через годину пiсля цукрового навантаження - 9,0 ммоль/л, через 2 години - 7,0 ммоль/л, через 3 години - 4,8 ммоль/л. Цi показники характернi для такого захворювання:
+ Прихований цукровий дiабет
- Цукровий дiабет I типу
- Цукровий дiабет II типу
- Хвороба Iценко-Кушiнга
-


До шпиталю було доставлено юнака 16-ти рокiв, хворого на iнсулiнозалежний цукровий дiабет. Рiвень глюкози у кровi пацiєнта складав 18 ммоль/л. Хворому було введено iнсулiн. Двi години потому рiвень глюкози зменшився до 8,2 ммоль/л, тому що iнсулiн:
+ Стимулює перетворення глюкози в печiнцi у глiкоген та ТАГ
- Стимулює транспорт глюкози через плазматичнi мембрани в головному мозку та печiнцi
- Гальмує синтез кетонових тiл iз глюкози
- Стимулює розщеплення глiкогену в печiнцi
- Стимулює розщеплення глiкогену у м’язах


При напруженiй фiзичнiй роботi у м’язовiй тканинi накопичується молочна кислота, яка дифундує в кров i пiдхоплюється печiнкою та серцем. Який процес забезпечує вiдновлення запасiв глiкогену в м’язах?
+ Цикл Корi
- Цикл лимонної кислоти
- Орнiтиновий цикл
- Цикл трикарбонових кислот
- Пентозофосфатний шлях


Виникнення нижчеперерахованих захворювань пов’язане iз генетичними факторами. Назвiть патологiю iз спадковою схильнiстю:
+ Цукровий дiабет
- Хорея Гентiнгтона
- Фенiлкетонурiя
- Серпоподiбноклiтинна анемiя
- Дальтонiзм


Хвора 46-ти рокiв скаржиться на сухiсть в ротi, спрагу, почащений сечопуск, загальну слабкiсть. У кровi: гiперглiкемiя, гiперкетонемiя. У сечi: глюкоза, кетоновi тiла. На ЕКГ: дифузнi змiни в мiокардi. Який найбiльш iмовiрний дiагноз?
+ Цукровий дiабет
- Алiментарна гiперглiкемiя
- Гострий панкреатит
- Нецукровий дiабет
- Iшемiчна хвороба серця


У хворого з дихальною недостатнiстю рН кровi 7,35. Визначення pCO2 показало наявнiсть гiперкапнiї. При дослiдженнi рН сечi вiдзначається пiдвищення її кислотностi. Яка форма порушення кислотноосновного стану в даному випадку?
+ Ацидоз газовий, компенсований
- Ацидоз метаболiчний, компенсований
- Ацидоз метаболiчний, декомпенсований
- Алкалоз газовий, компенсований
- Алкалоз газовий, декомпенсований


У хворого з дихальною недостатнiстю рН кровi 7,35. Визначення pCO 2 показало наявнiсть гiперкапнiї. При дослiдженнi рН сечi вiдзначається пiдвищення її кислотностi. Яка форма порушення кислотно-основного стану в даному випадку?
+ Ацидоз газовий, компенсований
- Ацидоз метаболiчний, компенсований
- Ацидоз метаболiчний, декомпенсований
- Алкалоз газовий, компенсований
- Алкалоз газовий, декомпенсований


У людини з хронiчним захворюванням нирок порушена їх видiльна функцiя. рН венозної кровi становить 7,33. Для корекцiї кислотно-лужного стану пацiєнту доцiльно внутрiшньовенно ввести розчин:
+ Бiкарбонату натрiю
- Хлориду натрiю
- Глюкози
- Хлориду калiю
- Хлориду кальцiю


У немовля с пiлороспазмом внаслiдок блювання, що часто повторювалося, з’явилися слабкiсть, гiподинамiя, iнодi судоми. Яка форма порушення кислотно-основного стану в нього спостерiгається?
+ Видiльний алкалоз
- Видiльний ацидоз
- Метаболiчний ацидоз
- Екзогенний негазовий ацидоз
- Газовий алкалоз


Хворий на гiпертонiчну хворобу разом з безсольовою дiєтою та з антигiпертензивними засобами, довгий час приймав гiдрохлортiазид, що зумовило порушення електролiтного балансу. Яке порушення внутрiшнього середовища виникло у хворого?
+ Гiпохлоремiчний алкалоз
- Метаболiчний ацидоз
- Гiперкалiємiя
- Гiпермагнiємiя
- Збiльшення об’єму циркулюючої кровi


У хворого 40-ка рокiв ознаки гiрської хвороби: запаморочення, задишка, тахiкардiя, рН кровi - 7,50, pCO2 - 30 мм рт.ст., зсув буферних основ +4 ммоль/л. Яке порушення кислотноосновного стану має мiсце?
+ Газовий алкалоз
- Негазовий алкалоз
- Негазовий ацидоз
- Газовий ацидоз
- Видiльний ацидоз


У результатi виснажуючої м’язової працi у робочого значно зменшилася буферна ємнiсть кровi. Надходження якої речовини у кров може бути причиною цього явища?
+ Лактат
- Пiруват
- 1,3-бiсфосфоглiцерат
- 3-фосфоглiцерат
-


У немовляти внаслiдок неправильного годування виникла виражена дiарея. Одним з основних наслiдкiв дiареї є екскрецiя великої кiлькостi бiкарбонату натрiю. Яка форма порушення кислотно-лужного балансу має мiсце в цьому випадку?
+ Метаболiчний ацидоз
- Метаболiчний алкалоз
- Респiраторний ацидоз
- Респiраторний алкалоз
- Не буде порушень кислотно-лужного балансу


У хворого струс головного мозку, що супроводжується повторним блюванням i задишкою. При обстеженнi вiдзначено: рН - 7,62; pCO2 - 40 мм рт.ст. Яке порушення кислотно-основного стану є у хворого?
+ Негазовий алкалоз
- Газовий алкалоз
- Негазовий ацидоз
- Газовий ацидоз
-


У пацiєнта у результатi тривалого блювання вiдбувається значна втрата шлункового соку, що є причиною порушення кислотно-лужного стану в органiзмi. Яка з перерахованих форм порушення кислотнолужного стану має мiсце?
+ Негазовий алкалоз
- Газовий ацидоз
- Негазовий ацидоз
- Газовий алкалоз
- Метаболiчний ацидоз


У новонародженої дитини з пiлоростенозом часте блювання, що супроводжується апатiєю, слабкiстю, пiдвищенням тонусу м’язiв, iнколи судомами. Яка форма порушення кислотноосновного стану розвинулася в хворого?
+ Негазовий алкалоз
- Газовий алкалоз
- Газовий ацидоз
- Метаболiчний ацидоз
- Видiльний ацидоз


У хворого пiсля вживання недоброякiсної їжi розвинулася дiарея. На наступний день у нього знизився артерiальний тиск, з’явились тахiкардiя, екстрасистолiя. pH кровi складає 7,18. Цi порушення є наслiдком розвитку:
+ Негазового ацидозу
- Газового ацидозу
- Негазового алкалозу
- Газового алкалозу
- Метаболiчного алкалозу


У чоловiка 32-х рокiв, хворого на пневмонiю, спостерiгається закупорка харкотинням дихальних шляхiв. В органiзмi хворого при цьому буде розвиватися така змiна кислотно-лужної рiвноваги:
+ Респiраторний ацидоз
- Метаболiчний ацидоз
- Респiраторний алкалоз
- Метаболiчний алкалоз
- Змiн не буде


При обстеженнi дитини лiкар виявив ознаки рахiту. Нестача якої сполуки в органiзмi дитини сприяє розвитку цього захворювання?
+ 1,25 [ОН]дигiдроксiхолекальциферол
- Бiотин
- Токоферол
- Нафтохiнон
- Ретинол


При обстеженнi в клiнiцi у чоловiка дiагностували гостру променеву хворобу. Лабораторно встановлено рiзке зниження вмiсту серотонiну в тромбоцитах. Порушення метаболiзму якої речовини є можливою причиною зниження тромбоцитарного серотонiну?
+ 5-окситрiптофан
- Тирозин
- Гiстидин
- Фенiлаланiн
- Серин


При обстеженнi хворого окулiст виявив збiльшення часу адаптацiї ока до темряви. Нестача якого вiтамiну може бути причиною такого симптому?
+ A
- B6
- B2
- C
- K


У хворого в кровi збiльшена концентрацiя пiрувату. Значна його кiлькiсть екскретується з сечею. Дефiцит якого вiтамiну має мiсце у хворого?
+ B1
- E
- B3
- B6
- B


Пiсля видалення у пацiєнта 2/3 шлунка у кровi зменшився вмiст гемоглобiну, кiлькiсть еритроцитiв, збiльшилися розмiри цих клiтин кровi. Дефiцит якого вiтамiну призводить до таких змiн у кровi?
+ B12
- C
- P
- B6
- PP


У хворого пiсля видалення жовчного мiхура утрудненi процеси всмоктування Ca через стiнку кишечнику. Призначення якого вiтамiну буде стимулювати цей процес?
+ D3
- PP
- C
- B12
- K


При дослiдженнi кровi хворого виявлено значне збiльшення активностi МВ-форм КФК (креатинфосфокiнази) та ЛДГ-1. Яку патологiю можна припустити?
+ Iнфаркт мiокарда
- Гепатит
- Ревматизм
- Панкреатит
- Холецистит


При обтурацiйнiй жовтяницi I жовчних норицях часто спостерiгається протромбiнова недостатнiсть. З дефiцитом в органiзмi якого вiтамiну це пов’язано?
+ K
- B6
- A
- C
- E


У хворого, який знаходиться на лiкуваннi з приводу вiрусного гепатиту В, з’явилися ознаки печiнкової недостатностi. Якi змiни кровi, що свiдчать про порушення бiлкового обмiну, найбiльш вiрогiдно спостерiгатимуться у цьому випадку?
+ Абсолютна гiпоальбумiнемiя
- Абсолютна гiперальбумiнемiя
- Абсолютна гiперфiбриногенемiя
- Бiлковий склад кровi не змiнений
- Абсолютна гiперглобулiнемiя


Людина впродовж тривалого часу вживала їжу, бiдну на метiонiн, внаслiдок чого у неї спостерiгалися розлади функцiї нервової та ендокринної систем. Це може бути наслiдком порушення синтезу:
+ Адреналiну
- Пiрувату
- Тиронiну
- Жирних кислот
- Глюкагону


У хворого, який проходить курс лiкувального голодування, нормальний рiвень глюкози у кровi пiдтримується головним чином за рахунок глюконеогенезу. З якої амiнокислоти при цьому у печiнцi людини найбiльш активно синтезується глюкоза?
+ Аланiн
- Лiзин
- Валiн
- Глутамiнова кислота
- Лейцин


Батьки дитини 3-х рокiв звернули увагу на потемнiння кольору його сечi при вiдстоюваннi. Об’єктивно: температура у нормi, шкiрнi покриви чистi, рожевi, печiнка не збiльшена. Назвiть iмовiрну причину даного стану:
+ Алкаптонурiя
- Гемолiз
- Синдром Iценка-Кушiнга
- Фенiлкетонурiя
- Подагра


У людини, що виконувала важку фiзичну роботу в умовах пiдвищеної температури навколишнього середовища, змiнилася кiлькiсть бiлкiв плазми кровi. Що саме має мiсце у даному випадку?
+ Вiдносна гiперпротеїнемiя
- Абсолютна гiперпротеїнемiя
- Абсолютна гiпопротеїнемiя
- Диспротеїнемiя
- Парапротеїнемiя


У хворого з частими кровотечами з внутрiшнiх органiв i слизових оболонок виявленi пролiн i лiзин у складi колагенових волокон. Через вiдсутнiсть якого вiтамiну порушено їх гiдроксилювання?
+ Вiтамiн C
- Вiтамiн K
- Вiтамiн A
- Тiамiн
- Вiтамiн E


У хворого, що страждає на спадкову хворобою Хартнупа, спостерiгаються пелагроподiбнi ураження шкiри, порушення розумового розвитку в результатi нестачi нiкотинової кислоти. Причиною цього захворювання є порушення такого процесу:
+ Всмоктування i реабсорбцiя в нирках триптофану
- Трансамiнування фенiлаланiну
- Декарбоксилювання триптофану
- Всмоктування i реабсорбцiя в нирках метiонiну
- Всмоктування i реабсорбцiя цистеїну


До фiбрилярних елементiв сполучної тканини належать колаген, еластин та ретикулiн. Вкажiть амiнокислоту, яка входить тiльки до складу колагену i визначення якої в бiологiчних рiдинах використовується для дiагностики захворювань сполучної тканини:
+ Гiдроксипролiн
- Пролiн
- Глiцин
- Лiзин
- Фенiлаланiн


Хворому, що страждає на хронiчний гепатит, для оцiнки знешкоджуючої функцiї печiнки було проведене навантаження бензоатом натрiю. За видiленням якої речовини з сечею судять про знешкоджуючу функцiю печiнки?
+ Гiпурова кислота
- Фенiлоцтова кислота
- Лимонна кислота
- Валерiанова кислота
- Щавелєва кислота


Пiд час катаболiзму гiстидину утворюється бiогенний амiн, що має потужну судинорозширюючу дiю. Назвiть його:
+ Гiстамiн
- Серотонiн
- ДОФА
- Норадреналiн
- Дофамiн


В кровi хворого виявлено високий вмiст галактози, концентрацiя глюкози знижена. Вiдмiчена розумова вiдсталiсть, помутнiння кришталика. Яке захворювання має мiсце?
+ Галактоземiя
- Лактоземiя
- Цукровий дiабет
- Стероїдний дiабет
- Фруктоземiя


У хворої дитини виявлена затримка розумового розвитку, збiльшення печiнки, погiршення зору. Лiкар пов’язує цi симптоми з дефiцитом в органiзмi галактозо-1-фосфатуридилтрансферази. Який патологiчний процес має мiсце у дитини?
+ Галактоземiя
- Фруктоземiя
- Гiперглiкемiя
- Гiпоглiкемiя
- Гiперлактатацидемiя


У кровi дитини виявлено високий вмiст галактози, концентрацiя глюкози понижена. Спостерiгаються катаракта, розумова вiдсталiсть, розвивається жирове переродження печiнки. Яке захворювання має мiсце?
+ Галактоземiя
- Цукровий дiабет
- Лактоземiя
- Стероїдний дiабет
- Фруктоземiя


Пацiєнту 33 роки. Хворiє 10 рокiв. Перiодично звертається до лiкаря зi скаргами на гострий бiль у животi, судоми, порушення зору. У його родичiв спостерiгаються подiбнi симптоми. Сеча червоного кольору. Госпiталiзований з дiагнозом - гостра перемiжна порфирiя. Причиною захворювання може бути порушення бiосинтезу такої речовини:
+ Гем
- Iнсулiн
- Жовчнi кислоти
- Простагландини
- Колаген


У дитини спостерiгається затримка фiзичного та розумового розвитку, глибокi порушення з боку сполучної тканини внутрiшнiх органiв, у сечi виявлено кератансульфати. Обмiн яких речовин порушений?
+ Глiкозамiноглiкани
- Колаген
- Еластин
- Фiбронектин
- Гiалуронова кислота


Амiак є дуже отруйною речовиною, особливо для нервової системи. Яка речовина бере особливо активну участь у знешкодженнi амiаку в тканинах мозку?
+ Глутамiнова кислота
- Лiзин
- Пролiн
- Гiстидин
- Аланiн


У немовляти спостерiгаються епiлептиформнi судоми, викликанi дефiцитом вiтамiну B6. Це спричинено зменшенням у нервовiй тканинi гальмiвного медiатора - γ-амiномасляної кислоти. Активнiсть якого ферменту знижена при цьому?
+ Глутаматдекарбоксилаза
- Аланiнамiнотрансфераза
- Глутаматдегiдрогеназа
- Пiридоксалькiназа
- Глутаматсинтетаза


Пiд час голодування м’язовi бiлки розпадаються до вiльних амiнокислот. В який процес найiмовiрнiше будуть втягуватися цi сполуки за таких умов?
+ Глюконеогенез у печiнцi
- Глюконеогенез у м’язах
- Синтез вищих жирних кислот
- Глiкогенолiз
- Декарбоксилування


При лабораторному дослiдженнi дитини виявлено пiдвищений вмiст у кровi та сечi лейцину, валiну, iзолейцину та їх кетопохiдних. Сеча має характерний запах кленового сиропу. Недостатнiсть якого ферменту характерно для цього захворювання?
+ Дегiдрогеназа розгалужених амiнокислот
- Амiнотрансфераза
- Глюкозо-6-фосфатаза
- Фосфофруктокiназа
- Фосфофруктомутаза


Недостатнiсть в органiзмi лiнолевої та лiноленової кислот призводить до ушкоджень шкiри, випадiння волосся, сповiльненого загоювання ран, тромбоцитопенiї, зниження опiрностi до iнфекцiйних захворювань. Порушення синтезу яких речовин найiмовiрнiше зумовлює вказанi симптоми?
+ Ейкозаноїди
- Iнтерлейкiни
- Iнтерферони
- Катехоламiни
- Кортикостероїди


Мати звернулася до лiкаря з приводу того, що у дитини 5-ти рокiв пiд дiєю сонячних променiв на шкiрi з’являються еритеми, везикулярний висип, свербiж шкiри. Лабораторнi дослiдження виявили зменшення вмiсту залiза у сироватцi кровi, збiльшення видiлення з сечею уропорфiриногену I. Найбiльш вiрогiдною спадковою патологiєю у дитини є:
+ Еритропоетична порфiрiя
- Метгемоглобiнемiя
- Печiнкова порфiрiя
- Копропорфiрiя
- Iнтермiтуюча порфiрiя


При тривалому лiкуваннi голодуванням у пацiєнта зменшилося спiввiдношення альбумiнiв i глобулiнiв у плазмi кровi. Що з наведеного буде наслiдком цих змiн?
+ Збiльшення ШЗЕ
- Зниження ШЗЕ
- Збiльшення гематокритного показника
- Зниження гематокритного показника
- Гiперкоагуляцiя


Основна маса азоту з органiзму виводиться у виглядi сечовини. Зниження активностi якого ферменту в печiнцi призводить до гальмування синтезу сечовини i нагромадження амонiаку в кровi i тканинах?
+ Карбамоїлфосфатсинтаза
- Аспартатамiнотрансфераза
- Уреаза
- Амiлаза
- Пепсин


Хворому з прогресуючою м’язовою дистрофiєю було проведено бiохiмiчне дослiдження сечi. Поява якої речовини у великiй кiлькостi в сечi може пiдтвердити захворювання м’язiв у даного хворого?
+ Креатин
- Порфiрини
- Сечовина
- Гiпурова кислота
- Креатинiн


У юнака 18-ти рокiв дiагностована м’язова дистрофiя. Пiдвищення в сироватцi кровi вмiсту якої речовини найбiльш iмовiрне при цiй патологiї?
+ Креатин
- Мiоглобiн
- Мiозин
- Лактат
- Аланiн


Хворому з пiдозрою на дiагноз "прогресуюча м’язова дистрофiя"був зроблений аналiз сечi. Наявнiсть якої сполуки в сечi пiдтверджує дiагноз?
+ Креатин
- Колаген
- Порфiрин
- Мiоглобiн
- Кальмодулiн


Хвора 46-ти рокiв довгий час страждає на прогресуючу м’язову дистрофiю (Дюшена). Змiни рiвня якого ферменту кровi є дiагностичним тестом в даному випадку?
+ Креатинфосфокiназа
- Лактатдегiдрогеназа
- Пiруватдегiдрогеназа
- Глутаматдегiдрогеназа
- Аденiлаткiназа


Для профiлактики атеросклерозу, iшемiчної хвороби серця, порушень мозкового кровообiгу рекомендується споживання жирiв iз високим вмiстом полiненасичених жирних кислот. Однiєю з таких жирних кислот є:
+ Лiнолева
- Олеїнова
- Лауринова
- Пальмiтоолеїнова
- Стеаринова


У чоловiка, який тривалий час невживав з їжею жирiв, але отримував достатню кiлькiсть вуглеводiв i бiлкiв, виявлено дерматит, погане загоювання ран, погiршення зору. Дефiцит яких компонентiв є причиною порушення обмiну речовин?
+ Лiнолева кислота, вiтамiни A, D, E,K
- Пальмiтинова кислота
- Вiтамiни PP , H
- Мiнеральнi солi
- Олеїнова кислота


Бiохiмiчний аналiз сироватки кровi пацiєнта з гепатолентикулярною дегенерацiєю (хвороба ВiльсонаКоновалова) виявив зниження вмiсту церулоплазмiну. У цього пацiєнта в сироватцi кровi буде пiдвищена концентрацiя таких iонiв:
+ Мiдь
- Кальцiй
- Фосфор
- Калiй
- Натрiй


При нестачi бiотину спостерiгається порушення синтезу вищих жирних кислот. Утворення якого iз зазначених метаболiтiв може бути порушено при цьому?
+ Малонiл КоА
- Сукцинiл КоА
- Пiруват
- Аланiн
- Серотонiн


Стеатоз виникає внаслiдок накопичення триацилглiцеролiв у гепатоцитах. Одним з механiзмiв розвитку цього захворювання є зменшення утилiзацiї нейтрального жиру ЛПДНЩ. Якi лiпотропнi речовини попереджують розвиток стеатозу?
+ Метiонiн, B6 , B12
- Аргiнiн, B2 , B3
- Аланiн, B1 , P P
- Валiн, B3 , B2
- Iзолейцин, B1 , B2


Дитина 3-х рокiв iз симптомами стоматиту, гiнгiвiту, дерматиту вiдкритих дiлянок шкiри була госпiталiзована. При обстеженнi встановлено спадкове порушення транспорту нейтральних амiнокислот у кишечнику. Нестача якого вiтамiну зумовила данi симптоми?
+ Нiацин
- Пантотенова кислота
- Вiтамiн A
- Кобаламiн
- Бiотин


Пiд час патронажу лiкар виявив у дитини симетричну шорсткiсть щiк, дiарею, порушення нервової дiяльностi. Нестача яких харчових факторiв є причиною такого стану?
+ Нiкотинова кислота, триптофан
- Лiзин, аскорбiнова кислота
- Треонiн, пантотенова кислота
- Метiонiн, лiпоєва кислота
- Фенiлаланiн, пангамова кислота


При обстеженнi чоловiка 45-ти рокiв, що тривалий час перебував на рослиннiй дiєтi, виявлено негативний азотистий баланс. Яка особливiсть рацiону стала причиною цього явища?
+ Недостатня кiлькiсть бiлкiв
- Надмiрна кiлькiсть води
- Надмiрна кiлькiсть вуглеводiв
- Недостатня кiлькiсть жирiв
- Недостатня кiлькiсть жирiв i бiлкiв


Пiсля хiмiчного опiку в хворого розвинувся стеноз стравоходу. Виникло рiзке схуднення вiд затрудненого прийому їжi. У кровi: ер.- 3, 0 • 1012 /л, Hb106 г/л, загальний бiлок - 57 г/л. Який вид голодування в хворого?
+ Неповне
- Бiлкове
- Повне
- Водне
- Абсолютне


У дитини виявлена схильнiсть до ожирiння, яка є результатом дiатезу. Назвiть вид дiатезу, при якому частiше може розвинутись ожирiння:
+ Нервово-артритичний
- Ексудативно-катаральний
- Лiмфатико-гiпопластичний
- Астенiчний
-


Чоловiк 70-ти рокiв хворiє на атеросклероз судин нижнiх кiнцiвок та iшемiчну хворобу серця. Пiд час обстеження виявлено порушення лiпiдного складу кровi. Надлишок яких лiпопротеїнiв є головною ланкою в патогенезi атеросклерозу?
+ Низької щiльностi
- Холестерин
- Високої щiльностi
- Промiжної щiльностi
- Хiломiкрони


До лiкарнi доставлено дитину 2-х рокiв з уповiльненим розумовим i фiзичним розвитком, що страждає на частi блювання пiсля прийому їжi. У сечi визначена фенiлпiровиноградна кислота. Наслiдком якого порушення є дана патологiя?
+ Обмiн амiнокислот
- Лiпiдний обмiн
- Вуглеводний обмiн
- Водно-сольовий обмiн
- Фосфорно-кальцiєвий обмiн


Дихальний коефiцiєнт у хворого складає 0,7. Це свiдчить, що у клiтинах людини переважає:
+ Окислення жирiв
- Окислення вуглеводiв
- Окислення бiлкiв
- Змiшане окислення жирiв та вуглеводiв
- Змiшане окислення жирiв та бiлкiв


В експериментi на кролi встановлено, що об’єм кисню, який споживається головним мозком за 1 хвилину, дорiвнює об’єму CO 2 , який видiляється клiтинами мозку в кров. Це свiдчить, що у клiтинах головного мозку має мiсце:
+ Окиснення вуглеводiв
- Окиснення жирiв
- Гiпокапнiя
- Гiпоксiя
- Окиснення бiлкiв


Мати зауважила занадто темну сечу у її 5-рiчної дитини. Дитина скарг не висловлює. Жовчних пiгментiв у сечi не виявлено. Поставлено дiагноз алкаптонурiя. Дефiцит якого ферменту має мiсце у дитини?
+ Оксидаза гомогентизинової кислоти
- Фенiлаланiнгiдроксилаза
- Тирозиназа
- Оксидаза оксифенiлпiрувату
- Декарбоксилаза фенiлпiрувату


У хворого дiагностовано алкаптонурiю. Вкажiть фермент, дефект якого є причиною цiєї патологiї:
+ Оксидаза гомогентизинової кислоти
- Фенiлаланiнгiдроксилаза
- Глутаматдегiдрогеназа
- Пiруватдегiдрогеназа
- ДОФА-декарбоксилаза


Хвора 36-ти рокiв страждає на колагеноз. Збiльшення вмiсту якого метаболiту найбiльш iмовiрно буде встановлено у сечi?
+ Оксипролiн
- Iндикан
- Креатинiн
- Сечовина
- Уробiлiноген


У пацiєнта стоматологiчного вiддiлення виявлено хворобу Педжета, що супроводжується деградацiєю колагену. Вирiшальним фактом для постановки дiагнозу було виявлення у сечi хворого пiдвищеного рiвня:
+ Оксипролiну
- Аргiнiну
- Триптофану
- Серину
- Аланiну


У новонародженої дитини спостерiгаються зниження iнтенсивностi смоктання, часте блювання, гiпотонiя. У сечi та кровi значно пiдвищена концентрацiя цитрулiну. Який метаболiчний процес порушений?
+ Орнiтиновий цикл
- ЦТК
- Глiколiз
- Глюконеогенез
- Цикл Корi


При обстеженнi жiнки 56-ти рокiв, що хвора на цукровий дiабет 1-го типу, виявлене порушення бiлкового обмiну, що при лабораторному дослiдженнi кровi проявляється амiноацидемiєю, а клiнiчно - уповiльненням загоєння ран i зменшенням синтезу антитiл. Який з перерахованих механiзмiв викликає розвиток амiноацидемiї?
+ Пiдвищення протеолiзу
- Гiперпротеїнемiя
- Зменшення концентрацiї амiнокислот у кровi
- Пiдвищення онкотичного тиску в плазмi кровi
- Збiльшення лiпопротеїдiв високої щiльностi


У кровi хворого виявлено пiдвищення активностi ЛДГ 4,5, АлАТ, карбамоїлорнiтинтрансферази. В якому органi можна передбачити розвиток патологiчного процесу?
+ Печiнка (можливий гепатит)
- Серцевий м’яз (можливий iнфаркт мiокарда)
- Скелетнi м’язи
- Нирки
- Сполучна тканина


У хлопчика 4-х рокiв пiсля перенесеного важкого вiрусного гепатиту мають мiсце блювання, втрата свiдомостi, судоми. У кровi - гiперамонiємiя. Порушення якого бiохiмiчного процесу викликало патологiчний стан хворого?
+ Порушення знешкодження амiаку в печiнцi
- Порушення знешкодження бiогенних амiнiв
- Посилення гниття бiлкiв у кишечнику
- Активацiя декарбоксилування амiнокислот
- Пригнiчення ферментiв трансамiнування


Пацiєнт 46-ти рокiв звернувся до лiкаря зi скаргами на болi в дрiбних суглобах нiг та рук. Суглоби збiльшенi, мають вигляд потовщених вузлiв. У сироватцi встановлено пiдвищений вмiст уратiв. Це може бути спричинене:
+ Порушенням обмiну пуринiв
- Порушенням обмiну вуглеводiв
- Порушенням обмiну лiпiдiв
- Порушенням обмiну пiримiдинiв
- Порушенням обмiну амiнокислот


При дослiдженнi сироватки кровi хворого виявлене пiдвищення рiвня аланiнамiнотрансферази (АЛТ) та аспартатамiнотрансферази (АСТ). Якi змiни на клiтинному рiвнi можуть призвести до подiбної ситуацiї?
+ Руйнування клiтин
- Порушення функцiї енергозабезпечення клiтин
- Порушення ферментних систем клiтин
- Руйнування генетичного апарату клiтин
- Порушення мiжклiтинних взаємовiдношень


Чоловiк 65-ти рокiв, який страждає на подагру, скаржиться на бiль в дiлянцi нирок. При ультразвуковому обстеженнi встановлена наявнiсть ниркових каменiв. Пiдвищення концентрацiї якої речовини є найбiльш вiрогiдною причиною утворення каменiв у даному випадку?
+ Сечова кислота
- Холестерин
- Бiлiрубiн
- Сечовина
- Цистин


В сечi новонародженого визначається цитрулiн та високий рiвень амiаку. Вкажiть, утворення якої речовини, найiмовiрнiше, порушене у цього малюка:
+ Сечовина
- Сечова кислота
- Амiак
- Креатинiн
- Креатин


На основi лабораторного аналiзу у хворого пiдтверджено дiагноз - подагра. Для встановлення дiагнозу було проведено визначення вмiсту:
+ Сечової кислоти в кровi та сечi
- Креатинiну в сечi
- Залишкового азоту в кровi
- Сечовини в кровi та сечi
- Амiаку в сечi


У 19-мiсячної дитини iз затримкою розвитку та проявами самоагресiї, вмiст сечової кислоти в кровi - 1,96 ммоль/л. При якому метаболiчному порушеннi це спостерiгається?
+ Синдром Леша-Нiхана
- Подагра
- Синдром набутого iмунодефiциту
- Хвороба Гiрке
- Хвороба Iценко-Кушiнга


На прийом до терапевта прийшов чоловiк 37-ми рокiв зi скаргами на перiодичнi iнтенсивнi больовi напади у суглобах великого пальця стопи та їх припухлiсть. У сечi: рiзко кисла реакцiя I рожеве забарвлення. З наявнiстю яких речовин можуть бути пов’язанi такi змiни?
+ Солi сечової кислоти
- Хлориди
- Амонiєвi солi
- Фосфат кальцiю
- Сульфат магнiю


Порушення процесiв мiєлiнiзацiї нервових волокон призводить до неврологiчних розладiв i розумової вiдсталостi. Такi симптоми характернi для спадкових i набутих порушень обмiну:
+ Сфiнголiпiдiв
- Нейтральних жирiв
- Вищих жирних кислот
- Холестерину
- Фосфатидної кислоти


У жiнки, що тривалий час дотримувалася дiєти з використанням очищеного рису, виявлений полiневрит (хвороба Берi-Берi). Вiдсутнiсть якого вiтамiну в їжi призводить до розвитку цього захворювання?
+ Тiамiн
- Аскорбiнова кислота
- Пiридоксин
- Фолiєва кислота
- Рибофлавiн


Внаслiдок дефiциту вiтамiну B1 порушується окисне декарбоксилювання α-кетоглутарової кислоти. Синтез якого з наведених коферментiв порушується при цьому?
+ Тiамiнпiрофосфат
- Нiкотинамiдаденiндинуклеотид
- Флавiнаденiндинуклеотид
- Лiпоєва кислота
- Коензим А


При алкаптонурiї вiдбувається надмiрне видiлення гомогентизинової кислоти iз сечею. С порушенням метаболiзму якої амiнокислоти пов’язано виникнення цього захворювання?
+ Тирозин
- Фенiлаланiн
- Аланiн
- Метiонiн
- Аспарагiн


У новонародженої дитини на пелюшках виявленi темнi плями, що свiдчать про утворення гомогентизинової кислоти. З порушенням обмiну якої речовини це пов’язане?
+ Тирозин
- Галактоза
- Метiонiн
- Холестерин
- Триптофан


У хворого в органiзмi знижений вмiст iонiв магнiю, якi потрiбнi для прикрiплення рибосом до гранулярної ендоплазматичної сiтки. Вiдомо, що це призводить до порушення бiосинтезу бiлка. Який саме етап бiосинтезу бiлка буде порушено?
+ Трансляцiя
- Транскрипцiя
- Реплiкацiя
- Активацiя амiнокислот
- Термiнацiя


До клiнiки потрапила дитина 1-го року з ознаками ураження м’язiв кiнцiвок та тулуба. Пiсля обстеження виявлений дефiцит карнiтину в м’язах. Бiохiмiчною основою цiєї патологiї є порушення процесу:
+ Транспорту жирних кислот у мiтохондрiї
- Регуляцiї рiвня Ca 2
+ в мiтохондрiях
- Субстратного фосфорилювання
- Утилiзацiї молочної кислоти
- Окисного фосфорилювання


Причиною захворювання на пелагру може бути переважне харчування кукурудзою i зниження у рацiонi продуктiв тваринного походження. Вiдсутнiсть у рацiонi якої амiнокислоти призводить до даної патологiї?
+ Триптофан
- Iзолейцин
- Фенiлаланiн
- Метiонiн
- Гiстидин


Пiсля обстеження хворому на сечокам’яну хворобу призначили алопурiнол - конкурентний iнгiбiтор ксантиноксидази. Пiдставою для цього був хiмiчний аналiз ниркових каменiв, переважною складовою яких є:
+ Урат натрiю
- Дигiдрат оксалату кальцiю
- Моногiдрат оксалату кальцiю
- Фосфат кальцiю
- Сульфат кальцiю


До лiкарнi надiйшов 9-рiчний хлопчик розумово i фiзично вiдсталий. При бiохiмiчному дослiдженнi кровi: пiдвищена кiлькiсть фенiлаланiну. Блокування якого ферменту може призвести до такого стану?
+ Фенiлаланiн-4-монооксигеназа
- Оксидаза гомогентизинової кислоти
- Глутамiнтрансамiназа
- Аспартатамiнотрансфераза
- Глутаматдекарбоксилаза


У дитини 1,5 рокiв спостерiгається вiдставання в розумовому i фiзичному розвитку, посвiтлiння шкiри i волосся, зниження вмiсту в кровi катехоламiнiв. При додаваннi до свiжої сечi декiлькох крапель 5% розчину трихлороцтового залiза з’являється оливково-зелене забарвлення. Для якої патологiї обмiну амiнокислот характернi данi змiни?
+ Фенiлкетонурiя
- Алкаптонурiя
- Тирозиноз
- Альбiнiзм
- Ксантонурiя


Одна з форм вродженої патологiї супроводжується гальмуванням перетворення фенiлаланiну в тирозин. Бiохiмiчною ознакою хвороби є накопичення в органiзмi деяких органiчних кислот, зокрема:
+ Фенiлпiровиноградна
- Лимонна
- Пiровиноградна
- Молочна
- Глутамiнова


У жiнки 36-ти рокiв має мiсце гiповiтамiноз B2 . Причиною виникнення специфiчних симптомiв (ураження епiтелiю, слизових оболонок, шкiри, рогiвки ока) iмовiрно є дефiцит:
+ Флавiнових коферментiв
- Цитохрому А1
- Цитохромоксидази
- Цитохрому В
- Цитохрому С


Спадкова гiперлiпопротеїнемiя I типу обумовлена недостатнiстю лiпопротеїнлiпази. Пiдвищення рiвня яких транспортних форм лiпiдiв в плазмi навiть натщесерце є характерним?
+ Хiломiкрони
- Лiпопротеїни низької густини
- Лiпопротеїни дуже низької густини
- Лiпопротеїни високої густини
- Модифiкованi лiпопротеїни


До лiкаря звернулась мати з приводу поганого самопочуття дитини - вiдсутнiсть апетиту, поганий сон, дратiвливiсть. При бiохiмiчному дослiдженнi в кровi виявлено вiдсутнiсть ферменту глюкоцереброзидази. Для якої патологiї це характерно?
+ Хвороба Гоше
- Хвороба Тея-Сакса
- Хвороба Нiмана-Пiка
- Хвороба Гiрке
- Хвороба Помпе


До лiкарнi надiйшла дитина 6-ти рокiв. Пiд час обстеження було виявлено, що дитина не може фiксувати погляд, не слiдкує за iграшками, на очному днi вiдзначається симптом "вишневої кiстки". Лабораторнi аналiзи показали, що у мозку, печiнцi та селезiнцi - пiдвищений рiвень ганглiозиду глiкометиду. Яке спадкове захворювання у дитини?
+ Хвороба Тея-Сакса
- Хвороба Вiльсона-Коновалова
- Синдром Шерешевського-Тернера
- Хвороба Нiмана-Пiка
- Хвороба Мак-Аргдля


У культурi клiтин, отриманих вiд хворого з лiзосомною патологiєю, виявлено накопичення значної кiлькостi лiпiдiв у лiзосомах. При якому з перелiчених захворювань має мiсце це порушення?
+ Хвороба Тея-Сакса
- Подагра
- Фенiлкетонурiя
- Хвороба Вiльсона-Коновалова
- Галактоземiя


Пiд час огляду дитини 11-ти мiсяцiв педiатр виявив викривлення кiсток нижнiх кiнцiвок i затримку мiнералiзацiї кiсток черепа. Нестача якого вiтамiну призводить до даної патологiї?
+ Холекальциферол
- Тiамiн
- Пантотенова кислота
- Бiофлавоноїди
- Рибофлавiн


При хворобi Вiльсона-Коновалова порушується транспорт мiдi, що призводить до накопичення цього металу в клiтинах мозку та печiнки. З порушенням синтезу якого бiлку це пов’язано?
+ Церулоплазмiн
- Металотiонеїн
- Транскобаламiн
- Гаптоглобiн
- Сидерофiлiн


У хворого 27-ми рокiв виявлено патологiчнi змiни печiнки i головного мозку. У плазмi кровi виявлено рiзке зниження, а в сечi - пiдвищення вмiсту мiдi. Встановлено дiагноз – хвороба Вiльсона. Активнiсть якого ферменту в сироватцi кровi необхiдно дослiдити для пiдтвердження дiагнозу?
+ Церулоплазмiн
- Карбоангiдраза
- Ксантиноксидаза
- Лейцинамiнопептидаза
- Алкогольдегiдрогеназа


У 12-ти рiчного хлопчика в сечi виявлено високий вмiст усiх амiнокислот алiфатичного ряду. При цьому вiдмiчена найбiльш висока екскрецiя цистину та цистеїну. Крiм того, УЗД нирок показало наявнiсть каменiв у них. Яка патологiя найбiльш вiрогiдна?
+ Цистинурiя
- Алкаптонурiя
- Цистит
- Фенiлкетонурiя
- Хвороба Хартнупа


ПМК-2


Повний тест (А - правильний варіант)


У трирiчної дитини з пiдвищеною температурою тiла пiсля прийому аспiрину спостерiгається посилений гемолiз еритроцитiв. Вроджена недостатнiсть якого ферменту могла викликати у дитини гемолiтичну анемiю?
+Глюкозо-6-фосфатдегiдрогеназа
-Глюкозо-6-фосфатаза
-Глiкогенфосфорилаза
-Глiцеролфосфатдегiдрогеназа
-γ-глутамiлтрансфераза


У хворого пiсля оперативного видалення кiсти пiдшлункової залози виник геморагiчний синдром з вираженим порушенням зсiдання кровi. Розвиток цього ускладнення пояснюється:
+ Активацiєю фiбринолiтичної системи
- Недостатнiм утворенням фiбрину
- Зменшенням кiлькостi тромбоцитiв
- Активацiєю протизгортальної системи
- Активацiєю фактору Крисмаса


У хворого 37-ми рокiв на фонi тривалого застосування антибiотикiв спостерiгається пiдвищена кровоточивiсть при невеликих пошкодженнях. У кровi зниження активностi факторiв згортання кровi II, VII, IX, X, подовження часу згортання кровi. Недостатнiстю якого вiтамiну обумовленi зазначенi змiни?
+ Вiтамiн K
- Вiтамiн C
- Вiтамiн D
- Вiтамiн E
- Вiтамiн B6


У хлопчика 3-х рокiв з вираженим геморагiчним синдромом вiдсутнiй антигемофiльний глобулiн А (фактор VIII) у плазмi кровi. Яка фаза гемостазу первинно порушена у цього хворого?
+ Внутрiшнiй механiзм активацiї протромбiнази
- Зовнiшнiй механiзм активацiї протромбiнази
- Перетворення протромбiну в тромбiн
- Перетворення фiбриногену в фiбрин
- Ретракцiя кров’яного згустку


У хворого опiкова хвороба ускладнилася ДВЗ-синдромом. Яку стадiю ДВЗ-синдрому можна запiдозрити, якщо вiдомо, що кров хворого згортається менше нiж за 3 хвилини?
+ Гiперкоагуляцiї
- Перехiдна
- Гiпокоагуляцiї
- Фiбринолiз
- Термiнальна


У хворого iз захворюванням печiнки виявлено зниження вмiсту протромбiну в кровi. Це призведе, перш за все, до порушення:
+ Другої фази коагуляцiйного гемостазу
- Першої фази коагуляцiйного гемостазу
- Судинно-тромбоцитарного гемостазу
- Фiбринолiзу
- Антикоагулянтних властивостей кровi


Видалення зуба у пацiєнта з хронiчним персистуючим гепатитом ускладнилось тривалою кровотечею. Яка причина геморагiчного синдрому?
+ Зменшення утворення тромбiну
- Збiльшення утворення тромбопластину
- Зменшення утворення фiбрину
- Збiльшення синтезу фiбриногену
- Посилення фiбринолiзу


У лiквiдатора наслiдкiв аварiї на АЕС пiд час перебiгу гострої променевої хвороби виник геморагiчний синдром. Що має найбiльше значення в патогенезi цього синдрому?
+ Тромбоцитопенiя
- Порушення структури стiнки судин
- Пiдвищення активностi факторiв фiбринолiзу
- Пiдвищення активностi факторiв систем протизсiдання кровi
- Зменшення активностi факторiв зсiдання кровi


У людини до травми гематокритний показник 40%. Яким вiн буде через добу пiсля втрати 750 мл кровi?
+ 0.3
- 0.4
- 0.55
- 0.45
- 0.5


У чоловiка 30-ти рокiв перед операцiєю визначили групову належнiсть кровi. Кров резус-позитивна. Реакцiю аглютинацiї еритроцитiв не викликали стандартнi сироватки груп 0 αβ (I), Аβ (II), Вα (III). Дослiджувана кров належить до групи:
+ 0 αβ (I)
- Аβ (II)
- Вα (III)
- АВ (IV)
-


У чоловiка 30-ти рокiв перед операцiєю визначили групову належнiсть кровi. Кров резус-позитивна. Реакцiю аглютинацiї еритроцитiв не викликали стандартнi сироватки груп 0αβ (I), Аβ (II), Вα (III). Дослiджувана кров належить до групи:
+ 0αβ (I)
- Аβ (II)
- Вα (III)
- АВ (IV)
-


На останньому мiсяцi вагiтностi вмiст фiбриногену в плазмi кровi в 2 рази вище за норму. Яку швидкiсть осiдання еритроцитiв слiд при цьому очiкувати?
+ 40-50 мм/годину
- 0-5 мм/годину
- 10-15 мм/годину
- 5-10 мм/годину
- 3-12 мм/годину


У вагiтної жiнки визначили групу кровi. Реакцiя аглютинацiї еритроцитiв вiдбулася зi стандартними сироватками груп 0αβ (I), Bα (III) i не вiдбулася зi стандартною сироваткою групи Aβ (II). Дослiджувана кров належить до групи:
+ Aβ (II)
- 0αβ (I)
- Bα (III)
- AB (IV )
-


Встановлено, що аглютинацiя еритроцитiв кровi реципiєнта викликали стандартнi сироватки I та II груп i не викликали - сироватка III групи I антирезусна сироватка. Кров якої групи за системами AB0 i резус можна переливати реципiєнту?
+ B, α (III) Rh−
- A, β (II) Rh−
- 0, α, β, (I) Rh+
- AB (IV ), Rh+
- AB (IV ), Rh−


Чоловiк 56-ти рокiв потрапив до клiнiки зi скаргами на загальну слабкiсть, бiль i печiння в язицi, вiдчуття онiмiння в кiнцiвках. У минулому перенiс резекцiю кардiального вiддiлу шлунка. У кровi: Hb- 80 г/л; ер.- 2, 0 • 10 12 /л; КП1,2, лейк.- 3, 5 • 10 9 /л. Який вид анемiї у цього хворого?
+ B12 -фолiєводефiцитна
- Гемолiтична
- Постгеморагiчна
- Апластична
- Залiзодефiцитна


Дiвчинка 10-ти рокiв часто хворiє на гострi респiраторнi iнфекцiї, пiсля яких спостерiгаються множиннi точковi крововиливи в мiсцях тертя одягу. Який гiповiтамiноз має мiсце в дiвчинки:
+ C
- B6
- B1
- A
- B2


Пацiєнт страждає на геморагiчний синдром, що проявляється частими носовими кровотечами, посттравматичними та спонтанними внутрiшньошкiрними та внутрiшньосуглобовими крововиливами. Пiсля лабораторного обстеження було дiагностовано гемофiлiю В. Дефiцит якого фактора згортання кровi обумовлює дане захворювання?
+ IX
- VIII
- XI
- V
- VII


У хворого спостерiгаються геморагiї, в кровi знижена концентрацiя протромбiну. Недостатнiсть якого вiтамiну призвела до порушення синтезу цього фактору згортання кровi?
+ K
- A
- D
- C
- E


У хворих з непрохiднiстю жовчовивiдних шляхiв пригнiчується зсiдання кровi, виникають кровотечi, що є наслiдком недостатнього засвоєння такого вiтамiну:
+ K
- A
- D
- E
- C


У вагiтної жiнки визначили групову належнiсть кровi. Реакцiю аглютинацiї еритроцитiв викликали стандартнi сироватки I, III груп, та не викликала сироватка II групи. Якою є група дослiджуваної кровi за системою АВ0?
+ А(II), β
- В(III), α
- О(I), α, β
- АВ (IV)
-


Пiсля прийому сульфанiламiдiв у хворого виникли лихоманка, блювання i стул з кров’ю. У кровi: лейк. 0, 9•109/л (гранул. - 0,7•109/л), лейкоаглютинiни. Який з термiнiв найбiльш точно характеризує виявленi змiни у кровi?
+ Агранулоцитоз
- Лейкопенiя
- Лейкоз
- Гемодилюцiя
-


У хворого, що перенiс 5 рокiв тому субтотальну резекцiю шлунка, розвинулась B12-фолiєводефiцитна анемiя. Який механiзм є провiдним у розвитку такої анемiї?
+ Вiдсутнiсть внутрiшнього фактора Касла
- Вiдсутнiсть зовнiшнього фактора Касла
- Порушення всмоктування вiтамiну B12 в тонкiй кишцi
- Дефiцит фолiєвої кислоти
- Дефiцит транскобаламiну


У хворої 36-ти рокiв, яка лiкувалася сульфанiламiдами з приводу респiраторної вiрусної iнфекцiї, в кровi гiпорегенераторна нормохромна анемiя, лейкопенiя, тромбоцитопенiя. В кiстковому мозку - зменшення кiлькостi мiєлокарiоцитiв. Яка це анемiя?
+ Гiпопластична
- Гемолiтична
- Постгеморагiчна
- B12-фолiєводефiцитна
- Залiзодефiцитна


Пiсля оперативного видалення частини шлунка у хворого порушилося всмоктування вiтамiну B 12 , вiн виводиться з калом. Розвинулася анемiя. Який фактор необхiдний для всмоктування цього вiтамiну?
+ Гастромукопротеїн
- Гастрин
- Соляна кислота
- Пепсин
- Фолiєва кислота


У районах Пiвденної Африки у людей розповсюджена серпоподiбноклiтинна анемiя, при якiй еритроцити мають форму серпа внаслiдок замiни в молекулi гемоглобiну амiнокислоти глутамiну на валiн. Чим викликана ця хвороба?
+ Генна мутацiя
- Порушення механiзмiв реалiзацiї генетичної iнформацiї
- Кросинговер
- Геномнi мутацiї
- Трансдукцiя


У хворого на мiкросфероцитарну гемолiтичну анемiю (хворобу Мiнковського-Шоффара), внаслiдок пiдвищення проникливостi мембрани еритроцитiв, у клiтину надходять iони натрiю та вода. Еритроцити набувають форму сфероцитiв i легко руйнуються. Який провiдний механiзм пошкодження еритроцитiв має мiсце в даному випадку?
+ Електролiтно-осмотичний
- Кальцiєвий
- Ацидотичний
- Протеїновий
- Нуклеїновий


Дитина 6-ти рокiв знаходиться на стацiонарному лiкуваннi з дiагнозом алергiчного ринiту. В кровi: змiни в лейкоцитарнiй формулi. Кiлькiсть яких клiтин лейкоцитарного ряду може бути збiльшена?
+ Еозинофiли
- Т-лiмфоцити
- В-лiмфоцити
- Базофiли
- Нейтрофiли


При аналiзi кровi у спортсмена виявлено: ер.- 5, 5 • 10 12 /л, Hb- 180 г/л, лейк.- 7 • 10 9 /л, н.- 64%, б.- 0,5%, е.- 0,5%, м.- 8%, л.- 27%. Такi показники свiдчать про стимуляцiю, перш за все:
+ Еритропоезу
- Лейкопоезу
- Лiмфопоезу
- Гранулоцитопоезу
- Iмуногенезу


В пробiрку, що мiстить розчин NaCl 0,9%, додана крапля кровi. Що вiдбудеться з еритроцитами?
+ Залишаться без змiн
- Осмотичний гемолiз
- Бiологiчний гемолiз
- Зморшкування
- Набухання


До приймального вiддiлення доставлено жiнку 38-ми рокiв з матковою кровотечею, що триває другу добу. Що з наведеного буде виявлено при аналiзi кровi хворої?
+ Зменшення гематокритного показника
- Еозинофiлiя
- Уповiльнення ШОЕ
- Лейкоцитоз
- Збiльшення колiрного показника


У приймально-дiагностичне вiддiлення доставили жiнку 38-ми рокiв з шлунковою кровотечею. Якi змiни найбiльш iмовiрнi з боку кровi через добу?
+ Зменшення гематокритного числа
- Лейкоцитоз
- Еритроцитоз
- Лейкопенiя
- Збiльшення гематокритного числа


У чоловiка 43-х рокiв з видаленою ниркою були виявленi симптоми анемiї. Що зумовило появу цих симптомiв?
+ Зниження синтезу еритропоетинiв
- Пiдвищене руйнування еритроцитiв
- Нестача залiза
- Нестача вiтамiну B12
- Нестача фолiєвої кислоти


У хворого в лейкограмi: лейкоцити - 14 • 10 9 /л; мiєлобласти - 71%, промiєлоцити, мiєлоцити, метамiєлоцити - 0%, паличкоядернi нейтрофiли - 6%, сегментоядернi - 13%; лiмфоцити - 7%, моноцити - 3%. Яка патологiя у хворого?
+ Мiєлобластний лейкоз
- Нейтрофiльний лейкоцитоз
- Хронiчний мiєлолейкоз
- Лiмфобластний лейкоз
- Хронiчний лiмфолейкоз


У хворого в обох щелепах рентгенологiчно виявлено численнi дефекти у виглядi гладкостiнних округлих отворiв. При гiстологiчному дослiдженнi - явища остеолiзису i остеопорозу при явищах слабкого кiсткоутворення. В сечi хворого знайдено бiлок БенсДжонса. Назвiть захворювання:
+ Мiєломна хвороба
- Хронiчний мiєлолейкоз
- Хронiчний еритромiєлоз
- Гострий мiєлолейкоз
- Гострий недиференцiйований лейкоз


Батьки для профiлактики кишкових iнфекцiй у дитини 3-х рокiв тривало застосовували антибiотики. Через мiсяць стан дитини погiршився. У кровi виражена лейкопенiя i гранулоцитопенiя. Який найбiльш вiрогiдний механiзм виявлених змiн у кровi?
+ Мiєлотоксичний
- Аутоiмунний
- Перерозподiльний
- Вiковий
- Гемолiтичний


Лiкар при дослiдженнi мазку кровi у пацiєнта з анемiєю встановив дiагноз - спадкова гемолiтична анемiя Мiнковського-Шофара. Виявлення у кровi яких характерних клiтин надало можливiсть лiкарю встановити дiагноз?
+ Мiкросфероцити
- Полiхроматофiли
- Мегалоцити
- Пойкiлоцити
- Анiзоцити


Хворий 2 роки тому перенiс операцiю резекцiї пiлоричного вiддiлу шлунка. Спостерiгається слабкiсть, перiодична поява темних кiл пiд очима, задишка. У кровi: Hb- 70 г/л, ер.- 3, 0 • 1012 /л, КП- 0,7. Якi змiни еритроцитiв у мазках кровi найбiльш характернi для даного стану?
+ Мiкроцити
- Мегалоцити
- Шизоцити
- Овалоцити
- Макроцити


У результатi радiацiйного випромiнювання ушкодженi стовбуровi гемопоетичнi клiтини. Утворення яких клiтин сполучної тканини буде порушено?
+ Макрофаги
- Фiбробласти
- Адiпоцити
- Меланоцити
- Перiцити


У чоловiка 50-ти рокiв при обстеженнi було виявлено зниження кiлькостi еритроцитiв у кровi та пiдвищення рiвня вiльного гемоглобiну в плазмi кровi (гемоглобiнемiя). КП становив 0,85. Який вид анемiї спостерiгається у хворого?
+ Набута гемолiтична
- Спадкова гемолiтична
- Гостра постгеморагiчна
- Хронiчна постгеморагiчна
- Анемiя внаслiдок порушення еритропоезу


Ураження хворого одноразовою дозою iонiзуючого випромiнювання спричинило розвиток кiстковомозкової форми променевої хвороби. Якi патологiчнi прояви з боку кровi будуть характерними в перiод удаваного благополуччя?
+ Наростаюча лiмфопенiя, лейкопенiя
- Перерозподiльчий лейкоцитоз, лiмфоцитоз
- Анемiя, лейкопенiя
- Тромбоцитопенiя, анемiя
- Тромбоцитопенiя, лейкоцитоз


Хворий 23-х рокiв скаржиться на слабкiсть, пiдвищення температури до 38 − 40℃. Об’єктивно: печiнка i селезiнка збiльшенi. У кровi: Hb- 100 г/л, ер.- 2, 9 • 1012 /л, лейк.- 4, 4 • 109/л, тромб.48 • 109/л, нейтрофiли сегментоядернi 17%, лiмфоцити - 15%, бластнi клiтини - 68%. Всi цитохiмiчнi реакцiї негативнi. Дайте гематологiчний висновок:
+ Недиференцiйований лейкоз
- Хронiчний мiєлолейкоз
- Гострий мiєлобластний лейкоз
- Гострий лiмфобластний лейкоз
- Гострий еритромiєлоз


У хворого, що надiйшов до хiрургiчного вiддiлення з ознаками гострого апендициту, виявленi наступнi змiни бiлої кровi: загальна кiлькiсть лейкоцитiв - 16 • 10 9 /л. Лейкоцитарна формула: б.- 0, е.- 2%, ю.- 2%, п.- 8%, с.- 59%, л.25%, м.- 4%. Як класифiкуються зазначенi змiни?
+ Нейтрофiлiя з регенеративним зсувом влiво
- Нейтрофiлiя з зсувом вправо
- Нейтрофiлiя з дегенеративним зсувом влiво
- Лейкемоїдна реакцiя за нейтрофiльним типом
- Нейтрофiлiя з гiперрегенеративним зсувом влiво


Пiд час роботи щодо лiквiдацiї наслiдкiв аварiї на АЕС, робiтник одержав дозу опромiнення 500 рентген. Скаржиться на головний бiль, нудоту, запаморочення. Якi змiни кiлькостi лейкоцитiв можна очiкувати в хворого через 10 годин пiсля опромiнення?
+ Нейтрофiльний лейкоцитоз
- Лiмфоцитоз
- Лейкопенiя
- Агранулоцитоз
- Лейкемiя


Жiнка 62-х рокiв скаржиться на частий бiль у дiлянцi грудної клiтки та хребта, переломи ребер. Лiкар припустив мiєломну хворобу (плазмоцитому). Який з перерахованих нижче лабораторних показникiв буде мати найбiльше дiагностичне значення?
+ Парапротеїнемiя
- Гiперальбумiнемiя
- Протеїнурiя
- Гiпоглобулiнемiя
- Гiпопротеїнемiя


У студента через 2 години пiсля iспиту в аналiзi кровi виявлено лейкоцитоз без iстотних змiн у лейкоцитарнiй формулi. Який найбiльш вiрогiдний механiзм розвитку лейкоцитозу?
+ Перерозподiл лейкоцитiв в органiзмi
- Посилення лейкопоезу
- Уповiльнення руйнування лейкоцитiв
- Уповiльнення мiграцiї лейкоцитiв у тканини
- Посилення лейкопоезу та зменшення руйнування лейкоцитiв


Чоловiк 26-ти рокiв перебуває в торпiднiй стадiї шоку внаслiдок автомобiльної аварiї. В кровi: лейк.- 3, 2 10 9 /л. Який головний механiзм в розвитку лейкопенiї?
+ Перерозподiл лейкоцитiв у судинному руслi
- Пригнiчення лейкопоезу
- Порушення виходу зрiлих лейкоцитiв з кiсткового мозку в кров
- Руйнування лейкоцитiв у кровотворних органах
- Пiдвищення видiлення лейкоцитiв з органiзму


З метою оцiнки адаптацiї до фiзичного навантаження лiкар провiв обстеження робiтникiв пiсля виконання важкої працi. Якi змiни в загальному аналiзi кровi можна виявити?
+ Перерозподiльчий лейкоцитоз
- Лейкопенiя
- Анемiя
- Гiпоальбумiнемiя
- Зсув лейкоцитарної формули влiво


У вагiтної жiнки 24-х рокiв пiсля тривалого блювання було зареєстровано зниження об’єму циркулюючої кровi. Про яку змiну загальної кiлькостi кровi може йти мова?
+ Полiцитемiчна гiповолемiя
- Проста гiповолемiя
- Олiгоцитемiчна гiповолемiя
- Полiцитемiчна гiперволемiя
- Олiгоцитемiчна гiперволемiя


У хворого через добу пiсля апендектомiї при аналiзi кровi виявили нейтрофiльний лейкоцитоз з регенеративним зсувом влiво. Який найбiльш iмовiрний механiзм розвитку абсолютного лейкоцитозу у периферичнiй кровi хворого?
+ Посилення лейкопоезу
- Перерозподiл лейкоцитiв в органiзмi
- Зменшення руйнування лейкоцитiв
- Уповiльнення емiграцiї лейкоцитiв у тканинi
- Активацiя iмунiтету


У пацiєнта встановлено гiповiтамiноз фолiєвої кислоти, що може призвести до порушення синтезу:
+ Пуринових та тимiдилових нуклеотидiв
- Пуринових нуклеотидiв та холестерину
- Тимiдилових нуклеотидiв та жирних кислот
- Гема та креатину
- Цитрату та кетонових тiл


У хворого скарги на загальну слабкiсть, пiдвищену втому, зниження апетиту i маси тiла. В анамнезi частi пневмонiї. На пiдставi клiнiчних даних та результатiв дослiдження периферiйної кровi у нього дiагностовано хронiчний лiмфолейкоз. Якi дегенеративнi змiни лейкоцитiв характернi для даного захворювання?
+ Тiнi Боткiна-Гумпрехта
- Тiльця Князьковi-Деле
- Зерна Амато
- Палички Ауера
- Токсична зернистiсть


У хворого в анамнезi: з дитинства вiдмiчався знижений рiвень гемоглобiну. Лiкування препаратами залiза не дає ефекту. У кровi: ер.- 3, 1 • 10 12 /л, ретик.16%, Hb- 85 г/л, КП- 0,75; в мазку кровi анiзоцити, пойкiлоцити, мiшенеподiбнi еритроцити, еритроцити з базофiльною зернистiстю, рiвень залiза у сироватцi 30 мкмоль/л. Для якої патологiї системи кровi характернi такi данi?
+ Таласемiя
- Залiзодефiцитна анемiя
- B 12 -дефiцитна анемiя
- Фолiєводефiцитна анемiя
- Гiпопластична анемiя


У хворої 19-ти рокiв з дитинства вiдмiчалося зниження гемоглобiну до 90-95 г/л. У кровi пiд час госпiталiзацiї: ер.- 3, 2 • 1012/л, Hb- 85 г/л, КП- 0,78; лейк.- 5, 6 • 109/л, тромб.- 210 • 109/л. В мазку: анiзоцитоз, пойкiлоцитоз, мiшенеподiбнi еритроцити. Ретикулоцити 6%. Лiкування препаратами залiза було неефективне. Яку патологiю системи кровi можна запiдозрити в даному випадку?
+ Таласемiя
- Ферментопатiя
- Мембранопатiя
- Серпоподiбноклiтинна анемiя
- Фавiзм


До клiнiки поступив чоловiк 40-ка рокiв, якого укусила гадюка. Де переважно буде проходити гемолiз еритроцитiв у цьому випадку?
+ У кровоносному руслi
- У клiтинах печiнки
- У клiтинах селезiнки
- У кiстковому мозку
- У паренхiмi нирок


У недоношеного немовляти спостерiгається жовтяниця. З нестачею у нього якого ферменту це пов’язано?
+ УДФ-трансглюкуронiдаза
- Лужна фосфатаза
- Кисла фосфатаза
- Каталаза
- НАД
+ - дегiдрогеназа


У кровi чоловiка 26-ти рокiв виявлено 18% еритроцитiв сферичної, сплощеної, кулястої та остистої форм. Iншi еритроцити були у формi двоввiгнутих дискiв. Як називається таке явище?
+ Фiзiологiчний пойкiлоцитоз
- Патологiчний пойкiлоцитоз
- Фiзiологiчний анiзоцитоз
- Патологiчний анiзоцитоз
- Еритроцитоз


Через рiк пiсля субтотальної резекцiї шлунка з приводу виразки малої кривизни виявленi змiни в аналiзi кровi - анемiя, лейко- i тромбоцитопенiя, КП-1,3, наявнiсть мегалобластiв та мегалоцитiв. Дефiцит якого фактору обумовив розвиток цiєї анемiї?
+ Фактор Касла
- Хлороводнева кислота
- Муцин
- Пепсин
- Гастрин


У чоловiка 40-ка рокiв внаслiдок посиленого гемолiзу еритроцитiв пiдвищився вмiст залiза в плазмi кровi. Який бiлок забезпечує його депонування в тканинах?
+ Феритин
- Гаптоглобiн
- Трансферин
- Транскортин
- Альбумiн


У синтезi пуринових нуклеотидiв беруть участь деякi амiнокислоти, похiднi вiтамiнiв, фосфорнi ефiри рибози. Коферментна форма якого вiтамiну є переносником одновуглецевих фрагментiв в цьому синтезi?
+ Фолiєва кислота
- Пантотенова кислота
- Нiкотинова кислота
- Рибофлавiн
- Пiридоксин


Жiнка 55-ти рокiв звернулася зi скаргами на тривалi циклiчнi матковi кровотечi протягом року, слабкiсть, запаморочення. Об’єктивно: блiдiсть шкiри. У кровi: Hb- 70 г/л, ер.- 3, 2 1012/л, КП- 0,6, лейк.- 6, 0 • 109/л, ретикулоцити - 1%; гiпохромiя еритроцитiв. Яка анемiя у хворої?
+ Хронiчна постгеморагiчна
- Гемолiтична
- Апластична
- B12 -фолiєводефiцитна
- Залiзодефiцитна


При обстеженнi в аналiзi кровi пацiєнта виявлено лейкоцитоз, лiмфоцитоз, клiтини Боткiна-Гумпрехта на тлi анемiї. Про яку хворобу слiд думати лiкарю?
+ Хронiчний лiмфолейкоз
- Гострий мiєлолейкоз
- Лiмфогранулематоз
- Iнфекцiйний мононуклеоз
- Мiєломна хвороба


При розтинi тiла померлого виявлена гiперплазiя кiсткового мозку плоских i трубчастих кiсток (пiоїдний кiстковий мозок), спленомегалiя (6 кг), гепатомегалiя (5 кг), збiльшення всiх груп лiмфатичних вузлiв. Якому захворюванню вiдповiдають виявленi змiни?
+ Хронiчний мiєлолейкоз
- Хронiчний лiмфолейкоз
- Мiєломна хвороба
- Справжня полiцитемiя
- Лiмфогрануломатоз


У хворого в кровi: ер.- 3, 0 • 10 12 /л; Hb90г/л; ретикул.- 0,5%. В мазку: пойкiлоцити, гiпохромнi еритроцити. Залiзо сироватки кровi - 80 мкмоль/л. Для якої патологiї це характерно?
+ Залiзорефрактерна анемiя
- Хвороба Мiнковського-Шоффара
- Залiзодефiцитна анемiя
- B 12 -дефiцитна анемiя
- Серпоподiбноклiтинна анемiя


Чоловiк 23-х рокiв пiсля ДТП надiйшов до лiкарнi у важкому станi iз черепно-мозковою травмою. Дихання характеризується судомним тривалим вдихом, який переривається коротким видихом. Для якого типу дихання це характерно?
+ Апнейстичне
- Гаспiнг-дихання
- Кусмауля
- Чейн-Стокса
- Бiота


До приймального вiддiлення лiкарнi надiйшов непритомний юнак з ознаками отруєння морфiном. Вiдзначається поверхневе та рiдке дихання, яке обумовлене пригнiченням дихального центру. Який тип недостатностi дихання виник при цьому?
+ Вентиляцiйна дисрегуляторна
- Вентиляцiйна обструктивна
- Вентиляцiйна рестриктивна
- Перфузiйна
- Дифузiйна


У групи альпiнiстiв на висотi 3000 метрiв було зроблено аналiз кровi. Виявлене зниження HCO 3 до 15 ммоль/л (норма 22-26 ммоль/л). Який механiзм зниження HCO 3 кровi?
+ Гiпервентиляцiя
- Посилення ацидогенезу
- Гiповентиляцiя
- Зниження амонiогенезу
- Зниження реабсорбцiї бiкарбонатiв у нирках


У хворого внаслiдок хронiчного обструктивного бронхiту на тлi задишки, тахiкардiї та цiанозу пiд час дослiдження газового складу кровi виявлено розвиток гiпоксемiї та гiперкапнiї. Яке порушення зовнiшнього дихання спостерiгається у хворого?
+ Гiповентиляцiя
- Гiпоперфузiя
- Гiперперфузiя
- Гiпердифузiя
- Гiпервентиляцiя


У людини в результатi патологiчного процесу збiльшена товщина гематоальвеолярного бар’єру. Безпосереднiм наслiдком цього буде зменшення:
+ Дифузiйної здатностi легень
- Резервного об’єму видиху
- Альвеолярної вентиляцiї легень
- Кисневої ємностi кровi
- Хвилинного об’єму дихання


Лiкар записав в iсторiї хвороби, що у хворого дихання поверхневе (знижена глибина дихання). Це означає, що зменшеним є такий показник зовнiшнього дихання:
+ Дихальний об’єм
- Життєва ємнiсть легень
- Функцiональна залишкова ємнiсть
- Ємнiсть вдиху
- Хвилинний об’єм дихання


У людини збiльшена вентиляцiя легень внаслiдок фiзичного навантаження. Який з наведених показникiв зовнiшнього дихання у неї значно бiльший, нiж у станi спокою?
+ Дихальний об’єм
- Життєва ємнiсть легень
- Резервний об’єм вдиху
- Резервний об’єм видиху
- Загальна ємнiсть легень


Пiд час обiду дитина поперхнулася i аспiрувала їжу. Почався сильний кашель, шкiра i слизовi цiанотичнi, пульс частий, дихання рiдке, видих подовжений. Яке порушення зовнiшнього дихання розвинулося у дитини?
+ Експiраторна задишка при асфiксiї
- Iнспiраторна задишка при асфiксiї
- Стенотичне дихання
- Альтернуюче дихання
- Дихання Бiота


При дослiдженнi зовнiшнього дихання лiкар попросив пацiєнта здiйснити максимально глибокий видих пiсля максимально глибокого вдиху для визначення такого показника:
+ Життєва ємнiсть легень
- Загальна ємнiсть легень
- Функцiональна залишкова ємнiсть
- Резервний об’єм видиху
- Киснева ємнiсть кровi


Людина зробила максимально глибокий видих. Як називається об’єм повiтря, що знаходиться в її легенях пiсля цього?
+ Залишковий об’єм
- Функцiональна залишкова ємнiсть легень
- Ємнiсть вдиху
- Резервний об’єм видиху
- Альвеолярний об’єм


Чоловiк 50-ти рокiв хворiє на хронiчний бронхiт, скаржиться на задишку пiд час фiзичного навантаження, постiйний кашель з вiдходженням харкотиння. При обстеженнi дiагностовано ускладнення - емфiзема легень. Чим вона зумовлена?
+ Зниження еластичних властивостей легень
- Зменшення альвеолярної вентиляцiї
- Зменшення розтяжностi легень
- Зменшення перфузiї легень
- Порушення вентиляцiйноперфузiйного спiввiдношення в легенях


Чоловiк 38-ми рокiв загинув при спробi пiдйому вантажу. Розвинувся колаптоїдний стан. На аутопсiї виявлений розрив обширної аневризми грудного вiддiлу аорти. За життя страждав на вiсцеральний сифiлiс. Який патологiчний процес в даному випадку обумовив зменшення мiцностi стiнки аорти, її розширення i розрив?
+ Зникнення еластичних волокон
- Зникнення колагенових волокон
- Атрофiя м’язового шару
- Змiни iнтими за типом "шагреневої шкiри"
- Новоутворення судин


У результатi поранення у чоловiка 35-ти рокiв настав повний розрив спинного мозку на рiвнi першого шийного сегменту. Як змiниться характер дихання?
+ Зупиниться
- Не змiниться
- Збережеться дiафрагмальне, щезне грудне
- Збережеться грудне, щезне дiафрагмальне
- Стане рiдким та глибоким


Анатомiчний мертвий простiр – це частина повiтря, яка залишається в повiтроносних шляхах пiсля видиху. В якiй iз наведених нижче ситуацiй вiдбудеться зменшення анатомiчного мертвого простору?
+ Накладання трахеостоми
- Нахил голови вперед
- Поворот лежачого пацiєнта на лiвий бiк
- Поворот лежачого пацiєнта на правий бiк
- Дихання через рот


У шахтаря виявлено фiброз легень, що супроводжувався порушенням альвеолярної вентиляцiї. Який механiзм виникнення цього порушення є провiдним?
+ Обмеження дихальної поверхнi легень
- Звуження верхнiх дихальних шляхiв
- Порушення нервової регуляцiї дихання
- Обмеження рухомостi грудної клiтки
- Спазм бронхiв


У пiдлiтка 12-ти рокiв, який хворiє на бронхiальну астму, виник тяжкий напад астми: виражена експiраторна задишка, блiдiсть шкiрних покривiв. Який вид порушення альвеолярної вентиляцiї має мiсце?
+ Обструктивний
- Рестриктивний
- Торако-дiафрагмальний
- Центральний
- Нервово-м’язовий


У людини пiсля довiльної тривалої затримки дихання збiльшилися частота й глибина дихання. Якi змiни у складi кровi стали причиною цього?
+ Пiдвищення pCO2
- Зниження pCO2
- Зниження pO2
- Пiдвищення pO2
- Пiдвищення pH


Щуру в плевральну порожнину введено 0,5 мл повiтря. Який тип недостатностi дихання виникає в даному випадку?
+ Рестриктивне порушення альвеолярної вентиляцiї
- Обструктивне порушення альвеолярної вентиляцiї
- Перфузiйний
- Дифузiйний
- Дисрегуляторне порушення альвеолярної вентиляцiї


У хворого iз вираженим пневмосклерозом пiсля перенесеного iнфiльтративного туберкульозу легень розвинулась дихальна недостатнiсть. До якого патогенетичного типу вона вiдноситься?
+ Рестриктивний
- Обструктивний
- Дисрегуляцiйний
- Рефлекторний
- Апнеїстичний


В експериментi на тваринi здiйснили перерiзку блукаючих нервiв з обох бокiв. Як при цьому змiниться характер дихання?
+ Стане глибоким i рiдким
- Стане поверхневим та частим
- Стане глибоким i частим
- Стане поверхневим та рiдким
- Дихання не змiниться


Внаслiдок дiї на органiзм електричного струму мiської електромережi впродовж 0,1 сек у напрямку "права рука-голова"у постраждалого спостерiгалась зупинка дихання. Вкажiть найбiльш iмовiрний механiзм цього ускладнення:
+ Тотальний паралiч дихального центру
- Рефлекторна зупинка дихання (больовий шок)
- Паралiч дихальних м’язiв
- Емоцiйний стрес
- Паралiч центрiв вдиху


Людина зробила спокiйних видих. Як називається об’єм повiтря, який мiститься у неї в легенях при цьому?
+ Функцiональна залишкова ємнiсть легень
- Залишковий об’єм
- Резервний об’єм видиху
- Дихальний об’єм
- Життєва ємнiсть легень


У неврологiчне вiддiлення з приводу мозкового крововиливу поступив хворий 62-х рокiв. Об’єктивно: стан важкий. Спостерiгається наростання глибини i частоти дихання, а потiм його зменшення до апное, пiсля чого цикл дихальних рухiв вiдновлюється. Який тип дихання у хворого?
+ Чейна-Стокса
- Кусмауля
- Бiота
- Гаспiнг-дихання
- Апнейстичне


При обстеженнi людини встановлено, що хвилинний об’єм серця дорiвнює 3500 мл, систолiчний об’єм - 50 мл. Якою є частота серцевих скорочень за хвилину у людини?
+ 70
- 60
- 50
- 80
- 90


У людини визначили частоту серцевих скорочень за пульсом. Вона дорiвнює 120/хв. Якою при цьому є тривалiсть серцевого циклу?
+ 0,5 с
- 0,7 с
- 0,8 с
- 0,9 с
- 1,0 с


У жiнки 30-ти рокiв хвилинний об’єм кровi у станi спокою становить 5 л/хв. Який об’єм кровi проходить у неї через судини легень за 1 хвилину?
+ 5 л
- 3,75 л
- 2,5 л
- 2,0 л
- 1,5 л


Визначте пульсовий i середньодинамiчний артерiальний тиск (мм рт.ст.) у обстежуваного, якщо вимiряний у нього артерiальний тиск становить 130/70 мм рт.ст.:
+ 60, 90
- 60, 80
- 50, 90
- 60, 100
- 50, 70


Подразнення правого блукаючого нерва спричинило рiзке сповiльнення атрiовентрикулярного проведення. На ЕКГ при цьому буде подовжений:
+ Iнтервал P − Q
- Комплекс QRST
- Зубець T
- Зубець P
- Iнтервал R − R


Пiд час реєстрацiї ЕКГ хворого з гiперфункцiєю щитоподiбної залози зареєстровано збiльшення частоти серцевих скорочень. Вкорочення якого елементу ЕКГ про це свiдчить?
+ Iнтервал R − R
- Сегмент P − Q
- Iнтервал P − Q
- Iнтервал P − T
- Комплекс QRS


У хворого через 12 годин пiсля гострого нападу загруднинного болю знайдено рiзке пiдвищення активностi АсАТ у сироватцi кровi. Вкажiть патологiю, для якої характерне це змiщення:
+ Iнфаркт мiокарда
- Вiрусний гепатит
- Колагеноз
- Цукровий дiабет
- Нецукровий дiабет


Пiд час хiрургiчного втручання на тонкiй кишцi у людини можлива рефлекторна зупинка серця. Якi рецептори в мiокардi необхiдно заблокувати, щоб попередити зупинку?
+ M-холiнорецептори
- H-холiнорецептори
- Пуриновi рецептори
- α-адренорецептори
- β-адренорецептори


У результатi побутової травми у пацiєнта виникла значна крововтрата, що супроводжувалося зниженням артерiального тиску. Дiя яких гормонiв забезпечує швидке вiдновлення кров’яного тиску, викликаного крововтратою?
+ Адреналiн, вазопресин
- Кортизол
- Статевi
- Окситоцин
- Альдостерон


У хiрурга пiсля проведення тривалої операцiї пiдвищився артерiальний тиск до 140/110 мм рт.ст. Якi змiни гуморальної регуляцiї можуть бути причиною пiдвищення артерiального тиску в даному випадку?
+ Активацiя симпатоадреналової системи
- Активацiя утворення i видiлення альдостерону
- Активацiя ренiн-ангiотензинової системи
- Активацiя калiкреїн-кiнiнової системи
- Гальмування симпатоадреналової системи


У тварини через 2 тижнi пiсля експериментального звуження ниркової артерiї пiдвищився артерiальний тиск. Зi збiльшенням дiї на судини якого фактора гуморальної регуляцiї це пов’язано?
+ Ангiотензин II
- Кортизол
- Альдостерон
- Вазопресин
- Дофамiн


Жiнка 49-ти рокiв звернулася до лiкаря зi скаргами на пiдвищену втомлюванiсть та появу задишки пiд час фiзичного навантаження. На ЕКГ: ЧСС50/хв, iнтервал P Q- подовжений, комплекс QRS- не змiнений, кiлькiсть зубцiв P перевищує кiлькiсть комплексiв QRS. Який вид аритмiї у пацiєнтки?
+ Атрiовентрикулярна блокада
- Екстрасистолiя
- Синусова брадикардiя
- Миготлива аритмiя
- Синоатрiальна блокада


У хворого 45-ти рокiв при аналiзi ЕКГ встановлено: ритм синусовий, число передсердних комплексiв бiльше числа шлуночкових комплексiв; прогресуюче подовження iнтервалу P – Q вiд комплексу до комплексу; випадiння окремих шлуночкових комплексiв; зубцi P та комплекси QRST без змiн. Назвiть тип порушення серцевого ритму:
+ Атрiовентрикулярна блокада II ступеня
- Синоаурiкулярна блокада
- Атрiовентрикулярна блокада I ступеня
- Внутрiшньопередсердна блокада
- Повна атрiовентрикулярна блокада


У хворої 45-ти рокiв при електрокардiографiчному обстеженнi виявлено такi змiни: iнтервал P − Q подовжений, при цьому випадає кожен другий або третiй комплекс QRST. Яке саме порушення провiдностi серця спостерiгається?
+ Атрiовентрикулярна блокада III ступеня
- Атрiовентрикулярна блокада повна
- Синоаурiкулярна блокада
- Внутрiшлуночкова блокада
- Атрiовентрикулярна блокада I ступеня


В експериментi на ссавцi зруйнували певну структуру серця, що призвело до припинення проведення збудження вiд передсердь до шлуночкiв. Що саме зруйнували?
+ Атрiовентрикулярний вузол
- Синоатрiальний вузол
- Пучок Гiса
- Нiжки пучка Гiса
- Волокна Пуркiн’є


На iзольованому серцi вивчалась швидкiсть проведення збудження у рiзних його дiлянках. Де була виявлена найменша швидкiсть?
+ Атрiовентрикулярний вузол
- Пучок Гiса
- Волокна Пуркiн’є
- Мiокард передсердь
- Мiокард шлуночкiв


В експериментi подразнюють гiлочки блукаючого нерва, якi iннервують серце. Це призвело до того, що припинилося проведення збудження вiд передсердь до шлуночкiв. Електрофiзiологiчнi змiни в яких структурах серця є причиною цього?
+ Атрiовентрикулярний вузол
- Пучок Гiса
- Синоатрiальний вузол
- Шлуночки
- Передсердя


У людини частота серцевих скорочень постiйно утримується на рiвнi 40 разiв за хвилину. Що є водiєм ритму серця у неї?
+ Атрiовентрикулярний вузол
- Синоатрiальний вузол
- Пучок Гiса
- Нiжки пучка Гiса
- Волокна Пуркiн’є


У пацiєнта тривалiсть iнтервалу P − Q на ЕКГ перевищує норму при нормальнiй тривалостi зубця P . Причиною цього є зменшення швидкостi проведення збудження:
+ Атрiо-вентрикулярним вузлом
- Сино-атрiальним вузлом
- Пучком Гiса
- Нiжками пучка Гiса
- Волокнами Пуркiн’є


Пiд час аналiзу електрокардiограми встановлено збiльшення тривалостi i амплiтуди зубця S. Деполяризацiя якої дiлянки серця порушена у хворого?
+ Базальнi вiддiли шлуночкiв
- Передсердя
- Верхiвка серця
- Боковi стiнки шлуночкiв
- Середня i нижня третина мiжшлуночкової перегородки


Пiд час роботи лiкарю–стоматологу доводиться довго стояти на ногах, що може призвести до застою кровi у венах нижнiх кiнцiвок та їх варикозного розширення. З порушенням якого механiзму венозного припливу кровi до серця це пов’язано?
+ Вiдсутнiсть скорочення скелетних м’язiв
- Градiєнт тиску
- Присмоктувальний ефект грудної клiтки
- Залишкова рушiйна сила серця
- Присмоктувально-тисковий помповий ефект дiафрагми на органи черевної порожнини


У хворого температура протягом 5-ти днiв коливалася в межах 39,5℃ 40,2℃. На 6-й день на тлi рiзкого зниження температури до 35,2℃ розвинувся колапс. Який головний механiзм колапсу?
+ Вазодилатацiя
- Гiпервентиляцiя
- Посилене потовидiлення
- Тахiкардiя
- Полiурiя


В експериментi у тварини в результатi проведеного перетинання депресорного нерва та руйнування каротидних клубочкiв розвинулась стiйка гiпертензiя. З порушенням якої функцiї нервової системи пов’язане це явище?
+ Вегетативна
- Вища нервова дiяльнiсть
- Рухова
- Сенсорна
- Трофiчна


В експериментi подразнюють гiлочки симпатичного нерва, якi iннервують серце. Це призвело до збiльшення сили серцевих скорочень, тому що через мембрану типових кардiомiоцитiв збiльшився:
+ Вхiд iонiв кальцiю
- Вихiд iонiв кальц iю
- Вихiд iонiв калiю
- Вхiд iонiв калiю
- Вхiд iонiв кальцiю та калiю


У хворого, що страждає на важку форму порушення водно-сольового обмiну, настала зупинка серця в дiастолi. Який найбiльш вiрогiдний механiзм зупинки серця в дiастолi?
+ Гiперкалiємiя
- Гiпернатрiємiя
- Дегiдратацiя органiзму
- Гiпокалiємiя
- Гiпонатрiємiя


Обстеження пацiєнта з високим артерiальним тиском показало в нього вторинну артерiальну гiпертензiю. Причиною такого стану є ренiнпродукуюча пухлина нирки. Що є головною ланкою в патогенезi вторинної артерiальної гiпертензiї в хворого?
+ Гiперпродукцiя ангiотензину 2, альдостерону
- Гiперпродукцiя кортизолу
- Гiперпродукцiя iнсулiну
- Недостатня продукцiя вазопресину
- Недостатня продукцiя катехоламiнiв


При проходженнi профiлактичного огляду у шахтаря лiкар встановив змiни функцiонального стану серця, що свiдчать про серцеву недостатнiсть в стадiї компенсацiї. Що з нижче перерахованого є головним пiдтвердженням компенсацiї дiяльностi серця?
+ Гiпертрофiя мiокарда
- Тахiкардiя
- Збiльшення артерiального тиску
- Задишка
- Цiаноз


У чоловiка 65-ти рокiв впродовж 15-ти рокiв була виражена артерiальна гiпертензiя. Останнiм часом систолiчний тиск почав знижуватися, а дiастолiчний залишився пiдвищеним. Який гемодинамiчний тип артерiальної гiпертензiї у хворого?
+ Гiпокiнетичний
- Нормокiнетичний
- Гiперкiнетичний
- Еукiнетичний
-


Хвора на ревматоїдний артрит пiсля трьохтижневого лiкування преднiзолоном почала скаржитись на перебої в роботi серця. З чим пов’язаний розвиток даного небажаного ефекту препарату?
+ Гiпокалiємiя
- Гiперкалiємiя
- Гiперурiкемiя
- Гiперглiкемiя
- Гiпоглiкемiя


Пiд час аналiзу ЕКГ людини з’ясовано, що у другому стандартному вiдведеннi вiд кiнцiвок зубцi P позитивнi, їхня амплiтуда 0,1 mV (норма - 0,05-0,25 mV), тривалiсть - 0,1 с (норма - 0,07-0,10 с). Вiрним є висновок, що у передсердях нормально вiдбувається процес:
+ Деполяризацiї
- Реполяризацiї
- Збудження
- Скорочення
- Розслаблення


У хворого внаслiдок травми розвинувся травматичний шок, у перебiгу якого мали мiсце наступнi порушення: АТ- 140/90 мм рт.ст., Ps- 120/хв. Хворий метушливий, багатослiвний, блiдий. Якiй стадiї шоку вiдповiдає цей стан?
+ Еректильна
- Латентний перiод
- Термiнальна
- Торпiдна
- Кiнцева


У хворого на ЕКГ виявлено збiльшення тривалостi комплексу QRS. Наслiдком чого це може бути?
+ Збiльшення часу охоплення збудженням шлуночкiв
- Порушення провiдностi у атрiовентрикулярному вузлi
- Збiльшення збудливостi передсердь
- Збiльшення збудливостi шлуночкiв та передсердь
- Збiльшення часу охоплення збудженням передсердь


При гострому iнфарктi в мiокардi виникає декiлька зон в осередку iнфаркту: зона некрозу, зона iшемiчного пошкодження i зона iшемiї. Зонi пошкодження на ЕКГ вiдповiдає:
+ Змiщення сегменту RS - T вище iзолiнiї
- Негативний зубець T
- Глибокий зубець Q
- Комплекс QRS типу QS
- Зниження зубця R


У собаки в дослiдi подразнювали на шиї периферичний вiдрiзок блукаючого нерва. При цьому спостерiгали такi змiни серцевої дiяльностi:
+ Зменшення частоти скорочень
- Збiльшення сили скорочень
- Збiльшення швидкостi атрiовентрикулярного проведення
- Збiльшення частоти та сили скорочень
- Збiльшення збудливостi мiокарда


У хворого на гострий мiокардит з’явилися клiнiчнi ознаки кардiогенного шоку. Який iз вказаних нижче патогенетичних механiзмiв є провiдним в розвитку шоку?
+ Зниження насосної функцiї серця
- Депонування кровi в органах
- Зниження дiастолiчного притоку до серця
- Зниження судинного тонусу
- Збiльшення периферичного опору судин


У тварини з недостатнiстю аортальних клапанiв розвинулась гiпертрофiя лiвого шлуночка серця. В окремих його дiлянках визначаються локальнi контрактури. Накопичення якої речовини в мiокардiоцитах обумовило контрактури?
+ Кальцiй
- Калiй
- Молочна кислота
- Вуглекислий газ
- Натрiй


У хворого з пересадженим серцем при фiзичному навантаженнi збiльшився хвилинний об’єм кровi. Який механiзм регуляцiї забезпечує цi змiни?
+ Катехоламiни
- Симпатичнi безумовнi рефлекси
- Парасимпатичнi безумовнi рефлекси
- Симпатичнi умовнi рефлекси
- Парасимпатичнi умовнi рефлекси


Жiнцi 54-х рокiв поставили попереднiй дiагноз: iнфаркт мiокарда. Характерною ознакою даного захворювання є суттєве пiдвищення в кровi активностi такого ферменту:
+ Креатинфосфокiназа
- Каталаза
- Г-6-ФДГ
- Альфа-амiлаза
- Аргiназа


Чоловiк 58-ми рокiв хворiє на атеросклероз судин головного мозку. При обстеженнi виявлена гiперлiпiдемiя. Вмiст якого класу лiпопротеїдiв у сироватцi кровi даного чоловiка найбiльш вiрогiдно буде пiдвищений?
+ Лiпопротеїди низької щiльностi
- Лiпопротеїди високої щiльностi
- Комплекси жирних кислот з альбумiнами
- Хiломiкрони
- Холестерин


У хлопчика 11-ти рокiв вмiст холестерину в сироватцi кровi до 25 ммоль/л. В анамнезi - спадкова сiмейна гiперхолестеринемiя, причиною якої є порушення синтезу бiлкiв-рецепторiв до:
+ Лiпопротеїнiв низької щiльностi
- Лiпопротеїнiв високої щiльностi
- Хiломiкронiв
- Лiпопротеїнiв дуже низької щiльностi
- Лiпопротеїнiв промiжної щiльностi


У вiддiлення реанiмацiї надiйшов чоловiк 47-ми рокiв з дiагнозом iнфаркт мiокарда. Яка з фракцiй лактатдегiдрогенази (ЛДГ) буде переважати в сироватцi кровi впродовж перших двох дiб захворювання?
+ ЛДГ1
- ЛДГ2
- ЛДГ3
- ЛДГ4
- ЛДГ5


У чоловiка 72-х рокiв довготривала хронiчна патологiя легень призвела до недостатностi клапанiв легеневої артерiї i трикуспiдального клапану, недостатностi кровообiгу за правошлуночковим типом. Який тип артерiальної гiпертензiї є причиною перевантаження серця об’ємом?
+ Легенева гiпертензiя
- Центрально-iшемiчна гiпертензiя
- Есенцiальна гiпертензiя
- Рефлексогенна гiпертензiя
- Сольова гiпертензiя


Пiд час об’єктивного обстеження хворого з дiагнозом: атеросклеротичний мiокардiосклероз, лiкар встановив феномен дефiциту пульсу. При якiй формi порушення серцевого ритму спостерiгається такий феномен?
+ Миготлива аритмiя
- Iдiовентрикулярний ритм
- Передсердно-шлуночковий ритм
- Брадикардiя
- Синусова екстрасистолiя


Пiд час тривалого бiгу у чоловiка 35-ти рокiв виникла гостра серцева недостатнiсть. Якi змiни iонного складу спостерiгаються у серцевому м’язi при цьому станi?
+ Накопичення в клiтинах мiокарда iонiв Na
+ i Ca2+
- Накопичення в клiтинах мiокарда iонiв K
+ i Mg2+
- Зменшення в клiтинах мiокарда iонiв Na
+ i Ca2+
- Зменшення в позаклiтинному просторi iонiв K
+ i Mg2+
- Збiльшення в позаклiтинному просторi iонiв Na
+ i Ca2+


Пiд час диспансерного огляду у чоловiка 36-ти рокiв, водiя за професiєю, артерiальний тиск склав 150/90 мм рт.ст. Скарги на шум у вухах наприкiнцi робочого дня та загальне нездужання, якi зникають пiсля вiдпочинку. Дiагностована гiпертонiчна хвороба. Який провiдний патогенетичний механiзм у цьому випадку?
+ Нейрогенний
- Нирковий
- Гуморальний
- Ендокринний
- Рефлексогенний


У хворого 59-ти рокiв, директора пiдприємства, пiсля перевiрки податкової iнспекцiї з’явився iнтенсивний пекучий бiль, локалiзований за грудниною, який iррадiює в лiву руку. Через 15 хвилин стан хворого нормалiзувався. Який можливий механiзм стенокардiї є провiдним у цього хворого?
+ Пiдвищення у кровi рiвня катехоламiнiв
- Атеросклероз коронарних судин
- Внутрiшньосудинна агрегацiя формених елементiв
- Тромбоз коронарних судин
- Функцiональне перевантаження серця


У водiя, який потрапив у ДТП, отримав травму та знаходиться у станi шоку, спостерiгається зменшення добової кiлькостi сечi до 300 мл. Який основний патогенетичний фактор цiєї змiни дiурезу?
+ Падiння артерiального тиску
- Зниження онкотичного тиску кровi
- Пiдвищення проникностi судин
- Зменшення кiлькостi функцiонуючих клубочкiв
- Вторинний гiперальдостеронiзм


Хворого на трансмуральний iнфаркт мiокарда лiвого шлуночка переведено до вiддiлення реанiмацiї у важкому станi. АТ70/50 мм рт.ст., ЧСС- 56/хв., ЧД- 32/хв. Зазначте головну ланку в патогенезi кардiогенного шоку:
+ Падiння серцевого викиду
- Падiння периферичного судинного опору
- Втрата води
- Крововтрата
- Втрата електролiтiв


Пiд час бiйки у чоловiка виникла зупинка серця внаслiдок сильного удару у верхню дiлянку передньої черевної стiнки. Який iз зазначених механiзмiв спричинив зупинку серця?
+ Парасимпатичнi безумовнi рефлекси
- Симпатичнi безумовнi рефлекси
- Парасимпатичнi умовнi рефлекси
- Симпатичнi умовнi рефлекси
- Периферичнi рефлекси


Жiнка 25-ти рокiв скаржиться на постiйний бiль у дiлянцi серця, задишку пiд час рухiв, загальну слабкiсть. Об’єктивно: шкiра блiда та холодна, акроцiаноз. Ps- 96/хв., АТ- 105/70 мм рт.ст. Межа серця змiщена на 2 см влiво. Перший тон над верхiвкою серця послаблений, систолiчний шум над верхiвкою. Дiагностовано недостатнiсть мiтрального клапана серця. Чим обумовлене порушення кровообiгу?
+ Перевантаження мiокарда збiльшеним об’ємом кровi
- Перевантаження мiокарда пiдвищеним опором вiдтоку кровi
- Пошкодження мiокарда
- Зниження об’єму циркулюючої кровi
- Збiльшення об’єму судинного русла


У спортсмена легкоатлета (бiгуна на довгi дистанцiї) пiд час змагань розвинулась гостра серцева недостатнiсть. В результатi чого виникла ця патологiя?
+ Перевантаження серця об’ємом
- Порушення вiнцевого кровообiгу
- Прямого пошкодження мiокарда
- Патологiя перикарда
- Перевантаження серця опором


У хворого 44-х рокiв на ЕКГ виявленi ознаки гiпертрофiї обох шлуночкiв та правого передсердя. Дiагностовано недостатнiсть тристулкового клапана. Який патогенетичний варiант порушення функцiї серця має мiсце при цiй недостатностi?
+ Перевантаження серця об’ємом
- Перевантаження серця опором
- Первинна мiокардiальна недостатнiсть
- Коронарна недостатнiсть
- Тампонада серця


У хворого на ессенцiальну артерiальну гiпертензiю розвинувся гiпертонiчний криз, що призвело до нападу серцевої астми. Який механiзм серцевої недостатностi є провiдним в даному випадку?
+ Перевантаження серця пiдвищеним опором
- Перевантаження серця збiльшеним об’ємом кровi
- Абсолютна коронарна недостатнiсть
- Пошкодження мiокарда
- Порушення надходження кровi до серця


Хворий 21-го року надiйшов до стацiонару з загостренням хронiчного тонзилiту. Скаржиться на слабкiсть, задуху при помiрному фiзичному навантаженнi. Температура 37,5℃. ЧСС- 110/хв. ЕКГ: ритм синусовий, iнтервал P Q подовжений. Яка аритмiя у хворого?
+ Передсердно-шлуночкова блокада I ст.
- Передсердно-шлуночкова блокада II ст.
- Внутрiшньопередсердна блокада
- Порушення внутрiшньошлуночкової провiдностi
- Передсердно-шлуночкова екстрасистолiя


Аналiз ЕКГ хворого виявив вiдсутнiсть зубця Р. Тривалiсть та амплiтуда QRS комплексу та зубця Т вiдповiдають нормi. Що є водiєм ритму серця даного пацiєнта?
+ Передсердно-шлуночковий вузол
- Синусовий вузол
- Пучок Гiса
- Волокна Пуркiньє
- Мiокард шлуночкiв


У здорової дорослої людини швидкiсть проведення збудження через атрiовентрикулярний вузол дорiвнює 0,02-0,05 м/с. Атрiовентрикулярна затримка забезпечує:
+ Послiдовнiсть скорочення передсердь та шлуночкiв
- Одночаснiсть скорочення обох передсердь
- Одночаснiсть скорочення обох шлуночкiв
- Достатню силу скорочення передсердь
- Достатню силу скорочення шлуночкiв


У хворого на iшемiчну хворобу серця вiдзначається гiпертрофiя мiокарда, тахiкардiя, зниження ХОК. Який з механiзмiв є провiдним в ушкодженнi кардiомiоцитiв у даному випадку?
+ Пошкодження специфiчних мембранних насосiв
- Збiльшення числа α та βадренорецепторiв
- Втрата Mg 2
+ кардiомiоцитами
- Втрата Ca 2
+ кардiомiоцитами
- Дегiдратацiя кардiомiоцитiв


У жiнки обмежений кровотiк у нирках, пiдвищений артерiальний тиск. Гiперсекрецiя якого гормону зумовила пiдвищений тиск?
+ Ренiн
- Адреналiн
- Норадреналiн
- Еритропоетин
- Вазопресин


До серцево-судинного вiддiлення надiйшов хворий зi скаргами на постiйний головний бiль у потиличнiй дiлянцi, шум у вухах, запаморочення. При обстеженнi: АТ180/110 мм рт.ст., ЧСС- 95/хв. Рентгенологiчно визначено звуження однiєї з ниркових артерiй. Активацiя якої з перерахованих систем викликала гiпертензивний стан хворого?
+ Ренiн-ангiотензинова
- Гемостатична
- Симпатоадреналова
- Кiнiнова
- Iмунна


У пацiєнта, який пiвтора мiсяця тому перенiс iнфаркт мiокарда, дiагностовано синдром Дреслера з характерною трiадою: перикардит, плеврит, пневмонiя. Який головний механiзм цього ускладнення?
+ Сенсибiлiзацiя органiзму антигенами мiокарда
- Зниження резистентностi до iнфекцiйних агентiв
- Активацiя сапрофiтної мiкрофлори
- Iнтоксикацiя органiзму продуктами некрозу
- Викидання у кров мiокардiальних ферментiв


У дорослої людини системний артерiальний тиск знизився з 120/70 до 90/50 мм рт.ст., що викликало рефлекторне звуження судин. У якому з зазначених органiв звуження судин буде найменшим?
+ Серце
- Шкiра
- Кишечник
- Скелетнi м’язи
- Печiнка


Внаслiдок стресу у похилої людини пiдвищився артерiальний тиск. Причиною цього є активацiя:
+ Симпато-адреналової системи
- Парасимпатичного ядра блукаючого нерва
- Функцiї щитоподiбної залози
- Функцiї кори наднирникiв
- Функцiї гiпофiзу


На ЕКГ пацiєнта мають мiсце такi змiни: зубець P - нормальний, iнтервал P − Q - вкорочений, шлуночковий комплекс QRST - розширений, зубець R двогорбий або двофазний. Яка iз форм аритмiї має мiсце у даного пацiєнта?
+ Синдром WPW (Вольфа-ПаркiнсонаУайта)
- Синдром Фредерiка (трiпотiння передсердь)
- Атрiовентрикулярна блокада
- Миготiння шлуночкiв
- Миготлива аритмiя


Хворий 39-ти рокiв з алкогольним цирозом печiнки скаржиться на задишку, загальну слабкiсть. Встановлено зниження артерiального тиску, розширення поверхневих вен передньої стiнки живота, спленомегалiю. Яке порушення гемодинамiки спостерiгається у хворого?
+ Синдром портальної гiпертензiї
- Недостатнiсть лiвого шлуночка серця
- Недостатнiсть правого шлуночка серця
- Колапс
- Тотальна серцева недостатнiсть


При аналiзi ЕКГ виявлено випадiння деяких серцевих циклiв PQRST. Наявнi зубцi та комплекси не змiненi. Назвiть вид аритмiї:
+ Синоатрiальна блокада
- Миготлива аритмiя
- Атрiовентрикулярна блокада
- Передсердна екстрасистола
- Внутрiшньопередсердна блокада


Пiд час емоцiйного збудження частота серцевих скорочень (ЧСС) у людини 30-ти рокiв досягла 112/хв. Змiна стану якої структури провiдної системи серця є причиною збiльшення ЧСС?
+ Синоатрiальний вузол
- Волокна Пуркiн’є
- Нiжки пучка Гiса
- Атрiовентрикулярний вузол
- Пучок Гiса


Хворому чоловiку 75-ти рокiв, у якого частота серцевих скорочень була 40/хвилину, iмплантували серцевий електростимулятор. Пiсля цього частота серцевих скорочень зросла до 70/хв. Функцiю якого вiддiлу серця взяв на себе електростимулятор?
+ Синоатрiальний вузол
- Атрiовентрикулярний вузол
- Нiжки Гiса
- Волокна пучка Гiса
- Волокна Пуркiн’є


На перехiд iз горизонтального положення у вертикальне система кровообiгу вiдповiдає розвитком рефлекторної пресорної реакцiї. Що з наведеного є її обов’язковим компонентом?
+ Системне звуження венозних судин ємностi
- Системне розширення артерiальних судин опору
- Зменшення об’єму циркулюючої кровi
- Зменшення частоти серцевих скорочень
- Зменшення насосної функцiї серця


У студента 18-ти рокiв пiд час фiзичного навантаження реографiчно зареєстровано перерозподiл кровотоку органiв. У яких судинах кровотiк пiдвищився найбiльшою мiрою?
+ Скелетнi м’язи
- Печiнка
- Головний мозок
- Нирки
- Шлунково-кишковий тракт


У хворого пiсля тривалого психоемоцiйного напруження спостерiгається пiдвищення артерiального тиску, що супроводжується серцебиттям, кардiалгiями, головним болем, запамороченням. Домiнуючим у формуваннi артерiальної гiпертензiї у даному випадку є збiльшення:
+ Тонусу артерiол
- Тонусу венул
- Об’єму циркулюючої кровi
- Частоти серцевих скорочень
- Серцевого викиду


При аналiзi ЕКГ необхiдно визначити, що є водiєм ритму серця. Зробити це можна на пiдставi вимiрювання:
+ Тривалостi iнтервалу R − R
- Амплiтуди зубцiв
- Напрямку зубцiв
- Тривалостi зубцiв
- Тривалостi комплексу QRST


У щурiв, що знаходяться у станi стресу, пiдвищенi м’язовий тонус та артерiальний тиск, збiльшений вмiст глюкози у кровi, посилена секрецiя кортикотропiну i кортикостероїдiв. У якiй фазi стресу знаходяться цi тварини?
+ Фаза протишоку
- Виснаження
- Фаза шоку
- Еректильна
- Термiнальна


У хворого виявлено екстрасистолiю. На ЕКГ при екстрасистолiчному скороченнi вiдсутнiй зубець P , комплекс QRS деформований, є повна компенсаторна пауза. Якi це екстрасистоли?
+ Шлуночковi
- Передсерднi
- Передсердно-шлуночковi
- Синуснi
-


Хворий був доставлений до лiкарнi в коматозному станi. В анамнезi цукровий дiабет. Об’єктивно: дихання Кусмауля, зниження артерiального тиску, у видихуваному повiтрi запах ацетону. Пiсля проведеної невiдкладної терапiї стан покращився. Який препарат було введено хворому?
+ Iнсулiн
- Адреналiн
- Iзадрин
- Букаркам
- Глiбенкламiд


У хворої встановлено порушення видiлення тиреотропного гормону гiпофiза. Зi зниженням функцiй якої частки гiпофiза це пов’язано?
+ Lobus anterior
- Infundibulum
- Lobus posterior
- Pars intermedia
-


У пацiєнта з пiдвищеним артерiальним тиском, тремором, тахiкардiєю, була дiагностовано доброякiсна пухлина мозкової речовини наднирникiв. Гiперсекрецiя якого гормону викликає таку симптоматику?
+ Адреналiн
- Глюкагон
- Iнсулiн
- Тироксин
- Соматотропiн


Недбалий студент раптово зустрiвся з деканом. Концентрацiя якого гормону найшвидше збiльшиться в кровi студента?
+ Адреналiн
- Тиреолiберин
- Кортикотропiн
- Кортизол
- Соматотропiн


У дiвчинки дiагностований адреногенiтальний синдром (псевдогермафродитизм). Надмiрна секрецiя яких гормонiв наднирникiв обумовила дану патологiю?
+ Андрогени
- Естрогени
- Мiнералокортикоїди
- Глюкокортикоїди
- Катехоламiни


У хворого вiдзначаються перiодичнi напади серцебиття (пароксизми), сильне потовидiлення, напади головного болю. При обстеженнi виявлена гiпертензiя, гiперглiкемiя, пiдвищення основного обмiну, тахiкардiя. При якiй патологiї наднирникiв спостерiгається подiбна картина?
+ Гiперфункцiя мозкового шару
- Гiпофункцiя мозкового шару
- Гiперфункцiя кори наднирникiв
- Гiпофункцiя кори наднирникiв
- Первинний альдостеронiзм


Хлопчик 5-ти мiсяцiв госпiталiзований з приводу тонiчних судом. Хворiє з народження. Об’єктивно: волосся жорстке, нiгтi витонченi та ламкi, шкiрнi покриви блiдi та сухi. В бiохiмiчному аналiзi кровi: кальцiй - 0,5 ммоль/л (норма - 0,75-2,5 ммоль/л), фосфор - 1,9 ммоль/л (норма - 0,646-1,292 ммоль/л). З чим пов’язанi цi змiни?
+ Гiпопаратиреоз
- Гiперпаратиреоз
- Гiперальдостеронiзм
- Гiпоальдостеронiзм
- Гiпотиреоз


У хворої внаслiдок запалення порушена ендокринна функцiя фолiкулярних клiтин фолiкулiв яєчника. Синтез яких гормонiв буде пригнiчений?
+ Естрогени
- Прогестерон
- Лютропiн
- Фолiкулостимулюючий гормон
- Фолiстатин


При оглядi пацiєнта виявлене надмiрне розростання кiсток i м’яких тканин обличчя, збiльшенi розмiри язика, розширенi мiжзубнi промiжки в збiльшенiй зубнiй дузi. Якi змiни секрецiї гормонiв у нього найбiльш вiрогiднi?
+ Збiльшена секрецiя соматотропного гормону
- Зменшена секрецiя соматотропного гормону
- Збiльшена секрецiя iнсулiну
- Зменшена секрецiя тироксину
- Зменшена секрецiя iнсулiну


Чоловiку 46-ти рокiв, що хворiє на дифузний токсичний зоб, була проведена операцiя резекцiї щитоподiбної залози. Пiсля операцiї вiдмiчаються вiдсутнiсть апетиту, диспепсiя, пiдвищена нервово-м’язова збудливiсть. Маса тiла не збiльшилася. Температура тiла у нормi. Чим, iз нижче перелiченого, обумовлений стан хворого?
+ Зниженням продукцiї паратгормону
- Зниженням продукцiї тироксину
- Пiдвищенням продукцiї кальцитонiну
- Пiдвищенням продукцiї тиреолiберину
- Пiдвищенням продукцiї тироксину


Хворий 42-х рокiв висуває скарги на сильне серцебиття, пiтливiсть, нудоту, порушення зору, тремор рук, пiдвищення артерiального тиску. З анамнезу: 2 роки тому було встановлено дiагноз феохромоцитома. Гiперпродукцiя яких гормонiв зумовлює цю патологiю?
+ Катехоламiни
- Альдостерон
- Глюкокортикоїди
- АКТГ
- Тиреоїднi гормони


У хворого 41-го року вiдзначається гiпонатрiємiя, гiперкалiємiя, дегiдратацiя, зниження артерiального тиску, м’язова слабкiсть, брадикардiя, аритмiя. З порушенням функцiй яких гормонiв це пов’язано?
+ Кортикостероїди
- Тиреоїднi
- Гормони пiдшлункової залози
- Статевi гормони
- Гормони мозкової речовини наднирникiв


Пiсля перенесеного сепсису у хворої 27-ми рокiв з’явився бронзовий колiр шкiри, характерний для аддiсонової хвороби. Механiзм гiперпiгментацiї полягає в пiдвищеннi секрецiї такого гормону:
+ Меланоцитстимулюючий
- Соматотропний
- Гонадотропний
- B-лiпотропний
- Тиреотропний


Пацiєнт 16-ти рокiв, що страждає на хворобу Iценко-Кушiнга, консультований з приводу надмiрної ваги тiла. При опитуваннi з’ясувалося, що енергетична цiннiсть спожитої їжi складає 1700-1900 ккал/добу. Яка провiдна причина ожирiння у даному випадку?
+ Надлишок глюкокортикоїдiв
- Нестача iнсулiну
- Надлишок iнсулiну
- Нестача глюкокортикоїдiв
- Гiподинамiя


Чоловiк середнього вiку виїхав до iншої країни на обiцяну йому роботу, але працевлаштуватися тривалий час йому не вдавалося. Якi з ендокринних залоз були виснаженi у цiєї людини найбiльше?
+ Наднирники
- Прищитоподiбнi
- Сiм’яники
- Пiдгрудинна
- Щитоподiбна


Пацiєнта турбують полiурiя (7 л на добу) i полiдипсiя. При обстеженнi не виявлено нiяких розладiв вуглеводного обмiну. Дисфункцiя якої ендокринної залози може бути причиною даних порушень?
+ Нейрогiпофiз
- Аденогiпофiз
- Острiвцi пiдшлункової залози
- Кора наднирникiв
- Мозкова речовина наднирникiв


Жiнка 44-х рокiв скаржиться на загальну слабкiсть, бiль у дiлянцi серця, значне збiльшення маси тiла. Об’єктивно: обличчя мiсяцеподiбне, гiрсутизм, АТ- 165/100 мм рт.ст., зрiст - 164 см, вага - 103 кг; переважно накопичення жиру на шиї, верхньому плечовому поясi, животi. Що є основним патогенетичним механiзмом ожирiння у жiнки?
+ Пiдвищення продукцiї глюкокортикоїдiв
- Зниження продукцiї тиреоїдних гормонiв
- Пiдвищення продукцiї iнсулiну
- Зниження продукцiї глюкагону
- Пiдвищення продукцiї мiнералокортикоїдiв


Спецiальний режим харчування призвiв до зменшення iонiв Ca2
+ в кровi. До збiльшення секрецiї якого гормону це призведе?
+ Паратгормон
- Тирокальцитонiн
- Вазопресин
- Соматотропiн
- Тироксин


У людини збiльшений вмiст iонiв кальцiю в плазмi кровi, зменшений – у кiстках. Надмiрна секрецiя якого гормону може спричинити такi змiни?
+ Паратгормон
- Тироксин
- Трийодтиронiн
- Тиреокальцитонiн
- Альдостерон


Внаслiдок вираженого зниження концентрацiї кальцiю в плазмi кровi у дитини 2-х рокiв виникли тетанiчнi скорочення дихальних i глоткових м’язiв. Зниження секрецiї якого гормону може бути причиною цього?
+ Паратгормон
- Тиреокальцитонiн
- Альдостерон
- Соматотропiн
- Кортизол


До лiкаря звернувся чоловiк 27-ми рокiв. При оглядi було виявлено збiльшення кистей, стоп та нижньої щелепи. Крiм того спостерiгалися деформацiя суглобiв (kiphosis), гормональнi порушення (iмпотенцiя, атрофiя яєчок). Функцiя якої залози порушена?
+ Передня частка гiпофiзу
- Наднирковi залози
- Шишкоподiбне тiло
- Щитоподiбна залоза
- Прищитоподiбнi залози


У дитини 2-х рокiв виникли судоми внаслiдок зниження концентрацiї iонiв кальцiю в плазмi кровi. Функцiя якого ендокринного органу знижена?
+ Прищитоподiбнi залози
- Гiпофiз
- Кора наднирникiв
- Шишкоподiбна залоза
- Тимус


До лiкаря звернулася жiнка 32-х рокiв зi скаргами на вiдсутнiсть лактацiї пiсля народження дитини. Дефiцитом якого гормону можна пояснити дане порушення?
+ Пролактин
- Соматотропiн
- Вазопресин
- Тиреокальцитонiн
- Глюкагон


Хворому тривалий час вводили високi дози гiдрокортизону, внаслiдок чого настала атрофiя однiєї з зон кори наднирникiв. Яка це зона?
+ Пучкова
- Клубочкова
- Сiтчаста
- Клубочкова i сiтчаста
-


При тиреотоксикозi пiдвищується продукцiя тиреоїдних гормонiв Т3 та Т4, розвиваються схуднення, тахiкардiя, психiчне збудження та iнше. Як саме впливають тиреоїднi гормони на енергетичний обмiн в мiтохондрiях клiтин?
+ Роз‘єднують окислення та окисне фосфорилювання
- Активують субстратне фосфорилювання
- Блокують субстратне фосфорилювання
- Блокують дихальний ланцюг
- Активують окисне фосфорилювання


Хворий знаходиться на облiку в ендокринологiчному диспансерi з приводу гiпертиреозу. До схуднення, тахiкардiї, тремтiння пальцiв рук, приєдналися симптоми гiпоксiї – головний бiль, втомлюванiсть, мерехтiння "мушок"перед очима. Який механiзм дiї тиреоїдних гормонiв лежить в основi розвитку гiпоксiї?
+ Роз’єднання окиснення та фосфорилування
- Гальмування синтезу дихальних ферментiв
- Конкурентне гальмування дихальних ферментiв
- Посилення синтезу дихальних ферментiв
- Специфiчне зв’язування активних центрiв дихальних ферментiв


Стресовий стан i больове вiдчуття у пацiєнта перед вiзитом до стоматолога супроводжуються анурiєю (вiдсутнiстю сечовидiлення). Це явище зумовлене збiльшенням:
+ Секрецiї вазопресину та адреналiну
- Активностi парасимпатичної нервової системи
- Активностi антиноцiцептивної системи
- Секрецiї вазопресину та зменшенням адреналiну
- Секрецiї адреналiну та зменшенням вазопресину


Хворий помилково прийняв надмiрну дозу тироксину. До яких змiн секрецiї тиреолiберину та тиреотропiну це призведе?
+ Секрецiя гормонiв зменшиться
- Секрецiя гормонiв збiльшиться
- Змiн секрецiї гормонiв не буде
- Секрецiя тиреолiберину збiльшиться, тиреотропiну - зменшиться
- Секрецiя тиреотропiну збiльшиться, тиреолiберину - зменшиться


У хворих на тиреотоксикоз спостерiгаються гiпертермiя, булiмiя, зменшення маси тiла, що пов’язане з порушенням:
+ Спряження окислення i фосфорилювання
- Розпаду АТФ
- Синтезу жирiв
- Циклу лимонної кислоти
- β-окиснення жирних кислот


У чоловiка 35-ти рокiв феохромоцитома. В кровi спостерiгається пiдвищений рiвень адреналiну та норадреналiну, концентрацiя вiльних жирних кислот зросла в 11 разiв. Активацiя якого ферменту пiд впливом адреналiну пiдвищує лiполiз?
+ ТАГ-лiпаза
- Лiпопротеїдлiпаза
- Фосфолiпаза
- Фосфолiпаза С
- Холестеролестераза


До лiкаря звернулися батьки хлопчика 10-ти рокiв, у якого вiдзначалося збiльшення волосяного покриву на тiлi, рiст бороди i вус, низький голос. Збiльшення секрецiї якого гормону можна припустити?
+ Тестостерон
- Соматотропiн
- Естроген
- Прогестерон
- Кортизол


Жiнка 38-ми рокiв звернулася до ендокринологiчної клiнiки з виразним тремором кiнцiвок. Гiперпродукцiя, якого гормону здатна викликати такi порушення?
+ Тироксин
- АКТГ
- Iнсулiн
- Адреналiн
- Соматостатин


У мешканцiв територiй з холодним клiматом в кровi збiльшений вмiст гормону, що має пристосувальне терморегуляторне значення. Про який гормон йдеться?
+ Тироксин
- Iнсулiн
- Глюкагон
- Соматотропiн
- Кортизол


У хворого з верхнiм типом ожирiння тривало вiдзначалися артерiальна гiпертонiя, гiперглiкемiя, глюкозурiя. Смерть настала вiд крововиливу у головний мозок. Пiд час патоморфологiчного дослiдження виявленi базофiльна аденома гiпофiзу, гiперплазiя кори наднирникiв. Який найбiльш вiрогiдний дiагноз?
+ Хвороба Iценка-Кушiнга
- Цукровий дiабет
- Акромегалiя
- Гiпофiзарний нанiзм
- Адипозогенiтальна дистрофiя


У пацiєнта, що звернувся до лiкаря, спостерiгається жовте забарвлення шкiри, сеча темна, кал темно-жовтого кольору. Пiдвищення концентрацiї якої речовини буде спостерiгатися в сироватцi кровi?
+ Вiльний бiлiрубiн
- Кон’югований бiлiрубiн
- Мезобiлiрубiн
- Вердоглобiн
- Бiлiвердин


У хворого з жовтяницею встановлено: пiдвищення у плазмi кровi вмiсту загального бiлiрубiну за рахунок непрямого (вiльного), в калi i сечi - високий вмiст стеркобiлiну, рiвень прямого (зв’язаного) бiлiрубiну у плазмi кровi в межах норми. Який вид жовтяницi має мiсце у хворого?
+ Гемолiтична
- Паренхiматозна
- Механiчна
- Хвороба Жильбера
-


У хворого з’явилися жовтушнiсть шкiри, склер та слизових оболонок. У плазмi кровi пiдвищений рiвень загального бiлiрубiну, в калi - рiвень стеркобiлiну, в сечi - уробiлiну. Який вид жовтяницi у хворого?
+ Гемолiтична
- Хвороба Жiльбера
- Паренхiматозна
- Обтурацiйна
- Холестатична


У 70-тi роки вченi встановили, що причиною важкої жовтяницi новонароджених є порушення зв’язування бiлiрубiну в гепатоцитах. Яка речовина використовується для утворення кон’югату?
+ Глюкуронова кислота
- Сечова кислота
- Сiрчана кислота
- Молочна кислота
- Пiровиноградна кислота


У хворого на жовтяницю у кровi пiдвищений вмiст прямого бiлiрубiну та жовчних кислот; у сечi вiдсутнiй стеркобiлiноген. При якiй жовтяницi можлива наявнiсть цих ознак?
+ Механiчна
- Печiнкова
- Паренхiматозна
- Гемолiтична
- Надпечiнкова


Хвора 48-ми рокiв надiйшла до клiнiки iз скаргами на слабкiсть, дратiвливiсть, порушення сну. Об’єктивно: шкiра та склери жовтого кольору. У кровi: пiдвищення рiвня загального бiлiрубiну з переважанням прямого. Кал - ахолiчний. Сеча - темного кольору (жовчнi пiгменти). Яка жовтяниця має мiсце в хворої?
+ Механiчна
- Гемолiтична
- Паренхiматозна
- Синдром Жiльбера
- Синдром Крiглера-Найяра


Хворий надiйшов до клiнiки зi скаргами на загальну слабкiсть, порушення сну. Шкiра має жовтий колiр. У кровi: збiльшена кiлькiсть прямого бiлiрубiну, жовчних кислот. Кал ахолiчний. Для якого стану характернi цi змiни?
+ Механiчна жовтяниця
- Гемолiтична жовтяниця
- Надпечiнкова жовтяниця
- Синдром Жiльбера
- Хронiчний холецистит


У хворого на пiдгострий септичний ендокардит при оглядi лiкар вiдзначив загальну слабкiсть i iктеричнiсть шкiри, склер i видимих слизових оболонок. У кровi виявлена збiльшена кiлькiсть непрямого бiлiрубiну. Що зумовлює жовтяничнiсть шкiри i слизових?
+ Надпечiнкова жовтяниця
- Жирова дистрофiя
- Гемосидероз
- Печiнкова жовтяниця
- Пiдпечiнкова жовтяниця


У юнака 20-ти рокiв дiагностовано спадковий дефiцит УДФ-глюкуронiлтрансферази. Пiдвищення якого показника кровi пiдтверджує дiагноз?
+ Непрямий (некон’югований) бiлiрубiн
- Прямий (кон’югований) бiлiрубiн
- Уробiлiн
- Стеркобiлiноген
- Тваринний iндикан


Хвора 28-ми рокiв потрапила до iнфекцiйної лiкарнi з приводу пожовтiння шкiри, склер, слизових оболонок. Лабораторно встановлене пiдвищення рiвня прямого бiлiрубiну у кровi. В сечi виявлений уробiлiноген i бiлiрубiн. Для якого з перелiчених захворювань характернi такi змiни?
+ Паренхiматозна жовтяниця
- Гемолiтична жовтяниця
- Iнфаркт нирки
- Туберкульоз нирки
- Механiчна жовтяниця


Для лiкування жовтяниць показано призначення барбiтуратiв, якi iндукують синтез УДФглюкуронiлтрансферази. Лiкувальний ефект при цьому обумовлений утворенням:
+ Прямого (кон’югованого) бiлiрубiну
- Непрямого (некон’югованого) бiлiрубiну
- Бiлiвердину
- Протопорфирину
- Гему


У хворого, який скаржився на бiль у дiлянцi лiвої лопатки, був дiагностований iнфаркт мiокарду. Назвiть вид болю у хворого?
+ Iррадiюючий (вiдбитий)
- Вiсцеральний
- Фантомний
- Перший (протопатичний)
- Другий (епiкритичний)


У людини нормальна чутливiсть шкiри пальця, але вiн не вiдчуває наявностi на ньому обручки. Який процес, спричинений впливом обручки, є причиною цього?
+ Адаптацiя рецепторiв
- Розвиток фiброзної тканини
- Порушення структури епiдермiсу
- Порушення кровообiгу
- Порушення структури рецепторiв


В результатi травми порушено цiлiснiсть переднього корiнця спинного мозку. Якi вiдростки яких нейронiв при цьому пошкодженi?
+ Аксони рухових нейронiв
- Дендрити рухових нейронiв
- Аксони чутливих нейронiв
- Дендрити чутливих нейронiв
- Дендрити вставних нейронiв


У пiддослiдного щура з паралiчем кiнцiвки спостерiгається зникнення сухожилкових i шкiрних рефлексiв, зниження м’язового тонусу, при цьому зберiгається здатнiсть м’язiв ураженої кiнцiвки вiдповiдати збудженням на пряму дiю постiйного струму. Який тип паралiчу вiдзначається у тварини?
+ В’ялий периферичний
- В’ялий центральний
- Спастичний периферичний
- Спастичний центральний
- Екстрапiрамiдний


Пiсля обстеження пацiєнта в клiнiцi нервових хвороб встановлена вiдсутнiсть звуження зiницi при дiї свiтла. З ураженням яких структур головного мозку це пов’язано?
+ Вегетативнi ядра 3 пари черепномозкових нервiв
- Червонi ядра середнього мозку
- Ретикулярнi ядра середнього мозку
- Ядра гiпоталамуса
- Ретикулярнi ядра довгастого мозку


У тварини збiльшений тонус м’язiв-розгиначiв. Це є наслiдком посиленої передачi iнформацiї до мотонейронiв спинного мозку такими низхiдними шляхами:
+ Вестибулоспiнальнi
- Медiальнi кортикоспiнальнi
- Ретикулоспiнальнi
- Руброспiнальнi
- Латеральнi кортикоспiнальнi


У чоловiка 60-ти рокiв пiсля iнсульту настав тривалий сон. Ураження яких структур ЦНС найбiльш iмовiрно призвело до цього стану?
+ Висхiдна частина РФ
- Мозочок
- Чорна субстанцiя
- Прецентральна звивина
- V-IX пари черепних нервiв


Внаслiдок травми у чоловiка 40-ка рокiв зруйнованi заднi корiнцi спинного мозку. Якi розлади будуть спостерiгатися в зонi iннервацiї цих корiнцiв?
+ Втрата всiх видiв чутливостi
- Порушення функцiї посмугованих скелетних м’язiв
- Порушення функцiї гладеньких м’язiв
- Втрата температурної та вiбрацiйної чутливостi
- Втрата больової чутливостi


У тварини в експериментi перерiзали заднi корiнцi спинного мозку. Якi змiнi вiдбуватимуться в зонi iннервацiї?
+ Втрата чутливостi
- Втрата рухових функцiй
- Зниження тонусу м’язiв
- Пiдвищення тонусу м’язiв
- Втрата чутливостi i рухових функцiй


Хворий 50-ти рокiв звернувся до клiнiки зi скаргами на загальну слабкiсть, втрату апетиту, аритмiю серця. Спостерiгається гiпотонiя м’язiв, млявi паралiчi, послаблення перистальтики кишечнику. Причиною такого стану може бути:
+ Гiпокалiємiя
- Гiпопротеїнемiя
- Гiперкалiємiя
- Гiпофосфатемiя
- Гiпонатрiємiя


Дитина 9-ми мiсяцiв харчується штучними сумiшами, якi не збалансованi за вмiстом вiтамiну B6 . У дитини спостерiгається пелагроподiбний дерматит, судоми, анемiя. Розвиток судом може бути пов’язаний з порушенням утворення:
+ ГАМК
- Гiстамiну
- Серотонiну
- ДОФА
- Дофамiну


Жiнка 68-ми рокiв скаржиться на вiдсутнiсть рухiв у правих руцi i нозi. Чотири мiсяцi тому перенесла iнсульт. Об’єктивно: рухи в правих кiнцiвках вiдсутнi, тонус м’язiв їх пiдвищений. Який стан спостерiгається у хворої?
+ Гемiплегiя
- Моноплегiя
- Параплегiя
- Тетраплегiя
-


У хворого пiсля перенесеної черепно-мозкової травми порушений акт ковтання. Який вiддiл мозку постраждав?
+ Довгастий мозок
- Середнiй мозок
- Промiжний мозок
- Кiнцевий мозок
- Таламус


В хронiчному експериментi на щурах стимулювали електричним струмом паравентрикулярнi та супраоптичнi ядра гiпоталамуса. Яку поведiнкову реакцiю це спричинило у тварин?
+ Збiльшення споживання води
- Зменшення споживання води
- Збiльшення споживання їжi
- Зменшення споживання їжi
- Вiдмова вiд їжi та рiдини


У пiддослiдної тварини пiд час експерименту подразнюють периферичний вiдрiзок блукаючого нерва. Якi з наведених змiн будуть спостерiгатися при цьому?
+ Зменшення частоти серцевих скорочень
- Збiльшення частоти серцевих скорочень
- Розширення зiниць
- Збiльшення частоти дихання
- Розширення бронхiв


У хворої 49-ти рокiв вiдзначається обмеження довiльних рухiв у лiвих кiнцiвках. Тонус м’язiв у лiвих руцi та нозi пiдвищений за спастичним типом, посиленi мiсцевi сухожилковi рефлекси, виявляються патологiчнi рефлекси. Який найбiльш iмовiрний механiзм призвiв до розвитку м’язової гiпертонiї та гiперрефлексiї?
+ Зниження гальмiвних низхiдних впливiв
- Активацiя мотонейронiв внаслiдок iнсульту
- Активацiя збуджуючих впливiв з вогнища iнсульту
- Активацiя синаптичної передачi iмпульсiв
- Гальмування мотонейронiв кори головного мозку


У чоловiка вiдмiчається випадiння функцiї медiальних половин сiткiвки. Який вiддiл провiдного шляху зорового аналiзатора уражений?
+ Зорове перехрестя
- Лiвий зоровий тракт
- Правий зоровий тракт
- Лiвий зоровий нерв
- Правий зоровий нерв


У людини звуженi зiницi. Чим це зумовлено?
+ Зростання тонусу парасимпатичних центрiв
- Зростання тонусу симпатичних центрiв
- Збiльшення активностi симпатоадреналової системи
- Дiя адреналiну
- Дiя норадреналiну


Отруєння ботулiнiчним токсином, який блокує вхiд iонiв кальцiю до нервових закiнчень аксонiв мотонейронiв, небезпечно для життя, бо загрожує:
+ Зупинкою дихання
- Зупинкою серця
- Розладом тонусу судин
- Розвитком блювання
- Розвитком проносу


При дослiдженнi гостроти слуху в коваля виявили втрату слуху на 50% у дiапазонi низьких частот i майже нормальну гостроту слуху в дiапазонi високих частот. Порушення яких структур слухової системи призвело до такого стану?
+ Кортiєв орган - ближче до гелiкотреми
- Кортiєв орган - ближче до овального вiконця
- Середня частина кортiєвого органу
- М’язи середнього вуха
- Барабанна перетинка


Хвора 75-ти рокiв доставлена до офтальмологiчного вiддiлення лiкарнi зi скаргами на погiршення зору. При об’єктивному дослiдженнi встановлена наявнiсть пухлини мозку, що розташована в дiлянцi лiвого зорового тракту. При цьому у хворої спостерiгається випадiння поля зору в:
+ Лiвих половинах сiткiвки обох очей
- Правих половинах сiткiвки обох очей
- Правих i лiвих половинах сiткiвки лiвого ока
- Правих i лiвих половинах сiткiвки правого ока
- Правих i лiвих половинах сiткiвок обох очей


У хворої 18-ти рокiв з’явились постiйнi запаморочення, нiстагм очей, скандована мова, невпевнена хода. Це свiдчить про порушення функцiї:
+ Мозочка
- Рухової кори
- Базальних ганглiїв
- Чорної субстанцiї
- Вестибулярних ядер


Внаслiдок черепно-мозкової травми у хворого розвинулись наступнi симптоми: iнтенцiйний тремор, дисметрiя, адiадохокiнез, дизартрiя. Яка структура головного мозку ушкоджена?
+ Мозочок
- Стрiатум
- Рухова кора
- Блiда куля
- Чорна речовина


За медичним показанням пацiєнту було проведено видалення частини однiєї iз структур ЦНС. В результатi видалення у пацiєнта розвинулися атонiя, астазiя, iнтенцiйний тремор, атаксiя, адiадохокiнез. Частина якої структури ЦНС була вилучена?
+ Мозочок
- Мигдалеподiбний комплекс
- Гiпокамп
- Базальнi ганглiї
- Лiмбiчна система


Депресiї та емоцiйнi розлади є наслiдком нестачi у головному мозку норадреналiну, серотонiну та iнших бiогенних амiнiв. Збiльшення їх вмiсту у синапсах можна досягти за рахунок антидепресантiв, якi гальмують такий фермент:
+ Моноамiнооксидаза
- Диамiнооксидаза
- Оксидаза L-амiнокислот
- Оксидаза D-амiнокислот
- Фенiлаланiн-4-монооксигеназа


У лабораторному експериментi на собацi вивчали будову центральних вiддiлiв слухової сенсорної системи. Була зруйнована одна з структур середнього мозку. Собака втратив орiєнтувальний рефлекс на звуковi сигнали. Яка структура була зруйнована?
+ Нижнi горбики чотиригорбикового тiла
- Верхнi горбики чотиригорбикового тiла
- Чорна речовина
- Ядра ретикулярної формацiї
- Червоне ядро


У реанiмацiйному вiддiленнi знаходиться хворий у коматозному станi. При дослiдженнi кровi вiдзначено збiльшення концентрацiї iонiв K
+ I зменшення - Ca +
+ , ацидоз, збiльшення рiвнiв сечовини, сечової кислоти. Який вид коми за етiологiєю найбiльш iмовiрний?
+ Ниркова
- Печiнкова
- Нейрогенна
- Дiабетична
- Гiпоглiкемiчна


Внаслiдок руйнування певних структур стовбуру мозку тварина втратила орiєнтувальнi рефлекси у вiдповiдь на сильнi свiтловi подразники. Якi структури було зруйновано?
+ Переднi горбки чотиригорбкового тiла
- Заднi горбки чотиригорбкового тiла
- Червонi ядра
- Вестибулярнi ядра
- Чорна речовина


Хворий звернувся до лiкаря зi скаргами на порушення вiдчуття рiвноваги, що з’явилося пiсля травми. Який нерв пошкоджено?
+ Присiнково-завитковий
- Трiйчастий
- Лицевий
- Промiжний
- Блукаючий


У кiшки з децеребрацiйною ригiднiстю потрiбно знизити тонус м’язiв. Цього можна досягти шляхом:
+ Руйнування вестибулярних ядер Дейтерса
- Подразнення отолiтових вестибулорецепторiв
- Подразнення вестибулярних ядер Дейтерса
- Подразнення вестибулослухового нерва
- Подразнення ампулярних вестибулорецепторiв


Пiд час футбольного матчу мiж вболiвальниками рiзних команд виникла сутичка. На фонi негативних емоцiй в одного учасника сутички були розширенi зiницi й пiдвищене серцебиття. Активацiя якої системи регуляцiї функцiй органiзму забезпечує такi вегетативнi змiни при негативних емоцiях?
+ Симпато-адреналова
- Гiпоталамо-гiпофiзарно-тиреоїдна
- Соматична нервова
- Парасимпатична нервова
- Метасимпатична нервова


В клiнiку доставлено чоловiка з травмою спини. Пiд час обстеження виявлено перелом хребцiв грудного вiддiлу. Пiд час об’єктивного огляду нейрохiрургом виявлено: нижче рiвня перелому з правого боку вiдсутня глибока чутливiсть, з лiвого боку – порушена температурна та тактильна чутливiсть. Яке ураження з боку спинного мозку є у хворого?
+ Синдром Броун Секара
- Хвороба Паркiнсона
- Судомний синдром
- Анестезiя
- Парастезiя


До вiддiлення нейрохiрургiї було доставлено чоловiка з втратою слуху внаслiдок травми голови. Порушення якої частки кори головного мозку може бути причиною цього?
+ Скронева
- Постцентральна звивина
- Тiм’яна
- Потилична
- Лобова


У чоловiка 33-х рокiв внаслiдок спинномозкової травми порушена больова та температурна чутливiсть, що обумовлено пошкодженням таких висхiдних шляхiв:
+ Спиноталамiчнi
- Медiальний спинокортикальний
- Заднiй спиномозочковий
- Латеральний спинокортикальний
- Переднiй спиномозочковий


У тварини зруйнували отолiтовi вестибулорецептори. Якi з наведених рефлексiв зникнуть внаслiдок цього у тварини?
+ Статокiнетичнi при рухах з лiнiйним прискоренням
- Статокiнетичнi при рухах з кутовим прискоренням
- Мiотатичнi
- Випрямлення тулуба
- Первиннi орiєнтувальнi


У хворого поперечний розрив спинного мозку нижче VI грудного сегменту. Як внаслiдок цього змiниться дихання?
+ Суттєво не змiниться
- Зупиниться
- Стане бiльш рiдким
- Стане бiльш глибоким
- Стане бiльш частим


До лiкарнi звернувся чоловiк 50-ти рокiв з розладами пам’ятi, болiсними вiдчуттями по ходу нервових стовбурiв, зниженням iнтелектуальних функцiй, порушеннями з боку серцево-судинної системи i явищами диспепсiї. В анамнезi хронiчний алкоголiзм. Дефiцит якого вiтамiну може викликати цi симптоми?
+ Тiамiн
- Нiацин
- Ретинол
- Кальциферол
- Рибофлавiн


У чоловiка 28-ми рокiв пiсля вогнепального поранення гомiлки розвинулася виразка на боцi пошкодження. Що є основним у патогенезi нейродистрофiї в даному випадку?
+ Травматизацiя периферичного нерва
- Психiчний стрес
- Порушення мiкроциркуляцiї
- Iнфекцiя
- Пошкодження тканини


При травмi периферичних нервiв виникає м’язова атрофiя, кiстки стають порозними i ламкими, на шкiрi i слизових виникають виразки. Яка функцiя нервових системи уражується у даному випадку?
+ Трофiчна
- Рухова
- Чутлива
- Вегетативна
- Вища нервова дiяльнiсть


У чоловiка пiсля гiпертонiчної кризи вiдзначається вiдсутнiсть довiльних рухiв в правих руцi та нозi, тонус м’язiв у цих кiнцiвках пiдвищений. Який вид розладу рухової функцiї спостерiгається у даному випадку?
+ Центральний паралiч
- Периферичний паралiч
- Периферичний парез
- Рефлекторний парез
- Центральний парез


Внаслiдок руйнування певних структур стовбуру мозку тварина втратила орiєнтувальнi рефлекси. Якi структури було зруйновано?
+ Чотиригорбкова структура
- Медiальнi ядра ретикулярної формацiї
- Червонi ядра
- Вестибулярнi ядра
- Чорна речовина


У хворого 48-ми рокiв на хронiчний гломерулонефрит наявнi набряки, АТ-210/100 мм рт.ст., ЧСС- 85/хв., межi серця розширенi. Який механiзм розвитку артерiальної гiпертензiї є головним?
+ Активацiя ренин-ангiотензинальдостеронової системи
- Пiдвищення ОЦК
- Пiдвищення продукцiї вазопресину
- Пiдвищення активностi симпатичного вiддiлу нервової системи
- Гiперфункцiя серця


У хворого 38-ми рокiв на 3-му роцi захворювання на системний червоний вiвчак виявлене дифузне ураження нирок, що супроводжується масивними набряками i вираженою протеїнурiєю. Що є найбiльш вiрогiдною причиною розвитку протеїнурiї у пацiєнта?
+ Аутоiмунне ушкодження нирок
- Асептичне ураження нирок
- Iшемiчне ушкодження нирок
- Запальне ураження сечового мiхура
- Запальне ураження сечовивiдних шляхiв


У хворого з гострою нирковою недостатнiстю на 6-й день проведення терапевтичних заходiв виникла полiурiя. Чим зумовлене зростання дiурезу на початку полiуричної стадiї гострої ниркової недостатностi?
+ Вiдновлення фiльтрацiї в нефронах
- Збiльшенням об’єму циркулюючої кровi
- Збiльшенням натрiйуретичного фактора
- Зменшенням альдостерону в плазмi
- Зменшенням вазопресину в плазмi


Хворий 55-ти рокiв хворiє на хронiчний гломерулонефрит протягом 15-ти рокiв. Якi змiни складу кровi або сечi найбiльш характерно свiдчать про обмеження секреторної функцiї нирок?
+ Гiперазотемiя
- Гiперглiкемiя
- Гiпопротеїнемiя
- Протеїнурiя
- Гiпо-, iзостенурiя


У пацiєнта з хронiчним захворюванням нирок розвинулась ниркова недостатнiсть. Який з показникiв найбiльш iмовiрно свiдчить про порушення реабсорбцiї в канальцях в даному випадку?
+ Гiпо- та iзостенурiя
- Гiперазотемiя
- Зниження клiренсу
- Гематурiя
- Лейкоцитурiя


Внаслiдок землетрусу чоловiк 50-ти рокiв двi доби перебував пiд завалом. Пiсля звiльнення з-пiд завалу рятiвниками у нього був встановлений синдром тривалого розчавлення. Виникнення якого ускладнення в подальшому найбiльш вiрогiдне?
+ Гостра ниркова недостатнiсть
- Гостра печiнкова недостатнiсть
- Гостра серцева недостатнiсть
- Гостра судинна недостатнiсть
- Гостра дихальна недостатнiсть


У результатi порушення технiки безпеки вiдбулося отруєння сулемою (хлористою ртуттю). Через 2 днi добовий дiурез склав 620 мл. У хворого з’явилися головний бiль, блювання, судоми, задишка, у легенях - вологi хрипи. Яка патологiя має мiсце?
+ Гостра ниркова недостатнiсть
- Хронiчна ниркова недостатнiсть
- Уремiчна кома
- Гломерулонефрит
- Пiєлонефрит


У пацiєнта встановлено порушення синтезу та видiлення вазопресину. В якому вiддiлi нефрона найбiльше порушиться процес сечоутворення?
+ Збiрна трубочка
- Проксимальний звивистий каналець
- Тонка частина петлi Генле
- Товста частина петлi Генле
- Клубочок


У хворого з масивними опiками розвинулась гостра недостатнiсть нирок, що характеризується значним i швидким зменшенням швидкостi клубочкової фiльтрацiї. Який механiзм її розвитку?
+ Зменшення ниркового кровотоку
- Ушкодження клубочкового фiльтра
- Зменшення кiлькостi функцiонуючих нефронiв
- Збiльшення тиску канальцевої рiдини
- Емболiя ниркової артерiї


У людини внаслiдок тривалого голодування швидкiсть клубочкової фiльтрацiї зросла на 20%. Найбiльш вiрогiдною причиною змiн фiльтрацiї в зазначених умовах є:
+ Зменшення онкотичного тиску плазми кровi
- Збiльшення системного артерiального тиску
- Збiльшення проникностi ниркового фiльтру
- Збiльшення коефiцiєнта фiльтрацiї
- Збiльшення ниркового кровотоку


Введення знеболюючого пацiєнту перед екстракцiєю зуба призвело до розвитку анафiлактичного шоку, який супроводжувався розвитком олiгурiї. Який патогенетичний механiзм зумовив зменшення дiурезу в данiй клiнiчнiй ситуацiї?
+ Зниження гiдростатичного тиску в капiлярах клубочкiв
- Пiдвищення гiдростатичного тиску в капсулi Шумлянського-Боумена
- Пошкодження клубочкового фiльтру
- Збiльшення онкотичного тиску кровi
- Зменшення кiлькостi функцiонуючих нефронiв


У хворого знижений синтез вазопресину, що призводить до полiурiї i, як наслiдок, до вираженої дегiдратацiї органiзму. У чому полягає механiзм розвитку полiурiї?
+ Зниження канальцевої реабсорбцiї води
- Зниження канальцевої реабсорбцiї iонiв Na
- Зниження канальцевої реабсорбцiї бiлку
- Зниження реабсорбцiї глюкози
- Збiльшення швидкостi клубочкової фiльтрацiї


У фiзично здорових молодих курсантiв пiсля важкого фiзичного навантаження при одноденному пiшому переходi на 50 км в сечi виявлено бiлок, рiвень якого в середньому не перевищував 1 г/л. Який рiзновид протеїнурiї мав мiсце?
+ Маршова
- Дегiдратацiйна
- Алiментарна
- Органiчна
- Несправжня


У жiнки 30-ти рокiв виникли набряки обличчя. При обстеженнi виявленi протеїнурiя (5,87 г/л), гiпопротеїнемiя, диспротеїнемiя, гiперлiпiдемiя. Для якого стану характерно таке поєднання симптомiв?
+ Нефротичний синдром
- Нефритичний синдром
- Хронiчний пiєлонефрит
- Гостра ниркова недостатнiсть
- Хронiчна ниркова недостатнiсть


У юнака 20-ти рокiв, через 2 тижнi пiсля перенесеної лакунарної ангiни, з’явилися скарги на загальну слабкiсть, набряки пiд очима. Пiсля обстеження хворому встановлено дiагноз: гострий гломерулонефрит. Якi патологiчнi змiни у складi сечi найбiльш вiрогiднi?
+ Протеїнурiя
- Цилiндрурiя
- Наявнiсть свiжих еритроцитiв
- Пiурiя
- Натрiйурiя


У населеному пунктi зареєстрований спалах гепатиту, який зв’язують з водним фактором. Який вiрус гепатиту мiг викликати спалах захворювань у цьому населеному пунктi?
+ E
- C
- D
- G
- B


У хворого 49-ти рокiв на гострий панкреатит виникала загроза некрозу пiдшлункової залози, що супроводжувалось надходженням у кров i тканини активних панкреатичних протеїназ i розщеплення тканинних бiлкiв. Якi захиснi фактори органiзму можуть iнгiбувати цi процеси?
+ α2 -макроглобулiн, α1 -антитрипсин
- Iмуноглобулiни
- Крiоглобулiн, iнтерферон
- Церулоплазмiн, трансферин
- Гемоплексин, гаптоглобiн


Хворому з гiперсекрецiєю шлункового соку лiкар рекомендував виключити з дiєти насиченi бульйони i овочевi вiдвари, тому що вони стимулюють шлункову секрецiю переважно через активацiю:
+ Вироблення гастрину
- Смакових рецепторiв
- Механорецепторiв ротової порожнини
- Механорецепторiв шлунка
- Вироблення секретину


У хворого нормально забарвлений кал, у складi якого є велика кiлькiсть вiльних жирних кислот. Причиною цього є порушення:
+ Всмоктування жирiв
- Гiдролiзу жирiв
- Жовчовидiлення
- Жовчоутворення
- Секрецiї лiпаз


Пацiєнт звернувся зi скаргами на гострий бiль у правому пiдребер’ї. При оглядi лiкар звернув увагу на пожовтiння склер хворого. Лабораторно: пiдвищена активнiсть АлАТ та негативна реакцiя на стеркобiлiн в калi. Для якого захворювання характернi такi симптоми?
+ Гепатит
- Гемолiтична жовтяниця
- Хронiчний гастродуоденiт
- Хронiчний колiт
- Хронiчний гастрит


Призначення доксициклiну гiдрохлориду викликало порушення симбiозу мiкробної флори в кишечнику. Визначити тип порушень при антибiотикотерапiї:
+ Дисбактерiоз
- Сенсибiлiзацiя
- Iдiосинкразiя
- Суперiнфекцiя
- Бактерiоз


У хворого на хронiчний гепатит виявлено значне зниження синтезу I секрецiї жовчних кислот. Який процес у найбiльшiй мiрi буде порушений у кишечнику цього хворого?
+ Емульгування жирiв
- Травлення бiлкiв
- Травлення вуглеводiв
- Всмоктування глiцерину
- Всмоктування амiнокислот


У людини порушено всмоктування продуктiв гiдролiзу жирiв. Причиною цього може бути дефiцит у порожнинi тонкої кишки:
+ Жовчних кислот
- Жовчних пiгментiв
- Лiполiтичних ферментiв
- Iонiв натрiю
- Жиророзчинних вiтамiнiв


Хворий пiсля вживання жирної їжi вiдчуває нудоту, млявiсть; з часом з’явилися ознаки стеатореї. У кровi холестерин - 9,2 ммоль/л. Причиною такого стану є нестача у кишечнику:
+ Жовчних кислот
- Триглiцеридiв
- Жирних кислот
- Фосфолiпiдiв
- Хiломiкронiв


Чоловiк 53-х рокiв звернувся зi скаргами на гострий бiль у правому пiдребер’ї. При оглядi лiкар звернув увагу на пожовтiлi склери хворого. Лабораторнi аналiзи показали пiдвищену активнiсть АЛТ та негативну реакцiю на стеркобiлiн у калi. Для якого захворювання характернi такi симптоми?
+ Жовчнокам’яна хвороба
- Гемолiтична жовтяниця
- Гепатит
- Хронiчний колiт
- Хронiчний гастрит


У юнака 16-ти рокiв пiсля перенесеного захворювання знижена функцiя синтезу бiлкiв у печiнцi внаслiдок нестачi вiтамiну K. Це може призвести до порушення:
+ Зсiдання кровi
- Швидкостi осiдання еритроцитiв
- Утворення антикоагулянтiв
- Утворення еритропоетинiв
- Осмотичного тиску кровi


При лабораторному дослiдженнi у хворого виявили стеаторею. Вкажiть фермент, недостатнiсть дiї якого призвела до виникнення цього симптому:
+ Лiпаза
- Амiлаза
- Пепсин
- Лактаза
- Хiмотрипсин


У новонародженого спостерiгається диспепсiя пiсля годування молоком. При замiнi молока розчином глюкози симптоми диспепсiї зникають. Недостатня активнiсть якого ферменту спостерiгається у новонародженого?
+ Лактаза
- Сахараза
- Мальтаза
- Амiлаза
- Iзомальтаза


При копрологiчному дослiдженнi встановлено, що кал знебарвлений, у ньому знайдено краплi нейтрального жиру. Найбiльш вiрогiдною причиною цього є порушення:
+ Надходження жовчi до кишечнику
- Кислотностi шлункового соку
- Секрецiї пiдшлункового соку
- Секрецiї кишкового соку
- Процесiв всмоктування в кишечнику


Пiсля вживання жирної їжi у хворого з’являються нудота та печiя, має мiсце стеаторея. Причиною такого стану може бути:
+ Нестача жовчних кислот
- Пiдвищене видiлення лiпази
- Порушення синтезу трипсину
- Нестача амiлази
- Порушення синтезу фосфолiпази


Хворий вiдзначає частi проноси, особливо пiсля вживання жирної їжi, схуднення. Лабораторнi дослiдження показали наявнiсть стеатореї; кал гiпохолiчний. Що може бути причиною такого стану?
+ Обтурацiя жовчних шляхiв
- Запалення слизової оболонки тонкої кишки
- Недостатнiсть панкреатичної лiпази
- Недостатнiсть панкреатичної фосфолiпази
- Незбалансована дiєта


Хворому на хронiчний гастрит зроблена внутрiшньошлункова рН-метрiя, за допомогою якої встановлено зниження кислотностi шлункового соку. Функцiя яких клiтин знижена?
+ Парiєтальнi екзокриноцити
- Головнi екзокриноцити
- Ендокриноцити
- Шийковi клiтини
- Додатковi клiтини


Пiсля переходу до змiшаного харчування у новонародженої дитини виникла диспепсiя з дiареєю, метеоризмом, вiдставанням у розвитку. Бiохiмiчна основа даної патологiї полягає у недостатностi:
+ Сахарази та iзомальтази
- Лактази та целобiази
- Трипсину та хiмотрипсину
- Лiпази та креатинкiнази
- Целюлази


До складу харчових рацiонiв обов’язково входять продукти, в яких є клiтковина. Вiдомо, що вона не перетравлюється ферментами травного тракту й не засвоюється органiзмом. Яку роль вiдiграє ця речовина?
+ Стимулює моторну функцiю травного каналу
- Гальмує моторну функцiю травного каналу
- Гальмує всмоктувальну функцiю травного каналу
- Гальмує секреторну функцiю травного каналу
- Гальмує процеси видiлення ферментiв травних сокiв


У хворого 30-ти рокiв iз гострим запаленням пiдшлункової залози (панкреатитом) виявлено порушення порожнинного травлення бiлкiв. Це може бути пов’язано iз недостатнiм синтезом та видiленням залозою такого ферменту:
+ Трипсин
- Пепсин
- Лiпаза
- Дипептидаза
- Амiлаза


Жiнка 45-ти рокiв декiлька рокiв хворiє на системний червоний вовчак у легкiй формi. При прогресуваннi захворювання (з’явився мiокардит) їй призначили преднiзолон як iмуносупресор. Через 2 мiсяцi прийому у хворої виникла шлункова кровотеча. Яка найбiльш iмовiрна її причина?
+ Ульцерогенна дiя
- Зменшення згортання кровi
- Пiдвищення артерiального тиску
- Подальше прогресування захворювання
- Збудження ЦНС


Проводять дуоденальне зондування. Що iз наведеного доцiльно ввести людинi пiд шкiру, щоб суттєво збiльшити надходження до дванадцятипалої кишки жовчi?
+ Холецистокiнiн-панкреозимiн
- Гастрин
- Секретин
- Нейротензин
- Соматостатин